Download as pdf or txt
Download as pdf or txt
You are on page 1of 493

Any screen.

Any time.
Anywhere.
Activate the eBook version
of this title at no additional charge.

Elsevier eBooks+ gives you the power to browse, search, and customize your content,
make notes and highlights, and have content read aloud.

Unlock your eBook today.


1. Visit http://ebooks.health.elsevier.com/
2. Log in or Sign up
3. Scratch box below to reveal your code
4. Type your access code into the “Redeem
Access Code” box
5. Click “Redeem”

It’s that easy! Place Peel Off


Sticker Here

For technical assistance:


email [email protected]
call 1-800-545-2522 (inside the US)
call +44 1 865 844 640 (outside the US)
Use of the current edition of the electronic version of this book (eBook) is subject to the terms of the nontransferable, limited license granted on
http://ebooks.health.elsevier.com/. Access to the eBook is limited to the first individual who redeems the PIN, located on the inside cover of this book,
at http://ebooks.health.elsevier.com/ and may not be transferred to another party by resale, lending, or other means.
2022v1.0
AL GRAWANY
Review of Surgery for
ABSITE AND
BOARDS
AL GRAWANY
Review of Surgery for
ABSITE AND BOARDS
THIRD EDITION

EDITORS
Christian de Virgilio, MD, FACS Areg Grigorian, MD
Chair Assistant Clinical Professor of Surgery
Department of Surgery Department of Surgery
Harbor-UCLA Meical Center Division of Trauma, Burns an Critical Care
Torrance, California; University of California, Irvine
Co-Chair Orange, California
College of Applie Anatomy;
Professor of Surgery
UCLA School of Meicine
Los Angeles, California

ASSOCIATE EDITORS

Amanda C. Purdy, MD Eric O. Yeates, MD Naveen Balan, MD


Surgical Resient Physician Resient Physician Surgical Resient
Department of Surgery Department of Surgery Department of Surgery
Harbor-UCLA Meical Center University of California, Irvine Harbor-UCLA Meical Center
Torrance, California Orange, California Torrance, California

ILLUSTRATOR
Stephanie Cohen, MD
Surgical Resient
Beth Israel Deaconess Meical Center
Boston, Massachusetts
1600 John F. Kenney Blv.
Ste 1800
Philaelphia, PA 19103-899

REVIEW OF SURGERY FOR ABSITE AND BOARDS, THIRD EDITION ISBN: 978-0-33-87054-

Copyright © 2023 by Elsevier Inc. All rights reserved.

No part of this publication may be reprouce or transmitte in any form or by any means, electronic or mechan-
ical, incluing photocopying, recoring, or any information storage an retrieval system, without permission in
writing from the publisher. Details on how to seek permission, further information about the Publisher’s permis-
sions policies, an our arrangements with organizations such as the Copyright Clearance Center an the Copyright
Licensing Agency, can be foun at our website: www.elsevier.com/permissions

This book an the iniviual contributions containe in it are protecte uner copyright by the Publisher (other
than as may be note herein).

Notice

Practitioners an researchers must always rely on their own experience an knowlege in evaluating an using
any information, methos, compouns, or experiments escribe herein.Because of rapi avances in the
meical sciences, in particular, inepenent verication of iagnoses an rug osages shoul be mae. To the
fullest extent of the law, no responsibility is assume by Elsevier, authors, eitors, or contributors for any injury
an/or amage to persons or property as a matter of proucts liability, negligence or otherwise, or from any
use or operation of any methos, proucts, instructions, or ieas containe in the material herein.

Previous eitions copyrighte 018 an 010.

Content Strategist: Jessica McCool


Content Development Specialist: Shweta Pant
Publishing Services Manager: Shereen Jameel
Project Manager: Beula Christopher
Design Direction: Ryan Cook

Printe in the Unite States of America.


Last igit is the print number: 9 8 7 6 5 4 3  1

AL GRAWANY
To my family, who always support me, and to all students of surgery, who motivate and inspire me to
always keep learning the art and science of medicine.

—Christian de Virgilio

I would not be where I am today if it wasn't for my mentors. Dr. de Virgilio—you are the reason I love surgical
education.Dr. Demetriades, you taught me trauma surgery but more importantly, you taught me how to be an
effective and inspiring teacher. Dr. Inaba, you have taught me how to be an effective leader both inside and outside
the operating room. Dr.Nahmias, you have taught me how to be an academician and researcher. And to my loving
wife, Rebecca Grigorian—a superhero mom and physician! Thank you all!

—Areg Grigorian
AL GRAWANY
Contributors

Mark Archie, MD Christian de Virgilio, MD, FACS


Assistant Clinical Professor of Surgery Chair
Department of Surgery Department of Surgery
Harbor-UCLA Medical Center Harbor-UCLA Medical Center
David Geffen School of Medicine at UCLA Torrance, California;
Los Angeles, California Co-Chair
College of Applied Anatomy;
Naveen Balan, MD
Professor of Surgery
Surgical Resident
UCLA School of Medicine
Department of Surgery
Los Angeles, California
Harbor-UCLA Medical Center
Torrance, California Benjamin DiPardo, MD
Jeremy M. Blumberg, MD Resident
Chief of Urology Department of Surgery
Harbor-UCLA Medical Center; UCLA
Associate Professor of Urology Los Angeles, California
David Geffen School of Medicine at UCLA
Richard Everson, MD
Los Angeles, California
Assistant Clinical Professor of Surgery
Nina M. Bowens, MD Department of Surgery
Assistant Professor Harbor-UCLA Medical Center
Department of Surgery David Geffen School of Medicine at UCLA
David Geffen School of Medicine at UCLA; Los Angeles, California
Associate Program Director, Vascular Surgery Program
Division of Vascular and Endovascular Surgery Mytien Goldberg, MD
Harbor-UCLA Medical Center Assistant Clinical Professor of Surgery
Torrance, California Department of Surgery
Harbor-UCLA Medical Center
Caitlyn Braschi, MD David Geffen School of Medicine at UCLA
Resident Physician Los Angeles, California
Department of Surgery
Harbor-UCLA Medical Center Areg Grigorian, MD
Torrance, California Assistant Clinical Professor of Surgery
Department of Surgery
Formosa Chen, MD, MPH
Division of Trauma, Burns and Critical Care
Health Sciences Assistant Clinical Professor
University of California, Irvine
Department of Surgery
Orange, California
David Geffen School of Medicine at UCLA
Los Angeles, California Joseph Hadaya, MD, PhD
Kathryn T. Chen, MD Resident Physician
Assistant Professor Department of Surgery
Department of Surgery David Geffen School of Medicine at UCLA
Harbor-UCLA Medical Center Los Angeles, California
Torrance, California
Danielle M. Hari, MD, FACS
Christine Dauphine, MD, FACS Division Chief, Surgical Oncology
Vice Chair, Education Department of Surgery
Department of Surgery Harbor-UCLA Medical Center;
Harbor-UCLA Medical Center Associate Professor
Torrance, California; Department of Surgery
Professor of Surgery David Geffen School of Medicine at UCLA
David Geffen School of Medicine at UCLA Los Angeles, California
Los Angeles, California vii
viii Contributors

Dennis Kim, MD Beverley A. Petrie, MD, FACS, FASCRS


Trauma Medical Director Professor of Surgery
Island Health Trauma Services Department of Surgery
Victoria, British Columbia, Canada David Geffen School of Medicine at UCLA
Los Angeles, California;
Catherine M. Kuza, MD, FASA Assistant Chief
Assistant Professor Division of Colon and Rectal Surgery, Department of Surgery
Department of Anesthesiology, Division of Critical Care Harbor-UCLA Medical Center
Keck School of Medicine of the University of Southern Torrance, California
California
Los Angeles, California Amanda C. Purdy, MD
Surgical Resident Physician
Steven L. Lee, MD, MBA Department of Surgery
Professor and Chief Harbor-UCLA Medical Center
Pediatric Surgery Torrance, California
UCLA Mattel Children’s Hospital
Los Angeles, California Shonda L. Revels, MD, MS
Assistant Professor
John McCallum, MD, MPH Department of Surgery
Assistant Professor UCLA
Department of Surgery Los Angeles, California
Harbor-UCLA Medical Center
Torrance, California Jordan M. Rook, MD
Resident Physician
Michael A. Mederos, MD Department of Surgery
Resident Physician David Geffen School of Medicine at UCLA
Department of Surgery Los Angeles, California
UCLA
Los Angeles, California Saad Shebrain, MBBCh, MMM, FACS
Program Director, Associate Professor of Surgery
Alexandra Moore, MD Department of Surgery
Surgery Resident Western Michigan University Homer Stryker M.D. School
Department of Surgery ofMedicine
UCLA Kalamazoo, Michigan
Los Angeles, California
Eric R. Simms, MD
Jeffry Nahmias, MD, MHPE Chief
Associate Professor Division of General, Bariatric and Minimally Invasive Surgery,
Department of Surgery Assistant Program Director of Surgery Residency
University of California, Irvine Department of Surgery
Orange, California Harbor-UCLA Medical Center
Torrance, California
Kristofer E. Nava, MD
Department of General Surgery Veronica Sullins, MD
Western Michigan University Homer D. Stryker School Assistant Clinical Professor of Surgery
of Medicine Department of Pediatric Surgery
Kalamazoo, Michigan David Geffen School of Medicine at UCLA
Los Angeles, California
Junko Ozao-Choy, MD, FACS
Vice Chair, Research Maria G. Valadez, MD
Department of Surgery General Surgery Resident
Harbor-UCLA Medical Center Department of Surgery
Torrance, California; Harbor-UCLA Medical Center
Associate Professor of Surgery Torrance, California
David Geffen School of Medicine at UCLA
Los Angeles, California Luis Felipe Cabrera Vargas, MD, MACC, FACS,
MACCVA, MFELAC
Joon Y. Park, MD President of the Future Surgeons Chapter of the Colombian
Surgery Resident Surgery Association,
Department of Surgery Professor of the Universidad Javeriana and Universidad El Bosque,
David Geffen School of Medicine at UCLA Fellow of Vascular Surgery of the Universidad Militar Nueva
Los Angeles, California Granada
Bogotá, Colombia

AL GRAWANY
Contributors ix

Zachary N. Weitzner, MD Eric O. Yeates, MD


Resident Physician Resident Physician
Department of Surgery Department of Surgery
UCLA University of California, Irvine
Los Angeles, California Orange, California

James Wu, MD Amy Kim Yetasook, MD


Assistant Clinical Professor of Surgery MIS and Bariatric Surgeon
Department of Surgery Department of Surgery
UCLA Medical Center Harbor-UCLA Medical Center
David Geffen School of Medicine at UCLA Torrance, California;
Los Angeles, California Assistant Professor
Department of Surgery
Tajnoos Yazdany, MD David Geffen School of Medicine at UCLA
Vice Chair of Education Los Angeles, California
Program Director, Obstetrics and Gynecology Residency,
Chief and Program Director, Female Pelvic Medicine and
Reconstructive Surgery,
Associate Professor
David Geffen School of Medicine at UCLA
Harbor-UCLA Medical Center
Torrance, California
AL GRAWANY
Foreword

It is an honor to write the forewor to the thir eition references, to provie a brief summary of essential rel-
of Review of Surgery for ABSITE and Boards by one of evant knowlege. The newest eition also inclues a
the foremost surgical eucators of our time, Dr. Chris- summary of “high-yiel” principles at the beginning
tian e Virgilio. This book grew out of his initial infor- of each chapter, which will further enhance the goal
mal attempts to improve ABSITE scores among his of rapi issemination of essential information on a
own resients at Harbor-UCLA. Over the years, this given topic.
effort has grown an expane, incluing collabora- In aition to serving as a valuable training tool for
tors from multiple institutions, to prouce a book that the in-service examination, it is our hope that this book
has become an essential tool in the surgical resient’s will also inspire the resient to augment their learn-
armamentarium. ing by elving into relevant sections of textbooks an
The most valuable aspect of this book, in my hum- online resources, incluing vieos an pocasts—all
ble opinion, is that in aition to questions testing part an parcel of the total eucational package freely
pure “iactic” knowlege—factois the resient is available to moern surgical trainees. The breath
expecte to learn by rote an memorize—there are an epth of multimeia eucation available toay is
many clinical questions that require an avance level enormous, compare to what I ha as a resient; con-
of cognitive effort. Here, the learner is expecte to syn- versely, the volume of knowlege an technical skills
thesize anatomic an physiologic knowlege within a new surgeons are expecte to learn an master has
clinical context an exercise surgical jugment base also increase signicantly.
on probabilities of ifferent outcomes. Too often, books The oubling of scientic knowlege, in meicine
specically targete at passing multiple-choice exam- an surgery, is now occurring at an exponential pace,
inations ten to skip the latter, in favor of questions an we nee all the help we can get to keep up! I am
that have easy answers—hence the common surgical grateful to Dr. e Virgilio an his colleagues for con-
aphorism that there are more exam questions on the tinuing to invest the effort necessary to upate this
clinical presentation of MEN- synrome than patients wonerful book, so it can continue to serve as a vital
with this isease! Writing questions that test esoteric resource for present an future surgeons.
minutiae is easy; writing questions that promote fur-
ther reaing an stuy of complex surgical scenarios Sharmila Dissanaike, MD, FACS, FCCM
is much harer. I applau Dr. e Virgilio an his col- Peter C. Canizaro Chair,
leagues for reaching this higher goal, while still inclu- University Distinguishe Professor of Surgery,
ing the “knowlege-regurgitation” questions that are Texas Tech University Health Sciences Center
an inevitable part of the stanarize exam process. Lubbock, Texas
Each question is followe by a thoughtful expla-
nation of the right answer, with accompanying

xi
AL GRAWANY
Preface

We are thrille about this thir eition of Review of Finally, we have ae illustrations from an increi-
Surgery for ABSITE and Boards, create to help stuents bly talente surgical illustrator, Dr. Stephanie Cohen,
of surgery prepare for the American Boar of Surgery who is a surgical resient at Beth Israel Deaconess. We
In-Training (ABSITE) an the American Boar of Sur- love her work so much that we aske her to make a
gery (ABS) Qualifying (written) Examination. The rawing for the cover!
original inspiration for the book stemme from a sur- The cover illustration, which combines elements of
gery review program we evelope at Harbor-UCLA art, music, an anatomy, remins us that Surgery is
Meical Center, esigne to stimulate the resients to both an art an a science. To master the arts requires
rea, improve performance on the ABSITE, an en- tremenous eication. Excellent surgical knowlege
hance their likelihoo of passing the ABS examinations is one characteristic that is paramount to becoming an
on the rst try. We were inspire to hear that the rst outstaning surgeon. This requires a lifelong commit-
two eitions prove to be a valuable resource. ment to reaing an then testing your knowlege. We
With that in min, we have strive to make the believe that the ieal way to acquire knowlege is to
3r eition even better with some exciting upates an create a year-roun reaing program. Strive to rea
changes. Areg Grigorian an I have ae three new aily, even if just for 15 minutes.
Assistant Eitors to our team, Drs. Amana Pury, Eric As with the original version, we believe that the
Yeates, an Naveen Balan. All are surgical resients; greatest value of our book lies in the esign of the
Drs. Pury an Balan at Harbor-UCLA an Dr. Yeates questions an the robust responses. The questions are
at UC Irvine. We hanpicke them because of their intene to make you think (try not to get frustrate if
outstaning recor of accomplishment in test taking you miss many of them!). We provie in-epth expla-
an question writing an their emonstrate strong nations for why we feel the correct answer is right an
interest in surgical eucation. We have also ae nu- why the incorrect answers are wrong. Please be aware
merous resients an surgical eucators from aroun that no textbook or review book has all the answers.
the country (an even one from Colombia) as contrib- Some questions an answers may be controversial. If
uting authors. Another important new feature is that you isagree with a question or think you foun an
we ae a summary of high-yiel information at the error, we woul love to hear back from you (our emails
beginning of each chapter. We feel this will serve as a are cevirgilio@lunquist.org an [email protected]u).
rapi-re way to brush up on key points. We have also We sincerely hope you n our review book useful.
ae new, high-yiel questions to remain up-to-ate
with the ever-changing an ynamic el of surgery. Christian de Virgilio and Areg Grigorian

xiii
AL GRAWANY
Acknowledgments

We would like to acknowledge the efforts of Elsevier for Specialist, Beula Christopher, Senior Project Manager,
the timely preparation and publication of this review and Ryan Cook, Book Designer. In addition, we would
book, in particular Jessica McCool, Content Strategist, like to thank the surgery faculty and residents at Harbor-
who helped with the development of this book and sup- UCLA and UC Irvine Medical Centers who assisted in
ported it throughout production, and the contributions the production and inspiration of this project.
made by Shweta Pant, Senior Content Development

xv
AL GRAWANY
Contents

PArt i: PATIENT CARE


1. Abdomen—General, 1
Naveen Balan, Areg Grigorian, and Christian de Virgilio
2. Abdomen—Hernia, 9
Amanda C. Purdy and Amy Kim Yetasook
3. Abdomen—Biliary, 19
Amanda C. Purdy and Danielle M. Hari
4. Abdomen—Liver, 33
Naveen Balan, Kathryn T. Chen, and Danielle M. Hari
5. Abdomen—Pancreas, 47
Joon Y. Park and Danielle M. Hari
6. Abdomen—Spleen, 65
Maria G. Valadez, Benjamin DiPardo, and Eric R. Simms
7. Alimentary Tract—Esophagus, 75
Amanda C. Purdy and Eric R. Simms
8. Alimentary Tract—Stomach, 87
Naveen Balan, Amy Kim Yetasook, and Kathryn T. Chen
9. Alimentary Tract—Small Bowel, 105
Zachary N. Weitzner, Formosa Chen, and Beverley A. Petrie
10. Alimentary Tract—Large Intestine, 123
Joseph Hadaya, Formosa Chen, and Beverley A. Petrie
11. Alimentary Tract—Anorectal, 143
Michael A. Mederos, Formosa Chen, and Beverley A. Petrie
12. Breast, 155
Naveen Balan, Junko Ozao-Choy, and Christine Dauphine
13. Endocrine Surgery, 175
Michael A. Mederos and James Wu
14. Skin and Soft Tissue, 197
Eric O. Yeates, Areg Grigorian, and Christian de Virgilio
15. Surgical Critical Care, 205
Eric O. Yeates and Dennis Kim
16. Trauma, 219
Naveen Balan, Caitlyn Braschi, and Dennis Kim

xvii
xviii ContEnts

17. Vascular—Arterial, 245


Amanda C. Purdy and Nina M. Bowens
18. Vascular—Venous, 265
Amanda C. Purdy and John McCallum
19. Vascular—Access, 275
Luis Felipe Cabrera Vargas, Mark Archie, and Christian de Virgilio
20. Transplant, 283
Joseph Hadaya, Areg Grigorian, and Christian de Virgilio
21. Thoracic Surgery, 291
Jordan M. Rook and Shonda L. Revels
22. Pediatric Surgery, 305
Alexandra Moore, Veronica Sullins, and Steven L. Lee
23. Plastic Surgery, 321
Amanda C. Purdy and Mytien Goldberg
24. Genitourinary, 327
Amanda C. Purdy and Jeremy M. Blumberg
25. Gynecology, 339
Amanda C. Purdy and Tajnoos Yazdany
26. Head and Neck, 347
Zachary N. Weitzner and James Wu
27. Nervous System, 357
Eric O. Yeates and Richard Everson

PArt ii: MEDICAL KNOWLEDGE


28. Anesthesia, 365
Eric O. Yeates and Catherine M. Kuza
29. Fluids, Electrolytes, and Acid-Base Balance, 379
Jordan M. Rook, Areg Grigorian, and Christian de Virgilio
30. Immunology, 391
Kristofer E. Nava and Saad Shebrain
31. Infection and Antimicrobial Therapy, 397
Eric O. Yeates and Jeffry Nahmias
32. Nutrition and Metabolism, 409
Eric O. Yeates, Areg Grigorian, and Christian de Virgilio
33. Oncology and Tumor Biology, 417
Alexandra Moore, Areg Grigorian, and Christian de Virgilio
34. Pharmacology, 427
Eric O. Yeates, Areg Grigorian, and Christian de Virgilio
35. Preoperative Evaluation and Perioperative Care, 437
Naveen Balan, Areg Grigorian, and Christian de Virgilio
36. Transfusion and Disorders of Coagulation, 451
Caitlyn Braschi, Joon Y. Park, and Eric R. Simms
37. Wound Healing, 465
Eric O. Yeates, Areg Grigorian, and Christian de Virgilio

AL GRAWANY
PART I PATIENT CARE

Abdomen—General
NAVEEN BALAN, AREG GRIGORIAN,
AND CHRISTIAN DE VIRGILIO
1
ABSITE 99th Percentile High-Yields
I. Enhance recovery after surgery (ERAS) – associate with a lower overall complication rate, although there
is no ifference in surgical complications or mortality
A. Preoperative optimization
1. Inclues preamission patient eucation on analgesia management after OR, control of meical
comorbiities, smoking cessation, prehabilitation, nutritional care, an correction of anemia
. Ieal patient is ASA 1 or , ambulatory, goo nutritional status; absolute contrainication is urgent
surgery, ASA 4–6, severely malnourishe, or immobile
B. Intraoperative management
1. Stanar anesthesia protocol, minimizing intraoperative uis, preventing intraoperative hypothermia,
maintain normal serum glucose, minimally invasive approach (when feasible), avoi routine use of rains
C. Postoperative care
1. Avoi routine use of nasogastric (NG) tubes, multimoal analgesia to minimize opioi use, use
of epiurals in laparotomy cases, use of TAP (transversus abominis plane) blocks, early urinary
catheter iscontinuation, an early mobilization

QUESTIONS
1. A 56-year-ol male unergoes laparoscopic 2. A 4-year-ol male unergoes laparotomy for an
peritoneal ialysis (PD) catheter placement. anterior abominal stab woun with peritoneal
Several months later the patient comes to the violation. A small perforation of the transverse
emergency epartment reporting problems colon is repaire primarily. While examining the
with his PD catheter. He reports that he can small bowel, an antimesenteric iverticulum is
instill ialysate without ifculty but is unable foun 10 cm proximal from the ileocecal junction.
to withraw ui through the catheter. His It is 3 cm in iameter, 3 cm in height, an there is
abomen is istene an he has mil abominal a brous ban extening from the iverticulum
pain. He is afebrile an not tachycaric. What is to the abominal wall. There is no palpable
the next best step? abnormality ajacent to the iverticulum an no
A. Prompt removal of PD catheter evience or history of GI bleeing. What is the
B. Abominal x-ray appropriate management of the iverticulum?
C. Instill tPA through the catheter A. Obtain aitional imaging postoperatively
D. Intraperitoneal antibiotics B. Diverticulectomy
E. Intravenous antibiotics C. Biopsy
D. Observation
E. Segmental resection
1
2 PArt i Patient Care

3. Which of the following is true about 8. A 50-year-ol male with cirrhotic ascites
intraabominal hypertension (IAH) an seconary to hepatitis C presents with fever,
abominal compartment synrome (ACS)? elevate white bloo cell count, an abominal
A. Diagnosis of ACS is establishe when pain. He has a history of esophageal varices. He
intraabominal pressure is greater than 0 has been on the liver transplant list for 6 months.
mmHg Paracentesis was performe an cultures were
B. Intraabominal hypertension is ene as sent. A single organism grows from the culture.
intraabominal pressure >1 mmHg Which of the following is true regaring this
C. Neuromuscular blockae reuces mortality in conition?
patients with ACS A. It is most likely ue to appenicitis
D. Paracentesis is contrainicate in patients with B. Prophylactic use of uoroquinolone can be
IAH use to prevent this conition
E. Cerebral perfusion is increase in ACS C. In aults, nephrotic synrome is the most
common risk factor
4. Which of the following is true regaring omental D. In chilren, E. coli is the most common isolate
torsion? E. He will likely nee an exploratory laparotomy
A. Seconary torsion is more common than
primary 9. A 74-year-ol male presents to clinic hoping to
B. If surgery is necessary, management consists of have his reucible umbilical hernia repaire
etorsion an omentopexy seconary to increasing but intermittent pain
C. Treatment is usually observation with pain an iscomfort. Two ays before his clinic visit,
control he ha been ischarge from the hospital for
D. The pain is usually in the left lower quarant unstable angina, for which he unerwent balloon
of the abomen angioplasty with placement of a bare metal
E. It typically prouces purulent-appearing coronary artery stent (BMS). When shoul his
peritoneal ui surgery be scheule?
A.  weeks
5. The most common organism isolate from the B. 1 month
infecte peritoneal ui of a patient with a PD C.  months
catheter is: D. 6 months
A. Beta-hemolytic streptococcus E. 1 year
B. Enterococcus
C. Escherichia coli 10. Which of the following is true regaring
D. Coagulase-negative staphylococcus abominal incisions an the prevention of
E. Coagulase-positive staphylococcus incisional hernias?
A. A 4:1 suture:woun length is the current
6. A 70-year-ol woman presents with progressive recommene closure length
abominal pain an abominal istention with B. There is no ifference in hernia occurrence
nonshifting ullness. A CT scan emonstrates between a running closure an an interrupte
loculate collections of ui an scalloping of the closure
intraabominal organs. At surgery, several liters C. A permanent monolament suture is
of yellowish-gray mucoi material are present on preferre in the closure of the fascia in a
the omentum an peritoneal surfaces. Which of running fashion
the following is true about this conition? D. Prophylactic use of mesh after open aortic
A. There is no role for surgical resection aneurysm surgery is not efcacious
B. It is most commonly of ovarian origin E. A 1-cm bite between each stitch is the
C. There is a strong genetic inuence recommene istance uring abominal
D. It is more common in males closure
E. Cytoreuctive surgery may be of benet

7. The most common cause of a retroperitoneal


abscess is:
A. Diverticulitis
B. Appenicitis
C. Renal infection
D. Tuberculosis of the spine
E. Hematogenous sprea from a remote location
CHAPtEr 1 Abdomen—General 3

11. A 55-year-ol obese male presents to the hospital 14. A woman presents with a rm, enlarging mass on
for his bariatric sleeve gastrectomy proceure. His her abominal wall. After appropriate workup,
comorbiities inclue iabetes an hypertension, she is iagnose with a esmoi tumor. Which of
an he states he was iagnose with “walking the following is true about this conition?
pneumonia”  weeks ago an place on A. There is a high rate of metastasis without
antibiotics, which he has nishe. Which of the proper treatment
following woul not be benecial if the SCIP B. The chance of local recurrence is low after
measures for preoperative an postoperative care appropriate intervention
are followe? C. These tumors ten to enlarge uring
A. Placing the patient on an insulin sliing menopause
scale to keep glucose levels between 80 an D. They occur most commonly in women after
10mg/L chilbirth
B. Clipping the patient’s abominal hair with an E. These tumors arise from proliferative
electric shaver before operating chonroblastic cells
C. Aministering anticoagulation on
postoperative ay 1 15. Which of the following is true regaring
D. Aministering antibiotics within 1 hour of retroperitoneal sarcomas?
surgery A. They are best manage by enucleation
E. Discontinuing antibiotics by postoperative B. Prognosis is best etermine by histologic grae
ay 1 C. Fibrosarcomas are the most common type
D. Lymph noe metastasis is common
12. A 3-year-ol female who is 4 weeks pregnant E. Raiation therapy is often curative for small
presents to the emergency epartment with acute sarcomas
onset of abominal pain, fever, an vomiting. She
states that the pain woke her up in the mile of 16. A 75-year-ol female with recently iagnose
the night with suen onset of epigastric pain atrial brillation, for which she was given an
that is now iffuse. She has no vaginal bleeing anticoagulant, presents with suen onset
an fetal monitoring emonstrates normal vitals abominal pain unrelate to oral intake. Surgical
for the fetus. Upon physical exam, the patient has history is remarkable for a total hip arthroplasty
iffuse tenerness with guaring throughout the 3 years ago. Her physical exam is signicant for
abomen, worse in the epigastric region. Pelvic a tener, palpable abominal wall mass above
examination is normal. She has a leukocytosis the umbilicus that persists uring exion of
of 15,000 cells/L. Abominal x-ray series shows abominal wall muscles. The mass is most likely
some ilate bowel loops but no other nings. relate to which of the following?
What is your next step in management of this A. A malignancy
patient? B. Bleeing from the superior epigastric artery
A. Abominal ultrasoun C. Occult trauma
B. CT scan of the abomen/pelvis with contrast D. An intraabominal infection
C. Amit an observe with serial abominal E. Bleeing from the inferior epigastric artery
exams
D. Exploratory laparotomy
E. Diagnostic laparoscopy

13. Which of the following is true regaring a rectus


sheath hematoma?
A. If locate above the umbilicus, it is more likely
to resemble an acute intraabominal process
B. If locate below the umbilicus, it is more likely
to cause severe bleeing
C. The majority are associate with a history of
trauma
D. Operative rainage is the treatment of choice
in most cases
E. Angiographic embolization is not useful
4 PArt i Patient Care

ANSWERS
1. B. PD catheters can become malpositione postopera- 3. B. IAH is ene as an intraabominal pressure
tively espite intraoperative conrmation of proper place- >1 mmHg. This is assesse by measuring the blaer pres-
ment. Instilling ialysate in the peritoneal cavity without the sure while the patient is paralyze. ACS is ene by IAH
ability to remove it may lea to abominal istention an >0 mmHg AND evience of en-organ malperfusion (i.e.,
mil pain. The rst step for a suspecte malpositione PD oliguria) (A). Patients who are mechanically ventilate often
catheter that may have been ippe or kinke is to obtain a have high peak pressures. Primary ACS occurs most com-
KUB. If the catheter appears malpositione, then a reason- monly after surgical proceures associate with massive
able next step woul be to return to the OR for iagnostic resuscitation an tense fascial closure. Seconary ACS is ue
laparoscopy to reposition the catheter. For catheters that are to meical conitions such as ascites or conitions requiring
clogge (resistance to instilling ialysate through the cath- resuscitation without an abominal proceure (i.e., signicant
eter or inability to instill ui), tPA can be use (C). Omen- burn injury). Nasogastric ecompression an neuromuscular
topexy or omentectomy can also be helpful in cases of a blockae are conservative measures to treat IAH but neither
malfunctioning catheter ue to obstruction. Peritonitis is a has been proven to signicantly reuce mortality (C). Reuc-
common complication of PD an accounts for 50% of techni- ing IAH with paracentesis shoul be performe rst in sec-
cal failures. This complication presents with abominal pain, onary ACS ue to ascites (D). In refractory cases an all other
fever, an clouy ialysate. The initial management involves cases of ACS, ecompressive laparotomy shoul be performe
intraperitoneal antibiotics, most commonly vancomycin, expeitiously to lower mortality. The pathophysiology of ACS
which cures 75% of cases without iscontinuation of PD (D). involves compression of the IVC, which can lea to elevate
Patients who continue to become increasingly septic may SVC pressures, an in turn increase intracranial pressures
require intravenous (IV) antibiotics as well (E). Any fungal resulting in ecrease cerebral perfusion pressures (E).
infection of PD requires prompt removal of the catheter (A). Reference: Muresan M, Muresan S, Brinzaniuc K, et al. How
Reference: Miller M, McCormick B, Lavoie S, Biyani M, Zim- much oes ecompressive laparotomy reuce the mortality
merman D. Fluoroscopic manipulation of peritoneal ialysis cathe- rate in primary abominal compartment synrome?: a single-
ters: outcomes an factors associate with successful manipulation. center prospective stuy on 66 patients. Medicine (Baltimore).
Clin J Am Soc Nephrol. 01;7(5):795–800. 017;96(5):e6006.

2. B. This patient has a Meckel iverticulum. This is a 4. A. It is important to be aware of omental torsion because
true intestinal iverticulum that results from the failure it reaily mimics an intraabominal perforation. Because
of the vitelline uct to obliterate uring the fth week it is typically very ifcult to iagnose preoperatively, the
of fetal evelopment. It is the most common congenital iagnosis is most often mae at surgery. Torsion of the omen-
anomaly of the GI tract. Pancreatic heterotopia is foun in tum escribes a twisting of the omentum aroun its vascular
a minority of cases. The most common heterotopic tissue peicle along the long axis. Primary torsion, in which case
foun in resecte specimens is gastric mucosa, which can there is no unerlying pathology, is extremely rare. Secon-
lea to ulcer formation an GI bleeing. Meckel with gas- ary torsion is much more common, an the torsion is usually
tric mucosa is locate at the antimesenteric borer; how- precipitate by a xe point such as a tumor, an ahesion,
ever, ulceration occurs in the opposite mesenteric borer of a hernia sac, or an area of intraabominal inammation.
the ileum. Symptomatic cases require surgical intervention. Omental torsion is much more common in aults in their
The management of an incientally iscovere asymptom- fourth or fth ecae of life. Chilren with torsion are typi-
atic Meckel iverticulum uring abominal exploration cally obese, likely contributing to a fatty omentum that pre-
is a controversial topic. Recently, it has been suggeste to isposes to twisting. Other factors that preispose a patient
selectively intervene on patients with risk factors, namely to torsion inclue a bi omentum an a narrowe omental
age <50, male sex, large iverticulum > cm in iameter, peicle. In primary omental torsion, the twiste omentum
presence of heterotopic tissue, palpation of abnormal no- tens to be localize to the right sie; thus, it is most com-
ules, or presence of brous bans. This patient has three monly confuse with acute appenicitis, acute cholecysti-
inications for removal incluing age <50, male sex, an tis, an pelvic inammatory isease (D). Complicating the
brous ban (D). The ectopic tissue in a Meckel iverticu- iagnosis is the fact that the omentum itself tens to migrate
lum secretes aci leaing to ulcer formation in the ajacent an envelop areas of inammation. Laparoscopy is ieal for
ileum. Thus a segmental bowel resection shoul be per- establishing the iagnosis an excluing other etiologies.
forme in cases of GI bleeing to inclue the iverticulum Treatment is to resect the twiste omentum, which can often
(E). Otherwise, a simple iverticulectomy is appropriate. be infarcte at the time of surgery, an to correct any other
Routine use of 99mTc-pertechnetate scans in asymptomatic relate conition that may be ientie (B, C). The ning of
patients is not inicate (A). Biopsy of a Meckel iverticu- purulent ui woul suggest another iagnosis because it is
lum is not typically require; however, the most common not consistent with omental torsion. The ui usually seen is
cancer in Meckel is carcinoi (C, D). serosanguinous (E).
Reference: Blouhos K, Boulas KA, Tsalis K, et al. Meckel’s iver- References: Chew DK, Holgersen LO, Frieman D. Primary
ticulum in aults: surgical concerns.Front Surg. 018;5:55. omental torsion in chilren. J Pediatr Surg. 1995;30(6):816–817.

AL GRAWANY
CHAPtEr 1 Abdomen—General 5

Sánchez J, Rosao R, Ramírez D, Meina P, Mezquita S, Gallaro cases. Hematogenous sprea is not a signicant contribut-
S. Torsion of the greater omentum: treatment by laparoscopy. Surg ing factor for seconary retroperitoneal abscesses (E). Other
Laparosc Endosc Percutan Tech. 00;1(6):443–445. common causes inclue retrocecal appenicitis (B), perfo-
Young TH, Lee HS, Tang HS. Primary torsion of the greater rate uoenal ulcers, pancreatitis, an iverticulitis (A). In
omentum. Int Surg. 004;89():7–75.
rare cases, patients may have Pott isease, which is a is-
seminate form relate to tuberculosis (D). Patients typically
5. D. Coagulase-negative staphylococci (Staphylococcus epi-
present with back, pelvic, ank, or thigh pain with associate
dermidis) is by far the most common cause of peritoneal cath-
fever an leukocytosis. Flank erythema may be present. Ki-
eter–relate infections (A–C). Staphylococcus aureus is coagu-
ney infections often have gram-negative ros such as Proteus
lase positive (E). Another ening feature of S. aureus is that
an E. coli. Treatment consists of broa-spectrum antibiotics
it is catalase positive. The iagnosis is mae by a combina-
an rainage, an ientication of the source. If the abscess
tion of abominal pain, evelopment of clouy peritoneal
is simple an unilocular, then CT-guie rainage is the
ui, an an elevate peritoneal ui white bloo cell count
treatment of choice. Operative rainage may be require for
greater than 100/mm3. Initial treatment consists of intraper-
complex abscesses.
itoneal antibiotics, which seem to be more effective than IV
antibiotics for a total of  weeks. If the infection fails to clear
8. B. Spontaneous (primary) bacterial peritonitis (SBP) is
base on abominal examination, clinical picture, or per-
ene as bacterial infection of ascitic ui in the absence
sistent peritoneal ui leukocytosis, then the catheter nees
of any surgically treatable intraabominal infection. Patients
to be remove an a temporary hemoialysis catheter will
usually present with fever, iarrhea, an abominal pain,
nee to be inserte. S. aureus an gram-negative organism
but if severe enough, they will also have altere mental
infections are less likely to respon to antibiotic manage-
status, hypotension, hypothermia, an a paralytic ileus.
ment alone.
However, 13% of patients will be completely asymptomatic.
6. E. Pseuomyxoma peritonei is a rare process in which Treatment is with antibiotics alone. Prophylactic antibiotics
the peritoneum becomes covere with semisoli mucus an (with uoroquinolones) to prevent SBP shoul be consi-
large loculate cystic masses. There is no familial preispo- ere in high-risk patients with cirrhosis, ascites, an history
sition (C). A useful classication erive from a large series of gastrointestinal bleeing (as in the present case). Patients
uses two categories: isseminate peritoneal aenomucinosis with cirrhosis who have low ascitic ui protein (<1.0 g/
(DPAM) an peritoneal mucinous carcinomatosis (PMCA). L) an those with a serum bilirubin greater than .5 mg/L
DPAM is histologically a benign process an is most often shoul also be starte on prophylactic antibiotics. Opsonic
ue to a rupture appenix. In one large series, appeniceal or bactericial activity of ascitic ui is relate to protein
mucinous aenoma was associate with approximately 60% concentration. One of the key features of primary peritoni-
of patients with DPAM. In patients classie as PMCA, the tis is that the isolate is usually a single organism an that
origin was either a well-ifferentiate appeniceal or intesti- organism usually is not an anaerobe. Seconary peritonitis
nal mucinous aenocarcinoma (B). Pseuomyxoma peritonei refers to peritonitis in the setting of a bowel perforation.
is most common in women age 50 to 70 years (D). It is often Thus, polymicrobial or anaerobic cultures shoul raise sus-
asymptomatic until late in its course. Symptoms are often picion for bowel perforation (A) an seconary peritonitis
nonspecic, but the most common symptom is increase (E). In aults, the most common pathogens in SBP are the
abominal girth. Physical examination may emonstrate a aerobic enteric ora E. coli an Klebsiella (C). In chilren with
istene abomen with nonshifting ullness. Management nephrogenic or hepatogenic ascites, group A Streptococcus,
is surgical, with cytoreuction of the primary an secon- S. aureus, an Streptococcus pneumoniae are common isolates
ary implants, incluing peritonectomy an omentectomy (D). The iagnosis is mae by paracentesis emonstrating
(A). If there is a clear origin at the appenix, a right colec- more than 50 neutrophils/mm3 of ascitic ui in the pres-
tomy shoul also be performe. If the origin appears to be ence of a correlating clinical presentation. This shoul be
the ovary, total abominal hysterectomy with bilateral salp- evaluate before initiating antibiotics because cultures will
ingo-oophorectomy is recommene. The recurrence rate is return falsely negative. An active infection is consiere a
very high (76% in one series). contrainication for liver transplantation.
References: Gough D, Donohue J, Schutt AJ, et al. Pseuo- References: Bell RB, Seymour NE. Abominal wall, omentum,
myxoma peritonei: long-term patient survival with an aggressive mesentery, an retroperitoneum. In: Brunicari FC, Anersen DK,
regional approach. Ann Surg. 1994;19():11–119. Billiar T, et al., es. Schwartz’s principles of surgery. 8th e. New York:
Hinson FL, Ambrose NS. Pseuomyxoma peritonei. Br J Surg. McGraw-Hill; 1990:1317–138.
1998;85(10):133–1339. Runyon BA. Monomicrobial nonneutrocytic bacterascites: a vari-
Ronnett BM, Zahn CM, Kurman RJ, Kass ME, Sugarbaker PH, ant of spontaneous bacterial peritonitis. Hepatology. 1990;1(4 Pt 1):
Schmookler BM. Disseminate peritoneal aenomucinosis an peri- 710–715.
toneal mucinous carcinomatosis: a clinicopathologic analysis of 109 Turnage RH, Li B, McDonal, JC. Abominal wall, umbili-
cases with emphasis on istinguishing pathologic features, site of cus, peritoneum, mesenteries, omentum an retroperitoneum. In:
origin, prognosis, an relationship to “pseuomyxoma peritonei.” Townsen CM Jr, Beauchamp RD, Evers BM, Mattox KL, es. Sabis-
Am J Surg Pathol. 1995;19(1):1390–1408. ton textbook of surgery: The biological basis of modern surgical practice.
17th e. Philaelphia: W.B. Sauners; 004:1171–1198.
7. C. Primary retroperitoneal abscesses are seconary to
hematogenous sprea while seconary retroperitoneal 9. B. Goo communication between the cariologist an
abscesses are relate to an infection in an ajacent organ. surgeon is essential before performing coronary interven-
The most common source of retroperitoneal abscesses is sec- tions in a patient who requires surgery. Coronary revascu-
onary, with renal infections accounting for nearly 50% of all larization before elective surgery is not recommene if the
6 PArt i Patient Care

patient has asymptomatic coronary artery isease (CAD). 11. A. The Surgical Care Improvement Project (SCIP) is
However, in the setting of an acute coronary synrome a national quality partnership of organizations intereste
(acute myocarial infarction [MI], unstable angina), a percu- in improving surgical outcomes that began in 006. Care is
taneous coronary intervention (PCI) is recommene before taken by all institutions to follow the recommenations by
surgery. The options are to perform balloon angioplasty the Joint Commission because all these outcomes are ocu-
alone or a a bare metal stent (BMS) or a rug-eluting stent mente an measure quarterly. The core measures inclue
(DES). The DES is the best long-term option, but it requires giving antibiotics within 1 hour of surgery (D) an iscon-
a longer elay of surgery. Thus, the ecision of which to use tinuing within 4 hours (E), Foley catheter removal by post-
epens on the urgency of the subsequent operation (urgent, operative ay , an hair removal by clipping on the ay of
time sensitive, or elective) an the feasibility of operating surgery. Shaving the hair off has been shown to increase the
with antiplatelet agents on boar. If the operation is urgent risk of infection (B). Other benecial measures inclue being
(within  weeks), a PCI with balloon angioplasty may be on appropriate venous thromboembolism (VTE) prophylaxis
best because the waiting perio for surgery is  weeks (A). If within 4 hours of surgery an glucose control. The impor-
the operation is time sensitive (–6 weeks), a BMS is a better tance of glucose control an surgical outcomes has been
option because it is less likely to suenly occlue as com- well establishe; however, in 009, the NICE-SUGAR trial
pare with angioplasty alone. However, one shoul wait 1 emonstrate that strict glucose control was actually associ-
month before performing surgery (C). Because this patient ate with worse outcomes. It is now wiely accepte that the
has a relatively symptomatic hernia, the operation is time goal shoul be to keep glucose levels below 180 mg/L (C).
sensitive. Finally, if a DES is place, the recommenation is Reference: NICE-SUGAR Stuy Investigators, Finfer S, Chit-
to wait 6 months before performing surgery (D, E). tock DR, etal. Intensive versus conventional glucose control in criti-
References: Fleisher LA, Fleischmann KE, Auerbach AD, etal. cally ill patients. N Engl J Med. 009;360(13):183–197.
014 ACC/AHA guieline on perioperative cariovascular evalua-
tion an management of patients unergoing noncariac surgery: 12. B. Fear of raiation exposure uring pregnancy shoul
a report of the American College of Cariology/American Heart not take preceence over quickly establishing the correct
Association Task Force on Practice Guielines. J Am Coll Cardiol.
iagnosis an initiating treatment. Base on the patient’s
014;64():e77–e137.
acute onset of symptoms an location, the presentation is
Guyatt GH, Akl EA, Crowther M, Gutterman DD, Schuünemann
HJ, American College of Chest Physicians Antithrombotic Therapy concerning for peritonitis, potentially ue to a perforate
an Prevention of Thrombosis Panel. Executive summary: anti- viscus, such as a peptic ulcer, or a close-loop bowel obstruc-
thrombotic therapy an prevention of thrombosis, 9th e: American tion. In this situation, the best next step woul be to per-
College of Chest Physicians Evience-Base Clinical Practice Guie- form a compute tomography (CT) scan of the abomen (A,
lines [publishe corrections appear in Chest. 141(4):119]. C–E). As a general rule, the care of the patient, not the fetus,
Dosage error in article text. Chest. 01;14(6):1698. shoul take rst priority. Base on the National Guieline
Dosage error in article text]. Chest. 01;141( suppl):7S–47S. Clearinghouse, expeitious an accurate iagnosing shoul
Livhits M, Ko CY, Leonari MJ, Zingmon DS, Gibbons MM, e take preceence over risk of ionizing raiation. The effects
Virgilio C. Risk of surgery following recent myocarial infarction.
of raiation exposure on the fetus epen on the gestational
Ann Surg. 011;53(5):857–864.
age an the amount of raiation. In general, the earlier the
gestational age, the greater the risk. High ose (>10 ras)
10. A. The material an the surgical technique use to close exposure early in pregnancy (within the rst 4 weeks) can
an open abomen are important eterminants of the risk of lea to fetal emise. However, such a high exposure excees
eveloping an incisional hernia. The European Hernia Soci- the ose of typical imaging (abominal x-ray is 00 mra
ety has recently come out with guielines recommening while abominal an pelvic CT is about 3–4 ras). Between
that a small bite closure be performe using at least a 4:1 8 an 15 weeks’ gestation, high-ose (>10 ras) raiation can
suture:woun length uring closure. It has also been shown lea to intrauterine growth retaration an central nervous
that running closure is superior to an interrupte closure (B). efects. Beyon 15 weeks (as in the present case), there o
Prophylactic use of mesh uring closure has been shown to be not appear to be any eterministic effects (ose-epenent
efcacious after open aortic aneurysm surgery because of the events such as fetal loss, congenital efects) on the fetus.
high rate of incisional hernia (D). A ranomize control trial Stochastic effects (those that are not ose epenent), such
looking at small bites compare to large bites has recently as the subsequent risk of cancer or leukemia, are increase
been performe, looking at 560 patients who receive either with exposure of 1 ra or more. The risk is about 1 cancer for
small, 5-mm bites 5 mm apart or large, 1-cm bites 1 cm apart. every 500 exposures. Conversely, if the pregnant patient with
They foun a statistically signicant reuce rate of hernia an acute abomen progresses to peritonitis an bowel per-
occurrence in the small bite group, which is now the recom- foration, the risk of fetal emise is very high. Thus, the risk
mene bite size an length (E). A slowly absorbable monol- of fetal miscarriage is higher with visceral perforation than
ament suture (polyioxanone suture [PDS]) has been shown with raiation exposure, an therefore all measures shoul
to also be the recommene suture in abominal closure (C). be taken for an accurate iagnosis. Magnetic resonance imag-
References: Deerenberg EB, Harlaar JJ, Steyerberg EW, et al. ing (MRI) is consiere a goo imaging option in pregnancy;
Small bites versus large bites for closure of abominal miline inci- however, its use in the emergent setting may be limite by
sions (STITCH): a ouble-blin, multicentre, ranomise controlle its availability. Ultrasoun is also useful but woul be more
trial. Lancet. 015;386(10000):154–160.
useful if the patient presente with right upper quarant
Muysoms FE, Antoniou SA, Bury K, etal. European Hernia Soci-
pain (suspecte biliary isease) or right lower quarant pain
ety guielines on the closure of abominal wall incisions. Hernia.
015;19(1):1–4. (suspecte appenicitis).
CHAPtEr 1 Abdomen—General 7

Reference: Khanelwal A, Fasih N, Kielar A. Imaging of acute Patients are typically in their thir or fourth ecae of life
abomen in pregnancy. Radiol Clin North Am. 013;51(6):1005–10. an present with pain, a mass, or both. They are classie
as either extra abominal (extremities, shouler), abominal
13. B. Rectus sheath hematomas are clinically signicant wall, or intraabominal (mesenteric an pelvic). There are no
because of the fact that they can easily be mistaken for an typical raiographic nings, but MRI may elineate mus-
intraabominal inammatory process. The etiology is an cle or soft-tissue inltration an is require in larger tumors
injury to an epigastric artery within the rectus sheath. In to elineate anatomic relations before surgical intervention.
most cases, there is no clear history of trauma (C). Particu- Core neele biopsy often reveals collagen with iffuse spin-
larly in the elerly who are taking oral anticoagulants, these le cells an abunant brous stroma, which may suggest a
typically occur spontaneously. Patients frequently escribe a low-grae brosarcoma; however, the cells lack mitotic activ-
suen onset of unilateral abominal pain, sometimes pre- ity. An open incisional biopsy of lesions larger than 3 to 4 cm
cee by a coughing t. In one series, 11 of 1 patients were is often necessary. Wie local excision with negative margins
women, an in another series, all 8 were women, with an is inicate for symptomatic esmoi tumors. Nonresect-
average age in the sixth ecae. Below the arcuate line, there able or incientally foun, asymptomatic, intraabominal
is no aponeurotic posterior covering to the rectus muscle. esmoi tumors (even if resectable) shoul be treate with
Therefore, hematomas below this line can cross the miline, nonsteroial antiinammatory agents (e.g., sulinac) an
causing a larger hematoma to form, an then cause bilateral antiestrogens, which have met with objective response rates
lower quarant pain resembling a perforate viscus. On of 50%. In regar to ajuvant therapy, recent retrospective
physical examination, a mass is often palpable. The Fothergill reviews have seen signicant reuctions in recurrence with
sign is the ning of a palpable abominal mass that remains raiation combine with surgery an even with raiation
unchange with contraction of the rectus muscles. This helps alone. More research is necessary for the use of chemotherapy
istinguish it from an intraabominal abscess, which woul agents, but it has been seen that when cytotoxic chemother-
not be palpable with rectus contraction. The iagnosis is best apy agents are use in inoperable esmoi tumors, there is a
establishe with a CT scan, which will emonstrate a ui 0% to 40% positive response. The aggressive nature of these
collection in the rectus muscle. The hematocrit shoul be tumors an high rate of occurrence make esmoi tumors the
closely monitore. Once the iagnosis is establishe, man- secon most common cause of eath in patients with FAP,
agement is primarily nonoperative an consists of resuscita- after colorectal carcinoma.
tion, monitoring of serial hemoglobin/hematocrit levels, an References: Ballo MT, Zagars GK, Pollack A, Pisters PW, Pollack
reversal of anticoagulation (D). However, one shoul be cau- RA. Desmoi tumor: prognostic factors an outcome after surgery,
tious with reversal of anticoagulation, as stable patients may raiation therapy, or combine surgery an raiation therapy. J Clin
benet from continue anticoagulation (e.g., recent mechan- Oncol. 1999;17(1):158–167.
ical valve). On rare occasions, angiographic embolization Hansmann A, Aolph C, Vogel T. High ose tamoxifen an
may be necessary (E). Surgical management, while rarely sulinac as rst-line treatment for esmoi tumors. Cancer.
necessary, woul involve ligation of the bleeing vessel an 004;100(3):61–60.
Janinis J, Patriki M, Vini L, Aravantinos G, Whelan JS. The phar-
evacuation of the hematoma.
macological treatment of aggressive bromatosis: a systematic
References: Berná JD, Zuazu I, Marigal M, García-Meina review. Ann Oncol. 003;14():181–190.
V, Fernánez C, Guirao F. Conservative treatment of large rectus
Nuyttens JJ, Rust PF, Thomas CR Jr, Turrisi AT 3r. Surgery
sheath hematoma in patients unergoing anticoagulant therapy.
versus raiation therapy for patients with aggressive bromato-
Abdom Imaging. 000;5(3):30–34.
sis or esmoi tumors: a comparative review of  articles. Cancer.
Zainea GG, Joran F. Rectus sheath hematomas: their pathogene-
000;88(7):1517–153.
sis, iagnosis, an management. Am Surg. 1988;54(10):630–633.
15. B. Most retroperitoneal tumors are malignant an com-
14. D. Desmoi tumors are unusual soft-tissue neoplasms prise approximately half of all soft-tissue sarcomas. The
that arise from fascial or bro-aponeurotic tissue. They are most common sarcomas occurring in the retroperitoneum
proliferations of benign-appearing broblastic cells with are liposarcomas, malignant brous histiocytomas, an
abunant collagen an few mitoses (E). Desmoi tumors leiomyosarcomas (C). Approximately 50% of patients will
o not metastasize (A); however, they are locally aggressive have a local recurrence an 0% to 30% will en up having
an have a very high local recurrence rate reaching almost istant metastases. Lymph noe metastases are rare (D).
50% (B). They have been associate with Garner synrome Retroperitoneal sarcomas present as large masses because
(intestinal polyposis, osteomas, bromas, an epiermal or they o not typically prouce symptoms until their mass
sebaceous cysts) an familial aenomatous polyposis (FAP), effect creates compression or invasion of ajacent struc-
which is why patients shoul be scheule for a colonos- tures. Symptoms may inclue gastrointestinal hemorrhage,
copy soon after iagnosis. In sporaic cases, surgical trauma early satiety, nausea, vomiting, an lower extremity swell-
appears to be an important cause. Desmoi tumors may ing. Retroperitoneal sarcomas have a worse prognosis than
evelop within or ajacent to surgical scars. Patients with nonretroperitoneal sarcomas. The best chance for long-term
FAP have a 1000-fol increase risk of the evelopment of survival is achieve with an en bloc, margin-negative resec-
esmoi tumors. Desmois are more common in women of tion. Tumor stage at presentation, high histologic grae,
chilbearing age, ten to occur after chilbirth, an may be unresectability, an grossly positive resection margins are
linke to estrogen. Oral contraceptive pills (OCP) have also strongly associate with increase mortality rates. Tumor
been foun to be associate with the occurrence of these grae is the most signicant preictor of outcome. Complete
tumors, whereas antiestrogen meications may lea to shrink- surgical resection is the most effective treatment for primary
age. They’ve been reporte to shrink after menopause (C). or recurrent retroperitoneal sarcomas (A, E). Surgical cure
8 PArt i Patient Care

can be limite because the margins are often compromise mass is palpable even uring exion of abominal wall mus-
by anatomic constraints. There is no ifference in survival cles, helping to ifferentiate this from an intraperitoneal pro-
between those who ha a resection with a grossly positive cess (Fothergill sign) (D). In a review of 16 patients by Mayo
margin an those with inoperable tumors. Unlike extremity Clinic, anticoagulation was associate with 70%. Above the
sarcomas, external beam raiation therapy is limite for ret- arcuate line, the etiology is often relate to a lesion to the
roperitoneal malignancies because there is a low tolerance superior epigastric artery within the rectus sheath (E). In
for raiation to surrouning structures. Postoperative an most cases, there is no clear history of trauma (C). In partic-
intraoperative raiation therapy have been shown to reuce ular, in the elerly who are taking oral anticoagulants, they
local recurrence, but further stuies are neee to etermine typically occur spontaneously. The most common treatment
if this leas to improve survival. for patients with rectus sheath hematomas is rest, analgesics,
Reference: Lewis JJ, Leung D, Wooruff JM, Brennan MF. Ret- an bloo transfusions as necessary. In general, coagulop-
roperitoneal soft-tissue sarcoma: analysis of 500 patients treate an athies are correcte; however, continuing anticoagulation
followe at a single institution. Ann Surg. 1998;8(3):355–365. may be pruent in select patients (e.g., biomechanical valve,
recent sale embolus). In extreme cases, angioembolization
16. B. This patient was recently iagnose with atrial may be require.
brillation an starte on oral anticoagulants. One shoul References: Alla VM, Karnam SM, Kaushik M, Porter J. Sponta-
suspect a rectus sheath hematoma in oler patients taking neous rectus sheath hematoma. West J Emerg Med. 010;11(1):76–79.
anticoagulants who present with the clinical tria of acute Cherry WB, Mueller PS. Rectus sheath hematoma: review of 16
abominal pain, an abominal wall mass, an anemia. The cases at a single institution. Medicine (Baltimore). 006;85():105–110.
Abdomen—Hernia
AMANDA C. PURDY AND AMY KIM YETASOOK 2
ABSITE 99th Percentile High-Yields
I. Abominal Wall Hernia
a. From skin to peritoneum: skin → fascia of Camper → fascia of Scarpa → external oblique → internal
oblique → transversus abominis → transversalis fascia → preperitoneal fat → peritoneum
b. Superior to arcuate line:
1. Anterior sheath comprise of aponeurosis of external oblique an the anterior half of the aponeurosis
of internal oblique
. Posterior sheath comprise of aponeurosis of transversus abominis an aponeurosis of the posterior
half of internal oblique; posterior sheath not present inferior to arcuate line
c. Ten to 15% of all incisions will evelop into ventral (incisional) hernia; woun infection after surgery
oubles risk of a hernia evelopment
. Miline epigastrium is a physiologic area of weakness in the abomen where patients can evelop
iastasis recti an/or epigastric hernia; risk factors inclue pregnancy an weight gain
e. Diastasis recti: attenuation of linea alba causing rectus muscle separation; when the rectus contract, a
bulge appears in the upper miline abomen; no fascial efect, not a hernia

II. Umbilical Hernias


a. Peiatric umbilical hernias
1. Congenital efect, repair by age 5 or sooner if symptomatic
b. Umbilical hernias in aults
1. Acquire efect, increase intraabominal pressure causes weakening of transversalis fascia an of
the umbilical ring
. Women are 3 times more likely to evelop umbilical hernias than men (ue to pregnancy), an up to
90% of women evelop an umbilical hernia uring pregnancy; incarceration occurs more in men
3. Cirrhotic patients with uncomplicate umbilical hernias shoul be meically optimize before
unergoing elective surgical repair; this inclues free water restriction, iuretics, an large volume
paracentesis (with infusion of albumin); mesh can be use

III. Inguinal Hernias


a. Cremaster muscle bers arise from internal oblique muscle, inguinal ligament from external oblique
muscle; the internal oblique an transversalis fascia form the internal ring of the inguinal canal; the
conjoint tenon is the lower common aponeurosis of the internal oblique an the transversus abominis
b. Though wiely believe as true, there is little evience to support physical activity with inguinal hernia
evelopment; inguinal hernias have increase type 3 collagen an ecrease type 1 collagen
c. During issection, an inirect hernia sac is foun on the anteromeial aspect of spermatic cor
. The genital branch of the genitofemoral nerve supplies sensation to the mons an labia majora in women,
an in men it supplies motor to the cremaster an sensation to the scrotum; it runs within the spermatic
cor an exits via the eep inguinal ring

9
10 PArt i Patient Care

e. The iliohypogastric nerve arises from the rst lumbar branch an travels between the transversus
abominis an the internal oblique muscles
f. The ilioinguinal nerve runs anterior to the spermatic cor in men or roun ligament in women an
passes through the supercial inguinal ring; supplies sensation to the upper meial thigh
g. Peiatric inguinal hernias (ue to a congenital failure of the processus vaginalis to close):
1. Repair only requires high ligation of the hernia sac (ligation at the internal ring)

IV. Hernia Repair


a. Open repair
i. Open repair with mesh (<10% recurrence rate)
1. Lichtenstein repair (tension free): mesh is suture meially to the transversus abominis arch, an
the lateral ege of mesh is suture to the inguinal ligament; mesh shoul overlap  cm over pubic
tubercle an  to 4 cm lateral to the internal ring
ii. Open repair without mesh (30%–50% recurrence rate)
1. Bassini repair: the conjoint tenon is suture to the inguinal ligament with interrupte,
nonabsorbable sutures
. McVay repair: the conjoint tenon is suture to Cooper ligament (also calle the pectineal
ligament); nee to expose Cooper ligament by incising transversalis fascia an entering
preperitoneal space; relaxing incision has to be mae in the anterior rectus sheath; repairs both
inguinal an femoral hernias
3. Shoulice repair: the transversalis fascia is incise an reapproximate, 4-layer closure with
running suture
b. Laparoscopic repair
i. Best option for recurrent hernias (previously treate with an open approach), bilateral hernias, an
obese patients; contrainicate in patients with large scrotal hernias or who have unergone prior
extensive lower abominal or pelvic surgery
ii. Mesh xation inferior to the iliopubic tract an lateral to the epigastric vessels (triangle of pain) is
avoie because of the risk of injury to the lateral femoral cutaneous nerve; similarly, avoi xation
below the internal ring, which is borere laterally by spermatic vessels an meially by vas eferens
(triangle of oom), as this risks injury to the external iliac artery an vein
iii. The two laparoscopic proceures: transabominal preperitoneal (TAPP) an totally extraperitoneal
(TEP) repair
1. TAPP: the peritoneal space is entere at umbilicus, peritoneum overlying inguinal oor is
issecte away as a ap, hernia is reuce, mesh is xe over internal ring opening in the
preperitoneal space, an peritoneum is reapproximate; can also examine contralateral sie
. TEP: the preperitoneal space is evelope w issecting balloon inserte between posterior rectus
sheath an rectus abominis an irecte towar pelvis inferior to arcuate ligaments; other ports
inserte into preperitoneal space without ever entering peritoneal cavity; avantage here is that
you never open peritoneum so no mesh exposure to abominal organs, so less ahesions
c. If chronic groin pain after hernia surgery, perform a pelvic MRI; risk of chronic groin pain after hernia
repair is 15% to 0%
. Most common location for breakown of laparoscopic hernia repair is the meial portion of the mesh;
often because the mesh is too small or that it was not appropriately attache meially

V. Femoral Hernia
a. Femoral triangle: femoral vein laterally, auctor longus meially an inguinal ligament superiorly
b. Bounaries of femoral canal: superior (inguinal), meial (lacunar ligament), lateral (femoral vein), an
posterior or oor (iliacus an psoas tenon; fascia of pectineus)
c. All femoral hernias shoul be repaire as they have a 15% to 0% risk of strangulation
CHAPtEr 2 Abdomen—Hernia 11

VI. Miscellaneous Hernias


a. Obturator hernia: Howship-Romberg sign is internal thigh pain with external rotation; repair for
obturator hernia shoul be an abominal approach as this allows access to the efect an reuction of
contents, may require incision of the obturator membrane
b. Petit hernia is boun by latissimus orsi, iliac crest, an external oblique muscle; Grynfeltt hernia is
boun by sacrospinous muscle, internal oblique, an the 1th rib
c. Spigelian hernia is efect through Spigelian fascia, which is area between semilunar line an lateral
borer of rectus abominis; majority occur just below arcuate line where no posterior rectus sheath exists
. A sports hernia is not a true hernia; it is a weakness of the inguinal oor eveloping in those with
signicant physical activity (athletes); pressure is place on the genital branch of the genital femoral
nerve an can lea to chronic groin pain; on exam, there may be tenerness with palpation of the
inguinal oor through the external ring, but no hernia is ientie with Valsalva

Fig. 2.1
12 PArt i Patient Care

Fig. 2.2

QUESTIONS
1. A 45-year-ol woman with iabetes mellitus 3. A 30-year-ol patient unerwent exploratory
an a BMI of 35 kg/m presents to clinic for an laparotomy for trauma. Which of the following
intermittent, painful bulge in her mi-abomen closure techniques is associate with the lowest
over an ol miline laparotomy scar. On exam, risk of eveloping an incisional hernia?
there is a reucible miline bulge with a 7 by A. Placing stitches 1 cm apart an 1 cm from the
3 cm fascial efect. She woul like to procee fascial ege
with surgery. What is the most appropriate B. Placing sutures 5 mm apart an 5 mm from
management? the fascial ege
A. Physical therapy referral for abominal wall C. Placing stitches 1 cm apart an 5 mm from the
strengthening fascial ege
B. Open hernia repair with onlay mesh D. Using running suture with a suture to woun
C. Open hernia repair with sublay mesh length ratio of :1
D. Laparoscopic hernia repair with mesh E. Using running suture with a suture to woun
E. Component separation an primary repair length ratio of 3:1

2. A 55-year-ol man with a history of 4. A 60-year-ol woman with chronic kiney isease
abominoperineal resection for rectal cancer two is unergoing elective peritoneal ialysis catheter
years ago has intermittent pain an fullness next placement. At her preoperative appointment, she
to his colostomy that is sometimes associate with is note to have a small, nontener, reucible
nausea an vomiting. On exam, his colostomy inguinal hernia. She says it has been there for
appears healthy, an no bulge is palpate. CT years an that it oes not bother her. What is the
emonstrates a loop of bowel supercial to the most appropriate management?
fascia that is ajacent to the stoma. What is the A. Peritoneal ialysis catheter placement alone
best management? B. Inguinal hernia repair with mesh with
A. Primary repair of parastomal hernia peritoneal ialysis catheter placement 6 weeks
B. Relocate the colostomy later
C. Repair with prosthetic mesh C. Inguinal hernia repair without mesh with
D. Repair with biologic mesh peritoneal ialysis catheter placement 6 weeks
E. Reassurance an return precautions later

AL GRAWANY
CHAPtEr 2 Abdomen—Hernia 13

D. Concurrent inguinal hernia repair with mesh C. Violation of the peritoneum uring a
an peritoneal ialysis catheter placement totally extraperitoneal (TEP) repair requires
E. Concurrent inguinal hernia repair without conversion to an open or transabominal
mesh an peritoneal ialysis catheter placement preperitoneal (TAPP) approach
D. Persistent numbness or pain of the lateral
5. The genital branch of the genitofemoral nerve: thigh is more common with open versus
A. is typically foun anteriorly on top of the laparoscopic repair
spermatic cor E. Laparoscopic repair will prevent him from
B. provies sensation to the base of the penis an eveloping a femoral hernia in the future
inner thigh
C. typically lies on the anterior surface of the 9. A 8-year-ol male patient is asking for avice
internal oblique muscle on whether to pursue open mesh repair or TEP
D. if cut will result in ipsilateral loss of repair of a newly iagnose, reucible right-sie
cremasteric reex inguinal hernia. What can you tell the patient
E. often intermingles with the iliohypogastric nerve about these two methos of repair?
A. Chronic pain is reuce with an open mesh
6. Which of the following is true regaring hernia repair
anatomy? B. Operative time is not signicantly ifferent
A. Poupart ligament is forme from the between the two
anteroinferior portion of the external oblique C. TEP repair is associate with a quicker return
aponeurosis to work an normal activities
B. The cremaster muscle arises from the D. Open mesh repair is associate with a higher
transversus abominis muscle rate of intraoperative complications
C. The genital branch of the genitofemoral nerve E. Recurrence is relatively common (>5%) no
passes through the supercial ring matter which metho is chosen
D. The femoral branch of the genitofemoral nerve
innervates the cremasteric muscle 10. One hour after laparoscopic repair of a left
E. Inirect hernias most often arise within the inguinal hernia, the patient complains of severe
borers of the rectus muscle, inferior inguinal burning groin pain. Which of the following is the
ligament, an inferior epigastric artery most appropriate recommenation?
A. Immeiate return to the OR for laparoscopy
7. Which of the following is true regaring the B. Nonsteroial antiinammatory rugs
arcuate line? C. Neurontin
A. It is usually locate a few centimeters above D. Opioi analgesia
the umbilicus E. Inject groin region with local anesthetic
B. Below this line, the internal oblique
aponeurosis splits 11. Ischemic orchitis after inguinal hernia repair is
C. Below this line, the rectus muscle lies on the most often ue to:
transversalis fascia A. Too tight a reconstruction of the inguinal ring
D. Below this line, the posterior rectus sheath is B. Preexisting testicular pathology
thinner C. Inavertent ligation of the testicular artery
E. Below this line, the external oblique muscle D. Completely excising a large scrotal hernia sac
oes not contribute to the anterior rectus E. Anomalous bloo supply to the testicle
sheath
12. A 45-year-ol man presents with an
8. A 55-year-ol male presents with a painful bulge asymptomatic right inguinal hernia. It is easily
in the left groin that rst appeare several months reuce with gentle pressure. Which of the
ago. His surgical history inclues a right-sie following is true about this conition?
open inguinal hernia repair. Upon examination A. The likelihoo of strangulation eveloping is
you also note a bulge in the right groin over his high without surgery
previous incision. Both masses are reucible. B. Without surgery, intractable pain will most
Which of the following is true regaring this likely evelop
patient’s conition? C. Waiting until symptoms evelop is a
A. Open repair is preferre reasonable alternative to surgery
B. In laparoscopic repair, failure to tack the mesh D. Laparoscopic repair is the best option
lateral to the inferior epigastric vessels can E. If the hernia is small, there is a lower chance of
lea to recurrence through the internal ring incarceration
14 PArt i Patient Care

13. A 5-month-ol previously full-term male infant D. Small, asymptomatic hernias can be clinically
presents with a tener left groin mass that has observe
been present for the past several hours. There is E. Primary closure has recurrence rates similar to
slight erythema over the skin. He is afebrile an those of mesh repair
his labs are normal. Which of the following is the
best next step? 16. Which of the following is true regaring femoral
A. Attempt manual reuction, an if successful, hernias?
scheule surgical repair when infant reaches 1 A. They are the most common hernia in females
year of age B. The Cooper ligament is consiere the anterior
B. Attempt manual reuction, an if successful, borer of the femoral canal
immeiately take to the operating room for C. They are lateral to the femoral vein
surgical repair D. Repair involves approximating the iliopubic
C. Attempt manual reuction, an if successful, tract to the Cooper ligament
scheule repair in  ays E. A Bassini operation is consiere an
D. Attempt manual reuction, an if successful, appropriate surgical option
scheule left-sie surgical repair with
contralateral groin exploration in  ays 17. A 55-year-ol woman presents with a painless
E. Take immeiately to the operating room for abominal wall bulge. She reports a successful
operative repair iet an exercise program an has lost almost
40 kg over the past  years. However, she is
14. Which of the following best escribes umbilical worrie because yesteray when she was
hernias in chilren? sitting up in be, she notice an upper miline
A. They have a signicant risk of incarceration. abominal bulge that looks like a large rige
B. Repair is inicate once an umbilical hernia is between her rib cage an belly button. On
iagnose physical exam the bulge becomes visible when
C. Repair shoul be performe if the hernia she lifts her hea off the be. Which of the
persists beyon 6 months of age following is true regaring her conition?
D. Most close spontaneously A. Surgical repair shoul be one immeiately
E. Repair shoul be performe only if the chil is before signs of incarceration evelop
symptomatic B. There are both congenital an acquire
etiologies
15. Which of the following is true regaring umbilical C. A strict regimen of abominal wall exercises
hernias in aults? usually results in complete resolution
A. Most are congenital D. The efect is limite to the transversalis fascia
B. Repair is contrainicate in patients with E. Typically these efects contain only
cirrhosis preperitoneal fat
C. Strangulation is less common than in chilren

ANSWERS
1. D. This patient has a symptomatic ventral incisional her- minimizing tension. This is unnecessary in this case, as the
nia. The best option for repair in this patient with multiple efect is only 3 cm wie, an a minimally invasive technique
risk factors for perioperative infection (iabetes an obe- is more appropriate (E). Abominal wall strengthening exer-
sity) is laparoscopic hernia repair with mesh. Compare to cises are the primary repair for rectus iastasis, which is an
open incisional hernia repair, laparoscopic repair has a lower attenuation of the linea alba in the superior abominal wall
incience of surgical site infection an is the best option for without a true hernia. This patient has a hernia, as evience
patients at risk for postoperative infection (C–D). Open an by fascial efect on physical exam (A).
laparoscopic ventral hernia repairs with mesh have similar Reference: Guielines for laparoscopic ventral hernia repair.
recurrence rate. Component separation is a technique where SAGES. Publishe June 7, 016. https://www.sages.org/publications/
the anterior rectus sheath is incise  cm lateral to the semi- guielines/guielines-for-laparoscopic-ventral-hernia-repair
lunar line in orer to primarily close large efects while
CHAPtEr 2 Abdomen—Hernia 15

2. C. This patient has a parastomal hernia. Although the inci- sensation to the sie of the scrotum an the labia. It is respon-
ence of parastomal hernias is higher with en ostomies than sible for the cremasteric reex. In women, it accompanies
with loop ostomies, this may simply be ue to loop ostomies the roun ligament of the uterus. The genital branch of the
getting reverse more often, an sooner than en ostomies genitofemoral nerve is part of the cor structures. It lies on
that are more often permanent. The majority of parastomal the iliopubic tract an accompanies the cremaster vessels (B).
hernias are asymptomatic an o not require intervention. The ilioinguinal nerve lies on top of the spermatic cor (A). It
However, this patient is experiencing symptoms with inter- innervates the internal oblique muscle an is sensory to the
mittent bowel obstruction an shoul unergo repair (E). The upper meial thigh ajacent to the genitalia. The nerve can
best option for management of a symptomatic parastomal her- sometimes splay out over the cor, making issection if-
nia is to take the ostomy own if appropriate. Unfortunately, cult. The iliohypogastric an ilioinguinal nerves arise from
this is not an option for this patient with a prior abomino- the T1-L1 level an intermingle. They provie sensation to
perineal resection (APR). The next best option is repair of the the skin of the groin, the base of the penis, an the upper
hernia with synthetic mesh using the Sugarbaker technique, meial thigh. The iliohypogastric nerve lies on the internal
where intraperitoneal mesh covers the entire efect, an the oblique muscle (C), provies sensory innervation from the
bowel leaing to the ostomy enters laterally between the skin overlying the pubis, an oes not intermingle with the
mesh an abominal wall. Biologic mesh is associate with genitofemoral nerve because they cross ifferent paths (E).
higher recurrence rates compare to prosthetic mesh (D). It Reference: Wantz GE. Testicular atrophy an chronic resiual
may be consiere for patients with signicant contamina- neuralgia as risks of inguinal hernioplasty. Surg Clin North Am.
tion. Primary repair of parastomal hernias has been largely 1993;73(3):571–581.
abanone ue to unacceptable recurrence rates of up to
70% (A). Ostomy relocation solves the problem at han (the 6. A. Poupart ligament is another name for the inguinal lig-
current symptomatic parastomal hernia); however, it is infe- ament. The inguinal ligament is forme from the anteroinfe-
rior to repair with mesh as there is a high risk of eveloping rior portion of the external oblique aponeurosis foling back
another parastomal hernia at the new ostomy site (B). on itself. It extens from the anterosuperior iliac spine to the
Reference: Hansson BM, Slater NJ, van er Velen AS, et al. Sur- pubic tubercle, turning posteriorly to form a shelving ege.
gical techniques for parastomal hernia repair: a systematic review of The cremaster muscle bers arise from the internal oblique
the literature. Ann Surg. 01;55(4):685–695. muscle an surroun the spermatic cor (B). The genital
branch of the genitofemoral nerve passes through the eep
3. B. After vertical miline abominal incision, approx- ring (C), whereas the ilioinguinal nerve passes through the
imately 10% to 0% of patients evelop incisional hernias. supercial ring. The genital branch innervates the cremas-
Ranomize controlle trials have shown that small (5 mm) ter muscle, whereas the femoral branch controls sensation to
fascial bites 5 mm apart have a signicantly lower rate of the upper lateral thigh (D). Inirect hernias arise lateral to
eveloping incisional hernia than large (1 cm) bites 1 cm the inferior epigastric vessels, whereas irect hernias arise
apart (A, C). Also, a suture to woun length ratio of at least meial to the inferior epigastric vessels. The lateral borer
4:1 is associate with less tension an a ecrease incience of the rectus muscle, inferior inguinal ligament, an inferior
of incisional hernia evelopment (D, E). epigastric artery ene the borers of Hesselbach triangle
References: Deerenberg EB, Harlaar JJ, Steyerberg EW, et al. an ene the location of a irect hernia (E).
Small bites versus large bites for closure of abominal miline inci-
sions (STITCH): a ouble-blin, multicentre, ranomise controlle
7. C. The arcuate line is locate below the umbilicus, typ-
trial. Lancet (London, England). 015;386(10000):154–160.
Millbourn D, Cengiz Y, Israelsson LA. Effect of stitch length on
ically one-thir the istance to the pubic crest (A). Between
woun complications after closure of miline incisions: a ranom- the costal margin an the arcuate line, the anterior rectus
ize controlle trial. Arch Surg. 009;144(11):1056–1059. sheath is mae up of a combination of the aponeurosis of the
external an internal oblique muscles. The posterior sheath is
4. D. Conitions that increase intraabominal pressure mae up of a combination of the aponeuroses of the internal
(cystic brosis, chronic lung isease, ventriculoperitoneal oblique an transverse abominal muscles. Below the arcu-
shunts, constipation, an peritoneal ialysis) are associate ate line, the anterior sheath is mae up of the aponeuroses
with higher risk for eveloping an inguinal hernia. Patients of all three abominal muscles (E). The internal oblique apo-
with small asymptomatic hernias are at risk for eveloping neurosis splits above the arcuate line to envelop the rectus
symptoms as their hernias enlarge uring peritoneal ial- abominis muscle (B). There is no posterior sheath below the
ysis. Therefore, everyone unergoing peritoneal ialysis arcuate line (D), an the transversalis fascia therefore makes
shoul be examine for presence of abominal hernias pre- up the posterior aspect of the rectus abominis muscle.
operatively. If a hernia is foun, the patient shoul unergo
concurrent herniorrhaphy at the time of peritoneal ialysis 8. E. This patient has bilateral inguinal hernias, one of
catheter placement (A–C). Hernia repair shoul be one with which is recurrent an shoul be offere a laparoscopic
mesh, as mesh is associate with ecrease recurrence rates repair. The avantages of this inclue the ability to visualize
an are safe in patients unergoing peritoneal ialysis (E). both sies through a single incision an a potentially eas-
Reference: Chi Q, Shi Z, Zhang Z, Lin C, Liu G, Weng S. Ingui- ier surgery in the setting of recurrence. It also protects the
nal hernias in patients on continuous ambulatory peritoneal ialysis: patient from eveloping a femoral hernia since the femoral
is tension-free mesh repair feasible? BMC Surg. 00;0(1):310. canal is covere by the mesh. Of note, femoral hernias are
known to evelop after open inguinal hernia repair. They
5. D. The genitofemoral nerve arises from the L1-L level. evelop on average sooner than a typical recurrence, sug-
The genital branch innervates the cremaster muscle an gesting that the original hernia was in fact a femoral one an
16 PArt i Patient Care

was misse at the original surgery. The two laparoscopic thought to be entrapment of the nerve uring surgery or
approaches inclue TEP an TAPP. TEP involves issecting postoperative scarring. Chronic groin pain is best worke up
a plane in the preperitoneal space, which may actually be with MRI. If conservative management oes not resolve the
avantageous when compare to TAPP because intraab- pain, operative exploration an ivision of the nerve(s) have
ominal ahesions are avoie (A). This oes not hol true met with success. The ieal approach in the setting of her-
for prior pelvic surgery as the preperitoneal space may be nia reoperation after open repair is to enter a space in which
obliterate in these patients, necessitating a TAPP. If the peri- the tissue planes have not been violate. The preferre man-
toneum is violate uring TEP, it is important to repair the agement is a laparoscopic retroperitoneal triple neurectomy,
efect to prevent ahesion formation postoperatively, but which allows a single stage approach to access the ilioingui-
it is not manatory to convert to a ifferent technique (C). nal, iliohypogastric, an genitofemoral nerves.
Though there are few absolute contrainications to laparo-
scopic hernia surgery, bowel ischemia with perforation or 11. D. Ischemic orchitis is thought to evelop as a result of
sepsis preclues the use of mesh, which is require in both thrombosis of veins of the pampiniform plexus, leaing to
TEP an TAPP. Tacking of the mesh in either laparoscopic testicular venous congestion. It has thus been terme con-
approach can reuce mesh migration but shoul be avoie gestive orchitis. The precise etiology of ischemic orchitis is
lateral to the epigastric vessels an inferior to the iliopubic unclear. The most commonly ientie risk factor is exten-
tract to avoi placement in the “triangle of oom” or the “tri- sive issection of the spermatic cor. This occurs particularly
angle of pain,” which contains the external iliac vessels an when a patient has a large hernia sac, an the entire istal
several nerves (lateral femoral cutaneous an femoral branch sac is issecte an excise. As such, it is recommene that
of genitofemoral, respectively) (B). Injury to these nerves is the sac instea is ivie an the istal sac left in situ. In
relatively specic to laparoscopic repairs (D). aition, the cor shoul never be issecte past the pubic
Reference: Fischer JE. Fischer’s mastery of surgery. Wolters Klu- tubercle. The presentation is that of a swollen, tener testicle,
wer Health/Lippincott Williams & Wilkins; Chicago, IL, 01. usually  to 5 ays after surgery. The testicle is often high
riing. This may eventually progress to testicular atrophy.
9. C. The preferre initial approach for an uncomplicate Scrotal uplex ultrasonography has been shown to be useful
inguinal hernia is still actively ebate within the surgi- in evaluating the perfusion of the testicle after hernia repair.
cal community. The LEVEL-trial specically compare TEP However, it oes not change the management of ischemic
repair versus open mesh repair an emonstrate reuce orchitis. Management is expectant. In the past, attempts to
pain in the immeiate postoperative perio an earlier return reexplore the groin were unertaken to try to loosen the
to work. However, this came at the expense of longer operat- inguinal ring, but this was not successful (A). The bloo sup-
ing room times an higher intraoperative complication rates ply to the testicle is via the testicular artery, but there are rich
(B, D). This seems to be consistent with the results of a New collaterals incluing the external spermatic artery an the
England Journal of Medicine (NEJM) stuy from 004 comparing artery to the vas. Thus, inavertent ligation of the testicular
open mesh repair to all methos of laparoscopic mesh repair. artery oes not typically lea to this complication (C). Preex-
However, they iverge on reporte recurrence rates, with the isting testicular pathology (B) or anomalous bloo supply (E)
NEJM stuy favoring open repair (recurrence of 4% versus to the testicle is not thought to contribute to ischemic orchitis
10.1%) while the LEVEL-Trial showe equivalent recurrence following inguinal hernia repair. However, ischemic orchi-
rates (3.0% for open an 3.8% for TEP) (E). The LEVEL-Trial tis can occur more frequently in recurrent inguinal hernia
also inicate an equivalent prevalence of chronic pain, which surgery using the anterior approach; thus, the laparoscopic
was not one of the outcomes in the NEJM article (A). approach shoul be consiere for recurrent hernias.
References: Langevel HR, van’t Riet M, Weiema WF, et al. References: Holloway B, Belcher HE, Letourneau JG, Kun-
Total extraperitoneal inguinal hernia repair compare with Lichten- berger LE. Scrotal sonography: a valuable tool in the evaluation of
stein (the LEVEL-Trial): a ranomize controlle trial. Ann Surg. complications following inguinal hernia repair. J Clin Ultrasound.
010;51(5):819–84. 1998;6(7):341–344.
Neumayer L, Giobbie-Hurer A, Jonasson O, et al. Open mesh Wantz GE. Testicular atrophy an chronic resiual neuralgia as
versus laparoscopic mesh repair of inguinal hernia. N Engl J Med. risks of inguinal hernioplasty. Surg Clin North Am. 1993;73(3):571–581.
004;350(18):1819–187.
12. C. A large prospective ranomize stuy in men
10. A. Severe groin pain eveloping in the recovery room emonstrate that watchful waiting for patients with
following laparoscopic hernia repair is most likely ue to a asymptomatic or minimally symptomatic inguinal hernias
stapling/tacking injury to a nerve. If this complication is sus- is an acceptable option for surgery (D). The patients were
pecte, the patient shoul return to the operating room to followe for as long as 9 years. Acute hernia incarceration
remove the offening tack. Acute groin pain is most likely without strangulation evelope in only one (0.3%) patient,
from injury to the ilioinguinal nerve. However, the most an acute incarceration with bowel obstruction evelope
commonly injure nerve uring laparoscopic hernia repair in only one (A). Approximately one-fourth of the watchful
is the lateral femoral cutaneous nerve (provies sensation waiting group eventually crosse over to receive surgical
to the lateral thigh). Injecting the groin with local anesthetic repair ue to increase hernia-relate pain (B). Smaller her-
may not relieve the pain an if it works, it will only be a nias ten to have a smaller neck, placing them at higher risk
temporary measure (E). Meical therapy is not appropriate for eveloping incarceration (E).
if the suspecte etiology is irritation of the nerve seconary Reference: Fitzgibbons RJ Jr, Giobbie-Hurer A, Gibbs JO, etal.
to stapling/tacking (B–D). Chronic groin pain may occur in Watchful waiting vs repair of inguinal hernia in minimally symptom-
10% to 5% of patients 1 year after surgery. The etiology is atic men: a ranomize clinical trial. JAMA. 006;95(3):85–9.
CHAPtEr 2 Abdomen—Hernia 17

13. C. The vast majority of inguinal hernias in chilren are the hernia through the ischemic skin, leaing to peritonitis
the inirect type ue to a persistent patent processus vagi- an eath. Thus, patients with cirrhosis an ascites shoul
nalis. Approximately 1% to 5% of chilren can evelop an unergo repair if there is evience that the skin overlying
inguinal hernia. However, the incience increases in preterm the hernia is thinning or becoming ischemic (B). However,
infants an those with a low birth weight. Right-sie her- repair shoul be elaye until after meical management of
nias are more common, an 10% of hernias iagnose at the ascites. If meical management fails an the skin over the
birth are bilateral. Incarceration is a more serious problem hernia is thinne an tense, then a transjugular portosystemic
in peiatric patients than in aults. Emergent operation on shunt shoul be consiere before repair. Alternatively, if the
an infant with an incarcerate hernia can be very challeng- patient is a caniate for liver transplant, the hernia can be
ing. Thus, it is preferable to try to reuce the hernia, which repaire uring the transplantation. Umbilical hernias have
is successful in 75% to 80% of cases, allow the inammation historically all been repaire by primary closure. Borrow-
to subsie over several ays, an then perform the repair ing from the low recurrence rates using mesh for inguinal
semielectively. The routine use of contralateral groin explo- hernias, umbilical hernias are now more frequently being
ration is not wiely supporte (D). For elective cases, one repaire using mesh, particularly those with large efects.
option is to perform laparoscopy via the hernia sac to look A recent prospective, ranomize stuy compare primary
for a contralateral hernia an, if foun, procee to repair. If closure with mesh repair. The early complication rates such
there are any signs of strangulation (e.g., leukocytosis, fever, as seroma, hematoma, an woun infection were similar in
elevate lactate), then manual reuction shoul be avoie, the two groups. However, the hernia recurrence rate was sig-
an the patient shoul be taken immeiately to the operating nicantly higher after primary suture repair (11%) than after
room for surgical intervention (E). In the patient escribe, mesh repair (1%) (E). Some authors are now avocating for
though the skin is erythematous, there are no signs of sys- the routine use of mesh for all ault umbilical hernias in the
temic toxicity. Methos to achieve reuction inclue the use absence of bowel strangulation.
of intravenous (IV) seation, Trenelenburg positioning, ice References: Arroyo A, García P, Pérez F, Anreu J, Canela F,
packs, an gentle irect pressure. Reuction without sub- Calpena R. Ranomize clinical trial comparing suture an mesh
sequent surgery is not appropriate. That being sai, infants repair of umbilical hernia in aults. Br J Surg. 001;88(10):131–133.
with anemia an history of prematurity are at signicantly Belghiti J, Duran F. Abominal wall hernias in the setting of cir-
rhosis. Semin Liver Dis. 1997;17(3):19–6.
increase risk of postoperative apnea an woul require
overnight monitoring.
Reference: Özemir T, Arıkan, A. Postoperative apnea after 16. D. Femoral hernias occur more commonly in females
inguinal hernia repair in formerly premature infants: impacts of ges- an have a high risk of incarceration. However, the most
tational age, postconceptional age an comorbiities. Pediatr Surg common overall hernia in females is an inirect inguinal
Int. 013;9(8):801–804. hernia (A). Bowel entering a femoral hernia passes own
a narrow femoral canal. This is because the femoral ring,
14. D. In chilren, umbilical hernias are congenital. They which serves as the entrance for the femoral canal, is very
are forme by a failure of the umbilical ring to close, causing rigi an unyieling. Thus, the xe neck of a femoral her-
a central efect in the linea alba. Most umbilical hernias in nia is prone to pinching off the bowel, putting the patient
chilren are small an will close by  years of age, particu- at risk for incarceration. The borers of the femoral canal
larly if the efect is less than 1 cm in size. As such, repair is are as follows: inguinal ligament (anterior) (B), Cooper
not always inicate at the time of iagnosis (B). Aition- ligament (posterior), femoral vein (lateral), an Poupart
ally, the ecision to perform an elective repair is not solely ligament (meial). Femoral hernias occur most commonly
etermine by the presence of symptoms (E). If closure oes lateral to the lymphatics an meial to the femoral vein,
not occur by age 4 or 5 years, elective repair is then consi- within the empty space (C). It is important to recognize
ere a reasonable option (C), even if the patient is asymp- that femoral hernias pass eep (posterior) to the inguinal
tomatic. If the hernia efect is large (> cm) or the family is ligament. As such, repairs to the inguinal ligament (such
bothere by the cosmetic appearance, repair shoul be con- as a Bassini operation an stanar mesh repair) will not
siere. Although umbilical hernias in chilren can incarcer- obliterate the efect (E). The femoral hernia can be xe
ate, this is very rare (A). If the chil presents with abominal either through a stanar inguinal approach or irectly
pain, bilious emesis, an a tener, har mass protruing over the bulge using an infrainguinal incision. The essen-
from the umbilicus, immeiate exploration an hernia repair tial elements of femoral hernia repair inclue issection
are inicate. an removal of the hernia sac an obliteration of the efect
in the femoral canal. This can be accomplishe by either
15. D. Unlike in chilren, umbilical hernias in aults are approximation of the iliopubic tract to the Cooper ligament
usually acquire (A). Risk factors are any conitions that or by placement of prosthetic mesh.
increase intraabominal pressure, such as pregnancy, obe- Reference: e Virgilio C, Frank PN, Grigorian A, es. Surgery: a
sity, an ascites. Overall strangulation of umbilical hernias case based clinical review. Springer; 015.
in aults is uncommon, but it occurs more often than in chil-
ren (C). Small, barely palpable an asymptomatic hernias 17. B. It is important to unerstan the ifference between
can be followe clinically. Larger or symptomatic hernias epigastric hernias an iastasis recti because the former is
shoul be repaire. In patients with cirrhosis an ascites, the a true hernia, which shoul be repaire, an the latter is
markely increase pressure causes the skin overlying the a benign conition. Diastasis recti is cause by increase
hernia to become thin an eventually ischemic. One of the separation of the rectus abominis muscles an a relative
most catastrophic complications in this setting is rupture of thinning of the linea alba, which can mimic a hernia. The
18 PArt i Patient Care

conition can be acquire, such as in multiparous women perforate through. Though small, they can cause signicant
where the repeate stretching of the abominal wall causes pain because of compression of the nerves traveling through
the rectus muscles to separate, or congenital, seconary to the efect. There is some evience to suggest that iastasis
more lateral attachment of the rectus muscles at birth. Clas- rectus may increase the risk for evelopment of an epigastric
sically, patients present after recent weight loss because this hernia an will make primary repair of epigastric hernias
allows for the lesion to be visible. There is no risk for stran- more challenging. Of note, patients with iastasis recti are at
gulation in iastasis recti because all of the facial layers are increase risk of abominal aortic aneurysms.
intact (A, D). Though several methos of surgical repair have References: Brunicari FC, Anersen DK, Schwartz SI.
been escribe, these are mainly cosmetic. In general, all that Schwartz’s principles of surgery. 10th e. McGraw-Hill Eucation.
is require is reassurance an abominal wall exercises to Köhler G, Luketina RR, Emmanuel K. Suture repair of pri-
help strengthen the musculature—though complete resolu- mary small umbilical an epigastric hernias: concomitant rectus
iastasis is a signicant risk factor for recurrence. World J Surg.
tion in aults is unlikely (C). In contrast, epigastric hernias
015;39(1):11–16.
are true hernias an represent a true efect in the linea alba.
Townsen CM, Jr, Beauchamp RD, Evers BM, Mattox KL, es.
They are generally small an contain either preperitoneal fat Sabiston textbook of surgery: the biological basis of modern surgical prac-
or part of the falciform ligament (E). They arise from efects tice. 17th e. Philaelphia, PA: W.B. Sauners; 004.
in the fascia in locations where neurovascular bunles
Abdomen—Biliary
AMANDA C. PURDY AND DANIELLE M. HARI 3
ABSITE 99th Percentile High-Yields
I. Physiology
A. Bile consists of water, bile salts, phospholipis, an cholesterol
B. Primary bile acis (cholic & chenoeoxycholic acis) become seconary bile acis when ehyroxylate
by gut bacteria (lithocholate an eoxycholate acis)
C. Mechanism of bile concentration in the gallblaer: active transport of NaCl into gallblaer mucosal
cells, passive absorption of water

II. Cholecystitis, Choleocholithiasis, an Cholangitis


A. Acute calculous cholecystitis: gallblaer inammation ue to gallstone impacte in the neck;
management is NPO, IV antibiotics, resuscitation, laparoscopic cholecystectomy
B. Acute acalculous cholecystitis: usually seen in critically ill patients in the ICU; US emonstrates
gallblaer wall thickening, pericholecystic ui, with no stones; HIDA if US is equivocal, tx is IV
antibiotics, resuscitation, an percutaneous cholecystostomy tube if critically ill versus laparoscopic
cholecystectomy if stable for surgery
C. Choleocholithiasis: may have obstructive jaunice; elevate irect bilirubin, may have transaminitis; US
emonstrates gallstones, ilate CBD, +/− stone in the CBD (sensitivity only 50%); tx is ERCP followe
up by laparoscopic cholecystectomy
1. If intraoperative cholangiogram is positive for choleocholithiasis: rst attempt to ush the stone
with saline; if it oesn’t work try ushing after giving 1 mg IV glucagon (relaxes sphincter of Oi); if
stones on’t clear, options inclue:
a) Postoperative ERCP
b) Transcystic CBD exploration—best for small stones, large cystic uct, or small CBD; generally
preferre over transuctal CBD exploration because it avois a CBD incision
c) Transuctal CBD exploration—best for large stones (>8–10 mm), large CBD, proximal stones
(above cystic uct), choleochotomy mae anterior to avoi vasculature laterally
D. Acute ascening cholangitis: Charcot tria (RUQ pain, fever, jaunice) presents in about 0%; Reynols
penta as hypotension an confusion; RUQ US: +/− gallstones, ilate CBD, elevate irect BR; tx is IV
antibiotics, ui resuscitation, pressors if in septic shock, followe by ERCP or percutaneous transhepatic
cholangiography (PTC) tube placement after resuscitation; laparoscopic cholecystectomy uring same
amission if ue to gallstones
E. Mirizzi synrome: large stone in the gallblaer neck compresses the common hepatic uct (CHD), can
cause CHD stricture or stula between the gallblaer an CHD; usually presents similar to cholecystitis
an iagnose uring cholecystectomy; manage with cholecystectomy

III. Choleochal cysts


A. Due to an anomalous pancreaticobiliary junction, with a fuse, long common pancreaticobiliary channel
allowing pancreatic enzymes to reux into the biliary tree leaing to inammation an cystic egeneration

19
20 PArt i Patient Care

B. More common in females an those of Asian escent, 60% iagnose before age 10
C. First step in workup is US but MRCP is best for iagnosis an preop planning
D. Associate with cholangiocarcinoma an gallblaer cancer; type III has very low risk of malignancy;
management for all types besies type III is surgical to ecrease subsequent malignancy risk
1. Management is base on location (escribe by Toani Classication):
. Type I (fusiform ilation, most common): cyst excision, Roux-en-Y hepaticojejunostomy,
cholecystectomy
3. Type II: cyst excision, primary closure, cholecystectomy
4. Type III: enoscopic sphincterotomy an cyst unroong
5. Type IVa: cyst excision, partial hepatectomy, Roux-en-Y hepaticojejunostomy, cholecystectomy
6. Type IVb: cyst excision, Roux-en-Y hepaticojejunostomy, cholecystectomy
7. Type V (Caroli isease): if only in one lobe of the liver—hepatic resection an cholecystectomy; if
bilobar or unresectable—liver transplant

IV. Gallblaer Polyps


A. Polypoi lesions of the gallblaer: cholesterolosis (most common, cholesterol-laen macrophages in the
lamina propria), aenomatous polyp (risk for gallblaer cancer)
B. Inications for cholecystectomy for gallblaer polyps: symptomatic, polyp >10 mm, primary sclerosing
cholangitis, an polyp of any size
C. If cholecystectomy is not inicate, shoul follow patient with serial US in 6 to 1 months

V. Gallblaer Aenocarcinoma (most common biliary malignancy)


A. Risk factors: gallstones, gallblaer polyp >10 mm, porcelain gallblaer with selective mucosal
calcication (as oppose to transmural calcication)
B. May present similarly to cholecystitis, often iagnose on pathology after cholecystectomy
C. Management:
1. T1a (into lamina propria) → cholecystectomy
. >T1b (into muscularis) OR >N1 → cholecystectomy, segment IVb & V hepatectomy, portal
lymphaenectomy (port site resection not inicate); followe by ajuvant chemotherapy
(gemcitabine an cisplatin)
3. If positive cystic uct margin, nee extrahepatic bile uct resection an hepaticojejunostomy

VI. Cholangiocarcinoma
A. Risk factors: primary sclerosing cholangitis, ulcerative colitis, choleochal cyst, liver uke infection
B. Can present with painless jaunice; suspect in patient with focal bile uct stenosis without history of
biliary surgery or pancreatitis; best imaging is MRCP
C. Unresectable if istant metastasis, which inclues multifocal hepatic isease an lymph noe mets
beyon the porta hepatis
D. For potentially resectable cholangiocarcinoma, start with iagnostic laparoscopy; goal of surgery is
negative margins; all surgery inclues portal lymphaenectomy; management epens on location:

Location/Classięcation Management
Lower 1/3 of extrahepatic bile duct Whipple
Middle 1/3 of extrahepatic bile duct Resection, hepaticojejunostomy
Upper 1/3 of bile duct Type I CHD (not to the conĚuence) If localized to one side—hemi-
AKA Klatskin tumor hepatectomy, extrahepatic
Type II CHD to the conĚuence
Further classięed with the bile duct excision, Roux-en-Y
Bismuth classięcation: Type IIIa CHD + RHD hepaticojejunostomy
Type IIIb CHD + LHD If unresectable hilar tumor ≤3cm
without nodal disease or distant
Type IV CHD + RHD + LHD mets—evaluate for transplant
CHAPtEr 3 Abdomen—Biliary 21

VII. Bile uct injuries (incience 0.3%–0.8%, most commonly ue to cystic uct stump leak)
A. Risk of bile uct injury higher with laparoscopic cases an elective (not emergent/urgent cases)
B. Principles of management: control sepsis, rain bile collections, an establish secure biliary rainage
C. Marke laboratory abnormalities are not typical; bilirubin may be elevate ue to systemic resorption;
US is initial imaging stuy, +/− HIDA
D. In immeiate postop perio, treat with IV antibiotics, ui resuscitation, percutaneous rainage an
ERCP with stent placement an/or sphincterotomy as this is sufcient for majority of cases; if not,
percutaneous transhepatic catheter require; if leak has not heale in 6 to 8 weeks, biliary reconstruction
is consiere with Roux-en-Y hepaticojejunostomy
E. If iscovere intraoperatively, repair only inicate if aequate hepatobiliary surgical experience is
available; otherwise, wie rainage an referral to higher level of care

Fig. 3.1 Biliary Cysts.


22 PArt i Patient Care

Questions
1. A 10-year-ol boy with sickle cell isease 4. Which of the following patients shoul be offere
presents with right upper quarant pain, nausea, a cholecystectomy?
vomiting, fever, an yellowing of the eyes for the A. A 40-year-ol woman with an incientally
past ay. He enies ark urine or light stool. On iscovere 6-mm gallblaer polyp
exam, he is febrile, hemoynamically stable, an B. A 30-year-ol man with asymptomatic
has a positive Murphy sign. He has leukocytosis, gallstones unergoing gastric bypass
elevate alkaline phosphatase, an elevate C. A 65-year-ol woman with asymptomatic
unconjugate bilirubin. On ultrasoun, there are gallstones an an incientally iscovere
gallstones, pericholecystic ui, an gallblaer porcelain gallblaer with selective mucosal
wall thickening, an CBD iameter is 4 mm. After calcication
starting IV ui resuscitation an IV antibiotics, D. A 50-year-ol man with a history of iabetes
what is the next step? an asymptomatic gallstones
A. MRCP E. A 1-year-ol boy with sickle cell isease an
B. ERCP asymptomatic gallstones
C. Percutaneous transhepatic cholangiography
D. Laparoscopic cholecystectomy 5. Which of the following is true regaring bile an
E. Cholecystostomy tube gallstones?
A. The primary bile acis are eoxycholic an
2. A 5-year-ol woman is unergoing elective lithocholic aci
laparoscopic cholecystectomy for symptomatic B. The primary phospholipi in bile is lecithin
cholelithiasis. When removing the gallblaer C. Cholecystectomy ecreases bile salt secretion
from the fossa, a -mm tubular structure is D. Brown pigmente gallstones are more likely to
completely transecte an is leaking bile. The be foun in the gallblaer versus the CBD
structure appears to come from the liver fossa an E. Bile consists of an equal part of bile salts,
enter irectly into the gallblaer. What is the phospholipis, an cholesterol
most appropriate management?
A. Laparoscopic clip placement 6. Which of the following is true regaring the
B. Repair over a T-tube gallblaer?
C. Roux-en-Y hepaticojejunostomy A. It passively absorbs soium an chlorie
D. Immeiate transfer to a hospital with a B. In the setting of cholelithiasis, cholecystokinin
hepatobiliary surgeon (CCK) can cause gallblaer pain that waxes
E. Complete cholecystectomy an plan for an wanes
postoperative ERCP C. It harbors an alkaline environment
D. Glucagon can help empty the gallblaer
3. A 45-year-ol male presents with hematemesis E. Its contraction is inhibite by vagal
two weeks after a motor vehicle accient in which stimulation
he suffere a liver injury that was manage
nonoperatively. Laboratory values are signicant
for an elevate total bilirubin an alkaline
phosphatase, as well as signicant anemia.
This patient is most likely to have which of the
following?
A. Arterioportal vein stula
B. Arteriohepatic vein stula
C. Arterial pseuoaneurysm
D. Portal venous pseuoaneurysm
E. Cavernous hemangioma
CHAPtEr 3 Abdomen—Biliary 23

7. A 75-year-ol woman presents to the emergency 11. Ultrasonography of the gallblaer reveals a
epartment with a -ay history of nausea, polypoi lesion. This most likely represents:
feculent vomiting, an obstipation. Her bloo A. a cholesterol polyp
pressure on amission is 80/60 mm-Hg, an B. aenomyomatosis
her heart rate is 10 beats per minute, both of C. a benign aenoma
which normalize after uis. Plain lms reveal D. aenocarcinoma
istene loops of small bowel with air–ui E. an inammatory polyp
levels an air in the biliary tree. Which of the
following is the best management option? 12. Which of the following is the correct pairing of
A. Small bowel enterotomy with removal of the anatomic structure an irection for retraction
gallstone plus uring a laparoscopic cholecystectomy?
B. Small bowel enterotomy with removal of the A. Gallblaer funus laterally
gallstone B. Gallblaer infunibulum laterally
C. Small bowel enterotomy with removal of C. Gallblaer boy laterally
the gallstone followe 8 weeks later by D. Gallblaer infunibulum cephala
cholecystectomy an takeown of stula E. Gallblaer funus meially
D. Small bowel resection to inclue area of
impacte gallstone 13. Hyrops of the gallblaer:
E. Small bowel resection to inclue area of A. Poses a signicantly increase risk of
impacte gallstone plus cholecystectomy an malignancy
takeown of the stula B. Is ue to a stone impacte in the cystic uct
C. Typically starts with an enteric bacterial
8. Jaunice with absent urine urobilinogen is most infection
consistent with: D. Is associate with marke right upper
A. Hepatitis quarant tenerness
B. Cirrhosis E. Results in the gallblaer getting lle with
C. Hemolysis bile-staine ui
D. Biliary obstruction
E. Sepsis 14. During a laparoscopic cholecystectomy for
symptomatic cholelithiasis, the surgeon
9. Which of the following is true regaring bile an inavertently transects the CBD. An experience
gallblaer isease? hepatobiliary surgeon is available. The best choice
A. Primary bile acis are forme by econjugation for operative repair is:
B. Bile acis are passively absorbe in the A. En-to-en CBD anastomosis
terminal ileum B. Choleochouoenostomy
C. Bile acis are responsible for the yellow color C. Choleochojejunostomy
of bile D. Hepaticouoenostomy
D. Bile uct stones occurring 1 year after E. Hepaticojejunostomy
cholecystectomy are consiere primary
common uct stones 15. The most common cause of benign bile uct
E. In between meals, gallblaer emptying is stricture is:
stimulate by motilin A. Ischemia from operative injury
B. Chronic pancreatitis
10. Which of the following is true regaring biliary C. Common uct stones
anatomy? D. Acute cholangitis
A. The right hepatic uct tens to be longer than E. Sclerosing cholangitis
the left an more prone to ilation
B. Venous return from the gallblaer is most
often via a cystic vein to the portal vein
C. Heister valves have an important role in the
gallblaer’s function as a bile reservoir
D. The CBD an pancreatic uct typically unite
outsie the uoenal wall
E. The arterial supply to the CBD erives
primarily from the left hepatic an right
gastric arteries
24 PArt i Patient Care

16. A 45-year-ol man has a 50% total boy 19. An 80-year-ol patient presents with nausea,
surface area thir-egree burn. Fever, marke fever, an right upper quarant pain an
leukocytosis, an right upper quarant pain tenerness. Ultrasonography reveals gallstones
evelop on hospital ay 7. His bloo pressure as well as air in the wall of the gallblaer. His
is 130/80 mm-Hg, an his heart rate is 110 beats temperature is 103.5°F an bloo pressure is
per minute. Ultrasonography shows a istene 70/40 mm-Hg. Meical therapy is initiate, an
gallblaer with gallblaer wall thickening an pressors are neee to maintain bloo pressure.
sluge. However, it is negative for gallstones. Which of the following is true regaring this
Antibiotics are initiate. The next step in conition?
management woul consist of: A. Metroniazole is an important antibiotic
A. Laparoscopic cholecystectomy choice
B. Compute tomography B. Emergent cholecystectomy is inicate
C. Hepatobiliary iminoiacetic aci (HIDA) scan C. Urgent percutaneous rainage is preferre
D. Percutaneous cholecystostomy over cholecystectomy
E. Upper enoscopy D. The most common organism is an anaerobic
gram-negative ro
17. During laparoscopic cholecystectomy, bile E. Perforation of the gallblaer is rare
appears to be emanating near the junction of
the CBD an cystic uct. Upon conversion to 20. Which of the following best escribes the role of
open cholecystectomy, the injury is note to be preoperative biliary rainage before a Whipple
a 3-mm longituinal tear in the anterolateral proceure in a patient with obstructive jaunice?
istal common hepatic uct. The uct itself A. It has been shown to ecrease the rate of
measures 7 mm in iameter. Management cholangitis
consists of: B. It has been shown to increase the rate of
A. Primary repair of the injury without a T tube woun infections
B. Primary repair of the injury over a T tube C. It shoul be performe routinely if the
C. Primary repair of the injury with a T tube bilirubin level is greater than 8 mg/L
place through a separate choleochotomy D. It has been shown to shorten the hospital stay
D. Hepaticojejunostomy E. It has been shown to ecrease the mortality
E. Choleochouoenostomy rate

18. Which of the following statements is 21. A 35-year-ol Chinese man presents with a
true regaring the use of intraoperative fever of 103.5°F, right upper quarant pain,
cholangiography (IOC) uring laparoscopic an jaunice. Laboratory values are signicant
cholecystectomy? for a white bloo cell count of 15,000 cells/L,
A. It helps prevent inavertent incision of the an alkaline phosphatase level of 400 U/L,
common bile uct (CBD) an a serum bilirubin level of 3.8 mg/L.
B. It is the best way to ientify clinically Magnetic resonance cholangiopancreatography
signicant common uct stones (MRCP) emonstrates a markely ilate CBD,
C. Routine use is justie because of its ability markely ilate intrahepatic ucts with several
to ientify anatomic anomalies of the hepatic intrahepatic uctal strictures, an multiple stones
ucts throughout the uctal system. Which of the
D. Routine use is helpful to ensure complete following is true regaring this conition?
removal of the gallblaer an cystic uct A. It is associate with close contact with ogs
E. Routine use is unnecessary an sheep
B. It is more commonly associate with black
pigment stones versus brown pigment stones
C. It more commonly affects males
D. Metroniazole is able to resolve the majority of
cases
E. Initial treatment is with enoscopic retrograe
cholangiopancreatography an transhepatic
cholangiography
CHAPtEr 3 Abdomen—Biliary 25

22. A 65-year-ol woman presents with symptoms 25. Choleochal cyst isease is thought to be cause
an signs of acute cholecystitis an unergoes by an abnormality of the:
an uneventful laparoscopic cholecystectomy. A. Bile uct smooth muscle
On postoperative ay 7, the pathology report B. Bile composition
inicates a supercial gallblaer carcinoma that C. Bile uct aventitia
invaes the perimuscular connective tissue. There D. Pancreaticobiliary uct junction
is no evience of istant metastasis on subsequent E. Bile uct mucosa
imaging. Which of the following woul be the
best management? 26. On CT scan, a type I choleochal cyst appears
A. Raiation an chemotherapy to be aherent to the posterior wall of the portal
B. Observation vein. Management consists of:
C. Reoperation with resection of liver segments A. Partial excision of the cyst, leaving posterior
IVB an V wall behin, an cholecystectomy with Roux-
D. Reoperation with resection of liver segments en-Y hepaticojejunostomy
IVB an V an regional lymph noe issection B. Complete excision of the cyst, cholecystectomy,
E. Reoperation with resection of liver segments an hepaticojejunostomy
IVB an V, regional lymph noe issection, C. Partial excision of the cyst, fulguration of
an resection of all port sites posterior cyst mucosa, an cholecystectomy
with Roux-en-Y hepaticojejunostomy
23. A 4-year-ol male presents with acholic stools D. Observation
an cola-colore urine. Alkaline phosphatase E. Roux-en-Y cyst jejunostomy
is 000 IU/L, AST is 78 IU/L, ALT is 88 IU/L,
an total bilirubin is .1 mg/L. Liver biopsy 27. Which of the following is the best management of
emonstrates periuctal concentric brosis a localize Klatskin tumor?
aroun macroscopic bile ucts. He is positive for A. Pancreaticouoenectomy (Whipple
perinuclear antineutrophil cytoplasmic antiboy proceure)
(p-ANCA). Which of the following is true about B. Resection of the entire extrahepatic biliary tree
this conition? with hepatic resection if necessary
A. It is more commonly associate with Crohn C. Resection of the mile thir of the biliary tree
isease than it is with ulcerative colitis with hepaticojejunostomy
B. Cancer antigen (CA) 19-9 levels shoul be D. Chemotherapy
etermine E. Raiation followe by chemotherapy
C. Enoscopic retrograe
cholangiopancreatography (ERCP) will 28. Which of the following is true regaring
preominantly emonstrate irregular cholangiocarcinoma?
narrowing of the intrahepatic biliary tree A. The majority are intrahepatic
D. Symptoms are often well controlle with B. Bismuth-Corlette type I cholangiocarcinoma
meical management occurs above the conuence of the right an
E. It is more common in females left hepatic ucts
C. Most patients benet from ajuvant
24. Which of the following is a feature of gallblaer chemoraiation after surgical intervention
cancer? D. It arises from malignant transformation in
A. Speckle cholesterol eposits are foun on the hepatocytes
gallblaer wall E. Resection with biliary-enteric bypass is
B. There are thickene noules of mucosa an consiere appropriate management in
muscle patients with early isease
C. Gallblaer cancer is more common in males
D. It is more likely to be accompanie by large
gallstones compare with smaller ones
E. Cancer invaing muscularis layer is manage
with cholecystectomy alone
26 PArt i Patient Care

Answers
1. D. This patient with sickle cell isease has acute calculous artery) uring laparoscopic cholecystectomy but may also
cholecystitis an shoul unergo laparoscopic cholecystec- occur following blunt an penetrating traumatic injuries.
tomy after ui resuscitation an initiation of antibiotics. The unerlying lesion is typically an arterial pseuoaneu-
Signs that point to acute cholecystitis in this case inclue: rysm that has a connection with the biliary tree (hence the
fever, positive Murphy sign, leukocytosis, an ultrasoun jaunice). It can also occur in association with gallstones,
nings of gallstones, gallblaer wall thickening, an peri- tumors, inammatory isorers, an vascular isorers.
cholecystic ui. MRCP is reasonable if there is concern for Treatment in most instances involves angiographic emboli-
possible choleocholithiasis. However, it is important to is- zation of the artery (thus angiography is most likely to be
tinguish obstructive jaunice from jaunice from hemolytic the therapeutic stuy of choice). Enoscopy may show bloo
anemia (as seen in this patient) (A). Although this patient has coming from the ampulla of Vater but will not typically be
jaunice, his labs show an increase unconjugated bilirubin. therapeutic (because the bleeing is coming from a hepatic
He also oes not have ark urine or acholic stools, an CBD artery pseuoaneurysm). The remaining answer choices are
iameter is normal. This is more consistent with hemolytic not thought to play a role in hemobilia (A, B, D, E).
anemia than with obstructive jaunice (in which you woul References: Ahrent SA, Pitt HA. Biliary tract. In: Townsen
expect conjugate bilirubinemia, ark urine, acholic stools, CM, Jr, Beauchamp RD, Evers BM, Mattox KL, es. Sabiston textbook
an CBD ilation). This young patient with sickle cell is- of surgery: the biological basis of modern surgical practice. 17th e. Phila-
ease has chronic hemolysis, which likely le to evelopment elphia: W.B. Sauners; 004:1597–164.
Bloechle C, Izbicki JR, Rashe MY, et al. (1994). Hemobilia:
of pigmente gallstones, an now cholecystitis. Sepsis can
presentation, iagnosis, an management. Am J Gastroenterol.
trigger increase hemolysis in patients with sickle cell isease
1994;89(9):1537–1540.
an is responsible for his perceive increase jaunice since Croce MA, Fabian TC, Spiers JP, Kusk KA. Traumatic
symptom onset. ERCP woul be an appropriate choice if there hepatic artery pseuoaneurysm with hemobilia. Am J Surg.
is a very high suspicion for choleocholithiasis or ascening 1994;168(3):35–38.
cholangitis; however, there is no evience of biliary obstruc- Nicholson T, Travis S, Ettles D, etal. Hepatic artery angiography
tion in this case (B). Percutaneous transhepatic cholangiogra- an embolization for hemobilia following laparoscopic cholecystec-
phy can also be use to ecompress the biliary tree, which is tomy. Cardiovasc Radiol. 1999;(1):0–4.
not inicate in this case (C). Cholecystostomy tube can be
consiere in patients with cholecystitis that are too unstable 4. C. Asymptomatic patients who are incientally is-
to unergo cholecystectomy, which is not true in this case (E). covere to have gallstones usually o not require surgery
because the lifetime risk of eveloping symptoms is <5%.
2. A. Ducts of Luschka are small ucts that originate in the There are, however, certain inications for cholecystectomy
gallblaer fossa an rain irectly into the gallblaer, as in asymptomatic patients. These inclue gallblaer polyps
escribe in this question. When transecte, they can cause ≥10 mm an a porcelain gallblaer with selective mucosal
bile leaks. When iscovere intraoperatively, the uct shoul calcication of the gallblaer because both have an asso-
be clippe or oversewn. More commonly these are iag- ciate malignancy risk (A). Historically, all patients with
nose postoperatively as a ui collection at the gallblaer porcelain gallblaer unerwent cholecystectomy because
fossa (biloma) an shoul be raine percutaneously an of the malignancy risk. It is now unerstoo that the risk is
an ERCP with sphincterotomy an stent placement shoul not as high as originally thought, an only selective muco-
be performe to encourage bile ow into the uoenum (E). sal calcication is associate with malignancy risk, while
Primary repair over a T-tube an Roux-en-Y hepaticojeju- transmural calcication is not. More extensive intramural
nostomy are the appropriate treatment for common bile uct eposits cause mucosal sloughing, which reuces the rate
injuries (with <50% luminal injury an >50% luminal injury, of aenocarcinoma, while the selective calcication yiels
respectively), which is not what is escribe in this case (B, to a continue inammatory stimulus. Thus, a stronger
C). If a common uct injury occurs at a hospital without a recommenation for prophylactic cholecystectomy is mae
surgeon who is experience in biliary reconstruction, the sur- for the selective mucosal calcication pattern in an asymp-
geon shoul place wie rains an then arrange transfer to a tomatic patient. Patients with cholelithiasis unergoing gas-
referral center. However, that is not necessary in this case (D). tric bypass are at increase risk for eveloping gallstones
References: Mercao MA, Domínguez I. Classication an man- because of rapi weight loss. However, most o not evelop
agement of bile uct injuries. World J Gastrointest Surg. 011;3(4):43–48. symptoms requiring cholecystectomy, an prophylactic cho-
Spanos CP, Syrakos T. Bile leaks from the uct of Luschka (sub- lecystectomy in these patients is not inicate (B). Diabetes is
vesical uct): a review. Langenbecks Arch Surg. 006;391(5):441–447. also not an inication for cholecystectomy in the absence of
symptoms (D). Patients with conitions that cause hemolytic
3. C. Hemobilia is a rare conition an presents with a anemia, such as sickle cell isease an hereitary sphero-
classic (Quinke) tria of upper gastrointestinal bleeing cytosis, are at increase risk of eveloping pigmente gall-
(hematemesis), combine with jaunice an right-sie stones. However, surgery for asymptomatic cholelithiasis in
upper abominal pain. It is most often a result of iatrogenic these patients is only recommene if they are unergoing
injury of the right hepatic artery (more common if there is another abominal operation (such as splenectomy for chil-
an aberrant right hepatic artery off the superior mesenteric ren with hereitary spherocytosis [E]).
CHAPtEr 3 Abdomen—Biliary 27

References: Warschkow R, Tarantino I, Ukegjini K, et al. Con- ajacent uoenum an causing air in the biliary tree, cre-
comitant cholecystectomy uring laparoscopic Roux-en-Y gastric ating a cholecystouoenal stula (the most common type
bypass in obese patients is not justie: a meta-analysis. Obes Surg. of biliary stula). Less commonly, the stula can be between
013;3(3):397–407. the gallblaer an the colon (hepatic exure) or the stom-
Overby DW, Apelgren KN, Richarson W, Fanelli R, Society of
ach. The stone typically loges in the narrowest portion of
American Gastrointestinal an Enoscopic Surgeons. SAGES guie-
the gastrointestinal tract—the istal ileum, near the ileoce-
lines for the clinical application of laparoscopic biliary tract surgery.
Surg Endosc. 010;4(10):368–386. cal valve. The iagnosis of gallstone ileus is mae preopera-
tively in only approximately half of cases because a history of
5. B. Bile consists of bile salts, phospholipis, an choles- biliary isease may be absent, pneumobilia may not be seen,
terol in the following concentrations: 80%, 15%, an 5%, the gallstone may not be visualize, or the abominal raio-
respectively (E). Normally, more than 95% of bile salts are graphic nings may be nonspecic. Because many of these
reabsorbe by the enterohepatic circulation an negative patients are elerly, have other major comorbiities, an are
feeback accounts for replacement of the 0.5 g loss of bile salts often markely ehyrate, initial surgical management
in the stool. The primary bile acis are cholic aci an che- shoul focus on relieving the obstruction. This is best accom-
noeoxycholic aci. The seconary bile acis are lithocholate plishe by a transverse enterotomy proximal to the palpable
an eoxycholate acis (A). Cholecystectomy has minimal stone an stone removal (C–E). It is also important to run
effect on bile aci secretion but oes increase enterohepatic the small bowel because a signicant portion of patients
circulation of bile salts (C). Pigment stones get their charac- will have more than one gallstone. Leaving the stula oes
teristic color from calcium bilirubinate. Brown pigment gall- not seem to lea to signicant morbiity on long-term fol-
stones occur more commonly in the setting of biliary stasis low-up. Most surgeons woul not recommen taking the
such as cholangitis an ten to form in the CBD. Black pig- patient back at a later time for stula takeown. A resection
ment stones are associate with hemolytic isorers an are of the small bowel is usually not necessary.
more likely to be foun within the gallblaer (D). References: Ahrent SA, Pitt HA. Biliary tract. In: Townsen
Reference: Osottir M, Hunter JG. Gallblaer. In: Bruni- CM, Jr, Beauchamp RD, Evers BM, Mattox KL, es. Sabiston textbook
cari FC, Anersen DK, Billiar TR, etal., es. Schwartz’s principles of of surgery: the biological basis of modern surgical practice. 17th e. Phila-
surgery. 8th e. New York: McGraw-Hill; 005:1187–100. elphia: W.B. Sauners; 004:1597–164.
Roríguez-Sanjuán JC, Casao F, Fernánez MJ, Morales DJ,
Naranjo A. Cholecystectomy an stula closure versus enterolithot-
6. D. The gallblaer concentrates an stores bile. It oes
omy alone in gallstone ileus. Br J Surg. 1997;84(5):634-637.
so by rapily absorbing soium an chlorie against a con- Tan YM, Wong WK, Ooi LLPJ. A comparison of two surgical
centration graient by active transport an passive water strategies for the emergency treatment of gallstone ileus. Singapore
absorption (A). The epithelial cells of the gallblaer secrete Med J. 004;45():69–7.
mucous glycoproteins an hyrogen ions into the gallbla- Halabi WJ, Kang CY, Ketana N, Lafaro KJ, Nguyen VQ, Sta-
er lumen. The secretion of hyrogen ions aciies the bile, mos MJ, Imagawa DK, Demirjian AN. Surgery for gallstone ileus:
increasing calcium solubility, an thus preventing its pre- a nationwie comparison of trens an outcomes. Ann Surg.
cipitation as calcium salts (C). Inammation of the gallbla- 014;59():39–35.
er mucosa seems to affect the ability to secrete hyrogen
ions, making the bile more lithogenic. Vagal innervation 8. D. Bilirubin is the result of the breakown of ol re
stimulates contraction of the gallblaer (E). CCK causes bloo cells into heme. Heme is broken own into biliver-
steay an tonic contraction. The term biliary colic is a mis- in an then bilirubin. Bilirubin is boun to albumin in the
nomer because postpranial gallblaer pain seconary to circulation, but as it reaches the liver, it is conjugate an
cholelithiasis oes not wax an wane but rather stays con- eventually enters the gastrointestinal tract. In the gastroin-
stant for up to several hours (B). The more appropriate term testinal tract, it is econjugate into urobilinogen by bacteria.
is symptomatic cholelithiasis. The gallblaer normally lls by Some urobilinogen gets reabsorbe in the gut, returns to the
contraction at the sphincter of Oi at the ampulla of Vater. liver, an is excrete in the urine, where it is eventually con-
In contrast, glucagon relaxes the sphincter of Oi an cre- verte to urobilin, giving urine its yellow appearance. The
ates the path of least resistance allowing the gallblaer to remaining urobilin is oxiize to stercobilin in the intestines,
empty into the uoenum. giving stool its brown appearance. In the presence of biliary
References: Ahrent SA, Pitt HA. Biliary tract. In: Townsen obstruction, less bilirubin enters the gut, less urobilinogen is
CM, Jr, Beauchamp RD, Evers BM, Mattox KL, es. Sabiston textbook mae, an therefore less appears in the urine. Less sterco-
of surgery: the biological basis of modern surgical practice. 17th e. Phila- bilin is mae an therefore the stools turn pale. Hemolysis
elphia, PA: W.B. Sauners; 004:1597–164. woul generate an increase in bilirubin an a corresponing
Osottir, M, Hunter, JG. Gallblaer. In: Brunicari FC, increase in urobilinogen in the gut an in the urine (C). The
Anersen DK, Billiar TR, etal., es. Schwartz’s principles of surgery. remaining answer choices o not play a signicant role in
8th e. New York: McGraw-Hill; 005:1187–100. bilirubin metabolism (A, B, E).
Reference: Ahrent SA, Pitt HA. Biliary tract. In: Townsen
7. B. The presentation is consistent with gallstone ileus. CM, Jr, Beauchamp RD, Evers BM, Mattox KL, es. Sabiston textbook
Gallstone ileus is a misnomer because it is actually a type of of surgery: the biological basis of modern surgical practice. 17th e. Phila-
mechanical small bowel obstruction. It occurs more com- elphia: W.B. Sauners; 004:1597–164.
monly in elerly females (>70 years). The most specic
stuy to help conrm iagnosis is a CT scan showing air 9. E. Cholesterol that has been conjugate with taurine or
in the biliary tree. It usually results from a large gallstone glycine is consiere a primary bile (cholic an chenoe-
(>.5 cm) that has eroe through the gallblaer into the oxycholic aci). Seconary bile acis are a result of bacterial
28 PArt i Patient Care

econjugation in the gastrointestinal tract (A). Although bile an retrieval of the specimen, (3) a 5-mm right-sie sub-
acis are passively absorbe along the entirety of the small costal port, an (4) an aitional 5-mm port inferior an lat-
intestine, they are actively absorbe only in the terminal eral to the subcostal port. The 5-mm ports allow graspers to
ileum (B). Bile acis are colorless, an the yellow hue of bile retract the gallblaer funus superiorly (A, E) an infun-
is a result of the pigmente biliverin (breakown prouct ibulum, or the neck, laterally. This is the ieal positioning to
of bilirubin) that is also foun in bile (C). Bile uct stones achieve the “critical view” an prevent CBD injury because
occurring after  years are consiere primary common uct it allows the cystic uct to remain perpenicular to the CBD.
stones an are often pigmente (D). During the fasting state, Excess cephala retraction of the gallblaer infunibulum
gallblaer emptying is stimulate by motilin. shifts the cystic uct in line with the CBD an is consiere
Reference: Luiking YC, Peeters TL, Stolk MF, et al. Motilin the most common cause of CBD injury (D). The gallblaer
inuces gall blaer emptying an antral contractions in the faste boy shoul not be use as a retraction site (C).
state in humans. Gut. 1998;4(6):830–835.
13. B. When a gallstone becomes impacte in the cystic
10. D. The left hepatic uct is longer than the right an is uct, the typical course is that acute cholecystitis will evelop
more likely to be ilate in the presence of istal obstruction in the patient. Less frequently, an acute infection oes not
(A). The spiral Heister valves within the cystic uct o not evelop in the patient even though the cystic uct remains
have any true valvular function (C). In approximately three- obstructe. In this situation, bile within the gallblaer
fourths of iniviuals, the CBD an the main pancreatic uct becomes absorbe, but the gallblaer epithelium continues
unite outsie the uoenal wall an traverse the uoenal to secrete glycoprotein (mucus). The gallblaer becomes
wall as a single uct. The bloo supply to the CBD runs along istene with mucinous material (E). This is known as
the lateral an meial walls at the 3 an 9 o’clock positions hyrops. The gallblaer may be palpable but oes not cre-
an comes from the right hepatic artery an retrouoenal ate the Murphy sign (D). Hyrops of the gallblaer may
artery (off gastrouoenal artery) (E). Thus, a transverse result in eema of the gallblaer wall an perforation.
hemitransection of the uct will likely interrupt the bloo Although hyrops may persist with few consequences, cho-
supply an rener a repair prone to ischemia an stricture. lecystectomy is generally inicate to avoi complications.
Venous return of the gallblaer is typically raine irectly Hyrops of the gallblaer oes not signicantly increase
to the parenchyma of the liver (B). the risk for malignancy (A). Although this can subsequently
Reference: Osottir, M, Hunter, JG. Gallblaer. In: Bruni- become infecte, enteric bacterial infection is not typically
cari FC, Anersen DK, Billiar TR, etal., es. Schwartz’s principles of responsible for the evelopment of hyrops (C).
surgery. 8th e. New York: McGraw-Hill; 005:1187–100.
Reference: Osottir, M, Hunter, JG. Gallblaer. In: Bruni-
cari FC, Anersen DK, Billiar TR, etal., es. Schwartz’s principles of
11. A. Most polypoi lesions of the gallblaer are benign, surgery. 8th e. New York: McGraw-Hill; 005:1187–100.
an of these, cholesterol polyps are the most common. They
are usually small (<10 mm), peunculate, an multiple. 14. E. The majority of common bile uct injuries occur iat-
They are usually seen in association with cholesterolosis. rogenically uring laparoscopic cholecystectomy in patients
Ultrasoun imaging often emonstrates hyperechoic foci with relatively benign gallblaer isease (e.g., symptomatic
with a comet tail artifact; unlike gallstones, these foci on’t cholelithiasis, acute cholecystitis). The management of an
prouce shaowing. Aenomyomatosis polyps are the sec- intraoperative bile uct injury epens on the type of injury
on most common (B). They appear as sessile polyps that an the clinical setting. If a small lateral injury (<50%) is
cause focal thickening of the wall. Inammatory polyps create in the CBD, this can be repaire by closing the uc-
are the thir most common (E). All three are benign an totomy over a T tube an leaving a rain. Conversely, if the
are pseuopolyps. Aenomas an aenocarcinomas of the common bile uct is transecte, this results in an interruption
gallblaer are generally larger than 10 mm. However, is- in the bloo supply to the uct an attempts at primary repair
tinguishing between a benign an a malignant polyp on will inevitably lea to stricture formation an recurrent epi-
ultrasonography is generally not reliable (C, D). Thus, when soes of cholangitis (A). Thus, if a transection is recognize
a polyp is foun on ultrasoun, the general inications for intraoperatively, an an experience hepatobiliary surgeon is
cholecystectomy are (1) a symptomatic polyp, () a polyp in available, it is best to repair it immeiately an to o so with
association with gallstones, (3) a polyp larger than 6 mm, an a biliary enteric bypass. Because most of these injuries will
(4) patient age over 50. For asymptomatic gallstone polyps be in the common bile uct, the best option is to perform a
that o not meet the above criteria, the recommene man- hepaticoenterostomy (B, C). A critical element of the repair
agement is follow-up ultrasoun in 6 months. is to perform a tension-free, mucosa-to-mucosa uct enteric
References: Ahrent SA, Pitt HA. Biliary tract. In: Townsen anastomosis. Hepaticouoenostomy has largely been aban-
CM, Jr, Beauchamp RD, Evers BM, Mattox KL, es. Sabiston textbook one for benign liver isease ue to ongoing enteric reux
of surgery: the biological basis of modern surgical practice. 17th e. Phila-
(D). It is also more technically challenging to perform because
elphia: W.B. Sauners; 004:1597–164.
it is ifcult to reach the uoenum to the hepatic uct; thus
Myers R, Shaffer E, Beck P. Gallblaer polyps: epiemiology, nat-
ural history an management. Can J Gastroenterol. 00:16(3):187-194.
most surgeons prefer a Roux-en-Y hepaticojejunostomy. If an
Shinkai H, Kimura W, Muto T. Surgical inications for small pol- experience hepatobiliary surgeon is not available, the best
ypoi lesions of the gallblaer. Am J Surg. 1998;175():114–117. option is to rain the area, place transhepatic catheters, an
refer the patient to higher level of care. If the injury is iscov-
12. B. A total of four trocar sites is typically place uring ere postoperatively an there has been a long elay, the best
laparoscopic cholecystectomy: (1) a 5-mm umbilical port for option is to perform transhepatic rainage an elay primary
the laparoscope, () a 1-mm epigastric port for issection repair for 6 to 8 weeks to allow the inammation to subsie.
CHAPtEr 3 Abdomen—Biliary 29

References: Ahrent SA, Pitt HA. Biliary tract. In: Townsen critically ill, the next stuy woul be a HIDA scan with sin-
CM, Jr, Beauchamp RD, Evers BM, Mattox KL, es. Sabiston textbook calie or morphine. A positive stuy ning woul emon-
of surgery: The biological basis of modern surgical practice. 17th e. Phil- strate nonlling of the gallblaer with visualization of the
aelphia: W.B. Sauners; 004:1597–164. tracer in the liver an small bowel. Morphine ecreases the
MacFayen BV Jr, Vecchio R, Ricaro AE, Mathis CR. Bile uct
rate of false-positive HIDA scan results because it leas to
injury after laparoscopic cholecystectomy: the Unite States experi-
sphincter of Oi contraction an thus increases the like-
ence. Surg Endosc. 1998;1(4):315–31.
Narayanan SK, Chen Y, Narasimhan KL, Cohen RC. Hepati- lihoo of lling of the gallblaer in the absence of chole-
couoenostomy versus hepaticojejunostomy after resection of cho- cystitis. A HIDA scan is not recommene in critically ill
leochal cyst: a systemic review an meta-analysis. J Pediatr Surg. patients in whom a elay in therapy can be potentially fatal
013;48(11):336–34. (C). Acalculous cholecystitis requires urgent intervention,
preferably cholecystectomy. The proceure can be attempte
15. A. Most benign bile uct strictures are iatrogenic an laparoscopically; however, there is a higher chance of ning
are ue to a technical error uring cholecystectomy, such as gangrenous cholecystitis an neeing to convert to open. If
excessive use of cautery, incorrect placement of a surgical the patient is too ill for surgery, percutaneous ultrasonogra-
clip, an overly aggressive issection near the CBD, all of phy or CT-guie cholecystostomy is the treatment option of
which may be the result of unclear anatomy (B–E). Regar- choice (B, D). Upper enoscopy is not inicate (E).
less of the cause, the eventual response is brosis an stric- Reference: Ahrent SA, Pitt HA. Biliary tract. In: Townsen
ture formation. As many as three-fourths of injuries that CM, Jr, Beauchamp RD, Evers BM, Mattox KL, es. Sabiston textbook
lea to strictures are not recognize at surgery, an as many of surgery: the biological basis of modern surgical practice. 17th e. Phila-
elphia: W.B. Sauners; 004:1597–164.
as one-thir occur 5 years or more after the operation. The
majority of iatrogenic strictures are short an occur in the
17. B. All of the provie options are potential repairs for
common bile uct an can present with an episoe of chol-
a bile uct injury. Sharp, clean, an small injuries in a large
angitis. The workup consists of ultrasonography, which will
CBD or common hepatic uct are more amenable to primary
etect ilate ucts proximal to the stricture, a compute
repair. Repair is generally performe over a T tube (A). It is
tomography scan to look for masses, an enoscopic retro-
important to bear in min that the CBD is supplie via two
grae cholangiography (ERCP) with enoscopic ultrasoun
main arteries running at the right an left borer of the uct,
(EUS), which can be both iagnostic an therapeutic. EUS can
entering at “3 o’clock” an “9 o’clock.” As such, injuries that
be helpful in etecting a tumor within the bile uct. During
are less than 50% in circumference are less likely to have
ERCP, a brushing of the bile uct shoul be taken for cytol-
interrupte the bloo supply on both sies an are therefore
ogy to rule out a malignancy. Management of focal benign
less likely to evelop ischemic stricture with primary repair.
strictures by a biliary enteric bypass or stenting remains
If the uct is transecte, nearly transecte (>50% circumfer-
ebatable because of the lack of ranomize trials an the
ence), or very small, a Roux-en-Y hepaticojejunostomy is rec-
lack of goo long-term follow-up with stenting. The pri-
ommene (D). Injuries to the proximal CBD can be treate
mary concern with stenting is that the strictures may become
with a hepaticojejunostomy (D), while injuries to the istal
obstructe an lea to recurrent cholangitis. Given the much
CBD can be treate with a choleochouoenostomy (E). If
less invasive nature of stenting, strong consieration shoul
the bile uct injury is the result of thermal injury, a primary
be given to this approach. If recurrent obstructive symptoms
repair with a T tube place through a separate choleochot-
subsequently evelop, a biliary enteric bypass shoul be
omy is the preferre approach (C).
performe.
References: Osottir, M, Hunter, JG. Gallblaer. In: Bruni-
References: Ahrent SA, Pitt HA. Biliary tract. In: Townsen cari FC, Anersen DK, Billiar TR, etal., es. Schwartz’s principles of
CM, Jr, Beauchamp RD, Evers BM, Mattox KL, es. Sabiston textbook surgery. 8th e. New York: McGraw-Hill; 005:1187–100.
of surgery: the biological basis of modern surgical practice. 17th e. Phila- Garen JO, e. Hepatobiliary and pancreatic surgery. 4th e. New
elphia: W.B. Sauners; 004:1597–164. York: Elsevier; 009:08.
Chun K. Recent classications of the common bile uct injury.
Korean J Hepatobiliary Pancreat Surg. 014;18(3):69–7. 18. E. The routine use of IOC to prevent bile uct injury is
Costamagna G, Shah SK, Tringali A. Current management of
controversial, but most surgeons woul say that routine use
postoperative complications an benign biliary strictures. Gastroin-
test Endosc Clin N Am. 003;13(4):635–648.
is unnecessary. Because the overall risk of bile uct injury
Lopez RR, Jr, Cosenza CA, Lois J, etal. Long-term results of metal- is so small, to ate there are no sufciently large-scale ran-
lic stents for benign biliary strictures. Arch Surg. 001;136(6):664–669. omize stuies to answer this question. Most likely, the use
Osottir M, Hunter, J. G. Gallblaer. In: Brunicari FC, Aner- of IOC will not prevent an injury to the CBD (A). However,
sen DK, Billiar TR, etal., es. Schwartz’s principles of surgery. 8th e. IOC seems to allow earlier recognition of a CBD injury an
New York: McGraw-Hill; 005:1187–100. prevent complete transection of the CBD. Although routine
Siriwarana HPP, Siriwarena AK. Systematic appraisal of the IOC will ientify unsuspecte CBD stones, in most instances,
role of metallic enobiliary stents in the treatment of benign bile uct CBD stones are suspecte preoperatively by abnormal liver
stricture. Ann Surg. 005;4(1):10–19. function tests, a ilate CBD, or a history of gallstone pancre-
atitis. In a nationwie retrospective analysis, CBD injury was
16. A. The presentation is consistent with acalculous cho- foun in 0.39% of patients unergoing cholecystectomy with
lecystitis. The initial stuy of choice is ultrasonography, IOC an in 0.58% of patients without IOC (unajuste rela-
which can be performe at the besie. Finings to conrm tive risk, 1.49). After controlling for patient-level factors an
the iagnosis woul inclue thickening of the gallblaer surgeon-level factors, the risk of injury was increase when
wall, sluge (as in this patient), an pericholecystic ui. If IOC was not use (ajuste relative risk, 1.71). Some sur-
the ultrasoun nings are negative an the patient is not geons prefer selective use of IOC an obtain what is known

AL GRAWANY
30 PArt i Patient Care

as the “critical view,” whereby the cystic uct an artery are pancreatic uct at the time of surgery, making the pancre-
carefully ientie an not clippe or cut until conclusive aticojejunostomy in a Whipple proceure easier to perform.
ientication has been mae. This is one by completely is- References: Ahrent SA, Pitt HA. Biliary tract. In: Townsen
secting the Calot triangle free of all fat an brous tissue an CM, Jr, Beauchamp RD, Evers BM, Mattox KL, es. Sabiston textbook
issecting the lower part of the gallblaer off the liver be, of surgery: the biological basis of modern surgical practice. 17th e. Phila-
such that only two skeletonize structures (the cystic uct elphia: W.B. Sauners; 004:1597–164.
Sewnath ME, Karsten TM, Prins MH, Rauws EJA, Obertop H,
an artery) are seen to be entering the gallblaer.
Gouma DJ. A meta-analysis on the efcacy of preoperative bili-
Reference: Sauners WB, Detry O, De Roover A, Detroz B. The
ary rainage for tumors causing obstructive jaunice. Ann Surg.
role of intraoperative cholangiography in etecting an preventing
00;36(1):17–7.
bile uct injury uring laparoscopic cholecystectomy. Acta Chirur-
Sohn TA, Yeo CJ, Cameron JL, Pitt HA, Lillemoe KD. Do preoper-
gica Belgica. 003;103():161–16.
ative biliary stents increase postpancreaticouoenectomy compli-
cations? J Gastrointest Surg. 000;4(3):58–67.
19. C. Emphysematous cholecystitis occurs in less than 1%
of acute cholecystitis cases. It is a isease that occurs pre-
ominantly in elerly iabetic men. The hallmark feature is 21. E. This patient presents with a history an nings
characterize by gas within the gallblaer wall or lumen. consistent with cholangiohepatitis, also known as recurrent
This can be seen on plain raiograph, ultrasoun, or com- pyogenic cholangitis. It is enemic in Asia, although the inci-
pute tomography (CT) scan. Gangrene of the gallblaer ence has been ecreasing. Cholangiohepatitis affects both
is present in three-fourths of all cases, an perforation of sexes equally (C). The etiology of cholangiohepatitis seems to
the gallblaer occurs in more than 0% of cases (E). In one be a combination of bacterial an parasitic (Clonorchis sinen-
large series, the mortality rate was 5% an the morbiity sis, Opisthorchis viverrini, an Ascaris lumbricoides) infections
rate was 50% espite aggressive treatment with broa-spec- in the biliary tree. The bacteria econjugate bilirubin, which
trum antibiotics an emergent surgery. In patients that are has a greater propensity to precipitate as bile sluge. Brown
unstable, an not eeme suitable for general anesthesia pigment stones form as a consequence of the sluge an
(such as a patient on pressors or multiple meical problems), ea bacterial cells (B). In aition, the nucleus of the stone
percutaneous rainage with cholecystostomy shoul be per- may harbor a parasite egg. The stones lea to recurrent epi-
forme rst. If the patient is more stable, cholecystectomy soes of cholangitis, liver abscesses, stricture formation, liver
is preferre (B). Although prior stuies suggeste open cho- failure, an an increase risk of cholangiocarcinoma. Recur-
lecystectomy was preferre, laparoscopic cholecystectomy rence is high. Initial treatment is with ERCP an transhepatic
is an acceptable approach, provie a low threshol for cholangiography. Patients often require multiple interven-
conversion an stanar principles are use. Antimicrobial tions to clear the biliary tree. The patient may eventually
coverage shoul inclue Clostridia perfringens, which is an require a biliary enteric bypass, but this woul not be the
anaerobic gram-positive ro an consiere the most com- initial proceure of choice. Metroniazole is the treatment of
mon cause of emphysematous cholecystitis (D). High-ose choice for amebic liver abscess (D). Hyati liver isease is a
penicillin shoul be starte immeiately (A). Other common liver cyst cause by Echinococcus an is associate with close
biliary pathogens associate with emphysematous cholecys- contact with ogs an sheep (A).
titis inclue Clostridia welchii, Escherichia coli, Enterococcus,
an Klebsiella. 22. D. Cancer of the gallblaer is preominantly aenocar-
References: Ahrent SA, Pitt HA. Biliary tract. In: Townsen cinoma. The majority of cases are iscovere in an avance
CM, Jr, Beauchamp RD, Evers BM, Mattox KL, es. Sabiston textbook state with istant metastases. Thus, the overall prognosis is
of surgery: the biological basis of modern surgical practice. 17th e. Phila-
very poor, with a 5-year survival rate of only 5%. The best
elphia: W.B. Sauners; 004:1597–164.
Tellez GS, Roriguez-Montes L, Fernanez e Lis J. Acute
chance of cure is if it is iscovere incientally at the time of
emphysematous cholecystitis: report of twenty cases. Hepatogastro- cholecystectomy. It is 17 times more likely to be iscovere in
enterology. 1999;46(8):144–148. patients following open cholecystectomy as compare with
laparoscopic cholecystectomy. Gallblaer cancer metas-
20. B. Several stuies have analyze the role of preoper- tasizes rst to the celiac axis lymph noes. Recent stuies
ative biliary rainage via ERCP an stenting in patients inicate that those that are iscovere incientally an are
with malignant obstructive jaunice who are to unergo a supercial, such as carcinoma in situ an T1 lesions (o not
Whipple proceure. Theoretically, relief of jaunice might exten into perimuscular connective tissue), an have neg-
improve the operative risk of the subsequent Whipple proce- ative margins, can be manage by cholecystectomy alone
ure. However, a large meta-analysis an single-center stu- (B), with a 100% 5-year survival. Those that are more locally
ies faile to show improve morbiity an mortality rates avance, such as T through T4 lesions (those that invae
with preoperative biliary rainage. In fact, the routine use the perimuscular connective tissue or irectly invae the
of preoperative biliary rainage seems to increase the risk liver), are treate with a raical cholecystectomy, which
of infectious complications incluing woun infection (10% inclues subsegmental resection of segments IVb an V, plus
with rainage versus 4% without) an increases the risk of hepatouoenal ligament lymphaenectomy, which results
pancreatic stula (10% with rainage versus 4% without). in prolonge survival (C). The caveat is that there must be no
Thus, it shoul only be use selectively (e.g., presence of evience of istant metastases. In one series of 48 patients,
cholangitis or severe, intractable pruritus). It has not been the overall 5-year survival rate was 13%, but it was 60% for
emonstrate to ecrease the risk of cholangitis (A), shorten patients who unerwent raical cholecystectomy. The rai-
hospital stay (D), or ecrease the mortality rate (E). Aition- cal cholecystectomy group ha signicantly longer survival
ally, obstructive jaunice provies the surgeon with a ilate than the simple cholecystectomy group for all stages except
CHAPtEr 3 Abdomen—Biliary 31

stage I (T1N0). Although port sites are associate with peri- exposure to carcinogens (nitrosamines, azotoluene). Obe-
toneal isease an ecrease survival, removing them oes sity has recently been shown to be a risk factor for a wie
not improve survival an shoul not be one routinely in all range of cancers, incluing the gallblaer (E). Speckle
patients with incientally iscovere gallblaer cancer (E). cholesterol eposits on the gallblaer wall are a feature of
Raiation therapy with uorouracil raiosensitization is the cholesterolosis an are not associate with an increase risk
most commonly use postoperative treatment. of cancer (A). Selective mucosal calcium eposits (porce-
References: Osottir, M, Hunter, J G. Gallblaer. In: Bruni- lain gallblaer) may have an increase risk of malignancy.
cari FC, Anersen DK, Billiar TR, etal., es. Schwartz’s principles of Thickene noules of mucosa an muscle in the gallblaer
surgery. 8th e. New York: McGraw-Hill; 005:1187–100. are a feature of aenomyomatosis (B). Tumor invaing the
Rei KM, Ramos-De la Meina A, Donohue JH. Diagnosis an lamina propria, but not yet invae all the way through an
surgical management of gallblaer cancer: a review. J Gastrointest
to the unerlying muscularis, is consiere T1a isease an
Surg. 007;11(5):671–681.
treate with simple cholecystectomy. Invasion to the uner-
Taner CB, Nagorney DM, Donohue JH. Surgical treatment of
gallblaer cancer. J Gastrointest Surg. 004;8(1):83–89.
lying muscularis is T1b isease an requires resection of liver
Pitt SC, Jin LX, Hall BL, Strasberg SM, Pitt HA. Inciental gall- segments IVb an V an regional lymph noe issection.
blaer cancer at cholecystectomy: when shoul the surgeon be sus- References: Osottir M, Hunter J. G. Gallblaer. In: Bruni-
picious? Ann Surg. 014;60(1):18–133. cari FC, Anersen DK, Billiar TR, etal., es. Schwartz’s principles of
surgery. 8th e. New York: McGraw-Hill; 005:1187–100.
23. B. Sclerosing cholangitis is characterize by the pres- Stephen AE, Berger DL. Carcinoma in the porcelain gallblaer:
ence of multiple inammatory brous thickenings resulting a relationship revisite. Surgery. 001;19(6):699–703.
Chen, G. L., Akmal, Y., DiFronzo, A. L., etal. (015).
in irregular narrowing of the entire biliary tree (C). It is pro-
gressive an as such leas eventually to biliary obstruction,
25. D. The exact etiology of choleochal cysts is unclear.
recurrent biliary infection, cirrhosis, an liver failure, as well
The most likely explanation is that there is an anomalous
as a signicantly increase risk of cholangiocarcinoma (in
pancreaticobiliary uct junction. Specically, the pancreatic
10%–0% of patients). All patients shoul be checke for an
uct joins the common bile uct more than 1 cm proximal
elevate level of CA 19-9. It is twice as common in men, an
to the ampulla, resulting in a long common channel. The
also tens to occur in younger patients (E). Risk factors for
long channel leas to free reux of pancreatic secretions
sclerosing cholangitis inclue inammatory bowel isease,
into the biliary tract, resulting in increase biliary pressures
pancreatitis, an iabetes. The strongest association is with
an inammatory changes in the biliary epithelium, which
ulcerative colitis (A). Approximately two-thirs of patients
eventually lea to ilation an cyst formation. Although an
have ulcerative colitis. In fact, it is usually iscovere in these
abnormal pancreaticobiliary junction is present in the major-
patients when an abnormal liver function test result is note.
ity of patients with choleochal cysts, it is not uniformly
Alkaline phosphatase is characteristically elevate out of pro-
seen. Choleochal cysts are more common in females an
portion to an elevate bilirubin level. Patients may test posi-
Asians. It classically presents in chilhoo with jaunice
tive for p-ANCA antiboies (in contrast to antimitochonrial
an an abominal mass accompanie by abominal pain.
antiboies for primary biliary cirrhosis). It is less commonly
In infants, it may be confuse with biliary atresia. However,
associate with Crohn isease. Other iseases associate
less than 50% of patients present with all three features, an
with sclerosing cholangitis inclue Rieel thyroiitis an
thus the iagnosis is often elaye. The most common pre-
retroperitoneal brosis. Removing the colon in patients with
sentation is nonspecic abominal pain. The iagnosis is
ulcerative colitis oes not affect the course of the sclerosing
mae by ultrasonography, which can sometimes etect the
cholangitis. In aition, the severity of inammation oes not
cyst antenatally. There are ve types. Type I is the most com-
preict the onset of malignancy. All newly iagnose patients
mon (90%) an consists of fusiform ilation of the bile uct.
with sclerosing cholangitis with or without an inammatory
Type V, also known as Caroli isease, is characterize by
bowel isease iagnosis shoul be scheule for a screen-
multiple intrahepatic ilations. Because of the risk of malig-
ing colonoscopy. Patients can be manage initially with ste-
nant egeneration, treatment involves excising the cyst with
rois, methotrexate, an cyclosporine, but the majority will
a biliary enteric bypass (typically hepaticojejunostomy). The
ultimately require more invasive treatment incluing biliary
risk of malignancy increases with the more avance age at
stenting (D). Currently, the best option is liver transplanta-
which the cyst is etecte. Type V (Caroli) will nee a par-
tion in patients who progress to liver failure.
tial liver resection or liver transplant. Biliary smooth muscle
Reference: Osottir, M, Hunter, JG. Gallblaer. In: Bruni-
(A), mucosa (E), uctal aventitia (C), an bile (B) are not
cari FC, Anersen DK, Billiar TR, etal., es. Schwartz’s principles of
surgery. 8th e. New York: McGraw-Hill; 005:1187–100.
thought to play a role in choleochal cyst isease.
References: Osottir, M, Hunter, J. G. Gallblaer. In: Bruni-
cari FC, Anersen DK, Billiar TR, etal., es. Schwartz’s principles of
24. D. Gallblaer cancer is two to three times more com-
surgery. 8th e. New York: McGraw-Hill; 005:1187–100.
mon in females (C). It is also more common in Native Amer- Toani T, Watanabe Y, Fujii T, Uemura S. Anomalous arrange-
icans in both North an South America. Approximately 90% ment of the pancreatobiliary uctal system in patients with a chole-
of patients with carcinoma also have gallstones. Large single ochal cyst. Am J Surg. 1984;147(5):67–676.
stones have a much higher risk of cancer than multiple small
stones, likely the result of creating more mucosal inam- 26. C. Type I choleochal cysts are the most common type
mation; large stones also are more likely to lea to chole- an are ilations of either the entire common hepatic uct an
cystoenteric stulas. Other risk factors inclue choleochal CBD or a segment of it. Management consists of excision of
cysts (which may be ue to an abnormal pancreaticobiliary the entire cyst an a biliary enteric bypass. An exception is if
junction), sclerosing cholangitis, gallblaer polyps, an the posterior wall of the cyst is stuck to the portal vein, which
32 PArt i Patient Care

occasionally occurs ue to ongoing inammation. Roux-en-Y approach using lobectomy is avocate. Ajuvant raiation
cyst jejunostomy alone woul not be sufcient (E). Dissection therapy has also not been shown to improve either quality of
of the posterior wall can sometimes be precarious because it life or survival in resecte patients. Patients with unresect-
may be stuck to the portal vein. In this case, the posterior wall able isease are often offere treatment with 5-uorouracil
shoul be left in situ an the mucosa fulgurate or curette alone or in combination with mitomycin C an oxorubicin,
(Lilly proceure) because this will still theoretically remove but the response rates are low. A Whipple proceure woul
the risk of malignancy. Type II choleochal cysts are ivertic- be appropriate for a istal CBD tumor (A).
ula that project from the CBD wall. Type III choleochal cysts References: Capussotti L, Muratore A, Polastri R, Ferrero A,
are foun in the intrauoenal portion of the CBD (also calle Massucco P. Liver resection for hilar cholangiocarcinoma: in-hospital
a choledochocele). Type IVa cysts are characterize by multi- mortality an longterm survival. J Am Coll Surg. 00;195(5):641–647.
ple ilations of the intrahepatic an extrahepatic biliary tree. Dinant S, Gerhars MF, Rauws EAJ, Busch ORC, Gouma DJ, van
Gulik TM. Improve outcome of resection of hilar cholangiocarci-
Most frequently, a large solitary cyst of the extrahepatic uct
noma (Klatskin tumor). Ann Surg Oncol. 006;13(6):87–880.
is accompanie by multiple cysts of the intrahepatic ucts.
Lygiakis N, Sgourakis G, Deemai G. Long-term results fol-
Type IVb choleochal cysts consist of multiple ilations that lowing resectional surgery for Klatskin tumors: a twenty-year per-
involve only the extrahepatic bile uct. Type V choleochal sonal experience. Hepatogastroenterology. 001;48(37):95–101.
cysts (Caroli isease) consist of ilations of the intrahepatic Osottir, M, Hunter, JG. Gallblaer. In: Brunicari FC,
biliary tree. Partial resection may be inicate for Type V cho- Anersen DK, Billiar TR, etal., es. Schwartz’s principles of surgery.
leochal cyst (A, B). There is no role for observation (D). 8th e. New York: McGraw-Hill; 005:1187–100.
References: Ahrent SA, Pitt HA. Biliary tract. In: Townsen
CM, Jr, Beauchamp RD, Evers BM, Mattox KL, es. Sabiston textbook 28. E. Cholangiocarcinoma arises from bile uct epithelium
of surgery: the biological basis of modern surgical practice. 17th e. Phil- (D). Although it can occur anywhere along the biliary tree, the
aelphia: W.B. Sauners; 004:1597–164. majority occurs extrahepatically, while only 0% are intrahe-
Osottir, M, Hunter, JG. Gallblaer. In: Brunicari FC,
patic (A). It is a locally aggressive cancer but can have irect
Anersen DK, Billiar TR, etal., es. Schwartz’s principles of surgery.
sprea to the liver an peritoneum. The Bismuth-Corlette
8th e. New York: McGraw-Hill; 005:1187–100.
Toani T, Watanabe Y, Fujii T, Uemura S. Anomalous arrange- classication system organizes cholangiocarcinoma by loca-
ment of the pancreatobiliary uctal system in patients with a chole- tion: type I occurs below the conuence of the left an right
ochal cyst. Am J Surg. 1984;147(5):67–676. hepatic ucts; type II occurs at the juncture of the left an
right hepatic ucts; type III involves either the left or right
27. B. Perihilar cholangiocarcinomas are also known as hepatic uct; an type IV involves seconary extensions of
Klatskin tumors. They are classie into four types base either the left or right hepatic ucts (B). MRCP is an appropri-
on whether they are limite to the common hepatic uct ate initial imaging stuy to ene the anatomy an plan for
(typeI), involve the bifurcation of the right an left hepatic surgical intervention. ERCP is the most valuable iagnostic
ucts (type II), or enter into the seconary right (type IIIa) or tool an allows for biopsy brushings. Intrahepatic isease can
left (type IIIb) intrahepatic ucts. Surgery is the only treat- be manage with hepatic wege resection while extrahepatic
ment that has shown potential for long-term survival, pro- isease nees resection with biliary-enteric bypass. However,
vie the tumor has no evience of istant sprea (D, E). this is only appropriate for patients that o not have exten-
Type I an II tumors involve resection of the entire extrahe- sive local isease (involvement of the portal vein trunk or
patic biliary tree with portal lymphaenectomy an bilateral hepatic arteries), noal involvement, or istant metastases.
Roux-en-Y hepaticojejunostomies (C). More recently, an even Distal cholangiocarcinoma will nee a pancreaticouoenec-
more aggressive approach has been taken for type I an II tomy. The National Comprehensive Cancer Network rec-
tumors to inclue a hemihepatectomy to achieve negative ommens consieration of chemoraiation in patients with
margins. Using this approach, several authors have shown positive margins or noal isease, but it shoul not be one
improve survival. For type III lesions, a similar aggressive routinely (C).
Abdomen—Liver
NAVEEN BALAN, KATHRYN T. CHEN, AND DANIELLE M. HARI 4
ABSITE 99th Percentile High-Yields
I. Hepatocellular Carcinoma
A. Etiology—arises in the setting of chronic inammation or cirrhosis
1. Hepatitis-C (most common), hepatitis-B, NAFLD (non-alcoholic fatty liver isease), EtOH, alpha-1-
antitrypsin eciency, hemochromatosis, Wilson isease, aatoxin exposure
B. Orthotopic liver transplant inications (MILAN criteria)
1. One lesion <5 cm or 3 or less lesions each <3 cm, no vascular invasion or metastasis
C. Liver resection inicate in goo surgical caniates
D. Options for patients who are poor surgical caniates: ablation (microwave/raiofrequency/
cryoablation), TACE (transarterial chemoembolization)
E. Future liver remnant (FLR): remaining liver following resection require to prevent liver failure
1. Healthy patient: 0% to 5% neee; injure liver (e.g., post-chemo): 30% neee; cirrhotic patient:
40% neee
. Portal vein embolization of isease segment to hypertrophy the contralateral (healthy) sie offere
to patients that fall below these threshols
F. Transjugular intrahepatic portosystemic shunt (TIPS)
1. Emergent inication: massive esophageal variceal blee refractory to meical an enoscopic
therapy, nees ecompression of the portal venous system; hepatic encephalopathy may evelop

Mass Background Imaging Management


Hemangioma Most common CT—hypodense with Most are observed, surgery resection vs enucleation
benign tumor peripheral nodular if symptomatic; can be associated with Kasabach-
enhancement MerriĴ syndrome (consumptive coagulopathy
from sequestration of platelets and congestive
heart failure)
Focal nodular Benign, second CT—hyperenhance Tc-99m sulfur colloid test if unclear diagnosis (FNH
hyperplasia most common on arterial phase will have normal/increased uptake), observation
(FNH) with central scar, only, no intervention needed
centrifugal ęlling on
delayed
Adenoma Associated CT—hyperenhance Cessation of OCP with follow-up imaging, if
with oral on arterial phase, persistent or enlarging then needs resection due
contraceptive centripetal ęlling on to rupture/malignant degeneration risk; resection
(OCP), steroid delayed phase recommended for all males; will have decreased
use uptake on sulfur colloid scan

33
34 PArt i Patient Care

Mass Background Imaging Management


Hydatid cyst Echinococcus CT/US—cystic lesions Albendazole, if failure then proceed with PAIR
granulosus with daughter cysts (puncture, aspiration, injection, reaspiration)
exposure
Amebic abscess Secondary to CT—low-density mass Serologic testing; once diagnosis conęrmed, then
Entamoeba with peripheral treat with metronidazole
histolytica enhancing rim
Pyogenic abscess Most due to biliary CT—double target or Percutaneous drain if >5 cm with antibiotics, if <5
infections cluster sign, air-Ěuid cm in size, then antibiotics alone
level
Hepatocellular Most common CT—rapid arterial If imaging is consistent and AFP elevated, no
carcinoma malignant tumor enhancement with need for biopsy; treat with resection, ablation,
(HCC) rapid washout on embolization, transplant
portal venous phase
Fibrolamellar More common in CT—central scar that Surgery can be curative, otherwise need
HCC women does not enhance chemotherapy vs embolization
CHAPtEr 4 Abdomen—Liver 35

Fig. 4.1
36 PArt i Patient Care

Questions
1. A 48-year-ol man with a history of Chil-Pugh 4. A 45-year-ol man with a history of alcohol
C cirrhosis seconary to nonalcoholic fatty liver abuse presents with recurrent UGI bleeing. His
isease is foun to have multifocal hepatocellular history is signicant for alcoholic pancreatitis. On
carcinoma. Imaging shows a -cm lesion in upper enoscopy, he is foun to have bleeing
segment VI, a 3-cm lesion in segment VII, an a from isolate gastric varices. The bleeing is
3-cm lesion in segment II. What is the appropriate controlle meically. On CT imaging, the portal
management? an superior mesenteric veins are patent, but
A. Liver transplantation the splenic vein is not visualize. Optimal
B. Right hepatectomy management for this patient woul be:
C. Chemotherapy A. Sie-to-sie portacaval shunt
D. Extene right hepatectomy B. Mesocaval shunt
E. Raiofrequency ablation of each lesion C. Distal splenorenal shunt
D. Long-term beta-blocker therapy
2. An otherwise healthy 6-year-ol woman is E. Splenectomy
foun to have anemia on her annual physical
exam. After workup, she is foun to have a 5. A 30-year-ol Hispanic man with a history
sigmoi colonic aenocarcinoma an multiple of alcohol abuse presents with a high fever,
hepatic lesions in the right hepatic lobe an right upper quarant pain, an leukocytosis.
several in the left hepatic lobe that are biopsy- Ultrasonography reveals a 5-cm ui collection
proven metastatic aenocarcinoma. Her future in the right lobe of the liver. On the CT scan, the
liver remnant is estimate to be 15%. Which of the ui collection shows a peripheral rim of eema.
following is recommene? The cause of the ui collection is most likely to
A. Chemotherapy alone be etermine by:
B. Colectomy followe by chemotherapy A. Bloo cultures
C. Concomitant colectomy an right hepatectomy B. Stool cultures
followe by chemotherapy C. Percutaneous aspiration of liver
D. Colectomy, postoperative chemotherapy, then D. Serologic tests
transarterial chemoembolization (TACE) of E. Liver function tests
liver lesions
E. TACE of liver lesions 6. Denitive management of the patient in
question 5 consists of:
3. An 8-year-ol girl presents with upper A. Oral metroniazole
gastrointestinal (UGI) bleeing. The physical B. Broa-spectrum antibiotics an open surgical
examination emonstrates splenomegaly. Her rainage
meical history is signicant for a prolonge C. Broa-spectrum antibiotics an early
stay in the neonatal intensive care unit at birth percutaneous aspiration of the abscess
ue to prematurity, complicate by necrotizing D. Broa-spectrum antibiotics an CT-guie
enterocolitis. She has no history of travel outsie catheter insertion to rain the abscess
the Unite States. Laboratory testing reveals a E. Broa-spectrum antibiotics an laparoscopic
hematocrit of 0% an normal bilirubin, albumin, rainage
an international normalize ratio. After ui
resuscitation, an upper enoscopy is performe 7. The most common benign tumor of the liver is:
that reveals esophageal varices. The patient is A. FNH
given octreotie an unergoes sclerotherapy. B. Hepatic aenoma
Which of the following stuies will most likely C. Hemangioma
etermine the cause of her UGI blee? D. Mesenchymal hamartoma
A. Duplex ultrasonography of the portal vein E. Inammatory pseuotumor
B. Duplex ultrasonography of the splenic vein
C. CT scan of the abomen
D. MRI of the abomen
E. Liver biopsy
CHAPtEr 4 Abdomen—Liver 37

8. Which of the following is true regaring liver 13. The best screening approach for etecting early
cysts associate with polycystic liver isease? HCC in patients with chronic viral hepatitis is:
A. Laparoscopic fenestration is the preferre A. Alpha-fetoprotein (AFP) level
treatment option B. AFP level an ultrasonography
B. It has an autosomal recessive inheritance C. Compute tomography
pattern D. Carcinoembryonic antigen (CEA) level
C. They are typically symptomatic E. Alkaline phosphatase level
D. Oral estrogen therapy can be helpful
E. Liver function tests are usually abnormal 14. A 36-year-ol woman presents with right upper
quarant pain, jaunice, evience of ascites, an
9. Which of the following is the best metho to an enlarge liver on physical examination. CT
prevent a rst blee in a patient with known large emonstrates marke hypertrophy of segment
esophageal varices? 1 of the liver, free ui in the peritoneum,
A. Beta-blockae an inhomogeneous contrast enhancement
B. Transjugular intrahepatic portosystemic shunt of the remainer of the liver. This most likely
(TIPS) placement inicates:
C. Sclerotherapy A. Bu-Chiari synrome
D. Enoscopic ligation B. Rupture hepatic aenoma
E. Selective portosystemic shunt placement C. Rupture hemangioma
D. Acute hepatitis
10. Which of the following is true regaring bile E. Schistosomiasis
acis?
A. Deoxycholic aci an lithocholic aci are 15. Which of the following is true regaring hepatic
primary bile acis aenomas?
B. Cholic an chenoeoxycholic acis are A. They o not occur in men
seconary bile acis B. They ten to appear “hot” on a sulfur colloi
C. Seconary bile acis are forme by intestinal liver scan
bacteria C. Rapi contrast enhancement on CT
D. After ingestion of foo, bile aci concentration istinguishes them from FNH
in the portal vein increases D. Rupture risk appears to be associate with
E. Ingestion of foo leas to an inhibition of tumor size
cholesterol 7-hyroxylase E. They contain an abunance of
nonparenchymal (Kupffer) cells
11. Which of the following is true regaring the
portal vein? 16. Which of the following treatments of a hyati
A. It typically has one or two valves cyst locate in the mi-right lobe of the liver
B. It supplies approximately one-thir of the is associate with the lowest recurrence rate,
bloo to the liver morbiity, an mortality?
C. The normal pressure is 10 to 1 mm Hg A. Long-term oral albenazole
D. It is forme by the conuence of the inferior B. Laparoscopic cyst excision with omentoplasty
mesenteric an splenic veins C. Long-term oral mebenazole
E. In the hepatouoenal ligament, it is usually D. Surgical total pericystectomy with pre- an
posterior to both the bile uct an hepatic postoperative albenazole
artery E. Percutaneous aspiration an injection of
scolicial agents
12. Focal noular hyperplasia (FNH):
A. Is typically symptomatic
B. Is usually centrally locate in the liver
C. Is best conrme with high-resolution
compute tomography (CT)
D. Poses a signicant risk of rupture
E. Is thought to be ue to an embryonic
isturbance in liver bloo ow
38 PArt i Patient Care

17. A 51-year-ol male with liver cirrhosis presents 22. The Moel for En-Stage Liver Disease (MELD)
with a moerately size, reucible, umbilical score:
hernia that occasionally causes pain. The skin A. Inclues an assessment of the severity of
is intact an there is no rainage. He has a ascites
signicant amount of ascites. Serum bilirubin, B. Inclues the presence of encephalopathy
albumin, an international normalize ratio are C. Is similar to Chil-Pugh in that they both use
normal. He has no encephalopathy. He oes not INR an serum creatinine
have any pain at the hernia site. Which of the D. Is not as useful as the Chil-Pugh classication
following woul be the most appropriate next E. Preicts 3-month mortality in patients
step in management? awaiting liver transplantation
A. TIPS placement
B. Six-liter paracentesis followe by intravenous 23. Which of the following is true regaring the bloo
(IV) albumin replacement supply to the liver?
C. Procee to surgical repair of the hernia A. The mile hepatic vein joins the right hepatic
D. Furosemie, spironolactone, an soium vein as it enters the inferior vena cava
restriction B. Veins from the cauate lobe rain primarily
E. Observation into the right hepatic vein
C. The ligamentum venosum marks the location
18. The most common ientiable source of a of the intrahepatic portal vein
pyogenic liver abscess is: D. A replace left hepatic artery most commonly
A. Seeing from the portal vein arises from a branch of the celiac axis
B. The biliary tree E. The proper hepatic artery gives rise to the
C. Hematogenous from enocaritis gastrouoenal artery in most instances
D. Direct extension of a nearby focus
E. Inammatory bowel isease 24. Which of the following is not consiere a poor
preictor of survival after hepatic resection for a
19. The principal meiators of brosis leaing to metastatic colorectal cancer?
cirrhosis in the liver are: A. Hepatic metastasis measuring 4 cm
A. Hepatocytes B. Noes positive in colon primary
B. Ito (liver stellate) cells C. Hepatic metastasis eveloping 6 months after
C. Enothelial cells primary resection
D. Kupffer cells D. Four small hepatic metastases
E. Clefts of Mall E. Very high CEA levels

20. Fibrolamellar carcinoma (FLC) of the liver: 25. The most common primary liver malignancy in
A. Is strongly associate with hepatitis B chilren is:
B. Most often occurs in elerly men A. HCC
C. Causes a marke elevation in AFP levels B. FLC
D. Often contains a central scar C. Intrahepatic cholangiocarcinoma
E. Has a worse prognosis than HCC D. Giant cell carcinoma
E. Hepatoblastoma
21. Which of the following is least likely to increase
the risk of HCC? 26. The most accurate test for assessment of hepatic
A. Toxins from Aspergillus reserve before major hepatic resection is:
B. Hyrocarbons A. Aminopyrine breath test
C. Smoking B. Inocyanine green clearance
D. Wilson isease C. Bromsulphthalein retention
E. Pesticies D. Sulfur colloi scan
E. Bile aci tolerance
CHAPtEr 4 Abdomen—Liver 39

27. Which of the following is true regaring Bu- 32. Which of the following is true regaring TIPS?
Chiari synrome? A. It is contrainicate in patients with poorly
A. It may benet from percutaneous angioplasty controlle ascites
an stenting B. It has a signicant risk of causing
B. Diagnosis is best mae by portal venography encephalopathy
C. The jaunice is cause by presinusoial liver C. It is consiere to be a selective shunt
failure D. It is best use for long-term portal
D. TIPS placement is contrainicate ecompression
E. Liver function test is often normal E. It has a low 1-year rate of shunt occlusion

28. In patients with fulminant hepatic failure, the 33. A 30-year-ol woman with symptoms an signs
complication that most frequently leas to eath is: of symptomatic cholelithiasis is foun to have
A. Renal failure gallstones an a 4-cm mass in the left lateral
B. Pneumonia lobe of the liver on an ultrasoun scan. The
C. Hypoglycemia patient takes oral contraceptives but no other
D. Intracranial hypertension meications. Contrast-enhance MRI reveals
E. Coagulopathy a lesion of low signal intensity with peripheral
noular enhancement, an T-weighte images
29. A 30-year-ol woman is foun to have an reveal high signal intensity. Management
inciental 3-cm mass in the liver on CT scan consists of:
that intensely enhances in the arterial vascular A. Laparoscopic cholecystectomy with a neele
phase. The lesion is “hot” on a technetium-99m– biopsy of the liver mass
macroaggregate albumin liver scan. Which of B. Laparoscopic cholecystectomy alone
the following is true about this lesion? C. A trial of contraceptive cessation
A. It is usually centrally locate D. Open cholecystectomy with a wege liver
B. It poses a signicant risk of rupture resection
C. It poses a signicant risk of malignancy E. Open cholecystectomy with a left lateral
D. It is thought to be cause by an embryologic segmentectomy
vascular injury
E. It is compose of sheets of hepatocytes with no 34. The most common cause of intrahepatic
Kupffer cells presinusoial portal hypertension is:
A. Alcohol
30. Which of the following is true regaring B. Bu-Chiari synrome
comparisons of amebic an pyogenic liver C. Schistosomiasis
abscesses? D. Hemochromatosis
A. Amebic abscesses have a much higher female E. PVT
preponerance
B. Mortality rates are similar 35. During iagnostic laparoscopy preceing
C. Both are more likely to occur in the left lobe pancreaticouoenectomy in a patient with
D. Percutaneous aspiration is more likely to be pancreatic cancer, a -mm, rm, white lesion is
neee with amebic abscesses note on the periphery of the liver. Which of the
E. Pyogenic abscesses are more likely to be multiple following is true?
A. The proceure shoul be aborte at this time
31. A 30-year-ol woman who is taking oral B. The most likely etiology is a bile uct
contraceptives is iscovere to have a 4-cm hamartoma
asymptomatic soli mass in the right lobe of the C. Biopsy of the lesion shoul not be one at this
liver on an ultrasoun scan. CT emonstrates time
a central stellate scar within the mass that D. The patient likely has abnormal liver function
enhances on arterial phase. Management tests (LFTs)
consists of: E. Wege resection of the liver shoul be
A. Observation performe
B. Discontinuing oral contraceptives, repeating
the CT scan in 6 months, an resection if the
mass has not ecrease in size
C. Resection of the mass with a 1-cm margin
D. Raiofrequency ablation
E. Formal hepatic lobectomy

AL GRAWANY
40 PArt i Patient Care

Answers
1. A. The most common inication for liver transplantation intraabominal sepsis (leaing to infectious seeing of the
is en-stage liver isease (not cancer). However, the Milan portal vein). Some patients may have congenital webs in the
criteria for liver transplantation arose in 1996 following a portal vein (leaing to stasis), an a smaller fraction have
prospective cohort stuy that foun orthotopic liver trans- inherite hypercoagulable states. In one stuy of 100 neo-
plantation for select cirrhotic patients with hepatocellu- nates who unerwent umbilical vein catheterization, portal
lar carcinoma (HCC) to be efcacious. The specic criteria vein ultrasonography emonstrate clinically silent PVT in
inclue patients with Chil-Pugh B or C cirrhosis an HCC: 43%, an only 56% ha complete or partial resolution. The
either 1 lesion ≤5 cm or ≤3 lesions all ≤3 cm. Aitionally, the etiology of PVT in aults is ifferent. It is more likely asso-
cancer cannot involve major vascular structures or have evi- ciate with malignancy an cirrhosis. In most chilren, PVT
ence of extrahepatic sprea. For Chils-Pugh A an early B is clinically silent until esophageal varices an UGI bleeing
cirrhotic pts with HCC that satisfy the Milan criteria, hepatic evelop. Patients with PVT an without any bleeing shoul
resection is an accepte option. A right hepatectomy woul be starte on anticoagulation. This also applies to asymp-
involve resection of segments V-VIII an an extene right tomatic patients because complete recanalization or partial
hepatectomy woul involve resection of segments IV-VI— resolution improves survival. Initial treatment of the blee-
neither option woul treat segment II (B, D). Chemotherapy ing varices is similar to that for aults an inclues the use
is reserve for patients with unresectable tumors, metastatic of sclerotherapy or baning as well as octreotie. Because
isease, or palliation (C). Raiofrequency ablation is useful PVT in chilren is not usually associate with cirrhosis,
in poor surgical caniates with multiple or small lesions but liver function is intact, an the overall prognosis for these
in this patient who has an inication for a surgical cure, it is chilren is reasonably goo. Nevertheless, a portosystemic
not appropriate (E). shunt shoul be consiere in patients who are refractory to
meical management.
2. A. About 5% of patients with colorectal cancer present References: Kim JH, Lee YS, Kim SH, Lee SK, Lim MK, Kim
with synchronous liver metastases, an 30% will evelop HS. Does umbilical vein catheterization lea to portal venous
liver metastases uring the course of their isease. Patients thrombosis? Prospective US evaluation in 100 neonates. Radiology.
with colorectal cancer an hepatic metastasis may be appro- 001;19(3):645–650.
priate surgical caniates with curative intent. Patients Schettino GCM, Fagunes EDT, Roquete MLV, Ferreira AR, Penna
FJ. Portal vein thrombosis in chilren an aolescents. J Pediatr
with liver metastases are consiere caniates for hepatic
(Rio J). 006;8(3):171–178.
resection base on the volume of liver remaining after
resection an not the actual number of tumors. In patients
4. E. The ning of isolate gastric varices, without esoph-
with normal liver function, a 0% remnant is recommene
ageal varices, is highly suggestive of splenic vein thrombo-
but in a patient that has unergone neoajuvant chemo-
sis. This conition leas to venous outow obstruction of the
therapy, a 30% to 35% remnant is recommene. Options
spleen, resulting in massively ilate short gastric veins. The
inclue colon-rst, liver-rst, an concomitant resection.
most common cause of splenic vein thrombosis is chronic
None of these three strategies emonstrates inferiority com-
pancreatitis, which leas to perivenous inammation. It has
pare to the others. The surgery shoul be iniviualize
been reporte to occur in 4% to 8% of patients with chronic
to the patient base on concern for complications from the
pancreatitis. Splenic vein thrombosis with gastric vari-
primary tumor, progression of liver isease, an ifcul-
ceal formation is referre to as left-sie or sinistral portal
ties in concomitant resection. However, the patient above
hypertension. The mortality rate for gastric variceal bleeing
has unresectable isease given the FLR of 15%. In this case,
excees 0%. Splenectomy is curative. Controversy exists
resection of the primary colon tumor is no longer avocate
as to whether prophylactic splenectomy is necessary when
in the absence of complications such as obstruction, blee-
asymptomatic gastric varices are iscovere in association
ing, or perforation (B, C). Chemotherapy alone is the appro-
with splenic vein thrombosis. A recent stuy suggests that
priate choice (D). TACE is primarily reserve for patients
gastric variceal bleeing from pancreatitis-inuce splenic
with hepatocellular carcinoma. It involves injecting chemo
vein thrombosis occurs in only 4% of patients. Thus, pro-
followe by embolization of a major tumor artery which is
phylactic splenectomy is not recommene in asymptomatic
often from hepatic artery (E).
patients, nor is it recommene concomitant with another
planne abominal operation. Bypass proceures carry a
3. A. Variceal bleeing in chilren is rare. The combination
higher risk of morbiity an woul not aress the unerly-
of esophageal varices an splenomegaly in the absence of
ing problem (A–C). Long-term beta-blocker therapy is use
evience of cirrhosis (normal hepatic function) is highly sug-
as a prophylactic agent in patients with esophageal varices
gestive of portal vein thrombosis (PVT). The iagnostic test
seconary to cirrhosis (D).
of choice is a uplex ultrasoun scan of the portal vein (B–E).
References: Agarwal AK, Raj Kumar K, Agarwal S, Singh S.
PVT likely occurs because of a combination of factors that Signicance of splenic vein thrombosis in chronic pancreatitis. Am J
contributes to the Virchow tria (injury, stasis, an hyperco- Surg. 008;196():149–154.
agulability). Many chilren with PVT have a history of neo- Heier TR, Azeem S, Galanko JA, Behrns KE. The natural his-
natal umbilical vein catheterization (leaing to portal venous tory of pancreatitis-inuce splenic vein thrombosis. Ann Surg.
injury), neonatal omphalitis (umbilical sepsis), or neonatal 004;39(6):876–880.
CHAPtEr 4 Abdomen—Liver 41

Weber SM, Rikkers LF. Splenic vein thrombosis an gastroin- combination with ultrasoun-guie neele aspiration: a compara-
testinal bleeing in chronic pancreatitis. World J Surg. 003;7(11): tive, prospective an ranomize stuy: treatment of amoebic liver
171–174. abscess. Trop Med Int Health. 003;8(11):1030–1034.
McGarr PL, Maiba TE, Thomson SR. Amoebic liver
5. D. The iagnosis of an amebic liver abscess is mae using abscess-results of a conservative management policy. S Afr Med J.
a combination of the clinical presentation, ultrasoun an CT 003;93():13–136.
scan features, an serologic testing. The causative organism
is Entamoeba histolytica. Humans ingest the cysts through a 7. C. Hemangiomas are the most common benign tumors
fecal-oral route. The cyst becomes a trophozoite in the colon of the liver. They are usually iscovere incientally an
an invaes the colonic mucosa, resulting in a iarrheal ill- are typically asymptomatic. Diagnosis is generally mae
ness. The organism then reaches the liver via the portal vein. by characteristic features of CT an MRI. The main issues
It leas to a liquefaction necrosis of the liver, leaing to the of which to be aware are that they can sometimes be if-
escription of an “anchovy paste” appearance of the ui, cult to istinguish from malignancy an that in chilren,
which is a combination of bloo an liquee hepatic tissue. in particular, giant hemangiomas (>5 cm) can lea to arte-
The infection is much more common in enemic areas such as riovenous shunting with congestive heart failure an
Central an South America, Inia, an Africa, or in inivi- thrombocytopenia seconary to consumptive coagulopa-
uals who have ha recent travel to those locations. Less than thy (Kasabach-Merritt synrome). Hemangiomas can be
one-thir of patients will have a history of a iarrheal illness. remove by parenchymal sparing enucleation (not by for-
Amebic liver abscesses are much more common in patients mal resection). FNH is a benign asymptomatic liver lesion
with a history of heavy alcohol consumption, suggesting that locate on the periphery of the liver an typically iscov-
alcohol increases susceptibility. CT scanning can help istin- ere incientally on CT scan (A). Hepatic aenomas present
guish amebic liver abscesses from other entities, such as a in young women an in association with oral contraceptive
pyogenic abscess an echinococcal cysts. The classic ning use (B). Mesenchymal hamartoma of the liver typically
on CT is that of a single ui collection in the right lobe with affects young males an is consiere a benign lesion that
a rim of peripheral eema. It may be that the preilection for may present with intraabominal enlargement an respira-
right lobe abscesses is ue to receiving more rainage (an tory istress particularly in the neonate (D). Inammatory
more bacteria) from the biliary an GI tract (via superior mes- pseuotumor is a benign liver lesion that requires neele
enteric an portal veins), as compare to the left lobe (via biopsy for correct iagnosis (E).
inferior mesenteric an splenic veins). So, it may be that the
right lobe receives more bacteria an bloo from GI an bil- 8. A. Polycystic liver isease is an autosomal ominant
iary infections. Culturing the liver abscess or stool oes not isorer that is seen in patients with polycystic kiney
usually yiel ameba (B). The best test to establish the iag- isease, or it can be seen with liver cysts alone (B). The
nosis is serologic testing using enzyme immunoassays. The majority of patients are asymptomatic from their liver, but
test is typically not reliable until 7 to 10 ays after the patient on rare occasion, large cysts can prouce severe abomi-
is infecte. Conservative meical management of amebic nal pain requiring intervention (C). Various strategies have
liver abscess is safe. Percutaneous ultrasonography-guie been use with varying egrees of success in symptom-
aspiration is inicate only in patients who fail to improve atic patients with liver cysts. Laparoscopic fenestration
clinically after 48 to 7 hours (C). Amebic liver abscesses may has emerge as the preferre treatment option an has a
lea to milly elevate transaminase an bilirubin levels, but low risk of bleeing. Percutaneous aspiration, instillation
these nings are nonspecic (E). Bloo cultures are not ini- of alcohol, an reaspiration (PAIR) is optimally suite
cate in the workup for amebic liver abscess (A). for patients with single cysts but has been use in poly-
References: Blessmann J, Binh HD, Hung DM, Tannich E, cystic liver patients with a ominant cyst. Formal lobec-
Burchar G. Treatment of amoebic liver abscess with metroniazole tomy is another option. When all other options have been
alone or in combination with ultrasoun-guie neele aspiration:
exhauste, liver transplantation has been successful. To
a comparative, prospective an ranomize stuy: treatment of
ate, there is no successful meical management. However,
amoebic liver abscess. Trop Med Int Health. 003;8(11):1030–1034.
McGarr PL, Maiba TE, Thomson SR. Amoebic liver patients are instructe to avoi factors that have been asso-
abscess-results of a conservative management policy. S Afr Med J. ciate with increase cyst growth. Hormone replacement
003;93():13–136. therapy with estrogens in particular has been linke to cyst
growth an shoul therefore be avoie (D). Recently, oct-
reotie has shown some preliminary promise in retaring
6. A. Amebic liver abscesses respon very well to oral
cyst growth. Liver function tests are typically normal but
metroniazole. Several stuies have investigate whether
can be elevate if there is gross isplacement of liver paren-
percutaneous rainage is neee. Given the rapi response
chyma by massive liver cysts (E).
to oral metroniazole, aspiration or catheter-irecte rain-
References: Abu-Wasel B, Walsh C, Keough V, Molinari M. Patho-
age is unnecessary in the majority of cases (B–E). Aspiration
physiology, epiemiology, classication an treatment options for
is only inicate if the iagnosis of amebic liver abscess is polycystic liver iseases. World J Gastroenterol. 013;19(35):5775–5786.
uncertain or if the patient oes not respon appropriately to Que F, Nagorney DM, Gross JB Jr, Torres VE. Liver resection an
antibiotics within a few ays. Metroniazole is aministere cyst fenestration in the treatment of severe polycystic liver isease.
for 7 to 10 ays. Gastroenterology. 1995;108():487–494.
References: Akgun Y, Tacyiliz IH, Celik Y. Amebic liver abscess: Sherstha R, McKinley C, Russ P, et al. Postmenopausal estro-
changing trens over 0 years. World J Surg. 1999;3(1):10–106. gen therapy selectively stimulates hepatic enlargement in women
Blessmann J, Binh HD, Hung DM, Tannich E, Burchar G. with autosomal ominant polycystic kiney isease. Hepatology.
Treatment of amoebic liver abscess with metroniazole alone or in 1997;6(5):18–186.
42 PArt i Patient Care

9. D. Because of the high risk associate with esophageal 11. E. The portal vein has no valves (A). It supplies approx-
varices, numerous stuies have been unertaken to try to imately 75% of the bloo ow to the liver compare with
prevent rst-time blees. The objective is to reuce portal 5% by the hepatic arteries (B). It is forme by the conuence
venous pressure to less than 1 mm Hg without aing mor- of the superior mesenteric an splenic veins (D). The normal
biity. Prophylaxis is important because the 1-year mortal- pressure in the portal vein is 3 to 5 mm Hg (C). The portal
ity rate is as high as 70% in cirrhotic patients. Prophylactic vein is most commonly locate posterior (Portal is Posterior)
sclerotherapy, TIPS placement, an portosystemic shunting to the common bile uct an hepatic artery in the hepatou-
have not been shown to be effective (C). Conversely, both oenal ligament.
prophylactic β-arenergic blockae an enoscopic ligation
have been shown to be effective. Two large, ranomize 12. E. FNH is usually an inciental ning on a CT scan
stuies emonstrate that enoscopic ligation is even more because most patients are asymptomatic (A), an it is not
effective than beta-blockae in blee prevention (A). The associate with a risk of rupture or subsequent malig-
former may be more appropriate in cases of meium to large nancy (D). A hallmark feature of FNH is the presence of a
esophageal varices. The combination of beta-blockae an hypoense central stellate scar on CT or magnetic resonance
enoscopic ligation is not recommene as it can increase imaging (MRI) that enhances with contrast. MRI is the stuy
the risk for averse effects without an ae benet. In of choice to conrm FNH an is often the test of choice to
patients who are caniates for liver transplantation an characterize liver lesions (C). FNH is usually locate on the
have esophageal bleeing that is not controlle by meical periphery of the liver (B). It may on occasion be ifcult to
management, TIPS is the best brige while awaiting trans- istinguish from hepatic aenoma or brolamellar hepa-
plantation. TIPS can also be use as part of the acute man- tocellular carcinoma. An early embryologic isturbance in
agement in patients with refractory variceal bleeing (B). liver bloo ow is the postulate cause of FNH, which is
Selective portosystemic shunt is reserve for patients that supporte by the nings of regenerative noules. Resection
have faile all other management options because this car- is inicate when patients are symptomatic or if a enitive
ries a signicant mortality rate an risk of hepatic enceph- iagnosis cannot be mae.
alopathy (E). It is rarely performe toay an only in an References: Gangahar K, Deepa S, Chintapalli N. MRI evalua-
emergency setting. tion of masses in the noncirrhotic liver. Appl Radiol. 014;43(1):0–8.
References: Psilopoulos D, Galanis P, Goulas S, et al. Eno- Wanless IR, Mawsley C, Aams R. On the pathogenesis of focal
scopic variceal ligation vs. propranolol for prevention of rst var- noular hyperplasia of the liver. J Hepatol. 1985;5(6):1194–100.
iceal bleeing: a ranomize controlle trial. Eur J Gastroenterol
Hepatol. 005;17(10):1111–1117. 13. B. Screening for HCC is only of potential benet in
Sarin SK, Lamba GS, Kumar M, Misra A, Murthy NS. Compari- patients at high risk of eveloping HCC. The role an best
son of enoscopic ligation an propranolol for the primary preven- test for screening for HCC in high-risk patients remain con-
tion of variceal bleeing. N Engl J Med. 1999;340(13):988–993. troversial. Stuies in Asian patients with chronic viral hepa-
titis showe that a combination of ultrasonography an AFP
10. C. Bile salts are mae in the liver an then secrete is an effective screening tool. Recommenations are that AFP
to be use in the biliary tree an the intestine. Bile is com- alone shoul not be use an that ultrasonography seems
pose of bile acis, pigments, phospholipis, cholesterol, to be more efcient (A). The benets of screening high-risk
proteins, an electrolytes. Bile salts are important for small white patients are unclear, as is its cost-effectiveness. CT
intestinal absorption of fats an vitamins. Cholic aci an imaging can help establish the iagnosis of HCC by emon-
chenoeoxycholic aci are primary bile acis (A). They are strating a hyperintense lesion on arterial phase an rapi
mae in the liver from cholesterol an then conjugate with washout on venous phase (C). CEA can be use as a tool
glycine an taurine in the hepatocytes. The seconary bile to measure response to treatment in patients with colorec-
acis are eoxycholic an lithocholic acis an are forme tal cancer (D). Alkaline phosphatase levels are not typically
by intestinal bacterial moication of the primary bile acis use for the iagnosis of HCC (E).
(B). As a result of enterohepatic circulation, 95% of bile acis References: Daniele B, Bencivenga A, Megna AS, Tinessa V.
are returne to the liver via the portal circulation. They are Alpha-fetoprotein an ultrasonography screening for hepatocellular
reabsorbe passively in the jejunum an actively in the carcinoma. Gastroenterology. 004;17(5 Suppl 1):S108–S11.
ileum. Bile salts are important in the absorption of ietary Tong MJ, Blatt LM, Kao VW. Surveillance for hepatocellular car-
cinoma in patients with chronic viral hepatitis in the Unite States of
fats an fat-soluble vitamins. Major resection of the istal
America. J Gastroenterol Hepatol. 001;16(5):553–559.
ileum results in fat malabsorption an eciency in fat-sol-
uble vitamins because it impairs the circulation of bile acis.
14. A. The patient most likely has Bu-Chiari synrome,
It also lowers cholesterol levels because more cholesterol is
a rare isorer cause by thrombosis of the hepatic infe-
use to make new bile salts. After ingestion of foo, bile aci
rior vena cava or the hepatic veins themselves that leas
concentration in the liver ecreases an the inhibition of cho-
to hepatic venous outow obstruction, postsinusoial liver
lesterol 7-hyroxylase ecreases, resulting in an increase of
failure, an cirrhosis. The classic tria inclues abomi-
bile aci secretion in the liver (D, E).
nal pain, ascites, an hepatomegaly. There are four forms:
References: D’Angelica M, Fong Y. The liver. In: Townsen CM,
acute, chronic, asymptomatic, an fulminant. It is often
Jr, Beauchamp RD, Evers BM, Mattox KL, es. Sabiston textbook of
surgery: the biological basis of modern surgical practice. 17th e. Phila-
associate with a hypercoagulable state that is either inher-
elphia: W.B. Sauners; 004:1513–1574. ite (protein C, protein S, factor V Leien, or antithrombin
Sieelaff TD, Curley SA. Liver. In: Brunicari FC, Anersen DK, III eciency) or acquire (myeloproliferative isorers,
Billiar TR, et al., es. Schwartz’s principles of surgery. 8th e. New polycythemia vera, thrombocytosis, pregnancy). It is more
York: McGraw-Hill; 005:1139–1186. common in women. The iagnosis can be mae by uplex
CHAPtEr 4 Abdomen—Liver 43

ultrasonography, which will show the thrombose hepatic 16. D. Cystic hyati isease of the liver is ue to infection
veins or inferior vena cava. The most prominent feature on by the tapeworm Echinococcus granulosus. Another species,
a CT scan is cauate lobe (segment I) hypertrophy an inho- Echinococcus multilocularis, causes alveolar echinococco-
mogeneous contrast enhancement. The treatment epens sis. Humans (an sheep) are intermeiate hosts, whereas
on the acuity of the presentation. Immeiate treatment is ogs are the enitive host. Diagnosis is establishe by an
with anticoagulation followe by percutaneous angioplasty enzyme-linke immunosorbent assay test for Echinococcus
with or without stenting. There are rare reports of successful antigen couple with an ultrasoun or CT scan. Characteris-
thrombolysis. Subsequent treatment epens on whether the tic features have le to four types escribe (Gharbi types): a
primary inication for an intervention is portal hypertension simple cyst (type I), a cyst with free-oating hyperechogenic
(TIPS or nonselective shunt) or liver failure (transplantation). material calle hydatid sand (type II), a cyst with a rosette
The remaining answer choices o not present with the afore- appearance suggesting a aughter cyst (type III), an a cyst
mentione nings (B–E). with a iffuse hyperechoic soli pattern (type IV). Treatment
References: Kim TK, Chung JW, Han JK, Kim AY, Park JH, Choi options for hyati isease inclue oral anthelmintic agents
BI. Hepatic changes in benign obstruction of the hepatic inferior (albenazole, mebenazole), laparoscopic or open cyst exci-
vena cava: CT nings. AJR Am J Roentgenol. 1999;173(5):135–14. sion with omentoplasty (B), formal liver resection, total peri-
Slakey DP, Klein AS, Venbrux AC, Cameron JL. Bu-Chiari syn- cystectomy, an PAIR (E). Drug therapy alone is curative
rome: current management options. Ann Surg. 001;33(4):5–57.
in only a small percentage of patients (A, C). The treatment
Wu T, Wang L, Xiao Q, et al. Percutaneous balloon angioplasty
of choice is a surgical total pericystectomy with pre- an
of inferior vena cava in Bu-Chiari synrome-R1. Int J Cardiol.
00;83():175–178. postoperative albenazole. This has been emonstrate to
have the lowest rates of recurrence, morbiity, an mortal-
ity. During aspiration or surgical treatment of hyati cysts,
15. D. Distinguishing between FNH an a hepatic ae- extreme caution must be taken to avoi rupture of the cyst.
noma is important because the management of the former Cyst rupture can result in release of protoscolices into the
is observation, whereas the treatment of hepatic aenomas peritoneal cavity an can lea to anaphylaxis.
often requires surgical resection because of their known risk References: Etlik O, Arslan H, Bay A, et al. Abominal hyati
of malignant egeneration an risk of hemorrhage an spon- isease: long-term results of percutaneous treatment. Acta Radiol.
taneous rupture. In a recent stuy, 70% of hepatic aenomas 004;45(4):383–389.
were symptomatic (abominal pain), 9% of resecte hepatic Georgiou GK, Lianos GD, Lazaros A, et al. Surgical management
aenomas ha evience of hemorrhage, an 5% ha malig- of hyati liver isease. Int J Surg. 015;0:118–1.
nancy present. Hepatic aenomas present in young women Kabaalioğlu A, Ceken K, Alimoglu E, Apayin A. Percutane-
ous imaging-guie treatment of hyati liver cysts: o long-term
in association with oral contraceptive use. Though rare in
results make it a rst choice? Eur J Radiol. 006;59(1):65–73.
men, they are associate with anabolic steroi use an gly-
Khuroo MS, Wani NA, Javi G, et al. Percutaneous rainage
cogen storage iseases (A). Most authors recommen a selec- compare with surgery for hepatic hyati cysts. N Engl J Med.
tive approach to the resection of hepatic aenomas (only 1997;337(13):881–887.
resect if symptomatic, >5 cm, or those that continue growing
espite cessation of oral contraceptive use on repeat imag- 17. D. Patients with cirrhosis are at increase risk for
ing), as rupture an malignant transformation risks are rare umbilical herniation ue to the increase intraabominal
for those <5 cm. Resection is recommene in men regar- pressure. The overlying skin can thin an eventually rup-
less of size. Differentiating FNH an hepatic aenoma is not ture, which is associate with high mortality. Chil-Pugh
always straightforwar. Both may show contrast enhance- A cirrhotics can procee with elective surgery after mei-
ment in the arterial phase of a CT scan, so this oes not help cal optimization. Chil-Pugh B cirrhotics have an increase
to ifferentiate them (C). FNH characteristically emon- risk uring surgery, an the ecision to operate shoul be
strates a central scar. Aenomas may emonstrate increase iniviualize. Chil-Pugh C is an absolute contrainica-
fat signal on MRI compare with FNH. When CT an MRI tion for elective surgery. Given that the patient above has
are unable to istinguish aenoma from FNH, a sulfur col- poorly controlle ascites, he is a Chil-Pugh B. Before surgi-
loi scan may be benecial because aenomas will appear cal intervention in this patient, meical therapy nees to be
“col” an FNHs “hot” because of the presence of Kupffer initiate (C). Fixing the umbilical hernia without aress-
cells (B–E). Raiofrequency ablation is another potential ing the unerlying ascites will increase the failure rate of
option in managing hepatic aenomas, especially when mul- the hernia repair. The initial treatment of ascites in a patient
tiple aenomas are present, or the patient is not a caniate with cirrhosis inclues a low-soium iet an the use of
for a major liver resection. the iuretics spironolactone an furosemie. In the major-
References: Cho SW, Marsh JW, Steel J, et al. Surgical manage- ity of patients, this approach is successful. If the ascites is
ment of hepatocellular aenoma: take it or leave it? Ann Surg Oncol. refractory to this management, the next step is large-volume
008;15(10):795–803. (4–6 L) paracentesis. The paracentesis shoul be followe
Daniele B, Bencivenga A, Megna AS, Tinessa V. Alpha-fetopro- by an IV infusion of 5% salt-poor albumin (B). If the asci-
tein an ultrasonography screening for hepatocellular carcinoma.
tes is still not responsive, serial large-volume paracentesis
Gastroenterology. 004;17(5 Suppl 1):S108–S11.
Herman P, Pugliese V, Machao MA, et al. Hepatic aenoma
can be use. TIPS is another option but shoul be reserve
an focal noular hyperplasia: ifferential iagnosis an treatment. for patients with reasonably goo liver function because
World J Surg. 000;4(3):37–376. those with avance liver isease will have a high risk of
Toso C, Majno P, Anres A, et al. Management of hepatocellular the evelopment of encephalopathy an hepatic ecompen-
aenoma: solitary-uncomplicate, multiple an rupture tumors. sation (A). In the latter patient, the ieal option woul be
World J Gastroenterol. 005;11(36):5691–5695. a liver transplantation. Peritoneovenous shunting is now
44 PArt i Patient Care

rarely use because it has a high rate of shunt clotting an Kakar S, Burgart LJ, Batts KP, Garcia J, Jain D, Ferrell LD. Clin-
can inuce isseminate intravascular coagulation. Obser- icopathologic features an survival in brolamellar carcinoma:
vation woul not be appropriate for a patient presenting comparison with conventional hepatocellular carcinoma with an
with worsening ascites (E). without cirrhosis. Mod Pathol. 005;18(11):1417–143.
Reference: Chouhury J, Sanyal AJ. Treatment of ascites. Curr
Treat Options Gastroenterol. 003;6(6):481–491. 21. D. Both hepatitis B an C virus infections are factors for
the evelopment of HCC, whereas hepatitis A is not. Cirrho-
18. B. The classic tria associate with pyogenic liver sis is not require for the evelopment of HCC, an HCC is
abscess is the same as Charcot tria for cholangitis. It consists not an inevitable result of cirrhosis. Chronic alcohol abuse
of right upper quarant pain, fever, an jaunice, although an smoking are also associate with an increase risk of
only 10% of patients have all three features. Pyogenic liver HCC (C). Aatoxin is linke to HCC (A). It is prouce by
abscess remains a highly lethal isease, with mortality rates, Aspergillus species an can be foun on contaminate pea-
even in more recent large series, ranging from 10% to 0%. nuts an other grains. Other hepatic carcinogens inclue
The most common etiology of pyogenic liver abscesses is the nitrites, hyrocarbons, solvents, pesticies, vinyl chlorie,
biliary tract. It is more likely to be associate with abnormal an Thorotrast (a contrast agent no longer use) (B, E).
liver function tests compare with other infectious hepatic HCC has also been linke to metabolic liver iseases such
etiologies (e.g., amebic abscess, hyati cyst) ue to its prox- as hereitary hemochromatosis. Wilson isease an primary
imity to the biliary tree. In most instances, management con- biliary cirrhosis have not been consistently emonstrate to
sists of IV antibiotics with percutaneous aspiration of the increase the risk of hepatocellular carcinoma.
abscess with or without catheter rainage. Other etiologies Reference: van Meer S, e Man RA, van en Berg AP, et al. No
inclue seeing of the portal vein from iverticular isease, increase risk of hepatocellular carcinoma in cirrhosis ue to Wilson
appenicitis (D), inammatory bowel isease (E), an sys- isease uring long-term follow-up: liver cancer in Wilson isease.
temic infections such as bacterial enocaritis (C). Amebic J Gastroenterol Hepatol. 015;30(3):535–539.
liver abscesses more commonly involve seeing from the
portal vein (A). However, in a high percentage of pyogenic 22. E. The MELD score is use to prioritize patients awaiting
liver abscesses, the source is unclear. liver transplantation an inclues the serum total bilirubin
Reference: Chu KM, Fan ST, Lai EC, Lo CM, Wong J. Pyogenic an serum creatinine levels an the international normalize
liver abscess. An auit of experience over the past ecae. Arch Surg. ratio (INR). The presence of encephalopathy or ascites oes
1996;131():148–15. not factor into this score (A, B). MELD was originally esigne
to preict mortality after a TIPS proceure. The score ranges
19. B. The Ito cells are also known as the hepatic stellate from 6 to 40. It has since been moie to a the serum
cells. They are locate in the space of Disse an are character-
soium level because low serum soium (<16 mEq/L) has
ize by the presence of lipi roplets because they store vita-
been shown to be an inepenent risk of mortality in liver
min A. Ito cells play an important role in the liver’s response
transplant recipients. The newly moie MELD score, in
to acute liver injury as well as in chronic liver injury. In these
combination with American Society of Anesthesiologists class
settings, the Ito cell ifferentiates into a myobroblast-like
an patient age, has been shown to be preictive of perioper-
cell that has a high capacity for brogenesis. The remain-
ative mortality in patients with cirrhosis unergoing a wie
ing answer choices o not play a role in meiating brosis
variety of surgical proceures. The MELD score removes
(A,D,E).
the subjectivity associate with other classication systems.
Reference: Hautekeete ML, Geerts A. The hepatic stellate (Ito)
In patients with en-stage liver isease awaiting transplan-
cell: its role in human liver isease. Virchows Arch. 1997;430(3):195–07.
tation, the 3-month mortality rate was 1.9% for those with a
20. D. FLC has been consiere to be a variant of HCC, but MELD score less than 9, whereas patients with a MELD score
recent stuies suggest that it is a istinct pathologic entity. of 40 or more ha a mortality rate of 71.3%. A MELD score >15
FLC generally occurs in younger patients (meian age 5 is require to be enliste on the liver transplant list. Chil-
years) an HCC in oler patients (meian age 55 years) (B). Pugh grae (base on bilirubin, albumin, INR, presence of
Unlike HCC, the majority of patients with FLC o not have ascites or encephalopathy) is another scoring system that can
cirrhosis, are not hepatitis-B positive, an o not have an ele- be use to measure hepatic reserve after hepatic resection (D).
vate AFP level (A–C). The tumor is usually well emarcate For each of the ve criteria, a point (1–3) is assigne. Chil-
an may have a central brotic area. This can make it har to Pugh A inclues 5 to 6 points (no mortality risk at 1 year),
istinguish from FNH. In the arterial phase of a CT scan, the Chil-Pugh B inclues 7 to 9 points (0% 1-year mortality
central scar in FNH enhances because it actually represents rate), an Chil-Pugh C inclues 10 to 15 points (55% 1-year
a vascular entity, whereas the central scar in FLC oes not mortality rate). INR an total bilirubin are the two variables
enhance. Likewise, the central scar in FNH is hyperintense the MELD an Chil-Pugh score share in common (C).
on gaolinium MRI. The prognosis overall tens to be better Reference: Wiesner R, Ewars E, Freeman R, et al. Moel
for en-stage liver isease (MELD) an allocation of onor livers.
than that of HCC, mostly because of the absence of cirrhosis,
Gastroenterology. 003;14(1):91–96.
but it still only carries a 5-year survival rate of 45% (E). It
is associate with elevate neurotensin levels. Treatment is
surgical resection. 23. D. The right hepatic vein rains segments V, VI, VII an
References: Ichikawa T, Feerle MP, Grazioli L, Maariaga VIII an enters the vena cava. The cauate lobe, situate in
J, Nalesnik M, Marsh W. Fibrolamellar hepatocellular carcinoma: the posterior right lobe, also rains irectly into the inferior
imaging an pathologic nings in 31 recent cases. Radiology. vena cava (B). The mile hepatic vein rains segments IVA,
1999;13():35–361. IVB, V, an VIII. The mile hepatic vein enters the inferior
CHAPtEr 4 Abdomen—Liver 45

vena cava jointly with the left hepatic vein via a common Reference: Seo T, Ano H, Watanabe Y, et al. Treatment of
orice (A). The left hepatic vein rains segments II an III. hepatoblastoma: less extensive hepatectomy after effective pre-
The roun ligament is a remnant of the umbilical vein an operative chemotherapy with cisplatin an Ariamycin. Surgery.
marks the location of the intrahepatic location of the left 1998;13(4):407–414.
portal vein. The ligamentum venosum is a remnant of the
uctus venosus an marks the borer between the cauate 26. B. In general, the Chil-Pugh scoring system is useful
lobe an the left lateral sector (C). In most instances, the com- in preicting hepatic reserve after hepatic resection. How-
mon hepatic artery gives rise to the gastrouoenal artery ever, it loses its preictive value in Chil-Pugh A patients.
an right gastric artery, after which the name changes to the The inocyanine green clearance test is a stuy for mea-
proper hepatic artery (E). The proper hepatic artery becomes suring hepatic reserve before hepatic resection in combina-
the right an left hepatic arteries. A replace right hepatic tion with the Chil-Pugh score. Inocyanine green bins to
artery arises from the superior mesenteric artery (most com- albumin an α1-lipoproteins in liver parenchymal cells an
monly) an is posterolateral to the portal vein. It is referre thus rapily clears from the plasma. It is then secrete in the
to as a replace artery because it replaces the right hepatic bile. Hepatic reserve is measure by the amount of inocy-
artery coming off the proper hepatic artery. This is in contrast anine green retaine in the plasma after 15 minutes. If more
to an accessory right hepatic artery, which also comes off the than 15% remains in the plasma at 15 minutes, this is con-
superior mesenteric artery (most commonly) but is in addi- siere abnormal (retention rate 15% = clearance rate 85%).
tion to the right hepatic artery coming off the proper hepatic The remaining choices are less effective stuies to assess for
artery. A replace left hepatic artery most commonly arises hepatic reserve (A, C–E).
from the left gastric artery (branch of the celiac axis). Reference: Schneier PD. Preoperative assessment of liver func-
tion. Surg Clin North Am. 004;84():355–373.
24. A. Several stuies have analyze preictors of poor
27. A. Bu-Chiari synrome is ue to thrombosis of the
long-term outcome after resection of hepatic metastasis from
hepatic veins or intrahepatic vena cava. It is often ue to an
colorectal cancer. In one stuy, the factors were positive
unerlying hypercoagulable state. It leas to postsinusoial
tumor margin, presence of extrahepatic isease, noe-pos-
portal hypertension because it is cause by hepatic venous
itive primary tumor, isease-free interval from primary
outow congestion (C). In contrast, presinusoial portal
tumor to metastases less than 1 months (C), more than one
hypertension evelops seconary to congestion within the
hepatic tumor, the largest hepatic tumor being larger than 5
intrahepatic portal system. Liver function is oftentimes nor-
cm, an a CEA level greater than 00 ng/mL. Using the last 5
mal in presinusoial portal hypertension while it is elevate
factors, the authors recommene against hepatic resection
in postsinusoial portal hypertension (E). Diagnosis is mae
for those with 3 or more points because the long-term out-
by CT scan an uplex ultrasoun scan of the hepatic veins
come was poor. In another large stuy, the factors for averse
(B). Initial management is with heparinization followe by
outcome were similar an inclue the number of hepatic
percutaneous angioplasty with or without stenting. Rare
metastases greater than three noe-positive primary tumor
reports exist of successful thrombolysis. TIPS has also been
(B), poorly ifferentiate primary tumor, extrahepatic is-
use successfully (D). Those with ecompensate liver func-
ease (D), tumor iameter 5 cm or larger, CEA level greater
tion may require liver transplantation.
than 60 ng/mL (E), an positive resection margin.
Reference: Slakey DP, Klein AS, Venbrux AC, Cameron JL.
References: Fong Y, Fortner J, Sun RL, Brennan MF, Blumgart Bu-Chiari synrome: current management options. Ann Surg.
LH. Clinical score for preicting recurrence after hepatic resection
001;33(4):5–57.
for metastatic colorectal cancer: analysis of 1001 consecutive cases.
Ann Surg. 1999;30(3):309–318.
Rees M, Tekkis PP, Welsh FKS, O’Rourke T, John TG. Evalu- 28. D. Cerebral eema an intracranial hypertension
ation of long-term survival after hepatic resection for metastatic (ICH) are the complications of fulminant hepatic failure
colorectal cancer: a multifactorial moel of 99 patients. Ann Surg. most likely to result in averse outcome an eath (A–C,
008;47(1):15–135. E). Liver failure is accompanie by high levels of ammonia,
which can be etoxie in astrocytes leaing to an accu-
25. E. Hepatoblastoma is the most common primary liver mulation of astrocyte glutamine. This is associate with
malignancy in chilren. It has been associate with familial increase intracellular osmolality an can lea to cerebral
polyposis synrome. It presents typically with an asymp- eema an eventually ICH. Thus, it is essential to monitor
tomatic abominal mass, anemia, thrombocytosis, an ICH as hepatic coma evelops with intracranial pressure
elevate AFP levels. Patients may also rst present with monitoring. This technology has been shown to be critical
precocious puberty seconary to increase beta–human cho- to the ongoing etermination of a patient’s caniacy for
rionic gonaotropin (β-hCG). Fetal histology has the best liver transplantation. Patients whose intracranial pressure
prognosis. Treatment is with chemotherapy rst an then increases to more than 0 mm Hg or whose cerebral per-
resection. Chemotherapy enables the subsequent hepatic fusion pressure ecreases to less than 60 mm Hg will have
resection to be less an may make tumors resectable that ini- a high risk of irreversible brain injury. If the intracranial
tially appear to be unresectable. FLC (B) has been consiere pressure is more than 50 mm Hg or the cerebral perfusion
to be a variant of HCC (A), but recent stuies suggest that it pressure is less than 40 mm Hg, transplantation is contra-
is a istinct pathologic entity. Focal bile uct stenosis in oler inicate. Coagulopathy in this patient population is not
male patients without any biliary instrumentation is highly consiere an absolute contrainication to invasive intra-
suggestive of intrauctal cholangiocarcinoma (C). Giant cell cranial pressure monitoring.
(osteoclast-like) carcinoma of the liver is rare (D) but is more Reference: Sass DA, Shakil A. Fulminant hepatic failure. Liver
commonly seen in bone tumors. Transpl. 005;11(6):594–605.
46 PArt i Patient Care

29. D. The patient has FNH. In contrast to hepatic aeno- Absolute contrainications to TIPS placement are polycystic
mas, FNH typically is not associate with symptoms an liver isease an right heart failure.
oes not pose any risks of rupture or malignant egeneration References: Colombato L. The role of TIPS in the management
(B, C). These lesions intensely enhance in the arterial vascular of portal hypertension. J Clin Gastroenterol. 007;41:S344–S351.
phase of axial imaging stuies. Characteristically, as many Testino G, Ferro C, Sumberaz A, et al. Type- hepatorenal syn-
as two-thirs of lesions will emonstrate a central scar that rome an refractory ascites: role of transjugular intrahepatic por-
tosystemic stent-shunt in eighteen patients with avance cirrhosis
enhances in the arterial phase (versus FLC, which remains
awaiting orthotopic liver transplantation. Hepatogastroenterology.
hypoense). The lesions are often peripherally locate (A).
003;50(54):1753–1755.
On a technetium-99m–macroaggregate albumin liver scan,
FNH appears “hot” because of the presence of Kupffer cells,
33. B. The MRI nings are characteristic of a hemangioma,
which take up sulfur colloi (E). The etiology is thought to
given the peripheral noular enhancement an the brightness
be the result of an early embryologic vascular injury. FNH
on T-weighte images. They have low-signal intensity on
is rarely symptomatic. In patients with symptoms relate
T1-weighte imaging. Hemangiomas are common benign liver
to FNH, resection is inicate. Because the lesions are often
lesions generally iscovere incientally on imaging stuies.
peripheral, minimally invasive (laparoscopic) approaches to
They may on occasion be ifcult to istinguish from other
resection shoul be avocate. Resection of the lesion with a
lesions. MRI nings ten to be more specic than CT scan
thin margin of normal liver parenchyma is curative, but for-
for hemangiomas. Rarely, hemangiomas are ifcult to iffer-
mal segmental resection shoul be consiere because such
entiate on MRI or CT scan. Hemangiomas can be enitively
proceures are associate with lower morbiity.
iagnose by a technetium-99–labele re cell scan with sin-
gle-photon emission CT. Diagnostic nings inclue ecrease
30. E. The male-to-female ratio for amebic liver abscesses is activity on early images an subsequent elaye lling from
approximately 10:1 versus 1.5:1 for pyogenic abscesses (A). the periphery. CT criteria that are specic for hemangioma
Three-fourths of liver abscesses involve the right lobe of the liver inclue iminishe attenuation on precontrast scan, peripheral
(C). Pyogenic abscesses are more likely to be multiple. Amebic contrast enhancement uring the ynamic bolus phase of scan-
abscesses ten to occur in younger patients an in enemic ning, an complete isoense ll-in on elaye imaging. Given
areas. Heavy alcohol consumption is commonly reporte for the vascular nature of hemangiomas, neele biopsy is contra-
amebic infection an is also a risk factor for pyogenic abscesses. inicate (A). Resection is also unnecessary (D, E). Hemangio-
The majority of amebic abscesses are manage with antibiotics mas are not associate with oral contraceptive use (C).
alone, whereas pyogenic abscesses often require aspiration or References: Freeny PC, Marks WM. Hepatic hemangioma:
catheter-base rainage (D). The mortality for patients with ynamic bolus CT. AJR Am J Roentgenol. 1986;147(4):711–719.
amebic liver abscesses is % to 4%; however, the mortality for Reimer P, Rummeny EJ, Dalrup HE, et al. Enhancement charac-
patients with pyogenic abscesses ranges from 10% to 0% (B). teristics of liver metastases, hepatocellular carcinomas, an heman-
giomas with G-EOB-DTPA: preliminary results with ynamic MR
31. A. The presence of a central stellate scar is consiere imaging. Eur Radiol. 1997;7():75–80.
iagnostic of FNH when the scar enhances in the arterial
phase. FNH is thought to be the result of a response to an 34. C. Portal hypertension is classie into three types:
in utero isturbance in liver bloo supply with a subsequent presinusoial, sinusoial, an postsinusoial. Distinguish-
liver regeneration. There oes not seem to be any link to oral ing between these causes is important because treatment may
contraceptive use an no risk of rupture or malignancy, so the iffer accoringly. Also, unlike the sinusoial an postsinu-
management is observation (B). The size of the FNH lesion soial types, presinusoial portal hypertension is more likely
oes not seem to be inuence by oral contraceptive use. to be associate with a preserve liver function. Presinusoi-
The only inications for surgery woul be if the iagnosis al hypertension is further ivie into intrahepatic an
cannot be mae preoperatively (particularly to istinguish extrahepatic causes. Extrahepatic causes inclue portal an
FNH from FLC) with certainty or if the patient has symptoms splenic vein thromboses (E). The most common intrahepatic
(although the presence of symptoms suggests another pathol- etiology is schistosomiasis (Schistosoma japonicum an Schis-
ogy) (C–E). Change in the size of FNH on follow-up is rare. tosoma mansoni). The infection by a uke leas to brosis an
Reference: Mathieu D, Kobeiter H, Maison P, et al. Oral contra- granulomatous reactions. In chilren, congenital hepatic
ceptive use an focal noular hyperplasia of the liver. Gastroenterol- brosis is another cause. Sinusoial causes inclue alcohol-
ogy. 000;118(3):560–564. ism an other causes of cirrhosis (A). Other etiologies inclue
hemochromatosis an Wilson isease (D). Postsinusoial
32. B. TIPS has been shown to be useful in patients who portal hypertension inclues Bu-Chiari synrome an
o not respon to meical management of variceal blee- congenital webs in the intrahepatic inferior vena cava (B).
ing. It is consiere to be a nonselective shunt an is highly
effective in the short term in preventing rebleeing (C, D). 35. B. Bile uct hamartomas are the most common lesions
However, because it is nonselective, it has a signicant risk of the liver seen uring laparotomy. They are often small (1–5
of encephalopathy. Thus, it shoul be use with caution in mm), rm, smooth, an white an occur in the periphery of
patients who alreay have marginal hepatic reserve. TIPS the liver. It is important to ifferentiate these from metastatic
is also useful in patients with refractory ascites (A). Recent lesions by taking intraoperative biopsies an sening them
stuies suggest that it is also useful as a brige to liver trans- as frozen specimens (C). If it is foun to be a metastatic lesion,
plantation in patients with hepatorenal synrome. It is not a the proceure shoul be aborte (A). Bile uct hamartomas
goo alternative to long-term portal ecompression because o not typically istort hepatic parenchyma an o not lea
the 1-year patency rate is only approximately 50% (E). to elevate LFTs. They o not nee to be resecte (E).
Abdomen—Pancreas
JOON Y. PARK AND DANIELLE M. HARI 5
ABSITE 99th Percentile High-Yields
I. Anatomic Variants of the Pancreas
a. Annular pancreas
i. Secon portion of uoenum entrappe in pancreatic hea from incomplete rotation of ventral
pancreatic bu, associate with Down synrome (trisomy 1)
ii. Symptoms: uoenal obstruction (nausea an nonbilious emesis)
iii. Treatment if symptomatic: nasogastric tube, uoenouoenostomy
b. Pancreatic ivisum
i. Failure of fusion of orsal (Santorini) an ventral (Wirsung) ucts so that the uct of Santorini rains
the majority of the pancreas via the minor papilla an the uct of Wirsung only rains the hea an
uncinate process via the major papilla
ii. Symptoms: most are asymptomatic, some have recurrent pancreatitis
iii. Treatment: enoscopic minor papilla sphincterotomy

II. Pancreatitis
a. Most common cause of acute pancreatitis: gallstone (# EtOH)
b. Most common cause of chronic pancreatitis: EtOH
c. First step in workup is serum amylase/lipase an abominal ultrasoun; pancreatitis is a clinical
iagnosis an oes not require CT imaging unless there is suspicion for a complicate pancreatitis (e.g.,
necrotizing pancreatitis, hemorrhagic, pseuocyst)
. Distinction between necrotizing pancreatitis an infecte necrotizing pancreatitis can be challenging to
make; patients with progression to infection will often have a worsening clinical course an CT nings
of extraluminal gas in the collection; FNA with culture shoul be performe to conrm the iagnosis but
is not require to begin treatment
e. Step-up approach to acute necrotizing pancreatitis (associate with improve mortality compare to
open necrosectomy):
i. NPO, ui resuscitation; IV broa-spectrum antibiotics not routinely aministere but shoul be
given to patients in septic shock
ii. Percutaneous versus enoscopic acute necrotic collection rainage
iii. If no improvement in 7 hours, procee to vieo-assiste retroperitoneal ebriement (VARD)
f. Peripancreatic ui collections

Interstitial pancreatitis Necrotizing pancreatitis


<4 weeks Acute peripancreatic Ěuid collection Acute necrotic collection
>4 weeks Pseudocyst Walled oě necrosis (WON)

47
48 PArt i Patient Care

i. Conservative management for asymptomatic an small (<6 cm) pseuocysts for at least 6-weeks (most <6
cm resolve spontaneously, nee mature wall for intervention)
ii. Intervention inications: symptomatic, persistent an/or enlarging; manage with enoscopic
cystogastrostomy for most
g. Chronic pancreatitis surgical inications: intractable pain following maximal meical management
i. Puestow: lateral pancreaticojejunostomy; ecompressive proceure for obstruction; ieal for ilate
main uct (≥6 mm) WITHOUT pancreatic hea enlargement
ii. Frey: lateral pancreatojejunostomy withuoenum-preserving coring outofthe pancreatic hea
without ivisionof the pancreas; ieal for ilate main uct (≥6mm) AND enlarge pancreatic hea
iii. Beger: uoenum-sparing resection of most of the pancreatic hea with ivision of the
pancreatic boy over the portal vein an reconstruction via a sie-to-sie an sie-to-en
pancreaticojejunostomy to rain the remaining hea an tail of the pancreas; ieal for enlarge
pancreatic hea with normal size main uct (<6 mm)

III. Pancreatic Cystic Lesions


a. Best iagnostic test: MRCP; then conrm iagnosis with enoscopic ultrasoun (EUS) an ne-neele
aspiration (FNA)

Diagnosis Notes Pathology Management


Mucinous cystic • Premalignant, 15% chance of Cyst walls with Resection
neoplasm malignancy “ovarian-type”
• MUCH more common in women stroma
(>95% women), age 40s–50s Fluid:
• Usually in the body or head of the • Mucinous
pancreas • Low amylase
• High CEA
Intraductal • Can be malignant (>60% risk of Fluid: Indications for resection:
papillary malignancy if any main duct • Mucinous • Any main duct component
mucinous component) • High amylase (main-duct or mixed)
neoplasm • Nearly equal in men & women, • High CEA • Branch-duct IPMN if ANY of the
(IPMN) age 60s–70s following:
• Imaging: Diěuse or segmental ◦ ≥3 cm
pancreatic duct enlargement ◦ Symptomatic (e.g., jaundice or
pancreatitis)
◦ Enhancing mural nodule
◦ Main duct dilation >1 cm
◦ Intraductal mucin
Serous cystic • No malignant potential Uniform, cuboidal, Resect if symptomatic, otherwise no
neoplasm • Most commonly in the tail glycogen-rich treatment indicated as there is no
• Most have symptoms (epigastric cells malignant potential
pain, nausea, vomiting) Fluid:
• MUCH more common in women • Serous
(90% women), age 70s • Low amylase
• Imaging: honeycomb-like lesion • Low CEA
with central scar and calcięcation
Pseudocyst • Suspect if recent episode of Fluid: Surgery indicated if at least 6 weeks
pancreatitis • Serous after episode of pancreatitis AND
• High amylase symptomatic OR >6 cm
• Low CEA Most managed with endoscopic
cystogastrostomy

IV. Pancreatic Neuroenocrine Tumors (PNETs)


a. Types:
i. Nonfunctional PNET: most common; usually present with large tumor because patients won’t have
symptoms until mass is large enough to cause mass effect
ii. Insulinoma: most common functional PNET; low malignant potential (90% benign); most are sporaic,
5% associate with MEN1; presents with Whipple tria (hypoglycemia, symptoms of hypoglycemia,
symptoms resolve with eating); iagnosis supporte by the following labs (measure while
hypoglycemic uring a 7 hour fast)—insulin >10 mcU/mL with elevate C-peptie (≥.5 ng/mL),
CHAPtEr 5 Abdomen—Pancreas 49

fasting insulin to glucose ratio >0.4, no sulfonylurea or meglitinie etecte; less likely to be etecte
with octreotie scan; if can’t localize with CT, can try 18-F-DOPA PET scan
1. Management:
b. Manage symptoms with small, frequent meals
c. If < cm AND > mm from the pancreatic uct: enucleate
. If > cm OR < mm from the pancreatic uct: formal resection
e. If not a surgical caniate: give iazoxie (inhibits release of insulin from beta islet cells)
iii. Gastrinoma: most common PNET in patients with MEN1 (0% associate with MEN1, 80% sporaic);
secon most common sporaic functional PNET; presents with refractory peptic ulcer isease (PUD),
iarrhea; most have low-grae malignant behavior; when iagnose, nee to test calcium, PTH,
prolactin (because associate with MEN1)
1. Diagnosis: conrme if all 3 of the following are true: PUD, serum gastrin >1000 (while off PPI
×7 hours at least), gastric pH <; if iagnosis is unclear, o secretin stimulation test: measure
baseline gastrin, give  U/kg IV secretin, measure gastrin levels Q5min for 30min; positive for
gastrinoma if gastrin increases by ≥00
. Management:
a. If < cm AND > mm from the pancreatic uct: enucleate
b. If > cm OR < mm from the pancreatic uct: formal resection
iv. Glucagonoma: from alpha islet cells; presents with 4 Ds: ermatitis (necrolytic migratory erythema),
iabetes, eep vein thrombosis, epression; most are malignant (75%); serum glucagon >1000;
most in istal pancreas so present late without obstructive jaunice, management: formal resection
(inclue splenectomy if oing istal pancreatectomy because of malignancy risk)
v. VIpoma: presents with high-volume watery iarrhea, ehyration, muscle cramping, cutaneous
ushing; most are malignant; labs that support the iagnosis: high VIP levels, hypokalemia,
achlorhyria, metabolic aciosis, hypercalcemia, hyperglycemia; management: formal resection
(inclue splenectomy if oing istal pancreatectomy)
vi. Somatostatinoma: from elta cells; mostly malignant (90%); can be associate with neurobromatosis
1; presents with steatorrhea, iabetes, gallstones, hypochlorhyria; iagnose with high fasting
somatostatin levels; management: formal resection

V. Pancreatic aenocarcinoma
a. Risk factors: increase age, smoking, obesity, new-onset iabetes in elerly
b. Workup an staging: CT pancreas protocol, CA19-9, CT chest/abomen/pelvis
c. Consier iagnostic laparoscopy to assess for M1 isease prior to resection or neoajuvant therapy
. Pancreatic aenocarcinomas with istant metastases (M1) are consiere unresectable an treate with
systemic therapy; those not associate with istant metastases are further classie below:

Classięcation Arterial Contact Venous Contact Initial Management


Resectable None SMV/PV: ≤180° AND no contour Surgery*
irregularity
IVC: no contact
Borderline Resectable SMA/Celiac: ≤180° SMV/PV: >180° and/or contour Neoadjuvant therapy, restage,
irregularity, appears reconstructable surgery if appropriate
IVC: contact
Locally Advanced SMA/Celiac: >180° Unreconstructable SMV/PV Neoadjuvant therapy, restage,
(encasement) surgery if appropriate

SMV = superior mesenteric vein, PV = portal vein, IVC = inferior vena cava, SMA = superior mesenteric artery.
*Can also consider neoadjuvant therapy especially for high-risk masses, but most common ABSITE answer is still surgery for resectable
disease.

AL GRAWANY
50 PArt i Patient Care

e. If patient has biliary obstruction on presentation an cannot procee irectly to surgery, can have
ERCP an stent placement; stent associate with increase risk of perioperative infection; shoul
obtain a new CA19-9 level after biliary ecompression
f. Generally, if a patient has a symptomatic pancreatic hea mass, you can procee with Whipple without a
biopsy
g. If planning on neoajuvant therapy, neeEUS-guie biopsy prior to treatment; after completion of
neoajuvant therapy, restage with CT an CA19-9 an resect if appropriate
h. All pancreatic aenocarcinoma gets ajuvant therapy
i. Benets of chemotherapy versus chemoraiation are not clear; either are appropriate

Fig. 5.1 Geography of Pancreatic Neuroenocrine Tumors.


CHAPtEr 5 Abdomen—Pancreas 51

Fig. 5.2 The Puestow Proceure.

Fig. 5.3 The Beger Proceure.


52 PArt i Patient Care

Fig. 5.4 The Frey Proceure.


CHAPtEr 5 Abdomen—Pancreas 53

Questions
1. A 50-year-ol man with a history of Roux-en-Y 4. Which of the following is true regaring
gastric bypass presents with epigastric pain an the role of enoscopic retrograe
fullness two months after an episoe of acute cholangiopancreatography (ERCP) an/or timing
pancreatitis. CT scan reveals an 8-cm pancreatic of surgery for acute biliary pancreatitis?
pseuocyst that abuts the gastric funus. What is A. In mil pancreatitis, laparoscopic
the most appropriate management? cholecystectomy can be safely performe
A. Enoscopic cystogastrostomy via the gastric within 48 hours of amission
remnant B. ERCP with sphincterotomy shoul be use
B. Percutaneous rainage routinely before surgery
C. Surgical cystogastrostomy via the gastric C. If the total bilirubin fails to normalize, ERCP
remnant with sphincterotomy shoul be performe
D. Surgical Roux-en-Y cyst-jejunostomy preoperatively
E. Repeat imaging in 4 weeks D. In severe pancreatitis, early cholecystectomy
reuces morbiity an mortality
2. A 55-year-ol woman with a history of coronary E. There is minimal risk of worsening the
artery isease is iagnose with a resectable 3-cm pancreatitis with the performance of ERCP
insulinoma in the tail of the pancreas. She ha a
percutaneous angioplasty with rug-eluting stent 5. Which of the following is true regaring
placement three weeks ago an is on aspirin an pancreatic cysts?
clopiogrel. Despite eating small, frequent meals, A. Serous cystaenoma has malignant potential
she continues to have signicant, intermittent B. Asymptomatic patients with mixe-type
light-heaeness, palpitations, an iaphoresis intrauctal papillary mucinous neoplasm
aily. What is the most appropriate next step in (IPMN) shoul unergo conservative
management? management
A. Octreotie C. Weight loss in patients with IPMN is mostly
B. Diazoxie attribute to an elevate level of TNF-alpha
C. Neoajuvant chemotherapy D. Mucinous cystaenoma usually occurs in
D. Continue aspirin an clopiogrel, procee women an in the boy or tail of the pancreas
with enucleation E. Asymptomatic mucinous cystaenoma can be
E. Hol clopiogrel, continue aspirin, procee manage with repeat imaging in 6 months
with istal pancreatectomy
6. A 55-year-ol man presents with a 1-hour
3. A 60-year-ol man with chronic pancreatitis is history of epigastric pain, nausea, an vomiting.
presenting for follow-up. Despite alcohol an He has iffuse mil abominal tenerness to
smoking cessation, oral analgesic meication, palpation. Laboratory values are signicant
celiac axis nerve block, an ERCP with stent for serum amylase of 800 U/L, serum glucose
placement, he continues to have severe pain of 130mg/L, chlorie of 104 mEq/L, white
an foul-smelling iarrhea. Imaging reveals bloo cell count of 1,000 cells/μL, serum
pancreatic calcication, an enlarge pancreatic soium of 15mEq/L, an triglycerie levels of
hea, an pancreatic uct iameter is 5 mm. 1800mg/L. The most likely explanation for the
What is the most appropriate management to hyponatremia is:
help resolve his symptoms? A. Excessive ui loss
A. Puestow proceure B. Inappropriate antiiuretic hormone response
B. Frey proceure C. Excessive free water replacement
C. Beger proceure D. Pseuohyponatremia
D. Minor papilla sphincterotomy E. Arenal insufciency
E. Whipple
54 PArt i Patient Care

7. Management of pancreatic lymphoma is by: 12. A 60-year-ol man presents with obstructive
A. Pancreaticouoenectomy jaunice, acholic stools, an weight loss. An
B. Chemotherapy abominal ultrasoun scan emonstrates a
C. Pancreaticouoenectomy with postoperative ilate biliary tree an no gallstones. A ynamic
chemotherapy contrast-enhance CT scan emonstrates a
D. Raiation therapy soli mass localize to the hea of the pancreas
E. Preoperative chemoraiation followe by without evience of istant metastasis, or
pancreaticouoenectomy aenopathy. Vascular involvement can’t be
exclue. The patient is otherwise in goo health.
8. Which of the following is true regaring pancreas Laboratory values are normal. Which of the
ivisum? following is the next step in the management?
A. The uct of Santorini ens in a blin pouch A. Exploratory laparotomy
B. The inferior portion of the pancreatic hea B. Diagnostic laparoscopy
rains through the uct of Santorini C. MRI
C. The majority of the pancreas rains through D. Enoscopic ultrasoun
the uct of Santorini E. Positron emission tomography (PET) scan
D. The uct of Wirsung rains through the minor
papilla 13. Which of the following is true regaring
E. The ucts of Wirsung an Santorini fail to alcohol an its relation to the pancreas an/or
evelop pancreatitis?
A. It inuces spasm of the sphincter of Oi
9. The preferre enitive treatment for recurrent B. It ecreases pancreatic secretion
acute pancreatitis ue to pancreas ivisum is: C. A single episoe of binge rinking cannot lea
A. Lateral pancreaticojejunostomy (Puestow to pancreatitis
proceure) D. The type of alcohol consume is an important
B. Pancreaticouoenectomy (Whipple proceure) risk eterminant
C. Minor papilla sphincterotomy E. It inhibits chymotrypsin
D. Major papilla sphincterotomy an pancreatic
uctal septotomy 14. A 48-year-ol male presents with vague abominal
E. Distal pancreatectomy pain of  weeks uration. He was recently
ischarge for an episoe of alcohol-relate
10. A 50-year-ol male with chronic pancreatitis pancreatitis. Laboratory exam is remarkable for
has faile meical management an is being a milly elevate serum amylase. A compute
consiere for more invasive treatment. Which tomography (CT) scan emonstrates a 4-cm well-
of the following is true regaring potential circumscribe peripancreatic ui collection with
interventions? homogenously low attenuation. The borers of the
A. Pancreaticouoenectomy (Whipple proceure) collection appear to be ill-ene. The patient is
is inappropriate for chronic pancreatitis afebrile an hemoynamically stable. What is the
B. Enoscopic proceures have been shown to be most appropriate next step?
superior to surgical treatment A. Intravenous (IV) antibiotics an uis
C. Lateral pancreaticojejunostomy (Puestow B. Amit an place the patient on nothing by
proceure) is appropriate if the pancreatic uct mouth (NPO)
is larger than 6 mm C. Percutaneous aspirate for carcinoembryonic
D. The most common inication for invasive antigen (CEA) level
intervention in chronic pancreatitis is poor D. Exploratory laparotomy
exocrine an enocrine function E. Observe
E. Long-term pain control is similar to either the
Puestow, Beger, or Frey proceure 15. Which of the following is the least favorable
management option for a chronic large pancreatic
11. Aenocarcinoma of the pancreas arises most often pseuocyst?
from which anatomic site? A. Enoscopic transpapillary rainage using a
A. Main pancreatic uct stent
B. Branch pancreatic uct B. Laparoscopic cystogastrostomy
C. Pancreatic acinus C. CT-guie rainage with a pigtail catheter
D. Ampulla of Vater D. Open Roux-en-Y cystojejunostomy
E. Pancreatic islet E. Enoscopic transgastric cystogastrostomy
CHAPtEr 5 Abdomen—Pancreas 55

16. A 65-year-ol man presents with a persistent 20. A 41-year-ol female presents with palpitations,
skin rash of the lower abomen an perineum, trembling, iaphoresis, an confusion. Serum
accompanie by intermittent vague left upper glucose is 48 mg/L an C-peptie level is
quarant pain an recent weight loss. A chemistry elevate. Her symptoms resolve with the
panel reveals serum glucose to be 160 mg/L, but aministration of a carbohyrate loa. Which of
results are otherwise unremarkable. CT reveals a the following is true regaring the most likely
large mass in the pancreas. Which of the following conition?
is true regaring the most likely conition? A. Elevate C-peptie an hypoglycemia rule out
A. This patient is at higher risk for venous an exogenous source
thromboembolic isease B. Patients will often have a mass in the neck of
B. The mass is most commonly in the hea of the the pancreas
pancreas C. The most sensitive stuy for localization is a
C. The secretory peptie responsible for the high-resolution CT scan
symptoms also stimulates exocrine pancreatic D. Recurrent lesions can be manage with
ow streptozocin an 5-FU
D. Patients often have associate hypokalemia E. It is the least common functional pancreatic
E. These are often benign lesions enocrine neoplasm

17. The most common cause of chronic pancreatitis 21. Octreotie scanning is most useful for localization
worlwie is: of which of the following tumors?
A. Gallstones A. VIPoma
B. Alcohol abuse B. Glucagonoma
C. Hereitary C. Pancreatic polypeptie-secreting tumor
D. Hypertriglyceriemia D. Gastrinoma
E. Infectious E. Insulinoma

18. A 35-year-ol cachectic woman presents with 22. Which of the following is true regaring
episoic severe watery iarrhea that has le to pancreatogenic (type 3) iabetes?
multiple hospital amissions for replacement A. Ketoaciosis is common
of uis an electrolytes over the course of B. The iabetes is easily controlle
several months. Stool cultures are repeately C. Peripheral insulin sensitivity is ecrease
negative an she has no history of travel abroa. D. Glucagon an pancreatic polypeptie (PP)
On examination, a mass is palpate in the levels are low
epigastrium/right upper quarant. CT reveals a E. Hyperglycemia is usually severe
large, bulky pancreatic mass with extension into
the superior mesenteric vein an ajacent organs. 23. A 30-year-ol nurse presents with intermittent
The best palliative management option for this iaphoresis, trembling, an palpitations. Her
patient’s symptoms is: fasting bloo sugar is 50 mg/L. Her insulin-
A. Octreotie to-C peptie ratio is greater than 1. Which of the
B. Streptozotocin following is the next step in management?
C. Embolization A. CT scan of the abomen
D. Chemotherapy B. Psychiatric counseling to iscuss sulfonylurea
E. Raiation therapy abuse
C. Psychiatric counseling to iscuss exogenous
19. A 65-year-ol male presents for evaluation of insulin abuse
yellowing skin. Review of systems is signicant D. Octreotie scan
for loose-tting clothes, fatigue, an night sweats. E. Magnetic resonance imaging
Laboratory evaluation is remarkable for elevate
total bilirubin. CT scan reveals a pancreatic mass.
Which of the following is least likely to contribute
to this conition?
A. History of cholecystectomy
B. Diabetes
C. Smoking
D. BRCA
E. Coffee consumption
56 PArt i Patient Care

24. A 60-year-ol alcoholic man presents with 28. Which of the following is true regaring anatomy
chronic, vague abominal pain. He enies a or the embryologic evelopment of the pancreas?
history of pancreatitis an is otherwise in goo A. The most commonly injure vessel uring
health. CT reveals a 6-cm multiloculate, septate issection behin the neck of the pancreas is
cyst at the tail of the pancreas. FNA of the cyst is the celiac vein
noniagnostic. Flui amylase an CEA are in the B. The pancreas receives its arterial supply from
high normal range. Management consists of: only the celiac artery
A. Distal pancreatectomy with possible C. The ventral pancreas constitutes the hea an
splenectomy part of the boy of the pancreas
B. CT-guie rainage of the cyst D. Venous rainage of the pancreas is to the
C. Enoscopic cystogastrostomy inferior vena cava
D. Roux-en-Y cystojejunostomy E. The uncinate process is orsal to the portal
E. Repeat imaging in 6 months vein an superior mesenteric artery

25. After a motor vehicle accient, persistent 29. A 35-year-ol man presents with severe
ascites evelops in a 55-year-ol man. Other abominal pain an iffuse abominal
than the ascites, CT nings are unremarkable. tenerness. CT scan with IV contrast
Paracentesis reveals clear ui with an amylase emonstrates areas of hypoattenuation in the
level of 5000 U/L. The patient fails an attempt at pancreas. His vitals are stable. His temperature is
bowel rest, parenteral nutrition, an paracentesis. 38.4°C. Which of the following is true regaring
Denitive management woul consist of: his conition?
A. Distal pancreatectomy A. Fine-neele aspiration (FNA) for culture
B. Placement of pigtail catheter shoul be performe
C. Roux-en-Y pancreaticojejunostomy B. Early IV antibiotics have emonstrate
D. Pancreaticouoenectomy improve survival
E. Placement of a transuoenal pancreatic uct C. Early necrosectomy ecreases morbiity
stent an mortality when compare with elaye
intervention
26. A 60-year-ol man presents with chronic D. The patient shoul be observe with repeat
epigastric abominal pain an jaunice. CT imaging if he eteriorates clinically
reveals iffuse swelling of the pancreas with E. Percutaneous rainage shoul be performe
compression of the intrapancreatic common
uct. Neele biopsy of the pancreas reveals 30. A 60-year-ol woman presents with gallstone
iffuse brosis an a plasma an lymphocytic pancreatitis. Which of the following is the best
inltrate. Serum IgG levels are increase. Primary preictor of a resiual gallstone persisting in the
management consists of: common bile uct?
A. Whipple proceure A. Persistent elevation of the total bilirubin level
B. Sterois B. A ilate common bile uct on amission
C. Chemotherapy C. Persistent elevation of the alkaline
D. Hepaticojejunostomy phosphatase level
E. ERCP with stenting D. Persistent elevation of the serum amylase level
E. Persistent abominal pain
27. A 61-year-ol female unergoes a
pancreaticouoenectomy (Whipple) operation. 31. Which of the following pancreatic cystic lesions is
On postoperative ay ve she becomes almost exclusively foun in a young female?
hypotensive, tachycaric, an has severe A. Serous cystic aenoma
abominal pain. Nasogastric tube emonstrates B. Mucinous cystic neoplasm
bilious output. She receives  L of uis an C. Sie-uct IPMN
BP improves to 110 mmHg. A CT scan reveals a D. Main-uct IPMN
signicant amount of free (with HU [hounsel E. Soli pseuopapillary epithelial neoplasm
units] of 5). The next step in her management is:
A. Angiography with embolization
B. Immeiate take back to the OR
C. IV octreotie rip
D. Transfuse bloo an transfer to ICU
E. Upper enoscopy
CHAPtEr 5 Abdomen—Pancreas 57

Answers
1. C. This patient has a symptomatic, large pancreatic consiere for metastatic insulinomas an is inappropriate
pseuocyst. Since it has been at least 6 weeks after his epi- in this case as the majority of insulinomas are benign (C).
soe of acute pancreatitis, an the pseuocyst is >6 cm, he References: Valgimigli M, Bueno H, Byrne RA, et al. 017 ESC
shoul be offere enitive treatment (E). The majority of focuse upate on ual antiplatelet therapy in coronary artery is-
pancreatic pseuocysts are manage with enoscopic cys- ease evelope in collaboration with EACTS: The Task Force for
togastrostomy as it is minimally invasive an has a high ual antiplatelet therapy in coronary artery isease of the European
Society of Cariology (ESC) an of the European Association for
success rate. In orer to perform enoscopic cystogastros-
Cario-Thoracic Surgery (EACTS). Eur Heart J. 018;39(3):13–60.
tomy, the pseuocyst must abut the gastric wall. However,
Gill GV, Rauf O, MacFarlane IA. Diazoxie treatment for insuli-
the above patient has a history of Roux-en-Y gastric bypass. noma: a national UK survey. Postgrad Med J. 1997;73(864):640–641.
The gastric funus is part of the remnant stomach an is not
easily accessible enoscopically. As such, an enoscopic cys-
3. C. This patient has chronic pancreatitis with persistent
togastrostomy woul not be routinely offere as this woul
pain espite meical management with celiac axis nerve
require the expertise of a highly skille enoscopist using
block an is therefore a surgical caniate. When etermin-
ouble-push balloon enoscopy techniques (A). Percutane-
ing which proceure to perform, there are two main factors to
ous rainage is not an ieal option because there is a high
consier: (1) if the pancreatic uct is ilate (≥6 mm), an ()
rate of pancreaticocutaneous stula formation an shoul
if the pancreatic hea is involve. In the case of an enlarge
be reserve for infecte pancreatic pseuocysts in patients
pancreatic hea an a normal-size main pancreatic uct
too unstable for enoscopy or surgery (B). Surgical options
(<6mm) (such as in this case), the most appropriate proceure
inclue cystogastrostomy or Roux-en-Y cyst-jejunostomy.
is the Beger proceure, which involves uoenum-sparing
In this case, the patient alreay has a Roux-en-Y bypass an
resection of most of the pancreatic hea with ivision of the
cystogastrostomy is the more appropriate option to rain the
pancreatic boy over the portal vein an reconstruction via a
pseuocyst without signicantly altering the anatomy (D).
sie-to-sie an sie-to-en pancreaticojejunostomy to rain
Reference: Nealon WH, Walser E. Surgical management of com-
plications associate with percutaneous an/or enoscopic manage-
the remaining hea an tail of the pancreas. When the pan-
ment of pseuocyst of the pancreas. Ann Surg. 005;41(6):948–957. creatic hea is not involve an there is pancreatic uctal
ilation of ≥6 mm, the Puestow proceure (lateral pancre-
2. B. This patient is experiencing symptoms of hypoglyce- aticojejunostomy) is most appropriate (A). When the pan-
mia seconary to her insulinoma espite ahering to eating creatic hea is involve an the pancreatic uct is ilate,
frequent, small meals. The treatment of choice for insulino- the most appropriate proceure is the Frey proceure, which
mas is surgical removal. If the insulinoma is small (< cm) involves coring out the pancreatic hea an then perform-
an > mm away from the pancreatic uct, enucleation can ing lateral pancreaticojejunostomy (B). A Whipple inclues a
be performe. Choice (D) is incorrect for two reasons: (1) it pancreaticouoenectomy an is a highly morbi operation
woul be inappropriate to procee with major pancreatic for benign isease (E). Minor papilla sphincterotomy is ini-
surgery while on ual antiplatelet therapy; an () because cate for pancreatitis in the setting of pancreas ivisum (D).
this patient’s tumor is 3 cm, the treatment of choice is resec-
tion with istal pancreatectomy an not enucleation (D). 4. A. The presence of gallstones is the most common cause
However, because of her history of percutaneous coronary of acute pancreatitis worlwie, which is thought to be ue
intervention with rug-eluting stent placement less than 1 to a gallstone causing transient obstruction at the ampulla
month ago, she shoul continue ual antiplatelet therapy (E). of Vater. In most cases, the inammation is mil to moer-
After rug-eluting stent placement, ual antiplatelet therapy ate, an the stone passes into the intestine spontaneously.
shoul ieally be continue for 6 months to minimize stent In patients with severe pancreatitis, early cholecystectomy
thrombosis. If urgent surgery is neee, clopiogrel can be is associate with an increase morbiity an mortality, so
temporarily hel prior to the 6-month mark but shoul not cholecystectomy shoul be elaye until the pancreatitis is
be hel within the rst 4 to 6 weeks when stent thrombosis resolve (D). In mil to moerate pancreatitis, the timing
risk is the highest. Therefore, this patient’s treatment shoul of surgery is not critical, an early cholecystectomy (within
be focuse on symptom management until she is reay for 48 hours) can be performe safely. However, long elays
surgery. Diazoxie is the initial meication of choice to con- result in as much as a 30% recurrence of pancreatitis. Rou-
trol symptoms in patients with insulinomas. It works by tine ERCP to etect the presence of common uct stones
inhibiting the release of insulin from beta islet cells. While is unnecessary because the probability of ning resiual
octreotie is a goo option to control symptoms from VIPo- stones is low an the risk of ERCP-inuce pancreatitis is
mas an glucagonomas, it oes not work reliably for insuli- signicant (B, E). Preoperative ERCP shoul be reserve for
nomas as insulinomas o not always contain somatostatin patients with concomitant cholangitis or clear evience of
receptors. Octreotie shoul only be consiere as symptom biliary obstruction (jaunice, persistent elevation of total bil-
management for insulinomas if octreotie scanning is posi- irubin >4 mg/L). Otherwise, an intraoperative cholangio-
tive (inicating that the tumor contains somatostatin recep- gram shoul be performe, an if a common bile uct stone
tors). Otherwise, octreotie will inhibit glucagon an actually is etecte, either a laparoscopic common uct exploration
worsen hypoglycemia (A). Chemotherapy is generally only or a postoperative ERCP shoul be performe (C).
58 PArt i Patient Care

References: Chang L, Lo S, Stabile BE, Lewis RJ, Toosie K, e References: Arcari A, Anselmi E, Bernuzzi P. Primary pancreatic
Virgilio C. Preoperative versus postoperative enoscopic retrograe lymphoma: a report of ve cases. Haematologica. 005;90(1), ECR09.
cholangiopancreatography in mil to moerate gallstone pancreati- Bouvet M, Staerkel GA, Spitz FR, et al. Primary pancreatic lym-
tis: a prospective ranomize trial. Ann Surg. 000;31(1):8–87. phoma. Surgery. 1998;13(4):38–390.
Kelly TR, Wagner DS. Gallstone pancreatitis: a prospective ran- Grimison P, Chin M, Harrison M. Primary pancreatic lympho-
omize trial of the timing of surgery. Surgery. 1988;104(4):600–605. ma-pancreatic tumors that are potentially curable without resection:
Rosing DK, e Virgilio C, Yaghoubian A, et al. Early cholecystec- a retrospective review of four cases. BMC Cancer. 006;6.
tomy for mil to moerate gallstone pancreatitis shortens hospital
stay. J Am Coll Surg. 007;05(6):76–766. 8. C. In pancreatic ivisum, the ucts of Wirsung an San-
torini fail to fuse (E). The result is that the majority of the
5. D. Serous cystaenoma is a benign true cyst that most pancreas rains through the uct of Santorini an through
commonly occurs in women an in the pancreatic hea. It is the lesser papilla. The inferior portion of the pancreatic hea
often asymptomatic, but large cysts (>4 cm) may cause vague an uncinate process rains through the uct of Wirsung
abominal pain. They o not nee to be resecte unless they an the major papilla (B, D). It is consiere a normal ana-
are symptomatic (A). Mucinous cystaenoma is consiere tomic variant an is seen in 10% of iniviuals. It is thought
premalignant, has a female preominance, occurs com- to lea to an increase risk of pancreatitis because the minor
monly in the boy or tail of the pancreas, an shoul always papilla sometimes cannot hanle the higher ow of pancre-
unergo resection (E). IPMN is ivie into three types base atic juices. In another more common variant, the uct of San-
on pancreatic uct involvement: main-uct, sie-branch, an torini ens in a blin pouch but still fuses with the Wirsung
mixe-type. Main-uct IPMN carries up to a 50% risk of har- uct (A).
boring malignant cells an shoul always be resecte in sur-
gically appropriate caniates. Mixe-type IPMN also has a 9. C. Pancreas ivisum can lea to recurrent episoes of
higher risk an shoul be remove (B). Sie-branch IPMN acute pancreatitis as well as chronic pancreatitis with intrac-
has a lower risk of malignancy an can be observe unless table pain. Unlike other forms of chronic pancreatitis, how-
it is symptomatic, larger than 3 cm, or associate with mural ever, marke ilation of the orsal uct is unusual. As such,
noules. The weight loss in patients with IPMN is mostly surgical ecompressive proceures are not successful (A, B).
attribute to exocrine insufciency from uct blockage an For patients with recurrent attacks of acute pancreatitis, the
not TNF-alpha cachexia (C). best option is sphincterotomy of the minor papilla because
the uct of Santorini is proviing the primary rainage to
6. D. Severe hypertriglyceriemia leas to a falsely low the pancreas. A stuy from Marseille foun a ecrease rate
soium level. Water is isplace in the serum by lipis, of acute pancreatitis in 4 patients after minor papilla sphinc-
resulting in an error in measurement. The anger is that terotomy an orsal uct stenting. The complication rate was
the clinician who is unaware may try to correct the hypo- lower with sphincterotomy than with stent insertion. Major
natremia with hypertonic saline, leaing to severe hyper- papilla sphincterotomy woul not likely be helpful because
natremia. Similarly, a signicantly elevate level of serum it rains a minority of the pancreas in pancreatic ivisum
glucose can also result in pseuohyponatremia. Excess (D). Distal pancreatectomy is typically not neee (E).
volume loss seconary to emesis can lea to a hypovole- Reference: Heyries L, Barthet M, Delvasto C, Zamora C, Ber-
mic hyponatremia but is accompanie by a hypochloremic nar JP, Sahel J. Long-term results of enoscopic management of
metabolic alkalosis (A). Patients with gastrointestinal (GI) pancreas ivisum with recurrent acute pancreatitis. Gastrointest
losses can have hyponatremia exacerbate by excessive free Endosc. 00;55(3):376–381.
water replacement (C). Arenal insufciency may lea to
hyponatremia seconary to the loss of action of alosterone 10. C. The most common inication for surgical interven-
at the istal convolute renal tubules but is accompanie tion in patients with chronic pancreatitis is chronic pain (D).
by severe refractory hypotension an marke hyperkale- Surgical rainage of a ilate pancreatic uct with istal
mia (E). obstruction is more effective than enoscopic approaches in
Reference: Howar J, Ree J. Pseuohyponatremia in acute patients with chronic pancreatitis (B). The Puestow proce-
hyperlipemic pancreatitis: a potential pitfall in therapy. Arch Surg. ure involves cutting open the length of the main pancreatic
1985;10(9):1053–1055. uct an anastomosing a Roux limb of jejunum to the uct
but requires a ilate uct (>6 mm). Both the Whipple pro-
7. B. Primary pancreatic lymphoma is extremely rare. Thus, ceure (for inammation limite to the pancreatic hea) an
the management approach is base on case series an expe- total pancreatectomy are options for the treatment of intrac-
rience with lymphoma at other sites. Patients with pancre- table chronic pancreatitis, although they are associate with
atic lymphoma may present with symptoms an CT nings greater morbiity than a rainage proceure (A). The Beger
suggestive of pancreatic aenocarcinoma, an as such, it proceure is another option, which resects the pancreatic
may be ifcult to iagnose preoperatively. However, sus- hea but spares the uoenum, stomach, an bile uct, but
picion of lymphoma shoul be raise in the presence of a this is a technically challenging proceure. The Frey proce-
large bulky pancreatic tumor or with more iffuse pancre- ure is similar to Beger but easier to perform since it avois
atic involvement. This is one situation in which CT-guie the transection of the pancreatic neck over the superior mes-
neele biopsy of the mass is inicate because the majority enteric vessels. The best long-term pain control is achieve
of stuies inicate that pancreatic lymphoma respons to with longituinal pancreaticojejunostomy with limite
chemotherapy as the primary moality. Surgery or raiation resection of the hea of the pancreas, which Beger an Frey
is not typically use in the management of pancreatic lym- both satisfy, with Frey being the preferre option (E). How-
phoma (A, C–E). ever, Frey requires a ilate uct an pancreatic hea.
CHAPtEr 5 Abdomen—Pancreas 59

References: Cahen DL, Gouma DJ, Nio Y, et al. Enoscopic ver- any aitional information beyon what is provie with
sus surgical rainage of the pancreatic uct in chronic pancreatitis. CT (E).
N Engl J Med. 007;356(7):676–684. References: Small W, Hayes JP, Suh WW. ACR appropriateness
DiMagno MJ, DiMagno EP. Chronic pancreatitis. Curr Opin Gas- criteria [r] borerline an unresectable pancreas cancer. Oncology.
troenterol. 01;8(5):53–531. 016;30(7):619–619.
Jawa ZAR, Kyriakies C, Pai M, et al. Surgery remains the Tummala P, Junaii O, Agarwal B. Imaging of pancreatic cancer:
best option for the management of pain in patients with chronic an overview. J Gastrointest Oncol. 011;(3):168–174.
pancreatitis: a systematic review an meta-analysis. Asian J Surg. Wang WL, Ye S, Yan S, et al. Pancreaticouoenectomy with por-
017;40(3):179–185. tal vein/superior mesenteric vein resection for patients with pan-
Roch A, Teysseou J, Mutter D, Marescaux J, Pessaux P. Chronic creatic cancer with venous invasion. Hepatobiliary Pancreat Dis Int.
pancreatitis: a surgical isease? Role of the Frey proceure. World J 015;14(4):49–435.
Gastrointest Surg. 014;6(7):19–135.
13. A. The exact mechanism by which alcohol inuces pan-
11. A. The majority of aenocarcinomas of the pancreas
creatitis is unclear. Ethanol inuces spasm of the sphincter
arise from the main pancreatic uct. Approximately 66%
of Oi, an this may lea to an increase in uctal pressure
of pancreatic aenocarcinomas evelop within the hea or
with a simultaneous brief stimulation of pancreatic secretion
uncinate process of the pancreas. The remaining answer
(B). It also increases pancreatic uct permeability, ecreases
choices can lea to pancreatic aenocarcinoma, but it occurs
pancreatic bloo ow, an inappropriately activates chymo-
less frequently (B, C, E). Carcinoma at the ampulla of Vater is
trypsin (E). Most patients with alcohol-relate pancreatitis
most commonly uoenal aenocarcinoma (D).
have a longstaning history of heavy rinking. The type of
Reference: Albores-Saavera J, Schwartz AM, Batich K, Henson
DE. Cancers of the ampulla of Vater: emographics, morphology,
alcohol consume is not important but rather the quantity
an survival base on 5,65 cases from the SEER program: Cancer of an uration (D). The mean amount consume in patients in
the Ampulla of Vater. J Surg Oncol. 009;100(7):598–605. whom pancreatitis evelops is 100 to 175 g/ay, although it
can rarely evelop after just one binge (C). Aitionally, the
12. D. In a patient with obstructive jaunice, the rst stuy risk of pancreatitis seems to be higher in patients who have a
to perform is an abominal ultrasoun scan. In the absence iet high in protein an fat.
of abominal pain an in the presence of weight loss, it is
highly likely that the iagnosis is malignancy. A ynamic, 14. E. The history of recent pancreatitis combine with the
contrast-enhance CT scan is highly effective in etermining history of vague abominal pain, elevate serum amylase,
the resectability of the mass. In cases where vascular involve- an CT scan emonstrating a peripancreatic ui collection
ment is not clear, enoscopic ultrasonography has aie in most likely represents pancreatic pseuocyst. Most patients
etermining resectability. Pancreatic cancer is consiere with pseuocyst o not nee amission an can continue
unresectable if the tumor is encasing or occluing the supe- to eat, although a low-fat iet is recommene. Amission
rior mesenteric vein or portal vein an causing vein contour an total parenteral nutrition (TPN) woul only be recom-
irregularity, as this is consiere unreconstructable. Ai- mene if they were unable to tolerate an oral iet (B). There
tionally, pancreatic cancer is consiere unresectable if the is no reason to start IV antibiotics because he is not present-
tumor is abutting or encasing the superior mesenteric artery, ing with an infecte pseuocyst (A). Initial management of
hepatic artery, or celiac trunk by more than 180°. More fre- pseuocysts is conservative via observation because most
quently, enoscopic guie biopsy is being performe. The spontaneously resolve. Pancreatic cyst CEA level is consi-
avantage of this approach is that there is no risk of tumor ere the most accurate tumor marker for iagnosing a muci-
seeing because the area through which the neele is passe nous pancreatic cystic lesion. However, in the present setting,
becomes part of the Whipple specimen. That being sai, in given the high suspicion for a pseuocyst, it woul not be
the situation in which the mass appears to be resectable, per- neee (C). Invasive interventions are inappropriate because
cutaneous or enoscopic ultrasonography–guie biopsy most pseuocysts resolve spontaneously (D). Preictors of
is not consiere necessary. Neele biopsy is prone to sam- failure for conservative management inclue pancreatic
pling error; therefore, a negative biopsy ning woul not pseuocysts larger than 6 cm or those that have persiste for
alter the plan to perform a Whipple proceure (A). Likewise, more than 6 weeks. CT or ultrasoun can be use to charac-
a positive biopsy ning woul not alter the operative eci- terize interval changes in pancreatic pseuocysts.
sion. Operative morbiity an mortality after the Whipple
proceure are sufciently low that one woul accept the low 15. C. Internal rainage is usually preferre to external
likelihoo (∼5%) that the lesion is benign. Biopsy shoul rainage for a symptomatic pancreatic pseuocyst that
be reserve for situations in which the lesion appears to be has faile to resolve with conservative therapy. External
unresectable because it may guie chemotherapy. It is also rainage is associate with a higher rate of complications,
inicate in situations in which the appearance of the mass incluing infection an pancreaticocutaneous stula. The
suggests other less common pathologies such as pancreatic only inication for percutaneous rainage is in a patient
lymphoma. Diagnostic laparoscopy is often one before with a ocumente or clinically apparent infected pancreatic
proceeing with a Whipple to conrm there are no obvious pseuocyst that is unstable for a surgical or enoscopic pro-
hepatic or peritoneal lesions (B). Suspecte lesions are sent ceure. Pseuocysts communicate with the pancreatic uctal
for a frozen sample. MRI may be a useful ajunct in patients system in 80% of cases. Internal rainage can be achieve
with equivocal nings on CT or in cases where hepatic enoscopically via a transmural approach or a transpap-
metastasis is suspecte (C). The role of PET in cancer workup illary approach. This is gaining popularity making it the
continues to evelop but as of now it is unclear if PET as new rst-line treatment for pancreatic pseuocyst. If there

AL GRAWANY
60 PArt i Patient Care

is portal hypertension (e.g., splenic vein thrombosis, uner- 18. A. The patient most likely has a VIPoma. It has also
lying cirrhosis, esophageal or gastric varices), then surgical been terme WDHA (watery iarrhea, hypokalemia, an
open internal rainage may be more appropriate. Options achlorhyria) an Verner-Morrison synrome. Patients have
inclue a cystogastrostomy, a Roux-en-Y cystojejunostomy, large-volume secretory iarrhea an can lose enormous
an a cyst uoenostomy (A–B, D–E). Cystogastrostomy amounts of uis an electrolytes. Diagnosis is by CT scan,
can be performe enoscopically, laparoscopically, or with a an most tumors have metastasize by the time of iagnosis.
combine approach. Failure of the enoscopic approach can Another useful imaging tool is enoscopic ultrasonography.
be preicte by the ning of major uctal isruption or ste- Even with istant metastasis, however, tumor ebulking,
nosis on enoscopic retrograe cholangiopancreatography hepatic artery embolization, an raiofrequency ablation of
(ERCP) or magnetic resonance cholangiopancreatography. liver metastasis are useful in controlling symptoms (C, E).
Regarless of the approach, biopsies of the cyst wall must be The best meical treatment of symptoms is achieve with
one to rule out malignancy. octreotie, a somatostatin analogue. Chemotherapy has no
References: Cantasemir M, Kara B, Kantarci F, Mihmanli role in the management of VIPoma (D). Streptozotocin is
I, Numan F, Erguney S. Percutaneous rainage for treatment of toxic to pancreatic beta cells an may be useful in the man-
infecte pancreatic pseuocysts. South Med J. 003;96():136–140. agement of insulinoma (B).
Nealon WH, Walser E. Surgical management of complications Reference: Nguyen HN, Backes B, Lammert F, et al. Long-term
associate with percutaneous an/or enoscopic management of survival after iagnosis of hepatic metastatic VIPoma: report of two
pseuocyst of the pancreas. Ann Surg. 005;41(6):948–957. cases with isparate courses an review of therapeutic options. Dig
Yusuf TE, Baron TH. Enoscopic transmural rainage of pancre- Dis Sci. 1999;44(6):1148–1155.
atic pseuocysts: results of a national an an international survey of
ASGE members. Gastrointest Endosc. 006;63():3–7.
19. E. Coffee rinking has not been shown to be a risk fac-
tor for pancreatic cancer. Factors that are associate with a
16. A. Glucagonoma can be remembere by the 4 Ds: ia-
risk for pancreatic cancer inclue smoking (strongest an
betes, ermatitis, eep vein thrombosis, an epression.
accounts for 5%–30% of all cases) (C), obesity, iabetes (B),
The rash is terme necrolytic migratory erythema an tens to
atypical multiple mole melanoma, hereitary pancreatitis
manifest on the lower abomen or perineum. The mass char-
(A), familial aenomatous polyposis, hereitary nonpolyp-
acteristically appears in the tail of the pancreas along with
osis colon cancer, BRCA2 (D), an Peutz-Jeghers synrome.
VIPoma (a neuroenocrine tumor that secretes vasoactive
The role of alcohol in pancreatic cancer is ebatable. More
intestinal polypeptie [VIP]). The responsible hormone, glu-
recently, a history of cholecystectomy an/or cholelithiasis
cagon, inhibits exocrine pancreatic ow (C). The iagnosis
has been emonstrate to be associate with an increase
of glucagonoma is conrme by measuring fasting gluca-
risk of pancreatic cancer (A).
gon levels. Because the tumors are in the istal pancreas, the
References: Fan Y, Hu J, Feng B. Increase risk of pancreatic
patient oes not usually present with jaunice; as such, the cancer relate to gallstones an cholecystectomy: a systemic review
iagnosis is often mae late when the tumor is large. Because an meta-analysis. Pancreas. 016;45(4):503–509.
glucagonoma is most commonly malignant, it shoul be Lowenfels AB, Maisonneuve P. Epiemiology an prevention of
remove with enucleation (if < cm) or by istal pancre- pancreatic cancer. Jpn J Clin Oncol. 004;34(5):38–44.
atectomy (E). Somatostatinoma can present with iabetes,
gallstones, steatorrhea, an hypochlorhyria an most com- 20. D. Insulinoma is the most common functional pan-
monly occurs in the hea of the pancreas along with pancre- creatic enocrine neoplasm (E). The classic feature is the
atic polypeptie-secreting tumor (B). Patients with VIPoma Whipple tria, which inclues symptomatic fasting hypo-
have large-volume secretory iarrhea an can lose enormous glycemia, a ocumente serum glucose level of less than
amounts of uis an electrolytes incluing potassium (D). 50mg/L, an relief of symptoms with the aministration
References: Vinik A, Feliberti E, Perry RR. Glucagonoma syn- of glucose. Patients will often present with recurrent epi-
rome. Endotext. 014;7:89–107. soes of syncope. They may also report palpitations, trem-
Schapiro H, Luewig RM. The effect of glucagon on the exocrine bling, iaphoresis, confusion or isorientation, an seizures.
pancreas. A review. Am J Gastroenterol. 1978;70(3):74–81. The iagnosis is conrme by emonstrating a low fasting
bloo sugar (insulin to glucose ratio of >0.3) an an elevate
17. B. For acute pancreatitis, gallstones an alcohol abuse C peptie level. However, the avent of newer antiiabetic
are by far the two most common etiologies, with a slightly meications such as sulfonylureas can also present with a
higher incience of biliary pancreatitis. Biliary pancreatitis, similar biochemical prole (A). Localization is achieve by
however, leas to chronic pancreatitis far less often (A). Alco- CT scan an enoscopic ultrasonography. On occasion, they
hol abuse is by far the most common cause of chronic pancre- cannot be localize preoperatively, in which case, intraoper-
atitis. Although hypertriglyceriemia, infection (often viral), ative ultrasonography is useful an is consiere the most
an hereitary synromes can lea to acute pancreatitis, sensitive imaging stuy. In contrast to the other functional
they occur less frequently than alcohol abuse an gallstones enocrine pancreatic neoplasms, an octreotie scan is poor
(C, D, E). at localizing insulinoma owing to the fact that these lesions
References: Fisher WE, Anersen DK, Bell RH, etal. Pancreas. may not express sufcient somatostatin receptors (C). They
In: Brunicari FC, Anersen DK, Billiar TR, et al., es. Schwartz’s
are evenly istribute throughout the hea, boy, an tail
principles of surgery. 8th e. New York: McGraw-Hill; 005:11–196.
Steer ML. Exocrine pancreas. In: Townsen CM, Jr, Beauchamp of the pancreas. There is no pancreatic tumor that character-
RD, Evers BM, Mattox KL, es. Sabiston textbook of surgery: the biolog- istically appears in the neck of the pancreas (B). The major-
ical basis of modern surgical practice. 17th e. Philaelphia: W.B. Saun- ity of insulinomas are benign (90%). They can be treate
ers; 004:1643–1678. with enucleation. Diazoxie inhibits insulin release an is
CHAPtEr 5 Abdomen—Pancreas 61

occasionally use for preoperative control of symptoms to insulin is proinsulin. Proinsulin is package in the pancre-
relate to hypoglycemia symptoms. For patients with recur- atic B cell, where it is cleave to insulin an C peptie, which
rent or metastatic malignant insulinoma, tumor ebulking are then release into the circulation at an equal ratio. Insu-
may be benecial as is the use of streptozocin an 5-FU. lin is cleare by the liver, whereas C peptie is cleare by
References: Dewitt CR, Hear K, Waksman JC. Insulin an the kiney an is cleare more slowly than insulin, such that
C-peptie levels in sulfonylurea-inuce hypoglycemia: a systemic the normal insulin-to-C peptie ratio is less than 1 uring
review. J Med Toxicol. 007;3(3):107–118. fasting. With a true insulinoma, both insulin an C peptie
Halfanarson TR, Rubin J, Farnell MB, Grant CS, Petersen GM. levels woul be elevate; however, the ratio woul still be
Pancreatic enocrine neoplasms: epiemiology an prognosis of
less than 1. Factitious hypoglycemia will present with an
pancreatic enocrine tumors. Endocr Relat Cancer. 008;15():409–47.
insulin-to-C peptie ratio greater than 1 only if the patient
is using exogenous insulin. In contrast, sulfonylurea abuse
21. D. Many pancreatic enocrine tumors have high con-
will have a ratio of less than 1 since it stimulates proinsu-
centrations of somatostatin receptors an can therefore be
lin release from the pancreas (B). Factitious hypoglycemia
image with a raiolabele form of the somatostatin ana-
has been reporte more frequently in health-care workers
logue octreotie (inium-111 pentetreotie). Octreotie
an is associate with a higher incience of suicie, epres-
scanning has the avantage of whole-boy scanning, which
sion, an personality isorers. Thus, the patient shoul be
is useful in gastrinomas because they can present in a wie
referre for psychiatric counseling. Octreotie scan (D) is not
area. Use in combination with enoscopic ultrasonography,
useful in the workup for insulinoma but CT, MRI, or eno-
it etects more than 90% of gastrinomas. It is also useful for
scopic ultrasoun may emonstrate a pancreatic mass (A, E).
localizing carcinoi tumors. As many as 90% of gastrinomas
References: Lebowitz M, Blumenthal S. The molar ratio of insu-
are foun in the Passaro triangle, an area ene by the junc-
lin to C-peptie: an ai to the iagnosis of hypoglycemia ue to sur-
tion of the cystic uct an common bile uct, the secon an reptitious (or inavertent) insulin aministration. Arch Intern Med.
thir portions of the uoenum, an the neck an boy of 1993;153(5):650–655.
the pancreas. Although a CT scan is also useful, an octreotie Waickus CM, e Bustros A, Shakil A. Recognizing factitious
scan is particularly helpful in localizing gastrinomas smaller hypoglycemia in the family practice setting. J Am Board Fam Pract.
than 1 cm. Somatostatinoma an VIPoma ten to be large 1999;1():133–136.
bulky tumors an are thus reaily seen by CT (A). Gluca-
gonoma may present with a mass seen in the pancreatic tail 24. A. It is important to be aware that not all ui-lle pan-
(B). Octreotie scanning will miss as many as 40% of insuli- creatic abnormalities in a patient with a history of rinking
nomas because they may not express sufcient somatosta- represent pseuocysts (B–E). Some of these lesions may rep-
tin receptors (E). Pancreatic polypeptie (PP) seems to have resent cystic neoplasms of the pancreas. Suspicion of a cystic
an important role in glucose metabolism. PP regulates the neoplasm shoul be particularly increase in the absence of
expression of the hepatic insulin receptor gene. PP-secreting a history of pancreatitis, as in this patient. A cystic neoplasm
tumor is rare an often asymptomatic but can be establishe shoul also be suspecte when the CT scan emonstrates
by the presence of an enhancing solitary pancreatic hea a soli component (septation) in the cystic lesion. The if-
tumor on CT imaging with elevate fasting PP level (C). ferential iagnosis inclues serous cystaenoma, mucinous
Reference: e Herer WW, Kwekkeboom DJ, Valkema R, et al. cystic neoplasm, intrauctal papillary-mucinous aenoma,
Neuroenocrine tumors an somatostatin: imaging techniques. J an soli pseuopapillary neoplasm. On a CT scan, a central
Endocrinol Invest. 005;8(11 Suppl International):13–136. scar is characteristic of a serous cystaenoma (although pres-
ent in only 0%), whereas the ning of peripheral eggshell
22. D. Diabetes in the setting of chronic pancreatitis or after calcications, although rare, is iagnostic of mucinous cystic
pancreatic resection is terme type 3 diabetes. It iffers from neoplasm an highly suggestive of cancer. In the patient pre-
type 1 an  iabetes in that it is associate with ecrease sente, the proceure of choice is surgical resection with is-
glucagon an PP levels an insulin ue to pancreatic loss tal pancreatectomy an splenectomy. This is base on several
or estruction. Because all three of these hormones regulate factors: the patient is having symptoms; he is a goo cani-
glucose levels, the ensuing iabetes is consiere to be if- ate for surgery; the lesion is reaily amenable to resection;
cult to control (B). Furthermore, peripheral insulin sensitivity an the lesion is large, has septations, an has multiple loc-
is increase, whereas hepatic insulin sensitivity is ecrease ulations. If, conversely, a patient has an incientally iscov-
(C). The result is that patients are prone to the evelopment ere pancreatic cyst without symptoms, surgery is generally
of hypoglycemia, but ketoaciosis an marke hyperglyce- recommene if the risk of surgery is low. Before surgery,
mia are rare (A, E). For iabetes to evelop as a result of pan- further stuies are recommene to attempt to etermine
creatitis, extensive estruction of the pancreas must occur. In the malignant potential. The workup may inclue MRI,
fact, resections involving up to 80% of an otherwise normal enoscopic ultrasonography to better elineate the mass,
glan can be one without enocrine insufciency. This may an CT-guie aspiration of the ui for amylase level an
help explain why not all post-Whipple patients evelop poor tumor markers (carcinoembryonic antigen, CA 19–9, CA 15,
glucose control. CA 7–4, CA 15–3).

23. C. Although the patient has symptomatic hypoglycemia, 25. E. After surgery, trauma, or bouts of pancreatitis, per-
seemingly consistent with an insulinoma, her insulin-to-C sistent ascites or pleural effusions can evelop. These are
peptie ratio is greater than 1. This combination, particularly generally cause by a isruption of the pancreatic uct,
in a health-care worker, is highly suggestive of factitious with free extravasation of pancreatic ui, leaing to the
hypoglycemia with exogenous insulin abuse. The precursor evelopment of an internal pancreatic stula, which is rare.
62 PArt i Patient Care

More commonly, the extravasate ui leas to the forma- gastrojejunostomy anastomosis if the afferent/efferent limbs
tion of a containe ui collection known as a pseuocyst. are to be evaluate. Transfusion of bloo is appropriate but
Management of pancreatic ascites or effusion rst requires transport interventional suite shoul be next, as the patient
establishing the iagnosis by obtaining a sample of the ui may have a heral blee followe by exsanguination (D).
an emonstrating a markely elevate amylase level an Octreotie has no role in the management of gastrouoe-
a protein level greater than 5 g/L. Serum amylase may be nal artery stump bleeing (C). One stuy emonstrate that
elevate from reassertion across the peritoneal membrane. wrapping the gastrouoenal artery stump using the falci-
The recommene management is a stepwise progression, form ligament uring surgery may ecrease the risk of this
rst with conservative management with bowel rest, paren- complication.
teral nutrition, placing the patient NPO, an paracentesis to References: Xu C, Yang X, Luo X. Wrapping the gastrou-
completely rain the ui. If this fails to resolve the internal oenal artery stump uring pancreatouoenectomy reuce
stula, ERCP with pancreatic stenting is recommene. If the stump hemorrhage incience after operation. Chin J Cancer.
this fails, surgery is inicate an shoul be tailore to the 014;6(3):99–308.
Han GJ, Kim S, Lee NK, et al. Preiction of late postoperative
location of the uctal injury (B). For istal uct isruptions,
hemorrhage after Whipple proceure using compute tomogra-
a istal pancreatectomy is recommene (A), whereas for
phy performe uring early postoperative perio. Korean J Radiol.
isruption of the boy, a Roux-en-Y pancreaticojejunos- 018;19():84–91.
tomy is performe (C). Whipple proceure (pancreati-
couoenectomy) is not neee (D). Conservative therapy 28. E. The ventral pancreas constitutes the uncinate
incluing somatostatin is successful in only approximately process an inferior portion of the hea of the pancreas,
50%, so nearly one-half will require an invasive proceure. leaving the remainer the embryologic remnant of the or-
References: Gómez-Cerezo J, Barbao Cano A, Suárez I, Soto A, sal pancreas (C). The uncinate process lies ventral to the
Rios JJ, Vazquez JJ. Pancreatic ascites: Stuy of therapeutic options
aorta but orsal to the portal vein an superior mesenteric
by analysis of case reports an case series between the years 1975
an 000. Am J Gastroenterol. 003;98(3):568–577.
artery. The most commonly injure vessel uring issec-
O’Toole D, Vullierme MP, Ponsot P, et al. Diagnosis an manage- tion behin the neck of the pancreas is the superior mes-
ment of pancreatic stulae resulting in pancreatic ascites or pleural enteric vein (A). The pancreas receives bloo supply from
effusions in the era of helical CT an magnetic resonance imaging. two sources: the celiac axis (superior pancreaticouoenal
Gastroenterol Clin Biol. 007;31(8–9 Pt 1):686–693. artery) an superior mesenteric artery (inferior pancreati-
couoenal artery) (B). Venous rainage of the pancreas is
26. B. Autoimmune pancreatitis is a form of chronic pan- to the portal system (D).
creatitis that is increasingly being recognize an can be
confuse with pancreatic lymphoma or pancreatic cancer. It 29. D. CT scan with IV contrast emonstrating areas of
presents most often as a iffusely enlarge hypoechoic pan- hypoattenuation (nonperfuse) in the pancreas in a patient
creas. A CT scan often shows iffuse narrowing of the main with this presentation is concerning ue to necrotizing pan-
pancreatic uct without the typical calcications seen with creatitis. It is important to note that the necrotic pancreas is
chronic alcoholic pancreatitis. Pathology reveals a plasma not usually infecte initially. Thus, initial management of
cell an lymphocytic inltrate. Laboratory values reveal necrotizing pancreatitis is conservative with the avoiance of
increase levels of IgG an often iabetes. Antiboies against early invasive interventions. FNA with culture might be con-
lactoferrin an carbonic anhyrase have been reporte, but siere later (because infecte necrosis typically evelops
they are not a specic ning. The treatment of choice is weeks later) in the course of the hospitalization if the patient
steroi therapy, an the isease respons well to this man- were to manifest evience of sepsis such as leukocytosis,
agement. Chemotherapy or invasive surgical/enoscopic tachycaria, refractory abominal pain, bacteremia, an/or
proceures are not necessary (A, C–E). persistent fevers (A). Prophylactic antibiotics for severe pan-
References: Ketikoglou I, Moulakakis A. Autoimmune pancre- creatitis shoul not be routinely aministere (B). In patients
atitis. Dig Liver Dis. 005;37(3):11–15. with proven (via neele aspiration) infecte necrosis, min-
Okazaki K. Autoimmune-relate pancreatitis. Curr Treat Options imally invasive percutaneous or enoscopic interventions
Gastroenterol. 001;4(5):369–375. (step-up approach) followe by vieo-assiste retroperito-
neal ebriement with the goal of postponing or obviating
27. A. This presentation is concerning for elaye blee- the nee for open surgery is preferre (E). Furthermore, early
ing following a pancreaticouoenectomy (Whipple) pro-
necrosectomy has been shown to increase morbiity an
ceure. This is most often ue to a gastrouoenal artery
mortality when compare with elaye intervention (C). In
stump leak. Flui with HU >5 is most consistent with
a patient that oes not appear to have an infecte necrotizing
bloo. CT may show a pseuoaneurysm, but this may not
pancreatitis, it is appropriate to approach management con-
always be present. On hospital ay 5, the tissue planes are
servatively with meical optimization an repeat CT scan if
often fragile, making it ifcult to control bleeing in the
there is a eterioration in clinical status. It is best to allow the
operating room (B). After resuscitation with bloo proucts,
patient to manifest the severity of the isease before invasive
the most appropriate next step involves performing an angi-
interventions.
ography with embolization. A bleeing ulcer is also in the
References: Bugiantella W, Ronelli F, Boni M, et al. Necrotiz-
ifferential, but less likely in the absence of blooy nasoga- ing pancreatitis: a review of the interventions. Int J Surg. 016;8
stric tube output an with CT nings, so upper enoscopy Suppl 1:S163–S171.
is not likely to be helpful (E). Esophagogastrouoenoscopy Mier J, León EL, Castillo A, Robleo F, Blanco R. Early versus
(EGD) nees to be selectively performe this early after sur- late necrosectomy in severe necrotizing pancreatitis. Am J Surg.
gery because the scope may compromise the freshly mae 1997;173():71–75.
CHAPtEr 5 Abdomen—Pancreas 63

30. A. Although elevation of alkaline phosphatase can be 31. E. Soli pseuopapillary epithelial neoplasm is A
seen with a resiual common bile uct stone, the best pre- rare tumor occurring almost exclusively in young women.
ictor is a persistent elevation of the total bilirubin (C). Amy- It has low malignant potential an for the majority of
lase is not typically elevate in this patient population (D). patients, the tumor can be resecte with curative intent
Because the pathophysiology of gallstone pancreatitis is tran- regarless of the size. Metastasis an recurrence are
sient obstruction of the ampulla of Vater by a gallstone, a sig- uncommon. Serous cystic aenoma also occurs most com-
nicant number of patients will have some egree of common monly in women, but this has no malignant potential an
bile uct ilation on amission; as such, common bile uct oes not nee to be resecte unless it is causing mass effect
ilation is not a specic ning (B). This iffers from patients (A). Mucinous cystic neoplasm is consiere a premalig-
with symptomatic cholelithiasis, in which uctal ilation is nant lesion, has a female preominance, occurs commonly
frequently associate with common uct stones. Persistent in the boy or tail of the pancreas, an shoul always
abominal pain can occur as a result of multiple etiologies unergo resection (B). Main-uct IPMN has a high risk
an shoul be appropriately worke up with history an of harboring malignant cells an shoul be resecte (D).
physical, laboratory stuies, an/or imaging, if necessary (E). Sie-uct IPMN can be manage conservatively unless it
References: Chan T, Yaghoubian A, Rosing D, et al. Total bili- is symptomatic, larger than 3 cm, or associate with mural
rubin is a useful preictor of persisting common bile uct stone in noules (C).
gallstone pancreatitis. Am Surg. 008;74(10):977–980. Reference: Frost M, Krige JE, Bornman PC. Soli pseuopapil-
Chang L, Lo SK, Stabile BE, Lewis RJ, e Virgilio C. Gallstone lary epithelial neoplasm—A rare but curable pancreatic tumour in
pancreatitis: a prospective stuy on the incience of cholangitis an young women. S Afr J Surg. 011;49():78–81.
clinical preictors of retaine common bile uct stones. Am J Gastro-
enterol. 1998;93(4):57–531.
Abdomen—Spleen
MARIA G. VALADEZ, BENJAMIN DIPARDO, AND ERIC R. SIMMS 6
ABSITE 99th Percentile High-Yields
I. Anatomy an Physiology
A. White pulp contains macrophages an both B an T lymphocytes
B. Re pulp is responsible for removing eforme or abnormal RBCs an nuclear remnants foun in RBCs
C. Splenocolic, gastrosplenic, splenorenal, an phrenicosplenic ligaments
1. Short gastric arteries are foun in the gastrosplenic ligament an can be a source of postoperative
hemorrhage
. Splenic artery lies anterior an superior to the splenic vein
3. Lack of normal peritoneal attachments results in a wanering spleen
D. Accessory spleen
1. Suspecte if peripheral bloo smear not consistent with asplenia after splenectomy or if recurrence of
primary pathology; splenic hilum is the most common location followe by tail of pancreas

II. Splenic masses


A. Most common benign splenic tumor: hemangioma
B. Most common primary splenic tumor: non-Hogkin lymphoma
C. Parasitic cysts (most common worlwie but rare in the Unite States):
1. Majority are hyati cysts seconary to echinococcus; treate with partial or total splenectomy ue to
risk of rupture
D. Nonparasitic cysts:
1. Cysts can be true cysts or pseuocysts, but this ifferentiation is ifcult to make preop; true cysts
or primary cysts have epithelial lining an are congenital; pseuocysts or seconary cysts lack an
epithelial lining an typically result from traumatic hematoma formation
. If asymptomatic, can be observe with serial imaging regarless of size; if symptomatic, treate with
partial or total splenectomy
E. Splenic abscesses
1. Etiology: bacteremia, trauma, hemoglobinopathies, splenic artery embolization, following acute
pancreatitis, immunosuppression, or trauma
. Most common organism: Streptococcus pneumoniae
3. Treatment: IV antibiotics followe by splenectomy (gol stanar)
4. If poor surgical caniate with thick-walle unilocular abscess, treat with percutaneous rainage

III. Splenectomy
A. Vaccinations
1. Vaccinate against encapsulate organisms: Streptococcus pneumoniae, Neisseria meningitidis, an
Haemophilus inuenzae; ieally  weeks before surgery;  weeks after if emergent

65
66 PArt i Patient Care

. Pneumococcal (PPSV3) vaccine shoulbegiven at least 8 weeks after the PCV13 vaccine with
revaccination at 5 years; meningococcal (MenACWY) vaccination shoul be given 8 weeks after
initial ose with revaccination every 5 years; also require yearly inuenza an COVID vaccination
B. Postsplenectomy consierations
1. Overwhelming postsplenectomy sepsis (OPSI) (<% of patients ue to loss of immunoglobulin M
[IgM])
a) Most cases occur early postsplenectomy, within rst  years, an most common in younger
patients unergoing splenectomy for hematologic isease; trauma splenectomy associate with
lowest risk of OPSI
b) Most common cause: Streptococcus pneumoniae infection
c) Treatment: thir-generation cephalosporins
. Peripheral smear nings:

Abnormality Description
Howell-Jolly bodies Nuclear remnants
Pappenheimer bodies Iron deposits
Heinz bodies Denatured hemoglobin
Target cells Thickened RBC membrane

IV. Hematologic Diseases

Condition Pathophysiology Management


Platelet Idiopathic Antiplatelet IgG produced by spleen Steroids ęrst, followed by IVIG;
disorders thrombocytopenic splenectomy for refractory cases
purpura (ITP)
Thrombotic Metalloproteinase deęciency causes Plasmapheresis is ęrst line;
thrombocytopenic defective vWF multimer cleaving splenectomy for refractory cases
purpura (TTP)
Hemolytic Hereditary Spectrin, ankyrin band 3 protein, Splenectomy is curative (if pigmented
anemias spherocytosis protein 4.2 abnormalities gallstones present, also need
prophylactic cholecystectomy)
Elliptocytosis Membrane protein defect Splenectomy rarely done
Pyruvate kinase Abnormal glucose metabolism Splenectomy not indicated
deęciency
G6PD deęciency X-linked recessive Splenectomy not indicated
Crises precipitated by drugs, fava
beans, etc.
Sickle cell anemia Hemoglobin defect causes sickling Splenectomy rarely indicated
Thalassemia Alpha or beta globulin defect Splenectomy indicated for symptomatic
interferes with RBC survival/ splenomegaly, high transfusion
production requirements
Autoimmune Autoantibodies to RBC antigens Cold: no splenectomy
hemolytic anemias Cold vs warm Warm: splenectomy if refractory to
steroid treatment
CHAPtEr 6 Abdomen—Spleen 67

Questions
1. A 40-year-ol female with ITP is about to unergo 3. A 1-year-ol boy presents with ecchymosis
a splenectomy. Her preoperative platelet count an fever of 101.°F. Laboratory exam is
is 40,000 cells/μL. Which of the following is true remarkable for platelet count of 30,000 cells/
regaring perioperative platelet transfusions for μL an hemoglobin of 8. mg/L. Peripheral
ITP uring splenectomy in this patient? bloo smear shows large an immature platelets.
A. Transfuse  units of platelets en route to OR Review of systems is signicant for an upper
B. Transfuse  units of platelets postoperatively respiratory tract infection three weeks prior. His
even if no intraoperative bleeing mother also notes that his urine has been pink.
C. Transfuse platelets upon clamping an ligating This is his secon amission for this constellation
the splenic vein even if no intraoperative of symptoms. Which of the following is true
bleeing regaring this conition?
D. Transfuse platelets following splenic artery A. Chilren are more likely to require
ligation even if no intraoperative bleeing splenectomy than aults
E. Transfuse platelets following splenic B. In chilren, intravenous immunoglobulin is
artery ligation if patient continues to have the initial approach to management
intraoperative bleeing C. The spleen is typically palpable on abominal
examination
2. A 9-year-ol male who unerwent emergent D. Chilren with platelet counts of 50,000 or
splenectomy for blunt abominal trauma fewer cells/μL shoul be hospitalize
presents to the emergency epartment 3 weeks E. In chilren, this conition is often precee by
postoperatively complaining of progressive left a viral illness
upper quarant abominal pain an fever. On
evaluation, he has a temperature of 38.5°C, a 4. Which of the following is the best inication for
heart rate of 1 bpm, an a bloo pressure of splenectomy?
11/68 mmHg. Labs emonstrate a white bloo A. Sarcoiosis
cell count of 1,000 cells/μL an a compute B. Gaucher isease
tomography (CT) scan shows a 7-cm ui C. Myelobrosis
collection in the left upper quarant. Which of the D. Hairy cell leukemia with neutropenia
following is the most likely iagnosis? E. Seconary hypersplenism in a cirrhotic patient
A. Iatrogenic colon perforation
B. Pancreatic stula 5. A 30-year-ol woman is foun to have a signet
C. Portal vein thrombosis ring calcication in the left upper quarant on a
D. OPSI plain abominal raiograph. A CT scan conrms
E. Postoperative hemorrhage a -cm splenic artery aneurysm just beyon the
take-off of the celiac axis. The pancreas appears
normal. Which of the following is true regaring
this conition?
A. It is an uncommon visceral artery aneurysm
B. In this patient, it is most likely a
pseuoaneurysm
C. It is associate with a ouble-rupture
phenomenon
D. The aneurysm typically arises in the proximal
portion of the splenic artery
E. Intervention is not neee
68 PArt i Patient Care

6. Two months after a splenectomy for ITP, the 11. A 7-year-ol girl with hemolytic anemia who has
patient is note to have petechiae an a ecrease faile conservative management is scheule for
in platelet count. A peripheral bloo smear is an elective splenectomy. Which of the following is
noteworthy for the absence of Howell-Jolly true regaring her conition?
boies. Which of the following is the best A. Preoperative right upper quarant
recommenation for a workup? ultrasonography shoul be performe
A. CT scan of the abomen B. An intraoperative search for accessory splenic
B. Bone marrow biopsy tissue is not necessary
C. No workup neee; aminister sterois C. The most common intraoperative complication
D. Raiolabele RBC scan is injury to the pancreas
E. No workup neee; aminister D. Open splenectomy shoul be performe
immunoglobulin E. Surgery shoul be elaye until 10 years of
age
7. Which of the following is true regaring ITP?
A. In aults, splenectomy shoul be performe 12. A 35-year-ol alcoholic male with human
once the iagnosis is establishe immunoeciency virus (HIV) unergoes a
B. A chronic form is more likely to evelop in splenectomy after being involve in a motor
aults than in chilren vehicle crash. Which of the following is true?
C. The iagnosis is effectively establishe by a A. The primary risk of OPSI is within the rst
peripheral bloo smear year after splenectomy
D. Immunoglobulin is ineffective in increasing B. Suspecte OPSI shoul initially be manage
the platelet count with a uoroquinolone
E. In aults, splenectomy shoul be elaye until C. The majority of OPSI cases are ue to
after the secon relapse Haemophilus inuenzae
D. Daily prophylactic antibiotic is recommene
8. Which of the following is true regaring TTP? E. Loss of immunoglobulin G (IgG) is what
A. It oes not lea to hemolysis preisposes postsplenectomy patients to OPSI
B. It is associate with liver failure
C. Splenectomy is the rst line of treatment in 13. Which of the following inications for
aults splenectomy poses the highest risk of
D. The Coombs test result is positive postsplenectomy sepsis?
E. The most common cause of eath is A. Trauma
intracerebral hemorrhage B. ITP
C. Hereitary spherocytosis
9. Which of the following is least likely to be seen in D. Thalassemia major
a postsplenectomy patient? E. Hereitary elliptocytosis
A. Erythrocytes containing iron eposits
B. Irregularly shape an fragmente RBCs 14. A 3-year-ol female with rheumatoi arthritis
C. Persistent monocytosis presents for evaluation of recurrent infections.
D. Acanthocytes Physical exam is signicant for splenomegaly.
E. Erythrocytes containing nuclear fragments Laboratory exam emonstrates marke
neutropenia. Which of the following is true
10. A 50-year-ol male has an incientally iscovere concerning this conition?
8-cm nonparasitic splenic cyst. Which of the A. Splenectomy is the initial treatment of choice
following is true about this conition? B. There is a tenency for upper extremity ulcers
A. Splenectomy shoul be performe to form in this patient population
B. Most are symptomatic an present with left C. The neutrophil count oes not improve with
upper quarant tenerness surgical intervention
C. It may secrete CA 19-9 D. Patients have antiboies against neutrophil
D. The patient shoul unergo percutaneous nuclei
aspiration E. Corticosterois are contrainicate
E. It is a common inciental ning
CHAPtEr 6 Abdomen—Spleen 69

15. Which of the following is true regaring 18. The most common inication for elective
hereitary spherocytosis? splenectomy is:
A. It is transmitte as an autosomal recessive trait A. Staging for Hogkin lymphoma
B. The spleen is typically smaller than normal B. Hereitary spherocytosis
C. Spherocytosis on bloo smear improves C. ITP
following splenectomy D. TTP
D. It is associate with leg ulcers E. Autoimmune hemolytic anemia
E. A positive irect Coombs test result conrms
the iagnosis 19. In comparing laparoscopic with open
splenectomy for hematologic isorers, which of
16. After splenectomy for a myeloproliferative isorer, the following is true?
a 40-year-ol woman presents with anorexia, A. Open splenectomy has better long-term results
abominal pain, an a low-grae fever. Her white with respect to response rates
bloo cell (WBC) count is 14,000 cells/μL an her B. The length of hospital stay is the same
platelet count is 500,000 cells/μL. A noncontrast C. The operative mortality rate is lower with
CT scan reveals iffuse small bowel eema an laparoscopic splenectomy
mil ascites. The most likely iagnosis is: D. Laparoscopic splenectomy has emerge as the
A. OPSI stanar of care
B. Portal vein thrombosis E. Laparoscopic splenectomy is frequently
C. Primary peritonitis associate with increase cost to the patient
D. Ischemic colitis
E. Perforate uoenal ulcer

17. The most common cause of spontaneous splenic


rupture worlwie is:
A. Leukemia
B. Malaria
C. Hemophilia
D. Hemolytic anemia
E. Hogkin lymphoma

Answers
1. E. Splenectomy for ITP can be safely performe in most shoul be consiere in a patient with fever an abominal
patients without the nee for platelet transfusions. Trans- pain after splenectomy. Although PVT might cause a fever,
fusing platelets preoperatively oes not reuce transfusion it woul not be associate with a left upper quarant ui
requirements intraoperatively (A). Platelet transfusion is collection (C). The iagnosis of PVT can be mae with CT
most effective after the splenic artery is ligate because the imaging emonstrating portal vein ilation an a lling
newly transfuse platelets are not at risk for sequestration (B, efect or with Duplex scan. OPSI is a life-threatening coni-
C). Splenic vein ligation woul not prevent platelet seques- tion cause by absent IgM, which requires prompt treatment
tration (C). Transfusions can be avoie if there is no intra- with broa-spectrum antibiotics. While a high inex of suspi-
operative bleeing (D). cion for OPSI shoul be maintaine in this patient with fever
Reference: Goel R, Ness PM, Takemoto CM, Krishnamurti L, after splenectomy, the left upper quarant ui collection
King KE, Tobian AAR. Platelet transfusions in platelet consumptive an graually progressive course make pancreatic stula a
isorers are associate with arterial thrombosis an in-hospital more likely iagnosis (D). Iatrogenic colon perforation woul
mortality.Blood. 015;15(9):1470–1476. present in the immeiate postoperative perio with systemic
signs of infection an worsening iffuse abominal pain
2. B. This patient presenting with fever, tachycaria, leu- (A). Similarly, postoperative hemorrhage will present in the
kocytosis, an a left upper quarant ui collection after immeiate postoperative perio with tachycaria an hypo-
emergent splenectomy likely has a pancreatic stula from tension requiring bloo proucts an possibly angioemboli-
iatrogenic injury to the tail of the pancreas, which lies in zation or surgical exploration (E). The most common source
the splenorenal ligament (B). Portal vein thrombosis (PVT) of bleeing after splenectomy is the short gastric arteries.

AL GRAWANY
70 PArt i Patient Care

3. E. ITP is an autoimmune isorer cause by the for- 5. C. Splenic artery aneurysms are the most common vis-
mation of antiplatelet IgG autoantiboies prouce in the ceral artery aneurysms (A). Women are four times more
spleen. Platelets are opsonize by the antiplatelet antiboies likely to be affecte than men. The aneurysm usually arises
an are then remove prematurely, leaing to the low plate- in the mile to istal portion of the splenic artery (D). The
let count. In aults, it is two to three times more common risk of rupture is very low an is likely epenent on size
in women, whereas it occurs with equal frequency in boys an hormonal inuences. Once rupture occurs, the mortality
an girls. Patients typically present with ecchymoses or pete- rate ranges from 35% to 50%. Splenic artery aneurysm is par-
chiae. Others may exhibit minor bleeing from the gums or ticularly problematic in pregnancy because rupture imparts
nose, excessive menstruation, or bloo in the stool or urine. a risk of mortality to both mother an fetus. Most patients
Life-threatening bleeing as an initial presentation is uncom- are asymptomatic an seek meical attention base on an
mon. In chilren, the presentation is often precee by a inciental raiographic ning (a ring-like calcication on a
viral illness. The spleen is usually not enlarge (C). The iag- plain abominal raiograph locate in the left upper qua-
nosis is one of exclusion an is base on the history, physi- rant). Inications for treatment of true aneurysms inclue the
cal examination, complete bloo count, an examination of presence of symptoms, pregnancy, an women of chilbear-
the peripheral smear, which shoul exclue other causes of ing age who inten to become pregnant. Pseuoaneurysms
thrombocytopenia. The peripheral bloo smear frequently are usually associate with inammatory processes, are
shows large, immature platelets. Bone marrow aspiration inherently unstable, an thus shoul be treate. For asymp-
is not routinely use but is appropriate in patients over the tomatic patients, size greater than  cm is an inication for
age of 60 an in patients consiering splenectomy. The bone surgery. Most splenic artery aneurysms can be observe;
marrow aspirate shows normal or increase megakaryo- however, because this woman is of chilbearing age, treat-
cytes. The management epens on the age of the patient, ment woul be inicate (E). The majority of splenic artery
the platelet count, an the severity of symptoms. In chilren, aneurysms are true aneurysms (B). Pseuoaneurysms occur
the majority present with mil cases that are self-limite most commonly in association with an episoe of severe
an o not nee any meical therapy (A–B). In fact, chil- pancreatitis with erosion into the vessel. The patient pre-
ren with platelet counts greater than 30,000 cells/μL shoul sente has no evience of pancreatitis. Splenic artery aneu-
not be hospitalize an o not routinely require treatment rysms are associate with a ouble-rupture phenomenon in
if they are asymptomatic or have only minor purpura (D). which there is an initial heral blee into the lesser sac an
In aults, that threshol is greater than 0,000/μL. The then rupture into the peritoneal cavity.
rst line of therapy is oral prenisone at a ose of 1 to
1.5 mg/kg/ay. Another effective therapy is intravenous 6. D. When a recurrence of a platelet count ecrease after
(IV) immunoglobulin, which is use if corticosterois are splenectomy for ITP evelops in a patient, one must consier
ineffective. Splenectomy is inicate for failure of meical the possibility of an accessory spleen that was misse. The
therapy, for prolonge use of sterois with sie effects, an presence of an accessory spleen is suggeste by the absence
for most cases of a rst relapse, particularly if there is pre- of Howell-Jolly boies on a peripheral bloo smear. This
operative bleeing. Patients with low platelet counts of less patient nees to be appropriately worke up starting with
than 10,000/μL shoul have platelets available for surgery raionuclie imaging to etermine if an accessory spleen is
but shoul not receive them preoperatively because they will present (C, E). The sensitivity of CT scan in ientifying an
be consume. Platelets shoul be given for those who con- accessory spleen is 60% (A). Bone marrow biopsy has no role
tinue to blee after ligation of the splenic peicle. The one (B). Ientication of an accessory spleen in a patient who
exception is if there is preoperative bleeing; platelets can be remains severely thrombocytopenic warrants surgical exci-
given before or at the time of incision uring splenectomy. sion of the accessory spleen. Rituximab may also be consi-
Urgent splenectomy plays a role in severe, life-threatening ere in this patient population.
bleeing, in conjunction with meical therapy in both aults References: Quah C, Ayiomamitis GD, Shah A, Ammori BJ.
an chilren. Splenectomy provies a permanent response Compute tomography to etect accessory spleens before laparo-
in 75% to 85% of patients. scopic splenectomy: is it necessary? Surg Endosc. 011;5(1):61–65.
Reference: George JN, Woolf SH, Raskob GE, et al. Iiopathic Ghanima W, Khelif A, Waage A, et al. Rituximab as secon-line
thrombocytopenic purpura: a practice guieline evelope by treatment for ault immune thrombocytopenia (the RITP trial): a
explicit methos for the American Society of Hematology. Blood. multicentre, ranomise, ouble-blin, placebo-controlle trial.
1996;88(1):3–40. Lancet. 015;385(9978):1653–1661.

4. D. General inications for splenectomy inclue symp- 7. B. Aults are more likely to get a chronic, more insiious
tomatic splenomegaly, hypersplenism, hemolytic anemia, form of ITP than chilren. In aults, women are affecte two
thrombocytopenia, or other cytopenia. Splenectomy is not to three times more often than men, whereas, in chilren, it
inicate for sarcoiosis, Gaucher isease, or myelobrosis, is equally common in boys an girls. The iagnosis of ITP is
unless they have hypersplenism (A–C). Splenectomy is not one of exclusion. The peripheral bloo smear shows a low
inicate for patients with portal hypertension (E). Hairy platelet count as well as large, immature platelets but oes
cell leukemia gets its name from hair-like cytoplasmic pro- not establish the iagnosis (C). IV immunoglobulin ther-
jections in lymphocytes that are seen on a peripheral smear. apy is effective in both chilren an aults in increasing the
Treatment is with chemotherapy, but splenectomy is useful platelet count (D). In aults, splenectomy is inicate for
in increasing cell counts, improving pain, an early satiety. failure of meical therapy (sterois, immunoglobulin), for
With newer chemotherapeutic agents, the role of splenec- prolonge use of sterois beyon 3 to 6 months, an for most
tomy is ecreasing. cases of a rst relapse (A, E).
CHAPtEr 6 Abdomen—Spleen 71

8. E. The rst line of treatment for TTP is plasma exchange be consiere. Laparoscopic splenectomy has emerge as
by removing the patient’s plasma an exchanging it with the gol stanar for most chilren (D). Intraoperatively,
fresh-frozen plasma (C). Splenectomy is not very effective before removal of the spleen, there shoul always be a search
in TTP an shoul be use as salvage therapy in refractory for an accessory spleen, particularly in a patient with a hema-
cases. Features of TTP inclue thrombocytopenia, microangio- tologic inication for splenectomy (B). There is no nee to
pathic hemolytic anemia, an neurologic complications. The elay surgery until 10 years of age (E). Most surgeons agree
pathophysiology involves abnormal platelet clumping, likely that the minimum accepte age is 5 years, but there have
ue to large multimers of von Willebran factor, which results been reports of splenectomy in patients as young as  years
in thrombotic episoes in the microvascular circulation. The ol. Although the pancreatic tail is at risk of injury, the most
narrowe lumens in the microvascular circulation lea to common intraoperative complication is hemorrhage that can
increase shear stress on re bloo cells (RBCs), causing them occur uring hilar issection (C).
to lyse (A). Symptoms an signs inclue petechiae; fever; References: Sheng J, Wu Y. A report of two cases of splenectomy
neurologic symptoms such as heaaches, seizures, an even in chilren younger than two years ol with hereitary spherocyto-
coma; an renal failure (B). The peripheral bloo smear shows sis. J Pediatr Surg Case Rep. 015;3():84–86.
schistocytes, nucleate RBCs, an basophilic stippling. The Vecchio R, Intagliata E, Marchese S, La Corte F, Cacciola RR, Cac-
ciola E. Laparoscopic splenectomy couple with laparoscopic chole-
most common cause of eath is intracerebral hemorrhage. TTP
cystectomy. JSLS. 014;18():5–57.
can be istinguishe from autoimmune hemolytic anemia, in
that the result of the Coombs test is negative in TTP (D).
Reference: Coppo P, Froissart A, French Reference Center for 12. D. OPSI is a signicant concern in the asplenic patient
Thrombotic Microangiopathies. Treatment of thrombotic thrombo- an can occur in 0.05% to % of postsplenectomy patients. It
cytopenic purpura beyon therapeutic plasma exchange. Hematology is ue to loss of IgM (E). These patients continue to be at an
Am Soc Hematol Educ Program. 015;(1):637–643. increase risk many years after splenectomy (A). Manage-
ment of OPSI requires prompt ientication an initiation
9. B. After splenectomy, target cells, Howell-Jolly boies of supportive care with a thir-generation cephalospo-
(erythrocytes containing nuclear fragments), Heinz boies, rin (B). The majority of OPSI cases are ue to Streptococcus
Pappenheimer boies (erythrocytes containing iron epos- pneumoniae (C), followe by H. inuenzae type B, Neisseria
its), an spur cells (acanthocytes) are seen (A, D–E). These meningitides, an group A streptococcus. Daily prophylactic
inclusions (boies) are normally pitte by the spleen. Leuko- antibiotic use is inicate for chilren younger than 5 an
cytosis, persistent monocytosis, an increase platelet counts for immunocompromise patients because they may not
commonly occur after splenectomy as well (C). The increase be able to mount an appropriate response to pneumococcal
in WBC count is primarily mature neutrophils. The white vaccination. Asplenic patients may also have mil egrees
bloo cell (WBC) count typically increases within 1 ay after of thrombocytosis an leukocytosis, Howell-Jolly boies in
splenectomy but may remain elevate for as long as several RBCs, an an increase number of target cells. Howell-Jolly
months. Asplenic patients have been foun to have subnor- boies are nuclear remnants in circulating erythrocytes that
mal IgM levels. The spleen is a major site of prouction for appear basophilic (blue). Normally, erythrocytes expel their
the opsonins properin an tuftsin, an splenectomy results DNA before exiting the bone marrow.
in ecrease serum levels of these proteins. Schistocytes References: Fishman D, Isenberg DA. Splenic involvement in
(irregularly shape an fragmente RBCs) are pathologic rheumatic iseases. Semin Arthritis Rheum. 1997;7(3):141–155.
an inicate either isseminate intravascular coagulation Piliero P, Furie R. Functional asplenia in systemic lupus erythe-
or traumatic hemolytic anemia (such as TTP). matosus. Semin Arthritis Rheum. 1990;0(3):185–189.
Theilacker C, Luewig K, Serr A, et al. Overwhelming postsple-
10. C. Nonparasitic splenic cysts are rare (E). They are most nectomy infection: A prospective multicenter cohort stuy. Clin
commonly asymptomatic, but when patients have symptoms, Infect Dis. 016;6(7):871–878.
they frequently complain of left upper quarant tenerness
with referre pain to the left shouler (B). Asymptomatic 13. D. All of the answer choices can lea to postsplenec-
cysts can safely be observe regarless of size (A). Ai- tomy sepsis (A–C, E). The incience an mortality rate for
tionally, percutaneous aspiration is met with high recurrence postsplenectomy sepsis are highest in patients with uner-
rates (D). Patients shoul be manage with observation an lying hematologic conitions, particularly thalassemia major
serial ultrasoun imaging to assess for interval growth. It has an sickle cell isease. Chilren have a higher risk than
been shown that splenic cysts may secrete tumor markers aults. In a large review, the incience of infection after sple-
such as CA 19-9, but they o not have malignant potential. nectomy in chilren (younger than 16 years ol) was 4.4%,
References: Boybeyi O, Karnak I, Tanyel FC, Ciftçi AO, Senocak compare with 0.9% in aults. Severe infection after sple-
ME. The management of primary nonparasitic splenic cysts. Turk J nectomy for benign isease was very uncommon, except in
Pediatr. 010;5(5):500–504. infants an chilren younger than 5 years of age. Patients are
Bresaola V, Pravisani R, Terrosu G, Risaliti A. Elevate serum CA also more susceptible to malaria.
19-9 level associate with a splenic cyst: which is the actual clinical
References: Davison RN, Wall RA. Prevention an manage-
management? Review of the literature. Ann Ital Chir. 015;86(1):–9.
ment of infections in patients without a spleen. Clin Microbiol Infect.
001;7(1):657–660.
11. A. In the peiatric population, preoperative workup Holsworth RJ, Irving AD, Cuschieri A. Postsplenectomy sep-
for hemolytic anemia shoul inclue a right upper quarant sis an its mortality rate: actual versus perceive risks. Br J Surg.
ultrasoun to look for cholelithiasis because these patients are 1991;78(9):1031–1038.
susceptible to eveloping pigment stones. If these are pres- Leonar AS, Giebink GS, Baesl TJ, Krivit W. The overwhelming
ent, concomitant splenectomy an cholecystectomy woul postsplenectomy sepsis problem. World J Surg. 1980;4(4):43–43.
72 PArt i Patient Care

14. D. The tria of rheumatoi arthritis, splenomegaly, of thrombocytosis after splenectomy an the setting of a
an neutropenia is calle Felty synrome. It is present in myeloproliferative isorer. PVT is uncommon (occurrence
3% of patients with rheumatoi arthritis. The pathophys- rate ranging from % to 8%) but not rare, an the greatest
iology involves the coating of the white bloo cell surface risk is in cases involving splenomegaly with a myeloprolifer-
with immune complexes, leaing to their sequestration an ative isorer. Postsplenectomy PVT typically presents with
clearance in the spleen. An increase risk of infections ue anorexia, abominal pain, leukocytosis, an thrombocytosis,
to neutropenia ensues. The size of the spleen can vary from as emonstrate in this patient. A high inex of suspicion,
nonpalpable to massively enlarge. Initial treatment with early iagnosis with contrast-enhance CT, an immeiate
corticosterois typically improves the neutrophil count, but anticoagulation are keys to successful treatment of PVT.
the effects are not always permanent (A, E). Hematopoietic Patients unergoing splenectomy shoul be treate with
growth factors an methotrexate have also been use. There eep venous thrombosis prophylaxis, incluing pneumatic
is a tenency for leg ulcers to form in these patients (B). Other compression evices, an with subcutaneous or low-molec-
inications for splenectomy inclue transfusion-epenent ular-weight heparin. OPSI is an uncommon complication in
anemia an profoun thrombocytopenia. Responses to postsplenectomy patients an may present with nonspecic
splenectomy are excellent, with more than 80% of patients u-like symptoms that rapily progress to fulminant sepsis
showing a urable increase in white bloo cell count. The (A). Primary peritonitis is often a monobacterial infection
neutrophil count typically improves immeiately, although occurring in cirrhotic patients with ascites (C). Ischemic coli-
the relative number of neutrophils may remain subnormal tis presents with left-sie abominal pain an blooy iar-
(C). However, neutrophil function improves. rhea in elerly patients with low-ow states, such as those
with severe ehyration, heart failure, shock, an trauma
15. D. Hereitary spherocytosis (HS) is an RBC membrane (D). Perforate uoenal ulcer initially presents with epigas-
isorer that leas to hemolytic anemia. It is autosomal tric pain, followe by iffuse tenerness, abominal rigiity,
ominant an the most common hemolytic anemia requir- an reboun tenerness (E).
ing splenectomy (A). It is ue to an inherite ysfunction or References: an’t Riet M, Burger JW, van Muiswinkel JM, Kaze-
eciency in one of the RBC membrane proteins (spectrin, mier G, Schipperus MR, Bonjer HJ. Diagnosis an treatment of por-
ankyrin, ban 3 protein, or protein 4.), which causes the tal vein thrombosis following splenectomy: portal vein thrombosis
membrane lipi bilayers to estabilize, leaing to a lack of following splenectomy. Br J Surg. 000;87(9):19–133.
membrane eformability. The spleen sequesters an estroys Winslow ER, Brunt LM, Drebin JA, Soper NJ, Klingensmith
ME. Portal vein thrombosis after splenectomy. Am J Surg.
these noneformable RBCs. Most patients are asymptom-
00;184(6):631–636.
atic, but they may have mil jaunice from hemolysis an
splenomegaly on physical examination (B). Laboratory fea-
tures inclue a mil to moerate anemia, a low mean cor- 17. B. Spontaneous rupture of the spleen is an uncommon
puscular volume, an elevate mean corpuscular hemoglobin ramatic abominal emergency that requires immeiate
concentration, an an elevate re cell istribution with. iagnosis an prompt treatment to ensure the patient’s sur-
Laboratory values also reect the hemolysis an rapi cell vival. Spontaneous rupture rarely occurs in a histologically
turnover with an elevate reticulocyte count, lactate ehy- proven normal spleen an in such cases is calle a true spon-
rogenase, an unconjugate bilirubin. Unlike autoimmune taneous rupture. Spontaneous rupture usually occurs in a
hemolytic anemia, the irect Coombs test result is negative isease spleen an is calle pathologic spontaneous rupture.
in HS (E). In HS, RBCs ten to lyse at lower concentrations of Infectious iseases have been cite in most cases involving
salt than normal. Splenectomy is curative for HS an serves splenic rupture but are rare in hematologic malignancies
as the sole moe of therapy, but patients continue to have espite frequent involvement of the spleen (A, E). Malaria
spherocytosis on bloo smear (C). Due to ongoing re cell is the number one cause worlwie an infectious mono-
lysis, gallstones are common. When gallstones are foun, nucleosis is the number one cause in the Unite States. With
prophylactic cholecystectomy is recommene, particularly malaria, changes in splenic structure can result in hematoma
in chilren. Another feature of HS is leg ulceration, which is formation, rupture, hypersplenism, torsion, or cyst forma-
another inication for early splenectomy. These ulcers heal tion. An abnormal immunologic response may result in mas-
after splenectomy. The cause of the ulceration is unclear but sive splenic enlargement. Spontaneous rupture of the spleen
may be a result of increase bloo viscosity that reuces is an important an life-threatening complication of Plas-
oxygen levels in the leg tissues. Alternatively, recent stuies modium vivax infection but is rarely seen in Plasmodium fal-
suggest that hemolysis leas to nitric oxie resistance, eno- ciparum malaria. Other less frequent causes of spontaneous
thelial ysfunction, an en-organ vasculopathy, as is seen splenic rupture inclue hemolytic anemia, hemophilia,
in sickle cell isease. myeloysplastic isorers, lupus, ialysis, an multiple
Reference: Kato GJ, McGowan V, Machao RF, et al. Lac- myeloma (C, D).
tate ehyrogenase as a biomarker of hemolysis-associate nitric Reference: Hamel CT, Blum J, Harer F, Kocher T. Nonopera-
oxie resistance, priapism, leg ulceration, pulmonary hyper- tive treatment of splenic rupture in malaria tropica: review of litera-
tension, an eath in patients with sickle cell isease. Blood. ture an case report. Acta Trop. 00;8(1):1–5.
006;107(6):79–85.
18. C. The most common inication for splenectomy is
16. B. This patient most likely has PVT; it shoul be sus- trauma to the spleen, whether iatrogenic or acciental. In
pecte in patients with fever an abominal pain after the past, staging for Hogkin isease was the most common
splenectomy. This patient is preispose to PVT forma- inication for elective splenectomy (A). ITP is now the most
tion because of her hypercoagulability from a combination frequent inication for splenectomy in the elective setting,
CHAPtEr 6 Abdomen—Spleen 73

followe by HS, autoimmune hemolytic anemia, an TTP that the total cost to the patient is less with the laparoscopic
(B, D–E). proceure ue to the shortene hospital stay (E). The lapa-
Reference: Schwartz SI. Role of splenectomy in hematologic roscopic approach has emerge as the stanar for nontrau-
isorers. World J Surg. 1996;0(9):1156–1159. matic elective splenectomy.
References: Beauchamp RD, Holzman MD, Fabian TC. Spleen.
19. D. The laparoscopic approach typically results in lon- In: Townsen CM, Jr, Beauchamp RD, Evers BM, Mattox KL, es.
ger operative times, shorter hospital stays, an lower mor- Sabiston textbook of surgery: the biological basis of modern surgical prac-
biity rates (B, E). It has similar bloo loss an mortality tice. 17th e. Philaelphia: W.B. Sauners; 004:1679–1710.
rates compare with open splenectomy (A, C). Cost analysis Parks AE, McKinlay, R. Spleen. In: Brunicari FC, Anersen DK,
reveals that higher operating room charges are seen with lap- Billiar TR, etal., es. Schwartz’s principles of surgery. 8th e. New
York: McGraw-Hill; 005:197–1318.
aroscopic splenectomy. However, several stuies have foun
Alimentary Tract—
Esophagus
AMANDA C. PURDY AND ERIC R. SIMMS 7
ABSITE 99th Percentile High-Yields
I. Anatomy an Surgical Approaches
A. Bloo supply:
1. Cervical esophagus→inferior thyroi artery
. Thoracic esophagus → irect branches off thoracic aorta
3. Abominal esophagus → branches from the left gastric an inferior phrenic arteries
B. Surgical approaches:
1. Cervical esophagus: left cervical incision
. Proximal an mi-thoracic esophagus: right posterolateral thoracotomy
3. Distal thoracic esophagus: left posterolateral thoracotomy
II. Hiatal Hernias
Types of Hiatal Hernias

Type Description Management


I Sliding hiatal hernia • If asymptomatic, no treatment indicated
GE junction slides above the diaphragm • If patient has GERD, can treat medically and consider
fundoplication and hiatal hernia repair if medical
management fails
II Paraesophageal hernia • Elective hiatal hernia repair and fundoplication
The fundus herniates through the hiatus recommended to reduce risk of future complications
(gastric volvulus, incarceration/strangulation)
III Sliding + paraesophageal hernia
Combination of types 1 and 2
*More common than type 2
IV Paraesophageal hernia that includes organs
besides the stomach

A. Gastric Volvulus
1. Gastric volvulus is a complication of large hiatal hernias, where the stomach herniates into the chest an
volvulizes;canlea to gastric ischemia an necrosis
. Borchart tria is present in 70% of acute gastric volvulus: severe epigastric pain, inability to vomit,
inability to pass nasogastric tube
3. First steps of management are NG tube ecompression an IV ui resuscitation; NG ecompression
may resolve volvulus; if NG tube cannot be place, patients nee emergent surgery

III. Barrett Esophagus (BE)


A. Intestinal metaplasia (stratie squamous epithelium → columnar epithelium with goblet cells) of the
istal esophagus seconary to aci reux; increase risk of esophageal aenocarcinoma
B. During the initial enoscopy:
75
76 PArt i Patient Care

1. If NO mucosal irregularities or noules are seen, procee with 4-quarant biopsies every  cm
within the segment of BE
. If any mucosal irregularities or noules ARE seen, enoscopically resect the irregular area(s) an obtain
4-quarant biopsies every 1 cm within the segment of BE
C. Meical management of GERD (PPI) & one of the following (base on enoscopy nings):

Pathology Finding Management


No dysplasia • Consider surveillance endoscopy with 4-quadrant biopsies every 2 cm within the segment of
BarreĴ esophagus every 3–5 years
Low-grade dysplasia • Endoscopic eradication (radiofrequency ablation, photodynamic therapy, or cryotherapy)
OR
• Surveillance endoscopy in 3–6 months with 4-quadrant biopsies every 1 cm within the
segment of BarreĴ esophagus
High-grade dysphasia • Endoscopic mucosal resection*Classically, high-grade dysplasia was treated with
esophagectomy; however, endoscopic resection is a much less invasive acceptable alternative

IV. Motility Disorers


A. Achalasia (classic tria: ysphagia [most common], regurgitation, weight loss)
1. Due to the progressive egeneration of ganglion cells in the myenteric plexus in the esophageal wall
. Esophagram: may see “bir’s beak” appearance
3. Manometry: no peristalsis in the esophageal boy an a hypertonic lower esophageal sphincter that
fails to completely relax uring swallowing
4. Management:
a) Heller myotomy: best option (to resolve the obstruction at the LES); myotomy of the inner circular
an outer longituinal muscles, extening 6 cm up the esophagus an  cm onto the stomach with
partial funoplication (to prevent subsequent reux) (incomplete myotomy may occur ue to
inaequate mobilization of GE junction)
b) Pneumatic enoscopic ilation: appropriate in patients who are not surgical caniates; will likely
nee repeate ilations over time
c) POEM (per oral enoscopic myotomy)
B. Divies ONLY circular muscle bers (cricopharyngeus), longer myotomy
C. Avantages: less invasive with similar symptom relief as Heller myotomy
D. Disavantages: technically emaning with long learning curve; up to 50% of patients can have aci
reux following the proceure
a) Nonoperative options (for poor operative risk patients, lose efcacy over time): calcium channel
blockers, nitrates, an botulism toxin injections; botulism toxin injections only work for a few months,
an shoul be avoie in patients who are surgical caniates as they may isrupt surgical planes
E. Zenker iverticulum (ysphagia, coughing, spontaneous regurgitation an halitosis)
1. A false (pulsion) iverticulum through Killian triangle ue to a failure of the cricopharyngeus muscle
to relax uring swallowing
. Management options:
a) Open surgery: cricopharyngeal myotomy +/− iverticulectomy or iverticulopexy; the most
important aspect is the myotomy, which xes the unerlying problem; if the iverticulum is large
(>5 cm) an causing mass effect, resection recommene
b) Enoscopic esophagoiverticulostomy: uses stapler or energy evices to ivie the
cricopharyngeus enoscopically; ONLY for large iverticula >3 cm (if iverticulum too small, the
cricopharyngeal myotomy will be incomplete)
c) Open surgery has lower recurrence rates an slightly higher
complication rates than enoscopic management

V. Esophageal Tumors
A. Leiomyoma (hypoechoic mass on enoscopic ultrasoun, more common in males)
1. Most common benign tumor of the esophagus, arise from the smooth muscle cells (mesenchymal); if
<5 cm, resect enoscopically
. If >5 cm, surgically enucleate (via VATS or laparoscopy)
CHAPtEr 7 Alimentary Tract—Esophagus 77

B. Esophageal cancer
1. Aenocarcinoma an squamous; aenocarcinoma more common in the Unite States
. Use enoscopic ultrasoun to etermine T stage, CT for N stage
3. Localize: limite to the mucosa or invaing lamina propria (an N0, M0)
a) Enoscopic resection for T1a = within the mucosa (to the lamina propria or muscularis mucosa)
b) Esophagectomy for T1b = within the submucosa
c) Neoajuvant chemoraiation rst if >T = to the muscularis propria
4. Regional: (noal isease but M0)
a) Neoajuvant chemoraiation rst, followe by esophagectomy
b) Nee 15 noes for proper oncologic staging
5. Distant (palliative care)
6. Esophagectomy, transhiatal approach (left cervical an abominal incisions) or transthoracic
approach (right thoracotomy an abominal incision); similar outcomes, transhiatal may be
associate with shorter length of hospital stay
7. Conuit choices after esophagectomy inclue the stomach (most common, nee to preserve the right
gastroepiploic artery for perfusion), the colon, an the jejunum
8. Special circumstances
a) Upper esophageal cancer (within 5 cm of the upper esophageal sphincter); chemoraiation as
primary treatment moality
b) Distal esophageal cancer (within 5 cm of GE junction)
(1) Some may originate in esophagus an others in stomach
() Depening on location, may sprea to meiastinal or abominal noes
(3) Most receive neoajuvant or perioperative chemoraiation prior to surgery
VI. Esophageal Perforation
A. Etiologies: iatrogenic by instrumentation (most common at cricopharyngeus), spontaneous (Boerhaave,
occurs 5 cm above GE junction on left), trauma, caustic ingestion
B. Workup: CXR (may see left-sie effusion), esophagram with gastrogran → if negative or inconclusive:
esophagram with thin barium
1. Gastrogran major side effect: pneumonitis if aspirated
. Barium major side effect: severe peritonitis/mediastinitis
C. Management: start with NPO, IV ui resuscitation, broa-spectrum antibiotics incluing antifungals
D. Nonoperative management: if patient hemoynamically stable, not septic, with mil symptoms, an a
containe perforation (minimal meiastinal contamination)
E. Surgical approach: right posterolateral thoracotomy for proximal or mile one-thir, left posterolateral
thoracotomy for lower one-thir
F. Management epens on etiology of perforation
1. Malignant obstruction
a) Early cancer: perform esophagectomy
b) Avance cancer: esophageal stenting
. Benign obstruction
a) If ue to achalasia, perform myotomy on contralateral sie
3. Normal esophagus
G. Repair of perforation
a) Exten myotomy to expose full length of mucosal injury (musical injury often longer than muscle
injury)
b) Debrie all nonviable tissue
c) Two-layer closure (mucosa with absorbable suture an muscle with nonabsorbable suture)
) Reinforce repair with intercostal ap
78 PArt i Patient Care

Fig. 7.1
CHAPtEr 7 Alimentary Tract—Esophagus 79

Questions
1. A 40-year-ol woman with a history of Raynau 4. A 35-year-ol woman is in the recovery room
isease presents for evaluation of ysphagia. after enoscopic ilation of a peptic stricture
Barium esophagram shows reux but no in the mi-thoracic esophagus. She begins to
structural abnormalities, an upper enoscopy complain of chest pain an oynophagia. She is
shows reux esophagitis. Manometry shows hemoynamically stable an chest raiograph
absent peristalsis of the istal esophagus an oes not show any free air or pleural effusion.
ecrease tone of the lower esophageal sphincter. Esophagram with water-soluble contrast is
Initial management consists of: normal. What is the most appropriate next step in
A. Laparoscopic Heller myotomy with partial management?
funoplication A. CT scan of the chest, abomen, an pelvis
B. Proton pump inhibitor an metoclopramie with IV an oral contrast
C. Pneumatic ilation B. Esophagram with thin barium
D. Calcium channel blocker C. Nasogastric tube placement
E. Nissen funoplication D. Left posterolateral thoracotomy
E. Right posterolateral thoracotomy
2. A 63-year-ol man with a history of GERD
presents for progressive ysphagia over the past 5. Which of the following is true regaring the
4months. Esophagram shows an irregular lesion in surgical approach, anatomy, or bloo supply to
the istal esophagus. Enoscopy with biopsy an the esophagus?
enoscopic ultrasoun conrms a  cm irregular A. Outer longituinal muscle is an extension of
mass with invasion into the submucosa. Imaging the cricopharyngeus muscle
oes not show any istant masses or abnormal B. Cervical esophagus is supplie by the inferior
lymph noes. What is the rst step in treatment? thyroi artery
A. Chemotherapy C. The narrowest point in the esophagus is at the
B. Chemoraiation aortic arch
C. Enoscopic resection D. Branches off the intercostal arteries are the
D. Enucleation major bloo supply to the thoracic esophagus
E. Esophagectomy E. The stanar surgical approach to the
miesophagus is a left thoracotomy
3. A 0-year-ol man presents to your ofce
after being hospitalize for lye ingestion. 6. Which of the following statements is true about
Esophagoscopy reveale a high-grae esophageal Mallory-Weiss synrome?
caustic injury, an the patient was treate with A. The chief pathologic ning is spontaneous
supportive care. The patient asks about the long- perforation of the esophagus
term complications after caustic injury to the B. It typically occurs on the right sie
esophagus. Which of the following is true? C. It is usually associate with air in the
A. He is at increase risk for esophageal meiastinum
aenocarcinoma D. Enoscopy shoul be performe to conrm
B. He shoul unergo enoscopic surveillance the iagnosis
beginning 6 weeks after ingestion E. Esophageal balloon tamponae is an
C. The most common complication is an appropriate option in cases of persistent
esophageal stricture bleeing
D. Ault caustic ingestion is less severe than
peiatric ingestion
E. Early use of a neutralizing agent ecreases the
risk of subsequent stricture formation

AL GRAWANY
80 PArt i Patient Care

7. A 40-year-ol female has been using a proton 11. Esophageal manometry performe in a
pump inhibitor (PPI) to control gastroesophageal patient with a true paraesophageal hernia will
reux isease (GERD) for the past 7 years. She emonstrate that the LES is:
is otherwise healthy. She was seen in clinic A. Above the normal position
an eeme a suitable caniate for enitive B. At the normal position
surgical intervention. During the operation, after C. Hypertensive
the phrenoesophageal ligament is mobilize, D. Hypotensive
her istal esophagus is inspecte, an it appears E. Short
shortene. Preoperative upper gastrointestinal
stuy i not ientify a hiatal hernia. Which of 12. Which of the following statements about a
the following will most likely nee to be one? paraesophageal hernia is true?
A. Procee with a stanar Nissen funoplication A. It is associate with anemia
B. Procee with a Dor funoplication B. It oes not pose a risk for incarceration an
C. Perform Collis gastroplasty strangulation
D. Abort the operation an initiate management C. Diagnosis is not reaily mae with upper
with sterois enoscopy
E. Take several biopsies before aborting the D. It is usually cause by a traumatic injury
operation E. It rarely requires operative repair

8. A 51-year-ol male has been unergoing yearly 13. Which of the following will preispose a patient
enoscopy with biopsy for Barrett esophagus to the evelopment of esophageal isease?
(BE). His most recent biopsy emonstrates high- A. LES length of 3 cm
grae ysplasia without noules. Which of the B. Resting LES pressure of 8 mm Hg
following is the best next step in management? C. Resting upper esophageal sphincter (UES)
A. Esophagectomy with reconstruction pressure of 70 mm Hg
B. Repeat enoscopy with biopsy in 3 months D. Abominal length less than 1 cm
C. Enoscopic raiofrequency ablation E. Relaxation of LES with swallowing
D. Antireux operation
E. Oncology referral for consieration of 14. A 5-year-ol male with cirrhosis an known
neoajuvant chemotherapy esophageal varices presents with a large
amount of hematemesis. Which of the following
9. Barrett esophagus: statements is true?
A. Is a congenital abnormality A. Beta blockae is ineffective for preventing
B. Occurs more frequently in women rebleeing
C. When iagnose, shoul be treate with an B. The most important next step is enoscopy for
antireux proceure to prevent cancer both iagnostic an therapeutic interventions
D. Diagnosis requires replacement of a 3-cm C. Prophylactic antibiotics o not improve
segment of the squamous cells by columnar survival
epithelium D. Early aministration of vasoactive rugs oes
E. Features the presence of goblet cells not improve outcomes
E. Enoscopic ban ligation has been
10. Which of the following is true regaring Barrett emonstrate to be superior to enoscopic
esophagus? sclerotherapy
A. PPIs are consiere a more effective treatment
option than H blockers
B. Dietary restrictions such as those use for
patients with GERD are not useful
C. Patients with short- an long-segment Barrett
esophagus have a similar risk of high-grae
ysplasia
D. Use of high-ose PPIs with aspirin is
contrainicate
E. Photofrin is a useful treatment moality
CHAPtEr 7 Alimentary Tract—Esophagus 81

15. A 59-year-ol iabetic male with a history of 18. Which of the following statements is true about
chronic obstructive pulmonary isease (COPD) Zenker iverticulum?
an prior congestive heart failure presents A. It is a true iverticulum
with a -year history of progressively ifcult B. It is best iagnose with esophagoscopy
swallowing. Esophagram emonstrates a ilate C. It is unlikely to cause aspiration
proximal esophagus with abrupt tapering istally. D. It is a pulsion iverticulum
Manometry shows high pressure in the lower E. Small iverticula (<3 cm) are best manage
esophageal sphincter (LES) at rest an failure enoscopically
of the LES to relax after swallowing. Upper
enoscopy is negative. Which of the following is 19. Over the past  years, a 50-year-ol man
true regaring this patient? repeately reporte ifculty swallowing, which
A. The unerlying conition is characterize he escribe as a lump in his throat. He has
by high-amplitue peristaltic waves of the notice expectoration of excess saliva, ysphagia,
esophagus intermittent hoarseness, regurgitation of unigeste
B. Laparoscopic esophagomyotomy with complete foo hours later, an some weight loss. Which of
funoplication is the treatment of choice the following is true of the most likely iagnosis?
C. A trial of calcium channel blockers shoul be A. Swallowing is easiest immeiately after waking
starte up in the morning an gets increasingly
D. Esophageal pneumatic ilation is the next step ifcult throughout the course of the ay
in management B. It is best manage with iverticulectomy alone
E. Peroral enoscopic myotomy (POEM) is the through a left cervical incision
treatment of choice C. It involves an outpouching of the muscularis
propria
16. During the course of an upper enoscopy for D. Esophagectomy will improve survival
manometry conrme achalasia, the enoscopist E. The patient shoul likely be starte on
thinks he may have cause an inavertent chemoraiation
perforation of the left lower istal esophagus.
The patient is stable an shows no signs of 20. A 39-year-ol male presents in clinic to iscuss his
sepsis. Esophagogram conrms a markely care before starting neoajuvant chemoraiation
ilate esophagus with a istal-free perforation. for esophageal cancer. His albumin is .4 mg/
Management consists of: L. Which of the following is true regaring
A. Intravenous (IV) antibiotics, placing patient nutritional optimization for this patient?
NPO (nothing by mouth), an close observation A. He shoul begin parenteral nutrition
B. Left thoracotomy, primary repair, longituinal B. Percutaneous gastrostomy tube shoul not be
myotomy on the contralateral sie offere
C. Laparoscopic primary repair an longituinal C. Esophageal stent placement has been consistently
myotomy on the ipsilateral sie emonstrate to improve nutritional status
D. Esophagectomy with immeiate D. Nasogastric tube insertion has been shown to
reconstruction improve nutritional status
E. Esophageal stent placement E. Stent migration an chest iscomfort are
uncommonly reporte in patients with
17. A 36-year-ol male presents for consultation esophageal stents
regaring an inciental esophageal mass seen
on compute tomography (CT) scan. This was 21. Which of the following is true regaring surgical
performe after he was involve in a motor intervention for esophageal cancer?
vehicle collision (MVC). He ha no serious A. Ivor Lewis esophagectomy involves an upper
injuries an was ischarge the same ay. Barium miline laparotomy an a left thoracotomy
swallow emonstrates a smooth, crescent-shape B. Transthoracic esophagectomy (TTE) is
lling efect. Which of the following is true associate with a shorter total hospital length
regaring this mass? of stay when compare to a transhiatal
A. Resection with a 1-cm margin is the treatment esophagectomy (THE)
of choice C. There is no ifference in mortality between the
B. They most commonly present with satellite use of TTE or THE in the surgical treatment of
tumors esophageal cancer
C. They have no risk of malignant egeneration D. TTE is associate with fewer complications
D. Esophageal ultrasonography may be useful when compare with THE
E. A preoperative enoscopic biopsy shoul be E. THE is performe with a right cervical incision
performe an miline laparotomy
82 PArt i Patient Care

Answers
1. B. This patient has ysphagia in the setting of sclero- injury. Most strictures occur within  months of ingestion.
erma, an autoimmune connective tissue isease most A less common long-term risk after caustic injury to the
common in women between the ages of 30 an 50. Ray- esophagus is the evelopment of squamous cell esophageal
nau occurs in 90% of scleroerma patients an presents carcinoma (A). Because of this risk, the American Society
with the classic tria of color change, incluing an initial for Gastrointestinal Enoscopy recommens routine eno-
pallor seconary to vasospasm, followe by cyanosis an scopic screenings every  to 3 years, beginning 10 to 0 years
rubor. Approximately 90% of patients have gastrointestinal after caustic esophageal injury (B). Ault caustic ingestion
involvement, an the esophagus is the most common site. tens to be more severe than peiatric cases (D). Neutraliz-
Patients with scleroerma evelop brous replacement of the ing agents are never inicate in caustic ingestions an o
smooth muscle layer of the esophagus, resulting in ecrease not prevent subsequent complications (E). Neutralizing reac-
peristalsis of the lower portion of the esophagus an hypo- tions between aciic an alkaline substances create exother-
tonicity of the lower esophageal sphincter which may pres- mic reactions that cause further thermal injury. Patients with
ent with ysphagia, reux, an peptic strictures. The most focal necrosis (grae III caustic injury) shoul be starte on
appropriate initial management is a PPI an a promotility broa-spectrum IV antibiotics.
agent. Antireux proceures (Nissen) are relatively contrain- References: Cheng HT, Cheng CL, Lin CH, et al. Caustic inges-
icate in patients with scleroerma, as up to 70% of patients tion in aults: the role of enoscopic classication in preicting out-
suffer from ysphagia postoperatively (E). Laparoscopic come. BMC Gastroenterol. 008;8(1):31.
Heller myotomy an pneumatic ilation are both acceptable ASGE Stanars of Practice Committee, Evans JA, Early
DS, et al. The role of enoscopy in Barrett’s esophagus an other
rst-line treatments for achalasia in patients who are goo
premalignant conitions of the esophagus. Gastrointest Endosc.
surgical caniates; however, they are not inicate for
01;76(6):1087–1094.
esophageal ysmotility ue to scleroerma (A, C). Patients
with achalasia will have absent peristalsis of the esophageal
4. B. This patient with chest pain an oynophagia after
boy an hypertonicity of the lower esophageal sphincter.
ilation of an esophageal stricture shoul be evaluate for
Calcium channel blockers, which may be inicate in acha-
iatrogenic esophageal perforation. Iatrogenic injuries are the
lasia, iffuse esophageal spasm, an nutcracker esophagus,
most common cause of esophageal perforations an occur
can actually worsen ysmotility in scleroerma (D).
at anatomic areas of narrowing, incluing the cricopharyn-
Reference: Carlson DA, Hinchcliff M, Panolno JE. Avances
in the evaluation an management of esophageal isease of systemic
geus an GE junction. When an esophageal perforation is
sclerosis. Curr Rheumatol Rep. 015;17(1):475. suspecte, initial workup inclues plain raiographs of the
neck, chest, an/or abomen epening on the suspecte
2. E. This patient has esophageal aenocarcinoma, the most location of the perforation. The next step is esophagram with
common esophageal malignancy in the Unite States. This water-soluble contrast. If normal, the next step is a thin bar-
patient has a T1b lesion with no evience of noal isease ium esophagram, which is more sensitive. CT with contrast
or istant metastasis an is a caniate for esophagectomy. is less sensitive than esophagram (A). Blin nasogastric tube
Important T stages to remember for esophageal aenocarci- placement is not appropriate in the setting of a suspecte
noma are T1a (invasion into the lamina propria or muscularis esophageal injury an may cause further injury (C). Surgical
mucosa), T1b (invasion into the submucosa), an T (invasion management prior to conrming the iagnosis is premature
into the muscularis propria). Patients with T1a lesions with no (D, E). If the patient is foun to have a free perforation of
abnormal lymph noes are caniates for enoscopic resec- the mi-thoracic esophagus, repair via a right posterolateral
tion (C). Patients with T1b lesions with no abnormal lymph thoracotomy woul be appropriate. Injuries to the istal
noes shoul procee with esophagectomy. Lymph noe thoracic esophagus are approache via a left posterolateral
involvement is irectly proportional to esophageal tumor thoracotomy.
epth or T stage. The incience of positive lymph noes is Reference: Blaergroen MR, Lowe JE, Postlethwait RW. Diag-
0% for T1a an 50% for T1b tumors. Patients with T lesions nosis an recommene management of esophageal perforation
an rupture. Ann Thorac Surg. 1986;4(3):35–39.
or any lymph noe involvement shoul procee with neoa-
juvant chemoraiation (A, B). Enucleation is not an appro-
priate treatment for esophageal aenocarcinoma, as a formal 5. B. The esophagus is a -layere muscular conuit con-
resection with lymph noe issection is warrante (D). Enu- necting the oropharynx to the stomach. The outer muscular
cleation is a treatment option for esophageal leiomyomas. layer is longituinal while the inner layer is circular an
Reference: National Comprehensive Cancer Network. Esoph- consiere an extension of the cricopharyngeus muscle (A).
ageal an Esophagogastric Junction Cancers (Version 5.00). Several anatomic areas of narrowing exist in the esophagus
https://www.nccn.org/professionals/physician_gls/PDF/esopha- with the cricopharyngeus muscle contributing to the narrow-
geal.pf. Accesse December 30, 00. est portion of the esophagus. Other anatomic areas of nar-
rowing occur at the aortic arch an the iaphragm (C). The
3. C. The most common complication after caustic injury to cervical esophagus is supplie by the thyrocervical trunk
the esophagus is stricture formation; the likelihoo of evel- off the subclavian artery. The major branches of the thyro-
oping a stricture is irectly correlate with the severity of cervical trunk can be remembere by the mnemonic “STAT”
CHAPtEr 7 Alimentary Tract—Esophagus 83

(suprascapular artery, transverse cervical artery, ascening extra esophageal isease (D). In the above patient, a biopsy
cervical artery, an inferior thyroi artery). The thoracic shoul be consiere. However, the long uration of GERD
esophagus is primarily supplie by branches irectly off the an absence of any systemic symptoms (fevers, night
aorta. The surgical approach to the esophagus can be ivie sweats, weight loss) make carcinoma unlikely, an thus the
into thirs. The istal thir of the esophagus is approache surgery shoul procee (E).
by a left thoracotomy, while the proximal an miesophagus Reference: Kunio NR, Dolan JP, Hunter JG. Short esophagus.
are approache with a right thoracotomy, as the aorta is in Surg Clin North Am. 015;95(3):641–65.
the way uring a left thoracotomy (E).
8. C. The management of BE with carcinoma has evolve
6. D. The mechanism of a Mallory-Weiss tear is similar to consierably in recent years. Esophagectomy with recon-
that of an esophageal perforation (Boerhaave synrome) struction was once consiere the stanar of care for
but iffers in that the injury is not full thickness (A). It is high-grae ysplasia, but this has been largely replace by
the result of forceful vomiting or coughing, such as after minimally invasive enoscopic techniques such as raiofre-
an alcohol rinking binge. The classic escription is retch- quency ablation (RFA) (A). A large meta analysis publishe
ing followe by vomiting of bloo. The presence of a hiatal in the New England Journal of Medicine emonstrates that
hernia is a preisposing factor an is foun in a majority RFA is associate with a high rate of isease eraication an
of patients. This situation exposes the LES to high pres- reuce risk of the evelopment of carcinoma. Although
sures, which results in a partial-thickness mucosal tear an no ranomize control trial currently exists to support this
bleeing most commonly 3 to 5 cm above the gastroesoph- recommenation, enoscopic therapy is now the favore
ageal junction on the left sie (B). Boerhaave synrome approach for high-grae ysplasia in BE without suspi-
results in a full-thickness tear causing esophageal perfo- cious noules. Enoscopic ablation or repeat enoscopy in
ration (A). These patients often present in sepsis with air 3 to 6 months with 4-quarant biopsies every 1 cm within the
in the meiastinum an a pleural effusion. Severe sepsis segment of BE are appropriate options in patients with low-
in the setting of esophageal perforation manates surgical grae ysplasia (B). These patients shoul also be offere an
intervention (C). Most bleeing from Mallory-Weiss tears antireux proceure such as a Nissen proceure or meical
stops spontaneously with nonsurgical management (E). management with PPI, even if asymptomatic. (D). Oncology
Patients shoul unergo enoscopy to conrm the iag- referral is premature because there is not yet a cancer iagno-
nosis. Recent stuies suggest that the area of bleeing is sis establishe for the above patient (E).
best manage by injecting sclerosing agents or epineph- References: Bennett C, Green S, Decaestecker J, et al. Surgery ver-
rine to prevent rebleeing. Esophageal balloon tamponae sus raical enotherapies for early cancer an high-grae ysplasia in
is contrainicate as it can convert a partial thickness tear Barrett’s oesophagus. Cochrane Database Syst Rev. 01;11:CD007334.
into a full-thickness esophageal laceration. Aitionally, it Almon M, Barr L. Management controversies in Barrett’s
will not stop the bleeing as it is usually arterial an not oesophagus. Gastroenterology. 014;49():195–05.
Shaheen NJ, Sharma P, Overholt BF, et al. Raiofrequency
venous. In cases not amenable to enoscopic therapy, oper-
ablation in Barrett’s esophagus with ysplasia. N Engl J Med.
ative management consists of oversewing the laceration
009;360():77–88.
through an anterior longituinal gastrotomy in the mile
thir of the stomach.
9. E. BE occurs in 5% to 7% of patients with GERD. It is
Reference: Llach J, Elizale JI, Guevara MC, et al. Enoscopic
an acquire pathology (A). The hallmark feature is the
injection therapy in bleeing Mallory-Weiss synrome: a ranom-
ize controlle trial. Gastrointest Endosc. 001;54(6):679–681.
presence of intestinal goblet cells, which signies intestinal
metaplasia, on enoscopic biopsy. It occurs more commonly
7. C. Roughly 15% of the ault population in the Unite in males with a 3:1 ratio (B). Once BE evelops, the risk of
States have GERD. Most patients can initially be manage aenocarcinoma is approximately 0.5% per year. In one
conservatively with the use of PPI. Inications for surgical large stuy, the prevalence of cancer was 4%. Management
intervention inclue failure of conservative management, of BE is initially meical, provie there is no evience of
patient preference for enitive intervention espite suc- severe ysplasia. However, surveillance for ysplasia is rec-
cessful meical management (e.g., patient woul like to ommene in patients with BE. If severe ysplasia is pres-
avoi lifelong nee for meication), an complications ent, enoscopic raioactive ablation or esophagectomy are
associate with GERD incluing Barrett esophagus or recommene (D). In patients with BE without ysplasia,
extra-esophageal manifestations (asthma, cough, hoarse- a ranomize stuy comparing meical management with
ness). The stanar surgical intervention involves a Nissen antireux surgery showe that there were no ifferences
funoplication. If a shortene esophagus is encountere between the two treatments with regar to preventing pro-
uring surgery (abominal length <1 cm), then a Collis gression to ysplasia an aenocarcinoma, although antire-
gastroplasty will nee to be performe to lengthen it an ux surgery was more efcient than meical treatment (C).
minimize tension uring antireux repair (A). In most References: Drewitz DJ, Sampliner RE, Garewal HS. The
incience of aenocarcinoma in Barrett’s esophagus: a prospec-
patients, about 3 cm of intraabominal esophagus can
tive stuy of 170 patients followe 4.8 years. Am J Gastroenterol.
be mobilize an thereby avoi the nee to lengthen the
1997;9():1–15.
esophagus. An anterior (Dor) funoplication may be con- Parrilla P, Martínez e Haro LF, Ortiz A, et al. Long-term results
siere in patients with unerlying esophageal ysmotility of a ranomize prospective stuy comparing meical an surgical
(B). Although scleroerma can present with a shortene or treatment of Barrett’s esophagus. Ann Surg. 003;37(3):91–98.
brotic esophagus, this is a iffuse process an will involve Peters, J. H, DeMeester, T. R. Esophagus an iaphragmatic
the entire esophagus. In aition, most patients will have hernia.
84 PArt i Patient Care

10. A. Although pharmacologic treatment for BE shoul make the iagnosis on a retroex view (C). Although incar-
be similar to that for GERD, most authorities agree that the ceration is rare, most surgeons recommen elective repair of
use of PPIs is more effective in treating patients with BE. The paraesophageal hernias because of the potential risk of stran-
ASPECT trial emonstrate that high-ose PPI an aspirin gulation (B, E). It is not typically precee by trauma (D).
chemoprevention therapy, especially in combination, sig-
nicantly an safely reuces the rate of cancer progression 13. D. Manometry is an important iagnostic tool to ien-
in patients with BE (D). Interestingly, in vivo stuies have tify preisposing conitions for esophageal isease. Char-
shown that nonsteroial antiinammatory rugs (NSAIDs) acteristics of an abnormal LES inclue resting pressure less
an statins can reuce the progression of cancer in patients than 6 mmHg (normal range is 6–6 mmHg), overall length
with Barrett esophagus. The ASPECT trial may provie more of less than  cm, an abominal length less than 1 cm (A, B).
powerful evience to suggest the use of NSAIDs in patients Relaxation of LES with swallowing is a function of the nor-
with Barrett esophagus for chemoprophylaxis. Long-seg- mal swallowing mechanism an ysfunction will increase
ment Barrett esophagus has a higher risk for high-grae the risk for the evelopment of achalasia (E). The resting
ysplasia (C). Photofrin has not been emonstrate to be a UES is 60 to 80 mmHg (C). High UES pressures will preis-
useful moality (E). Dietary restrictions are helpful in Barrett pose patients to pulsion iverticulum an ifculty with
esophagus an inclue the avoiance of fatty foos, choco- swallowing.
late, peppermint, alcohol, coffee, ketchup, mustar, or vine-
gar (B). 14. E. Acute variceal bleeing (AVB) is the leaing cause of
References: Cameron JL, Cameron AM. The management of upper GI bleeing in patients with cirrhosis, an the man-
Barrett’s esophagus. In: Cameron JL, Cameron AM, es. Current sur- agement can be challenging. Early recognition an inter-
gical therapy. 11th e. Philaelphia: W.B. Sauners; 014. vention are important because the progression to sepsis an
Shapiro J, van Lanschot JJB, Hulshof MCCM, et al. Neoajuvant multiorgan failure confers a ismal prognosis with over 90%
chemoraiotherapy plus surgery versus surgery alone for oesopha- mortality. The most important next steps in a cirrhotic pre-
geal or junctional cancer (CROSS): long-term results of a ranomise
senting with AVB involve the airway, breathing, an circu-
controlle trial. Lancet Oncol. 015;16(9):1090–1098.
Jankowski JAZ, e Caestecker J, Love SB, et al. Esomeprazole an
lation (ABCs). Airway management shoul take preceence
aspirin in Barrett’s oesophagus (AspECT): a ranomise factorial over controlling AVB (B). After the ABCs, the recommene
trial. Lancet. 018;39(10145):400–408. approach involves a combination of vasoactive rugs (oct-
reotie) an enoscopic intervention. Meical management
11. B. Hiatal hernias are ivie into three types. Type I, shoul be initiate as soon as possible because it can reuce
or a sliing hiatal hernia, is the most common. In this her- the rate of active bleeing an improve the yiel of eno-
nia, the gastroesophageal junction moves upwar into the scopic intervention (D). Several ranomize controlle trials
posterior meiastinum along with part of the stomach, such have been performe comparing enoscopic ban ligation
that the LES is above its normal position (A). The major- versus enoscopic sclerotherapy an have emonstrate
ity of these hernias are asymptomatic. Those who o have the superiority of the former in both controlling bleeing
symptoms typically experience heartburn an regurgitation. an safety prole. Infection has been emonstrate to be an
In type II, or paraesophageal hernias, the gastroesophageal important preictor of mortality in AVB. Patients that receive
junction an therefore the LES are in their normal positions, prophylactic uoroquinolones have been shown to have
as is the caria. However, the gastric funus is islocate a reuce incience of AVB an improve survival (C). In
upwar. The LES is neither hypertensive nor hypotensive patients with chronic esophageal varices, beta-blockers can
(C, D). A type III hernia is a combination of types I an II. be use to prevent episoes of rebleeing (A).
A hypertensive LES is characteristic of achalasia. In GERD, References: Calès P, Masliah C, Bernar B, et al. Early aminis-
the LES pressure is low. GERD seems to begin from gastric tration of vapreotie for variceal bleeing in patients with cirrhosis.
istention. The istention leas to a shortening of the LES N Engl J Med. 001;344(1):3–8.
Hou MC, Lin HC, Liu TT, et al. Antibiotic prophylaxis after eno-
(E). As the sphincter shortens, its resting pressure ecreases.
scopic therapy prevents rebleeing in acute variceal hemorrhage: a
The location of the LES (in the normal abominal position or
ranomize trial. Hepatology. 004;39(3):746–753.
in the meiastinum) is important in GERD. Loss of abom- Jensen DM, Kovacs T, Ranall G. Emergency sclerotherapy vs
inal length of the LES causes a ecrease in LES pressure rubber ban ligation for actively bleeing esophageal varices in a
because it is no longer subjecte to the positive pressure of ranomize prospective stuy. Gastrointest Endosc. 1994;40:41.
the abomen. Lo GH, Lai KH, Cheng JS, et al. Emergency baning ligation ver-
sus sclerotherapy for the control of active bleeing from esophageal
12. A. A paraesophageal hernia, or type II hiatal hernia, is varices. Hepatology. 1997;5(5):1101–1104.
also calle a rolling-type hiatal hernia. The wiene hiatus per-
mits the funus of the stomach to protrue into the chest, 15. D. This patient has achalasia, a primary motility isor-
anterior an lateral to the boy of the esophagus. The gastro- er of the esophagus, specically of the LES. The pathogen-
esophageal junction remains below the iaphragm. The her- esis is presume to be neurogenic egeneration of ganglion
niate gastric funus rotates in a counterclockwise irection cells, which can be iiopathic or infectious (i.e., Chagas
an is prone to becoming incarcerate an strangulate. This isease from Trypanosoma cruzi). The egeneration results
herniate portion of the stomach evelops mucosal erosions in a failure of the LES to relax on swallowing, leaing to an
(Cameron ulcers) that can lea to chronic bloo loss an increase in intraluminal esophageal pressure, marke esoph-
anemia in up to one-thir of patients. Patients can also have ageal ilation (with an air–ui level on raiograph), an
ysphagia, heartburn, an abominal pain. Diagnosis can loss of progressive peristalsis in the boy of the esophagus.
be mae by a barium swallow. Upper enoscopy can reaily The classic tria of symptoms is ysphagia, regurgitation,
CHAPtEr 7 Alimentary Tract—Esophagus 85

an weight loss. There are four basic treatment options, all requires an esophagectomy (e.g., cancer, severe burn), imme-
of which are consiere palliative proceures as there is no iate esophagectomy with reconstruction is recommene
cure. Accoring to recent American College of Gastroenter- if feasible (limite inammation an minimal elay) (D).
ology Clinical Guielines, initial therapy shoul be either Stenting is generally reserve for unresectable cancer (E). An
grae pneumatic ilation or laparoscopic surgical myot- iatrogenic perforation in a patient with achalasia will nee to
omy with a partial funoplication in patients t to unergo have the perforation aresse as iscusse above an will
surgery. Esophageal pneumatic ilation is a better option in also nee enitive management of the unerlying isease
patients with higher operative risk an is safer than previ- provie the person is not extremely ill. The treatment of
ously thought, but patients will often require multiple ila- choice is a left thoracotomy, primary repair, an longitui-
tions over time. For low-risk patients or those who have faile nal myotomy on the contralateral sie with or without fun-
balloon ilation, a laparoscopic esophagomyotomy with an oplication. Laparoscopic repair is increasing in popularity
anterior funoplication (Dor) or partial, 70-egree posterior but will still nee a myotomy on the contralateral sie of the
funoplication (Toupet) shoul be performe. A recent mul- perforation (C).
ticenter, ranomize controlle trial foun that although a References: Fernanez FF, Richter A, Freuenberg S, Wenl K,
lower percentage of patients with a Toupet funoplication Manegol BC. Treatment of enoscopic esophageal perforation. Surg
ha an abnormal 4-hour pH test when compare with a Dor Endosc. 1999;13(10):96–966.
funoplication, the ifferences were not statistically signi- Hunt DR, Wills VL, Weis B, Jorgensen JO, DeCarle DJ, Coo IJ.
Management of esophageal perforation after pneumatic ilation for
cant, an either approach woul be appropriate. A complete
achalasia. J Gastrointest Surg. 000;4(4):411–415.
funoplication, or a Nissen, has a high chance of causing
recurrent ysphagia in this patient population (B). Meical
management with calcium channel blockers an nitroglyc- 17. D. Leiomyomas are the most common benign tumor
erin can help relax the LES, but this treatment only relieves in the esophagus, accounting for more than 50% of benign
symptoms in less than 10% of patients. These meications tumors. However, benign masses constitute only 10% of
are only consiere in patients who are not appropriate sur- esophageal tumors. They have a small risk of malignant
gical caniates (C). In high-risk elerly patients, injection of egeneration (C). Leiomyomas only become symptom-
the LES with botulinum toxin can provie short-term relief. atic when they are very large (>5 cm). Otherwise, they are
Botulinum toxin shoul be avoie in patients who woul incientally iscovere uring the course of other stuies.
otherwise be appropriate surgical caniates because it can They have a characteristic appearance on barium swallow
ruin the anatomic planes require for surgery. Nutcracker of a smooth, crescent-shape lling efect that encroaches
esophagus is characterize by high-amplitue peristaltic on the lumen. On enoscopy, the mucosa is usually intact,
waves of the esophagus (A). Esophageal iverticula can be an the tumor moves up an own with swallowing. If
associate with a hypertrophic upper esophageal sphincter. it has the characteristic appearance, the tumor shoul not
POEM is an emerging option but requires a long learning unergo biopsy because of an increase risk of mucosal
curve. Up to 50% of patients can have aci reux following perforation. This can create scarring that may affect later
the proceure (E). efforts at resection (E). Esophageal ultrasonography is
References: Campos GM, Vittinghoff E, Rabl C. Enoscopic an very useful in the iagnosis of leiomyomas because it will
surgical treatments for achalasia: a systemic review an meta-analysis. emonstrate a homogeneous region of hypoechogenicity.
Ann Surg. 009;49(1):45–57. Treatment is to enucleate the mass, which can be one via
Hoogerwerf WA, Pasricha PJ. Achalasia: treatment options revis- a vieoscopic approach with intraoperative esophagoscopy
ite. Can J Gastroenterol. 000;14(5):406–409. (A). The cell of origin of these tumors is mesenchymal. The
Rawlings A, Soper NJ, Oelschlager B, et al. Laparoscopic Dor ver- average age at presentation is 38 years, an they are twice
sus Toupet funoplication following Heller myotomy for achalasia: as common in males an most commonly locate in the
results of a multicenter, prospective, ranomize-controlle trial. lower two-thirs of the esophagus. Leiomyomas are usu-
Surg Endosc. 01;6(1):18–6.
ally solitary, but multiple tumors are seen in as many as
Vaezi M, Richter J, Wilcox C. Botulinum toxin versus pneumatic
10% of patients (B).
ilatation in the treatment of achalasia: a ranomize trial. Gut.
1999;44():31–39. Reference: Aurea P, Grazia M, Petrella F, Bazzocchi R. Giant leio-
myoma of the esophagus. Eur J Cardiothorac Surg. 00;(6):1008–1010.
16. B. The ecision of how to procee in an iatrogenic
esophageal perforation epens on ve factors: whether it 18. D. A Zenker iverticulum is a false esophageal iver-
is a free or containe perforation, the uration of time that ticulum that oes not contain all layers of the esophagus; it
the perforation has been present, the unerlying pathology is also a type of pulsion iverticulum (A). A pulsion ivertic-
in the esophagus, whether severe inammation is present ulum forms at a point of weakness an is ue to alterations
at surgery, an the patient’s conition. As a general rule, in luminal pressure. Conversely, a traction iverticulum is
if the perforation is containe, as shown on an esophago- from external pulling on the esophageal wall, such as from
gram, management can be conservative (A). If it is a small iname lymph noes with tuberculosis. Zenker iverticu-
free perforation, surgery is inicate with primary repair lum is the most common type of esophageal iverticulum.
with or without an intercostal muscle ap. Resection of an It usually presents in oler patients (>60 years). It charac-
injure esophagus with cervical esophagostomy (spit s- teristically arises at a point of weakness, most commonly at
tula), gastrostomy, an feeing jejunostomy is reserve for the Killian triangle, which is forme by the inferior bers of
situations in which there has been a long elay in iagnosis the inferior constrictor muscle an the superior borer of the
(>7 hours), severe inammation is present, or the patient cricopharyngeus muscle. Patients typically present with ys-
is extremely ill or isable (B). If the unerlying isease phagia, regurgitation of unigeste foo, halitosis, episoes
86 PArt i Patient Care

of aspiration, an salivation (C). With the characteristic his- 20. B. Patients with newly iagnose esophageal cancer
tory, the rst iagnostic stuy is a barium swallow. In the frequently present with poor nutritional status, which only
absence of other pathology (such as an irregular mucosa), worsens after starting neoajuvant therapy. As such, although
enoscopy is not neee (B). Treatment is surgical by either nutritional optimization is an important component in the man-
open or enoscopic techniques. The open technique involves agement of esophageal cancer, the optimal approach remains
cervical esophagomyotomy with stapling an amputation of unene. Percutaneous gastrostomy, however, shoul be
the iverticulum. The enoscopic technique involves ivi- iscourage because it may compromise the gastric conuit
sion of the common wall between the iverticulum an the neee uring esophageal reconstruction an will elay che-
esophagus. Stuies have shown that results with the eno- motherapy for an aitional  to 4weeks. The role for paren-
scopic technique are better with larger iverticula (E). Diver- teral nutrition is limite because of its high cost an high rate
ticula smaller than 3 cm are too short to accommoate one of complications (A). Nasogastric tube insertion can lea to
cartrige of staples an to allow complete ivision of the migration of the tube an aspiration (D). Esophageal stents are
sphincter; therefore, this size is consiere a contrainication frequently offere because they can signicantly improve the
to this technique. ysphagia associate with esophageal cancer. Unfortunately, its
References: Bonavina L, Bona D, Abraham M, Saino G, Abate role in improving nutritional status has ha inconsistent results
E. Long-term results of enosurgical an open surgical approach for in the literature (C). Stent migration an chest iscomfort are
Zenker iverticulum. World J Gastroenterol. 007;13(18):586–589. common an lea to the frequent removal of the stents (E).
Collar JM, Otte JB, Kestens PJ. Enoscopic stapling technique Aitional stuies are neee to etermine the best approach
of esophagoiverticulostomy for Zenker’s iverticulum. Ann Thorac
for nutritional optimization in this patient population.
Surg. 1993;56(3):573–576.
References: Jones CM, Grifths EA. Shoul oesophageal stents
Narne S, Cutrone C, Bonavina L, Chella B, Peracchia A. Eno-
be place before neo-ajuvant therapy to treat ysphagia in patients
scopic iverticulotomy for the treatment of Zenker’s iverticulum:
awaiting oesophagectomy? Best evience topic (BET). Int J Surg.
results in 10 patients with staple-assiste enoscopy. Ann Otol Rhi-
014;1(11):117–1180.
nol Laryngol. 1999;108(8):810–815.
Mão-e-Ferro S, Serrano M, Ferreira S, et al. Stents in patients
with esophageal cancer before chemoraiotherapy: high risk of com-
19. A. Cricopharyngeal ysfunction has multiple causes, plications an no impact on the nutritional status. Eur J Clin Nutr.
incluing such neurogenic an myogenic etiologies as 016;70(3):409–410.
stroke, multiple sclerosis, peripheral neuropathy, Parkinson Naharaja V, Cox MR, Eslick GD. Safety an efcacy of esoph-
isease, an ermatomyositis. The exact cause is unknown, ageal stents preceing or uring neoajuvant chemotherapy for
but the primary theory is that the cricopharyngeus muscle, esophageal cancer: a systemic review an meta-analysis. J Gastroin-
which is normally in a state of tonic contraction, fails to relax test Oncol. 014;5():119–16.
an allow the passage of foo into the cervical esophagus.
This prouces a Zenker iverticulum, which is consiere 21. C. Surgical intervention in esophageal cancer is an
a false iverticulum (only involves an outpouching of the area of active research. The three stanar approaches
mucosa an submucosa) an can be conrme with a bar- inclue TTE, THE, an a combination of the two using a
ium swallow (C). Enoscopic evaluation of a suspecte three-incision esophagectomy. TTH was initially escribe
Zenker iverticulum is iscourage as it can lea to an iat- as a two-stage proceure by Dr. Ivor Lewis in which he per-
rogenic perforation. Patients escribe ifculty swallowing forme mobilization of the stomach using an upper miline
foo, which worsens throughout the ay as the iverticulum laparotomy incision followe by resection of the esophagus
increasingly gets lle with foo. Another key element of using a right thoracotomy incision several ays later (A).
the iagnosis is the classic history of an inability to hanle A large multicenter prospective stuy comparing THE an
saliva secretion, such that the patient escribes expectoration TTE faile to emonstrate any ifference in overall mortality
of saliva. Patients also report hoarseness. Diverticulectomy an morbiity between the two approaches (D). However,
is often performe uring surgery for a Zenker iverticu- THE has been shown in several stuies to be associate with
lum. However, the most important aspect of management a lower total hospital length of stay (B). THE is performe
is cricopharyngeal myotomy, which is necessary to correct with a left cervical incision an miline laparotomy (E). It is
the unerlying pathology (B). Weight loss results from a often performe for patients with istal esophageal cancer.
ecrease caloric intake. Although one shoul always be References: D’Amico TA. Outcomes after surgery for esopha-
suspicious of carcinoma in a patient with ifculty swallow- geal cancer. Gastrointest Cancer Res. 007;1(5):188–196.
ing an weight loss, the long uration of symptoms makes Hulscher JB, Tijssen JG, Obertop H, van Lanschot JJ. Transtho-
carcinoma unlikely (D, E). racic versus transhiatal resection for carcinoma of the esophagus: a
References: Cameron JL, Cameron AM. The management of meta-analysis. Ann Thorac Surg. 001;7(1):306–313.
Barrett’s esophagus. In: Cameron JL, Cameron AM, es. Current Litle VR, Buenaventura PO, Luketich JD. Minimally invasive resec-
surgical therapy. 11th e. Philaelphia: W.B. Sauners; 014. tion for esophageal cancer. Surg Clin North Am. 00;8(4):711–78.
Cameron JL, Cameron AM. The management of pharyngeal Rentz J, Bull D, Harpole D, et al. Transthoracic versus transhiatal
esophageal (Zenker) iverticula. 11th e. Philaelphia, PA: W.B. esophagectomy: a prospective stuy of 945 patients. J Thorac Cardio-
Sauners; 014. vasc Surg. 003;15(5):1114–110.
Alimentary Tract—Stomach
NAVEEN BALAN, AMY KIM YETASOOK, AND KATHRYN T. CHEN 8
ABSITE 99th Percentile High-Yields
I. Ulcers
A. Peptic ulcer isease (PUD): imbalance of pepsin/aci an mucosal protection
1. Almost always cause by Helicobacter pylori (gram-negative spirochete) an NSAID overuse
. Triple therapy: PPI, clarithromycin, amoxicillin, or metroniazole ×14 ays
3. Daintree Johnson classication for types of gastric ulcers
a) Type I: along lesser curvature in the antrum, solitary, not aci associate
b) Type II: prepyloric, solitary, aci associate
c) Type III: prepyloric an uoenal, one in each location, aci associate
) Type IV: proximal stomach/caria, solitary, not aci associate
e) Type V: anywhere in the stomach, usually multiple, NSAID associate
4. Biopsy all gastric ulcers: higher risk of cancer

Operation Procedure Result Complication


Antrectomy Resection of distal stomach (need Removal of antral G cells Incomplete antrectomy can lead
reconstruction with Billroth I to continued ulcer disease
or II)
Truncal Ligation of anterior and posterior Removal of vagal Loss of parasympathetic
vagotomy vagal trunks 4 cm proximal to stimulation to gastric innervation to pylorus which
(TV) GEJ body parietal cells, prevents gastric emptying
decreasing acid secretion (primarily to solid foods)
Selective Ligation of anterior and posterior Lower rate of delayed gastric
vagotomy (SV) vagal trunks distal to hepatic/ emptying but higher risk of
celiac branches recurrent ulcer disease
Highly selective Similar to SV except preserves Lowest rate of delayed gastric
vagotomy crow’s foot ębers innervating emptying but highest risk of
(HSV) pylorus and antrum recurrent ulcer disease
Pyloroplasty Widen the pyloric channel, Improved gastric emptying Can lead to rapid gastric
multiple approaches (usually with TV/SV) emptying

B. Surgical inications: surgery for bleeing PUD entails oversewing of the ulcer (to ligate the bleeing
artery) with consieration for truncal vagotomy (to ecrease aci secretion); more time-consuming
proceures (selective vagotomy) reserve for elective ulcer surgery, as are antrectomy an Billroth II
(when obstruction complicates PUD)

87
88 PArt i Patient Care

1. Truncal vagotomy an selective vagotomy always nee rainage proceure (e.g., pyloroplasty)
whereas highly selective vagotomy oes not

Duodenal Ulcer*
Problem Treatment
Bleeding Oversew bleeder
Perforation Graham patch

*For gastric ulcer, ęrst-line treatment is same; however, ęrst biopsy ulcer to rule out malignancy.

II. Upper Gastrointestinal Blee


A. Meical management: hyration, PPI bi (noninferior to continuous PPI) ×7 hours
B. Proceural interventions: enoscopic hemostasis attempts ×, angioembolization(usually GDA); n
look EGD only for high-risk lesion (visible vessel or aherent clot)
C. Surgery inications: multiple faile enoscopies, hemoynamic instability, associate perforation

III. Bariatric Surgery


A. Inications:
1. BMI >40 (morbi obesity) or BMI 35 to 40 with ≥1 weight-relate comorbiity (iabetes,
hypertension, hyperlipiemia, GERD, obstructive sleep apnea, pseuotumor cerebri, severe life-
limiting arthritis)
B. Sleeve gastrectomy: most commonly performe, lower excess boy weight (EBW) loss an incience
of vitamin eciency; postoperative complications inclue staple line leak, bleeing, e novo reux,
incisura stricture, twisting/kinking of sleeve, weight regain
C. Roux-en-Y gastric bypass: higher EBW loss an incience of vitamin eciency; postoperative
complications inclue anastomotic leak, internal hernia, marginal ulcer, perforate marginal ulcer,
bile reux, jejunojejunostomy, intussusception, roux stasis synrome, gastrogastric stula, umping
synrome, an weight regain
D. Pulmonary embolus is leaing cause of eath, occurring within a few weeks after ischarge
E. Common postoperative complications:

Complication Symptoms Etiology Management


Alkaline reĚux Bilious emesis does Bile reĚux into the stomach Workup with EGD (pathology:
(requires surgery NOT relieve usually with Billroth II foveolar hyperplasia with
more than any other epigastric pain anatomy mucosal congestion and
postgastrectomy edema); conęrm with HIDA
syndrome) demonstrating biliary secretion
into stomach/esophagus;
treatment: convert to Roux-en-Y
Small intestinal Bloating, chronic Bacterial overgrowth Labs for B12 deęciency, macrocytic
bacterial overgrowth watery diarrhea in blind loop due to anemia, conęrm with lactulose
(SIBO) immotility breath test and treat with
antibiotics (rifaximin)
Early dumping Nausea, vomiting, Hyperosmolar load and Eat smaller, more frequent meals
syndrome diarrhea 15–30 min release of serotonin, with low carbs, high protein, and
postprandially neurotensin, bradykinins, fat throughout the day, separate
and enteroglucagon solids from liquids, octreotide is
most eěective
Late dumping Dizziness, fatigue, High carbohydrate load Similar to above; if refractory,
syndrome diaphoresis 1–3 h causing hyperinsulinemia convert to Roux-en-Y
postprandially leading to reactive
hypoglycemia
Marginal ulcer Nausea, pain, and NSAIDs, smoking, H. pylori Discontinue NSAIDs, treat H. pylori,
perforation/bleeding infection, acid exposure to give acid suppression therapy,
at anastomosis small bowel bleeding or perforation may need
surgery
CHAPtEr 8 Alimentary Tract—Stomach 89

IV. Gastric Cancer


A. Aenocarcinoma: risk factors inclue H. pylori infection, iet (high salt, nitrosamine-containing foos),
obesity, smoking, ETOH, hereitary iffuse gastric cancer (CDH1 mutations)
1. Staging: EGD for biopsy, EUS for T/N stage, CT chest/abomen/pelvis, +/− PET-CT, iagnostic
laparoscopy to evaluate for peritoneal isease
. Early gastric cancer: cTis or T1a-enoscopic or surgical resection; T1b-surgical resection with 5 cm
gross margins to achieve R0 resection
3. Locoregional isease: T+ or noal involvement nee neoajuvant chemotherapy followe by
surgical resection an ajuvant chemotherapy
4. Surgical options: istal versus subtotal versus total gastrectomy with R0 goal, nee 16 lymph noes;
CDH1 nees total gastrectomy, also acceptable option for palliation in stage IV isease
5. Siewert-Stein classication:
a) Type I: epicenter within 1 to 5 cm above GE junction, treat as esophageal cancer
b) Type II: epicenter within 1 cm above an  cm below GE junction, treat as esophageal cancer
c) Type III: epicenter within  to 5 cm below GE junction, treat as stomach cancer
6. Incience of PROXIMAL gastric cancer has been rising the fastest (ue to obesity an reux);
proximal gastric cancer is two times more common in men an white race
7. H. pylori is PROTECTIVE against proximal gastric cancer but associate with increase risk of istal
gastric cancer
B. Gastrointestinal stromal tumor (GIST)
1. Enoscopy: submucosal mass with central umbilication an ulceration
. Essential features: majority ue to mutations in protooncogene KIT (most common) or PDGFRA;
spinle cell histology; can occur anywhere along GI tract, but stomach is most common (60%)
followe by jejunum/ileum
3. High-risk features for recurrence: tumor size >5 cm or >5 mitoses/hpf
4. Imatinib: tyrosine kinase inhibitor, highest response rate with exon 11 KIT mutation
a) Neoajuvant therapy: use when surgical resection will result in signicant morbiity
b) Ajuvant therapy: high-risk features for recurrence an tumor rupture uring surgery
C. MALT lymphoma: B-cell lymphoma, arises in the setting of chronic H. pylori infection
1. Histology: lymphoepithelial lesions are pathognomonic
. Treatment: triple or quaruple antibiotic therapy will eraicate cancer in majority of early cases

AL GRAWANY
90 PArt i Patient Care

Fig. 8.1
CHAPtEr 8 Alimentary Tract—Stomach 91

Fig. 8.2
92 PArt i Patient Care

Questions
1. A 56-year-ol male with fatigue unergoes 3. A 41-year-ol female presents to the emergency
upper enoscopy after initial workup shows a epartment with acute severe abominal pain
microcytic anemia. He is foun to have an ulcer an nausea but no vomiting. She reports a history
with irregular borers with biopsy showing of uncomplicate Roux-en-Y gastric bypass
a ense lymphoi inltrate with prominent years ago but enies other abominal surgeries.
lymphoepithelial lesions. Which of the following Abominal exam reveals mil tenerness
is true for this malignancy? without guaring or reboun tenerness. A CT
A. Tumors with chromosomal translocation scan shows ilate proximal small bowel but no
t(11;18) respon poorly to antibiotics intraabominal free air or ui. What is the next
B. After successful treatment, yearly enoscopy best step in the management of this patient?
is use for surveillance A. Avise the patient to eat smaller, more
C. It is most commonly a result of a gram- frequent meals
positive ro B. Exploratory laparotomy
D. There is no role for surgical resection C. Nasogastric tube ecompression followe by
E. Early-stage isease requires chemoraiation as 4-hour water-soluble contrast challenge
rst-line treatment D. Antibiotic therapy
E. Upper enoscopy
2. A 46-year-ol female was incientally foun
to have a peunculate mass along the greater 4. A 40-year-ol male with severe epigastric pain
curvature of the stomach on CT imaging is foun to have multiple uoenal ulcers on
following a motor vehicle collision. Further EGD. What is the normal location of the cells that
workup with enoscopy shows a submucosal secrete the majority of the hormone that is being
mass with central umbilication an ulceration overprouce in this patient?
that is foun to be CD117-positive after biopsy. A. Stomach boy
Which of the following is true regaring the B. Stomach antrum
management of this lesion? C. Pancreas
A. It is consiere a raiosensitive tumor D. Duoenum
B. The highest response rate to therapy involves E. Jejunum
mutations in the KIT proto-oncogene at exon
11 5. Three years after a laparoscopic Roux-en-Y gastric
C. It arises from an enoermal-erive bypass (LRYGB), a 45-year-ol male presents with
component symptoms an signs of a small bowel obstruction
D. Most patients become symptomatic early in (SBO). He reports a 150-lb weight loss. Which of
the course of their isease the following is the most likely etiology?
E. Early tumors can be treate with enoscopic A. An internal hernia
mucosal resection B. Ahesions
C. Roux compression ue to mesocolon scarring
D. Kinking of the jejunojejunostomy
E. Incarcerate abominal wall hernia
CHAPtEr 8 Alimentary Tract—Stomach 93

6. A 79-year-ol male with chronic back pain an 9. Which of the following is true regaring gallstone
chronic obstructive pulmonary isease (COPD) isease after weight loss surgery?
requiring supplemental oxygen presents to the A. The rate of postoperative cholecystectomy is
emergency epartment (ED) with epigastric the same regarless of the type of weight loss
abominal pain that starte suenly  ays surgery
ago. His abominal examination is signicant B. Prophylactic cholecystectomy shoul be
for epigastric tenerness but is otherwise performe at the time of surgery in most
unremarkable. A compute tomography (CT) patients
scan emonstrates a small amount of free air C. Ursoiol is recommene for 6 months after
uner the right hemiiaphragm but no contrast gastric bypass surgery
extravasation. An upper gastrointestinal (GI) D. Decrease secretion of calcium an mucin
water-soluble contrast stuy emonstrates a contributes to gallstone formation after weight
uoenal ulcer but no extravasation. Which of the loss surgery
following is the best management? E. Acute cholecystitis after weight loss surgery is
A. Nasogastric tube ecompression, intravenous uncommon
(IV) antibiotics, an proton pump inhibitor
(PPI) 10. Which of the following is the rst manifestation of
B. Exploratory laparotomy gastric leak following Roux-en-Y gastric bypass?
C. Diagnostic laparoscopy A. Abominal pain
D. Oral antibiotics, clear liqui iet for  weeks, B. Tachycaria
an follow-up in clinic C. Nausea
E. Serial abominal exam in the ED for 6 D. Increase serum glucose
to 8hours an, if improving, he may be E. Tachypnea
ischarge with oral antibiotics
11. A 45-year-ol male with a history of laparoscopic
7. Which of the following is true regaring the gastric baning 5 years ago presents to the ED
management of obesity? with complaints of pain at his port site. He rst
A. Inications for bariatric surgery inclue a notice it several ays ago after he got his gastric
boy mass inex (BMI) greater than 30 with ban ajuste in clinic. On exam, the port site
weight-relate comorbiities or BMI greater appears erythematous, warm, an is tener
than 35 to palpation. He is afebrile an normotensive.
B. Sibutramine acts by inhibiting pancreatic Which of the following is the best next step?
lipase A. CT of the abomen
C. Roux-en-Y gastric bypass (RYGB) oes not B. Amit to the hospital, start IV antibiotics an
have a restrictive component ui resuscitation
D. RYGB has a lower 30-ay mortality compare C. EGD
with biliopancreatic iversion (BPD) D. Incision an rainage
E. Patients with obesity-relate comorbiities o E. Discharge with oral antibiotics
not nee to attempt nonoperative management
before obesity surgery

8. Which of the following is the gol stanar for


the iagnosis of gastroparesis?
A. Upper enoscopy
B. Plain abominal x-rays
C. Nuclear meicine scan
D. CT
E. It is consiere a clinical iagnosis.
94 PArt i Patient Care

12. A 60-year-ol man presents with a 1-hour 15. Which of the following is true regaring TNM
history of worsening epigastric pain. He has a (tumor, noes, an metastases) staging for gastric
history of uoenal ulcer, an the results of a aenocarcinoma?
recent biopsy  weeks earlier were negative for A. Compute tomography scan is the most
Helicobacter pylori. Upright chest raiograph accurate means of etermining T an N
emonstrates free air uner the iaphragm. The staging
patient is hemoynamically stable. At surgery, B. The accuracy of enoscopic ultrasoun (EUS)
a perforate uoenal ulcer is foun with mil is higher for N stage than T stage
peritoneal contamination. Which of the following C. Fifteen lymph noes are require for an
is the best management option? oncologic resection to appropriately stage the
A. Graham patch of uoenal ulcer patient
B. Graham patch of uoenal ulcer with truncal D. Magnetic resonance imaging (MRI) with
vagotomy an pyloroplasty gaolinium shoul be routinely performe
C. Truncal vagotomy an antrectomy with E. T3 invaes the subserosa
Billroth I reconstruction
D. Truncal vagotomy an antrectomy with 16. Which of the following is associate with
Billroth II reconstruction hypergastrinemia?
E. Graham patch of uoenal ulcer with a highly A. Diabetes
selective vagotomy B. Hypothyroiism
C. Hyperparathyroiism
13. Which of the following is true regaring D. Chronic gastritis
postvagotomy iarrhea? E. D-cell hyperplasia
A. It is effectively treate with octreotie
B. It oes not improve with oral cholestyramine 17. A 46-year-ol male unergoes a istal
C. Cariovascular manifestations are common gastrectomy for a tumor in the gastric antrum that
D. Most patients require the creation of a reverse was biopsy proven to be aenocarcinoma. The
jejunal segment specimen is sent for pathology. Pathology reveals
E. Diarrhea may improve with the aministration microscopic evience of tumor at the margins.
of coeine Which of the following most accurately escribes
this resection?
14. A 45-year-ol woman is unergoing an A. D1 resection
exploratory laparotomy for Zollinger-Ellison B. D resection
synrome (ZES). Preoperative localization stuies C. R0 resection
faile to emonstrate the location of the tumor. D. R1 resection
At surgery, no obvious tumor is seen espite E. R resection
an extensive Kocher maneuver an careful
inspection. An intraoperative ultrasoun scan is 18. Which of the following is consiere to be a risk
negative. The next step in the management woul factor for gastric cancer?
be: A. Pernicious anemia
A. Closing the abomen B. Bloo group O
B. Distal pancreatectomy an splenectomy C. Carbonate aciic soa
C. Proximal pancreaticouoenectomy D. Female sex
D. Blin proximal uoenotomy E. Diabetes
E. Blin istal uoenotomy
19. Which of the following is true regaring the types
of gastric ulcers?
A. Type II ulcers are the most common
B. Type IV ulcers occur near the gastroesophageal
junction
C. Type I ulcers usually have increase aci
secretion
D. Type III ulcers are associate with ecrease
aci secretion
E. Type I gastric ulcers are prepyloric
CHAPtEr 8 Alimentary Tract—Stomach 95

20. Which of the following is true regaring 26. Which of the following is true regaring ZES?
gastrointestinal stromal tumor (GIST)? A. Symptoms ecrease with fasting
A. The extent of the tumor is best etermine B. Ulcers are most often locate in the istal
preoperatively by enoscopy uoenum
B. They arise from smooth muscle cells C. It is most commonly familial
C. Malignant potential is reaily etermine by D. It is the most common functional
histologic features neuroenocrine tumor
D. They can be manage by laparoscopic wege E. Treatment with proton pump inhibitors (PPIs)
resection can control symptoms in the majority of
E. They rarely present with GI bleeing patients

21. Which of the following is true regaring 27. A 70-year-ol man presents with an 8-hour
postgastrectomy bile reux? history of acute abominal pain an a history of
A. It is more likely to occur after a Billroth I than melena. On examination, the patient is febrile to
a Billroth II reconstruction 101°F, with a bloo pressure of 105/70 mmHg
B. Symptoms usually correlate with the amount an a heart rate of 130 beats per minute an
of bile entering the stomach has iffuse abominal tenerness with reboun
C. In symptomatic patients, meical management an guaring. The rectal examination is guaiac
is generally effective positive. Laboratory values are signicant for a
D. Creation of a Roux-en-Y gastrojejunostomy is white bloo cell count of 16,000 cells/μL an a
an effective surgical option hematocrit of 6%. CT emonstrates extravasation
E. Most patients with bile reux into the stomach of oral contrast in the proximal uoenum. After
will evelop symptoms resuscitation, management consists of:
A. Closure of the perforation with omental patch
22. The best test for localization of a gastrinoma is: plus an HSV
A. MRI B. Closure of the perforation an omental patch
B. CT via the open approach
C. Abominal ultrasoun C. Perform uoenotomy over perforation,
D. Octreotie scan oversew posterior ulcer, close uoenotomy,
E. Selective angiography an place omental patch
D. Vagotomy an antrectomy with oversewing of
23. The best test to conrm eraication of H. pylori the posterior ulcer an omental patch
after treatment is: E. Closure of the perforation an omental patch
A. H. pylori serology via laparoscopic approach
B. Urea breath test
C. Rapi urease test 28. A 50-year-ol woman with a history of iabetes
D. Histologic biopsy presents with symptoms of early satiety, nausea,
E. Antral mucosal biopsy with culture vomiting, an epigastric pain. Upper enoscopy
reveals a large mass of unigeste foo particles
24. Which of the following is true regaring a highly in the stomach that is partially obstructing the
selective vagotomy (HSV)? pylorus. Which of the following is true regaring
A. The anterior an posterior vagal trunks are this conition?
ivie A. Most patients require surgery
B. The nerve of Grassi is spare B. It can be treate with oral aministration of
C. The anterior Latarjet nerve is ivie cellulase
D. The crow’s feet to the antrum are spare C. Psychiatric treatment is critical in long-term
E. The celiac branch is ivie management
D. The patient likely has patchy areas of alopecia
25. The most common metabolic isorer after E. Peptic ulcer isease is a risk factor
gastric resection is a eciency of:
A. Iron
B. Vitamin B1
C. Folate
D. Calcium
E. Vitamin D
96 PArt i Patient Care

29. A 70-year-ol man presents to the ED with 32. A 4-year-ol alcoholic male with recurrent
suen onset of severe epigastric pain associate episoes of pancreatitis presents to the ED with
with retching but with little vomitus. His one episoe of hematemesis in the morning.
bloo pressure is 140/90 mmHg an his heart He oes not appear to have any active bleeing
rate is 90beats per minute. Attempts by the currently. CT scan emonstrates splenic artery
ED physician to place a nasogastric tube are thrombosis. Lipase an liver function tests are
unsuccessful. An upright chest raiograph reveals normal. EGD emonstrates isolate gastric
a large gas bubble just above the left iaphragm. varices that are not currently bleeing an
Which of the following is true regaring this one -cm ulcer at the angularis. Which of the
conition? following enoscopic features confers the lowest
A. The stomach is likely twiste along the axis, risk of rebleeing?
transecting the lesser an greater curvature A. Oozing ulcer
B. In chilren it is largely ue to a B. Nonbleeing ulcer with overlying clot
paraesophageal hernia C. Nonbleeing visible vessel
C. It is associate with Bergman’s tria D. Visible ulcer base
D. Percutaneous gastrostomy tube for enitive E. Flat pigmente spot
management is acceptable in select patients
E. It is initially manage conservatively for the 33. Which of the following is true regaring gastric
majority of patients polyps?
A. Funic gastric polyps have the highest risk of
30. Which of the following escribes the association harboring malignant cells
between Sister Mary Joseph noule an gastric B. Aenomatous gastric polyps are the most
cancer? common type
A. A metastatic left axillary lymph noe C. Hamartomatous polyps are associate with
B. A metastatic left supraclavicular lymph noe H.pylori infection
C. An ovarian mass from gastric metastasis D. Heterotopic polyps most commonly present
D. Umbilical metastasis suggesting with gastrointestinal bleeing
carcinomatosis E. Inammatory polyps o not have a risk of
E. An anterior noule palpable on rectal malignancy
examination suggesting rop metastasis
34. Bleeing from a Dieulafoy gastric lesion is ue to:
31. A 68-year-ol woman presents with an upper GI A. Antral vascular ectasia
hemorrhage. She has a history of ulcer isease B. Abnormal gastric rugal fols
an has recently complete a treatment for H. C. Ingeste foreign material
pylori. Upper enoscopy reveals brisk arterial D. An abnormal submucosal vessel
bleeing from a uoenal ulcer locate on the E. A premalignant lesion
posterior wall. Despite numerous attempts to
control the bleeing enoscopically, the ulcer 35. The most sensitive an specic iagnostic test for
continues to blee. The patient has receive 4 gastrinoma is:
units of bloo. Her hematocrit is 5%, her bloo A. Basal an stimulate gastric aci outputs
pressure is 110/60 mmHg, an her heart rate is B. Octreotie scan
10 beats per minute. Which of the following is C. Fasting serum gastrin
the best management option? D. Calcium stimulation test
A. Duoenotomy, oversewing the ulcer, truncal E. Secretin stimulation test
vagotomy, an pyloroplasty
B. Duoenotomy an oversewing the ulcer
C. Truncal vagotomy an antrectomy with
Billroth I reconstruction
D. Truncal vagotomy an antrectomy with
Billroth II reconstruction
E. Highly selective vagotomy
CHAPtEr 8 Alimentary Tract—Stomach 97

Answers
1. A. Lymphoepithelial tissue on biopsy is virtually pathog- ahesive SBOs, which can be ecompresse with vomiting,
nomonic for gastric MALT lymphoma, an inolent malig- obstruction of the afferent loop via an internal hernia causes
nancy primarily thought to arise from chronic H. pylori a close loop obstruction with a high risk of perforation with
infection (gram-negative spirochete) (C). Treatment with urgent surgical exploration inicate to relieve the obstruc-
triple therapy or quaruple therapy antibiotics to erai- tion. Eating smaller, more frequent meals is the rst-line
cate H. pylori is the rst-line treatment for patients with treatment of early umping synrome postgastrectomy (A).
early-stage 1 or  isease. Most cases take 1 year to achieve Nasogastric tube ecompression with water-soluble contrast
remission; however, although rare, it can take up to 3 years. challenge is the conservative management for ahesive SBO
Refractory cases, as well as stage 3 an 4 isease, require an is not appropriate in this patient (C). Antibiotic therapy
chemoraiation using CHOP (cyclophosphamie, oxoru- is a treatment for small intestine bacterial overgrowth (SIBO)
bicin, vincristine, an prenisone) (E). Surgical resection is but not for bowel obstruction (D). While upper enoscopy
reserve for cases complicate by perforation, bleeing, or woul be useful in the iagnosis of reux gastritis or mar-
obstruction (D). Tumor biology affects the response to antibi- ginal ulcer, it woul be of low utility in this patient (E).
otics, namely the t(11;18) chromosomal translocation that has
a <5% response to antibiotics alone an requires aitional 4. B. This patient with multiple uoenal ulcers likely has
treatment with raiation therapy or rituximab. After suc- Zollinger-Ellison synrome seconary to hypersecretion of
cessful treatment with antibiotics, surveillance enoscopy is gastrin from a gastrinoma. Gastrin is usually prouce by
neee 3 months after treatment to check for eraication of antral G cells an acts on parietal cells to prouce hyrochlo-
H. pylori an to evaluate for recurrence (B). ric aci an chief cells to prouce pepsinogen, both cell types
Reference: Liu H, Ye H, Ruskone-Fourmestraux A, et al. of which are most preominant in the stomach boy (A).
T(11;18) is a marker for all stage gastric MALT lymphomas The pancreas is the site of the secretion of many hormones
that will not respon to H. pylori eraication. Gastroenterology. incluing somatostatin from D cells, insulin from beta cells,
00;1(5):186–194. an glucagon from alpha cells (C). The uoenum an jeju-
num are the sites of the secretion of cholecystokinin (CCK)
2. B. Gastric GIST can be iagnose on upper enoscopy, from I cells an secretin from K cells (D, E). The uoenum is
classically as a submucosal mass with central umbilication the most common site for gastrinomas (50%–88%), followe
an ulceration. On pathologic evaluation, the majority by the pancreas (5%).
express CD117 (c-kit), as well as CD-34. It arises from meso-
ermal-erive components an grows intraluminal (C). 5. A. The most common etiology of small bowel obstruction
Patients thus present late with obstruction or they outgrow in the Unite States is ahesions from previous abominal
the bloo supply, presenting with necrosis an hemorrhage surgery. However, this oes not hol true for patients that
into the gastric lumen (D). GISTs are not raiosensitive or have previously ha an LRYGB. In this proceure, a poten-
responsive to traitional chemotherapy (A). However, the tial hernia site (Petersen space hernia) is create, increasing
KIT proto-oncogene encoes for a receptor tyrosine kinase the risk for the evelopment of an internal hernia, which is
that, when mutate, becomes constitutively active an the most common cause of SBO in this patient population
leas to mitogenic activity an tumorigenesis. Imatinib, a with an incience of 1% to 5%. This potential space results
tyrosine kinase inhibitor (TKI), has emerge as an import- from herniation of intestinal loops through a efect in the
ant ajunct in the management of gastric GISTs, with high mesentery an between small bowel limbs, transverse meso-
response rates: 90% for exon 11 mutations an 50% for exon colon, an the retroperitoneum. Aitionally, when com-
9 mutations in KIT. Neoajuvant therapy with imatinib can pare to its open counterpart, the laparoscopic approach
be use to ownsize tumors when upfront surgical resec- further facilitates a Petersen hernia because of the ecrease
tion woul result in signicant morbiity. Ajuvant ima- frequency of postoperative ahesions, which seemingly have
tinib is inicate in patients with high-risk features for a physiologic role of preventing bowel mobility, an thus,
recurrence: extragastric tumors >5 cm or >5 mitoses/50 hpf internal herniation. Risk of SBO is signicantly higher with
or gastric tumors >10 cm or >5 mitoses/50 hpf, or patients a retrocolic versus an antecolic approach. Roux compression
with tumor rupture. The most common averse effect of ue to mesocolon scarring is the secon most common etiol-
imatinib is eema. While small Tis or T1a gastric aenocar- ogy for SBO in patients with LRYGB followe by ahesions
cinomas can be treate with enoscopic resection, complete (B, C). Kinking of the jejunojejunostomy an incarcerate
resection of gastric GISTs typically requires at least a wege abominal wall hernia occur less frequently (D, E).
resection (E). Reference: Champion JK, Williams M, Husain S, Johnson AR.
Small-bowel obstruction after laparoscopic Roux-en-Y gastric bypass:
3. B. One of the more feare postoperative complications etiology, iagnosis, an management. Arch Surg. 003;13(4):988–993.
following Roux-en-Y gastric bypass is acute afferent loop
synrome with small bowel obstruction (SBO) of the bil- 6. A. Nonoperative management for perforate peptic
iopancreatic limb. A high inex of suspicion is neee to ulcer isease is gaining popularity an is now accepte as an
iagnose this complication, usually base on patient symp- appropriate rst-line management for poor surgical cani-
toms an evience of proximal SBO on imaging. Unlike most ates (e.g., COPD using home oxygen) who are stable, have
98 PArt i Patient Care

no evience of peritonitis, an have no contrast extravasa- proceures result in up to 50% resolution of weight-relate
tion. Conservative management is also more favorable if the comorbiities an up to 50% excess weight loss. Compare
uration of symptoms has laste more than 4 hours. By this to BPD, RYGB has a lower 30-ay mortality an is slightly
time the perforation has typically been seale. Self-sealing favore by surgeons as it is technically easier to perform.
of the perforation is achieve by either ahesion formation RYGB has a slightly higher mean excess weight loss at  years
to the cauate lobe, the greater omentum, the gallblaer, compare to sleeve gastrectomy, but sleeve gastrectomy has
or the falciform ligament. In one stuy, only 3 out of 109 a higher perioperative leak rate. Both proceures are equally
patients manage nonoperatively evelope an intraab- effective in eliminating type  iabetes mellitus.
ominal abscess (which can be manage with antibiotics References: Duarte MIX e T, Bassitt DP, Azeveo OC e,
an percutaneous rainage). This may speak to the intrinsic Waisberg J, Yamaguchi N, Pinto Junior PE. Impact on quality of life,
immune function of the omentum an the fact that the upper weight loss an comorbiities: a stuy comparing the biliopancre-
GI tract has a low bacterial loa. Eighty percent of nonoper- atic iversion with uoenal switch an the bane Roux-en-Y gas-
tric bypass. Arq Gastroenterol. 014;51(4):30–37.
ative cases respon favorably, an morbiity is not signi-
Santry HP, Gillen DL, Lauerale DS. Trens in bariatric surgical
cantly increase. Patients eeme appropriate caniates
proceures. JAMA. 005;94(15):1909–1917.
for nonoperative management shoul be amitte, place O’Brien P. Surgical treatment of obesity. Endotext. 016;19:9–46.
NPO (nothing by mouth), an given IV ui resuscitation, Zingmon DS, McGory ML, Ko CY. Hospitalization before an
IV antibiotics covering gram-negative an anaerobic organ- after gastric bypass surgery. JAMA. 005;94(15):1918–194.
isms, an PPIs. Nasogastric tube insertion is critical to help
ecompress the stomach an allow the perforation to heal. 8. C. Gastroparesis is ene as elaye gastric emptying
CT scan may be consiere for patients who fail to improve without a mechanical cause for obstruction. Although iabe-
or those who eteriorate clinically. Surgery is the next step tes is the most common known cause of gastroparesis (9%),
for patients who fail conservative management (B, C). Out- iiopathic gastroparesis occurs more frequently (36%). The
patient follow-up is not appropriate because nonoperative most common symptoms are nausea, early satiety, an
management shoul be performe in a monitore setting abominal bloating. Most patients o not have abominal
with frequent abominal exams an follow-up esophagogas- pain. Although symptoms alone can be suggestive of this
trouoenoscopy (EGD) to ensure that the perforation has conition, it nees to be conrme by imaging (E). Gastric
seale (D, E). emptying scintigraphy (elaye gastric emptying stuy) is
References: Nusree R. Conservative management of perforate the gol stanar in iagnosing gastroparesis. This involves
peptic ulcer. Thai J Surg. 005;6:5–8. asking the patient to eat a small meal along with a raioac-
Hanumanthappa MB, Gopinathan S, Guruprasa R. A non- tive tracer. The rate of emptying is measure 1, , 3, an 4
operative treatment of perforate peptic ulcer: a prospective stuy
hours after the meal is ingeste. If more than 10% of the meal
with 50 cases. J Clin Diagnostic Res. 01;41:4161.
remains in the stomach after 4 hours, the stuy is consiere
consistent with gastroparesis (A, B, D).
7. D. Obesity has been linke to multiple comorbiities,
incluing hypertension an iabetes, an is on the rise. As 9. E. Gallstone formation occurs in 30% to 5% of patients
such, many clinicians have turne to meical management unergoing weight loss surgery, but only 7% to 15% are
an/or bariatric surgery to help ght this epiemic in cases symptomatic. Among those 7% to 15% who o become
where iet an exercise fail. Two FDA-approve meications symptomatic, acute cholecystitis is uncommon. Rapi
to help treat obesity inclue sibutramine an orlistat. Sibutra- weight loss is a known risk factor for cholelithiasis. In fact,
mine blocks the presynaptic uptake of serotonin, thereby excess weight loss greater than 5% is consiere the stron-
potentiating its anorexic effects in the CNS. Orlistat inhibits gest preictor of postoperative cholecystectomy an occurs
pancreatic lipase, which ecreases ietary fat absorption an more commonly in patients who have ha a gastric bypass
results in weight loss (B). A signicant complication limiting versus laparoscopic baning or sleeve gastrectomy (A). Sev-
its use for most patients is severe atulence. Inications for eral mechanisms have been shown to contribute to gallstone
weight loss surgery inclue BMI >35 with associate obesity- formation uring weight loss incluing increase secretion
relate comorbiities (e.g., hypertension, iabetes) or BMI of calcium an mucin into bile, increase concentrations of
>40 (A). Aitionally, all patients will nee to emonstrate arachionic aci erivatives, an bile stasis seconary to
that they have successfully attempte an faile nonoper- stringent ietary restrictions postoperatively (D). Prophy-
ative weight loss management such as iet an exercise lactic cholecystectomy at the time of weight loss surgery
programs (E). Patients will also nee to be evaluate by a has been a point of ebate in the surgical community. Pro-
physiatrist an eeme suitable for the proceure. The four ponents argue that it helps prevent the morbiity of symp-
stanar approaches in the Unite States inclue laparo- tomatic biliary isease an avois the nee for treatments
scopic gastric baning, sleeve gastrectomy, BPD, an RYGB. such as enoscopic retrograe cholangiopancreatography
Laparoscopic gastric baning an sleeve gastrectomy are (ERCP), which can be particularly challenging in this patient
consiere restrictive proceures as they physically limit population (e.g., RYGB). However, it has been shown in sev-
the intake of foo. BPD is consiere a malabsorptive pro- eral large stuies that the rate of postoperative cholecystec-
ceure as it involves constructing an alimentary channel is- tomy remains uner 15%; therefore, the routine removal of
tally to the GI tract an thereby preventing the absorption the gallblaer uring weight loss surgery is not currently
of caloric intake. RYGB is consiere a combine approach supporte by the American Society of Metabolic an Bar-
an involves creating a small restricte gastric remnant iatric Surgery (B). In contrast, symptomatic patients may
(restrictive component) an a roux-limb from the stomach unergo concomitant cholecystectomy safely. Ursoiol after
to the istal jejunum (malabsorptive component) (C). These gastric bypass can signicantly ecrease the rate of gallstone
CHAPtEr 8 Alimentary Tract—Stomach 99

formation, but because it has not been shown to be cost effec- Naef M, Naef U, Mouton WG, Wagner HE. Outcome an compli-
tive an lea to improve outcomes, it is not routinely rec- cations after laparoscopic Sweish ajustable gastric baning: 5-year
ommene (C). results of a prospective clinical trial. Obes Surg. 007;17():195–01.
References: D’Hont M, Sergeant G, Deylgat B, Devrient D, Stroh C, Hohmann U, Will U, etal. (008).
Van Rooy F, Vansteenkiste F. Prophylactic cholecystectomy, a man-
atory step in morbily obese patients unergoing laparoscopic 12. E. In the majority of patients with a perforate uo-
Roux-en-Y gastric bypass? J Gastrointest Surg. 011;15(9):153–1536. enal ulcer, simple closure of the ulcer with an omental
Shiffman ML, Shamburek RD, Schwartz CC, Sugerman HJ, Kel- (Graham) patch is all that is necessary (A). This is then fol-
lum JM, Moore EW. Gallblaer mucin, arachionic aci, an bile lowe by treatment of H. pylori. In aition, a Graham patch
lipis in patients who evelop gallstones uring weight reuction. alone shoul be use if the patient is unstable, if there is
Gastroenterology. 1993;105(4):100–108.
extensive exuative peritonitis, or if the perforation is long
Sugerman HJ, Brewer WH, Shiffman ML, et al. A multicenter,
placebo-controlle, ranomize, ouble-blin, prospective trial of pro-
staning (>4 hours). However, in the setting of a patient
phylactic ursoiol for the prevention of gallstone formation following with a known ulcer iathesis who has either alreay been
gastric-bypass-inuce rapi weight loss. Am J Surg. 1995;169(1):91–96. treate for H. pylori or is H. pylori negative, an ulcer surgery
Tucker ON, Fajnwaks P, Szomstein S, Rosenthal RJ. Is concomi- shoul be ae to the operation, provie the patient is
tant cholecystectomy necessary in obese patients unergoing lapa- a goo operative risk, is hemoynamically stable, an oes
roscopic gastric bypass surgery? Surg Endosc. 008;(11):450–454. not have extensive peritonitis. The options are either to per-
Villegas L, Schneier B, Provost D, et al. Is routine cholecys- form a highly selective vagotomy (HSV) or a vagotomy an
tectomy require uring laparoscopic gastric bypass? Obes Surg. pyloroplasty (B). An HSV is the preferre approach in the
004;14():06–11. stable goo-risk patient, provie the surgeon is comfort-
able with the proceure. Pyloroplasty is typically performe
10. B. The rate of obesity is rising in the Unite States an along with a vagotomy because the wiene outlet from the
an increasing number of patients are unergoing weight
stomach to the uoenum helps circumvent any unwante
loss surgery. Gastric leak in the early postoperative perio
effects of the ecrease gastric peristalsis an overall change
may be an inication to go back to the operating room, so
in gastric emptying patterns that occur following vagotomy.
early recognition of this complication is important. The rst
The entire proceure can be performe laparoscopically in
manifestations of a gastric leak are tachycaria an fever (A,
select patients. Truncal vagotomy an antrectomy (C, D)
C–E). This may also be accompanie by tachypnea, abomi-
is generally not recommene in the setting of perforation
nal pain, chest pain, oliguria, an/or hypotension.
because of the high associate morbiity an mortality rates.
Reference: Bekehit M, Katri K, Nabil W. Earliest signs an man-
References: Caiere GB, Bruyns J, Himpens J, Van Alphen P,
agement of leakage after bariatric surgeries: single institute experi-
Verturyen M. Laparoscopic highly selective vagotomy. Hepatogastro-
ence. Alexandria J Med. 013;49(1):9–33.
enterology. 1999;46(7):1500–1506.
Joran PH Jr, Thornby J. Perforate pylorouoenal ulcers: long-
11. A. Laparoscopic gastric baning involves placing an term results with omental patch closure an parietal cell vagotomy.
inatable balloon aroun the proximal stomach at the angle Ann Surg. 1995;1(5):486–488.
of His. A properly place lap ban will have an approxi- Siu WT, Leong HT, Law BKB, et al. Laparoscopic repair for
mately 45° upwar angle from the horizontal plane on a perforate peptic ulcer: a ranomize controlle trial. Ann Surg.
plain lm of the abomen. The proceure was very popular 00;35(3):313–319.
when it rst appeare but lost traction after subsequent stu-
ies emonstrate that it was far inferior to gastric bypass. 13. E. Postvagotomy synromes inclue iarrhea, gas-
Aitionally, patients with laparoscopic bans were more tric atony, an incomplete vagotomy (leaing to recurrent
likely to require revisions for complications associate with ulceration). Diarrhea follows truncal vagotomy an may be
the gastric ban. One such complication is ban erosion (BE) confuse with umping synrome. The iarrhea associate
into the stomach an/or ajacent organs. This may present with vagotomy occurs more frequently an is not associ-
as port site erythema (inammation tracking own the tube), ate with the other cariovascular manifestations seen with
fooling the clinician into thinking the patient may only have umping synrome (C). The initial treatment is similar to
an overlying skin infection. In fact, most patients with BE that for umping synrome, with ietary moications
presenting with port site erythema o not have a subfascial such as frequent small meals with ecrease ui intake
port infection. BE can occur many years after surgery, an an an increase in ber. A propose mechanism of the iar-
one propose mechanism involves overtightening of the rhea is an increase in stool bile salts. Oral cholestyramine is
ban (e.g., after clinic visit). CT of the abomen shoul be often helpful because it bins bile salts (B). Loperamie an
performe in patients suspecte of having BE an, if foun, coeine have also been shown to elay intestinal transit time
the port site shoul be completely eate an the patient an improve symptoms. In the very rare patient who oes
shoul be scheule for laparoscopic removal of the ban. not respon to meical management, reversal of a segment
EGD may emonstrate BE if it has completely eroe into of jejunum is effective in slowing transit time an improving
the gastric lumen but may miss partial BE (C). Incision an iarrhea (D). Octreotie is not effective for postvagotomy
rainage are not inicate because there is no abscess (D). iarrhea an may make the situation worse by ecreasing
The patient shoul be monitore for the evelopment of a pancreatic secretions an thus increasing steatorrhea (A).
subsequent port site infection, but the rst step is to get a CT References: Duncombe V, Bolin T, Davis A. Double-blin trial of
scan (B–E). cholestyramine in post-vagotomy iarrhea. Gut. 1977;18(7):531–535.
References: Dilorenzo N, Lorenzo M, Furbetta F. Intragastric O’Brien JG, Thompson DG, Mcintyre A. Effect of coeine an lop-
gastric ban migration an erosion: an analysis of multicenter expe- eramie on upper intestinal transit an absorption in normal subjects
rience on 177 patients. Surg Endosc. 013;7(4):1151–1157. an patients with postvagotomy iarrhea. Gut. 1988;9(3):31–318.
100 PArt i Patient Care

14. D. More than 80% of gastrinomas are localize preoper- References: Seino Y, Matsukura S, Inoue Y, Kaowaki S, Mori
atively. For those that cannot be localize, surgical explora- K, Imura H. Hypogastrinemia in hypothyroiism. Am J Dig Dis.
tion is still inicate because excision of the primary tumor 1978;3():189–191.
leas to a ecrease rate of liver metastasis. When exploring, Korman MG, Laver MC, Hansky J. Hypergastrinemia in chronic
renal failure. BMJ. 197;1(5794):09–10.
it is important to be aware that 80% of gastrinomas are foun
within the gastrinoma (Passaro) triangle, an area ene by
the junction of the cystic uct an common bile uct, the
17. D. R0 resection is resection of all gross an microscopic
tumors (C). R1 inicates removal of all macroscopic isease
secon an thir portions of the uoenum, an the neck
but microscopic margins are positive for isease. An R
an boy of the pancreas. As many as 60% of gastrinomas
resection inicates that gross resiual isease is left behin
are within the wall of the uoenum, primarily in the rst
(E). A D1 resection (A) refers to removal of perigastric lymph
an secon portions an can be very small. Thus, the next
noes; D (B) refers to the aitional resection of lymph
maneuver woul be to perform a blin proximal uoenot-
noes along the name vessels aroun the stomach. A D3
omy to manually palpate the uoenal wall for tumors. Clos-
resection is a D resection plus removal of para-aortic lymph
ing the abomen (A) woul be inappropriate. Blin istal
noes.
pancreatectomy an splenectomy (these share bloo supply)
(B) or istal uoenotomy (E) woul have very low yiels. A
pancreaticouoenectomy (Whipple proceure) (C) woul 18. A. Risk factors for gastric cancer inclue ietary factors
not be inicate in this setting. It is potentially inicate for such as a large consumption of smoke meats, pickle foos,
multiple uoenal or proximal pancreatic hea tumors that high nitrates, an high salt, whereas a iet high in fruits an
coul not be enucleate. vegetables may be protective (D, E). Other risk factors inclue
smoking, low socioeconomic status, Black race, H. pylori infec-
15. C. Achieving an aequate lymphaenectomy with a tion, chronic atrophic gastritis, bloo type A, previous partial
≥15 lymph noe harvest uring an oncologic resection of gastrectomy, achlorhyria, pernicious anemia, polyps (ae-
gastric cancer is important in accurately staging the patient nomatous an hyperplastic), male sex, an certain familial
an reucing the noal false negative rate. Staging of gas- synromes such as hereitary nonpolyposis colorectal can-
tric cancer involves epth of invasion (T1 invaes lamina cer, Li-Fraumeni synrome, familial aenomatous polyposis,
propria; T, muscularis propria or subserosa; T3, serosa; T4, an Peutz-Jeghers synrome (B, D). Peutz-Jeghers synrome
ajacent structures), noes, an istant metastasis (E). EUS is associate with a markely increase risk of cancer in the
is the best moality for assessing tumor epth of invasion esophagus, stomach, small bowel, colon, pancreas, breast,
an noal status. It is approximately 80% accurate in eter- lung, uterus, an ovary, with a cumulative 93% risk of can-
mining whether the tumor is transmural (invaing serosa, cer. Carbonate aciic soa has not been shown to increase
T3) but only 50% accurate in assessing whether pathologi- the risk for cancer (C). Gastric cancer has been categorize by
cally enlarge lymph noes are present (B). EUS seems to Lauren into intestinal an iffuse types base on histology.
be more accurate with avance isease than early isease. The intestinal type is thought to be more relate to environ-
CT scanning is the preferre metho for etermining istant mental factors, is associate with chronic gastritis, an is well
metastases, but it is not as useful for T an N staging (A). The ifferentiate. The iffuse type is usually poorly ifferenti-
routine use of MRI an positron emission tomography scan- ate an associate with signet rings an occurs in younger
ning for staging of gastric cancer has not as yet been estab- patients an in association with familial isorers an with
lishe (D). N1 isease inclues 1 to 6 regional noes; N, 7 type A bloo. The iffuse type has a worse prognosis.
to 15 regional noes; an N3, more than 15 regional noes. References: Bernt H, Wilner GP, Klein K. Regional an
References: Puli SR, Batapati Krishna Rey J, Bechtol ML, social ifferences in cancer incience of the igestive tract in the
Antillon MR, Ibah JA. How goo is enoscopic ultrasoun for German Democratic Republic. Neoplasma. 1968;15(5):501–515.
TNM staging of gastric cancers? A meta-analysis an systematic Giariello FM, Brensinger JD, Tersmette AC, et al. Very high
review. World J Gastroenterol. 008;14(5):4011–4019. risk of cancer in familial Peutz-Jeghers synrome. Gastroenterology.
Willis S, Truong S, Gribnitz S, Fass J, Schumpelick V. Enoscopic 000;119(6):1447–1453.
ultrasonography in the preoperative staging of gastric cancer: accu- Wyner EL, Kmet J, Dungal N, Segi M. An epiemiological
racy an impact on surgical therapy. Surg Endosc. 000;14(10):951–954. investigation of gastric cancer. Cancer. 1963;16(11):1461–1496.
Xi W, Zhao C, Ren G. Enoscopic ultrasonography in preop-
erative staging of gastric cancer: etermination of tumor invasion 19. B. Gastric ulcers have been categorize into ve types.
epth, noal involvement an surgical respectability. World J Gastro- The most common is the type I lesion (≈60%) (A), which is
enterol. 003;9():54–57. locate near the angularis incisura at the borer between the
antrum an the funus, usually along the lesser curve. These
16. D. When consiering gastrinoma, it is important to patients usually have normal or ecrease aci secretion.
be aware of the ifferential iagnosis of an elevate gastrin Type II gastric ulcers are locate in the funus an are asso-
level. Causes of hypergastrinemia with increase aci prouc- ciate with a concomitant uoenal ulcer. Type III gastric
tion inclue gastrinoma, G-cell hyperplasia (not D-cell) (E), ulcers are prepyloric. Both types II an III gastric ulcers are
retaine antrum after istal gastrectomy, renal failure, an gas- usually associate with increase gastric aci secretion. Type
tric outlet obstruction. Hypergastrinemia with normal or low III ulcers are thought to behave like uoenal ulcers. Type IV
aci prouction inclues pernicious anemia, postvagotomy gastric ulcers are locate near the gastroesophageal junction.
states, use of aci-suppressive meication, an chronic gas- Like type I ulcers, type IV gastric ulcers have normal or low
tritis. Hypothyroiism is associate with a low gastrin level, aci prouction an are associate with impaire mucosal
whereas hyperthyroiism increases gastrin levels (B). Diabetes efense. Type V gastric ulcers are consiere a iffuse pro-
(A) an hyperparathyroiism (C) o not affect gastrin levels. cess an are associate with NSAID use.
CHAPtEr 8 Alimentary Tract—Stomach 101

20. D. GISTs were previously calle leiomyomas or leiomyo- 22. D. More than 90% of gastrinomas have receptors for
sarcomas because they were thought to arise from smooth somatostatin. Octreotie scanning (somatostatin receptor
muscle cells, but they in fact originate from mesenchymal scintigraphy) has been shown to be the most sensitive test
components (from Cajal cells) (B). They stain positive for for localization of gastrinomas. However, successful local-
CD117 (c-kit). They are most commonly foun in the stom- ization epens on size an location. Somatostatin receptor
ach an, although rare, they are the most common mesen- scintigraphy is poor for very small tumors (<1.1 cm) an for
chymal tumors of the intestinal tract. Because they are not small primary uoenal tumors. Duoenal gastrinomas are
epithelial tumors an grow in the wall of the stomach, they best localize by enoscopic ultrasonography. Abominal
ten to be large at the time of presentation. They cause muco- ultrasoun is not helpful (C). Failure to etect the tumor pre-
sal ulceration an frequently present with GI bleeing (E). operatively shoul not preempt surgical exploration because
Large tumors may also prouce symptoms of weight loss, an aitional 33% will be foun at surgery. CT an angiog-
abominal pain, an fullness an early satiety. An abominal raphy may also be useful ajuncts in etecting gastrinoma
mass may be palpable. An enoscopic biopsy specimen may (B, E). Asie from MRI’s utility in etecting liver metastasis,
be negative in as many as one-half of cases ue to sampling it is not often employe in the workup for a presume gas-
error because most of the tumor is submucosal (A). A CT trinoma (A).
scan provies a better assessment of the extent of the tumor. Reference: Alexaner HR, Fraker DL, Norton JA, et al. Prospec-
Determining whether a GIST is malignant is not straightfor- tive stuy of somatostatin receptor scintigraphy an its effect on
war because there are no iscriminating cellular features operative outcome in patients with Zollinger-Ellison synrome. Ann
(C). The malignant potential is etermine by mitotic activ- Surg. 1998;8():8–38.
ity (>5 mitoses/50 high power el) with 1 cm. Lymph noe
issection is not necessary because tumors sprea hema- 23. B. A urea breath test is the best way to conrm erai-
togenously an lymph noe metastasis is extremely rare. cation of H. pylori. The test relies on the fact that the bacte-
Wege resection with 1 cm margins is aequate treatment ria hyrolyze urea. The patient is given raiolabele urea to
in most cases. This can be performe laparoscopically. How- ingest orally. If H. pylori is present, the urea will be converte
ever, microscopically positive margins have not been emon- to ammonia an raiolabele bicarbonate, which is then
strate to affect survival. exhale as carbon ioxie. The amount of exhale carbon
References: Dempsey DT. Stomach. In: Brunicari FC, Aner- ioxie is quantie. Positive H. pylori serology (A) provies
sen DK, Billiar TR, etal., es. Schwartz’s principles of surgery. 8th e. evience of current infection if the patient has never been
New York: McGraw-Hill; 005:933–996. treate for it but will remain positive even after successful
Mercer DW, Robinson EK. Stomach. In: Townsen CM, Jr, Beau- treatment; thus, it is not useful in this setting. Antral mucosa
champ RD, Evers BM, Mattox KL, es. Sabiston textbook of surgery: the biopsy (E) with histologic examination (D) for the organism
biological basis of moern surgical practice. 17th e. Philaelphia: is the gol stanar test. It is useful in the initial evaluation
W.B. Sauners; 004:165–13.
of patients with upper GI symptoms because it permits eval-
Novitsky YW, Kercher KW, Sing RF, Henifor BT. Long-term out-
uation of the stomach via enoscopy at the time of biopsy.
comes of laparoscopic resection of gastric gastrointestinal stromal
tumors. Ann Surg. 006;43(6):738–745.
However, given its invasive nature an increase cost, it is
Sexton JA, Pierce RA, Halpin VJ, et al. Laparoscopic gastric resection not routinely recommene to conrm eraication. Cultures
for gastrointestinal stromal tumors. Surg Endosc. 008;(1):583–587. of the gastric mucosa are not routinely available at every lab-
Malangoni MA, etal. Stomach. In: Cameron JL, e. Current surgi- oratory, an a repeat enoscopy is require. The rapi urease
cal therapy. 8th e. Philaelphia: Mosby; 004:67–100. test, also known as the campylobacter-like organism (CLO)
test (C), is ieally use if another enoscopy an biopsy are
21. D. Bile reux into the stomach can occur without previ- being performe. The stuy requires placing a sample of gas-
ous surgery, but in most instances it follows ablation of the tric mucosa in a urea solution an then using a pH inicator
pylorus, such as after gastric resection or pyloroplasty. After to emonstrate the prouction of ammonia.
such proceures, most patients will have bile in the stomach
on enoscopic examination, along with some egree of gross 24. D. HSV is also known as a parietal cell vagotomy or
or microscopic gastric inammation. However, only a small proximal gastric vagotomy. The goal of the operation is to
fraction of patients will have a signicant egree of symptoms ivie the vagal nerves of the proximal two-thirs of the
such as nausea, epigastric pain, an bilious vomiting consis- stomach where the parietal cells are locate an preserve the
tent with alkaline (bile) reux gastritis (B, E). Symptoms often istal thir to maintain antral function an thus not require
evelop months or years after the inex operation. The if- a rainage proceure (such as a pyloroplasty). This results
ferential iagnosis inclues afferent or efferent loop obstruc- in fewer complications than the classic truncal vagotomy.
tion, gastric stasis, an small bowel obstruction. These other The operation spares the main anterior an posterior vagal
iagnoses can be rule out using a combination of abomi- trunks (A) but ivies the branches of the anterior an pos-
nal raiographs, upper enoscopy, an abominal CT scan. terior Latarjet nerves that irectly innervate the proximal
A hepatoiminoiacetic aci (HIDA) scan is particularly help- stomach (C). The istal 7 cm (approximately) of nerves,
ful for emonstrating bile reux. Bile reux an gastritis are known as the crow’s feet, are spare. Likewise, the celiac
more likely to occur after Billroth II reconstruction (A) than an hepatic branches are spare (E). Proximally, it is import-
after Billroth I an least likely after vagotomy an pyloro- ant to ivie the nerve of Grassi, which is a branch off the
plasty. Meical management of symptomatic patients is not posterior trunk of the vagus (B). It is often referre to as the
particularly effective (C). The surgical proceure of choice is criminal nerve of Grassi because failure to ivie this branch
to convert the Billroth II into a Roux-en-Y gastrojejunostomy leas to a higher ulcer recurrence rate. With the recognition
with a lengthene jejunal limb (at least 45 cm). of H. pylori as the main etiology of peptic ulcer, the role of
102 PArt i Patient Care

surgery has greatly iminishe. HSV is still inicate in cer- The majority of anterior perforate ulcers can be manage
tain rare situations, such as patients who o not respon to by simple ulcer closure with an omental (Graham) patch.
meical management, patients who are bleeing who o not This can be achieve via an open or laparoscopic approach.
respon to enoscopic management, or with perforation in In this patient, one must rule out a bleeing posterior ulcer.
patients with a longstaning ulcer iathesis. This woul best be achieve via an anterior uoenotomy
across the pylorus. If a posterior ulcer is ientie, it shoul
25. A. Gastric resection leas to numerous isturbances in be oversewn.
metabolism. These inclue eciencies of iron, vitamin B1 References: Dasmahapatra KS, Suval W, Machieo GW.
(B), folate (C), fat-soluble vitamins (E), an calcium (D). Of Unsuspecte perforation in bleeing uoenal ulcers. Am Surg.
these, iron eciency is the most common. Iron is absorbe 1988;54(1):19–1.
in the uoenum an is facilitate by an aciic environment. Hunt PS, Clarke G. Perforation in patients with bleeing ulcer.
After gastric resection, overall iron intake is ecrease, an ANZ J Surg. 1991;61(3):183–185.
Stabile BE, Hary HJ, Passaro E. “Kissing” uoenal ulcers. Arch
the reuce aciity impairs absorption. Reuction in the
Surg. 1979;114(10):1153–1156.
parietal cell mass from gastric resection leas to a ecrease in
intrinsic factor, which is necessary for the enteric absorption
28. B. Bezoars are accumulations of inigestible material in
of vitamin B1, occurring in the terminal ileum. This leas
the stomach. Bezoars often prouce nonspecic symptoms
to a megaloblastic anemia. Furthermore, an aciic environ-
an are usually foun incientally in patients unergoing
ment facilitates the bioavailability of vitamin B1. Vitamin
upper gastrointestinal enoscopy or imaging. There are two
B1 eciency usually only evelops when at least one-half
types. Phytobezoars are compose of unigeste vegeta-
of the stomach is resecte. Fat malabsorption can occur after
ble matter (as in this patient). Risk factors for phytobezoars
gastrectomy (particularly with a Billroth II reconstruction)
inclue previous gastric surgery an gastroparesis such as
because of inaequate mixing of foo with bile an igestive
from iabetes. Peptic ulcer isease is not a risk factor (E).
enzymes. This leas to a ecrease absorption of fat-soluble
Bezoars prouce obstructive symptoms but can also cause
vitamins. Calcium is absorbe in the uoenum an small
ulceration an bleeing. Diagnosis is suggeste by an upper
bowel an is also facilitate by an aci environment. Long-
GI series an conrme by enoscopy. Treatment generally
term eciencies manifest as osteoporosis. Folate eciency
consists of a combination of enzymatic egraation, eno-
is rare.
scopic isruption, irrigation, an removal. Enzyme therapy
can be performe with papain (present in meat tenerizers)
26. E. ZES (gastrinoma) is cause by uncontrolle secre- or with cellulase. However, the use of papain has been associ-
tion of gastrin by a pancreatic or a uoenal neuroenocrine ate with hypernatremia, gastric ulceration, an esophageal
tumor. Most cases are sporaic, but 0% are inherite (C). perforation, such that cellulase is preferre. More recently,
The inherite or familial form of gastrinoma is associate nasogastric Coca-Cola lavage has been successfully use.
with multiple enocrine neoplasia type 1. Gastrinoma is the The mechanism responsible is believe to be a combination
most common functional neuroenocrine tumor in multiple of the mucolytic effect of soium bicarbonate (NaHCO3)
enocrine neoplasia type 1 but insulinoma is the most com- an igestion of the bezoar by CO bubbles, all of which is
mon overall (D). The most common symptoms are epigastric exaggerate by the cola’s aciity. Trichobezoars are com-
pain, gastroesophageal reux, an iarrhea. The massive pose of hair. It occurs most commonly in girls an young
aci hypersecretion leas to a secretory iarrhea that persists women who swallow their hair (trichophagia). Interestingly,
even with fasting (A). The majority will have emonstra- most have long hair with patchy areas of alopecia (D), an
ble peptic ulceration that is most commonly locate in the many have an unerlying psychiatric isorer; thus psychi-
proximal uoenum (B). Unlike typical ulcers, those associ- atric care is important in prevention (unlike phytobezoars)
ate with gastrinoma on occasion will be foun in the istal (C). The hair creates a cast of the stomach an strans of
uoenum or jejunum. Ulcers in these locations shoul raise hair can exten into the small bowel (the so-calle Rapunzel
suspicion for gastrinoma, as shoul recurrent or refractory synrome). Large trichobezoars are likely to require surgical
peptic ulcers, ulcers in association with secretory iarrhea, removal because they are less likely to respon to enzymatic
ning gastric rugal hypertrophy or esophagitis-relate egraation (A).
stricture on enoscopy, bleeing or perforate ulcer, family References: Bonilla F, Mirete J, Cuesta A, Sillero C, González
history of ulcer, an ulcers in the setting of hypercalcemia M. Treatment of gastric phytobezoars with cellulase. Rev Esp Enferm
or kiney stones. PPIs are highly effective in relieving the Dig. 1999;91(1):809–814.
symptoms of ZES, although enitive treatment consists of Laas SD, Triantafyllou K, Tzathas C, Tassios P, Rokkas T, Raptis
localizing an resecting the tumor. SA. Gastric phytobezoars may be treate by nasogastric Coca-Cola
Reference: Meijer JL, Jansen JB, Lamers CB. Omeprazole in the lavage. Eur J Gastroenterol Hepatol. 00;14(7):801–803.
treatment of Zollinger-Ellison synrome an histamine H-antagonist Walker-Renar P. Upate on the meicinal management of phy-
refractory ulcers. Digestion. 1989;44 Suppl 1:31–39. tobezoars. Am J Gastroenterol. 1993;88(10):1663–1666.

27. C. The presentation of oral contrast extravasation in 29. D. Gastric volvulus is associate with Borchart tria
the proximal uoenum (or free air uner the iaphragm) (suen onset of severe upper abominal pain, recurrent
combine with melena, anemia, an guaiac-positive stool retching without vomitus, an an inability to pass a naso-
is highly suggestive of a “kissing” uoenal ulcer. This rep- gastric tube). Etiology is either primary (ue to congenital
resents a rare combination of an anterior uoenal ulcer that changes in the gastric ligaments) or seconary to anatomic
perforates into the peritoneum an a synchronous posterior abnormalities, usually paraesophageal or iaphragmatic
ulcer that eroes into the gastrouoenal artery an blees. hernias. Even if gastric volvulus is associate with anatomic
CHAPtEr 8 Alimentary Tract—Stomach 103

abnormalities, these o not always nee to be aresse for management inclue the presence of shock or a large ulcer
enitive management. In elerly patients or poor surgical (>cm). Even when bleeing recurs after having been con-
caniates who cannot tolerate a long operation, once the trolle enoscopically, enoscopic treatment can again be
stomach is etorse (either enoscopically or surgically), attempte with a high rate of success, thus avoiing sur-
enitive therapy can consist of as little as a gastropexy gery. The bleeing is usually from a posterior ulcer that has
usually via percutaneous gastrostomy tube. Bergman tria eroe into the gastrouoenal artery (remember anterior
(mental status changes, petechiae, an yspnea) is seen with ulcers cause a free perforation an peritonitis, posterior
fat emboli synrome (C). The volvulus can be either orga- ulcers penetrate an blee). Surgical management ecisions
noaxial (twisting aroun the axis between the gastroesoph- shoul be base on the hemoynamic stability of the patient,
ageal junction an pylorus), which is twice as common, or the patient’s overall meical conition, an whether the
mesenteroaxial (twisting along the axis between the lesser patient has a history of ulcer isease that has been treate
an greater curvature) (A). Gastric volvulus most commonly for H. pylori. In the patient who is actively bleeing, the
occurs in association with a iaphragmatic efect. The stom- uoenum shoul be opene across the pylorus as is use
ach becomes trappe in the efect an twists. In chilren, the in a pyloroplasty. The ulcer be shoul be oversewn with
efect is congenital (such as a Bochalek hernia), whereas multiple gure-of-eight sutures. If the patient has a history
in aults, it is more often traumatic or seconary to parae- of ulcers that have been treate for H. pylori an is stable
sophageal hernias (B). Gastric volvulus can also occur in in the operating room, an ulcer operation shoul be per-
the absence of a iaphragmatic efect. In such situations, forme. The best option in this type of emergent setting is
there is typically a congenital absence of intraperitoneal vis- to perform a truncal vagotomy an to close the longituinal
ceral attachments. It is seen in association with a wanering uoenotomy in a transverse fashion as with a pyloroplasty.
spleen, a conition in which the spleen also lacks peritoneal If the patient is a high surgical risk an unstable, another
attachments an is prone to torsion. Gastric volvulus is a sur- option woul be to simply perform a smaller uoenotomy,
gical emergency because there is a high risk of gastric necro- oversew the ulcer, simply close the uoenotomy, an treat
sis if it is unrecognize (E). If the stomach is compromise, postoperatively for H. pylori (B). Although vagotomy an
a gastric resection may be neee. If a volvulus is foun antrectomy are another option, they woul selom be use
without necrosis an without a iaphragmatic efect, then in the emergent setting because of the higher associate
etorsion an gastropexy are performe. morbiity rate (D, E). An HSV (C) woul not aress the
References: Carter R, Brewer LA 3r, Hinshaw DB. Acute gas- actively bleeing ulcer.
tric volvulus. A stuy of 5 cases. Am J Surg. 1980;140(1):99–106. References: Brullet E, Calvet X, Campo R, Rue M, Catot L,
Uc A, Kao SC, Saners KD, Lawrence J. Gastric volvulus an Donoso L. Factors preicting failure of enoscopic injection therapy
wanering spleen. Am J Gastroenterol. 1998;93(7):1146–1148. in bleeing uoenal ulcer. Gastrointest Endosc. 1996;43():111–116.
Wasselle JA, Norman J. Acute gastric volvulus: pathogenesis, Lau JY, Sung JJ, Lam YH, et al. Enoscopic retreatment compare
iagnosis, an treatment. Am J Gastroenterol. 1993;88(10):1780–1784. with surgery in patients with recurrent bleeing after initial eno-
scopic control of bleeing ulcers. N Engl J Med. 1999;340(10):751–756.
30. D. A metastatic left supraclavicular lymph noe is
calle the Virchow node (Troisier sign) (B). Intraabominal 32. D. Recurrent episoes of acute pancreatitis preispose
cancers ten to metastasize to the left seconary to lymph patients to eveloping splenic vein thrombosis, which can
rainage into the left subclavian vein via the thoracic uct. result in isolate gastric varices. Historically, patients were
A metastatic left axillary lymph noe from gastric cancer is offere a splenectomy as a prophylactic measure to prevent
calle an Irish node (A). A Blumer shelf is a palpable no- severe upper GI bleeing. However, with improve imaging
ule on rectal examination suggesting a rop metastasis (E). we are better able to ientify splenic vein thrombosis, an
An ovarian mass from a gastric metastasis is also known as we now know that only 4% of patients will have clinically
Krukenberg tumor. (C) An umbilical noule (Sister Mary signicant gastric variceal bleeing, so routine splenectomy
Joseph noe) suggests carcinomatosis. Although associ- has fallen out of favor. This patient also has a concomitant
ate with gastric cancer, it may represent any metastatic ulcer, which coul have been contributing to hemateme-
lesion, most commonly from an intraabominal cancer. sis. The Forrest classication graes peptic ulcers base on
It was name after Dr. William Mayo’s surgical assistant, enoscopic features an allows the clinician to etermine
who mae the observation while scrubbing patients for gas- risk of rebleeing. The risk ecreases in the following orer:
tric surgery that those with umbilical noules ha wiely active spurting bleeing (17%–100%), active oozing bleeing
metastatic an unresectable gastric cancer. Current recom- (17%–100%), nonbleeing visible vessel (0%–81%), aherent
menations are that if such noules are foun on physical clot (14%–36%), at pigment spot (0%–13%), an clean vis-
examination, the patient shoul unergo ne-neele aspira- ible ulcer base (0%–10%) (A–C, E). Although patients with
tion because such umbilical noules may sometimes repre- high-risk peptic ulcers (active bleeing/oozing, nonbleeing
sent benign isease. visible vessel) may benet from a secon-look enoscopy,
References: Fleming MV, Oertel YC. Eight cases of Sister Mary current guielines recommen against routine secon-look
Joseph’s noule iagnose by ne-neele aspiration. Diagn Cyto- enoscopy.
pathol. 1993;9(1):3–36. References: Forrest JH, Finlayson NDC, Shearman DJC. Enos-
Giner Galvañ V. Sister Mary Joseph’s noule. Its clinical signi- copy in gastrointestinal bleeing. Lancet. 1974;304(7877):394–397.
cance an management. An Med Interna. 1999;16(7):365–370. Heier TR, Azeem S, Galanko JA, Behrns KE. The natural his-
tory of pancreatitis-inuce splenic vein thrombosis. Ann Surg.
31. A. Bleeing from uoenal ulcers can be controlle 004;39(6):876–88.
enoscopically in the majority of patients; thus surgery Laine L, Jensen DM. Management of patients with ulcer blee-
is rarely inicate. Preictors of failure of enoscopic ing. Am J Gastroenterol. 01;107(3):345–360.
104 PArt i Patient Care

33. E. Hyperplastic polyps are by far the most common who is not controlle enoscopically. Antral vascular ecta-
gastric polyps (70%–90%) (B). Other types inclue aeno- sia (A) is seen in a conition known as watermelon stomach
matous, hamartomatous, inammatory (pseuopolyps), an can lea to signicant acute or chronic GI bloo loss.
funic glan, an heterotopic. Hyperplastic polyps are seen Dilate mucosal bloo vessels containing thrombus, muco-
in association with chronic atrophic gastritis, which is ue sal bromuscular ysplasia, an hyalinization are prominent
to H. pylori infection (C). Hyperplastic polyps are further features. It erives its name from the mucosal vessels that
classie into polypoi foveolar hyperplasia an typical create parallel lines in the mucosal fols (B). The stomach
hyperplastic polyps. Polypoi foveolar hyperplasia oes not is typically not enlarge. It is seen preominantly in elerly
seem to have malignant potential, whereas the typical hyper- women with autoimmune isease or elerly males with
plastic polyp has an approximately % chance of eveloping cirrhosis.
malignancy. Aenomatous polyps have the highest risk of Reference: Selinger CP, Ang YS. Gastric antral vascular ectasia
malignancy (10%–0%), an the risk of malignancy seems (GAVE): an upate on clinical presentation, pathophysiology, an
to be relate to size an histology (greater risk for villous treatment. Digestion. 008;77():131–137.
than tubular) (A). Funic gastric polyps are associate with
long-term PPI use, an the risk of cancer is negligible. Ai- 35. E. The most sensitive an specic test for gastrinoma
tionally, hamartomatous, inammatory, an heterotopic pol- (ZES) is the secretin stimulation test. An IV bolus of secre-
yps o not seem to have a risk of malignancy. Heterotopic tin is aministere, an gastrin levels are checke before
polyps are usually the result of ectopic pancreatic tissue an an after injection. An increase in serum gastrin of 10 pg/
are typically benign lesions without clinical signicance (D). mL or greater has the highest sensitivity an specicity for
However, large heterotopic polyps can lea to obstruction gastrinoma. There are numerous other causes of hypergas-
an intussusception. Treatment for most polyps is simply trinemia. They can be ivie into those associate with an
enoscopic polypectomy. Aitional surgical resection is increase aci prouction an those with a ecrease aci
recommene for polyps that are sessile an larger than  prouction (A). In the latter situation, the hypergastrinemia
cm, those with areas of invasive tumor, an those that cause is reactive ue to hypo- or achlorhyria. In aition to ZES,
symptoms (bleeing or pain). G-cell hyperplasia, gastric outlet obstruction, an retaine
References: Orlowska J, Jarosz D, Pachlewski J, Butruk E. antrum after Billroth II reconstruction are associate with
Malignant transformation of benign epithelial gastric polyps. Am J increase aci prouction. Reactive hypergastrinemia is
Gastroenterol. 1995;90(1):15–159. seen with atrophic gastritis, pernicious anemia, an gastric
Jalving M, Koornstra JJ, Wesseling J, Boezen HM, DE Jong cancer; in patients receiving H-receptor antagonists an
S, Kleibeuker JH. Increase risk of funic glan polyps uring PPIs; an after vagotomy. Hypergastrinemia is also seen
long-term proton pump inhibitor therapy. Aliment Pharmacol Ther. in chronic renal failure ue to ecrease catabolism. Given
006;4(9):1341–1348. this broa ifferential, fasting serum gastrin levels (C) are
not sufciently specic to establish the iagnosis of ZES in
34. D. A Dieulafoy lesion is a congenital malformation in the majority of patients unless gastrin levels are extremely
the stomach (typically on the lesser curvature) characterize high (>1000pg/mL). The secretin stimulation test has higher
by a submucosal artery that is abnormally large an tortu- sensitivity an specicity than the calcium stimulation test
ous. As a result of its relatively supercial location, it may (D). The calcium stimulation test is use if the secretin test
eroe through the mucosa an become expose to gastric result is negative an there is a high suspicion for ZES in
secretions, leaing to massive upper GI hemorrhage. On the presence of hypergastrinemia. Once the iagnosis of ZES
enoscopy, the mucosa of the stomach appears normal, an is establishe, a nuclear octreotie scan (B) seems to be the
the only ning is a pinpoint area of mucosal efect with most sensitive test to localize the tumor.
brisk arterial bleeing. The lesion may easily be misse if the Reference: Berna M, Hoffmann K, Long S. Serum gastrin in
bleeing is not active. Dieulafoy lesion is not premalignant Zollinger-Ellison synrome: II. Prospective stuy of gastrin provoc-
(E) an is not associate with the ingestion of foreign mate- ative testing in 93 patients from the National Institutes of Health
rial (C). Treatment is enoscopic, via electrocautery, heater an comparison with 537 cases from the literature: evaluation of
probe, or injection with a sclerosing agent. Surgery, which iagnostic criteria, proposal of new criteria, an correlations with
consists of a wege resection, is reserve for the rare patient clinical an tumoral features. Medicine. 006;(6):331–364.
9
Alimentary Tract—
SmallBowel
ZACHARY N. WEITZNER, FORMOSA CHEN,
AND BEVERLEY A. PETRIE

ABSITE 99th Percentile High-Yields


I. Duoenum
A. Releases alkaline mucus from Brunner glans to neutralize gastric aci
B. S cells release secretin (inhibits gastric aci, stimulates pancreatic bicarb, increases bile prouction in
liver) an I cells release cholecystokinin (CCK) (inhibits gastric emptying, stimulates pancreatic enzyme
prouction, increases bile, gallblaer contraction, satiety)
C. Leafy appearing villi, absorb iron, eroe in celiac isease
D. Bulb: 1st segment; ulcerations ue to Helicobacter pylori, Zollinger–Ellison synrome
E. Descening: n segment; pancreatic an common bile uct empty
F. Transverse: 3r segment; anterior to inferior vena cava, aorta, vertebral column
G. Ascening: 4th segment; joins jejunum; ens at ligament of Treitz
H. Bloo supply: celiac axis via gastrouoenal artery an SMA axis via pancreaticouoenal arteries

II. Jejunum
A. Does not have Brunner glans; thus marginal ulcers are more likely with Billroth II than with Billroth I
B. Begins at ligament of Treitz
C. Ientiable by long vasa recta from SMA an plicae circularis
D. Dense villi for absorption of water, lipis, NaCl, glucose, amino acis

III. Ileum
A. Three meters long, short vasa recta from SMA, atter mucosa with Peyer patches lymphoi tissue
B. Terminal ileum (TI) is chief site of absorption of B1, folate, an bile salts (conjugate in TI, unconjugate
elsewhere in ileum)

IV. Absorptive Defects


A. Diagnostic tests
1. Suan re stain: etects presence of fat in stool
. Schilling test: etects B1 eciency; oral labele B1 given with unlabele IM B1 to force urinary
excretion; if raiolabele B1 in urine, able to absorb B1; if none, repeate with PO intrinsic factor; if
labele B1 in urine → pernicious anemia; if not, primary small bowel (SB) malabsorption
3. D-xylose test: PO D-xylose given, if not excrete in urine, then primary SB absorption ecit
B. Short Gut Synrome (SGS)
1. Dene by malabsorption ue to loss of intestinal length; iagnosis base on symptoms
. Due to both loss of absorption an ecrease transit time
3. Risk of SGS with less than 180 cm of small bowel; lower risk with competent ileocecal valve
a) Having 50% to 100% of functional large intestine is equivalent to 50 cm small bowel
b) Ileum can aapt after resection by ilating, elongating, increasing size of villi to increase
absorption, more important than jejunum in SGS

105
106 PArt i Patient Care

4. Complications: total parenteral nutrition (TPN) associate liver isease, cholelithiasis, calcium oxalate
nephrolithiasis, coagulopathy, bacterial overgrowth, sepsis
5. Tx: prevention of sepsis, slow transit (loperamie, etc.), reuce GI secretion with octreotie an PPI,
restrict oxalate in iet
6. Teuglutie: GLP- agonist to promote intestinal absorption an health
7. Surgical tx: slow transit by interposing colon, reversing intestinal segments, lengthening proceures
(STEP), intestinal transplantation

V. Structural Disease
A. Duoenal iverticula
1. Most common small bowel iverticula; most are asymptomatic
. Operate only if symptomatic with small bowel obstruction (SBO), biliary obstruction, concern for
malignancy

VI. Mechanical Disease


A. Small bowel obstruction
1. Most commonly ue to ahesions in rst worl; hernia elsewhere
. Early small bowel follow-through (SBFT) to etermine likelihoo of resolution with conservative
management
3. Close-loop obstruction may have normal lactate
4. Avoi nitrous oxie uring anesthesia as gas thir spaces
5. Law of LaPlace: P=T/r
6. Malignancy is suspecte for SBO in a virgin abomen; always check for a hernia
B. Stricture
1. Heineke-Mikulicz: stricture <7 cm, longituinal incision on antimesenteric bowel close in transverse
fashion
. Finney: stricture 7 to 15 cm, longituinal enterotomy on antimesenteric bowel suture in sie-to-sie
isoperistaltic enteroenterostomy
3. Jaboulay: stricture 10 to 0 cm,  enterotomies anastomose in sie-to-sie fashion bypassing
stricture segment
4. Michelassi: stricture >15 cm or multiple consecutive strictures, bowel ivie at mipoint of stricture,
stenotic areas overlappe, longituinal enterotomies along stricture, then anastomose in sie-to-sie
isoperistaltic fashion

VII. Small Bowel Neoplasms


A. Carcinoi: most common SB neoplasm
1. Slow-growing tumors of Kulchitsky enterochromafn cells, usually nonfunctional, carcinoi
synrome when functional
. Most commonly locate in ileum followe by rectum
3. Strong esmoplastic reaction an mesenteric brosis surrouning can obstruct
4. Carcinoi synrome: secretes serotonin, histamine, VIP, braykinin, prostaglanins which cause
ushing, iarrhea, bronchoconstriction, an right-sie valvular heart isease (most commonly
tricuspi insufciency); tx=octreotie
5. Elevate chromogranin-A, pancreatic polypeptie, NSE, 5-HIAA
6. Resect if possible; can ebulk liver mets
7. Gastric carcinoi (types I an II have hypergastrinemia; type III oes not)
a) Type I: autoimmune etiology associate with atrophic gastritis; associate with
hypergastrinemia an in setting of chronic atrophic gastritis or pernicious anemia, rarely
malignant, usually women, small an multicentric; if less than 1 cm without concerning risk
factors, can remove enoscopically
b) Type II: associate with the Zollinger-Ellison synrome (hypergastrinemia) an MEN-1; grows
slowly but more likely to metastasize to lymph noes an istant sites; somatostatin analogs may
be initiate an result in tumor regression; nees gastric resection
c) Type III: usually poorly ifferentiate neuroenocrine cell, normal gastrin level, high rate of
cancer, an usually nees partial gastrectomy an lymph noe issection
CHAPtEr 9 Alimentary Tract—SmallBowel 107

8. Appeniceal carcinoi:
a) < cm at tip: appenectomy
b) > cm or at base of appenix: right hemicolectomy
9. Bronchial carcinoi:
a) Carcinoi tumors may be iagnose by bronchoscopy, appearing as pink or purple friable
enobronchial masses covere by intact epithelium
b) Tx: Complete surgical resection with meiastinal lymph noe sampling or issection, regarless of
the presence of noal involvement
10. Diagnosis
a) Octreotie scan: best for localizing; inium 111-labele somatostatin scintigraphy
b) Ga-68 DOTATATE PET/CT: better than octreotie scan for localizing NETs
c) Chromogranin A level most sensitive for etection
B. Aenocarcinoma
1. Secon most common primary SB tumor
. Risk factors: smoking, EtOH, peptic ulcer isease, celiac isease, Crohn isease, FAP, HNPCC, PJS
3. Goal is R0 resection with 10 mesenteric lymph noes, may nee Whipple
C. Lymphoma
1. From lymphoi tissues such as Peyer patches, thus more common in jejunum an ileum
. B-cell NHL most common, better prognosis than T-cell lymphomas
3. Enteropathy-associate T-cell lymphoma has poor prognosis, associate with celiac
4. Tx is surgical resection with ajuvant CHOP or R-CHOP chemotherapy
D. Metastases
1. Melanoma most common; colon, breast, lung, kiney also seen

VIII. Fistulas an Ileostomies


A. High output stula >500 cc/ay, unlikely to spontaneously resolve
B. Check for FRIENDS: foreign boies, raiation, inammation/inammatory bowel isease, neoplasm,
istal obstruction, sterois/sepsis
C. High ileostomy output consiere >100 cc/ay an manage with antimotility agents, maintain
hyration
D. Loss of bicarb leas to uric aci nephrolithiasis in high output stulae an ostomies

IX. Crohn Disease


A. Abominal pain, iarrhea +/– hematochezia, ileitis, obstruction, perianal isease, lateral ssures
B. Aphthous ulcers, anemia, vitamin eciencies, malnutrition, cholesterol gallstones, oxalate kiney
stones, blin loop synrome, stulae, megaloblastic anemia (ecrease B1)
C. Extraintestinal sx that resolve with resection: uveitis, erythema noosum
D. Extraintestinal sx that o not resolve with resection: ankylosing sponylitis, pyoerma gangrenosum
E. Tx: inuction therapy with sterois (sometimes iniximab), maintenance therapy with biologics an
mesalamine/sulfasalazine
F. Skip lesions, rare rectal involvement but common perianal involvement, fat creeping, eep transmural
ulcers that are serpiginous, common stenotic lesions, noncaseating granulomas on histology
G. Surgery to treat obstruction, maintain intestinal length; o not perform stulotomies
108 PArt i Patient Care

Fig. 9.1 Heineke-Mikulicz Stricturoplasty an Finney Stricturoplasty.


CHAPtEr 9 Alimentary Tract—SmallBowel 109

Questions
1. A 55-year-ol woman with a history of total 4. Two weeks after an open aortic aneurysm repair,
abominal colectomy with en ileostomy for the patient presents with marke abominal
refractory Crohn isease presents with a bulge istention without nausea or vomiting. There is
ajacent to her ileostomy. The bulge has been no tenerness on abominal examination. Plain
present for months an has always been easily lms are unremarkable. CT scan reveals a large
reucible. However, she frequently evelops amount of ascites but is otherwise unremarkable.
bloating, obstipation, an low ostomy output Paracentesis reveals turbi ui that is culture
requiring manual reuction of the bulge. What is negative. Flui analysis reveals a triglycerie
the best treatment option for this patient? level of 400 mg/L. The white bloo cell
A. Primary parastomal hernia repair count is 600 cells/μL with a preominance of
B. Relocation of her ileostomy with mesh closure lymphocytes. Which of the following is true about
of the previous efect this conition?
C. Parastomal hernia repair with mesh A. The patient shoul be place on total parental
D. Ileal pouch-anal anastomosis nutrition (TPN) an NPO
E. Observation B. Octreotie is not useful
C. The patient shoul immeiately be reexplore
2. A 6-year-ol female presents to clinic for her D. Interventional raiology (IR) embolization is
-week follow-up appointment after unergoing rst-line treatment
a low anterior resection with iverting loop E. Most patients respon to a high-protein, low-
ileostomy for rectal cancer. Her incisions are fat iet with meium-chain triglyceries
healing well. She states her ostomy has put
out 1.9 L per ay an she has ouble her 5. Which of the following is true regaring short
aily ui intake ue to increase thirst. Her bowel synrome in aults?
serum creatinine remains normal an she has A. The presence of an intact ileocecal valve
no electrolyte abnormalities. What is the best reuces malabsorption
treatment option for this patient? B. It is ene as less than 300 cm of the resiual
A. Amission to the hospital for IV hyration small bowel
B. Oral loperamie an close outpatient C. Resection of the ileum is better tolerate than
monitoring resection of the jejunum
C. Revision to a more istal ostomy D. The presence of an intact colon oes not alter
D. Methylnaltrexone an close outpatient the severity
monitoring E. It is most commonly cause by multiple
E. Observation operations requiring small bowel resection

3. A 55-year-ol man with a history of heavy 6. Which of the following is true regaring the
nonsteroial antiinammatory rugs (NSAID) management of short bowel synrome?
use is amitte with a perforate antral ulcer. A. Glutamine shoul be avoie
He unergoes a Billroth II reconstruction. On B. Octreotie is the cornerstone of management
postoperative ay 4, he evelops acute abominal C. Coeine is contrainicate
pain an hemoynamic instability. Which of the D. Early enteral feeing is inicate
following complications is most likely causing E. Patients who require TPN after 6 months will
this presentation? require permanent TPN
A. Anastomotic leak of gastrojejunostomy
B. Efferent limb synrome
C. Duoenal stump blowout
D. Marginal ulcer
E. Internal hernia
110 PArt i Patient Care

7. A 6-year-ol boy has short bowel synrome 11. Which of the following is the best test for
cause by migut volvulus that evelope uring prognosis an monitoring treatment response in
infancy an has since been epenent on TPN, carcinoi tumors?
which he has tolerate well. He has approximately A. Platelet serotonin levels
8 cm of small bowel remaining with an intact B. 4-hour urinary 5-HIAA test
colon. The small bowel is markely ilate C. Serum chromogranin A levels
without evience of small bowel obstruction. D. Serum serotonin levels
Which of the following is the best option? E. Neuron-specic enolase
A. Serial transverse enteroplasty proceure (STEP)
B. Continue with TPN 12. Which of the following is true regaring small
C. Small bowel transplantation bowel obstruction?
D. Small bowel tapering proceure A. The most common worlwie etiology is
E. Tapering an lengthening proceure (Bianchi) ahesions from prior surgery
B. It is more frequent with upper intestinal than
8. Which of the following is true regaring small lower intestinal surgery
bowel neoplasms? C. In a complete close-loop obstruction, serum
A. Aenocarcinoma is the most common type lactate can be normal
B. Small bowel lymphoma most commonly D. Partial obstruction symptoms typically
occurs in the uoenum improve within 4 hours with conservative
C. The incience of primary small intestinal management
cancers is increasing E. Abominal pain isproportionate to exam
D. Five-year survival is higher for nings occurs early in the setting of
aenocarcinoma compare with carcinoi obstruction
tumors
E. Small bowel lymphoma is primarily treate by 13. Which of the following is true regaring
chemotherapy uoenal iverticula?
A. They ten to occur on the antimesenteric sie
9. A 68-year-ol woman presents with an of the bowel
exacerbation of congestive heart failure an acute B. Most are ientie in young patients
abominal pain. Physical examination of the C. Treatment with enoscopic interventions is
abomen is signicant for mil iffuse abominal contrainicate
tenerness but no reboun or guaring. CT D. Malabsorption ue to bacterial overgrowth
arteriography of the abomen emonstrates within the iverticula manates surgery
iffuse narrowing of the superior mesenteric E. When iscovere incientally at surgery, they
artery (SMA) an its branches but no vascular shoul be left alone
occlusion, pneumatosis, free air, or portal venous
gas. Which of the following is an appropriate 14. Which of the following is the most common cause
management option? of obscure GI bleeing in aults?
A. IV heparin rip A. Small intestine angioysplasia
B. Exploratory laparotomy B. Meckel iverticulum
C. Aggressive ui resuscitation C. Crohn isease
D. Intraarterial papaverine D. Infectious enteritis
E. Increase cariac output E. Vasculitis

10. Which of the following is true regaring 15. Which of the following is true regaring GISTs of
carcinoi? the small bowel?
A. The majority of carcinoi synrome is from A. Most patients are symptomatic with GI
appeniceal tumors that have metastasize bleeing
B. The most common symptom of carcinoi B. They stain positive for CD134
synrome is iarrhea C. Patients eeme caniates for chemotherapy
C. Chromogranin A will not be elevate in shoul receive it for 1 year
nonfunctioning tumors D. A patient with a 6-cm tumor shoul receive
D. Patients are at an increase risk for glossitis ajuvant chemotherapy
E. Urinary 5-hyroxyinoleacetic aci (5-HIAA) E. Malignancy is primarily etermine by
is not sensitive for etecting metastatic evience of local invasion
carcinoi
CHAPtEr 9 Alimentary Tract—SmallBowel 111

16. A hernia sac containing a Meckel iverticulum is 20. Which of the following is true regaring Peutz-
known as: Jeghers synrome?
A. Petit hernia A. Patients shoul begin breast an cervical
B. Littre hernia cancer screening at age 5
C. Spigelian hernia B. It is autosomal recessive
D. Richter hernia C. Small bowel obstruction is uncommon
E. Grynfeltt hernia D. Prophylactic colectomy is recommene to
most patients starting at age 0
17. Superior mesenteric artery (SMA) (Wilkie) E. These patients are not at increase risk for
synrome: small bowel cancer
A. Involves the secon portion of the uoenum
B. Causes venous outow obstruction from the 21. Which of the following is correct with regars to
left kiney Crohn isease?
C. Is best iagnose with arteriography A. Mesenteric fat wrapping is consiere
D. Shoul initially be manage with a high pathognomonic
caloric intake iet B. Symptoms of ankylosing sponylitis improve
E. Is best manage by gastrojejunostomy with resection of isease bowel
C. The majority of patients with an initial
18. A 45-year-ol woman with a history of presentation of terminal ileitis progress to
laparotomy an 5000 cGy of abominal an Crohn isease on long-term follow-up
pelvic irraiation for ovarian cancer 10 years ago D. Exaggerate skin injury after minor trauma
presents with symptoms an signs of an acute (pathergy) is a commonly associate conition
bowel obstruction. CT scan shows a complete E. Pyoerma gangrenosum is commonly foun
small bowel obstruction at the level of the mi on the initial presentation of Crohn isease
jejunum with no evience of any masses. Which
of the following is true about this conition? 22. Which intestinal cells have been implicate in
A. If a stricture is present, it is best manage by the formation of gastrointestinal stromal tumors
strictureplasty (GISTs)?
B. Sterois shoul be aministere A. Goblet cells
C. Acute raiation enteritis is ue to an B. Interstitial cells of Cajal
obliterative arteritis C. Enteroenocrine cells
D. The risk of this complication increases in the D. Paneth cells
setting of iabetes E. Absorptive enterocytes
E. The egree of raiation amage is not affecte
by whether the patient receive chemotherapy 23. A 46-year-ol woman is about to unergo hepatic
resection for a metastatic carcinoi tumor.
19. A 75-year-ol male with a history of chronic During anesthesia inuction, her bloo pressure
obstructive pulmonary isease (COPD) presents ecreases to 80 mmHg systolic an her heart rate
to the ED with a 1-ay history of abominal increases to 110 beats per minute. Her entire boy
istention an nausea. He enies abominal pain. appears ushe. Her temperature is normal, as is
Abominal examination is benign. Laboratory en-tial CO. Management consists of:
values are normal. CT scan emonstrates free A. Corticosterois
air uner the iaphragm an thin-walle, air- B. Antihistamine
lle cysts within the bowel wall. Which of the C. Octreotie
following is true regaring this conition? D. Abort operation
A. Laparotomy is inicate E. Dantrolene
B. The primary form occurs more commonly than
the seconary form
C. It is unlikely to be relate to the patient’s
COPD
D. It is most commonly seen in the ileum
E. It is associate with steroi use
112 PArt i Patient Care

24. A 70-year-ol woman presents with vague 27. Which of the following is true regaring Crohn
abominal pain, iarrhea, steatorrhea, an isease?
anemia with an elevate mean corpuscular A. It is more common in iniviuals of high
volume. Her meical an surgical history is socioeconomic status
unremarkable. A CT scan of the abomen an B. The most common inication for surgery is
pelvis is negative. An upper GI series an small perforation
bowel follow-through are signicant only for a C. It has a unimoal istribution
large jejunal iverticulum. Which of the following D. It is more prevalent in females
is true regaring this patient? E. The most common initial presentation is an
A. It is typically cause by an autoimmune acute onset of abominal pain an iarrhea
etiology
B. A long-chain triglycerie iet may be helpful 28. The earliest lesion characteristic of Crohn isease
C. The iverticulum shoul be resecte is:
D. Broa-spectrum antibiotics are inicate A. Aphthous ulcer
only if the patient presents with a fever an B. Caseating granuloma
leukocytosis C. Noncaseating granuloma
E. Vitamin B1 is inicate D. Cobblestone mucosa
E. Serosal thickening
25. A 57-year-ol male with no past surgical history
presents with  ays of abominal pain, nausea, 29. Which of the following is the best therapeutic
an vomiting. On exam he is istene an option for mil active Crohn isease?
tympanic an is milly tener to palpation A. Sulfasalazine
without reboun or guaring. He has no groin B. Prenisone
hernias. Compute tomography (CT) scan C. Buesonie
emonstrates multiple ilate loops of small D. Metroniazole
bowel with a transition point in the istal E. Iniximab
small bowel, with some ajacent mesenteric
fat straning. He has a mil leukocytosis. His 30. Which of the following is true regaring the
last bowel movement was 1 ay ago. He has principles of operative management of the small
not passe atus for over a ay. He has not ha bowel in Crohn isease?
any similar symptoms previously. A nasogastric A. The optimal margin is at least 4 cm beyon
(NG) tube is place, intravenous (IV) uis are grossly visible isease
aministere, an the patient is place NPO B. Frozen section shoul be obtaine to conrm
(nothing by mouth). Which of the following is the the absence of active isease in at least one
best next step in management? margin
A. A 4-hour trial of NG tube suction, then C. A 3-cm stricture segment of uoenum is
exploratory laparotomy if high output best manage by resection
continues D. A 10-cm stricture segment of jejunum
B. Exploratory laparotomy can be manage by a Heineke-Mikulicz
C. Water-soluble oral contrast challenge strictureplasty rather than by resection
D. Water-soluble oral an rectal contrast E. Strictures longer than 10 cm are best manage
challenge by resection
E. Diagnostic laparoscopy

26. Which of the following has been shown to be the


most efcacious means of reucing postoperative
ileus in patients unergoing bowel resection?
A. Early ambulation
B. Gum chewing
C. Alvimopan
D. Ketorolac combine with reuction in opioi
use
E. Nasogastric intubation
CHAPtEr 9 Alimentary Tract—SmallBowel 113

Answers
1. C. This patient presents with a chronic, reucible para- as compare to after a Billroth I, because the alkaline envi-
stomal hernia. The enitive treatment for parastomal her- ronment secrete by Brunner glans in the uoenum is not
nia repair is ostomy reversal; however, given this patient’s present in the jejunum. However, marginal ulcers, which are
Crohn isease, ostomy takeown with ileoanal anastomosis present on the jejunal sie of the anastomosis, present with
or ileal pouch-anal anastomosis is contrainicate (D). Given abominal pain an possible upper GI bleeing (D). Any
the recurrent iscomfort an intermittent obstruction from proceure that can result in ahesions has the potential to
her hernia, simple observation woul not be appropriate (E). cause an internal hernia, though the highest chance is after a
Once taught as the surgical treatment of choice for parasto- gastric bypass. The presentation of internal hernia is typically
mal hernia repair, ostomy relocation is no longer avise as inolent with vague abominal pain, nausea, an vomiting
it creates the potential for a new parastomal hernia an for an can be confuse for gastroenteritis or peptic ulcer is-
hernias from prior ostomy site an laparotomy incisions. ease. A high inex of suspicion is require, an the iagnosis
SAGES now recommens against ostomy relocation as the often requires iagnostic laparoscopy for conrmation (E).
treatment of parastomal hernias (B). Primary parastomal
hernia repair results in high tension with a high rate of her-
nia recurrence (A). The most effective treatment for paras- 4. E. The patient has chylous ascites. In Western countries,
tomal hernias ajacent to ostomies that are unable to be chylous ascites is most often ue to malignancy an cirrho-
reverse is mesh repairs (C). Stanar approaches inclue sis, whereas infectious etiologies such as tuberculosis an
the Sugarbaker repair, in which an unerlay mesh is place lariasis preominate in Eastern an eveloping countries.
on the efect with the stoma exiting the peritoneum at the Other causes inclue postlaparotomy inammatory isor-
sie of the mesh, an the keyhole approach, when the stoma ers, trauma, raiation therapy, congenital lymphatic abnor-
is brought through a hole create in the mesh. malities, an pancreatitis. The operations most associate
Reference: Gillern S, Bleier JIS. Parastomal hernia repair an with this complication inclue aortic aneurysm repair, ret-
reinforcement: the role of biologic an synthetic materials. Clin Colon roperitoneal lymph noe issection, inferior vena cava sur-
Rectal Surg. 014;7(4):16–171. gery, an liver transplantation, because these are operations
in which retroperitoneal lymphatics are most likely to be
2. B. This patient presents with high ostomy output, usu- interrupte. The mechanisms thought to lea to the evel-
ally ene as over 1. L per ay. High ostomy output can opment of chylous ascites inclue exuation of chyle ue
result in ehyration, loss of bicarbonate resulting in uric to obstruction of the cisterna chyli, irect leakage of chyle
aci nephrolithiasis, an skin breakown. In the absence of through a lymphoperitoneal stula, an exuation through
ehyration or electrolyte abnormalities requiring inpatient ilate retroperitoneal vessels. The iagnosis of chylous
amission, high ostomy output can be manage as an out- ascites is best establishe by analysis of the ui. Chyle typ-
patient with close follow-up an titration of oral loperamie ically has a turbi appearance; however, it may be clear in
or Lomotil. When high ostomy output results in symptom- fasting patients. Elevate triglycerie levels in the ui are
atic ehyration or electrolyte isturbances, amission for consiere iagnostic, usually above 00 mg/L, although
inpatient hyration an monitoring is recommene (A). some use a threshol above 110 mg/L. In aition, the
Ostomy revision is not typically require for high ostomy white bloo cell count is greater than 500, with a preom-
output an iverting loop ileostomies are usually istal inance of lymphocytes. The total protein level is between
enough to aequately absorb ui (C). Methylnaltrexone is .5 an 7.0g/L. Cultures are negative, except for cases of
an opioi antagonist use to treat opioi-inuce ileus an tuberculosis, in which aenosine eaminase is also positive
is not absorbe enterally (D). Observation is not appropriate in the ui. The initial treatment of chylous ascites is to
for high ileostomy output (E). aminister a high-protein, low-fat iet with meium-chain
triglyceries. This iet minimizes chyle prouction an
3. C. The most serious complication after a Billroth II recon- ow. Meium-chain triglyceries are absorbe by the intes-
struction is a uoenal stump blowout. In a Billroth II, the tinal epithelium an are transporte to the liver through the
pylorus an antrum are resecte, an the rst portion of the portal vein an o not contribute to chylomicron formation.
uoenum is oversewn. The biliary system rains into the Conversely, long-chain triglyceries are converte to mono-
uoenum, which then rejoins path of foo at a surgically glyceries an free fatty acis, which are then transporte
constructe gastrojejunostomy. The inammation cause by to the intestinal lymph vessels as chylomicrons. If this iet
a perforate ulcer results in inammation of the uoenum, regimen fails, placing the patient NPO an on TPN with oct-
which may leak after oversewing. Aitional causes of uo- reotie has been shown to be useful in patients with postop-
enal stump blowout inclue afferent limb obstruction an erative chylous ascites (A, B). If these meical approaches
pancreatitis. Efferent limb synrome is the result of obstruc- fail, then lymphoscintigraphy is often useful to localize
tion of the efferent limb istal to the gastrojejunostomy, lymph leaks an the site of obstruction. In some instances,
resulting in bile reux an bilious emesis (B). Anastomotic IR can percutaneously inject glue to stop leak. Surgical reex-
leak of the gastrojejunostomy is possible but less common ploration with localization an closure of the lymphatic
an catastrophic than uoenal stump blowout. A marginal leak shoul be performe if leak persists beyon  weeks
ulcer is more likely to occur after a Billroth II reconstruction, (C). Alternatively, in facilities with capabilities to perform
114 PArt i Patient Care

percutaneous lymphangiography, embolization of lymphat- Intestinal aaptation occurs over a perio of 1 to  years in
ics may be attempte after faile ietary management (D). most aults. Thus, the nal etermination of whether per-
This latter complication may be ue to a high plasminogen manent TPN will be necessary is not etermine until after
concentration in the ascitic ui. this perio (E).
Reference: Cárenas A, Chopra S. Chylous ascites. Am J Gastro-
enterol. 00;97(8):1896–1900. 7. A. Many patients with short bowel synrome can even-
tually iscontinue TPN, particularly if the bowel length
5. A. The total length of small bowel is approximately 0 is more than 10 cm in aults or more than 60 cm in chil-
feet (each foot is equal to ≈30 cm), or approximately 600 cm ren. Treatment options for short bowel synrome epen
(6 m). Short bowel synrome is ene as the presence of on the length of small bowel remaining, whether the rem-
less than 180 cm of resiual an functional small bowel in nant small bowel is markely ilate, whether the patient
ault patients (B). Thus, resection of less than 50% of the remains TPN epenent, an whether multiple complica-
small intestine is generally well tolerate. In approximately tions of TPN have evelope such as catheter-relate infec-
75% of cases, short bowel synrome results from one mas- tions, vena cava thrombosis, an liver amage (B). A short
sive small bowel resection, as oppose to multiple sequen- remnant (<90 cm in aults, <30 cm in chilren) of small
tial resections (E). In aults, the most common etiologies bowel poses a challenging ilemma. If the remnant of small
inclue acute mesenteric ischemia, malignancy, an Crohn bowel is short an markely ilate without evience of
isease. In peiatric patients, the most common etiologies obstruction, the best option woul be an intestinal lengthen-
inclue intestinal atresia, migut volvulus, an necrotizing ing proceure. The ilate bowel lens itself to lengthening
enterocolitis. Resection of the jejunum is better tolerate by applying a series of transverse linear staples on the mes-
than resection of the ileum because the absorption of bile enteric borer an then on the antimesenteric borer. The
salts an vitamin B1 occurs in the ileum (C). An intact ile- proceure is known as the serial transverse enteroplasty
ocecal valve is thought to reuce malabsorption because it proceure. The Bianchi proceure is another option. How-
increases the resience time of the chyme in the small intes- ever, it is technically much more emaning an associate
tine. Likewise, an intact colon is important because it has a with a risk of creating ischemia an anastomotic leaks an
tremenous water-reabsorbing capacity an electrolytes an thus has a higher complication rate an an increase nee
can also absorb fatty acis (D). With an intact colon, a shorter for reoperation (E). Tapering of the small bowel alone woul
small bowel remnant is tolerate. The key to preventing be inicate for patients with a longer small bowel remnant
short bowel synrome is avoiance of excessive small bowel (>60 cm in chilren) who have marke bowel istention
resection. In Crohn patients, the use of strictureplasty as with evience of stasis an bacterial overgrowth (D). Taper-
oppose to resection is recommene when possible. Also, ing alone woul not be appropriate in someone with such a
one shoul resect only obviously ea bowel in acute mesen- short segment of small bowel. Small bowel transplantation
teric ischemia, leaving marginal bowel in situ an perform- is also an option but is reserve for the patient with a short
ing a secon-look proceure. segment an who is TPN epenent (such as this patient) in
whom, in aition, complications have evelope from the
6. D. In the early phase of short bowel synrome, treat- TPN, as mentione (C). If liver failure has evelope in the
ment is irecte at slowing intestinal transit; reucing GI patient, small bowel transplantation can be combine with
secretions; an maintaining nutrition, ui, an electrolyte liver transplantation.
balance. Transit time is slowe by the aministration of nar- References: Kim HB, Lee PW, Garza J, Duggan C, Fauza D,
cotics such as coeine an iphenoxylate, as well as with the Jaksic T. Serial transverse enteroplasty for short bowel synrome: a
antimotility agents Lomotil (iphenoxylate an atropine) case report. J Pediatr Surg. 003;38(6):881–885.
an loperamie (C). Massive small bowel resection is asso- Suan D, Thompson J, Botha J, et al. Comparison of intestinal
ciate with hypergastrinemia an aci hypersecretion. The lengthening proceures for patients with short bowel synrome.
increase aciity in the small bowel results in the inhibition Ann Surg. 007;46(4):593–601.
of igestive enzymes. This can be controlle with H-receptor
antagonists or proton pump inhibitors such as omeprazole 8. C. Malignant tumors of the small bowel are rare. How-
an thus shoul be starte in all patients with short gut syn- ever, the incience has nearly ouble since the 1970s. The
rome (B). Nutrition is achieve with the institution of TPN. most common tumor is carcinoi (37.4%), followe by ae-
In aition, enteral feeing shoul be institute as soon as nocarcinoma (36.9%), lymphoma (17%), an GISTs (8%)
postoperative ileus has resolve. Enteral feeing assists in (A). Small bowel lymphomas most commonly involve the
the process of intestinal aaptation an prevents the evelop- ileum (as o carcinois), whereas aenocarcinomas are most
ment of villous atrophy associate with being NPO for a pro- common in the uoenum (periampullary), an GISTs are
longe perio of time. Glutamine is helpful because it serves evenly istribute throughout the small bowel (although
as a trophic factor for the gut an is consiere the principal most common in the stomach) (B). Small bowel lympho-
fuel of the small intestine (A). Cholestyramine is also useful mas are preominantly the non-Hogkin type. In chilren
in controlling iarrhea ue to unabsorbe bile salts. The role younger than age 10, they are the most common intestinal
of octreotie is controversial. Short-term use leas to a reuc- neoplasm. The propensity for involvement of the ileum is
tion in iarrhea, but long-term use may lea to steatorrhea, ue to its high concentration of lymphoi tissue (C). The
gallstones, an an inhibition of intestinal aaptation. More primary treatment of small bowel lymphoma (as well as all
recently, a high-carbohyrate, low-fat enteral iet rich in glu- other small bowel malignancies) is surgical resection inclu-
tamine combine with growth hormone aministration has ing the affecte mesentery (E). There is no clear, well-ene
shown promise in improving intestinal absorptive capacity. role for raiation therapy or chemotherapy for the majority
CHAPtEr 9 Alimentary Tract—SmallBowel 115

of small bowel malignancies. The exception is the use of 10. D. While it was long believe that the appenix was
Gleevec (imatinib mesylate) for GISTs. The 5-year survival the most common source of carcinoi tumor, a large SEER
rate is higher for carcinoi compare with aenocarcinoma atabase stuy foun that the small intestine accounte
(64.6% versus 3.5%) (D). for 55% of cases, followe by the rectum (0%), an then
References: Balthazar EJ, Noorhoorn M, Megibow AJ, Goron the appenix (17%). The most common location in the
RB. CT of small-bowel lymphoma in immunocompetent patients small bowel is the ileum (A). Carcinoi synrome most
an patients with AIDS: comparison of nings. AJR Am J Roentge- commonly presents with ushing followe by iarrhea
nol. 1997;168(3):675–680. an bronchospasms (B). Most gut carcinoi tumors o not
Bilimoria KY, Bentrem DJ, Wayne JD, Ko CY, Bennett CL, Talam-
cause the synrome because vasoactive substances (sero-
onti MS. Small bowel cancer in the Unite States: changes in epie-
tonin, histamine, opamine, substance P, prostaglanins)
miology, treatment, an survival over the last 0 years. Ann Surg.
009;49(1):63–71. from these tumors enter the portal vein an are metabo-
lize by the liver before reaching the systemic circulation.
9. E. The presentation is most consistent with nonocclusive For carcinoi synrome to evelop, these substances nee
mesenteric ischemia, which accounts for approximately 0% to be release irectly into the systemic circulation. Thus,
to 30% of acute mesenteric ischemia cases. This conition the synrome evelops in the setting of bronchial carci-
typically affects elerly patients an presents in the set- nois (which o not rain into the liver), retroperitoneal
ting of a ecrease in cariac output, such as after an acute invasion (where retroperitoneal veins rain irectly into
myocarial infarction, exacerbation of congestive heart fail- the systemic circulation), or in the presence of liver metas-
ure, or after cariac surgery. There are no laboratory tests tasis. A 4-hour urinary 5-HIAA test is highly sensitive an
to establish the iagnosis of bowel ischemia with certainty, specic for etecting metastatic carcinoi an is consiere
although the presence of lactic aciosis is consiere omi- the gol-stanar test to establish the iagnosis (E). How-
nous. The initial iagnostic test of choice for suspecte acute ever, it is not as sensitive for etecting nonfunctional carci-
mesenteric ischemia is CT angiography. It is helpful in ien- noi tumors. Screening for a carcinoi tumor (as oppose
tifying the etiology, which inclues an embolus that woul to establishing the iagnosis of carcinoi synrome)
be visualize as an occlusion just istal to the origin of the is probably best achieve with serum chromogranin A
SMA; acute thrombosis of the SMA, which woul appear as because it will be elevate in both functioning an non-
an occlusion in association with iffuse calcications within functioning tumors (C). Normally, most ietary tryptophan
the vessel; mesenteric venous thrombosis, which woul is converte into nicotinic aci (niacin, vitamin B3). In the
emonstrate a lack of contrast lling of either the portal or presence of carcinoi tumors, there is a shift towar con-
superior mesenteric vein; an nonocclusive mesenteric isch- version to 5-hyroxytryptophan, which is then converte
emia, which woul simply show iffuse spasm. The stan- to serotonin. Serotonin is then metabolize to 5-HIAA. The
ar treatment for SMA embolus is operative embolectomy shift away from conversion to tryptophan to nicotinic aci
with resection of ischemic or infarcte bowel, although there can result in pellagra, which can present with iarrhea,
are some reports of the use of thrombolytic therapy in the ermatitis (rough scaly skin, glossitis, angular stomatitis),
absence of signs of bowel compromise (B). The treatment ementia, an/or hypoalbuminemia.
for acute thrombosis is surgical bypass from either the aorta References: Swain CP, Tavill AS, Neale G. Stuies of tryptophan
or the iliac artery to the more istal SMA. For mesenteric an albumin metabolism in a patient with carcinoi synrome, pel-
lagra, an hypoproteinemia. Gastroenterology. 1976;71(3):484–489.
venous thrombosis, the treatment is heparin alone, provie
Nobels FR, Kwekkeboom DJ, Coopmans W, et al. Chromogranin
there is no suggestion of infarcte bowel (A). For nonocclu- A as serum marker for neuroenocrine neoplasia: comparison with
sive mesenteric ischemia, the goal of treatment is to restore neuron-specic enolase an the alpha-subunit of glycoprotein hor-
intestinal bloo ow, which is most successfully one by mones. J Clin Endocrinol Metab. 1997;8(8):6–68.
correcting the unerlying cause to improve cariac output. Zuetenhorst JM, Taal BG. Metastatic carcinoi tumors: a clinical
This may be accomplishe with inotropes in the setting of review. Oncologist. 005;10():13–131.
cariogenic shock. In aition to supportive care, selective Maggar MA, O’Connell JB, Ko CY. Upate population-base
intraarterial infusion of a vasoilator, such as papaverine review of carcinoi tumors. Ann Surg. 004;40(1):117–1.
hyrochlorie into the SMA to reverse splanchnic vasocon-
striction, can be helpful but is not rst-line therapy when 11. C. Serum chromogranin A is the most sensitive marker
meical management of cariac ysfunction may be success- for etecting neuroenocrine tumors in general. It has also
ful (D). Aggressive ui resuscitation shoul be use with been shown to be the most useful marker for etecting
caution as nonocclusive mesenteric ischemia often occurs in recurrence an response to treatment. Because the level of
the setting of ecompensate congestive heart failure (as in chromogranin A correlates with tumor buren, it is a useful
the above patient), which may worsen with multiple ui marker for treatment response. A high level correlates with
boluses (C). The mortality rate for nonocclusive mesenteric a worse prognosis. Platelet serotonin level is also useful in
ischemia is approximately 50%. etecting carcinoi tumors (A). However, platelets become
References: Bassiouny HS. Nonocclusive mesenteric ischemia. rapily saturate with serotonin; thus, it is not a useful
Surg Clin North Am. 1997;77():319–36.
tool for monitoring treatment response (D). 5-HIAA is also
Kozuch P, Brant L. Review article: iagnosis an management
of mesenteric ischemia with an emphasis on pharmacotherapy. Ali- thought to be useful; however, several stuies inicate that
ment Pharma Ther. 005;1(3):01–15. chromogranin A is more sensitive for recurrence an a bet-
Trompeter M, Braza T, Remy CT, Vestring T, Reimer P. Non- ter prognosticator (B). Neuron-specic enolase has a high
occlusive mesenteric ischemia: etiology, iagnosis, an interven- specicity but a low sensitivity for the etection of carcinoi
tional therapy. Eur Radiol. 00;1(5):1179–1187. tumor (E).
116 PArt i Patient Care

References: Bajetta E, Ferrari L, Martinetti A, et al. Chromogr- Atluri P, Karakousis GC, Porrett PM, Kaiser LR, es. Surgical
anin A, neuron specic enolase, carcinoembryonic antigen, an review: an integrated basic and clinical science study guide. n e. Phil-
hyroxyinole acetic aci evaluation in patients with neuroeno- aelphia: Lippincott Williams an Wilkins; 005.
crine tumors. Cancer. 1999;86(5):858–865.
Eriksson B, Oberg K, Strisberg M. Tumor markers in neuroen- 13. E. Acquire iverticula consist of mucosa an submu-
ocrine tumors. Digestion. 000;6 Suppl 1(1):33–38. cosa but lack a complete muscularis an are thus consiere
Janson ET, Holmberg L, Strisberg M, et al. Carcinoi tumors: false iverticula. They are most commonly locate in the sec-
analysis of prognostic factors an survival in 301 patients from a on portion of the uoenum near the ampulla of Vater an
referral center. Ann Oncol. 1997;8(7):685–690.
are referre to as periampullary iverticula. They arise on
Nikou GC, Lygiakis NJ, Toubanakis C, et al. Current iagno-
sis an treatment of gastrointestinal carcinois in a series of 101
the mesenteric borer in areas of weakness in the bowel wall
patients: the signicance of serum chromogranin-A, somatostatin where bloo vessels penetrate (A). Periampullary iverticula
receptor scintigraphy an somatostatin analogues. Hepatogastroen- are associate with cholangitis, pancreatitis, an sphincter
terology. 005;5(63):731–741. of Oi ysfunction. Duoenal iverticula are also associ-
ate with choleocholithiasis. These latter complications
are thought to be ue to the location of the periampullary
12. C. Mechanical SBO is the most frequently encountere iverticulum, which may lea to obstruction an stasis of
surgical isorer of the small intestine an, in the Unite the common uct. The majority of patients presenting with
States, is most commonly ue to intraabominal ahesions biliary complications who are iscovere to have a uoenal
relate to previous abominal surgery. However, worlwie iverticulum can be safely treate enoscopically (C). If this
it is most commonly ue to a hernia (A). The risk of reamis- is not successful, surgical iverticulectomy is recommene.
sion for ahesions is greatest for patients unergoing lower Care must be taken uring iverticulectomy to ientify an
abominal surgery an seems to be in the 9% range long preserve the sphincter, which may require cannulation of the
term (B). Diagnosis of obstruction can be mae with CT scan, common bile uct. These false iverticula are also foun in
small bowel series, or enteroclysis (uoroscopic examina- the jejunum an ileum. They are istinguishe from a Meckel
tion of the small bowel using liqui contrast). The majority iverticulum, which is a true iverticulum present at birth.
of patients can be manage nonoperatively with nasogastric Duoenal iverticula are most often iscovere between
ecompression an nutritional support. This is successful in ages 56 an 76 years uring upper enoscopy, enoscopic
65% to 81% of patients, an resolution of symptoms most retrograe cholangiopancreatography, or abominal imag-
commonly occurs within 48 hours (D). However, any signs ing in as many as 6% of patients (B). They are asymptomatic
an symptoms suggestive of ischemic bowel are an ini- in the majority of patients, an thus surgery is not recom-
cation for urgent operative intervention. The incience of mene if they are iscovere incientally either on imaging
strangulation is no greater than with SBO that presents later. or intraoperatively. Complications are estimate to occur in
Features of strangulate obstruction such as abominal pain 6% to 10% of patients. They may cause symptoms of mal-
isproportionate to abominal nings are suggestive of absorption ue to bacterial overgrowth within the ivertic-
intestinal ischemia an are not usually an early ning (E). ula. This can be treate with antibiotics (D). Less commonly,
Serum lactate levels are 90% sensitive an 87% specic for bleeing can arise within the iverticulum, or iverticulitis
the presence of bowel ischemia. However, it is possible that can evelop, leaing to perforation, which usually occurs
patients with a complete close-loop obstruction (more com- into the retroperitoneum. Perforation requires laparotomy,
monly with volvulus) can have a normal lactate level. This is an closure of the uoenal efect can be challenging an
because obstruction of venous rainage prevents lactic aci may require placing a loop of jejunum over the efect as a
prouce by enterocytes from reaching systemic circulation. serosal patch.
If nasogastric ecompression fails to resolve the obstruction, References: Kenney RH, Thompson MH. Are uoenal iver-
surgery is inicate. The timing of surgery is ebatable. In ticula associate with choleocholithiasis? Gut. 1988;9(7):1003–1006.
one large series, surgery was recommene for failure of Tham TCK, Kelly M. Association of periampullary uoe-
nasogastric ecompression after 6 ays an in another stuy nal iverticula with bile uct stones an with technical success
after 10 to 14 ays. The morbiity an mortality rates of early of enoscopic retrograe cholangiopancreatography. Endoscopy.
small bowel obstruction are very low. 004;36(1):1050–1053.
References: Ellozy SH, Harris MT, Bauer JJ, Gorne SR, Kreel Vaira D, Dowsett JF, Hatel AR, et al. Is uoenal iverticulum
I. Early postoperative small-bowel obstruction: a prospective eval- a risk factor for sphincterotomy? Gut. 1989;30(7):939–94.
uation in 4 consecutive abominal operations. Dis Colon Rectum.
00;45(9):114–117. 14. A. The majority of lesions responsible for GI bleeing
Matter I, Khalemsky L, Abrahamson J, Nash E, Sabo E, Elar S. are seen with upper enoscopy or colonoscopy. Obscure GI
Does the inex operation inuence the course an outcome of ahe- bleeing refers to persistent or recurrent bleeing for which
sive intestinal obstruction? Eur J Surg. 1997;163(10):767–77. no source has been ientie by these moalities. Obscure
Parker MC, Ellis H, Moran BJ, et al. Postoperative ahesions: bleeing can be either occult (meaning not visible to the
ten-year follow-up of 1,584 patients unergoing lower abominal eye) or overt (such as melena an hematochezia). In most
surgery. Dis Colon Rectum. 001;44(6):8–89. instances, the source of obscure bleeing is the small bowel.
Stewart R, Page C, Brener J. The incience an risk of early
Small intestine angioysplasias account for 75% of cases
postoperative small bowel obstruction: a cohort stuy. Am J Surg.
1987;154(6):643–647.
of obscure bleeing in aults (B–E). Other causes inclue
Tavakkoli A, Ashley SW, Zinner MJ. Small Intestine. In: Bruni- Crohn isease, infectious enteritis, neoplasms, an vascu-
cari F, Anersen DK, Billiar TR, Dunn DL, Hunter JG, Matthews JB, litis. A Meckel iverticulum is the most common cause of
Pollock RE, es. Schwartz’s principles of surgery. 10th e. New York: obscure GI bleeing in chilren. Localization of small bowel
McGraw-Hill Eucation; 015:1146–1151. lesions is ifcult with stanar stuies. Options inclue
CHAPtEr 9 Alimentary Tract—SmallBowel 117

push enteroscopy an small bowel barium stuies, capsule triangle (Grynfeltt triangle) (E) are more common than those
enoscopy, raiolabele re bloo cell scanning, an angiog- through the inferior lumbar triangle (the Petit triangle) (A).
raphy (although these latter two are only useful in the setting The Petit triangle is boune by the external oblique mus-
of active bleeing). cle, latissimus orsi muscle, an iliac crest. The Grynfeltt tri-
Reference: Pennazio M, Santucci R, Rononotti E, et al. Out- angle is boune by the quaratus lumborum muscle, the
come of patients with obscure gastrointestinal bleeing after cap- 1th rib, an the internal oblique muscle. A spigelian hernia
sule enoscopy: report of 100 consecutive cases. Gastroenterology. occurs through the spigelian fascia, which is compose of the
004;16(3):643–653. aponeurotic layer between the rectus muscle meially an
the semilunar line laterally (C). Nearly all spigelian hernias
15. D. GISTs were previously terme leiomyomas or leiomyo- occur in the spigelian belt locate below the umbilicus but
sarcomas. It now seems that they are mesenchymal tumors. above the epigastric vessels. The absence of posterior rectus
GISTs are classie into three types: spinle cell (70%), epi- fascia may contribute to an inherent weakness in this area. A
thelioi type (0%), an mixe spinle an epithelioi cell Richter hernia occurs when only the antimesenteric borer of
type (10%). GISTs stain positive for CD34, the human pro- the bowel herniates through the fascial efect (D). It involves
genitor cell antigen, as well as for CD177, the c-kit proto-on- only a portion of the circumference of the bowel. As such,
cogene protein (B). The stomach is the most common site in incarceration an strangulation may occur in the absence of
the GI tract. Small bowel GISTs may be inciental iscover- any evience of bowel obstruction.
ies at surgery for other isorers. The majority of patients Reference: Skanalakis PN, Zoras O, Skanalakis JE, Mirilas
are asymptomatic (A). However, those that o present with P. Spigelian hernia: surgical anatomy, embryology, an technique of
symptoms ten to be very large an bulky at presentation. repair. Am Surg. 006;7(1):4–48.
In one large stuy, the meian size of a symptomatic GIST
was 11 cm. They ten to present with evience of obstruction 17. D. The SMA leaves the aorta at a ownwar an acute
or GI bleeing. The stanar treatment is surgical resection angle. SMA synrome or Wilkie synrome is a rare conition
with 1 cm margins. However, microscopically positive mar- characterize by compression of the thir portion of the uo-
gins have not been emonstrate to affect survival. GISTs of enum by the SMA as it passes over this portion of the uo-
the small intestine carry a high mortality rate, likely ue to enum (A). It occurs most often in the setting of profoun
the late presentation. Only 8% of patients were alive at a weight loss. Factors that preispose to the conition inclue
meian follow-up of 0 months in one stuy. Determining supine immobilization, scoliosis, placement of a boy cast,
whether a GIST is benign or malignant is ifcult because an eating isorers. Symptoms inclue profoun nausea
seemingly benign tumors may behave in a malignant fash- an vomiting, abominal istention, weight loss, an post-
ion with local recurrence. The risk of malignancy can be pranial epigastric pain, which varies from intermittent to
remembere by “the rule of 5s”: tumors >5 cm or >5 mito- constant, epening on the severity of the uoenal obstruc-
ses per 50 high-power el (E). The ajuvant treatment of tion. Weight loss usually occurs before the onset of symp-
GISTs inclues chemotherapy with imatinib (Gleevec), a toms. It is believe to occur more commonly in women, likely
tyrosine kinase inhibitor. In one stuy, imatinib controlle seconary to the increase prevalence of anorexia. However,
tumor growth in as many as 85% of avance GISTs. Cur- a recent stuy of SMA synrome among intellectually is-
rently, imatinib is recommene for unresectable, metastatic, able chilren showe that it preominantly affects males.
or recurrent lesions. Ajuvant therapy shoul continue for a The iagnosis can be mae by a CT scan, which emon-
total of 3 years (C). Patients that harbor an exon 9 KIT muta- strates a ecrease aortomesenteric angle an a ecrease
tion will require a higher ose of imatinib (800 mg aily ver- istance between the aorta an the SMA, as well as evience
sus 400 mg). The most useful inicators of survival an the of obstruction of the uoenum (C). It can also be iagnose
risk of metastasis inclue the size of the tumor at presenta- by a barium upper GI series or hypotonic uoenography,
tion, the mitotic inex, location within the GI tract, an the emonstrating abrupt or near-total cessation of ow of bar-
absence of tumor rupture. ium from the uoenum to the jejunum. Conservative mea-
References: Blay JY, Bonvalot S, Casali P, et al. Consensus meet- sures that are trie initially are primarily focuse on weight
ing for the management of gastrointestinal stromal tumors. Report gain to increase the mesenteric root fat pa. The operative
of the GIST Consensus Conference of 0-1 March 004, uner the treatment is uoenojejunostomy (E). Nutcracker synrome
auspices of ESMO. Ann Oncol. 005;16(4):566–578. is characterize by compression of the left renal vein by the
Crosby JA, Catton CN, Davis A, et al. Malignant gastrointestinal aorta, superior to the uoenum (B).
stromal tumors of the small intestine: a review of 50 cases from a References: Ason DE, Mitchell JE, Trenkner SW. The superior
prospective atabase. Ann Surg Oncol. 001;8(1):50–59. mesenteric artery synrome an acute gastric ilatation in eating
Dematteo RP, Ballman KV, Antonescu CR, et al. Ajuvant ima- isorers: a report of two cases an a review of the literature. Int J
tinib mesylate after resection of localise, primary gastrointestinal Eat Disord. 1997;1():103–114.
stromal tumour: a ranomise, ouble-blin, placebo-controlle Agrawal GA, Johnson PT, Fishman EK. Multietector row
trial. Lancet. 009;373(9669):1097–1104. CT of superior mesenteric artery synrome. J Clin Gastroenterol.
Joensuu H, Eriksson M, Sunby Hall K, et al. One vs three years of 007;41(1):6–65.
ajuvant imatinib for operable gastrointestinal stromal tumor: a ran- Geskey JM, Erman HJ, Bramley HP, Williams RJ, Shaffer ML.
omize trial: A ranomize trial. JAMA. 01;307(1):165–17. Superior mesenteric artery synrome in intellectually isable chil-
ren. Pediatr Emerg Care. 01;8(4):351–353.
16. B. A hernia sac containing a Meckel iverticulum is
calle a Littre hernia. Lumbar hernias can be either congen- 18. D. The small-intestinal epithelium is acutely susceptible
ital or acquire an occur in the lumbar region of the poste- to raiation injury because raiation has its greatest impact
rior abominal wall. Hernias through the superior lumbar on rapily proliferating cells. Raiation-inuce injury to the
118 PArt i Patient Care

bowel can present with acute or chronic enteritis. Approx- seen with collagen vascular isease, celiac sprue, Crohn is-
imately 75% of patients unergoing raiation therapy for ease, use of sterois, an in immunoecient states. More
abominal an pelvic cancers evelop acute raiation enteri- ominously, it is also associate with ischemic bowel. Thus,
tis transiently. Chronic raiation enteritis results from an it is important to recognize that not all cases of pneumato-
obliterative arteritis in the submucosal vessels, while acute sis are benign. In neonates, it is most commonly associate
raiation enteritis is a transient perio of nausea, vomiting, with necrotizing enterocolitis. The ning of pneumatosis
iarrhea, an abominal pain that occurs aroun 3 weeks intestinalis in association with necrotizing enterocolitis oes
after treatment (C). This leas to progressive submucosal not manate surgical exploration. It is also seen with pyloric
brosis an stricture formation. Not infrequently, patients stenosis, Hirschsprung isease, an other causes of bowel
with raiation-inuce injury may evelop a small bowel obstruction. Pneumoperitoneum can rarely be the result of a
obstruction. The risk of raiation enteritis correlates with benign case of pneumatosis intestinalis because the air-lle
the amount of raiation receive. It is uncommon if the total cysts are thin-walle an can burst.
raiation ose is less than 4000 cGy. The risk of raiation References: Mularski RA, Ciccolo ML, Rappaport WD. Nonsur-
amage increases if the patient receive chemotherapy or has gical causes of pneumoperitoneum. West J Med. 1999;170(1):41–46.
unerlying vascular isease or iabetes (E). Early symptoms Peter S, Abbas M, Kelly K. The spectrum of pneumatosis intesti-
of raiation amage inclue iarrhea, abominal pain, an nalis. Arch Surg. 003;138(1):68–75.
Hsueh KC, Tsou SS, Tan KT. Pneumatosis intestinalis an pneu-
malabsorption an are usually self-limite. The treatment of
moperitoneum on compute tomography: beware of non-therapeutic
acute raiation enteritis inclues antispasmoic agents, anal-
laparotomy. World J Gastrointest Surg. 011;3(6):86–88.
gesic agents, an antiiarrheal agents. Sterois are not use
in the management of raiation enteritis (B). Only a small
group of patients with chronic raiation enteritis will require
20. A. Peutz-Jeghers synrome features mucocutaneous
melanotic pigmentation an hamartomatous polyps (not
surgery for either SBO from stricture formation or stulas.
aenomatous) of the small intestine. It is an autosomal om-
Unlike Crohn isease, for which strictureplasty is use, it is
inant inherite synrome (B). The skin lesions are foun in
not recommene for raiation enteritis because there is a
the circumoral region of the face, buccal mucosa, forearms,
high risk of tissue breakown (A). The extent of macroscopic
palms, soles, igits, an perianal area, whereas the hamarto-
raiation injury is ifcult to etermine on gross inspection.
mas are usually in the jejunum an ileum. The most common
Extensive lysis of ahesions shoul be avoie because this
symptom is recurrent colicky abominal pain (C). Symptoms
creates a risk of an enterotomy an subsequent stula for-
of a bowel obstruction evelop in as many as 50% of patients,
mation as well. The two main surgical proceures are pri-
which is usually ue to intussusception or obstruction by the
mary resection with reanastomosis or bypass. If the source
polyp itself. Hemorrhage or chronic anemia can also occur as
of obstruction is a loop of bowel stuck in the pelvis, it is best
a result of the polyps. The polyps can also unergo aeno-
treate with a bypass rather than an attempt to take own
matous change. Patients are at signicantly increase risk of
the ahesions an risk injury.
eveloping cancer in the GI tract (esophagus, stomach, small
References: Gallan RB, Spencer J. Natural history an surgical
management of raiation enteritis. Br J Surg. 1987;74(8):74–747.
intestine, colon, an pancreas) an extraintestinal cancer
Tavakkoli A, Ashley SW, Zinner MJ. Small Intestine. In: Bruni- (testis, breast, uterus, ovary). Female patients shoul begin
cari F, Anersen DK, Billiar TR, Dunn DL, Hunter JG, Matthews JB, breast an cervical cancer screening starting at age 5. Over
Pollock RE, es. Schwartz’s principles of surgery. 10th e. New York: the long term, cancer evelops in as many as 90% of patients.
McGraw-Hill Eucation; 015:1146–1151. Compare with the general population, they are at 500 times
increase risk of the evelopment of small intestine cancer
19. E. Pneumatosis intestinalis is a raiographic ning (E). Operative intervention is only inicate in the presence
an not a isease unto itself. Its iscovery on imaging is of symptoms (D).
vexing because it can be a completely benign ning, or it References: Boarman LA, Pittelkow MR, Couch FJ, et al. Asso-
can be associate with life-threatening bowel ischemia. It ciation of Peutz-Jeghers-like mucocutaneous pigmentation with
has been ivie into primary an seconary pneumatosis breast an gynecologic carcinomas in women. Medicine (Baltimore).
intestinalis. The primary form is less common an is terme 000;79(5):93–98.
pneumatosis cystoides intestinalis (B). It consists of thin-walle, Giariello FM, Brensinger JD, Tersmette AC, et al. Very high
risk of cancer in familial Peutz-Jeghers synrome. Gastroenterology.
air-lle cysts within the bowel wall, usually in the colon,
000;119(6):1447–1453.
but it can occur anywhere in the GI tract (D). It is an inci-
Wu YK, Tsai CH, Yang JC. Gastrouoenal intussusception ue
ental ning, an the iagnosis is reaily mae on plain to Peutz-Jeghers synrome: a case report. Hepatogastroenterology.
raiograph or CT scan. The gas can appear as linear, curvi- 1994;41():134–136.
linear, bubbly, or cystic. There is no specic treatment (A). van Lier MGF, Wagner A, Mathus-Vliegen EMH, Kuipers EJ,
Seconary pneumatosis intestinalis occurs when there is an Steyerberg EW, van Leeram ME. High cancer risk in Peutz-Jeghers
unerlying isease process. The exact cause of pneumatosis synrome: a systematic review an surveillance recommenations.
intestinalis is unclear, but there seem to be several pathways Am J Gastroenterol. 010;105(6):158–164.
that allow gas to enter the bowel wall. Immunoecient an
inammatory bowel states lea to a loss of mucosal barrier 21. A. The ning of “creeping fat” or mesenteric fat wrap-
function that may permit air to enter the bowel wall. Bowel ping is a gross feature of Crohn isease that is consiere
obstruction leas to gas formation uner pressure. Alter- pathognomonic. It inicates the encroachment of mesenteric
ations in bacteria ora, with invasion of the bowel wall, like- fat onto the serosal surface of the bowel. The presence of fat
wise lea to gas formation. In aults, seconary pneumatosis wrapping correlates well with the presence of unerlying
intestinalis is most often associate with COPD (C). It is also acute an chronic inammation. A recent stuy suggests that
CHAPtEr 9 Alimentary Tract—SmallBowel 119

aiponectin, an aipocyte-specic protein with antiinam- more rarely, a carcinoi crisis can manifest with hyperten-
matory properties foun in mesenteric aipose tissue, may sion. Octreotie is effective for a hypertensive crisis as well.
play an important role in the inammation seen in Crohn Ajunctive treatment with antihistamines may also be of
isease. Terminal ileitis refers to any acute inammation of benet ue to frequent histamine release from carcinoi
the istal ileum ajacent to the ileocecal valve an is there- tumors (B). If the above measures o not resolve the crisis,
fore not pathognomonic. Terminal ileitis is associate with then aborting the proceure may be necessary (D). Dantro-
numerous infectious causes incluing Yersinia enterocolitica lene is the preferre choice of management for malignant
an pseuotuberculosis, Mycobacterium, cytomegalovirus (in hyperthermia (E). This iagnosis is supporte by an increase
acquire immunoeciency synrome), Salmonella, Campy- in en-tial CO. Corticosterois are not use in the man-
lobacter, an Shigella, among others. The ning of terminal agement of carcinoi crisis (A).
ileitis oes not warrant bowel resection. Overall, a minority References: Bax NDS, Woos HF, Batchelor A, Jennings M. Oct-
of patients (10% in one stuy) who present with terminal ile- reotie therapy in carcinoi isease. Anticancer Drugs. 1996;7(Suppl
itis progress to Crohn isease on long-term follow-up (C). 1):17–.
The majority of extraintestinal manifestations in inamma- Warner RR, Mani S, Profeta J, Grunstein E. Octreotie treatment
of carcinoi hypertensive crisis. Mt Sinai J Med. 1994;61(4):349–355.
tory bowel isease improve with bowel resection but anky-
losing sponylitis an primary sclerosing cholangitis o not
(B). Pyoerma gangrenosum is rarely the initial presentation 24. E. The patient has a blin loop synrome, which is ue
of Crohn isease (E). These patients present with small pap- to bacterial overgrowth (A). Symptoms inclue iarrhea, ste-
ules often on the lower extremities that resemble a “cat’s atorrhea, megaloblastic anemia, weight loss, abominal pain,
paw” appearance an can progress to larger ulcerations with an eciencies of fat-soluble vitamins. The megaloblastic
necrotic centers. Rarely, patients evelop pathergy, a con- anemia is ue to the utilization of vitamin B1 by the bacte-
ition in which minor trauma leas to the evelopment of ria. The unerlying cause may be an intestinal abnormality
large an ifcult-to-heal ulcers (D). Debriement of these such as a iverticulum, stula, an intestinal stricture, or it
lesions shoul be avoie because this worsens the lesion. may follow a Billroth II proceure. In the patient presente,
Iniximab or another tumor necrosis factor-alpha inhibitor the large jejunal iverticulum is likely the etiology. The iag-
shoul be use. nosis can be conrme by various means. A barium stuy
References: Menachem Y, Gotsman I. Clinical manifestations of is useful to ene the anatomic abnormality. The d-xylose
pyoerma gangrenosum associate with inammatory bowel is- test involves ingesting xylose, which is metabolize by the
ease. Isr Med Assoc J. 004;6():88–90. bacteria. Excessive CO in the breath conrms the iagno-
Yamamoto K, Kiyohara T, Murayama Y, et al. Prouction of ai- sis. Cultures of the small intestine can be obtaine; however,
ponectin, an anti-inammatory protein, in mesenteric aipose tissue
passing an intestinal tube istal enough to obtain an ae-
in Crohn’s isease. Gut. 005;54(6):789–796.
quate culture can be challenging. Another useful stuy is the
Hatemi I, Hatemi G, Celik AF, et al. Frequency of pathergy phe-
nomenon an other features of Behçet’s synrome among patients
Schilling test. Oral raiolabele vitamin B1 is aministere
with inammatory bowel isease. Clin Exp Rheumatol. 008;6(4 along with parenteral unlabele vitamin B1. The unlabele
Suppl 50):S91–S95. vitamin B1 saturates liver receptors. Thus, if the oral raio-
labele vitamin B1 is properly absorbe an liver receptors
22. B. There are four main cell types in the small intestine: are saturate, the raiolabele vitamin B1 will be excrete
absorptive enterocytes (E), which make up 95% of intestinal in high concentrations in the urine. With pernicious anemia
cells; goblet cells (A); Paneth cells (D); an enteroenocrine an blin loop synrome, oral absorption will be low, an
cells (C). Goblet cells secrete mucus. Paneth cells secrete sev- thus urinary excretion of raiolabele vitamin B1 will be
eral substances incluing lysozyme, tumor necrosis factor, low. When the test is repeate after the aition of intrin-
an cryptiins, which assist in host mucosal efense. There sic factor, vitamin B1 excretion will increase, whereas with
are more than 10 istinct types of enteroenocrine cells that blin loop synrome, vitamin B1 excretion will remain
secrete various gut hormones. The interstitial Cajal cell is a low. The initial treatment of blin loop synrome consists
specialize cell of mesoermal origin that seems to regulate of broa-spectrum antibiotics incluing metroniazole with
peristalsis. It is referre to as an intestinal pacemaker cell. tetracycline as well as vitamin B1 supplementation given
The cells normally express KIT, a tyrosine kinase receptor. parenterally. This shoul be given to all patients present-
These cells have been implicate as the cells of origin of ing with blin loop synrome (D). Prokinetic agents o not
GISTs. seem to help. In aition, ietary moications such as a
References: Miettinen M, Majii M, Lasota J. Pathology an lactose-free iet are useful because patients with blin loop
iagnostic criteria of gastrointestinal stromal tumors (GISTs): a synrome often become lactose intolerant. Meium-chain
review. Eur J Cancer. 00;38:S39–S51. triglycerie iets are more reaily absorbe than long-chain
Sircar K, Hewlett BR, Huizinga JD, Chorneyko K, Berezin I, Ri- triglyceries because they o not require igestive enzymes
ell RH. Interstitial cells of Cajal as precursors of gastrointestinal (B). Resection of the iverticulum is not recommene ini-
stromal tumors. Am J Surg Pathol. 1999;3(4):377–389.
tially (C). Surgery shoul be reserve for patients who fail
repeate meical management attempts.
23. C. The patient has a carcinoi crisis. This has been References: Ross CB, Richars WO, Sharp KW, Bertram PD,
escribe after anesthetic inuction as well as after other Schaper PW. Diverticular isease of the jejunum an its complica-
stressful situations such as biopsies or invasive proceures. tions. Am Surg. 1990;56(5):319–34.
Carcinoi crisis is characterize by hypotension, broncho- Woos K, Williams E, Melvin W, Sharp K. Acquire jejunoileal
spasms, ushing, an tachycaria. The primary treatment is iverticulosis an its complications: a review of the literature. Am
IV octreotie aministere as a bolus of 50 to 100 μg. Even Surg. 008;74(9):849–854.
120 PArt i Patient Care

25. C. Surgical ogma has state that a small bowel obstruc- approve for short-term (maximum 15 oses over 5 ays)
tion (SBO), in the absence of prior surgery or visible exter- in-hospital use only. Patients on long-term narcotics (e.g., for
nal hernia, requires surgical intervention, as the ifferential chronic pain) shoul not use Alvimopan because this popu-
invariably are all surgical iseases, such as internal hernia, lation has an increase risk of myocarial infarction.
appenicitis, intussusception, inammatory bowel isease, References: Luwig K, Enker WE, Delaney CP, et al. Gastro-
malignancy, or obstructe Meckel iverticulum. However, in intestinal tract recovery in patients unergoing bowel resection:
a recent stuy, as many as 40% of SBO in patients without results of a ranomize trial of alvimopan an placebo with a
prior history of surgery resolve nonoperatively. Of those stanarize accelerate postoperative care pathway. Arch Surg.
008;143(11):1098–1105.
that require surgical intervention, the majority were foun
Wolff BG, Weese JL, Luwig KA, et al. Postoperative ileus-relate
to have ahesions espite no prior operations (A). Most cen-
morbiity prole in patients treate with alvimopan after bowel
ters have transitione to a water-soluble oral contrast chal- resection. J Am Coll Surg. 007;04(4):609–616.
lenge to help ecie which patients with ahesive SBOs will Su’a BU, Hill AG. Perioperative use of chewing gum affects the
require surgical intervention. This is being performe even inammatory response an reuces postoperative ileus following
in patients without prior abominal surgery. This entails major colorectal surgery. Evid Based Med. 015;0(5):185–186.
performing nasogastric ecompression for two hours fol- Kotagal M, Hakkarainen TW, Simianu VV, Beck SJ, Alfonso-
lowe by aministration of a water-soluble contrast either by Cristancho R, Flum DR. Ketorolac use an postoperative complica-
mouth or via NG tube. This is followe by plain lms 8 hours tions in gastrointestinal surgery. Ann Surg. 016;63(1):71–75.
later. Patients with plain lms emonstrating contrast in the
colon after 8 hours are unlikely to require surgical interven- 27. A. Crohn isease is the most common primary surgi-
tion while those without contrast in the colon after 8 hours cal isease of the small bowel. Acute onset of abominal
are more likely to fail nonoperative management (C). Rectal pain an iarrhea is not the most common presentation for
contrast is not typically use in the workup nor in manage- Crohn isease; the majority of patients rst present with an
ment of ahesive SBO (D). insiious onset of vague abominal iscomfort (E). It has a
Reference: Ng YYR, Ngu JCY, Wong ASY. Small bowel obstruc- bimoal istribution, with one large peak in the secon an
tion in the virgin abomen: time to challenge surgical ogma with thir ecaes of life an a secon smaller peak in the sixth
evience: Small bowel obstruction in the virgin abomen. ANZ J ecae (C). Several risk factors for Crohn isease have been
Surg. 018;88(1–):91–94. ientie, incluing living in northern latitues, Ashkenazi
Jewish escent, smoking, an a familial inheritance. The rel-
26. C. Postoperative ileus remains a major source of pro- ative risk among rst-egree relatives of patients with Crohn
longe hospitalization in patients unergoing abominal isease is as high as 14 to 15 times greater than in the gen-
surgery. The use of early ambulation, early postoperative eral population. It is also more common in urban areas an
feeing protocols, an routine nasogastric intubation have in patients with a high socioeconomic status. Most stuies
not been shown to be associate with earlier resolution of suggest that Crohn isease is approximately of equal prev-
postoperative ileus (A, E). Reucing opioi use in combi- alence in females an males (D). Breastfeeing may also
nation with the use of nonsteroial antiinammatory rugs be protective against the evelopment of Crohn isease.
such as ketorolac has been shown to reuce the uration of Although meical management is the rst-line treatment for
ileus in most stuies. The mechanism may be a combination Crohn isease, about 75% of patients will ultimately nee
of the reuction in opiois an the antiinammatory prop- surgery. The most common reasons for surgery inclue s-
erties of ketorolac. However, ketorolac has been associate tula, abscess, an obstruction; perforation is quite rare (B).
with an increase risk of operative site an gastrointestinal References: Passier JLM, Srivastava N, van Puijenbroek EP.
(GI) bleeing as well as ui retention (D). Recently, ketoro- Isotretinoin-inuce inammatory bowel isease. Neth J Med.
lac has been also shown to increase the risk of reamission 006;64():5–54.
an reinterventions after GI surgery. Another rug that has Strong SA. Surgical management of Crohn’s isease. In: Holz-
been investigate is erythromycin, which is useful for gast- heimer RG, Mannick JA, es. Surgical treatment: evidence-based and
roparesis because it works by its agonistic effect on the moti- problem-oriented. Munich: Zuckschwert; 001:714–75.
lin receptor. However, it oes not seem to be useful for ileus
an shoul be avoie in cases of obstruction, as woul all 28. A. In the early stages of Crohn isease, patients emon-
promotility agents (B). Metoclopramie is a opaminergic strate small supercial ulcers in the mucosa known as aph-
antagonist with antiemetic an prokinetic properties, but it thous ulcers. These supercial ulcers are often surroune
has also not been shown to be useful for ileus. Gum chewing by a halo of erythema. The ulcers form as a result of submu-
has ha conicting results in the literature, but a recent ran- cosal lymphoi follicle expansion. As the isease progresses,
omize controlle trial from New Zealan emonstrate a the ulcers coalesce to form larger ulcers, which are stellate
signicant reuction in postoperative ileus in patients with shape, as well as eep linear ulcers. Further coalescence of
colorectal cancer unergoing bowel resection (7% versus the ulcers leas to a cobblestone appearance (D), which is a
48%). The most efcacious agent, however, is alvimopan hallmark of Crohn isease. Other hallmarks of Crohn isease
(Entereg), which has been emonstrate in ranomize inclue noncaseating granulomas (C), transmural inamma-
stuies to improve postoperative ileus in patients unergo- tion, serosal thickening (E), an “skip lesions,” meaning that
ing bowel resection. Alvimopan is an opioi receptor antago- the areas of intestinal inammation are iscontinuous. The
nist. It bins μ-opioi receptors in the GI tract an selectively noncaseating granulomas are foun in both areas of active
inhibits the opioi effects on GI function an motility while isease, an grossly normal-appearing intestine is seen in
not affecting opioi analgesia. It is the rst US Foo an Drug all layers of the bowel wall an in mesenteric lymph noes
Aministration–approve rug for postoperative ileus. It is (B). Because the inammation is transmural, iname loops
CHAPtEr 9 Alimentary Tract—SmallBowel 121

of bowel become ahere to one another, thereby leaing to rugs act by inhibiting DNA synthesis an thus suppressing
brosis, stricture formation, intraabominal abscess, stulas, the function of T cells an natural killer cells. A secon-line
an, rarely, free perforation. agent for maintenance of remission is methotrexate.
Reference: Levine MS. Crohn’s isease of the upper gastrointes-
tinal tract. Radiol Clin North Am. 1987;5(1):79–91.
30. D. Approximately three-fourths of patients with Crohn
29. A. Numerous pharmacologic agents are use to treat isease will eventually require surgery. Inications for sur-
Crohn isease. Treatment options shoul be ivie into gery inclue failure of meical management, intestinal
those use for maintenance therapy for mil active isease, obstruction, stula, abscess, bleeing, an perforation. In
those use to treat an acute exacerbation, an rugs for chilren, growth retaration is another inication. Because
maintaining remission. In patients with mil active isease, patients with Crohn isease will often require repeat opera-
the most commonly use rug is sulfasalazine, an amino- tions, it is important to avoi unnecessary resection of small
salicylate that acts as an antiinammatory agent. This is bowel because this puts the patient at risk of short bowel syn-
particularly useful in patients with colitis an ileocolitis. rome. As such, several principles of surgical management
Mesalamine is another antiinammatory agent in the same shoul be followe. Surgical resection shoul be limite to
family as sulfasalazine. It seems to have fewer sie effects the segment of bowel that is causing the complication. Other
owing to the fact that it is activate by colonic bacteria, thus areas of active isease shoul be left alone, provie they
limiting its action to the colon. For acute are-ups, the treat- are not causing obvious complications. Resection margins of
ment of choice remains corticosterois, prenisone in partic-  cm beyon grossly visible isease are recommene (A).
ular. Prenisone is highly effective in inucing remission (in Resection margins have not been shown to affect recurrence.
approximately three-fourths of patients); however, ue to the The presence of microscopic isease in the resection margin
sie effects of long-term use, it is not recommene for long- also oes not aversely affect outcome or recurrence. Thus,
term prevention of remission (B). Buesonie (C), a syn- frozen section is unnecessary (B). When the inication for
thetic glucocorticoi, is another option. It has an avantage surgery is SBO, strictureplasty has been shown to be equally
over prenisone in that it has a markely reuce systemic effective as resection for jejunal an ileal isease while sparing
absorption an thus fewer long-term sie effects. Neverthe- bowel length. Two types of strictureplasty are recommene:
less, it can also suppress the arenal glan. If corticosterois the Heineke-Mikulicz pyloroplasty (for strictures <1 cm in
are ineffective in inucing remission, the next step woul length) an the Finney pyloroplasty (for strictures ≤5 cm
be to aminister iniximab (E), a monoclonal antiboy that in length) (E). A potential rawback of these techniques is
targets tumor necrosis factor-alpha. Care must be use in that they may potentially leave an unetecte malignancy
aministering iniximab. Because it targets tumor necrosis behin. Thus, uring the course of a strictureplasty, biopsy
factor-alpha, a cytokine that regulates inammatory reac- specimens of any intraluminal ulcerations shoul be taken.
tions, patients who receive iniximab are at increase risk of Duoenal Crohn isease is much less common, an thus
acquiring opportunistic infections such as tuberculosis an guielines are less clear. However, current recommenations
aspergillosis. It is also associate with activation of latent are to perform a bypass of uoenal strictures, such as with
multiple sclerosis, emyelinating central nervous system a gastrojejunostomy an uoenojejunostomy, epening
isorers, an worsening congestive heart failure. Iniximab on the location. Duoenal resection is not recommene (C).
has also been shown to be effective in healing complex stu- Duoenal strictureplasty has been rarely reporte. For colon
las associate with Crohn isease. Rarely, it has been associ- isease, resection is recommene, again limiting resection
ate with T-cell lymphoma an almost exclusively in young to the isease segment causing symptoms. In a metaanal-
teenage males. Antibiotics have an ajunctive role in the ysis, 90% of recurrences occurre at nonstrictureplasty sites.
treatment of infectious complications associate with Crohn References: Fazio V, Marchetti F, Church M. Effect of resection
margins on the recurrence of Crohn’s isease in the small bowel: a
isease (D). They are use to treat patients with perianal is-
ranomize controlle trial. Ann Surg. 1996;4(4):563–571.
ease, enterocutaneous stulas, an active colon isease an
Tichansky D, Cagir B, Yoo E, Marcus SM, Fry RD. Stricture-
ai in situations in which bacterial overgrowth has occurre. plasty for Crohn’s isease: meta-analysis. Dis Colon Rectum.
Once remission has been achieve after an acute are-up, it 000;43(7):911–919.
is important to maintain remission. Although corticosterois Yamamoto T, Fazio VW, Tekkis PP. Safety an efcacy of stricture-
woul theoretically be useful, the sie effects preclue long- plasty for Crohn’s isease: a systematic review an meta-analysis: W.
term aministration. Iniximab is use to maintain remis- Donal Buie, M.., eitor. Dis Colon Rectum. 007;50(11):1968–1986.
sion, as are azathioprine an 6-mercaptopurine. These latter
Alimentary Tract—
LargeIntestine
JOSEPH HADAYA, FORMOSA CHEN, AND BEVERLEY A. PETRIE 10
ABSITE 99th Percentile High-Yields
I. Aequate Colonoscopy Metrics
A. Shoul be able to intubate the cecum in ≥90% of all cases an in ≥95% of cases when the inication is
screening in a healthy ault
B. Aenoma etection rates of at least 5% in patients >50 years ol who are unergoing screening
colonoscopy
C. The mean withrawal time is ≥6 minutes in colonoscopies with normal results that are performe in
patients with intact anatomy
D. Mucosally base peunculate polyps an sessile polyps < cm in size are resecte or ocumentation of
unresectability is mae
E. Perforation rates shoul not excee 1 in 500 colonoscopies overall, an 1 in 1000 screening colonoscopies
F. Incience of polypectomy bleeing shoul be <1%
G. Postpolypectomy bleeing is manage nonoperatively in ≥90% of cases

II. Initial Colonoscopy Screening Guielines

Risk factor Screening


Normal risk Start at age 50 and continue every 10 years; alternatives to colonoscopy
include annual fecal occult blood test (FOBT) or Ěexible sigmoidoscopy
every 5 years or annual FOBT plus Ěexible sigmoidoscopy every 5 years
First-degree family member with Start at age 40 or 10 years prior to ęrst-degree members diagnosis age;
colorectal cancer prior to age 60 continue screening every 5 years
Two ęrst-degree family members with Start at age 40 or 10 years prior to ęrst-degree members diagnosis age;
colorectal cancer diagnosis at any age continue screening every 5 years
Ulcerative colitis (UC) Colonoscopy every 1–2 years starting 8 years after onset of disease or at same
time of primary sclerosing cholangitis (PSC) diagnosis
Familial adenomatous polyposis (FAP) Annual sigmoidoscopy starting at age 10–12
Lynch syndrome Colonoscopy every 1–2 years starting at age 20, or 10 years prior to age of
diagnosis of youngest aěected family member
Personal cancer history Colonoscopy at 1 year following resection, or 6 months if no preoperative
completion colonoscopy was performed; then every 3 years

123
124 PArt i Patient Care

III. Postcolonoscopy Frequency Guielines Base on Finings

Finding Recommended interval for surveillance colonoscopy


No abnormalities 10 years
1–2 tubular adenomas <10 mm 7–10 years
3–4 tubular adenomas <10 mm 3–5 years
5–10 tubular adenomas <10 mm 3 years
Adenoma >10 mm 3 years
Adenoma with tubulovillous or villous histology 3 years
Adenoma with high-grade dysplasia 3 years
>10 adenomas 1 year
Piecemeal resection of adenoma ≥20 mm 6 months

A. Surgically resect polyps that are unable to be enoscopically remove


1. If T1 lesion present, polypectomy is sufcient if all the following are true: (1) margins are clear of ysplasia
an cancer by 1 to  mm (peunculate), or submucosal invasion <1 to  mm (nonpeunculate), ()
no lymphovascular or perineural invasion is present, (3) moerate-well ifferentiate lesion, (4) en bloc
enoscopic resection; all others require a formal oncologic resection with aequate lymphaenectomy

IV. Staging of Colon Cancer an Management

Stage TNM staging Management


Stage I T1-2, N0, M0 Colectomy
Stage II T3-4, N0, M0 Colectomy (with en bloc resection of adjacent organs, if needed); high-risk
features (obstruction, T4, <12 LN harvested, high-grade tumor, perineural or
lymphovascular invasion) should get adjuvant chemotherapy
Stage III Any T, N1 or N2, M0 Colectomy with adjuvant chemotherapy
Stage IV Any T, Any N, M1 Diěuse metastatic disease: systemic chemotherapy
Oligometastatic liver/lung disease: neoadjuvant vs adjuvant chemotherapy,
staged vs simultaneous colectomy and metastatectomy

V. Familial Aenomatous Polyposis (FAP)


A. Autosomal ominant, mutations in APC gene (tumor suppressor); up to 5% are de novo mutations;
mean age for polyp evelopment is 16 to 18 years, with iffuse polyposis at age 0 to 40, colon cancer in
approximately 100%, left-sie
B. Increase risk of cancer is ue to substantial number of aenomas at early age rather than intrinsic
malignant potential of each aenoma (versus hereitary nonpolyposis colorectal cancer)
C. Surgery: generally, by age 0, or for high-grae ysplasia, suspecte colorectal cancer, gastrointestinal
bleeing, rapi increase in polyp number or inability to survey; surgical options: proctocolectomy
with ileal–pouch anal anastomosis (IPAA), total colectomy with ileorectal anastomosis (IRA) or en
ileostomy
D. Postcolectomy surveillance necessary (risk for malignancy at rectal cuff or within ileal pouch)

VI. Hereitary Nonpolyposis Colon Cancer (HNPCC/Lynch Synrome)


A. Autosomal ominant, mutations in DNA mismatch repair (MMR) genes (MLH1, MSH2, MSH6, PMS2)
B. Risk of colorectal cancer 1% to 90%, occurs at younger age than average risk; aenomacarcinoma
sequence progresses faster in HNPCC than sporaic isease, synchronous lesions common (7%) as well
as metachronous
C. Amsteram II criteria (family history-base screening): at least 3 relatives with colorectal or Lynch-
associate cancer, involving at least  generations, at least 1 iagnose before age 50, at least 1 is a rst-
egree relative of the other two, an familial aenomatous polyposis exclue (“3--1-1-0 rule”)
CHAPtEr 10 Alimentary Tract—LargeIntestine 125

VII. Ulcerative Colitis an Crohn’s Disease


A. Ulcerative colitis
1. Contiguous inammation from rectum extening proximally, spares anus, mucosal isease only,
crypt abscesses are common
. Extraintestinal manifestations inclue ankylosing sponylitis, primary sclerosing cholangitis, uveitis,
pyoerma gangrenosum (ankylosing sponylitis an primary sclerosing cholangitis o not improve
after proctocolectomy)
3. Management: mesalamine or iniximab for maintenance therapy; sterois for acute ares
4. Inications for elective surgery inclue growth failure in chilren, intractable isease espite
escalating meical therapy, chronic steroi epenence, ysplasia, or malignancy
5. Elective surgical options: total proctocolectomy with ileal pouch–anal anastomosis (IPAA), total
abominal colectomy with ileorectal anastomosis (in rare cases where rectum is spare, although
rectum must be screene for malignancy), total abominal colectomy with en ileostomy
6. Pouchitis: inammation/infection of pouch, treat with ciprooxacin or metroniazole
7. Emergent surgery: severe colitis with toxic megacolon, perforation, or bleeing; perform total
abominal colectomy with en ileostomy (possible pouch formation later)
B. Crohn’s isease
1. Transmural inammation with skip lesions, any segment affecte (terminal ileum most common) but
generally spares rectum; strictures, stula, granulomas an creeping fat common
. Extraintestinal manifestations inclue arthralgias, megaloblastic anemia, erythema noosum
3. Maintenance therapy for mil isease inclues mesalamine or immunomoulator (azathioprine,
6-mercaptopurine, methotrexate); for moerate to severe isease, iniximab (or other anti-TNF
therapy) monotherapy or iniximab plus azathioprine
4. Acute isease manage with sterois, anti-TNF agents, or immunomoulators
5. Surgery reserve for complications (strictures, perforation, obstruction, malignancy, stula); most
commonly strictures

VIII.Sigmoi an Cecal Volvulus


A. If no evience of peritonitis, ischemia, or perforation, sigmoi volvulus can be manage with exible
sigmoioscopy to etorse an reuce bowel, place rectal tube (ecompression), an allow for bowel
preparation; high risk of recurrence, so sigmoiectomy with primary anastomosis shoul be performe
within 1 to 3 ays
B. For peritonitis, ischemia, or inability to etorse: emergency resection (with en colostomy)
C. Cecal volvulus is not amenable to enoscopic therapy an shoul be manage surgically with right
colectomy ue to high risk of recurrence

IX. Clostridium difcile Colitis


A. Anaerobic, gram-positive ro; healthcare acquire infection, can present up to 3 to 4 weeks after
antibiotic aministration (even single ose); upate 01 guielines below

Clinical Treatment
Initial episode Preferred: ędaxomicin 200 mg BID for 10 days
Alternative: oral vancomycin 125 mg 4× daily for 10 days OR
If above unavailable, oral metronidazole 500 mg ×3 daily for 10 days
1st recurrence Preferred: ędaxomicin 200 mg BID for 10 days, OR BID for 5 days followed by once every
other day for 20 days
Alternative: vancomycin 125 mg 4× daily for 10 days
2nd and 3rd recurrence Preferred: ędaxomicin 200 mg BID for 10 days, OR BID for 5 days followed by once every
other day for 20 days
Alternative: vancomycin 125 mg 4× daily for 10 days followed by rifaximin 400 mg 3× daily
for 20 days
4th recurrence Fecal microbiota transplantation
Fulminant infection Vancomycin 500 mg 4× daily; if ileus, consider adding rectal instillation of vancomycin;
(hypotension, ileus, IV metronidazole (500 mg every 8 hours) should be administered with oral or rectal
megacolon*) vancomycin, particularly if ileus is present

*Should consider total abdominal colectomy as well


126 PArt i Patient Care

X. Diverticulitis
A. Mil uncomplicate iverticulitis, minimal to no comorbiities, no systemic signs of infection, minimal
pain, an able to tolerate oral intake) can be treate without antibiotics; for abscesses >3 cm in size,
percutaneous rainage recommene
B. After resolution of complicate iverticulitis, the patients shoul unergo colonoscopy if none recently
(higher risk of associate malignancy)
C. For patients with complicate iverticulitis (stula, obstruction, stricture or abscess requiring rainage),
elective colectomy typically recommene
D. For patients with uncomplicate iverticulitis, ecision for elective colectomy may be consiere for
those with multiple, recurrent amissions for uncomplicate iverticulitis that ecrease quality of life;
young age shoul not be a eterminant to recommen surgery
E. For patients presenting with iffuse peritonitis or failure of nonoperative management, urgent
sigmoiectomy shoul be offere an if expertise is present, minimally invasive surgery shoul be
employe
F. Primary anastomosis is preferable to Hartmann proceure in hemoynamically stable,
immunocompetent patients younger than 85 years, even in Hinchey III or Hinchey IV isease (LADIES
trial); however, this ecision shoul be iniviualize
G. Hartmann proceure preferre in hemoynamically unstable patients, immunocompromise, those on
pressors

Category Findings/symptoms Management


Emergent Conditions
Hinchey I Pericolonic abscess Antibiotics, bowel rest, and percutaneous drainage
if abscess >3 cm
Hinchey II Pelvic abscess
Hinchey III Purulent peritonitis Surgery: Hartmann procedure vs resection with
primary anastomosis, diverting ileostomy
Hinchey IV Feculent peritonitis
(controversial)
Elective
Smoldering diverticulitis Chronic abdominal pain Sigmoidectomy if disease is debilitating
Stricture Thin stools, constipation
Colovesical ęstula Pneumaturia, fecaluria, recurrent UTI Sigmoidectomy, ęstula takedown (postoperative
Foley catheter to decompress bladder)

XI. Lower GI Bleeing


A. Initial evaluation: colonoscopy is preferre in stable patients, allows for iagnosis, localization, potential
intervention, an proctoscopy
1. Shoul be performe as soon as patient is resuscitate an has complete aequate bowel
preparation, which may only inclue a Fleet enema prior to scope; limite utility without
preparation, with low cecal intubation rates an obstructe views; nasogastric tube can be place
for aministrating bowel preparation an to rule out upper GI blee; if bleeing briskly, consier
simultaneous upper enoscopy
B. If bleeing briskly, an source not visualize with colonoscopy, consier angiography (therapeutic,
accurate localization) or tagge RBC scan (most sensitive, etects blee <0.5cc/min)
C. If hemoynamically unstable, upper GI bleeing rule out with EGD, hemorrhois rule out with
proctoscopy, an bleeing source still not foun, but high suspicion for istal to ileocecal valve, procee
with total abominal colectomy (segmental resection iscourage)

XII. Ischemic Colitis: blooy iarrhea an left-sie abominal pain in elerly with volume epletion, risk
factors inclue cariovascular isease an a short interval of volume epletion/hypotension; watershe
areas (splenic exure) most commonly affecte
A. Cross-sectional imaging generally shows nonspecic segmental colon thickening; best stuy is
enoscopy, which emonstrates clearly emarcate patchy areas of erythema an ulceration
1. Conservative management with ui resuscitation typically effective; antibiotics to prevent bacterial
translocation
CHAPtEr 10 Alimentary Tract—LargeIntestine 127

Questions
1. A 4-year-ol man with no past meical history 4. A 55-year-ol woman unergoes laparoscopy
presents with a 5-ay history of left lower for presume appenicitis. At surgery, she is
quarant abominal pain. He is foun to have foun to have perforate appenicitis with what
sigmoi iverticula with associate pericolonic appears to be peritoneal stuing. The patient
straning an mesenteric lymphaenopathy on unergoes appenectomy an biopsy of the
CT imaging. He is treate with IV antibiotics, peritoneum. Final pathology reveals appeniceal
his pain resolves, a iet is restarte, an he is aenocarcinoma. Subsequent workup reveals no
transitione to oral antibiotics an ischarge. evience of aitional metastatic sprea to the
Which of the following is most appropriate? liver or lungs. Further treatment woul consist of:
A. A probiotics A. No further treatment
B. Scheule elective sigmoi colectomy B. Systemic chemotherapy
C. Scheule colonoscopy C. Intraperitoneal chemotherapy
D. No further recommenations D. Cytoreuctive surgery an hyperthermic
E. Scheule repeat CT scan of the abomen an intraperitoneal chemotherapy
pelvis with IV contrast E. Cytoreuctive surgery an systemic
chemotherapy
2. A 68-year-ol female presents to the emergency
epartment (ED) with obstipation, nausea, an 5. A 73-year-ol female with no signicant meical
graually worsening abominal istention. problems is foun to have a 3-cm hepatic
She is afebrile with normal vital signs an has exure mass on screening colonoscopy. A
moerate istention on examination with mil biopsy emonstrates moerately ifferentiate
abominal tenerness. A CT scan of the abomen aenocarcinoma. Her laboratory tests are
an pelvis suggests a large bowel obstruction notable for microcytic anemia an normal liver
with a transition point in the left (escening) function tests. Which of the following is the most
colon an multiple hypoattenuating masses appropriate preoperative staging strategy?
in the liver an base of the lungs. There is also A. CT scan of the chest, abomen, an pelvis, an
evience of small bowel ilation. He is afebrile transrectal enoscopic ultrasoun
an hemoynamically stable. A nasogastric B. CT scan of the chest, abomen, an pelvis, an
tube is place. What is the next best step in the carcinoembryonic antigen
management of this patient? C. CT scan of the chest, abomen, an pelvis, MRI
A. Colonoscopy an uncovere stent placement of the brain, an carcinoembryonic antigen
B. Colonoscopy an covere stent placement D. PET/CT of the chest, abomen, an pelvis, MRI
C. Left colectomy of the brain, an carcinoembryonic antigen
D. Diverting loop ileostomy E. PET/CT of the chest, abomen, an pelvis,
E. Initiate inpatient chemotherapy MRI of the brain

3. Which of the following is true about colonic 6. A 45-year-ol woman with a 15-year history
physiology? of pancolitis from UC unergoes surveillance
A. The colon absorbs the majority of water in the colonoscopy. No polyps are etecte. Ranom
gastrointestinal tract biopsy samples are taken, an nal pathology
B. Soium is absorbe actively via Na+,K+- nings reveal high-grae ysplasia from
ATPase the sigmoi colon region. Recommene
C. Ammonia reabsorption is unaffecte by management woul be:
luminal pH A. Repeat colonoscopy in 6 months with
D. Chlorie is secrete aitional ranom biopsies
E. It prouces no nutrients B. Sigmoi colectomy
C. Total colectomy with ileorectal anastomosis
D. Total proctocolectomy with ileostomy
E. Restorative proctocolectomy with ileal pouch–
anal anastomosis
128 PArt i Patient Care

7. The earliest manifestation of ulcerative colitis is: 12. A 75-year-ol woman presents with mil iffuse
A. Mucosal ulcerations abominal pain an iarrhea that is positive on
B. Mucosal eema fecal immunochemical test. Her meical history
C. Plasmacytosis is negative. Her WBC count is normal, as is her
D. Pseuopolyps hematocrit. A CT scan shows mil thickening of
E. Crypt abscesses the colonic wall at the splenic exure with some
associate pericolic fat straning. Which of the
8. With appenicitis uring pregnancy, the factor following is the best next step in management?
most strongly associate with fetal mortality is: A. Diagnostic laparoscopy
A. Fetal gestational age B. Exploratory laparotomy
B. Open appenectomy instea of laparoscopy C. IV antibiotics an ui hyration
C. Maternal comorbiities D. Colonoscopy
D. Appeniceal rupture E. Mesenteric angiography
E. Delay in antibiotic aministration
13. A 65-year-ol institutionalize patient presents
9. Which of the following is true about hereitary with a -ay history of abominal istention,
nonpolyposis colon cancer (HNPCC) (Lynch nausea, an obstipation. Physical examination
synrome)? is signicant for marke istention with mil
A. It is not associate with a higher risk of upper iffuse abominal tenerness, no guaring,
genitourinary tract cancer. an no reboun. The WBC count is 10,000
B. It is consiere an autosomal recessive cells/μL. Plain lms reveal a massively ilate,
synrome. inverte U-shape (omega sign) loop of bowel.
C. Screening colonoscopy shoul begin at age 1. Management shoul consist of:
D. Colonic malignancy has the same prognosis as A. Enoscopic etorsion
sporaic cancer. B. Enoscopic etorsion followe by sigmoi
E. Moie Amsteram criteria requires one colectomy uring the same hospitalization
family member to be iagnose before age 40. C. Enoscopic etorsion followe by elective
sigmoi colectomy in the case of a recurrence
10. Which of the following is true regaring familial D. Exploratory laparotomy with sigmoi
juvenile polyposis? colectomy, on-table lavage, an primary
A. It is autosomal recessive anastomosis
B. The polyps are hamartomas E. Exploratory laparotomy with sigmoi
C. The risk of colon cancer is 100% by age 50 colectomy, proximal colostomy, an oversewn
D. Once a polyp is etecte, total rectal stump
proctocolectomy is recommene
E. There is no association with upper GI 14. A 38-year-ol woman presents with a 1-ay
malignancy history of nausea, vomiting, abominal istention,
an obstipation. The physical examination is
11. Which of the following is true regaring colonic signicant for istention with a tympanic mass
polyps? in the left upper quarant an mil abominal
A. Tubulovillous aenomas have a lower tenerness. She is hemoynamically stable
malignancy risk than tubular aenomas with no leukocytosis or lactic aciosis. A plain
B. Sessile serrate polyps shoul be resecte abominal raiograph reveals a markely ilate,
C. The polyps in Peutz-Jeghers synrome are kiney-shape loop of bowel with haustral
hyperplastic markings that project from the right lower
D. Pseuopolyps are commonly foun in FAP quarant to the left upper quarant. The iagnosis
E. In an aenomatous polyp, the risk of malignancy is conrme with CT scan lacking evience of
is relate to its location in the GI tract bowel malperfusion. Which of the following is
likely to be the best treatment option?
A. Cecostomy tube placement
B. Operative etorsion with cecopexy
C. Right hemicolectomy with an ileostomy an
mucus stula
D. Initial enoscopic etorsion with a subsequent
right hemicolectomy
E. Right colectomy with primary anastomosis
CHAPtEr 10 Alimentary Tract—LargeIntestine 129

15. Which of the following is true regaring 18. A 47-year-ol morbily obese male unerwent
iverticular iseases of the lower GI tract? emergent sigmoiectomy with en colostomy
A. They occur most commonly in the escening creation yesteray morning for perforate
colon iverticulitis. Evaluation of the colostomy on
B. The rectum can be affecte morning rouns reveals iffusely usky mucosa.
C. Incientally iscovere cecal iverticula On examination with a test tube an light, the
require surgical management usky area appears to be supercial to the fascia.
D. Elective sigmoi resection shoul be precee Management consists of:
by a mechanical bowel preparation with oral A. Reexploration in the operating room (OR),
an IV perioperative antibiotics resection of ischemic colon, an stoma
E. They are associate with a long, reunant relocation
colon B. Reexploration in the OR, segmental colon
resection, an placement of stoma at the same
16. A 50-year-ol male is unergoing a screening site
colonoscopy uner intravenous (IV) seation. C. Reexploration in the OR, on-table bowel prep,
Near the en of the proceure, he briey becomes an primary colonic anastomosis
unresponsive, requiring a sternal rub to arouse D. Observation an reevaluate the colostomy in
him. IV anesthetics are weane off an the 1 to 4 hours
proceure is complete. In the recovery room, a E. IV antibiotics
chest x-ray is performe to rule out an aspiration
event before ischarge. There is no consoliation 19. A 75-year-ol male with chronic constipation
in the lungs, but free air is seen uner the presents with severe abominal pain an fever.
iaphragm. The patient has no complaints, the CT scan shows free air an straning in the colon.
abomen is soft, he woul like to eat, an he has The colon an rectum appear to be ilate an
normal vital signs. Which of the following is the lle with large masses of stool. Intraoperatively,
best next step? a roun perforation about 1 cm in iameter is
A. Exploratory laparotomy foun in the colon with thickene balls of stool
B. Diagnostic laparoscopy protruing out. Which of the following is true
C. Serial abominal exam for 6 hours regaring this patient?
D. Amit to hospital, IV antibiotics, an bowel A. The perforation is most likely to occur at the
rest splenic exure
E. Discharge home B. It is associate with nonsteroial
antiinammatory rug (NSAID) use
17. Ten years after an abominoperineal resection for C. It is best manage by primary closure an
locally avance rectal cancer, a patient presents washout
with a hernia ajacent to his stoma that causes D. Anticholinergic agents coul have prevente
him iscomfort an interferes with the placement this conition
of his colostomy bag. It has been increasing in E. The perforation is usually at the mesenteric
size over the last several months. Which of the borer
following is true regaring this conition?
A. Chronic obstructive pulmonary isease
(COPD) is the strongest risk factor
B. Treatment for this patient inclues weight loss
an a support evice such as a hernia belt
C. Stoma relocation is the superior treatment
D. Prophylactic mesh placement at the initial
operation ecreases risk of this complication
E. This complication is more common with loop
ileostomy than en colostomy
130 PArt i Patient Care

20. A 71-year-ol female with COPD is recovering 23. A 65-year-ol woman presents with massive
from pneumonia in the intensive care unit bleeing per rectum. Her initial bloo pressure
(ICU). She is on a ventilator. Her abomen is in the ED is 80/60 mmHg, with a heart rate of
acutely istene, an she has not ha a bowel 10 beats per minute. After volume resuscitation,
movement in several ays. Imaging emonstrates the bloo pressure increases to 10/80 mmHg.
a cecum measuring 8 cm in iameter with gas A nasogastric aspirate is negative for bloo. A
pattern of istention extening to the rectum. besie anoscopy is performe an hemorrhoial
There is no stool in the rectal vault. Her vital bleeing is rule out, an the patient remains
signs are stable. Her octor woul like to start hemoynamically stable. The next step in her
neostigmine. Which of the following is true management is:
regaring the aministration of neostigmine for A. Colonoscopy
this patient’s conition? B. Mesenteric arteriography
A. History of coronary artery isease is C. Tagge re cell scan
consiere a contrainication D. Upper enoscopy
B. History of secon-egree heart block is E. Exploratory laparotomy
consiere a contrainication
C. Neostigmine shoul not be given as a 24. A 35-year-ol patient with a history of ulcerative
continuous infusion colitis who has unergone restorative total
D. Neostigmine is effective in 0% of patients proctocolectomy with an ileal pouch–anal
with this conition anastomosis presents with a 3-ay history of
E. If a bolus of neostigmine is not successful, abominal pain, increase bowel movements,
repeat boluses shoul be avoie hematochezia, an fever. Which of the following
is true regaring this conition?
21. For the patient in question 0, neostigmine A. Biopsy is typically not require
an enoscopic ecompression fail to improve B. This is an uncommon complication
symptoms. She appears more istene an C. Use of probiotics is not helpful
uncomfortable. Repeat x-ray shows cecum is now D. Urgent excision of the J-pouch is often
10 cm in iameter. She is taken to the operating necessary
room. Intraoperatively, her colon appears E. Ciprooxacin is more effective treatment than
eematous an ilate to 10 cm, but there are metroniazole
no signs of ischemic bowel ientie. Which of
the following is the most appropriate treatment 25. Which of the following is true about familial
option? aenomatous polyposis?
A. Total abominal colectomy with ileoanal A. Microsatellite instability is a major contributor
anastomosis to this isease
B. Proctocolectomy with ileal pouch–anal B. It is not associate with extraintestinal
anastomosis manifestations
C. Placement of cecostomy tube C. Patients with the gene mutation shoul begin
D. Transanal retrograe colonic insertion of a screening with exible sigmoioscopy at age
long multiperforate Faucher tube 0
E. Subtotal colectomy with en ileostomy D. Patients unergoing prophylactic
proctocolectomy have a lower subsequent risk
22. A 3-year-ol male iagnose with ulcerative of eveloping periampullary carcinoma
colitis 1 year ago presents to the emergency E. Upper enoscopy shoul be performe every
epartment with jaunice skin. He is amitte 1 to 3 years
an workup is consistent with primary sclerosing
cholangitis. Which of the following is aitionally
recommene?
A. Immeiate screening colonoscopy
B. Immeiate colonoscopy with ranom biopsies
C. Colonoscopy with ranom biopsies at 8 to 10
years after his UC iagnosis
D. Screening colonoscopy at age 50
E. Symptom-riven colonoscopy as neee
CHAPtEr 10 Alimentary Tract—LargeIntestine 131

26. A 10-year-ol boy with acute myelogenous 29. Five ays after appenectomy, liqui stool
leukemia presents with right lower quarant is note to be coming out of the right lower
abominal pain an tenerness. He recently quarant woun. The patient is hemoynamically
complete chemotherapy. His temperature is stable, afebrile, an tolerating an oral iet. Which
10°F an WBC count is 900 cells/μL. A CT of the following is true about this conition?
scan reveals inammation an thickening of the A. The patient shoul have nothing by mouth
right colon an straning in the ajacent fat. an be place on parenteral nutrition
Management consists of: B. Octreotie shoul be starte
A. IV antibiotics, bowel rest, an IV uis C. The patient shoul immeiately unergo
B. Right hemicolectomy with primary reexploration an a cecostomy
ileotransverse colostomy D. The patient shoul immeiately unergo
C. Right hemicolectomy with ileostomy an reexploration an a right colectomy
mucous stula E. The conition resolves spontaneously in most
D. Cecostomy instances
E. Appenectomy
30. Which of the following is true regaring
27. A 40-year-ol man presents with a 5-ay history chemotherapy for colon carcinoma?
of right lower quarant abominal pain, anorexia, A. The combination of 5-uorouracil an
an fever. On physical examination, he is focally leucovorin prolongs survival in stage IV colon
tener in the right lower quarant, an a mass cancer
is palpable. A CT scan shows a small (<1 cm) B. Raiation therapy is commonly use in
abscess surrouning an iname appenix. After combination with chemotherapy in the
ui resuscitation an intravenous antibiotics, management of colon cancer
which of the following is the most appropriate C. Bevacizumab (Avastin) has not been shown to
management? prolong survival in stage IV colon cancer
A. CT-guie rainage followe by interval D. 5-uorouracil an leucovorin prolong survival
appenectomy in patients with stage III colon cancer
B. Initial nonoperative management followe by E. Bevacizumab (Avastin) is a monoclonal
interval appenectomy antiboy against epiermal growth factor
C. Laparoscopic appenectomy receptor
D. Open appenectomy
E. Nonoperative management 31. A 40-year-ol man unergoes an appenectomy
for acute appenicitis. Final pathology reveals
28. A 55-year-ol man is unergoing a screening a 1.1-cm carcinoi at the base of the appenix.
colonoscopy. A benign-appearing 1-cm Lymph noes are negative. Which of the
peunculate polyp is remove from the sigmoi following is true about this conition?
colon with a col snare. Four hours later, severe A. No further treatment is necessary
left lower quarant pain evelops in the patient. B. There is a signicant chance that carcinoi
A CT scan reveals free intraperitoneal air with synrome will evelop in the patient
minimal fat straning aroun the sigmoi colon. C. The patient shoul receive chemotherapy
The situation is best manage by: D. The patient shoul unergo reexploration an
A. Diverting proximal colostomy a right colectomy
B. Resection of sigmoi colon with an en E. Most appeniceal carcinois are .5 cm or
colostomy an oversew of the rectum larger when iscovere
C. Resection of the sigmoi colon with primary
anastomosis 32. An important source of energy for colonocytes,
D. Primary closure of the perforation particularly in the setting of iversion colitis, is:
E. Broa-spectrum antibiotics an nasogastric A. Ketone boies
ecompression B. Glucose
C. Amino acis
D. Propionate
E. Glutamine
132 PArt i Patient Care

33. A 15-year-ol boy presents to a colorectal clinic 37. A 50-year-ol woman presents with symptoms
with a family history of familial polyposis. an signs of acute appenicitis. At surgery, there
APC gene testing is performe, an the result is a large amount of gelatinous ascites with
is positive. Flexible sigmoioscopy reveals peritoneal implants. This most likely represents:
eight polyps in the sigmoi. Colonoscopy A. Benign ovarian tumor
reveals no other polyps. Polyps are consistent B. Appeniceal mucinous aenoma
with aenomatous polyps without evience C. Tuberculous appenicitis
of malignancy. Which of the following is the D. Salmonella enteritidis
recommene management? E. Yersinia enterocolitica
A. Repeat sigmoioscopy in 6 months
B. Cyclooxygenase- inhibitors, repeat 38. A 75-year-ol woman presents to clinic for
sigmoioscopy in 6 months follow-up after four episoes of uncomplicate
C. Total colectomy with ileorectal anastomosis iverticulitis in the past year, each of which
D. Total proctocolectomy with continent require a 5-ay hospitalization for IV antibiotics
ileostomy an bowel rest. The patient is a iabetic. Previous
E. Restorative proctocolectomy with ileal pouch– CT scans emonstrate inammation in the
anal anastomosis sigmoi colon with fat straning. Subsequent
colonoscopy reveale iverticula throughout
34. The most common presentation for appeniceal the majority of the transverse, escening,
aenocarcinoma is: an sigmoi colon, but was negative for other
A. Palpable abominal mass pathology. Which of the following is the most
B. Acute appenicitis correct surgical intervention?
C. Ascites A. Total colectomy with ileoproctostomy
D. Inciental ning uring unrelate abominal B. Sigmoi colectomy with proximal margin
surgery at an area without any hypertrophy of the
E. Chronic anemia muscularis propria an istal margin where
the taenia splay out
35. The most common perianal lesion in Crohn’s C. Left colectomy with proximal margin where
isease is: there is cessation of iverticula an istal
A. Fissures margin where the taenia splay out
B. Skin tags D. Sigmoi colectomy with proximal margin
C. Perianal abscess at an area without any hypertrophy of the
D. Perianal stulas muscularis propria an istal margin at the
E. Hemorrhois rectosigmoi junction
E. Left colectomy with proximal margin where
36. A 10-year-ol boy presents with symptoms there is cessation of iverticula an istal
an signs suggestive of acute appenicitis. margin at the rectosigmoi junction
An ultrasoun shows enlarge hypoechoic
mesenteric lymph noes an an absence of 39. A hernia containing an appenix is known as:
a thickene or ilate blin-ening tubular A. Petit hernia
structure. Which of the following is true about B. Amyan hernia
this conition? C. Littre hernia
A. A iagnostic laparoscopy shoul be performe D. Spigelian hernia
B. This conition usually causes more peritoneal E. Grynfeltt hernia
irritation than appenicitis
C. The WBC count tens to be higher than with 40. A 35-year-ol man presents with a 1-ay history
appenicitis of anorexia, right lower quarant pain an
D. It occurs with equal frequency in aults an tenerness, an low-grae fever. At surgery,
chilren the appenix appears normal. However, both
E. It is usually associate with an anteceent the cecum an terminal ileum appear re an
upper respiratory tract infection iname. Management woul consist of:
A. Right hemicolectomy
B. Appenectomy
C. Close woun without further intervention
D. Biopsy of the cecal wall
E. Biopsy of the terminal ileum
CHAPtEr 10 Alimentary Tract—LargeIntestine 133

41. A 15-year-ol boy presents with a 5-ay history 43. Inciental appenectomy is BEST inicate in
of right lower quarant pain an a fever of 103°F. which of the following circumstances?
On examination, he has right lower an right A. During gastric bypass surgery in a 45-year-ol
upper quarant tenerness. Total bilirubin is man
3mg/L an alkaline phosphatase is 50 IU/L. B. During hysterectomy in a 30-year-ol woman
CT with contrast emonstrates multiple ensities C. During small bowel resection in a 30-year-ol
in the right lobe of the liver, a phlegmon in the woman with Crohn’s isease
right lower quarant, an straning aroun the D. During laparoscopic cholecystectomy in a
superior mesenteric vein with air bubbles within 5-year-ol woman
the vein. The clinical picture most likely represents: E. During a Whipple proceure in a 50-year-ol
A. Amebic liver abscess man
B. Pylephlebitis
C. Carcinoi synrome
D. Metastatic aenocarcinoma
E. Inammatory bowel isease (IBD)

42. The most common worlwie intestinal parasite


causing appenicitis is:
A. Enterobius vermicularis
B. Strongyloides stercoralis
C. Ascaris lumbricoides
D. Echinococcus granulosus
E. Clonorchis sinensis

Answers
1. C. This patient is presenting with a rst episoe of References: Sharma PV, Eglinton T, Hier P, Frizelle F. System-
uncomplicate iverticulitis, which is generally limite atic review an meta-analysis of the role of routine colonic evalu-
to pericolonic inammation an phlegmon. Mesalamine, ation after raiologically conrme acute iverticulitis. Ann Surg.
rifaximin, an probiotics are not typically recommene to 014;59():63–7.
Tehranian S, Klinge M, Saul M, Morris M, Diergaare B, Schoen
reuce risk of iverticulitis recurrence. However, patients
RE. Prevalence of colorectal cancer an avance aenoma in
with chronic smolering isease may notice improvement patients with acute iverticulitis: implications for follow-up colo-
in symptoms with these ajuncts. To rule out an associ- noscopy. Gastrointest Endosc. 00;91(3):634–640.
ate malignancy, the American Society of Colon an Rectal Hall J, Hariman K, Lee S, et al. The American Society of Colon
Surgeons recommens that all patients with complicate an Rectal Surgeons clinical practice guielines for the treatment of
iverticulitis shoul unergo colonoscopy at 6 to 8 weeks left-sie colonic iverticulitis. Dis Colon Rectum. 00;63(6):78–747.
after resolution of symptoms if colonoscopy has not been
performe in the past year. For uncomplicate iverticu- 2. A. This patient presents with a large bowel obstruction,
litis, this recommenation shoul be iniviualize to the most likely seconary to metastatic colon cancer given the
patient; however, most practitioners woul recommen imaging nings of bilateral lung noules an liver masses.
colonoscopy in those with high-risk nings on CT imaging The rst step in managing a large bowel obstruction is to rule
(mesenteric lymphaenopathy) or those with clinical recov- out a close-loop obstruction, which is surgical emergency.
ery that is atypical. The rate of colon cancer in the setting of Patients with a competent ileocecal valve an an obstruct-
iverticulitis ranges from 0.5% to .7% for uncomplicate ing colonic mass shoul be consiere to have a close-loop
isease, an as high as 11% for complicate iverticulitis. obstruction an shoul unergo urgent surgical ecompres-
In the setting of a single episoe of uncomplicate ivertic- sion (i.e., colostomy or ileostomy) or resection (C, D). Patients
ulitis that has fully resolve, sigmoi colectomy is not rec- with evience of small bowel ilation an no systemic signs
ommene (B, D). This patient’s symptoms appear to have of infection likely have an incompetent ileocecal valve an
resolve, so there is no inication for repeat imaging (E). In may be appropriate caniates for briging therapy which
the case of persistent or recurrent symptoms, chronic smol- can allow for proper staging an an elective oncologic
ering iverticulitis shoul be consiere in the ifferential resection. Colonic stenting has been increasingly utilize
iagnosis, as sigmoi resection is effective in resolution of to manage left-sie large bowel tumors presenting with
pain in this setting. obstruction. As this patient oes not have a tissue iagnosis,
134 PArt i Patient Care

colonoscopy an stent placement will serve both iagnostic colon cancer, but is useful in assessing persistence of isease
an therapeutic purposes. Uncovere stents are superior to following resection. Transrectal enoscopic ultrasoun (A) is
covere stents in the management of large bowel obstruction useful in locoregional staging for rectal cancer but not colon
as they are associate with fewer complications, lower rates cancer. PET/CT scans are not use routinely for staging of a
of stent migration, an longer uration of patency (B). Che- newly iagnose colon cancer (C–E) but be useful to assess
motherapy shoul not be initiate without tissue iagnosis rising CEA levels following initial surgical treatment.
an impening obstruction (E). References: National Comprehensive Cancer Network (NCCN).
Reference: Kaplan J, Strongin A, Aler DG, Siiqui AA. NCCN Guielines for Patients. Version .01: Colon Cancer.
Enteral stents for the management of malignant colorectal obstruc- Upate January 1, 01. Accesse March 7, 0. https://www.
tion. World J Gastroenterol. 014;0(37):1339–1345. nccn.org/patients/guielines/content/PDF/colon-patient.pf.
Norholm-Carstensen A, Wille-Jørgensen PA, Jorgensen LN,
3. B. The colon is responsible for both water an electrolyte Harling H. Ineterminate pulmonary noules at colorectal cancer
reabsorption. Water absorption averages 1 to  L per ay but staging: a systematic review of preictive parameters for malig-
nancy. Ann Surg Oncol. 013;0(1):40–4030.
can be as much as 5 L. However, the small intestine (mostly
jejunum) is where the majority of water absorption occurs
(A). Soium is absorbe actively via Na+,K+-ATPase with
6. E. The risk of the evelopment of colon cancer in
patients with UC increases with time. By 0 years, colon
water following passively. Chlorie is actively absorbe,
cancer will evelop in approximately 10% of patients. Thus,
not secrete, through a chlorie–bicarbonate exchange (D).
surveillance colonoscopy is recommene. Colon cancer
Bacteria fermentation in the colon prouces short-chain fatty
evelops in UC in the absence of polyps. In aition, areas
acis, which are a primary source of energy for colonocytes
of ysplasia may not be reaily apparent on stanar colo-
(E). Decreasing colonic pH (as occurs with lactulose) results
noscopy. As such, once a patient has ha UC for 8 years,
in a ecrease in ammonia reassertion (C).
colonoscopic surveillance is recommene annually there-
after. In aition to biopsies of areas of suspicion, ranom
4. D. For patients with peritoneal stuing from appen-
biopsies are recommene because at ysplasia evelops
iceal aenocarcinoma, cytoreuctive surgery with hyper-
in these patients. The ning of even high-grae ysplasia
thermic intraperitoneal chemotherapy (HIPEC) has shown
is an inication for surgery. Repeat colonoscopy woul be
promise in patients without evience of istant organ metas-
inappropriate (A). Some authors recommen surgery even
tasis. In a large series in which complete cytoreuction was
for low-grae at ysplasia because the risk of malignancy
ene as tumor noules less than .5 mm in iameter
is also signicantly increase; a recent metaanalysis emon-
remaining after surgery, patients with complete cytoreuc-
strate that these patients have a nine times increase risk
tion an aenomucinosis pathology ha a 5-year survival
of having colorectal cancer compare with patients who
rate of 86%. Incomplete cytoreuction ha a 5-year survival
are ysplasia free. Dysplasia in a at (nonpolypoi) lesion
rate of only 0%. Systemic or intraperitoneal chemotherapy
is concerning because it is more ifcult to monitor with
alone leas to lower survival rates (B, C, E). Offering no
follow-up screening. The curative operation is a restorative
treatment to a patient with peritoneal stuing seconary to
proctocolectomy with an ileal pouch–anal anastomosis
appeniceal aenocarcinoma woul not be appropriate (A).
(B–D). In aition to ysplasia, the inications for colec-
HIPEC is being use for colorectal, gastric, an ovarian can-
tomy in patients with UC inclue toxic megacolon, severe
cer, as well as intraperitoneal mesothelioma.
lower GI bleeing, an intractable isease that oes not
References: Jaffe BM, Berger DH. Appenix. In: Brunicari FC,
respon to meical management.
Anersen DK, Billiar TR, etal., es. Schwartz’s principles of surgery.
8th e. New York: McGraw-Hill; 005:1119–1138. References: Ullman T, Croog V, Harpaz N, Sachar D, Itzkowitz
Lally KP, Cox C, Anrassy RJ. Appenix. In: Townsen CM, Jr, S. Progression of at low-grae ysplasia to avance neoplasia in
Beauchamp RD, Evers BM, Mattox KL, es. Sabiston textbook of sur- patients with ulcerative colitis. Gastroenterology. 003;15(5):1311–1319.
gery: The biological basis of modern surgical practice. 17th e. Philael- Thomas T, Abrams KA, Robinson RJ, Mayberry JF. Meta-analysis:
phia: W.B. Sauners; 004:1381–1400. cancer risk of low-grae ysplasia in chronic ulcerative colitis. Ali-
Sugarbaker PH, Chang D. Results of treatment of 385 patients ment Pharmacol Ther. 007;5(6):657–668.
with peritoneal surface sprea of appeniceal malignancy. Ann Surg
Oncol. 1999;6(8):77–731. 7. B. Mucosal eema is the earliest ning on enoscopy.
Sugarbaker PH, Jablonski KA. Prognostic features of 51 colorectal As the isease avances, friable mucosa an ulcerations
an 130 appeniceal cancer patients with peritoneal carcinomatosis evelop (A). A “lea pipe” colon is a feature of longstan-
treate by cytoreuctive surgery an intraperitoneal chemotherapy. ing UC seen on barium enema an is the result of a loss of
Ann Surg. 1995;1():14–13. haustral markings an shortening of the colon. Although
crypt abscesses are almost always seen with UC, other
5. B. This patient is presenting with a rst-time iagnosis inammatory conitions of the colon can also present with
of colon cancer. Preoperative CT scan of the abomen an crypt abscesses (E). Finings on gross appearance in Crohn’s
pelvis to evaluate tumor extension, lymphatic sprea, an colitis that are not characteristic of UC inclue a thickene
metastases is recommene for all colon cancer patients. mesentery, thickene bowel wall, segmental isease, an
Routine staging with CT scan of the chest is frequently per- “creeping fat” or “fat wrapping.” On microscopic examina-
forme but controversial, as metastatic pulmonary isease tion, Crohn’s isease is transmural, whereas UC is limite to
is uncommon. In a systematic review of 5873 patients that the mucosa an submucosa. Noncaseating granulomas are a
unerwent preoperative staging chest CT, 9% ha an ine- hallmark feature of Crohn’s isease, whereas crypt abscesses
terminate pulmonary noule, of which only 10% were malig- are characteristic of UC. Plasmacytosis (increase in plasma
nant. Carcinoembryonic antigen has low iagnostic yiel for cells in lamina propria) can be foun in both UC an Crohn’s
CHAPtEr 10 Alimentary Tract—LargeIntestine 135

isease (C). Pseuopolyps are seen in both UC an Crohn’s with sporaic colorectal cancer patients. J Exp Clin Cancer Res. 008;
isease (D). 7(1):39.
Watson P, Lin KM, Roriguez-Bigas MA, et al. Colorectal carci-
noma survival among hereitary nonpolyposis colorectal carcinoma
8. D. It is important to remember that appeniceal perfora-
family members. Cancer. 1998;83():59–66.
tion is the most important variable in etermining fetal mor- van er Post RS, Kiemeney LA, Ligtenberg MJL, et al. Risk of
tality uring pregnancy; thus, it is imperative to make the urothelial blaer cancer in Lynch synrome is increase, in particu-
iagnosis early. Conversely, a general anesthetic increases the lar among MSH mutation carriers. J Med Genet. 010;47(7):464–470.
risk of premature labor. A recent large stuy was conucte
comparing appenicitis in more than 3000 pregnant women 10. B. Familial juvenile polyposis is an autosomal om-
with more than 94,000 nonpregnant women. The stuy inant (A) isorer (just like hereitary nonpolyposis colon
foun that the rate of negative appenectomy was higher in cancer [HNPCC], FAP, an Peutz-Jeghers synrome). It is a
pregnant women compare with nonpregnant women (3% completely ifferent entity from FAP. The polyps are hamar-
versus 18%). Rates of fetal loss an early elivery were con- tomas (also calle juvenile polyps), not aenomas. Hamarto-
sierably higher in women with complex appenicitis (6% mas are benign growths compose of histologically normal
an 11%, respectively) compare with negative (4% an an mature cells foun in abnormal locations an congu-
10%, respectively) an simple (% an 4%, respectively) rations. However, the hamartomas can egenerate into ae-
appenicitis. Complex appenicitis an a negative appen- nomas an malignancy, causing risk of colon cancer, but not
ectomy remaine risks for fetal loss on multivariate anal- to the same egree as with FAP. The lifetime risk is approxi-
ysis. Interestingly, laparoscopy was associate with a higher mately 10% to 38% (versus 100% for FAP) (C). Because of this
rate of fetal loss compare with open appenectomy (os risk an because many polyps occur on the right sie, colo-
ratio of .31) (B). Ultrasonography has been extremely useful noscopic (rather than exible sigmoioscopic) surveillance
in helping iagnose appenicitis. If nings are equivocal, is recommene, beginning at approximately 10 to 1 years
magnetic resonance imaging (MRI) shoul be performe. of age. Unlike FAP, in which the presence of polyps equates
One must strive to avoi unnecessary appenectomies that with a nee for a restorative proctocolectomy, if a polyp is
place the fetus at risk; however, elays in operative care for seen, it shoul be snare an sent to pathology (D). In most
appenicitis likewise place the fetus at risk. instances, it will be a hamartoma. If aenomatous changes
References: McGory ML, Zingmon DS, Tillou A, Hiatt JR, are seen, then a colectomy shoul be performe, an if the
Ko CY, Cryer HM. Negative appenectomy in pregnant women rectum is spare, an ileorectal anastomosis can be one
is associate with a substantial risk of fetal loss. J Am Coll Surg. with close surveillance. Approximately 15% to 0% of Peu-
007;05(4):534–540.
tz-Jeghers synrome patients evelop stomach or uoenal
Lim HK, Bae SH, Seo GS. Diagnosis of acute appenicitis in preg-
cancers, so upper enoscopic surveillance is recommene
nant women: value of sonography. AJR Am J Roentgenol. 199;159(3):
539–54. by age 5 (E).
References: Dunlop MG, British Society for Gastroenterology,
Association of Coloproctology for Great Britain an Irelan. Gui-
9. D. Stage for stage, colonic malignancy in Lynch syn- ance on gastrointestinal surveillance for hereitary non-polyposis
rome has the same prognosis as sporaic cancer. In a small colorectal cancer, familial aenomatous polypolis, juvenile polyp-
number of sporaic colon cancers, microsatellite instability osis, an Peutz-Jeghers synrome. Gut. 00;51(Suppl 5):V1–V7.
an inappropriate DNA methylation leas to impaire DNA Howe JR, Mitros FA, Summers RW. The risk of gastrointestinal
mismatch repair, increasing the risk for eveloping colon can- carcinoma in familial juvenile polyposis. Ann Surg Oncol. 1998;5(8):
cer. Lynch synrome (or HNPCC) arises because of errors in 751–756.
the mismatch repair genes that coe for the DNA mismatch
repair enzymes. It is an autosomal ominant synrome with 11. B. Sessile serrate polyps often harbor ysplasia or car-
an increase risk of colorectal carcinoma an other malig- cinoma in situ an shoul be resecte. Given their at mor-
nancies, with a lifetime risk of approximately 80% for colon phology, they are at risk of not being etecte or unergoing
cancer, 0% for gastric cancer, an a high risk of enometrial incomplete removal an may contribute to interval cancer
an upper genitourinary tract cancer (A, B). The colon can- evelopment. Patients with serrate polyposis synrome
cers are more commonly right sie (as oppose to left sie (previously known as hyperplastic polyposis synrome)
in sporaic cancer); as such, screening requires colonoscopy, can present with upwar of 100 polyps in the colon, some
which is recommene either at age 5 or 10 years less than of which are premalignant. Patients with this synrome are
the age at which colon cancer evelope in other family consiere to be at increase risk for colon cancer. Aeno-
members (whichever is earlier) (C). Patients with FAP shoul matous polyps are consiere neoplastic an are ivie
begin screening much earlier (age 10–1). Upper enoscopy into three types: tubular (<5% risk of malignancy), tubulo-
screening is also recommene starting at age 50. The moi- villous (0% risk of malignancy), an villous (40% risk of
e Amsteram criteria for clinical iagnosis of HNPCC can malignancy) (A). Polyp size is also an important etermi-
be remembere by the 3--1-1 rule: 3 or more relatives with nant, with polyps smaller than 1 cm having an extremely
histologically verie cancers in the colon, enometrium low risk of malignancy vs a nearly 50% risk of malignancy
small intestine, or pelvis;  or more successive generations in polyps larger than  cm. The location of a polyp oes not
affecte; 1 or more relatives iagnose before age 50 (E); an affect the risk of malignancy (E). Most colon cancers evelop
1 shoul be a rst-egree relative of the other two. Aition- from aenomatous polyps. Peutz-Jeghers synrome is char-
ally, FAP must be rule out to iagnose Lynch synrome. acterize by hamartomatous polyps (C). They present with
References: Stigliano V, Assisi D, Cosimelli M, et al. Survival GI bleeing an intussusception. Although hamartomatous
of hereitary non-polyposis colorectal cancer patients compare polyps are not consiere premalignant, they can egenerate
136 PArt i Patient Care

into aenomatous polyps, so there is a risk of malignancy. 13. B. This patient has a sigmoi volvulus. The common
The polyps in Peutz-Jeghers synrome occur primarily in enominator in sigmoi volvulus is a large, reunant colon,
the small intestine, but they can also occur in the colon an which is frequently associate with chronic constipation.
rectum. Patients have melanin spots on the buccal mucosa Iniviuals with chronic constipation (elerly or institution-
an lips. Because of the iffuse nature of polyps throughout alize), a high-ber iet (leas to an elongate an reun-
the GI tract, surgery is only inicate if there is evience of ant colon), or megacolon (Chagas isease) are preispose.
obstruction or bleeing or evience that a polyp has uner- Patients present with symptoms an signs of an acute large
gone aenomatous change. Inammatory polyps or pseuo- bowel obstruction. The important issues are the following:
polyps are islans of regenerating mucosa seen most often in 1. Establishing the correct iagnosis. This can generally be
IBD or after mucosal injury (D). one by classic raiographic nings of a markely ilate
References: Correa P, Strong JP, Reif A, Johnson WD. The epie- colon with a “bent inner tube” appearance or an omega
miology of colorectal polyps: prevalence in New Orleans an inter- sign.
national comparisons. Cancer. 1977;39(5):58–64. 2. Determining whether the patient alreay has an ischemic
Hyman NH, Anerson P, Blasyk H. Hyperplastic polyposis or ea bowel. This can be achieve via evience
an the risk of colorectal cancer. Dis Colon Rectum. 004;47(1):
of systemic toxicity (laboratory tests) an physical
101–104.
examination (peritonitis), an if these are present, the
patient nees a laparotomy an sigmoi colectomy (D,
12. C. Ischemic colitis occurs primarily in elerly patients E). Sometimes this is also seen at the time of enoscopy.
at an average age of 70 years an may present with lower GI 3. Unerstaning the value of enoscopic etorsion.
bleeing. Fecal immunochemical test has replace the oler This can be performe with either a rigi
fecal occult guaiac bloo test because it has been shown proctosigmoioscope or a exible enoscope.
to have superior aherence, usability, accuracy, sensitivity, 4. Being aware that there is a high recurrence rate (as high
an better etection of occult bleeing. Unlike acute small as 40%). Thus, after etorsion, a recommenation shoul
bowel ischemia, which evelops in association with mesen- be mae for a subsequent colectomy (A, C).
teric arterial or venous occlusive isease, colonic ischemia 5. Distinguishing it from cecal volvulus, which cannot
is rarely the result of major vascular occlusion. Rather, it usually be enoscopically etorse an requires surgery
usually occurs as a result of a low-ow state such as severe (right hemicolectomy).
ehyration. As such, mesenteric angiography is typically Reference: Chung YF, Eu KW, Nyam DC, Leong AF, Ho YH,
not helpful (E). It tens to evelop in watershe areas of Seow-Choen F. Minimizing recurrence after sigmoi volvulus:
bloo supply, such as the splenic exure (most common), Recurrence after sigmoi volvulus. Br J Surg. 1999;86():31–33.
known as Grifth’s point, where collaterals are present
between the superior mesenteric artery an inferior mes- 14. E. Cecal or cecocolic volvulus is much less common
enteric artery (specically, the mile colic artery an than sigmoi volvulus. It occurs in younger patients. There
the ascening branch of the left colic artery, respectively); are two types: axial ileocolic volvulus (90%) an cecal bascule
Sueck’s critical point (rectosigmoi junction), where col- (10%). In the former, the cecum rotates up an over to the left
laterals are present between the sigmoi artery an superior upper quarant. Cecal bascule occurs when the cecum ips
rectal artery; an the ileocecal area. In aition to avance upwar an anterior in a horizontal plane. It is thought to
age, risk factors for ischemic colitis inclue unerlying car- be ue to a congenital anomaly leaing to a lack of xation
iovascular isease, iabetes, vasculitis, an hypotension. of the cecum to the retroperitoneum, an as such, the termi-
Most cases are mil an result in painless, blooy iarrhea. nal ileum, cecum, an ascening colon twist an can become
More severe cases can result in bacterial translocation with ischemic. It can sometimes be har to iagnose raiographi-
fever an leukocytosis or, rarely, full-thickness necrosis cally because the patient will often also emonstrate ilate
with peritonitis. The iagnosis is via a combination of the loops of small bowel with air–ui levels, giving the appear-
history an examination, plain lms to rule out an acute ance of a small bowel obstruction. Unlike sigmoi volvulus,
abomen an that sometimes will show signs of mucosal enoscopic ecompression for cecal volvulus is very ifcult
eema (thumb printing), an CT scan (nonspecic colonic (D). The treatment of choice is to perform a right colectomy
wall eema an fat straning). The surgeon nees to be with primary anastomosis; this is feasible espite no bowel
aware that the ifferential iagnosis inclues colon can- preparation. There is a high recurrence rate after operative
cer an IBD. As such, colonoscopy shoul eventually be etorsion an cecopexy (B). If the right colon is alreay gan-
performe, although it is not necessary in the acute phase grenous (e.g., leukocytosis, aciosis, high fever, peritonitis),
(D). The exception is ischemic colitis after aortic surgery, in right hemicolectomy with ileostomy an mucus stula is the
which case the enoscopy assists in the iagnosis, an CT treatment of choice (C). However, this patient’s presentation
scan nings may be har to interpret ue to postsurgical makes this ning unlikely. Cecostomy tubes are inappropri-
changes. Most patients are treate meically with bowel ate in a patient who is young an otherwise a goo surgical
rest an broa-spectrum antibiotics. Surgery is reserve for caniate who woul benet from enitive treatment (A).
patients who eteriorate an/or have evience of iffuse Reference: Habre J, Sautot-Vial N, Marcotte C, Benchimol D.
peritonitis or enoscopy in whom shows necrosis (A, B). Caecal volvulus. Am J Surg. 008;196(5):e48–e49.
References: Balthazar EJ, Yen BC, Goron RB. Ischemic colitis:
CT evaluation of 54 cases. Radiology. 1999;11():381–388. 15. D. Sigmoi iverticular isease is thought to occur ue
Schreuers EH, Grobbee EJ, Spaaner MCW, Kuipers EJ. to a low-ber iet leaing to constipation. A long, reun-
Avances in fecal tests for colorectal cancer screening. Curr Treat ant colon, foun in populations that have a high-ber
Options Gastroenterol. 016;14(1):15–16. iet, increases the risk of eveloping volvulus (E). Sigmoi
CHAPtEr 10 Alimentary Tract—LargeIntestine 137

iverticula are consiere false because they are compose of References: Hiranyakas A, Ho YH. Laparoscopic parastomal
only mucosa an submucosa. They occur at points of weak- hernia repair. Dis Colon Rectum. 010;53(9):1334–1336.
ness between the taeniae coli, where bloo vessels penetrate Kann BR. Early stomal complications. Clin Colon Rectal Surg.
the colonic wall. They occur most commonly in the sigmoi 008;1(1):3–30.
Tam KW, Wei PL, Kuo LJ, Wu CH. Systematic review of the use
colon on the mesenteric sie of the antimesenteric taenia
of a mesh to prevent parastomal hernia. World J Surg. 010;34(11):
(A). They occur ue to increase intraluminal pressure, so
73–79.
they are consiere pulsion iverticula. Because the taeniae Vierimaa M, Klintrup K, Biancari F, et al. Prospective, ranomize
splay out at the rectum, iverticula o not evelop in the stuy on the use of a prosthetic mesh for prevention of parastomal her-
rectum (B). Asymptomatic iverticula (sigmoi or cecal) o nia of permanent colostomy. Dis Colon Rectum. 015;58(10):943–949.
not require surgical management (C). Patients eeme sur-
gical caniates for sigmoi resection are best serve with 18. D. Ischemia or necrosis of the stoma is a recognize
a mechanical bowel preparation with IV an oral periopera- complication of a colostomy creation. It is more likely to
tive antibiotics. This ecreases the rate of surgical site infec- occur in situations in which the inferior mesenteric artery
tion by more than 50%. was ligate high, near the aorta, such that the stoma is rely-
Reference: Kim EK, Sheetz KH, Bonn J, et al. A statewie col- ing on the marginal artery. It is important to evaluate the
ectomy experience: the role of full bowel preparation in preventing extent of ischemia before proceeing irectly to the operating
surgical site infection. Ann Surg. 014;59():310–314. room. This is can be accomplishe by placing a clear test tube
into the ostomy an using a penlight to evaluate the mucosa
16. D. Pneumoperitoneum in a symptomatic patient almost own to the level of the fascia or via enoscopy. If the isch-
always necessitates emergency surgery an is often ue to emia is evient own to the level of the fascia, the patient
visceral perforation (A, B). However, colonoscopy (less so nees reexploration an revision; otherwise it may progress
with iagnostic versus therapeutic) can lea to benign pneu- to full-thickness necrosis, perforation, an stool spillage into
moperitoneum an is believe to be ue to microperforation the peritoneum. The type of operation will epen on the
an/or the transmural passage of air seconary to insufa- extent of ischemia (A–C). In contrast, if the ischemia is super-
tion. Patients with benign pneumoperitoneum, no abomi- cial, it can be observe, an a mucocutaneous junction will
nal pain, an no systemic signs of sepsis (fever, leukocytosis) form by seconary intention. This may lea to recession of
can be treate with IV antibiotics an bowel rest. Due to the the stoma or stricture but can be ealt with later when the
scarcity of benign pneumoperitoneum in the literature, these patient recovers from surgery. In aition, it may be techni-
recommenations are base on several case reports an one cally ifcult, if not impossible, to gain aitional length in a
prospective stuy. Serial abominal exam shoul also be per- morbily obese patient to refashion the stoma; thus, a return
forme but in aition to amission, antibiotics an bowel to the OR shoul be avoie if it can be one safely. There is
rest (C). The patient shoul not be ischarge home if there no role for IV antibiotics (E).
is concern for a perforate viscus (E). Reference: Kim JT, Kumar RR. Reoperation for stoma-relate
Reference: Pearl JP, McNally MP, Elster EA, DeNobile JW. complications. Clin Colon Rectal Surg. 006;19(4):07–1.
Benign pneumoperitoneum after colonoscopy: a prospective pilot
stuy. Mil Med. 006;171(7):648–649. 19. B. This patient has a stercoral ulceration complicate
by perforation. This is a rare conition occurring primarily
17. D. Parastomal hernias are a relatively common compli- in elerly patients suffering from chronic constipation. It is
cation of stoma creation with an estimate occurrence rate as thought that a har fecaloma leas to local ischemia, ulcer
high as 50%, with en colostomy having the highest risk an formation, an subsequent perforation. The antimesenteric
loop ileostomy having the lowest risk (E). Other risk factors borer of the rectosigmoi colon is the most likely location
inclue oler age, woun infection, obesity, malnutrition, owing to its unique characteristics incluing lower water
immunosuppression, IBD, an COPD (A). While parastomal content, poorer blooy supply, an higher pressure secon-
hernia can be asymptomatic, the estimate reoperation rate ary to a narrowe intraluminal iameter (A–E). The iagno-
for patients with this conition is somewhere aroun 30%. sis is suggeste with the following four criteria: (1) a roun
A stuy publishe in the Worl Journal of Surgery in 010 antimesenteric colonic perforation >1 cm in iameter; ()
emonstrate a reuction of parastomal hernia rate from colon full of stool protruing through the perforation site; (3)
55% to 7.8% with the placement of mesh uring the inex evience of multiple pressure ulcers an acute inammation
operation. Another stuy showe reuction of clinically sig- aroun the perforation; an (4) absence of external injury,
nicant parastomal hernia but no ifference in CT etectable iverticulitis, or obstruction ue to neoplasms or ahesions.
parastomal hernias with the use of mesh. Prophylactic mesh Patients most commonly present with iffuse abominal
placement is being use more commonly as more stuies pain an fever. The iagnosis is selom mae before sur-
suggest improve outcomes. Patients with asymptomatic gery. Since inammation an ulcer formation exten beyon
parastomal hernia shoul be manage with a support evice the immeiate bowel surrouning the perforation, a simple
such as a hernia belt an weight loss. However, this patient closure or limite colonic resection shoul be avoie (C).
is complaining of pain an ifculty applying his colostomy Thus, a formal colon resection with proximal colostomy
bag, which are both inications for repair or relocation of the (Hartmann proceure) is recommene. Stercoral ulcer per-
stoma (B). Prosthetic mesh repair is consiere the preferre foration has a high mortality rate. Chronic constipation is a
surgical approach because relocating the stoma is associate common problem affecting many people, but stercoral ulcer-
with the same high risk of hernia formation as the initial ation is rare; this suggests there may be aitional preispos-
stoma (C). A common repair is the Sugarbaker technique, in ing factors contributing to this entity. Several reports have
which mesh is place aroun the ostomy as a ap valve. shown an association of NSAID use with the evelopment
138 PArt i Patient Care

of stercoral perforation. Aitionally, anticholinergic agents evience of ischemic or perforate colon, then colectomy
will worsen chronic constipation an contribute to this com- can be avoie (E). In this scenario, a right transverse or left
plication (D). lower quarant sigmoi colostomy tube is often use with
References: Huang WS, Wang CS, Hsieh CC, Lin PY, Chin a success rate of 95%. This functions as a “blow-hole” colos-
CC, Wang JY. Management of patients with stercoral perforation tomy at one or several points in the bowel an is more effec-
of the sigmoi colon: report of ve cases. World J Gastroenterol. tive than a formal colostomy at proviing pressure relief. A
006;1(3):500–503. less effective option inclues the transanal insertion of a large
Mauer CA, Renzulli P, Mazzucchelli L. Use of the accurate iag-
multiperforate tube (Faucher tube) guie to the proximal
nostic criteria may increase incience of stercoral perforation of the
ege of ilate colon by the surgeon’s hans (D). Next, the
colon. Dis Colon Rectum. 000;43(7):991–998.
Patel VG, Kalakuntla V, Fortson JK, Weaver WL, Joel MD, Ham-
surgeon performs manual compressive maneuvers to milk
mami A. Stercoral perforation of the sigmoi colon: report of a rare the colonic content towar the tube. If there is any concern of
case an its possible association with nonsteroial antiinammatory compromise bowel, a total or subtotal colectomy shoul be
rugs. Am Surg. 00;68(1):6–64. performe. Primary anastomosis shoul be avoie because
Serpell JW, Nicholls RJ. Stercoral perforation of the colon. Br J this has a high leak rate (A, B). This is a highly morbi proce-
Surg. 1990;77(1):135–139. ure, with mortality estimate to be up to 40%.
References: Caves PK, Crockar HA. Pseuo-obstruction of the
20. B. This patient has acute colonic pseuoobstruction large bowel. Br Med J. 1970;(5709):583–586.
or Ogilvie synrome. It often occurs in critically ill patients Vanek VW, Al-Salti M. Acute pseuo-obstruction of the colon
without any signs of mechanical obstruction. The pathophys- (Ogilvie’s synrome): An analysis of 400 cases. Dis Colon Rectum.
iology is not completely unerstoo, is likely multifactorial, 1986;9(3):03–10.
an is thought to occur seconary to paralysis of the bowel Vogel JD, Feingol DL, Stewart DB, et al. Clinical practice guie-
lines for colon volvulus an acute colonic pseuo-obstruction. Dis
allowing for passive istention. Stable patients without
Colon Rectum. 016;59(7):589–600.
any systemic signs of compromise bowel shoul initially
unergo conservative management with bowel rest, nasoga-
stric tube suction, ecompressive rectal tube, an electrolyte
22. B. PSC is a progressive an estructive isease of the
entire biliary tree seconary to an inammatory process. It is
repletion. Neostigmine is a reversible cholinesterase inhibi-
estimate that up to 80% of patients with PSC have IBD, with
tor that has been emonstrate in ranomize controlle tri-
UC being most common. It has been emonstrate that PSC
als to have an improve response over placebo (reuction in
signicantly increases the risk of colorectal cancer in these
cecum iameter of 5 cm compare to  cm). Up to 80% to 90%
patients. UC patients shoul typically unergo screening
of patients have a favorable response to a single IV injection
colonoscopy with ranom biopsies starting 8 years from the
of  mg neostigmine (D). For those who o not respon, a
time of their IBD iagnosis (C). However, patients iagnose
secon an thir aministration can be given. Alternatively,
with PSC shoul receive a colonoscopy with ranom biopsies
a continuous infusion of neostigmine at a rate of 0.4 to 0.8
promptly at the time of PSC iagnosis an continue every 1
mg/hour over 4 hours can be given an has been shown
to  years thereafter. Patients without IBD or family history
to have successful results (C). Contrainications to the use
of colorectal cancer shoul begin screening at age 50 or after
of neostigmine inclue acute urinary retention, acute coro-
presenting with worrisome symptoms (D, E). Patients with
nary artery synrome, asthma, bronchospasm, an secon-
family history of colorectal cancer shoul begin screening 10
or thir-egree heart block (A). All patients being given
years before the age of iagnosis of any rst-egree relative
neostigmine shoul be place on cariac monitoring, an a
with colorectal cancer.
syringe prelle with atropine shoul be place at besie
References: Razumilava N, Gores GJ, Linor KD. Cancer sur-
an reay for immeiate use. Enoscopic etorsion with or
veillance in patients with primary sclerosing cholangitis. Hepatology.
without placement of a ecompression tube is a viable option 011;54(5):184–185.
in the absence of perforation, bowel ischemia, or peritonitis, Zheng HH, Jiang XL. Increase risk of colorectal neoplasia in
particularly for those who o not respon to neostigmine patients with primary sclerosing cholangitis an inammatory
an can ai in ruling out a mechanical obstruction. bowel isease: a meta-analysis of 16 observational stuies. Eur J Gas-
References: Ponec RJ, Sauners MD, Kimmey MB. Neostigmine troenterol Hepatol. 016;8(4):383–390.
for the treatment of acute colonic pseuo-obstruction. N Engl J Med.
1999;341(3):137–141. 23. A. The most common cause of lower GI bleeing, iver-
Van Der Spoel J, Ouemans-Van Straaten HM, Stoutenbeek CP.
ticulosis, accounts for more than one-half of cases. Rarely,
American Society for Gastrointestinal Enoscopy guieline on the
massive lower GI bleeing can be the result of an upper GI
role of enoscopy in the management of acute colonic pseuo-
obstruction an colonic volvulus. Gastrointest Endosc. 001;91():
source. Placing a nasogastric tube an aspirating for bloo
8–35. are important rst steps after resuscitation. Likewise, hem-
Vogel JD, Feingol DL, Stewart DB, et al. Clinical practice guie- orrhois can rarely be the cause an can easily be rule out
lines for colon volvulus an acute colonic pseuo-obstruction. Dis with anoscopy while the patient is stabilize. The manage-
Colon Rectum. 016;59(7):589–600. ment algorithm epens on the patient’s response to initial
resuscitation. If the patient stabilizes, she shoul unergo
21. C. If conservative therapy fails in the management of colonoscopy as soon as a bowel preparation is complete
Ogilvie synrome or if there is any concern for compro- (A). If the patient continues to blee, the next step is to per-
mise bowel, surgery shoul be consiere. The three sur- form either mesenteric arteriography or a tagge re bloo
gical options inclue tube colostomy, transanal insertion of cell scan (nuclear scintigraphy) using technetium-99m (B,
a long multiperforate rainage tube, an total or subtotal C). Arteriography can be both iagnostic an therapeutic
colectomy with an ostomy. If uring laparotomy there is no (angioembolization). However, it is invasive an bleeing
CHAPtEr 10 Alimentary Tract—LargeIntestine 139

must be brisk (0.5–1 mL/min). It is also not as feasible to can wait until age 50 for screening because they are consi-
repeat the stuy in the case of a patient that stoppe blee- ere to have the same risk as the normal population. Aeno-
ing an reblees. Nuclear scanning etects bleeing at a mas can evelop throughout the gastrointestinal (GI) tract
much slower rate (only 0.1 mL/min), an since the raio- in FAP patients, an in particular in the uoenum, an
active agent remains labele on the re bloo cell for some patients are at risk of the evelopment of periampullary car-
time, repeat images can be obtaine for up to 4 hours. If cinoma. Therefore, upper enoscopy for surveillance every
the patient cannot be stabilize an the source is not iscov- 1 to 3 years starting at age 5 to 30 years shoul be recom-
ere, the patient shoul be taken to the operating room for mene. Prophylactic proctocolectomy oes not ecrease the
an exploratory laparotomy with intraoperative enoscopy or risk of eveloping periampullary carcinoma, an it remains
colonoscopy (D–E). If the source cannot be localize, a total a common cause of morbiity in this patient population (D).
colectomy shoul be performe. Once the iagnosis of FAP has been mae an polyps are
Reference: Farner R, Lichliter W, Kuhn J, Fisher T. Total colec- eveloping, treatment is surgical. FAP may also be associ-
tomy versus limite colonic resection for acute lower gastrointesti- ate with extraintestinal manifestations such as congenital
nal bleeing. Am J Surg. 1999;178(6):587–591. hypertrophy of the retinal pigment epithelium, esmoi
tumors, epiermoi cysts, manibular osteomas, an central
24. E. Pouchitis is a nonspecic inammation of the ileal nervous system tumors (B). In a small number of sporaic
reservoir that can occur after an ileoanal pouch creation or colon cancers, microsatellite instability leas to impaire
in a continent ileostomy reservoir. Pouchitis can be acute or DNA mismatch repair an thus an inability to ensure the
can become chronic. Symptoms inclue increase iarrhea, elity of a copie DNA stran, increasing the risk for evel-
hematochezia, abominal pain, fever, an malaise. The oping cancer (A).
iagnosis is establishe via a combination of the history,
enoscopic nings, an histology from biopsy samples. 26. A. In the neutropenic patient with leukemia who
Enoscopy with biopsy is important to rule out uniag- presents with acute abominal pain, one must suspect
nose Crohn’s isease (A). It is the most common long-term neutropenic enterocolitis, which is commonly referre to
complication of this proceure, with an incience as high as typhlitis. The typical patient presents with abominal
as 30% to 55% (B). Clostridium difcile-associate pouchitis pain an tenerness, fever, an iarrhea in association with
shoul be rule out. The cause is unknown; it may be ue severe neutropenia (ene as an absolute neutrophil count
to fecal stasis within the pouch, but emptying stuies o not <1000 cells/μL). A CT scan is helpful in ruling out perfo-
conrm this. A recent Cochrane stuy showe that cipro- ration an in the case of typhlitis will show thickening of
oxacin is more effective than metroniazole for inucing the cecal wall with pericolic straning. Some reports have
remission of acute pouchitis. Most patients will respon also shown the utility of ultrasonography in establishing
rapily to either oral preparations or enemas. Patients with the iagnosis via the emonstration of cecal thickening. The
chronic pouchitis may require ongoing suppressive anti- majority of patients respon to bowel rest an IV antibiot-
biotic therapy. Salicylate an stool enemas have also been ics. The mortality rate in chilren in contemporary series
use with some success. Reintrouction of normal ora by is 8% to 10%. Surgery shoul be reserve for patients with
probiotics has been shown to be useful in chronic cases (C). signs of perforation, although the nee for surgical inter-
Rarely, the pouch requires excision, but this woul not be vention is low (B–E).
one urgently (D). References: Schlatter M, Snyer K, Freyer D. Successful non-
References: Gionchetti P, Amaini C, Rizzello F, Venturi A, Pog- operative management of typhlitis in peiatric oncology patients.
gioli G, Campieri M. Diagnosis an treatment of pouchitis. Best Pract J Pediatr Surg. 00;37(8):1151–1155.
Res Clin Gastroenterol. 003;17(1):75–87. Sloas MM, Flynn PM, Kaste SC, Patrick CC. Typhlitis in chilren
Holubar SD, Cima RR, Sanborn WJ, Pari DS. Treatment an with cancer: a 30-year experience. Clin Infect Dis. 1993;17(3):484–490.
prevention of pouchitis after ileal pouch-anal anastomosis for chronic
ulcerative colitis. Cochrane Database Syst Rev. 010;6:CD001176.
Maiba TE, Bartolo DC. Pouchitis following restorative procto- 27. E. Patients who present with a protracte history consis-
colectomy for ulcerative colitis: incience an therapeutic outcome. tent with acute appenicitis an a palpable mass are likely to
J R Coll Surg Edinb. 001;46(6):334–337. have a perforate an walle-off abscess. They are best man-
Shen B, Achkar JP, Lashner BA, et al. A ranomize clinical trial age by nonoperative therapy (IV antibiotics, bowel rest).
of ciprooxacin an metroniazole to treat acute pouchitis. Inamm Several large stuies have shown a low recurrence rate in
Bowel Dis. 001;7(4):301–305. patients that unergo nonoperative management, so the par-
aigm in acute care surgery has now shifte such that inter-
25. E. FAP is a rare autosomal ominant isease that val appenectomy is not performe in most patients with
accounts for approximately 1% of colon cancers. It is ue to a perforate appenicitis (B). Taking such a patient to the
a mutation in the aenomatous polyposis coli (APC) tumor operating room for an open or laparoscopic appenectomy
suppressor gene on chromosome 5q. Synromes that are is acceptable (C, D). However, the intense inammation an
consiere variants of FAP inclue attenuate FAP (elaye scarring will make the operation ifcult an signicantly
polyp growth), Turcot synrome, an Garner synrome. If increase the chances of having to perform an ileocecectomy
unrecognize or untreate, cancer can evelop in all patients (E). Aitionally, routine CT-guie rainage of abscesses is
by age 35 to 40 years; in fact, polyps often begin at puberty. not recommene particularly when the abscess is small (A).
They eventually can evelop thousans of polyps. As such, Reference: Kaminski A, Liu ILA, Applebaum H, Lee SL,
rst–egree relatives of FAP patients who are APC positive Haigh PI. Routine interval appenectomy is not justie after
shoul begin screening at age 10 to 1 years by exible sig- initial nonoperative treatment of acute appenicitis. Arch Surg.
moioscopy (C). Relatives who are APC mutation negative 005;140(9):897–901.
140 PArt i Patient Care

28. D. In etermining management of this case, one must than 1 cm rarely exten outsie of the appenix an are
consier the inications for the colonoscopy, the timing of treate simply by appenectomy. A right colectomy is ini-
the perforation, an the intraoperative nings. Because the cate for tumors larger than 1 cm with extension into the
polyp is peunculate an benign appearing, one can pre- mesoappenix or the base an for those that are larger than
sume that it has been completely remove an that further  cm an locate at the tip (A). In contrast, aenocarcinoma
colon resection is not neee. The vast majority of colonic of any size an at any location in the appenix is treate with
injuries, whether iatrogenic or from penetrating trauma, a right colectomy. Appeniceal carcinois rarely cause carci-
can be repaire primarily. Furthermore, this patient has noi synrome because wiesprea liver metastases are rare
presumably unergone a bowel prep, so the bacterial loa an there is no relation to tumor size an the evelopment of
is ecrease. Resection with colostomy (A–C) woul be carcinoi synrome (B). There is no role for raiation or che-
reserve for patients with longstaning perforation an motherapy for appeniceal carcinoi (C). In one large series,
iffuse fecal contamination. Conservative management (E) the overall 5-year survival rate for localize lesions was 94%,
woul be inappropriate for a patient with an iatrogenic an 84.6% for regional invasion, an 33.7% for istant metasta-
symptomatic colonic perforation. The approach use for ses. In approximately 15% of patients, noncarcinoi tumors
polypectomy shoul be consiere by the surgeon an may at other sites were also evient.
impact surgical approach. If electrocautery was require References: Jaffe BM, Berger DH. Appenix. In: Brunicari FC,
uring the initial polypectomy, unrecognize thermal injury Anersen DK, Billiar TR, etal., es. Schwartz’s principles of surgery.
may lea to failure of primary repair. 8th e. New York: McGraw-Hill; 005:1119–1138.
Sanor A, Molin IM. A retrospective analysis of 1570 appeni-
29. E. The patient has a cecal stula. The most common ceal carcinois. Am J Gastroenterol. 1998;93(3):4–48.
Stinner B, Kisker O, Zielke A, Rothmun M. Surgical manage-
causes are slippage of the suture or necrosis of the remaining
ment for carcinoi tumors of small bowel, appenix, colon, an rec-
appeniceal stump. Colocutaneous stulas, being low-out-
tum. World J Surg. 1996;0():183–188.
put stulas, are not associate with losses of large amounts
of ui, electrolytes, or nutrients. Therefore, total parenteral 32. D. Diversion colitis can occur after fecal iversion.
nutrition is not necessary to maintain aequate nutrition (A). When the fecal stream is iverte, colonocytes are not
Spontaneous closure is the rule in the majority of patients. expose to intraluminal nutrients an the eciency of
Cross-sectional imaging shoul be performe to rule out an these compouns can lea to mucosal atrophy an subse-
intraabominal abscess or ui, as intraabominal sepsis quent inammatory colitis. Short-chain fatty acis (SCFAs)
shoul be aresse. Surgery is not an appropriate initial (acetate, butyrate, an propionate) are prouce by bacte-
management option (C, D). Patients can be fe a low-resiue rial fermentation of ietary carbohyrates such as lactulose.
iet because absorption is mostly complete by the time the SCFAs are an important source of energy for the colonic
contents reach the cecum. Octreotie oes not help in assist- mucosa, an their use is consiere the rst-line treatment
ing closure of a cecal stula (B). If the stula fails to close, (as rectal enema) for iversion colitis. The energy is use by
one must suspect the possibility of either a neoplasm in the colonocytes for processes such as active transport of soium.
cecum, IBD, tuberculosis, or istal obstruction. Ketone boies, glucose, or amino acis (glutamine) are not
Reference: Hale DA, Molloy M, Pearl RH, Schutt DC, use as an energy source of colonocytes (A–C, E).
Jaques DP. Appenectomy: a contemporary appraisal. Ann Surg.
Reference: Harig JM, Soergel KH, Komorowski RA, Woo CM.
1997;5(3):5–61.
Treatment of iversion colitis with short-chain-fatty aci irrigation.
N Engl J Med. 1989;30(1):3–8.
30. D. Current guielines inicate that stage I (noe nega-
tive, invaes submucosa) colon cancer oes not nee chemo- 33. E. In a patient who tests positive for the APC gene,
therapy. The role of chemotherapy in stage II (noe negative, screening via sigmoioscopy is recommene starting at age
invaes subserosa or irect invasion of ajacent organ) colon 10 to 1 years. Once polyps are etecte, the recommenation
cancer remains ebatable. The combination of 5-uoroura- is to remove the entire colon an rectum (A). Cyclooxygen-
cil an leucovorin prolongs survival in stage III colon cancer ase- inhibitors were shown to slow the growth of polyps in
(positive lymph noes, no istant metastasis) but not stage patients with FAP in a ranomize stuy, but recent stuies
IV (A). Until recently, there was no effective chemotherapy inicate that these rugs increase the risk of eath from
for stage IV cancers. Two recent rugs have been approve cariovascular events (B). The best option is a restorative
for stage IV colon cancer. They have been shown to prolong proctocolectomy with an ileal pouch–anal anastomosis. Total
life but not cure this avance-stage cancer an are very abominal colectomy with ileorectal anastomosis is another
costly (C). Cetuximab (Erbitux) is a monoclonal antiboy option, but it requires careful lifelong surveillance of the rec-
that targets epiermal growth factor receptor. Bevacizumab tal mucosa for polyps (C). Total proctocolectomy with conti-
(Avastin) is a monoclonal antiboy against vascular eno- nent ileostomy may be another option. If possible, avoiing
thelial growth factor A (E). Raiation therapy is not com- an ostomy shoul be consiere in a young patient (D).
monly use in the management of colon cancer but is use
commonly in combination with chemotherapy for patients 34. B. Primary aenocarcinoma of the appenix presents
with rectal cancer (B). most commonly as acute appenicitis. For this reason, it is
always important to check the nal pathology of the appen-
31. D. Most carcinois are foun at the tip of the appenix. iceal specimen. Patients are at increase risk of synchro-
As such, they are not usually the cause of appenicitis but nous neoplasms, particularly in the colon; thus, examination
rather are inciental nings. Over 95% of carcinoi tumors of the large intestine shoul be one with full colonoscopy.
of the appenix are less than  cm in size (E). Tumors less Denitive treatment consists of a right colectomy regarless
CHAPtEr 10 Alimentary Tract—LargeIntestine 141

of the size of the tumor. If the nal pathology reveals appen- been calle peritoneal mucinous carcinomatosis an features
iceal cancer, the patient shoul be taken back for a right col- extensive proliferative epithelium, cytologic atypia, an a
ectomy. In one series, the 5-year survival rate after curative high mitotic rate. Treatment consists of aggressive removal of
resection was 61% an 31%. The remaining answer choices all peritoneal implants as well as an appenectomy. Intraper-
are not typically a common presentation of appeniceal ae- itoneal chemotherapy likewise shows promising results. The
nocarcinoma (A, C–E). 5-year survival rate is approximately 50% but varies greatly
References: Fujiwara T, Hizuta A, Iwagaki H, et al. Appeniceal by histology. Tuberculous peritonitis often presents as slowly
mucocele with concomitant colonic cancer. Report of two cases. Dis progressive abominal istention ue to ascites, combine
Colon Rectum. 1996;39():3–36. with fever, weight loss, an abominal pain (C). Characteris-
Ito H, Osteen RT, Bleay R, Zinner MJ, Ashley SW, Whang EE. tic features at surgery are multiple whitish noules scattere
Appeniceal aenocarcinoma: long-term outcomes after surgical
over the visceral an parietal peritoneum. Salmonella enter-
therapy. Dis Colon Rectum. 004;47(4):474–480.
itiis typically presents with iarrhea, nausea, an vomiting
with stool leukocytes (D). It can rarely lea to intestinal per-
35. B. The most common perianal lesion in Crohn’s isease foration, most commonly through an ulcerate Peyer patch.
is a skin tag, followe by ssures (A). Fissures are tears in Yersinia infections can lea to mesenteric aenitis, coli-
the anoerm, an most are supercial an in the posterior tis, an ileitis that can present in a similar fashion to acute
miline (poorer bloo supply). A eep ssure or one in an appenicitis (E). Yersinia infections can also cause appenici-
unusual location (lateral) shoul raise concern for Crohn’s tis. Meigs synrome is seen in patients with a benign ovarian
isease. Crohn’s isease oes increase the risk of eveloping tumor (A) an presents with ascites an pleural effusion that
hemorrhois as well as perianal abscesses an stulas (C–E). resolve after resection of the tumor.
Most patients with anal manifestations will have Crohn’s Reference: Wirtzfel DA, Roriguez-Bigas M, Weber T, Petrelli
isease elsewhere. Perianal involvement is extremely rare NJ. Disseminate peritoneal aenomucinosis: a critical review. Ann
with UC. Surg Oncol. 1999;6(8):797–801.

36. E. The presentation an nings are consistent with 38. B. Previously, it was recommene that all patients
acute mesenteric aenitis (pseuoappenicitis). It is associ- shoul unergo surgery after the secon episoe of uncom-
ate with Y. enterocolitica, Helicobacter jejuni, Campylobacter plicate iverticulitis. However, several large stuies have
jejuni, an Salmonella or Shigella species, an streptococcal refute this, an it is now recommene that surgical inter-
infections of the pharynx. It occurs more commonly in chil- vention be offere on a case-by-basis basis, taking into
ren an is often precee by an upper respiratory infection account the number of episoes, age, comorbiities, sever-
(D). It is a iagnosis of exclusion. Physical examination typ- ity of attacks, an impact on quality of life. In particular, a
ically reveals more vague an iffuse tenerness, without lower threshol for surgery is recommene for iabetic an
signicant guaring, as oppose to the localize tenerness immunocompromise (taking sterois) patients. In contrast,
seen in appenicitis (B). Leukocytosis is usually present in it is recommene that all cases of complicate iverticuli-
patients with acute mesenteric aenitis with WBC counts tis be offere enitive surgical intervention after the acute
between 10 an 15 × 103 cells/μL, similar to those foun in conition has resolve. One of the principles of surgery for
patients with appenicitis (C). CT may show generalize iverticulitis is that one only nees to resect iname, thick-
lymphaenopathy in the small bowel mesentery, but these ene colon, espite the presence of iffuse iverticula (A, C,
nings are nonspecic. The iagnosis is often mae intra- E). Once the istal colon is remove, the intraluminal pres-
operatively. There is no nee for noal biopsy (A). sure will ecrease an the majority of the proximal iver-
Reference: Abel-Haq NM, Asmar BI, Abuhammour WM, ticula will resolve. Recurrence is primarily the result of an
Brown WJ. Yersinia enterocolitica infection in chilren. Pediatr Infect inaequate istal resection, which inavertently may leave
Dis J. 000;19(10):954–958. behin sigmoi iverticula. Because iverticula o not occur
in the rectum, the istal resection margin shoul be taken at
37. B. Pseuomyxoma peritonei is a confusing term because normal-appearing rectum (D). The rectum can be ientie
it has been applie to several ifferent pathologies. It has by the fact that the taenia splays out.
been use in reference to any progressive process in which References: Bullar KM, Rothenberger DA. Colon, rectum,
the peritoneal cavity becomes lle with a thick gelatinous an anus. In: Brunicari FC, Anersen DK, Billiar TR, et al., es.
substance. This gelatinous substance is thought to arise from Schwartz’s principles of surgery. 8th e. New York: McGraw-Hill;
005:1055–1118.
mucus-secreting cells from a perforate, mucus-proucing
Chapman J, Davies M, Wolff B, et al. Complicate iverticulitis:
tumor, which can be either benign or malignant an can orig-
is it time to rethink the rules? Ann Surg. 005;4(4):576–581.
inate from the appenix, small bowel, or ovary. Even if these Lipman JM, Reynols HL. Laparoscopic management of iver-
cells are benign, once it has sprea throughout the perito- ticular isease. Clin Colon Rectal Surg. 009;(3):173–180.
neum, the substance is ifcult to eraicate, an with time, Stocchi L. Current inications an role of surgery in the manage-
the patient’s small bowel becomes mechanically obstructe. ment of sigmoi iverticulitis. World J Gastroenterol. 010;16(7):804–817.
If the source is a malignant tumor, the 5-year survival rate is
signicantly reuce. The most common source of this coni- 39. B. An Amyan hernia is one containing the Appen-
tion is a benign mucinous cystaenoma of the appenix. The ix. The importance of an Amyan hernia is that it can be
new terminology has been coine isseminate peritoneal confuse with a stanar strangulate hernia. Manage-
aenomucinosis to ene patients with mucinous peritoneal ment shoul consist of appenectomy without the use of
implants that arise from a benign aenoma of the appenix. mesh. It is name after Clauius Amyan, who performe
This is the most common variety. A more aggressive form has the rst appenectomy in Lonon in 1746. The patient was
142 PArt i Patient Care

an 11-year-ol boy with a scrotal hernia that containe the aministration of broa-spectrum antibiotics, removal of the
appenix perforate by a pin. Petit hernia is a type of lumbar infectious source (in this case by appenectomy), an antico-
hernia locate in the inferior lumbar triangle (A). It is boun agulation (for the suspecte thrombose superior mesenteric
by the iliac crest inferiorly, the external oblique muscle ante- vein). Neither carcinoi synrome (C) nor IBD (E) are likely
riorly, an the latissimus orsi muscle posteriorly. Littre her- to present with elevate alkaline phosphatase an total bili-
nia is a hernia containing Meckel iverticulum (C). Spigelian rubin, an neither ts the clinical history of the patient.
hernia is a hernia through the linea semilunaris an between References: Chang YS, Min SY, Joo SH, Lee SH. Septic throm-
two layers of abominal wall, making these ifcult if not bophlebitis of the porto-mesenteric veins as a complication of acute
impossible to palpate (D). Grynfeltt hernia is another type of appenicitis. World J Gastroenterol. 008;14(8):4580–458.
lumbar hernia foun in the superior lumbar triangle, which Vanamo K, Kiekara O. Pylephlebitis after appenicitis in a chil.
J Pediatr Surg. 001;36(10):1574–1576.
is boun by the quaratus lumborum muscle on its oor, the
internal oblique muscle anteriorly, an the 1th rib superi-
orly (E).
42. C. The association of parasites with appenicitis is
somewhat controversial. The ebate is whether the para-
Reference: Logan MT, Nottingham JM. Amyan’s hernia: a case
report of an incarcerate an perforate appenix within an ingui-
site is an inciental ning or the actual cause. Ascariasis
nal hernia an review of the literature. Am Surg. 001;67(7):68–69. is the most common parasite worlwie, with an estimate
1.4 billion persons infecte. The majority of infections occur
40. C. Although only a minority of patients (10%) who pres- in the low- an mile-income countries (LMIC) of Asia
ent with terminal ileitis progress to Crohn’s isease on long- an Latin America but are becoming more common in the
term follow-up, the surgeon shoul always consier this Unite States owing to increase international travel an
iagnosis. The inications for resection woul inclue free emigration from LMIC. E. vermicularis (pinworm) is the
perforation, stula, or stricture. The iagnosis can be con- secon most common parasite (A). Intestinal parasites can
fuse with appenicitis. Provie the cecum is not iname, cause appenicitis by obstructing the lumen. Thus, it is
the appenix shoul be remove to avoi confusion in the always important to check the nal pathology; therapy with
future because recurrent abominal pain may evelop in a helminthicie is necessary postoperatively. Mebenazole,
the patient. However, in the presence of active inamma- pyrantel pamoate, an albenazole are the rugs of choice.
tion of the cecum, appenectomy shoul not be performe S. stercoralis (threaworm) can lea to pneumonitis, malab-
because there is a higher risk of an enterocutaneous stula sorption, an bleeing ulcers (B). E. granulosus can lea to
formation (B). Similarly, biopsy shoul be avoie because hyati cyst isease (D). C. sinensis (Chinese liver uke) can
this increases the risk for enterocutaneous stula formation increase the risk of pigmente (brown) gallstones an chol-
as well (D, E). Therefore, closure of the woun without fur- angiocarcinoma (E).
ther intervention is the correct management for this patient.
This patient shoul subsequently receive a colonoscopy with 43. C. When eciing whether to perform an inciental
ranom biopsies to look for evience of inammatory bowel appenectomy uring another proceure, one must factor in
isease. the lifelong risk of appenicitis versus the risks of appen-
ectomy an the aitional costs. Because the lifelong risk
41. B. Pylephlebitis is essentially an infectious inamma- of appenicitis is only 8.6% in men an 6.7% in women,
tion of the portal venous system. These veins rain the gas- inciental appenectomy is rarely recommene. In a
trointestinal tract. It typically begins within the small veins large stuy of patients unergoing cholecystectomy with
raining an area of infection within the abomen an is most an without inciental appenectomy, low-risk patients
often associate with iverticulitis an appenicitis. Exten- unergoing appenectomy showe a signicant increase
sion of the thrombophlebitis into larger veins can lea to in nonfatal complications (os ratio of 1.53). Particular cir-
septic thrombophlebitis of the portal vein or its tributaries cumstances in which inciental appenectomy (uring the
(superior mesenteric vein, splenic vein) as well as multiple course of another operation) woul be recommene are
small liver abscesses. Due to laminar ow patterns, the bac- for chilren about to unergo chemotherapy (ue to risk of
teria are more likely to loge an form abscesses in the right subsequent typhlitis), in the isable (i.e., para/quariple-
lobe of the liver. Similarly, amebic liver abscesses also form gic) who cannot react normally to abominal pain, Crohn’s
in the right lobe but are usually singular (A). Patients with isease patients (because they have a signicant risk of sub-
pylephlebitis are usually not jaunice but have elevate sequent abominal pain) whose cecum is free of macroscopic
liver enzymes (particularly alkaline phosphatase). Pylephle- isease (to minimize risk of postoperative cecal stula), an
bitis was much more common in the preantibiotic era, but iniviuals who are about to travel to remote places where
it has become very rare ue to major avances in antibiotic there is no access to meical/surgical care. The patients in
an surgical treatment. Air bubbles or thrombi of the portal the remaining answer choices (A, B, D, E) woul not benet
venous system are key nings of pylephlebitis on CT scan from an inciental appenectomy.
(D). The reporte mortality rate is as high as 30% to 50%. Reference: Wen SW, Hernanez R, Naylor CD. Pitfalls in non-
Because of the rarity, establishe management protocols ranomize outcomes stuies: the case of inciental appenectomy
are lacking. The most pruent approach seems to be rapi with open cholecystectomy. JAMA. 1995;74(1):1687–1691.
Alimentary Tract—Anorectal
MICHAEL A. MEDEROS, FORMOSA CHEN, AND BEVERLEY A. PETRIE 11
ABSITE 99th Percentile High-Yields
I. Anorectal Abscess: Cryptoglanular Abscess Infection of Glans in Crypts at Dentate Line
A. Types: perianal (subcutaneous), intersphincteric, ischiorectal, supralevator
B. Treatment consierations:
1. Every anorectal abscess requires rainage; antibiotics not typically require unless overlying cellulitis
or systemic signs of infection
. Approximately 30% of patients will evelop a stula-in-ano after incision an rainage
3. “Outwar” rainage when an abscess enters, or passes through, skeletal muscle (i.e., levator ani,
external sphincter); best for subcutaneous (perianal) or ischiorectal abscesses (together account for
90%); all others (intersphincteric, supralevator) shoul be raine internally through the rectum/anal
canal (e.g., incision of internal sphincter along length of abscess for intersphincteric abscess)
4. “Horseshoe” abscess (bilateral abscesses arising from the eep postanal space); treatment inclues
external rainage of bilateral ischiorectal fossae an open posterior rainage or internal rainage of
posterior abscess

II. Fistula-in-Ano: Chronic Form of Perianal Abscess in Which the Abscess Cavity Does Not Heal Completely;
Instea, It Becomes an Inammatory Tract With a Primary Opening (Internal Opening) in the Anal Crypt at the
Dentate Line an a Seconary Opening (External Opening) in the Perianal Skin
A. Types: submucosal/supercial, intersphincteric (0%–45%), transsphincteric (30%–60%),
suprasphincteric (<0%), extrasphincteric (%–5%)
B. Treatment consierations:
1. Ensure abscesses are raine an sepsis controlle prior to enitive measures
. Utilize techniques that will have the lowest risk of recurrence an sphincter ysfunction; evaluate all
patients for baseline fecal incontinence prior to enitive therapy
3. Rule out Crohn isease prior to enitive surgical management of complex stulas; stulas in Crohn
respon to Crohn meical management (e.g., iniximab) an may worsen with surgery
4. Surgical techniques: initial management either stulotomy or seton placement
a) Seton: initial management for unraine abscess relate to stula, an to mature a stulous tract
prior to enitive surgical intervention
b) Fistulotomy: ieal for low or supercial simple stulas involving less than one-thir of the
internal sphincter complex, an known internal/external openings
c) Fistulectomy: associate with larger efects, higher risk of incontinence, an without higher
healing rates compare to stulotomy; usually not use
) Cutting seton: may be use for stulas involving more than one-thir of the internal sphincter in
an attempt to supercialize the tract for stulotomy
e) Ligation of intersphincteric stula tract (LIFT) proceure: ieal for transsphincteric stulas with
mature tracts

143
144 PArt i Patient Care

f) Enorectal avancement ap: goo option for high an complex anal stulas; goal is to cover the
internal opening with ap of mucosa, submucosa, an rectal wall
g) Fibrin glue an plugs: relatively ineffective treatment; lowest risk of incontinence but associate
with high rates of persistent or recurrent isease

III. Pilonial Disease


A. Acquire foreign boy reaction to rupture hair follicles in the intergluteal cleft; pilonial isease oes
not involve the anal canal, ifferentiating it from stula-in-ano
B. Pilonial isease without concurrent abscess can be treate with weight loss, shaving of intergluteal area,
avoiing prolonge sitting, phenol application, an improving hygiene
C. Acute pilonial abscess requires incision an rainage (off miline)
D. Denitive therapy shoul be performe after the inammatory perio has resolve; igital rectal exam
shoul be performe to evaluate for retrorectal fullness
1. Principles of enitive surgical management:
a) Determine extent of pilonial isease by ientifying all “pits” an sinus tracts; a stula probe an
methylene blue are useful ajuncts
b) Chronic sinus tracts may be unroofe an marsupialize with healing by seconary intention
c) Extensive an complex pilonial isease requires excision own to the sacrococcygeal
fascia closure techniques inclue various avancement an rotational aps; rhomboi ap
reconstruction is associate with the lowest recurrence rate

IV. Anal Intraepithelial Neoplasia (AIN) an Squamous Cell Cancer (SCC)
A. Human papillomavirus (HPV) high-risk strains: 16 an 18 (HPV 6 an 11 associate with conyloma
acuminata)
B. Low-grae squamous intraepithelial lesion (LSIL) = AIN-1
1. Low-grae ysplasia, has the potential to progress to HSIL
. Surveillance in immunocompetent iniviuals; low risk for progression to HSIL
C. High-grae squamous intraepithelial lesion (HSIL) = AIN- an AIN-3
1. Premalignant lesion requiring intervention; 10% to 0% of lesions progress to SCC
. Manage with ablative or topical therapies (e.g., electrocautery, imiquimo, trichloracetic aci, 5-FU)
D. Anal canal SCC (cannot be completely visualize with istraction of gluteal cheeks) management:
1. Chemoraiation with moie Nigro protocol: 5-FU, mitomycin C, pelvic raiation (50–54 Gy) for all
except T1N0 (00 NCCN guielines), which requires WLE
. Follow-up: physical exam (igital rectal exam, anoscopy, inguinal LN palpation) starting 8 to 1
weeks after cXRT
a) If no evience of isease on exam: follow with exams every 3 to 6 months for 5 years
b) If persistent isease on exam: reexamine in 4 weeks, then every 3months; if still persistent
>6 months after cXRT, then biopsy, restage, an abominoperineal resection (APR) (if not
metastatic)
c) If progressive isease on exam: biopsy,restage, an APR (if not metastatic)
3. Metastatic: enitive chemo +/− raiation; immunotherapy (e.g., PDL1 inhibitors)
E. Perianal SCC (previously anal margin): completely visualize on istraction of the gluteal cheeks an
within 5 cm of the anal verge
1. If Tis-T1, N0, well to moerately ifferentiate: may be amenable to wie local excision with 1-cm
margins if negative margins can be achieve without affecting sphincter function
. Treat like anal canal SCC if tumor oes not meet the above criteria

V. Rectal Prolapse
A. Complete rectal prolapse (procientia) is characterize by concentric mucosal fols/rings versus raial
fols seen with incomplete (mucosal or hemorrhoial) prolapse
B. Requires colonoscopy to rule out other conitions prior to elective surgery to aress prolapse
C. Acute management of rectal prolapse is reuction; if can’t reuce, use sugar to ecrease eema an
reattempt
D. Surgery for enitive management
1. Transabominal approach for patients who are goo surgical caniates (<10% recurrence)
CHAPtEr 11 Alimentary Tract—Anorectal 145

a) Rectopexy (with or without mesh), a sigmoiectomy if reunant sigmoi colon or history of
constipation
b) Perineal approach for patients with signicant comorbi conitions or limite lifespan (up to 30%
recurrence at  years)
(1) Altemeier proceure: perineal rectosigmoiectomy (also inicate for incarcerate an/or
strangulate complete prolapse)
() Delorme proceure: mucosal stripping an muscle plication

VI. Rectal Aenocarcinoma


A. Preop evaluation: surgeon exam, pelvic MRI or enorectal ultrasoun, CT chest/abomen/pelvis
1. Pelvic MRI is the preferre imaging moality over enorectal ultrasoun to etermine epth of
invasion an noal involvement
B. Neoajuvant chemoraiation inications: tumors T3 or greater (stage ), N+ (stage 3), an/or if
ecreasing tumor size may spare sphincter an avoi APR
C. Ajuvant chemotherapy inications: stage  tumors with high-risk features (obstructive lesion, T4,
poorly ifferentiate) an stage 3 rectal cancer (N+)
D. Total mesorectal excision: excision of the rectum an all the pararectal lymph noes within the
mesorectum
1. Lowers risk of local recurrence an shoul be performe in all low- an mi-rectal cancers
E. Transanal local resection
1. The major ownsie is that total mesorectal excision (removes local lymph noes an ecreases
local recurrence) is not one with local excision; thus there is a higher local recurrence rate (7%–1%
compare with 5%–10% for a transabominal approach)
. Can be consiere in patients at lowest risk for lymph noe metastases; all of the following must be
true to qualify: T1 tumor within 8 cm from the anal verge, ≤3 cm, mobile an nonxe, <30% bowel
circumference, no perineural or lymphovascular invasion, well-to-moerately ifferentiate, able to
resect with >3 mm negative margins, no noal involvement on imaging
3. Intact full-thickness specimen with goal of 1.0 cm margins (although >3 mm margins are accepte)
146 PArt i Patient Care

Fig. 11.1 Anorectal Abscesses an Fistulas.

Fig. 11.2 Goosall Rule.


CHAPtEr 11 Alimentary Tract—Anorectal 147

Questions
1. A 3-year-ol male presents with anal pain. He 5. A 68-year-ol male presents to the emergency
reports some staining of bloo on tissue paper room with abominal istension, obstipation, an
after bowel movements. On physical exam, his weight loss. On CT scan, the patient is foun to
abomen is soft an benign. He has no perirectal have colonic ilation, ecompresse small bowel,
masses an is foun to have a lateral perianal an an obstructing mass in the rectum. On igital
ssure. What is the most likely cause? rectal exam, the mass is palpate at about 4 cm
A. Passage of a har stool from the anal verge. What is the next best step in
B. Receptive anal intercourse management?
C. Crohn isease A. Rectal stent placement
D. Ulcerative colitis B. Loop colostomy creation
E. Perirectal abscess C. Neoajuvant chemoraiation
D. Abominoperineal resection
2. A 61-year-ol male is referre by his primary E. Loop ileostomy creation
care physician for persistent anemia. He reports
a 0-poun unintentional weight loss an 6. A 55-year-ol male with cirrhosis complicate by
previous episoes of hematochezia. He unergoes esophageal varices presents with bright re bloo
colonoscopy in which a 5-cm fungating mass is per rectum. Nasogastric lavage reveals bilious
foun 10 cm from the anal verge. He is scheule ui. Hematocrit is 5, an patient is given 
for surgery. Which of the following is the most units of bloo. Colonoscopy emonstrates bloo
important surgical factor for reucing the risk of in the rectal vault an blue-tinte submucosal
local recurrence? elevations in the istal rectum an anal canal.
A. Twelve or more lymph noes harveste He is hemoynamically stable. What is the best
B. Total mesorectal excision treatment option next step?
C. Five-centimeter raial margin A. Meical management
D. One-centimeter negative istal margin B. Transjugular intrahepatic portosystemic shunt
E. Proximal ligation of the inferior mesenteric (TIPS)
vessels C. Hemorrhoiectomy
D. Injection sclerotherapy
3. A 45-year-ol male is foun to have a -cm lesion E. Balloon tamponae
in the anal canal that is biopsie an foun to be
squamous cell carcinoma. There is no evience of 7. Which of the following is true regaring
istant isease or aenopathy. What is the next hiraenitis suppurativa?
best step in management? A. It may mimic a complex anal stula
A. Abominoperineal resection B. There is no role for topical clinamycin in
B. Wie local excision perineal hiraenitis suppurativa
C. Chemotherapy an raiation C. Raical excision with skin grafting is typically
D. Topical imiquimo necessary
E. Observation D. It may progress beyon the anal verge into the
anal canal
4. A 30-year-ol female with stula-in-ano who E. It is not associate with keloi formation
was treate at an outsie hospital 3 months ago
now presents to you for further management. On
exam, there is a seton with the external opening
of the stula lateral to the anus. On igital rectal
exam, the internal opening is above the levator
complex. What is the best treatment option?
A. Enorectal mucosal avancement ap
B. Fibrin glue
C. Cutting seton
D. Ligation of intersphincteric stula tract
E. Fistulotomy
148 PArt i Patient Care

8. Twelve hours after hemorrhoial baning, a 13. A 65-year-ol woman presents to the emergency
45-year-ol man presents to the emergency epartment with severe perianal pain for 1
epartment reporting rectal an abominal pain hours that came on after straining uring a
an an inability to urinate. His temperature is bowel movement. Physical examination reveals
10°F an heart rate is 110 beats per minute. an exquisitely tener perianal mass with
Management consists of: bluish iscoloration uner the perianal skin.
A. Placement of a Foley catheter Management consists of:
B. Broa-spectrum IV antibiotics A. Stool softeners an sitz baths
C. Broa-spectrum antibiotics an rectal B. Rubber ban ligation
examination with the patient uner anesthesia C. Stab incision an rainage with the patient
D. Stool softeners an oral antibiotics uner local anesthesia in the emergency
E. In-an-out catheterization of blaer an stool epartment
softeners D. Elliptical excision of skin an rainage with
the patient uner local anesthesia in the
9. A 50-year-ol woman presents with rectal pain, emergency epartment
incomplete rectal voiing, an bright re bloo E. Rectal examination with the patient uner
an mucus per rectum. Colonoscopy reveals a general anesthesia with incision an rainage
solitary rectal ulcer in the istal rectum on the
anterior wall. A biopsy specimen of the ulcer 14. Which of the following is true regaring
shows chronic inammation. Management anogenital warts?
consists of: A. Human papillomavirus (HPV) types 6 an 11
A. Transanal excision of the ulcer preispose to malignancy
B. Perineal rectosigmoiectomy B. No association exists with squamous
C. Abominal rectopexy intraepithelial lesions
D. High-ber iet an efecation training C. Treatment epens on location an extent of
E. Rectal xation with prosthetic sling isease
D. Immunomoulator therapy is ineffective when
10. The most common cause of a rectovaginal stula use topically
is: E. Vaccine against HPV oes not prevent
A. Carcinoma of the rectum anogenital warts
B. Crohn isease
C. Obstetric injury 15. Which of the following is true regaring chronic
D. Ulcerative colitis anal ssures?
E. Raiation A. Topical iltiazem is rst-line treatment
B. Topical nitroglycerin an botulinum toxin
11. Approximately 3 hours after a hemorrhoiectomy, injection have similar results as rst-line
a patient continues to have bleeing from the therapies
anus. The nurse has change the pa multiple C. Topical nitrates are superior to topical
times an has attempte to pack the rectum with iltiazem
gauze. What is the next best step in management? D. Anterior ssures are more common in men
A. Rubber baning the bleeing site E. Lateral internal sphincterotomy is the-gol
B. Rectal packing with epinephrine gauze stanar treatment
C. Suture ligation
D. Ice packs 16. Hirschsprung isease presenting in an ault:
E. Foley catheter balloon compression A. Does not occur
B. Is not associate with the RET mutation
12. A 60-year-ol woman presents with severe C. Is best iagnose by a barium enema
perianal itching that is constant throughout the D. Requires a pull-through proceure for
ay. Examination reveals minimal erythema an enitive management
excoriations in the perianal region. Which of the E. Can be treate with anorectal myomectomy
following is the best initial treatment?
A. Exam uner anesthesia
B. Biopsy an/or culture
C. Oral antibiotics
D. Application of nonscente barrier cream
E. Intravenous sterois
CHAPtEr 11 Alimentary Tract—Anorectal 149

17. A 56-year-ol male patient is foun to have rectal 21. Which of the following is true regaring the bloo
aenocarcinoma just proximal to the entate line. supply to the rectum?
Which of the following is true about wie local A. The superior an mile rectal arteries arise
excision (WLE) of such a lesion? from the inferior mesenteric artery
A. WLE is an option provie the tumor is 4 cm B. The mile rectal veins rain into the internal
or less iliac veins
B. Inguinal lymph noe metastases o not occur C. The inferior rectal veins rain into the inferior
with rectal cancers above the entate line mesenteric vein
C. The presence of lymphatic invasion preclues D. The superior rectal veins rain into the inferior
WLE vena cava
D. WLE is reasonable provie the invasion E. There is excellent collateralization between the
remains within the serosa superior an mile rectal arteries
E. WLE is not a recommene option
22. An 80-year-ol woman with multiple
18. The recommene initial treatment of anal canal signicant meical comorbiities presents with
melanoma is: rectal prolapse. She has a history of chronic
A. Abominoperineal resection (APR) constipation. Colonoscopy nings are negative.
B. Wie local excision (WLE) Treatment woul be best achieve via:
C. WLE with regional lymph noe issection A. Fixation of the rectum with prosthetic sling
D. Raiation therapy (Ripstein repair)
E. Raiation therapy an chemotherapy B. Anterior resection with rectopexy
C. Thiersch anal encirclement
19. A 30-year-ol male presents with reness, pain, D. Resection of perineal hernia an closure of the
an uctuance in the intergluteal cleft, about 4 cm cul-e-sac (Moschcowitz proceure)
posterior to the anus. There is consierable hair E. Perineal rectosigmoiectomy (Altemeier
ajacent to the lesion. Which of the following is proceure)
the most appropriate management?
A. Incision an rainage in the intergluteal cleft 23. Which of the following statements are true
B. Incision an rainage lateral to the intergluteal regaring perianal isease in association with
cleft Crohn isease?
C. En bloc excision of the sinus tract with ap A. Anal stulas ten to have a single tract
reconstruction B. Magnetic resonance imaging (MRI) is not
D. Excision with primary closure particularly helpful
E. Unroong the tract an marsupializing C. The liberal use of multiple setons is helpful
D. Iniximab is ineffective in healing these
20. A 35-year-ol man with leukemia an severe stulas
neutropenia presents with severe anal pain. E. Aggressive use of stulotomy provies the
Physical examination at the besie emonstrates best chance of cure
inuration but no obvious uctuance in the
perianal region. Which of the following is the best 24. A 3-year-ol male who recently engage in
management? unprotecte anoreceptive sexual intercourse
A. Intravenous (IV) antibiotics only presents with severe rectal pain with
B. Besie anoscopy an, if uctuant mass mucopurulent ischarge. Which of the following
etecte, then besie incision an rainage etiologies is most likely?
C. Besie anoscopy an, if uctuant mass A. Chlamydia trachomatis
etecte, then operative incision an rainage B. Neisseria gonorrhoeae
D. Examination uner anesthesia with wie C. Treponema pallidum
ebriement of perianal area D. Haemophilus ducreyi
E. Examination uner anesthesia with biopsy of E. Shigella species
inurate areas an incision an rainage,
even if no pus is etecte
150 PArt i Patient Care

25. Which of the following is true regaring stula- C. Fistulas are categorize base on their
in-ano? relationship to the anal mucosa
A. Drainage of an anorectal abscess rarely results D. Surgical treatment is etermine by the
in a persistent stula-in-ano internal an external opening of the stula
B. The internal opening is generally easily E. Injecting hyrogen peroxie or methylene blue
ientiable into the external opening is contrainicate

Answers
1. C. Typical anal ssures are preominantly locate in the Anal canal SCC inclues lesions that are incompletely visu-
posterior miline (90%) an, less commonly, the anterior alize with spreaing of the gluteal cheeks, while perianal
miline. The two leaing theories for eveloping an anal s- SCC inclues lesions that are completely visualize with
sure are relative ischemia of the posterior miline an high spreaing of the gluteal cheeks to a raius of 5 cm from the
mechanical stress at the posterior location. Constipation, anus. First-line treatment for locoregional anal canal SCC
passage of large forme or har stools, instrumentation, an is chemoraiation with 50 to 54 Gy of raiation with 5-FU
receptive anal intercourse may cause tears in the anoerm an mitomycin. Abominoperineal resection is reserve for
in the posterior an anterior locations (A,B). Atypical s- patients who have persistent or recurrent isease 6 months
sures can form anywhere in the anal canal an are relate to or greater after chemoraiation (A). Perianal SCC may be
unerlying conitions such as Crohn isease, tuberculosis, amenable to wie local excision for Tis-T1 lesions (select T
HIV, leukemia, an anal neoplasms. Ulcerative colitis gen- lesions) if there is no signicant involvement of the sphincter
erally oes not affect the anus (D). Perirectal abscesses may complex an 1 cm margins can be obtaine (B). Lesions that
be relate to unerlying stula-in-ano but are not generally o not meet those criteria, have evience of noal isease,
associate with anal ssures (E). or are poorly ifferentiate are treate like anal canal SCC
Reference: Lu KC, Herzig DO. Anal ssure. In: Steele SR, Hull with chemoraiation. Observation is an appropriate option
TL, Saclaries TJ, Senagore AJ, Whitlow CB, es. The ASCRS textbook for patients with a low-grae squamous intraepithelial lesion
of colon and rectal surgery. 3r e. Springer International Publishing; (AIN 1) since these lesions often regress an are less likely to
016:05–14. progress to SCC. Conversely, high-grae squamous intraep-
ithelial lesions (AIN /3) are consiere premalignant an
2. B. Total mesorectal excision (TME) allows for a complete may be treate with ablative therapy or topical agents, such
resection of the rectal tumor an raining of lymph noes, as imiquimo (D). SCC by enition is invasive cancer an
achieving tumor-free circumferential an istal margins. In shoul not be observe (E).
patients treate with TME, local recurrence rates were lower References: Samani T, Nash GM. Anal cancer. In: Steele SR,
compare to those who unerwent conventional surgery Hull TL, Saclaries TJ, Senagore AJ, Whitlow CB, es. The ASCRS
(9% versus 16%, respectively). Lymph noe issection is an textbook of colon and rectal surgery. 3r e. Springer International Pub-
important tenet of cancer surgery. A stanar TME removes lishing; 016:357–371.
the pararectal noes. Further, 1 noes may not be harveste in Ajani JA, Winter KA, Gunerson LL, et al. Fluorouracil, mitomy-
resection for a mi- or low-rectal cancer, especially in patients cin, an raiotherapy vs uorouracil, cisplatin, an raiotherapy for
carcinoma of the anal canal: a ranomize controlle trial. JAMA.
who receive neoajuvant raiation (A). A lateral lymph noe
008;99(16):1914–191.
issection of the common, external, an internal iliac noes as
James RD, Glynne-Jones R, Meaows HM. Mitomycin or cis-
well as the obturator noes may have a small ecrease in local platin chemoraiation with or without maintenance chemotherapy
recurrence when compare with stanar TME in patients for treatment of squamous-cell carcinoma of the anus (ACT II): a
who i not receive neoajuvant raiation, but the number of ranomise, phase 3, open-label, × factorial trial. Lancet Oncol.
noes harveste is not pertinent. A istal resection margin of 1 013;14(6).
cm is recommene for low rectal cancers. The negative istal
an raial margins can be achieve with a TME (C, D). Rou- 4. A. Several treatments for stula-in-ano have been
tine high (proximal) ligation of the inferior mesenteric vessels escribe. Determining the type of stula is important when
oes seem to affect outcomes in the absence of obvious proxi- eciing which treatment is best suite for the patient. “Low”
mal inferior mesenteric noal involvement (E). stulas inclue those that involve less than one-thir of the
Reference: Katz MHG. Proctectomy. In: American College of internal sphincter complex. These stulas are often amenable
Surgeons, Katz MHG, es. Operative standards for cancer surgery: vol- to stulotomy with low risk of causing sphincter ysfunction
ume II: esophagus, melanoma, rectum, stomach, thyroid. 1st e. Lippin- (E). Fistulas that involve more than one-thir of the internal
cott Williams an Wilkins; 018:134–145. sphincter complex are consiere “high.” Cutting setons are
those that are perioically tightene, facilitating supercial-
3. C. Anal squamous cell carcinoma (SCC) is categorize by ization of the stula tract for eventual stulotomy/stulec-
its location: anal canal or perianal (previously anal margin). tomy. However, this treatment is associate with a signicant
CHAPtEr 11 Alimentary Tract—Anorectal 151

risk of anal incontinence for extrasphincteric stulas (C). serpentine. It is important to ifferentiate varices from
Fibrin glue acts as a sealant an has the lowest risk of anal hemorrhois because certain interventions on a mistaken
incontinence of the options liste, but it is associate with a varix, like hemorrhoiectomy, coul be catastrophic (C). Ini-
very low success rate an is not recommene (B). Enorec- tial treatment for any acute gastrointestinal blee inclues
tal mucosal avancement aps are an option for complex prompt resuscitation an correction of coagulopathy. Naso-
stulas (high transsphincteric, suprasphincteric, an extras- gastric tube emonstrating nonblooy bilious ui sug-
phincteric stulas) with a success rate between 60% an 93%. gests that this patient oes not have an upper GI blee an
The ligation of intersphincteric stula tract (LIFT) proceure so EGD woul not be necessary (A). Patients with cirrhosis
is another option for high transsphincteric stulas, with a an stigmata of portal hypertension incluing rectal varices
success rate of about 70%. However, this is not an option for must rst be meically optimize with soium restriction
an extrasphincteric stula (D). an oral iuretics (furosemie, spironolactone). First-line
References: Santoro GA, Abbas MA. Complex anorectal stu- enoscopic intervention inclues injection sclerotherapy or
las. In: Steele SR, Hull TL, Saclaries TJ, Senagore AJ, Whitlow CB, enoscopic ban ligation. However, large anorectal varices
es. The ASCRS textbook of colon and rectal surgery. 3r e. Springer may not be amenable to baning. Pneumatic tamponae is a
International Publishing; 016:45–74. goo measure to stop active bleeing an is use as a brige
Steele SR, Kumar R, Feingol D, Rafferty JL, Buie WD, The Stan-
to a enitive intervention (E). TIPS is a useful intervention
ars Practice Task Force, the American Society of Colon an Rectal
to relieve the portal venous pressure an reuce variceal
Surgeons. Practice parameters for the treatment of perianal abscess
an stula-in-ano. Dis Colon Rectum. 011;54:1465–1474.
bleeing. However, it is associate with an increase risk of
Williams JG, Farrans PA, Williams AB, et al. The treatment of hepatic encephalopathy an shoul only be consiere as a
anal stula: ACPGBI position statement. Colorectal Dis. 007;9 Suppl last resort (B). An alternative for persistent bleeing inclues
4:18–50. angioembolization. Surgery is rarely inicate.
Jarrar A, Church J. Avancement ap repair: a goo option for Reference: Robertson M, Thompson AI, Hayes PC. The man-
complex anorectal stulas. Dis Colon Rectum. 011;54:1537–1541. agement of bleeing from anorectal varices. Curr Hepatol Rep.
017;16(4):406–415.
5. B. Decompression is critical when aressing any
obstructive lesions in any portion of the gastrointestinal 7. A. Hiraenitis suppurativa is isease of the follicular
tract. This patient with an obstructing rectal cancer will epithelium, involving areas containing cutaneous apocrine
likely require neoajuvant therapy; however, the obstruction sweat glans. It occurs in the armpits, groin, uner the
must be aresse rst (C). Rectal stents are for obstructive breasts, an between the buttocks. The typical appearance
lesions in the mi to high rectum an serve as a brige for is of multiple open comeones with sinus tracts an small
surgery. However, stenting a low obstructing rectal cancer abscesses. Scarring can lea to keloi formation (E). It can
is associate with chronic pain, tenesmus, worse quality mimic complex anal stula isease but stops at the anal verge
of life, an stent migration (A). In general, emergent resec- because there are no apocrine sweat glans in the anal canal
tion of a locally avance, obstructing rectal cancer without (D). It can also mimic perianal Crohn isease. Initial treat-
proper staging an omitting multimoality therapy shoul ment is with warm compresses an lifestyle changes such
be avoie because this may potentially compromise onco- as weight loss, wearing loose-tting clothes, cessation of
logic outcomes (D). A proximal iverting ostomy is an ieal smoking, an local hygiene. Topical antibiotics (e.g., clina-
option for a low obstructing rectal cancer. A loop colostomy mycin) an biologic therapy, such as aalimumab, have also
(e.g., transverse or sigmoi loop colostomy) will effectively been use with some success (B). If this fails, surgery may be
relieve the obstruction (B). A loop ileostomy is a viable neee to incise an rain acute abscesses an unroof stu-
option if there is evience of small bowel ilation, suggesting las with ebriement of granulation tissue. Raical excision
an incompetent ileocecal valve. However, a loop ileostomy an skin grafting are almost never necessary (C).
woul not relieve the obstruction in this case, where the ile- Reference: Dunn KM, Rothenberger DA. Colon, rectum, an
ocecal valve is competent, evience by the ecompresse anus. In: Brunicari F, Anersen DK, Billiar TR, Dunn DL, Hunter
small bowel (E). A loop colostomy woul provie effective JG, Matthews JB, Pollock RE. es. Schwartz’s principles of surgery.
venting whether there is small bowel ilation or not. 10th e. McGraw Hill Eucation; 015:133.
References: You YN, Hariman KM, Baffor A, et al. The
American Society of Colon an Rectal Surgeons clinical practice
guielines for the management of rectal cancer. Dis Colon Rectum.
8. C. Sepsis after the treatment of hemorrhois has been
00;63(9):1191–1.
escribe after baning, sclerotherapy, an staple hemor-
Pisano M, Zorcolo L, Merli C, et al. 017 WSES guielines on rhoiectomy. Although very rare, it is life threatening. It is
colon an rectal cancer emergencies: obstruction an perforation. most common in immunocompromise patients. The patient
World J Emerg Surg. 018;13(1):36. usually presents within the rst 1 hours after the proce-
ure but can present in a elaye fashion. The most com-
6. D. The inex of suspicion for rectal varices shoul be mon symptoms are severe perineal pain, fevers, an urinary
high in this patient with cirrhosis an portal hypertension. retention. Appropriate management of sepsis after hemor-
Rectal varices are the result of portosystemic shunting from rhoiectomy inclues hospital amission, ui resuscitation,
the inferior mesenteric vein an superior rectal veins via an IV antibiotics with coverage of gram-negative ros an
the mile an inferior rectal veins ue to unerlying por- anaerobes. Examination with the patient uner anesthesia is
tal hypertension. Characteristics of rectal varices that help recommene to rule out a necrotizing infection that may
ifferentiate them from internal hemorrhois are that they require ebriement. Conservative management with mei-
o not prolapse an they originate from the rectum. A sub- cal management is not appropriate for a patient suspecte of
mucosal varix often has a bluish-gray hue an may appear having sepsis (A,B,D,E).
152 PArt i Patient Care

References: Cirocco WC. Life threatening sepsis an mortality surgically correctable causes (prolapsing hemorrhois, s-
following staple hemorrhoiopexy. Surgery. 008;143(6):84–89. sure, neoplasm, stula), antibiotic use, noninfectious er-
McClou JM, Jameson JS, Scott AND. Life-threatening sepsis fol- matologic causes (seborrhea, psoriasis, contact ermatitis),
lowing treatment for haemorrhois: a systematic review. Colorectal an systemic iseases (jaunice, iabetes). However, the
Dis. 006;8(9):748–755.
majority of pruritus ani is iiopathic an often relate to
local hygiene (both overzealous an inaequate hygiene).
9. D. Solitary rectal ulcer synrome is an uncommon is-
Treatment focuses on removal of irritant, maintaining goo
orer that can be confuse with malignancy because the
perianal hygiene, ietary ajustments, an avoiing scratch-
patient presents with rectal bleeing, pain, an evience of
ing (A–C). Maintaining perianal hygiene is an important
straining uring bowel movements. It is a benign process
aspect of treatment. However, patients shoul be counsele
cause by an internal intussusception from chronic strain-
that the perianal region shoul not be scrubbe vigorously
ing, leaing to repetitive trauma to the mucosa. On proc-
an that use of scente proucts shoul be avoie as these
toscopy, noules or a mass may be foun, in which case the
can exacerbate the pruritus. Hypoallergenic an unscente
term colitis cystica profunda is use. Biopsy shoul be per-
moisturizing cream an barrier creams can be applie if ry
forme to exclue malignancy. The iagnosis of an internal
skin is an issue (D). Biopsy an/or culture of the region may
intussusception can be conrme with anorectal manometry
be necessary if the symptoms persist espite treatment (B).
an efecography. Treatment is nonoperative an inclues
Hyrocortisone ointment can provie symptomatic relief
a high-ber iet, efecation training to avoi straining, an
but shoul not be use for prolonge perios ue to risk of
laxatives or enemas. Either abominal or perineal repair, as
ermal atrophy that may lea to more pruritus (E).
for a patient with rectal prolapse, is recommene for failure
Reference: Ansari P. Pruritus ani. Clin Colon Rectal Surg.
of meical management (B, C). Transanal excision of a rectal 016;9(1):38–4.
ulcer is consiere in the management of rectal cancer after
etermining the extent of tumor invasion through the bowel 13. D. Hemorrhois shoul be istinguishe as being
wall an evaluating the ajacent lymph noes (A). Rectal x- either internal or external. Internal ones arise above the en-
ation with prosthetic sling can be consiere in the case of tate line an as such are insensate. They may cause painless
rectal procientia (E). bleeing uring straining to efecate, may prolapse, or may
Reference: Felt-Bersma R, Cuesta M. Rectal prolapse, rectal even become strangulate. If they strangulate, they can cause
intussusception, rectocele, an solitary rectal ulcer synrome. Gas-
pain ue to intense spasm of the anal sphincter. External
troenterol Clin North Am. 001;30(1):199–.
hemorrhois originate below the entate line, are covere
10. C. A rectovaginal stula is most often ue to an obstetric with anoerm, an may cause iscomfort such as itching,
injury after a vaginal elivery in association with episiotomy, but generally only cause severe pain if they become throm-
typically in primigravias. Other causes inclue inamma- bose. Treatment of thrombose external hemorrhois, as
tory bowel isease (Crohn isease more than ulcerative coli- in this case, consists of excision an rainage of the throm-
tis) (B, D), carcinoma of the rectum (A), raiation therapy bose hemorrhoi with the patient uner local anesthesia.
for pelvic malignancies (E), an, rarely, perianal abscesses To prevent recurrence or inaequate rainage, it is important
an iverticulitis. It can also be iatrogenic uring low ante- to excise an ellipse of skin an not simply perform a stab
rior resections, particularly in women who have ha a hys- avulsion (C–E). Do not rubber ban thrombose external
terectomy. Treatment for low stulas is with an enorectal hemorrhois because this is not well tolerate by patients
avancement ap, an for high stulas (more likely ue to seconary to severe pain (B). Nonoperative management is
neoplasm, Crohn isease, raiation), management is via a acceptable if the patient has ha symptoms for more than
transabominal approach with resection of the affecte rec- 7 hours an the pain is alreay beginning to subsie (A).
tal segment. Numerous stuies have shown that local anesthesia is well
tolerate.
11. C. Bleeing can occur immeiately or, in the case of Reference: Jongen J, Bach S, Stübinger SH, Bock JU. Exci-
hemorrhoial baning, after 7 to 10 ays, when the necrotic sion of thrombose external hemorrhoi uner local anesthe-
sia: a retrospective evaluation of 340 patients. Dis Colon Rectum.
stump sloughs off. Options for the management of bleeing
003;46(9):16–131.
inclue rectal packing with epinephrine gauze (B), ice packs
(D), an balloon compression with a Foley catheter (E). The
majority of bleeing is mil an resolves with simple mea-
14. C. Conyloma acuminata (anogenital warts) is cause
by HPV. There are at least 66 types of HPV. Types 6 an
sures. However, if bleeing is copious, the patient shoul be
11 are foun in benign anogenital warts, whereas types 16
taken back to the operating/proceure room, where visual-
an 18 behave more aggressively an are more frequently
ization is better, anesthesia is aequate, cautery can be use,
associate with ysplasia an malignant transformation
an suture ligation can be performe.
(A). There is an association with squamous intraepithe-
References: Jongen J, Bock JU, Peleikis HG, Eberstein A, Pster
K. Complications an reoperations in staple anopexy: learning by lial lesions an squamous cell carcinoma (B). Conylomas
oing. Int J Colorectal Dis. 006;1():166–171. occur in the perianal region, the squamous epithelial of anal
Ravo B, Amato A, Bianco V, et al. Complications after sta- canal, an occasionally the mucosa of the istal rectum. The
ple hemorrhoiectomy: can they be prevente? Tech Coloproctol. treatment epens on location an extent of isease. The
00;6():83–88. options inclue caustic agents (poophyllin, trichloroacetic
aci, nitric aci), cryotherapy, fulguration, surgical excisions,
12. D. Pruritus ani is a common problem with a multitue of antineoplastic preparations (5-FU), laser therapy, interferon,
etiologies. The possible etiologies inclue perianal infection, immunomoulator therapy (imiquimo), ciofovir, an
CHAPtEr 11 Alimentary Tract—Anorectal 153

surgical excision (D). There are vaccines against HPV that rains into both the inferior mesenteric noes an internal
potentially prevent anogenital warts (E). iliac noes. Rectal cancers just proximal to the entate line
Reference: Goron PH. Conyloma acuminatum. In: Goron can potentially sprea to inguinal lymph noes, so a careful
PH, Nivatvongs S, es. Principles and practice of surgery for the colon, inguinal examination for lymphaenopathy is an important
rectum, and anus. 3r e. CRC Press; 007:61–74. part of the physical examination in these patients (B). WLE
is an option in a limite number of cases for rectal aeno-
15. E. Anal ssures are thought to evelop as the result of carcinoma (E). Inications for WLE in rectal aenocarcinoma
the passage of har stools, causing trauma to the anoerm inclue size <3 cm, T1 status (invaes only submucosa) (D),
istal to the entate line an typically in the posterior loca- less than 30% involvement of bowel wall, proximity within
tion owing to its poorer bloo supply. Anterior ssures are 8 cm of anal verge, mobile an nonxe lesion, an well/
more common in women (D). Given their istal location, moerately ifferentiate, no lymphovascular/perineu-
anal ssures cause exquisite pain with each efecation, often ral invasion or tumor buing on tissue biopsy, or noal
accompanie by bloo on the toilet paper. In general, non- involvement on imaging. WLE nees a 1-cm raial an -mm
operative treatment starts by softening the stool with ber, eep margin. If WLE is contrainicate, abominoperineal
increase water intake, an sitz baths (i.e., topical agents are resection or low anterior resection is appropriate.
not always rst-line treatment) (A). Numerous topical agents Reference: Whitefor MH. Local excision of rectal neoplasia. In:
have been use with varying egrees of success, incluing Steele SR, Hull TL, Saclaries TJ, Senagore AJ, Whitlow CB, es. The
% liocaine jelly, nitroglycerin ointment (0.%), topical il- ASCRS textbook of colon and rectal surgery. 3r e. Springer Interna-
tional Publishing; 016:495–505.
tiazem, an topical arginine (a nitric oxie onor). Topical
nitrates o not have superior healing rates compare with
iltiazem (C). In fact, nitrates are associate with sie effects
18. B. Melanoma of the anal canal is extremely rare, an the
overall prognosis is poor. Given its rarity, establishe man-
such as heaaches in as many as 30% of patients, an thus
agement protocols are lacking. Raiation therapy an chemo-
topical iltiazem is use more frequently. Botulinum toxin
therapy can be consiere as ajuvant therapy epening on
injections have similar efcacy as nitroglycerin as rst-line
melanoma epth an staging (D, E). However, surgical resec-
treatment (B). Surgery is generally reserve for those for
tion is the initial treatment. A recent metaanalysis showe
whom meical management fails. Surgical management
no stage-specic survival avantage of APR over WLE (A).
involves a lateral internal sphincterotomy. Though it can
As such, WLE is the recommene management. Lymph
be performe on either sie, the right sie is more likely to
noe issection has not been shown to improve survival but
avoi hemorrhoial tissue. The internal sphincter shoul be
may incur signicant morbiities (C).
ivie an only the length of the ssure as this ensures the
References: Droesch JT, Flum DR, Mann GN. Wie local exci-
lowest rate of incontinence. Fissurectomy has inferior heal-
sion or abominoperineal resection as the initial treatment for ano-
ing rates compare to lateral internal sphincterotomy base rectal melanoma? Am J Surg. 005;189(4):446–449.
on two ranomize trials. Singer M, Mutch MG. Anal melanoma. Clin Colon Rectal Surg.
Reference: Stewart DB Sr, Gaertner W, Glasgow S, Migaly J, 006;19():78–87.
Feingol D, Steele SR. Clinical practice guieline for the manage-
ment of anal ssures. Dis Colon Rectum. 017;60(1):7–14. 19. B. Pilonial isease is theorize to exist ue to rup-
ture hair follicles in the intergluteal region. These ingrown
16. E. Hirschsprung isease rarely presents in aults (A). hairs may become infecte an present as an abscess in the
In this setting, the patient typically has a lifelong history sacrococcygeal region. However, this is one theory of the
of constipation an fecal impaction. A careful history will origin; the true etiology is still unknown. Pilonial isease
often reveal symptoms ating back to infancy. In most cir- can either present acutely with an abscess or chronically
cumstances, Hirschsprung isease presenting in an ault (prior rainage). Acute isease is best treate with incision
consists of a short segment of aganglionosis. Although a bar- an rainage of the abscess lateral to the intergluteal cleft,
ium enema can be iagnostic if an extremely ilate proxi- as oppose to irectly in the cleft because the latter creates
mal colon, transitional zone, an contracte istal colon an constant friction in the woun an therefore heals poorly
rectum are seen, it may miss short-segment Hirschsprung (A). The remaining answer choices are use for chronic is-
isease if the rectal tube is introuce too far past the anal ease (E). Although there is not a “gol stanar” for chronic
canal, bypassing the contracte segment (C). As such, the pilonial cyst management, the preferre treatment option
iagnosis is establishe by a rectal mucosal biopsy specimen epens on whether the pilonial cyst is simple or complex.
emonstrating aganglionosis. As in chilren, Hirschsprung Excision with primary closure off the miline for a simple,
isease is associate with the RET mutation in a percentage noninfecte pilonial cyst is the most appropriate treatment
of patients (B). Although pull-through proceures, such as option (D). Complex pilonial cysts may require an en bloc
the Soave or Duhamel operation, are performe in chilren excision of the sinus tract with a ap reconstruction (C). A
an in those with long segments of aganglionosis, an anorec- rhomboi ap is the favore approach.
tal myomectomy can be performe in aults with short-seg- Reference: Dunn KM, Rothenberger DA. Colon, rectum, an
ment aganglionosis (D). anus. In: Brunicari F, Anersen DK, Billiar TR, Dunn DL, Hunter JG,
Reference: Wu J, Schoetz D, Coller J. Treatment of Hirschsprung’s Matthews JB, Pollock RE. es. Schwartz’s principles of surgery. 10th e.
isease in the ault: report of five cases. Dis Colon Rectum. McGraw Hill Eucation; 015:133.
1995;38(6):655–659.
20. A. Perianal pain may evelop in neutropenic patients,
17. C. The upper an mile rectum mostly rain into yet the iagnosis of a perianal abscess may be ifcult given
the inferior mesenteric noes, whereas the lower rectum the lack of inammatory response to infection. In a severely
154 PArt i Patient Care

neutropenic patient with no uctuance, IV antibiotics alone to etermine the extent of isease. Antibiotics (metronia-
is consiere appropriate treatment. In patients who o zole, ciprooxacin) are use in treatment of stulas to con-
not improve, or who subsequently evelop uctuance, an trol symptoms an sepsis, but stulas ten to recur when
examination with the patient uner anesthesia shoul be per- the antibiotics are iscontinue. Immunomoulators (cyc-
forme to rule out an abscess that requires rainage (B, C). losporine, tacrolimus, mercaptopurine, azathioprine, an
Any areas of inuration shoul be biopsie to exclue a iniximab) have been use as well with varying egrees of
leukemia inltrate an culture to ai in the selection of success. Of these, iniximab seems to be the most effective
antimicrobial agents (E). Wie ebriement of perianal area (D). The liberal use of setons is recommene. Aggressive
woul not be inicate for a perianal abscess (D). use of stulotomy shoul be avoie for low intersphinc-
teric, suprasphincteric, or extrasphincteric stulae because it
21. B. The superior rectal arteries arise from the inferior is associate with elaye healing an an increase risk of
mesenteric artery, which provies bloo to the upper rectum. incontinence (E).
The mile rectal artery arises from the internal iliac artery References: Davis BR, Kasten KR. Anorectal abscess an stula.
an the inferior rectal artery arises from the puenal artery In: Steele SR, Hull TL, Saclaries TJ, Senagore AJ, Whitlow CB, es.
(branch of the internal iliac artery), which provie bloo to The ASCRS textbook of colon and rectal surgery. 3r e. Springer Inter-
the rest of the rectum an the anal canal (A). Rich collaterals national Publishing; 016:15–44.
exist between the rectal arteries such that they are relatively Gol SL, Cohen-Mekelburg S, Schneier Y, Steinlauf A. Perianal
stulas in patients with Crohn’s isease, part 1: current meical
resistant to ischemia. Suak’s point marks the superior rectal
management. Gastroenterol Hepatol (NY). 018;14(8):470–481.
an mile rectal junction. It is consiere a watershe area
an thus is unique in that it has a poor bloo supply (E). The
24. B. Proctitis typically presents with pain, tenesmus, rec-
mile rectal arteries are the least consistent an are absent
tal bleeing, iarrhea, an mucous ischarge. It can be ue
in as many as three-fourths of patients. The venous rain-
to a bacterial infection, viral infection, trauma, raiation, or
age follows the arterial supply (C). The superior rectal veins
inammatory bowel isease. Bacterial proctitis is often ue
rain into the inferior mesenteric vein an then to the portal
to sexually transmitte isease an is associate with anal
vein (D), whereas the mile an inferior rectal veins rain
intercourse. N. gonorrhoeae is the most common bacterial
into branches of the internal iliac veins an into the inferior
cause, followe by Chlamydia, which tens to prouce fewer
vena cava.
symptoms (A). T. pallidum, H. ducreyi, an Shigella species
are uncommon causes of proctitis (C–E). Bacterial proctitis
22. E. Procientia (rectal prolapse) is much more common can also be ue to nonsexually transmitte iseases, pri-
in women than men. It is most common in elerly women. marily in association with inammatory bowel isease.
In young men, it is more often associate with psychiatric Treatment of bacterial proctitis is with antibiotics, whereas
isease. It involves all layers of the rectum an starts 6 to 7 for proctitis in association with inammatory bowel is-
cm from the anal verge. As a general rule, aults with rectal ease, the treatment inclues sterois an 5-aminosalicylic
prolapse require surgery, whereas chilren can often be man- aci enemas.
age nonoperatively. Proceures are ivie into abominal
an perineal proceures. In general, abominal proceures 25. D. Drainage of an anorectal abscess provies a cure for
are associate with a lower recurrence rate but a higher com- the majority of patients, with 6% to 50% going on to evelop
plication rate than perineal proceures. As such, abominal a persistent stula-in-ano (A). Most stulas are cryptoglan-
proceures are use for younger, lower-risk patients, an ular in origin. Other causes, though less common, inclue
perineal proceures are use for oler, higher-risk patients trauma, Crohn isease, malignancy, raiation, an infections
(A, B). Recent stuies have shown favorable results with the (tuberculosis, actinomycosis, an chlamyia). The external
perineal rectosigmoiectomy in elerly high-risk patients. opening of the stula is usually obvious, whereas the inter-
The perineal rectosigmoiectomy has a 15% recurrence rate nal one is often har to ientify (B). Fistulas are categorize
an is a goo option for oler patients. Another well-ac- base on their relationship to the anal sphincter complex
cepte perineal operation is the Delorme proceure, which (intersphincteric, transsphincteric, an suprasphincteric)
involves reeng the rectal mucosa. The Thiersch anal encir- (C). Surgical treatment is etermine by the location of the
clement is no longer use (C). Moschcowitz proceure is internal an external openings an the course of the stula
more often performe for the management of vaginal pro- tract (D) an may inclue simple stulotomy, raining or
lapse (D). cutting seton, brin glue injection, brin plug, ligation of
Reference: Williams JG, Rothenberger DA, Maoff RD, Gol- intersphincteric stula tract (LIFT) proceure, or anorectal
berg SM. Treatment of rectal prolapse in the elerly by perineal rec- avancement ap. Gently injecting hyrogen peroxie or
tosigmoiectomy. Dis Colon Rectum. 199;35(9):830–834.
methylene blue into the external opening may help ientify
the internal opening (E). The main goal of treatment is to
23. C. Anal stulas in association with Crohn isease ten treat an eliminate sepsis while at the same time maintain-
to be complex an have multiple stulous tracts (A). MRI is ing continence.
particularly helpful to etect the extent of the stula tract an Reference: Davis BR, Kasten KR. Anorectal abscess an stula.
ientify abscesses an to visualize the anal sphincter an pel- In: Steele SR, Hull TL, Saclaries TJ, Senagore AJ, Whitlow CB, es.
vic oor muscle (B). These patients shoul also unergo sig- The ASCRS textbook of colon and rectal surgery. 3r e. Springer Inter-
moioscopy, colonoscopy, an small bowel follow-through national Publishing; 016:15–44.
Breast
NAVEEN BALAN, JUNKO OZAO-CHOY,
AND CHRISTINE DAUPHINE 12
ABSITE 99th Percentile High-Yields
I. Breast Imaging
A. Screening mammogram guielines: q1- years with initiation at 45-50 years ol
1. 3D mammography or igital breast tomosynthesis for high-risk or ense breast patients
B. Ultrasoun (US) as part of iagnostic workup of breast symptom
1. <30 years ol: US can be use as solitary evaluation
. >30 years ol: US an iagnostic mammogram must be use
C. MRI
1. BRCA1/BRCA2 (an other hereitary breast synromes): annual MRI from ages 5 to 9; those 30+
nee annual MRI + mammogram (alternating q6months)
. Gail risk with >0% lifetime cancer risk: annual MRI + mammogram (alternating q6months)
3. Useful in workup of axillary noal metastasis with unknown breast primary

Category Deęnition Management Example


BI-RADS 0 Need more information Additional imaging Abnormal screening mammogram
BI-RADS 1 Normal Routine screening Normal breast tissue
BI-RADS 2 Benign Routine screening Ovoid, smooth solid mass, stable ęndings
BI-RADS 3 Probably benign Interval imaging (3–6 mo) Benign-appearing clustered calcięcations
BI-RADS 4 Suspicious Biopsy Indeterminate clustered calcięcations
BI-RADS 5 Highly suggestive for cancer Biopsy Spiculated mass, branching calcięcations
BI-RADS 6 Biopsy-proven cancer Deęnitive Treatment Mass with biopsy-proven malignancy

II. Benign Breast Disease


A. Fibrocystic isease
1. Pathologic iagnosis: microcysts, brosis, hyperplasia, apocrine metaplasia, an aenosis; commonly
foun on stereotactic breast biopsy of calcications
B. Mastitis/breast abscess
1. Mastitis: oral antibiotics with gram-positive coverage; continue breastfeeing if lactating
. Abscess: smoking, nipple rings, nipple cleft are risk factors; aspiration is preferre, Incision an
ebriement (I&D) only if aspiration not possible/abscess is supercial, if multiple recurrences
consier surgical excision
a) IV antibiotics an hospitalization ifsevere; icloxacillin for lactating patient
b) Milk stula (complication of I&D): stula will resolve with cessation of breastfeeing
c) Associate mass: biopsy to rule out malignancy; neee to conrm granulomatous mastitis (a
chronic autoimmune inammatory isorer); stain for AFB, fungi
155
156 PArt i Patient Care

C. Nipple ischarge
1. Physiologic: milky, green, gray, yellow, blue, bilateral, stimulation-inuce
a) Check TSH, prolactin if spontaneous bilateral milky ischarge (if elevate, then MRI for
prolactinoma)
. Pathologic: serous/blooy, unilateral, spontaneous; most common cause of unilateral blooy nipple
ischarge is intrauctal papilloma (malignancy rate is 7%)
D. Mastalgia
1. Clinical breast exam; iagnostic imaging only neee for focal breast pain; generalize mastalgia not
associate with malignancy
. Reassure patient, use supportive bras, can also use anazol an tamoxifen if persistent
E. Abnormal imaging with iscorant benign pathology
1. Pathologic ning iscorant from BIRADS 5 imaging, then surgical excision require

III. Breast Cancer


A. Lumpectomy: for nonrecurrent uctal carcinoma in situ (DCIS) or invasive cancer (followe by raiation
in most)with small/isolate lesions amenable tobreast conservation an without pathogenicmutation
for hereitary breast cancer
B. Mastectomy: for DCIS or invasive cancer with locally avance/multiple lesions, recurrence/
pathogenic mutation for hereitary cancer, or contrainication to raiation (prior raiation treatment,
connective tissue isorer, homozygous ATM gene mutation)
C. Sentinel lymph noe biopsy: perform with mastectomy/lumpectomy
1. Blue ye: contrainicate in pregnancy; Lymphazurin—rare anaphylaxis; methylene blue—can cause
tissue/skin necrosis
. Sulfur colloi (technetium Tc 99m): raiotracer use in lymphoscintigraphy
3. Resect all raio “hot” (>10% of hottest noe), blue, an/or palpable LNs; no minimum require
4. False-negative rate ecreases with more lymph noes harveste postneoajuvant chemo
5. Higher noal ientication with both ye an raiotracer injection in subareolar space
D. Axillary lymph noe issection:
1. Inicate if any SLNs positive postneoajuvant, > positive SLNs in lumpectomy, any positive noes
in mastectomy, inammatory breast cancer, if unable to n noes on SLNB for invasive cancer
. Increase risk of lympheema, chronic pain, thoracoorsal (latissimus orsi—unable to auct arm),
an long thoracic nerve injury (serratus anterior—winge scapula)
E. Neoajuvant chemotherapy: given for locally avance breast cancer in orer to attempt breast
conservation therapy (lumpectomy), inammatory breast cancer, noe-positive isease, an strongly
consiere in triple-negative or HER2+ cancer
F. Early breast cancer (T1-T, N0, M0): lumpectomy + SLNB + raiation versus mastectomy + SLNB;
+/− chemotherapy/enocrine therapy base on receptor status/gene signature prole
G. Locally avance breast cancer (T3-T4, N1/N): neoajuvant chemotherapy, then surgery followe by
raiation therapy; enocrine therapy base on receptor status (gene signature not inicate); aitional
ajuvant chemotherapy if HER2+ an/or resiual isease
CHAPtEr 12 Breast 157

IV. Special Consierations

Lesion History Imaging/histology Management


Simple cyst Painful, enlarges Oval or round, well-circumscribed, Aspiration if symptomatic; if bloody,
with menses anechoic on US send for cytology
Complex cyst Painful, palpable Thickened wall, septations, solid Core needle biopsy of the solid
mass features, intracystic mass, components or wall
bloody
Fat necrosis Prior trauma Lucent centered calcięcations Core needle biopsy usually not needed
unless diagnosis is uncertain
Fibroadenoma Mobile, enlarging Well circumscribed, smooth, Surgical excision if growing or
mass, usually develops central popcorn symptomatic; no routine excision
young patients calcięcations as mass involutes <2 cm
with age
Phyllodes tumor Rapidly growing Leaf-like projections with Wide local excision with 1 cm margins;
mass epithelial and stromal if malignant, no axillary evaluation
components (benign and needed because nodal spread is rare
malignancy types)
Intraductal Unilateral bloody Branching ębrovascular core, Surgical excision to rule out
papilloma nipple discharge overlying epithelial and concomitant malignancy (peripheral
myoepithelium; most common papillomas have higher risk of
cause of unilateral bloody malignancy)
discharge
Paget disease Eczematoid Large cells with pale cytoplasm Full-thickness skin biopsy of involved
changes to and prominent nucleoli nipple then central lumpectomy vs
nipple mastectomy, if invasive cancer or
DCIS is also found, then treat as such
Radial scar Detected Spiculated with central lucency, Excisional biopsy to rule out
(complex abnormality on ębroelastic core with malignancy
sclerosing lesion) screening surrounding radiating ducts/
lobules
Atypical ductal/ Atypical hyperproliferation of Excisional biopsy, no margins needed;
lobular ductal or lobular epithelium consider 5 y of antiestrogen therapy
hyperplasia/LCIS after excision to lower risk of
subsequent cancer
DCIS Branching, pleomorphic, or - Mastectomy vs lumpectomy
widespread calcięcations - SNLB if: mastectomy, lumpectomy in
on mammogram, subtypes UOQ, or central lumpectomy with
(comedo, cribriform, papillary, removal of nipple
pseudopapillary) - 2-mm margins needed during
excision
Invasive cancer Abnormal Spiculated mass, pleomorphic - Mastectomy vs lumpectomy with
screening; or widespread calcięcations, radiation
Palpable mass invasion of cancer cells - SLNB vs ALND
- No ink on tumor for margin

A. Inammatory breast cancer (can be misiagnose as mastitis)


1. Clinical: erythema, peau ’orange changes, usually breast inammation that has faile meical
management an has rapily progresse over the course of weeks
. Negative skin biopsy oes not rule out the iagnosis; still nee to perform core neele biopsy of
unerlying mass or lymph noe; pan-CT require to evaluate for istant metastases (0%–35% of patients)
3. Pathology: skin biopsy shows ermal lymphovascular tumor emboli an obstruction of ermal
lymphatics (cause of eema an peau ’orange)
4. Treatment: neoajuvant chemotherapy, moie raical mastectomy without immeiate
reconstruction, ajuvant raiation (no role for SLNB even if clinical response).
B. Male breast cancer
1. Consier genetic testing for all men with breast cancer, otherwise treat as with women; no nee for
prophylactic bilateral mastectomy in males with BRCA (ifferent than women)
. Tamoxifen is superior to anastrozole for hormone therapy
158 PArt i Patient Care

C. Breast cancer uring pregnancy


1. First trimester: if no abortion, mastectomy + SLNB versus ALND, chemotherapy safe in n trimester
or later, but raiation/enocrine/trastuzumab therapy must be postpartum
. Secon to early 3r trimester neoajuvant chemotherapy (if inicate), then mastectomy versus
lumpectomy + SLNB versus ALND + raiation/enocrine/trastuzumab postpartum
D. Invasive breast cancer in patients >70 years of age
1. For luminal A (strongly ER/PR-positive, HER2-negative) cancers, may omit raiation postlumpectomy
if negative margins an negative SLNB; may omit SLNB if poor operative caniate
E. ACOSOG Z0011
1. SLNB for T1/T, clinically noe-negative cancer in patients who procee irectly to breast
conservation therapy (lumpectomy) without prior chemotherapy
a) If more than two noes return positive → nee completion ALND
b) must then get chemotherapy an raiation for equivalent survival

V. Meications
A. Enocrine therapy
1. Selective estrogen receptor moulator (SERM): block effect of estrogen on tissue
a) Tamoxifen: increase venous thromboembolism risk, increase risk of enometrial
aenocarcinoma, pre- or postmenopausal
b) Raloxifene: prevent osteoporosis, lower rate of uterine cancer, postmenopausal
. Aromatase inhibitor (AI): blocks conversion of anrogens to estrogens in peripheral tissues in
postmenopausal women; ecreases bone ensity (serial DEXA scans for osteoporosis)
3. Anastrozole outperforms tamoxifen for postmenopausal patients, can cause myalgias; switch to
letrozole or exemestane if not tolerate (other AIs)
B. Chemotherapy
1. Anthracyclines (oxorubicin [Ariamycin]): irreversible cariotoxicity
. Taxols (paclitaxel): numbness, tingling, burning of hans/feet
C. HER2 receptor inhibitor (trastuzumab): reversible cariotoxicity; cannot be use in pregnancy

Fig. 12.1
CHAPtEr 12 Breast 159

Questions
1. A 56-year-ol woman with cT1N0 invasive uctal 4. A 45-year-ol woman with ER-negative, PR-
carcinoma (IDC) of the left breast (ER+/PR+/ negative, HER2-negative right invasive uctal
HER2-) unergoes lumpectomy with sentinel breast cancer that is 3 cm in size an positive
noe biopsy. Pathology emonstrates a 1 mm axillary lymph noes also has fullness in the
caual margin an conrms no lymph noe right supraclavicular area. Ultrasoun-guie
involvement. A 1-gene assay recurrence score is ne-neele aspiration of a supraclavicular noe
8 (low). Which of the following escribes the next reveals metastatic breast cancer. Which of the
best management? following is escribes the best management of
A. Reexcision of the caual margin, raiation this patient?
therapy, enocrine therapy A. Chemotherapy followe by moie raical
B. Raiation therapy an enocrine therapy mastectomy an chest wall raiation extene
C. Chemotherapy, raiation therapy, enocrine to supraclavicular fossa
therapy B. Chemotherapy, followe by moie raical
D. Raiation therapy alone mastectomy with excision of supraclavicular
E. Enocrine therapy alone noe an stanar chest wall raiation
C. Moie raical mastectomy with excision
2. A 50-year-ol woman unergoes further of supraclavicular noe, followe by
mammographic workup of an abnormality chemotherapy an stanar chest wall raiation
etecte on screening. Subsequent iagnostic D. Moie raical mastectomy with excision
mammographic views are coe BI-RADS 5, an of supraclavicular noe, followe by
a core neele biopsy is performe, showing usual chemotherapy an chest wall raiation
uctal hyperplasia, apocrine metaplasia, an extene to supraclavicular fossa
aenosis. Which of the following statements is E. Palliative chemotherapy only, no role for
TRUE regaring the next step in management? surgical resection
A. Excision of the mammographic abnormality
shoul be performe 5. A 78-year-ol woman with mil ementia,
B. Excision of the mammographic abnormality chronic obstructive pulmonary isease (COPD),
shoul be performe, followe by iabetes, en-stage renal isease, an a prior
chemoprophylaxis with tamoxifen lower extremity amputation for peripheral
C. Excision is not necessary, but vascular isease has an episoe of severe chest
chemoprophylaxis with tamoxifen is pain 1 ay after unergoing core biopsy of a
recommene 1.5-cm left breast mass. EKG shows an acute
D. Neither excision nor chemoprophylaxis is MI. Angiogram emonstrates a critical stenosis
recommene of the left anterior escening artery, an a
E. Breast MRI shoul be performe to rug-eluting stent is place. She is subsequently
etermine nee for further excision an/or place on antiplatelet therapy. Her biopsy results
chemoprophylaxis later reveal invasive uctal carcinoma that is
low grae, 90% estrogen receptor (ER) an
3. Which of the following is true regaring Polan progesterone receptor (PR) positive, an HER2
synrome? negative. Her axilla is clinically negative. What is
A. It typically presents as a bilateral conition the BEST therapeutic approach to this patient?
B. Women are more commonly affecte than A. Plan lumpectomy an sentinel noe biopsy for
men 1 weeks post stent placement
C. It can be associate with excess hair in the B. Plan mastectomy an sentinel noe biopsy for
chest/axillary region 1 weeks post stent placement
D. It is ue to an x-linke autosomal recessive C. Refer for neoajuvant chemotherapy, with plan
genetic isorer for subsequent lumpectomy an sentinel noe
E. It typically presents with abnormal igits on D. Refer for palliative chemotherapy as she is not
the ipsilateral upper extremity a surgical caniate
E. Initiate neoajuvant enocrine therapy, with
plan for subsequent lumpectomy
160 PArt i Patient Care

6. A 50-year-ol female presents to your clinic 9. Which of the following is TRUE regaring
complaining of generalize, nonfocal cyclical chemotherapy for the treatment of breast cancer?
breast pain in her left breast. Her clinical breast A. Neoajuvant chemotherapy has been shown
examination is normal, an she has a negative to have better outcomes for ER-positive,
screening mammogram within the past 6 months. HER2-negative breast cancer compare with
You avise her that: ajuvant chemotherapy
A. Breast pain is frequently associate with breast B. In patients with ER-negative, PR-negative,
cancer HER2-negative (triple-negative) breast
B. Oral contraceptives are not associate with cancer, complete response to neoajuvant
breast pain chemotherapy is achieve in the vast majority
C. Pharmacologic agents are not recommene in of patients
the treatment of breast pain C. 1 gene assay recurrence score may be use
D. The t of her bra is an important consieration to guie chemotherapy treatment in noe-
as a cause of breast pain negative early-stage ER+ breast cancer
E. Breast pain is not very responsive to treatment D. Chemotherapy is most effective in inltrating
uctal cancers that have low Ki67
7. Which of the following is TRUE of sentinel lymph E. Chemotherapy is inicate if the breast cancer
noe (SLN) biopsy? is proven to be invasive
A. Ientication of SLNs by either the blue ye
or raioactive colloi is successful in the vast 10. Which of the following is TRUE regaring
majority of cases positive lymph noes in breast cancer?
B. SLN biopsy shoul not be performe in A. Involvement of internal mammary lymph
women with breast cancer an conrme- noes is consiere stage IV isease
noal isease who are unergoing B. In the setting of breast conservation,
neoajuvant therapy completion axillary lymph noe issection
C. There is no role in DCIS shoul be performe if the sentinel lymph
D. Utilization of the technetium raiocolloi is noe biopsy is positive
contrainicate in pregnancy C. In a patient with a core biopsy-proven positive
E. The false-negative rate is extremely low lymph noe an no primary lesion etecte,
axillary lymph noe issection an serial
8. Which of the following is TRUE regaring 6-month mammograms are recommene
raiotherapy for the treatment of breast cancer D. In the setting of mastectomy with a positive
after breast conservation? sentinel lymph noe, raiotherapy to the
A. Raiotherapy shoul be performe even if no axilla can be consiere instea of performing
lymph noes are positive for cancer axillary lymph noe issection
B. Raiotherapy is recommene as treatment E. Following neoajuvant chemotherapy for a
for a positive margin after lumpectomy 6-cm primary breast cancer with associate
C. Whole breast raiotherapy is most effective 3-cm noes, if all lesions have isappeare on
when given concurrently with chemotherapy ultrasoun imaging axillary issection is no
D. Raiotherapy is less efcacious with small longer necessary
tumors compare to larger ones
E. Higher energy raiation exerts more amage
to the skin
CHAPtEr 12 Breast 161

11. A 5-year-ol woman with a boy mass inex 14. Which of the following statements is true
(BMI) of 5 is recommene to unergo a regaring the lymphatic anatomy of the breast?
mastectomy for a 9-cm segmental istribution of A. Axillary lymph noes are organize into three
calcications that were shown to be DCIS on core levels with respect to the pectoralis major
neele biopsy. The calcications are about 1 cm muscle
from the nipple. Physical exam an ultrasoun B. In a stanar axillary issection for breast
of the axilla are negative. The patient has a small cancer, only level I an II noes are remove
breast contour an esires reconstruction of her C. Approximately 30% of the lymphatic rainage
breast. In aition to sentinel noe biopsy, which from the breast goes to the contralateral lymph
of the following woul be the BEST management? noes
A. Nipple-sparing mastectomy with immeiate D. Rotter noes are technically level I noes
tissue expaner placement E. Batson plexus is a network of lymphatics that
B. Nipple-sparing mastectomy with elaye rain the subareolar portion of the breast
reconstruction
C. Skin-sparing mastectomy with immeiate 15. Which of the following is true regaring
tissue expaner placement gynecomastia?
D. Skin-sparing mastectomy with elaye A. It is consiere a risk factor for male breast
reconstruction cancer
E. Total mastectomy with elaye reconstruction B. Alcohol is not a risk factor
after ajuvant therapy C. It is uncommon after age 50
D. It is ue to accumulation of subareolar fat.
12. After a moie raical mastectomy, a 45-year- E. It is associate with use of proton-pump-
ol woman reports new-onset weakness in the inhibitors (PPI)
ipsilateral arm when pulling own on a cor to
ajust the blins in her home. On examination, 16. Which of the following is least likely to contribute
she has ifculty when attempting to internally to the evelopment of breast infection (mastitis/
rotate an auct her arm. What is the best abscess)?
explanation for her ecits? A. Nipple ring insertion
A. Transection of the intercostobrachial nerve B. Granulomatous mastitis
B. Application of surgical clips across the long C. Smoking
thoracic nerve D. Hiraenitis
C. Transection of the thoracoorsal nerve E. Alcohol intake (> rinks/ay)
D. Cautery injury to the supraclavicular nerve
E. Retractor injury to the meial pectoral nerve 17. A 44-year-ol woman presents with a palpable
tener mobile mass in the upper outer quarant
13. A 35-year-ol woman presents with burning of her left breast. The overlying skin is normal
pain an reness along the anterolateral right an there is no aenopathy on exam. Ultrasoun
breast. On exam, a rm tener cor coul be examination reveals a .5-cm cystic lesion. An
palpate just below the skin from the shouler ultrasoun-guie cyst aspiration is performe.
tracking own towar the lateral breast. Which Which of the following is true?
of the following is TRUE regaring the initial A. The ui shoul be sent for cytologic
management of this isease? examination only if it is bloo tinge
A. Mammogram an ultrasoun shoul be B. Straw-colore ui shoul prompt a core
performe neele biopsy
B. Systemic anticoagulation shoul be initiate C. The presence of septations is associate with a
C. Antibiotics covering gram-positive bacterial low recurrence rate of the cyst after aspiration
strains shoul be aministere D. Thickness of the cyst wall oes not correlate
D. A short course of oral corticosteroi therapy with cancer risk
shoul be prescribe E. Viscous gel-like ui is a poor prognostic sign
E. An incisional biopsy of the skin shoul be
performe
162 PArt i Patient Care

18. Nipple ischarge is most suspicious of breast 22. A 50-year-ol woman has unergone stereotactic
cancer in which of the following women? neele biopsy of a 4 cm area of abnormal
A. A 35-year-ol woman with bilateral brown calcications, showing high-grae uctal
ischarge that is only visible with squeezing of carcinoma in situ (DCIS). Which of the following
the nipple choices is the most appropriate treatment for this
B. A 45-year-ol woman with unilateral serous patient?
ischarge that is spontaneous A. Lumpectomy alone
C. A 30-year-ol woman who is lactating an B. Moie raical mastectomy
notices unilateral blooy nipple ischarge that C. Lumpectomy an sentinel lymph noe biopsy
is spontaneous D. Lumpectomy an sentinel lymph noe biopsy,
D. A 50-year-ol woman with greenish-colore followe by whole breast raiotherapy
ischarge bilaterally that is sometimes E. Lumpectomy an sentinel lymph noe biopsy
spontaneous with intraoperative raiotherapy
E. A 40-year-ol woman with bilateral milky
ischarge that occurs spontaneously onto her 23. BRCA1 an BRCA2 are:
bra A. Protooncogenes
B. Cyclin-epenent kinase
19. Which of the following statements is TRUE C. Tumor suppressor genes
regaring tamoxifen therapy? D. Mismatch repair genes
A. It has been shown to reuce the risk of E. Tyrosine kinases
eveloping breast cancer by 90% in patients
that are consiere high risk 24. Which of the following is TRUE of invasive
B. Its primary serious sie effect is loss of bone lobular carcinoma of the breast?
mineral ensity A. It is more commonly associate with
C. In ER-positive invasive breast cancer, optimal pleomorphic lobular carcinoma in-situ (LCIS)
uration of therapy for patients uner 50 is 10 as oppose to nonpleomorphic LCIS
years B. Lobular cancers are typically hormone
D. It is more effective when aministere receptor-negative
concurrently with chemotherapy C. Breast conservation therapy is contrainicate
E. Treatment with tamoxifen is safe in the secon D. Invasive lobular cancers typically appear on
an thir trimesters of pregnancy mammogram an ultrasoun as a iscrete
mass
20. Which of the following is most characteristic of a E. Lobular cancers comprise 40% of all invasive
malignant lesion as seen on ultrasoun imaging? breast cancers
A. Taller-than-wie measurements
B. Hypoechoic mass 25. A 8-year-ol lactating woman presents with a
C. Anechoic mass -ay history of right breast pain an reness
D. Homogenous internal structure that is progressively worsening. On examination,
E. Bilateral ege shaowing a 4-cm area of skin ajacent to the nipple-areolar
complex is erythematous an tener, with
21. A 45-year-ol premenopausal woman unergoes some focal eema an no etectable uctuance.
stereotactic core neele biopsy of calcications Focuse ultrasoun conrms the absence
seen on screening mammogram. The biopsy of a ui collection. The appropriate initial
reveals atypical uctal hyperplasia (ADH). Which management woul consist of:
of the following is TRUE about the management A. Image-guie core neele biopsy
of this patient? B. Cessation of breast-feeing an/or pumping
A. Tamoxifen shoul be prescribe C. Incision an rainage
B. The lesion shoul be completely excise with a D. Oral antibiotics
negative margin E. Mammography
C. No further excision is require if the
calcications were completely remove
D. Prophylactic bilateral mastectomy shoul
strongly be consiere
E. Sentinel lymph noe biopsy shoul be
performe along with excision of the lesion
CHAPtEr 12 Breast 163

26. A 40-year-ol woman presents with a 10-cm 29. A 50-year-ol woman presents to her primary
right breast mass. She notes that it has been octor with a palpable mass in the upper outer
rapily growing, an the weight of the mass quarant of her right breast. It has been present
causes her right breast to rest lower than her left. an unchange for 3 months, an she has no
Pathology from a core neele biopsy reveale personal or family history of breast or ovarian
a broepithelial lesion with notable leaike cancer. On examination, there is a 1.5-cm rm,
projections of the stroma. Which of the following nontener mass with no associate skin or
statements is true of this lesion? nipple abnormalities an no lymphaenopathy.
A. Stromal hypercellularity is the pathologic Mammography is performe an there is no
feature that typically istinguishes this lesion evience of mass, asymmetry, or calcication. It is
from broaenoma reporte as normal. What is the next appropriate
B. It commonly emonstrates an aggressive step?
growth pattern similar to breast cancer, A. Observation, with repeat physical exam in
inltrating surrouning tissues as it enlarges 3 months
C. Sentinel lymph noe biopsy has become B. Orer a repeat mammogram in 3 to 6 months
stanar in malignant cases C. Orer MRI of the breast
D. The aition of raiotherapy is recommene D. Orer focuse breast ultrasoun
in most patients to prevent recurrence after E. Excision of the mass
lumpectomy
E. Surgical margins of at least  cm are 30. MRI of the breast is best inicate in which of the
recommene following scenarios?
A. 45-year-ol woman with a 1-cm area of
27. A 55-year-ol woman was foun on routine microcalcications that is excise an
mammography to have a new, 1.7 cm, stellate pathology emonstrates atypical uctal
lesion with a translucent area in the central hyperplasia
portion. Which of the following best escribes B. 45-year-ol, average-risk woman with focal
appropriate management of this lesion? breast pain an normal mammogram an
A. Observation only ultrasoun
B. Repeat mammography in 6 months C. 45-year-ol woman with inltrating carcinoma
C. MRI of the breast foun in an axillary noe with a negative
D. Image-guie core neele biopsy followe by mammogram an ultrasoun
wire-localize excision D. 45-year-ol woman with microcalcications
E. Image-guie core neele biopsy, followe by that are excise an pathology emonstrates
wire-localize wie excision with a negative DCIS with comeo necrosis
margin E. 45-year-ol woman with a 1-cm area of
microcalcications that is excise an
28. Which of the following is TRUE regaring pathology emonstrates lobular carcinoma in
intrauctal papilloma? situ
A. The presence of blooy ischarge is
concerning for atypia or malignant regions 31. A 45-year-ol woman presents with a 10 mm area
within the papilloma of suspicious microcalcications on mammogram.
B. Peripherally locate papillomas (istant from Stereotactic core neele biopsy reveals only LCIS.
the nipple) are associate with an increase in Wire localize excisional biopsy successfully
subsequent breast cancer risk removes all calcications but there is classic LCIS
C. The most common presenting symptom is a at the margins. Which of the following is the most
nontener, smooth, mobile noule beneath the appropriate NEXT step in management?
nipple-areolar complex A. Bilateral prophylactic mastectomies, with or
D. When associate with blooy ischarge, without reconstruction
excision with a small margin is recommene B. Reexcision to clear margins
E. Breast MRI is inicate prior to excision to C. No further therapy
rule out malignancy D. Sentinel lymph noe biopsy to stage the
ipsilateral axilla
E. Lifelong tamoxifen
164 PArt i Patient Care

32. A 48-year-ol female is being evaluate for a new 36. A 1-year-ol woman with a strong family history
left breast mass that was foun on mammogram. of breast cancer has just learne she is a carrier
She reports having two alcoholic rinks per ay, of a BRCA1 germline mutation. Which of the
is an active smoker with a ve-pack-per-year following is TRUE regaring this mutation?
smoking history, an has a mother who was A. Breast cancers associate with BRCA1
iagnose with breast cancer at age 65. Her past mutations are typically hormone receptor
meical history is signicant for atypical uctal negative
hyperplasia that was excise 6 years previously. B. BRCA1 mutations are consiere “gain of
Which of the following factors is associate with function” mutations
the highest risk of breast cancer in this patient? C. BRCA mutations account for nearly half of all
A. Age breast cancers
B. Mother with a history of breast cancer D. Her lifetime risk of eveloping breast cancer
C. Daily alcohol intake can be reuce by half if she takes tamoxifen
D. Smoking E. Male relatives of the patient have a 100-fol
E. History of atypical uctal hyperplasia risk of eveloping breast cancer if they are
carriers of the mutation
33. Which histologic type of DCIS is most likely to
progress to invasive uctal cancer? 37. A 56-year-ol woman is iagnose with a 1.5 cm
A. Comeo breast cancer, which is estrogen an progesterone
B. Micropapillary receptor negative with no overexpression of
C. Papillary HER2/neu. Her axillary exam is normal. Asie
D. Cribriform from axillary evaluation by sentinel lymph
E. Soli noe biopsy, what is the most appropriate
recommenation for breast cancer therapy?
34. A 45-year-ol woman unergoes breast- A. Lumpectomy alone
conservation therapy for DCIS. The nal B. Lumpectomy plus hormonal therapy
pathology shows no evience of invasion an a C. Lumpectomy plus raiotherapy
0. mm cranial margin. The NEXT appropriate D. Lumpectomy plus chemotherapy
step is: E. Lumpectomy plus raiotherapy an
A. No further surgery; shoul initiate raiation chemotherapy
therapy
B. No further surgery; shoul initiate tamoxifen 38. Which of the following patients with a 1.5-cm
C. Reexcision of the close margin only invasive uctal breast cancer woul be the most
D. Reexcision of the close margin an perform appropriate for breast-conserving therapy?
sentinel noe biopsy A. 33-year-ol woman who is 10 weeks pregnant
E. Mastectomy at iagnosis
B. 58-year-ol woman who has a history of
35. A woman with a history of glioblastoma, left lumpectomy in the same breast for previous
lower limb osteosarcoma as a teenager an breast T1N0 breast cancer
cancer at the age of 40 is likely to have which of C. 55-year-ol woman with ipsilateral palpable
the following: lymph noes that appear abnormal on
A. Cowen synrome ultrasoun
B. Li-Fraumeni synrome D. 5-year-ol woman with scleroerma
C. Peutz-Jeghers synrome E. 50-year-ol woman with synchronous,
D. Ataxia-telangiectasia multicentric ipsilateral invasive lobular cancer
E. BRCA2 mutation
CHAPtEr 12 Breast 165

39. Which of the following is the most important 42. A 65-year-ol female unerwent left moie
preictor of 10-year isease-specic survival for raical mastectomy followe by chemotherapy
breast cancer? an raiation therapy for a stage II breast cancer
A. Primary tumor size when she was 40 years ol. She has ha long-
B. Histologic grae staning swelling of her ipsilateral arm an
C. Total number of positive lymph noes recently evelope raise purple noules along
D. Estrogen-receptor status the anterior upper arm. Which of the following is
E. Age at time of iagnosis TRUE regaring treatment of this lesion?
A. Treatment of this conition is largely
40. A 55-year-ol woman presents with 1 month conservative
of breast erythema an swelling. On physical B. Bevacizumab (angiogenesis inhibitor) plus
examination an mammogram, there is no paclitaxel has emerge as the treatment of
evience of a breast mass. However, there is choice
iffuse skin thickening an eema associate C. Concurrent Ariamycin-base chemotherapy
with a 3-cm lymph noe in the axilla. A trial of an raiation are consiere the optimal
broa-spectrum antibiotics has been ineffective. A treatment strategy
core neele biopsy reveals inltrating carcinoma D. Surgical resection is the optimal primary
that is 10% estrogen-receptor positive an HER2/ treatment moality
neu negative. Which of the following statements is E. Laser an raiofrequency ablation treatments,
TRUE regaring her management? followe by low-ose raiation therapy
A. Tamoxifen shoul be initiate immeiately
B. Moie raical mastectomy shoul be 43. A 58-year-ol postmenopausal woman with
performe as soon as possible to increase a history of right breast cancer presents with
chances of survival a new 1.-cm noule within the scar of her
C. Raiation therapy shoul be performe lumpectomy incision. Her prior therapy inclue
concurrently with chemotherapy to improve a negative sentinel noe biopsy, raiotherapy,
response rates an chemotherapy, an she is currently taking
D. Chemotherapy alone shoul be initiate tamoxifen. Core neele biopsy reveals recurrent
immeiately inltrating uctal carcinoma that is hormone
E. Antibiotics shoul be continue because of the receptor-positive an HER2 negative. Which of
infectious signs the following is TRUE regaring her conition?
A. Sentinel noe biopsy cannot be performe again
41. Batson plexus provies a potential metastatic B. Lumpectomy (excision of skin with margin)
route of breast cancer to: is recommene if patient esires breast
A. Supraclavicular noes conservation
B. Bone C. Tamoxifen shoul be continue for an
C. Liver aitional 5 years
D. Arenal glans D. She shoul unergo bilateral mastectomy
E. Lung E. Mastectomy is recommene

44. A 65-year-ol woman presents with a longstaning


history of a scaly eczematoi rash involving her
right nipple an extening onto the areola. The
rash has not resolve espite aily applications of
steroi cream. Other than the skin changes on the
nipple, the physical examination of the breast is
unremarkable. A mammogram is negative. Which
of the following is TRUE regaring this conition?
A. A new steroi cream shoul be aministere
B. A punch biopsy of the skin is recommene
C. If nipple ischarge is present, cytologic
examination of the ischarge is frequently
iagnostic
D. This lesion is precancerous
E. Mastectomy is rarely require to treat this
conition
166 PArt i Patient Care

45. Which of the following is TRUE of breast 46. The primary serious averse reaction to
lymphoma? trastuzumab that requires monitoring is which of
A. Primary breast lymphoma is preominantly a the following?
T-cell lymphoma A. Hepatic toxicity
B. Seconary breast lymphomas are much more B. Renal toxicity
common than primary breast lymphoma C. Cariac toxicity
C. Primary breast lymphoma oes not respon D. Pulmonary toxicity
well to the chemotherapy that is stanarly E. Bone marrow toxicity (aplastic anemia)
use for nonbreast lymphoma
D. Breast lymphoma has a preilection for central
nervous system recurrence
E. Treatment of breast lymphoma tens to require
mastectomy with noe issection in most cases

Answers
1. B. Genomic proling has emerge as an important future breast cancer risk) (B, C). Breast MRI has no role in
ajunct in etermining which early breast cancer patients iscorance of BI-RADS 5 imaging an benign core biopsy
benet from ajuvant chemotherapy. There are multiple nings (E).
assays available an are generally utilize in ER-positive,
HER2-negative, noe-negative invasive breast cancers to 3. E. Polan synrome is a sporaic congenital isorer
provie “low” versus “high” systemic recurrence risk scores. that classically affects the unilateral breast (A), chest wall,
For high scores, chemotherapy has emonstrate a survival an upper extremity. It is present in at least 1 in 100,000 ini-
benet over enocrine therapy alone; whereas for low scores, viuals, occurs more commonly on the right than left (:1
enocrine therapy alone has similar survival to enocrine to 3:1), an affects men more often than women (3:1) (B).
plus chemotherapy. This patient has a low recurrence score, Unerevelopment or absence of the pectoralis, serratus, an
eliminating benet from chemotherapy (C). Aitionally, latissimus orsi muscles, symbrachyactyly (fuse, missing,
following breast conservation therapy with lumpectomy an an/or shortene igits), shortene forearm, extrocaria,
SLNB, raiation is recommene to reuce local recurrence rib abnormalities, absent axillary hair (C), athelia, imin-
risk (E). While excision of DCIS is recommene to have ishe subcutaneous fat localize over the ipsilateral chest
a  mm margin, resection of invasive cancer only requires wall, an renal agenesis or hypoplasia (rare) have all been
no ink on tumor (A). Ajuvant enocrine therapy reuces escribe as characteristics of Polan synrome. The cause
the risk of cancer recurrence for hormone receptor-positive is thought to be ue to interruption in the vascular supply
cancers (D). to the affecte chest wall an upper extremity in utero (not
Reference: NCCN National Comprehensive Cancer Network a genetic isorer) resulting in hypoplasia of the chest wall
Clinical Practice Guielines in Oncology; Breast Cancer – BINV- muscles (D).
an BINV-6 pages of Version .01. http://www.nccn.org
4. A. Involve supraclavicular noes enote an N3c noal
2. A. Raiologic-pathologic iscorance is an important stage, which is a stage III breast cancer. Breast cancer woul
concept in etermining further management of breast lesions be stage IV if more istant noes (contralateral, periaortic,
after a percutaneous breast biopsy. Core neele biopsies hilar) are involve or cancer has metastasize to bone, brain,
inherently have a egree of sampling error when performe, lung, visceral organ, etc., for which palliative chemotherapy
an can unerestimate the lesion since only a small part is woul be inicate (E). Regaring treatment of supraclavicu-
sample. If the pathologic result is not concorant (i.e., con- lar noes, chemotherapy an raiation to the supraclavicular
sistent) with the raiologic ning, then surgical excision is fossa without surgical resection of the supraclavicular noe
manate to evaluate the entire tissue. Specically, if mam- is the recommene approach (B, C). Resection of the supra-
mographic nings are highly suspicious for malignancy clavicular noe may be recommene if not fully treate by
an are coe as BI-RADS 5, but core biopsy results are chemotherapy an raiation but it woul not be the planne
benign, then excisional biopsy shoul be performe as this enitive therapy (C, D).
inicates a iscorance between the imaging an pathology.
(C, D) Ductal hyperplasia, apocrine metaplasia, an aenosis 5. E. In elerly patients an in patients with multiple
are elements of brocystic change, a benign breast isease comorbiities that, in an of themselves, limit a patient’s
that oes not manate chemoprophylaxis with tamoxifen survival, the stanar treatment algorithms may be altere
(which is recommene for atypical lesions that increase to reuce potential averse effects of treatment that have
CHAPtEr 12 Breast 167

lower margins of benet in patients with limite life spans. Reference: Boughey JC, Suman VJ, Mittenorf EA, et al. Senti-
Given her recent myocarial infarction, immeiate surgi- nel lymph noe surgery after neoajuvant chemotherapy in patients
cal risk is extremely high (A, B). Therefore, a 3- to 6-month with noe-positive breast cancer: the ACOSOG Z1071 (Alliance)
course of aromatase inhibitor therapy followe by reimaging clinical trial. JAMA. 013;310(14):1455–1461.
to assess response is the best course of action. This patient
has a luminal A type breast cancer (strongly ER/PR-positive, 8. A. Raiotherapy works by irectly amaging DNA
HER2-negative, an low grae) that woul likely emon- within cells, not by inucing ischemia. It exerts most of
strate a low genomic prole signature signaling no benet for its effect uring the M phase of the cell cycle by inucing
chemotherapy (C, D) Furthermore, cariac toxicities relate formation of free oxygen raicals. As such, raiation ther-
to chemotherapy (oxorubicin) woul likely be higher risk apy is more efcacious with smaller tumors that have a
for this patient with cariac morbiities. higher oxygen potential (D). Higher energy raiation has a
skin-preserving effect as the maximal ionizing potential is
6. D. Breast pain is a common breast complaint among not reache until the raiation beam reaches eeper struc-
women an is a common reason for referral to specialty breast tures (E). Aitionally, it has been shown to be most effective
clinics although primary care physicians are able to work up when use sequentially after chemotherapy instea of con-
an treat breast pain (E). Breast pain is typically not associ- currently (C). Nearly all patients unergoing lumpectomy
ate with breast cancer although focal breast noncyclic breast for invasive an noninvasive breast cancer are caniates
pain in patients without recent breast imaging may warrant at for raiotherapy. NSABP B17 establishe that raiotherapy
least a screening mammogram or focuse breast ultrasoun signicantly reuces local recurrence when aministere
(A). Oral contraceptives, hormone therapy, psychiatric agents after lumpectomy (A). Postmastectomy raiotherapy is gen-
(serotonin an norepinephrine reuptake inhibitor, an anti- erally inicate for locally avance isease (T4, 4-cm tumor
psychotics) as well as cariovascular agents (spironolactone, size or greater, an 4 or more lymph noes positive). In gen-
igoxin) are known to cause mastalgia in some patients (B). If eral, positive margins shoul be excise an raiotherapy
breast pain is nonfocal an cyclical, conservative treatments not relie upon to clear margins (B). ACOSOG Z0011 trial
such as smoking cessation, iet moication an ecrease emonstrate that patients with early invasive breast can-
in caffeine intake, evaluating the t of the patient’s bra are cer (T1 an T) unergoing breast-conserving therapy o
important rst consierations (D). For continue an severe not nee completion axillary issection if the sentinel lymph
breast pain, pharmacologic agents such as vitamin E, eve- noe biopsy is positive as there is no ifference in mortality.
ning primrose oil, NSAIDs, tamoxifen, an anazol are other Reference: Giuliano AE, McCall LM, Beitsch PD, et al. ACOSOG
Z0011: a ranomize trial of axillary noe issection in women
options for treatment (C). The majority of patients with breast
with clinical T1- N0 M0 breast cancer who have a positive sentinel
pain will n relief in this treatment algorithm.
noe. J Clin Oncol. 010;8(18_suppl):CRA506-CRA506. oi:10.100/
Reference: Cornell LF, Sanhu NP, Pruthi S, Mussallem DM. jco.010.8.18_suppl.cra506.
Current management an treatment options for breast pain. Mayo
Clin Proc. 00;95(3):574–580.
9. C. Chemotherapy plays an important role in treating
7. A. Sentinel lymph noe biopsy is typically inicate occult istant metastatic isease in invasive cancer. How-
as an axillary staging proceure for patients with clinically ever, not all invasive cancers benet from chemotherapy (E).
noe-negative breast cancer. The success rate for ientifying Those that are low-grae, small, lymph-noe negative, an
the sentinel noes when using both a blue ye an raio- have low S-phase fractions (<5%) or Ki67 (<0%) (markers
active colloi is 95% or higher (A), an the false-negative of proliferation) have minimal to no benet over enocrine
rate (inaccurately etermining the axilla to be negative for therapy alone (D). Gene expression prole assays, such as
metastatic cancer) is aroun 10% (E). The clinical recur- the 1 gene assay, that test which genes are being expresse
rence rate in the axilla after negative sentinel lymph noe in cancer tissue have been able to categorize patients into
biopsy is 0.3%. By enition, DCIS is noninvasive an there- groups that will likely benet from chemotherapy versus
fore cannot be associate with positive noes in the axilla. those that will not (C). Chemotherapy time preoperatively
However, when DCIS is foun on core biopsy, the remaining versus postoperatively has the same survival outcome for
lesion may contain invasive cancer (i.e., upstage), so senti- all cancer subtypes (A). However, newer research may be
nel noe biopsy is recommene in cases where the upstage emonstrating benets for triple-negative an HER2-pos-
rate is highest (high-grae DCIS, comeonecrosis, associa- itive cancers. In triple-negative breast cancer, a complete
tion with a mass lesion, > cm) or if breast lymphatics will pathologic response (no more tumors seen at surgery) after
be remove/isrupte at the primary surgery (mastectomy neoajuvant chemotherapy is approximately 40% (B).
or lumpectomy in upper outer quarant of the breast) pre-
cluing performance of sentinel noe biopsy at a secon 10. D. In general terms, having grossly positive axillary
proceure if invasive cancer is unexpectely foun (C). In lymph noes is an inication for axillary issection. How-
women with invasive breast cancer an biopsy-proven noal ever, the ACOSOG Z0011 trial establishe equivalent sur-
isease, SLNB is still consiere after neoajuvant therapy vival with axillary issection an no axillary issection in
if the noal be is raiologically an clinically negative as patients with early-stage breast cancer unergoing lumpec-
a subset of patients will have ownstage noal isease tomy an raiation therapy who have three or fewer posi-
following neoajuvant therapy (B). Raioactive technetium tive noes (B). The AMAROS trial further establishe that
is low ose, an has been observe as safe for use in preg- women with early breast cancer an no clinically palpable
nancy (D). It is actually the blue yes that require caution in noes coul unergo raiotherapy in place of axillary is-
pregnancy. section if lymph noes were etermine to be positive (D).
168 PArt i Patient Care

However, for patients with locally avance isease an pal- presents elsewhere, it usually causes an acute onset of pain
pable lymph noes, axillary issection remains an important an tenerness. It is a result of an inammatory-throm-
component of treatment to prevent local recurrence (E). In a botic process an not an infectious or autoimmune isease
patient with an isolate positive noe, magnetic resonance (C, D). Risk factors inclue recent trauma or surgery to
imaging (MRI) looking for a primary is inicate, if no pri- the local area, heavy lifting, tight clothing, an unerlying
mary is foun, most woul recommen mastectomy (C). malignancy. Monor isease typically presents over the lat-
Involvement of internal mammary lymph noes represents eral aspect of the breast an eventually turns into a palpa-
avance local isease (cNb or cN3) not metastatic isease ble cor or har mass. The veins most commonly involve
(A). inclue the lateral thoracic vein, the thoracoepigastric vein,
Reference: Giuliano AE, McCall LM, Beitsch PD, et al. ACOSOG an, less frequently, the supercial epigastric vein. The is-
Z0011: a ranomize trial of axillary noe issection in women orer is benign, self-limite, an not itself malignant or a
with clinical T1- N0 M0 breast cancer who have a positive sentinel risk factor for breast cancer. Mammography an ultrasoun
noe. J Clin Oncol. 010;8(18_suppl):CRA506-CRA506. oi:10.100/ are typically performe to exclue unerlying malignancy.
jco.010.8.18_suppl.cra506.
Otherwise, the iagnosis is largely clinical an biopsy is not
necessary (E). Treatment consists of nonsteroial antiinam-
11. C. Nipple-sparing mastectomy is contrainicate in matory rugs an warm compresses. Antibiotics, systemic
patients with extensive intrauctal cancer, associate nipple anticoagulation, an corticosterois are not warrante (B).
ischarge, Paget isease, or cancer within a -cm istance Reference: Mayor M, Burón I, e Mora JC, et al. Monor’s is-
of the nipple (A, B). Whereas total mastectomy woul be an ease. Int J Dermatol. 000;39(1):9–95.
oncologically soun operation, this patient esires recon-
struction an has no contrainications to immeiate place- 14. B. Axillary lymph noes are classically organize into
ment of expaners an sparing the skin (D, E). six anatomic groups base on their anatomic location (lat-
eral, pectoral, scapular, central, subclavicular, an interpec-
12. C. Moie raical mastectomy inclues, by eni- toral). However, a more clinically useful classication is into
tion, a resection of level I an II axillary noes along with the levels base on their location relative to the pectoralis minor
entire breast parenchyma uner skin aps. Several important muscle, with level I being locate lateral (most inferior) to
nerves resie in the axilla, injury to which can lea to signif- the muscle borer, level II being locate behin the pecto-
icant motor an sensory ecits. Avoiance of intraopera- ralis minor, an level III noes meial (A). Rotter noes are
tive use of neuromuscular blockae uring anesthesia an interpectoral (between the pectoralis major an minor mus-
careful ientication of the long thoracic an thoracoorsal cles) an are technically level II noes (D). In a stanar axil-
nerves are key to avoiing inavertent injury. The intercos- lary issection, level I an II noes are remove. There are
tobrachial nerve is the lateral cutaneous branch of the secon approximately 0 to 30 lymph noes in the average axilla,
intercostal nerve. Resection oes not lea to any motor loss, an the lymphatic rainage is fairly preictable, following
but it can cause loss of sensation over the meial aspect of the a hierarchical pattern to the rst echelon of noes, followe
upper arm (A). The long thoracic nerve courses along the lat- by seconary an then tertiary echelons. This pattern is the
eral chest wall in the miaxillary line on the serratus anterior basis of the principle for sentinel lymph noe biopsy. Most
muscle to innervate it. The serratus anterior muscle abucts of the lymphatic rainage of the breast is to the axilla, with
an laterally rotates the scapula an hols it against the chest rainage to the contralateral breast being rare (C). For this
wall. Injury to the long thoracic nerve results in a winge reason, it is not stanar to remove (or even check for) sen-
scapula (B). The thoracoorsal nerve courses lateral to the tinel noes in the ipsilateral supraclavicular an internal
long thoracic nerve on the latissimus orsi muscle, following mammary or contralateral lymph noe stations. The network
the course of the subscapular artery. It innervates the latis- of lymphatics that rains the subareolar region is calle Sap-
simus orsi muscle. The latissimus orsi muscle aucts, pey, an it is important because this is the principle behin
extens, an meially rotates the upper arm. Injury to this subareolar injection of blue ye an raiocolloi for sentinel
nerve generally oes not cause a major isability, but it can lymph noe mapping. Batson plexus is instea a network of
lea to ifculty in arm auction an meial rotation. Fur- venous rainage that is thought to be a route for metastasis
thermore, preservation of this nerve an vessels is important to the spine (E).
if a subsequent latissimus orsi ap is being consiere. The
meial pectoral nerve runs lateral to or through the pectoral 15. E. Gynecomastia is an asymptomatic conition result-
minor muscle, actually lateral to the lateral pectoral nerve, ing from the abnormal benign proliferation of glanular
with both innervating the pectoralis minor an major mus- breast tissue concentrically behin the nipple areolar com-
cles. Injury to the meial pectoral nerve may lea to atrophy plex in men. It is not consiere a risk factor for breast can-
of the clavicular portion of the pectoralis muscles, resulting cer (A). After examination, it is not uncommon to n that
in atrophy of the pectoralis muscle (E). The anterior branches most patients, in fact, have pseuogynecomastia, which is
of the supraclavicular nerve are sensory nerves that supply a an accumulation of subareolar fat without a proliferation in
limite area of skin over the upper aspect of the breast, an glanular tissue (D). There are three stages where gyneco-
therefore injury woul not result in a motor ecit (D). mastia is more common - infancy, puberty, an after age 50.
The stimulation of breast growth is attribute to an imbal-
13. A. Monor isease is a thrombophlebitis involving one ance of the effects of estrogen versus testosterone. Oler
or more of the supercial anterior chest wall veins (lateral patients are more vulnerable to this imbalance an thus
thoracic vein, thoracoepigastric vein, or the supercial epi- up to 70% of patients oler than 50 have senescent gyne-
gastric vein). Similar to supercial thrombophlebitis that comastia (C). Spironolactone increases the metabolism an
CHAPtEr 12 Breast 169

clearance of testosterone; marijuana alters the hypothalam- cyst (or perhaps an enlargement of a ifferent nearby cyst) is
ic-pituitary-gonaal axis; uremia relate to ESRD causes pro- common after aspiration, an no feature preicts high or low
longe half-life of luteinizing hormone (LH), which leas to risk of cyst recurrence (C). However, if a cyst recurs within
ecrease secretion of LH an ecrease testosterone levels;  weeks of the aspiration proceure, this shoul spark suspi-
an cimetiine increases plasma prolactin levels, all of which cion an consieration for biopsy.
are well-escribe causes of gynecomastia. It is also associ-
ate with PPI. Alcohol is a risk factor for both gynecomastia 18. B. Nipple ischarge is consiere “pathologic” if it is
an breast cancer (B). Mammography is excellent in ifferen- serous or blooy in color, unilateral, emanating from a sin-
tiating true gynecomastia from malignant isease with a sen- gle uct only, copious in amount, or spontaneous. When a
sitivity an specicity exceeing 90%. However, the positive woman experiences pathologic ischarge after the age of
preictive value for cancer is low, as woul be expecte with 50, it is particularly more worrisome. Brown, green, white
such a low incience of malignancy in this patient popula- (milky), yellow, an blue ischarge is more commonly
tion. If the patient is bothere by the appearance of gyneco- “physiologic” an can usually be expresse from multiple
mastia, antiestrogens such as tamoxifen are frequently use ucts an/or bilaterally on examination. Blooy an serous
with success. Rarely, patients will require surgical removal. types shoul raise concern for malignancy. Breast-feeing
References: Johnson RE, Mura MH. Gynecomastia: patho- women can commonly have bloo-tinge milk in the rst
physiology, evaluation, an management. Mayo Clin Proc. weeks of pregnancy. This conition requires only observa-
009;84(11):1010–1015. tion, as it is most often self-limite. In the case of pathologic
ischarge, mammography an breast ultrasoun shoul be
16. E. Nonlactational breast infections preominantly performe in an attempt to ientify an occult malignancy
occur when there is an obstruction or pseuo obstruction of causing the ischarge. If negative, uctal excision is rec-
the lactiferous uct. The most common organism remains ommene as both iagnostic an therapeutic. Malignant
Staphylococcus aureus. Trauma to the nipple, which inclues lesions are foun in fewer than 10% of cases. From this list
the placement of nipple rings, causes scarring an obstruc- of patients, choice B is the most suspicious for breast cancer
tion (A). Granulomatous mastitis is an inammatory lesion (A, C–E).
of the breast, which may be autoimmune in nature, but is
often recurrent an associate with superinfections of the 19. C. Tamoxifen is a selective estrogen-receptor mou-
inammatory mass (B). Smoking causes a change in the epi- lator (SERM) that acts competitively at the estrogen recep-
thelium of the breast uct (keratinizing squamous metapla- tor to halt cell ivision. Inications for its use are to reuce
sia) that leas to keratin plugs that obstruct the ucts (C). cancer risk in high-risk patients an as a cancer therapy in
Hiraenitis is a skin infection that is cause by obstruction men an women with estrogen receptor-positive noninva-
of the apocrine sweat glans, which often occurs in the peri- sive an invasive breast cancer (D). It is also consiere
areolar, axillary, an inframammary regions of the breast (D). benecial in women who suffer from cyclical mastalgia who
Alcohol oes not have a known irect association with breast have severe symptoms that have faile other measures. In
abscess. the NSABP-P01 trial, high-risk patients (5-year Gail risk
>1.67% or lobular carcinoma in situ [LCIS]) experience a
17. A. Breast cysts are overwhelmingly a benign entity, 50% risk reuction in subsequent noninvasive an inva-
occurring most frequently in women between the ages of 35 sive breast cancers. A 90% risk reuction is associate with
an 50. The typical presentation is that of a painful smooth, prophylactic mastectomy, not tamoxifen (A). The ecision
mobile rm mass that often uctuates in size accoring to to give tamoxifen must always weigh the possible benet
the timing of a woman’s menstrual cycle. The exact etiology against the potential sie effects. Tamoxifen is associate
is largely unknown, but it is clear that hormones play a role with the evelopment of enometrial aenocarcinoma an
in the course of isease. Breast cysts largely isappear after with an increase risk of venous thromboembolism an cat-
menopause, so the presence of a cyst in a postmenopausal aract formation (B). It shoul be aministere for 10-years
woman shoul raise concern. The vast majority of breast following surgical resection in premenopausal women with
cysts are terme “simple cysts” an o not require any breast cancer (C). If a woman is pregnant, tamoxifen therapy
action at all. The presence of a simple cyst oes not elevate shoul be halte to avoi fetal exposure (can cause craniofa-
an iniviual’s risk of subsequent breast cancer. Aspiration cial malformations an ambiguous genitalia) an reinitiate
is primarily recommene if a woman is symptomatic, or if after pregnancy an lactation (E).
the cyst was inavertently iscovere on mammographic
imaging, an the sonographer cannot enitively etermine 20. A. On ultrasoun, lesions that are anechoic are ui
a sonographic lesion to be cystic or concorant with the lle (i.e., cysts), an lesions that are hypoechoic are soli (C).
mammographic abnormality. Though most ui aspirate Benign an malignant masses can appear hypoechoic, but hav-
from breast cysts is straw-colore an watery, a viscous gel- ing a homogeneous internal structure is a benign characteris-
like aspirate is common an not worrisome unless it con- tic (B, D). Bilateral ege shaowing is also a typically benign
tains bloo (B, E). There is no nee to sen cyst aspirate for ning on ultrasoun as echoes are eecte off a smooth-
cytologic evaluation unless it is blooy. In the case of blooy borere roune mass an appear as ark shaows below
aspirate, core neele biopsy shoul also be performe on each ege of a lesion (E). Taller-than-wie measurements
the cyst wall. If suspicious features such as intracystic sep- enote a lesion that is inltrative of the natural elements of
tations, thickene walls, an intracystic mass are present, the breast, which run parallel to the chest wall. Lesions that
these cysts are calle “complicate cysts,” an core neele are benign are typically wier-than-tall an grow along the
biopsy is recommene (D). Recurrence of a simple breast natural elements of the breast.
170 PArt i Patient Care

21. A. Atypical uctal hyperplasia, along with atypical lob- can then lea to cancer formation. A common example is
ular hyperplasia an at epithelial hyperplasia, is classie the MSH2 an MLH1 associate with Lynch synrome (D).
as a “proliferative lesion with atypia.” As such, it is associ- Tyrosine kinases an cyclin-epenent kinases are groups of
ate with up to ve times higher relative risk of breast cancer enzymes that are important for cell regulation an play key
than normal breast tissue. The risk is higher with multifocal roles in evelopment of many cancers (B, E).
lesions. Stromal brosis an apocrine metaplasia o not have
an increase risk for breast cancer an thus o not nee any 24. A. Invasive lobular cancers comprise only 15% of all
aitional workup (D). Though ADH is a benign iagnosis, invasive breast cancers an arise from the terminal lobular
it is morphologically similar to low-grae uctal carcino- components of the lactiferous ucts (E). These cancers are
ma-in-situ an must be less than  mm in size to be terme typically hormone receptor positive an ten to occur in
ADH. Surgical excision is recommene if ADH is iag- postmenopausal women (B). Histologically, lobular cancers
nose on core biopsy because of reporte rates of upstaging grow in a linear pattern inltrating between tissue planes
(ning cancer) of 0% to 30% (C). It is not important to have rather than istorting them. This growth pattern explains
a negative margin, but excision of the initial abnormal area why lobular cancer can be very iniscrete on mammogram
must be containe in the surgical specimen for pathologic an ultrasoun (poorly ene borers) (D). Although it can
evaluation (B). Incorrect targeting (i.e., not seeing the clip on be ifcult to etermine the extent of the lesion an mas-
specimen raiograph) shoul spark consieration for retar- tectomy is often recommene for lobular cancers, breast
geting an reexcision. Tamoxifen is a stanar recommena- conservation is an acceptable option an is associate with
tion after excisional biopsy conrms the absence of cancer. low rates of recurrence (C). Pleomorphic LCIS is an aggres-
Patients enrolle in the NSABP P-01 trial were ranomize sive form of LCIS an has higher chance of having unerly-
to tamoxifen versus placebo, an those who took tamoxifen ing lobular carcinoma. Nonpleomorphic LCIS has a higher
ha a 50% reuction in subsequent invasive an noninva- chance of harboring uctal carcinoma.
sive carcinoma of the breast. Axillary staging is not inicate
given that ADH is benign, an bilateral prophylactic mastec- 25. D. Mastitis commonly complicates lactation an is
tomy shoul not be recommene for this relatively low-risk characterize by erythema, warmth, an tenerness of the
lesion (E). breast. It can often be associate with fever an malaise. The
majority of patients present without an associate abscess.
22. D. From the NSABP B-17 trial, lumpectomy plus raio- The etiology is thought to be ue to bacteria ascening in
therapy was establishe as superior to lumpectomy alone, the uctal tree of the breast through the nipple, couple
given the signicant reuction of ipsilateral breast tumor with relative milk stasis from intermittent clogging of ucts
recurrence rates with the aition of raiotherapy (A, C). an long intervals between feeings. The initial treatment
Sentinel lymph noe biopsy is not absolutely inicate for inclues the aministration of antibiotics covering S. aureus
DCIS when performing lumpectomy. However, it is often (icloxacillin), hot compresses with breast massage, an
performe in cases of high-grae DCIS, an in those with continuation of breastfeeing or pumping to evacuate static
suspicion for microinvasion, mass present on imaging, an/ milk (B). Han evacuation may be necessary if the breast is
or negative hormone receptors, to reuce the nee for sec- too tener to allow feeing or pumping. Incision an rain-
on surgeries if occult invasive isease is foun within the age or percutaneous aspiration are usually not warrante in
specimen. Aitionally, in most cases of lumpectomy (an the absence of a clear area of uctuance or a ui collection
all cases of mastectomy), the lymphatic rainage has been seen on ultrasoun (C). Mammography is typically not help-
remove along with the tumor, an thus sampling the sen- ful in the workup of mastitis, often resulting in false-positive
tinel noe woul be impossible. Planne axillary issection nings (mass, skin thickening). However, if symptoms an
(as part of a moie raical mastectomy) woul not be signs of reness an skin thickening persist, mammography
inicate for in situ isease (B). Intraoperative raiotherapy an core neele biopsy with or without skin biopsy shoul
is now consiere appropriate accoring to ASTRO guie- be performe to rule out inammatory breast carcinoma
lines for partial breast irraiation for “low-risk” DCIS that is (A,E).
.5 cm or smaller, low or intermeiate grae, an with mar-
gins greater than 3 (yes, three) mm. However, this scenario 26. A. The escribe lesion is a phylloes tumor, also
escribes a larger lesion that is high-grae an is therefore historically referre to as cystosarcoma phylloes. These
not appropriate for IORT (E) tumors are rare, accounting for fewer than 1% of breast neo-
plasms, an consist of both an epithelial component an a
23. C. BRCA1 an BRCA2 are examples of tumor suppres- cellular, spinle cell stromal component that forms a charac-
sor genes, which normally regulate an inhibit growth of teristic leaike structure (hence the term phyllodes). They are
abnormal cells. A mutation in both copies of a tumor sup- preominantly benign, but borerline malignant an malig-
pressor gene such as the BRCA gene (usually one inherite nant variants occur in up to 40% of cases. Phylloes tumors
an one acquire) leas to loss of this protective function typically occur in women uring the fth ecae of life an
an unregulate growth of abnormal cells goes unchecke. commonly present as a fast-growing, rm, mobile mass in
On the other han, protooncogenes (such as ras) typically the breast. At large sizes, the contours of the tumor are often
coe for proteins that stimulate cell growth, an mutations visible beneath a thin stretche layer of skin, an the size an
in these genes lea to upregulate cell ivision an therefore weight of the tumor cause the breast to take on the shape of
cancer (A). Mismatch repair genes coe for proteins that rec- a “tearrop.” On imaging, phylloes tumors appear similar
ognize DNA errors an repair them, making them a type of to broaenomas, with istinct well-circumscribe margins
tumor suppressor gene. Mutations in mismatch repair genes an macrolobulations (B). Core neele biopsy is the stanar
CHAPtEr 12 Breast 171

for obtaining a tissue iagnosis, particularly in a woman Cawson JN, Malara F, Kavanagh A, Hill P, Balasubramanium G,
over 40 years of age. However, benign phylloes tumors can Henerson M. Fourteen-gauge neele core biopsy of mammograph-
still be ifcult to istinguish from broaenoma with core ically evient raial scars: is excision necessary?: is excision neces-
sampling alone, most often being reporte as a “broepi- sary? Cancer. 003;97():345–351.
Fasih T, Jain M, Shrimankar J, Staunton M, Hubbar J, Grifth
thelial lesion,” which require excision in orer to make the
CDM. All raial scars/complex sclerosing lesions seen on breast
iagnosis. Distinguishing features of benign phylloes from screening mammograms shoul be excise. Eur J Surg Oncol.
broaenoma are largely base on stromal hypercellularity 005;31(10):115–118.
an morphology. Recent stuies suggest that the best way to Jacobs TW, Byrne C, Colitz G, Connolly JL, Schnitt SJ. Raial
istinguish the two lesions is by the proportion of iniviual scars in benign breast-biopsy specimens an the risk of breast cancer.
long spinle nuclei (>30% is reliable for phylloes tumors) N Engl J Med. 1999;340(6):430–436.
ami isperse stromal cells. Excision with a clear margin of
breast tissue is the treatment of choice for the vast majority 28. B. Intrauctal papilloma is a benign intraepithelial
of phylloes tumors, even malignant ones as long as a mar- tumor of the breast uctal tissues. When it occurs as a single
gin greater than 1 cm is achievable (E). For larger, borerline centrally locate lesion, it is classie as a nonproliferative
an malignant lesions, mastectomy may be require, but this lesion of the breast an confers no subsequent increase risk
is not common. Borerline malignant an malignant forms of breast cancer (aenosis, brosis, an squamous/apocrine
of the isease are associate with high local recurrence rates metaplasia are other examples). A papilloma can grow as
an metastasis via a hematogenous route, most commonly to large as a few centimeters in iameter an most commonly
the lungs. Therefore, sentinel noe biopsy an axillary is- presents with spontaneous, unilateral blooy or serosan-
section are not inicate, given that phylloes tumors very guinous nipple ischarge (C). It is the most common cause
rarely metastasize to lymph noes (C). Raiotherapy is not of blooy ischarge, which oes not increase the likelihoo
generally use after lumpectomy (as it is in breast cancer) of associate malignancy (A). When excise, no margin is
since phylloes are most often benign an, even in malignant require (D). Intrauctal papillomas may be localize to the
variants, raiotherapy has questionable benet (D). Chemo- periphery, sparing the main uct; these are often multiple
therapy has not been proven effective with these tumors an an are associate with a small increase in subsequent breast
is typically not recommene. cancer risk. Although invasive an noninvasive carcinomas
References: Chen WH, Cheng SP, Tzen CY, et al. Surgical treat- must be rule out with iagnostic mammogram an focuse
ment of phylloes tumors of the breast: retrospective review of 17 ultrasoun examination (even with a negative mammogra-
cases. J Surg Oncol. 005;91(3):185–194. phy), malignancy accounts for fewer than 10% of cases of
Krishnamurthy S, Ashfaq R, Shin HJ, Sneige N. Distinction of blooy nipple ischarge (E).
phylloes tumor from broaenoma: a reappraisal of an ol prob-
lem. Cancer. 000;90(6):34–349. 29. D. It is important to note that mammography alone is
insufcient to etermine whether to perform further iagnos-
27. D. Raial scars (RSs) (<1 cm) an complex sclerosing tic workup of a palpable breast mass. Reportely up to 10%
lesions (CSLs) (>1 cm) are, in an of themselves, benign an of palpable malignancies can be misse if reliant only on the
are classie as proliferative lesions without atypia (papil- results of a mammogram as a result of varying breast en-
lomatosis an sclerosing aenosis are two other examples). sity because some breast cancers can be mammogram occult.
As such, they are associate with a milly increase risk of Therefore, choosing to observe, reevaluate or repeat the mam-
subsequent breast cancer (1.5– times normal). These lesions mogram in 3 months woul elay the iagnosis of a possible
can mimic carcinomas of the breast on mammography given malignancy (A, B). Orering aitional breast imaging is the
their stellate appearance. However, presence of a translucent stanar approach when there is a palpable ning an the
central area of fat within the lesion is the classical ning on initial mammogram is negative. Focuse breast ultrasoun is
imaging. Although these lesions have a specic appearance the recommene stuy to further assess the palpable area.
on mammography, core neele biopsy is necessary to exclue There is no role for MRI at this point given the information
malignancy (A–C). Histologically, RS an CSL are character- provie (C). If the lesion is conrme on ultrasoun an is
ize by a broelastic core from which ucts an lobules rai- soli, core neele biopsy is then inicate. All breast imag-
ate. Though biopsy rarely reveals atypia, carcinoma-in-situ ing reports follow a stanarize reporting system an use a
or invasive cancer, upstaging is not uncommon after exci- well-establishe lexicon of escriptive terms. The Breast Imag-
sion. Therefore, when core biopsy emonstrates RS or CSL, ing Reporting an Data System (BI-RADS) category classi-
excisional biopsy of the entire lesion is generally recom- cation for mammograms uses a 0- to 6-point scale as follows:
mene (E). It is notable that newer stuies have suggeste 0, assessment incomplete an aitional imaging require;
that excisional biopsy may not be necessary in cases where 1, negative; , benign ning; 3, probably benign ning; 4,
vacuum-assiste neele cores provie large volume biopsy suspicious abnormality; 5, highly suspicious of malignancy;
specimens, atypical epithelial hyperplasia is absent, an 6, known biopsy-proven malignancy. Recommenations by
when mammographic nings are consistent with histologic category for nonpalpable nings are as follows: 0, shoul
nings. Regarless, it is important for the raiologist an obtain aitional stuies (such as ultrasonography); 1 an ,
pathologist to alert the surgeon to the presence of an RS ue continue routine screening; 3, short-term follow-up mammo-
to its increase risk of associate an subsequent malignancy. gram in 6 months; 4, perform neele biopsy; 5, biopsy an
References: Alleva DQ, Smetherman DH, Farr GH Jr, Ceer- treatment; 6, continue with treatment plan.
bom GJ. Raial scar of the breast: raiologic-pathologic correlation References: Eberl MM, Fox CH, Ege SB, Carter CA, Mahoney
in  cases. Radiographics. 1999;19 Spec No(suppl_1):S7–S35; iscus- MC. BI-RADS classication for management of abnormal mammo-
sion S36–S37. grams. J Am Board Fam Med. 006;19():161–164.
172 PArt i Patient Care

Kerlikowske K, Smith-Binman R, Ljung BM, Gray D. Evalua- remains controversial. It is generally unerstoo, but not
tion of abnormal mammography results an palpable breast abnor- enitively proven, that smoking is a signicant risk factor
malities. Ann Intern Med. 003;139(4):74–84. an, as such, smoking cessation is usually recommene to
reuce risk (D). Having a history of uctal or lobular atypia
30. C. MRI has very few absolute inications for iagnos- is associate with 3.5 to 5 times increase risk, an a history
tic workup of breast lesions. Perhaps the most establishe of lobular carcinoma-in-situ (LCIS) carries a 7 to 10 times
is evaluating for a primary breast cancer in a patient with increase risk. Most of the remaining risk factors are relate
known noal metastasis an no obvious lesion within the to increase exposure to estrogen an inclue an increase
breast. Having ADH or LCIS in a group of calcications number of menstrual cycles such as young age at menarche,
woul require wire-localize excisional biopsy, an MRI ol age at menopause, an nulliparity.
has no role (A, E). A focus of DCIS requires wie excision,
followe by ajuvant treatments (D). In ense breasts, there 33. A. DCIS is further classie histologically into micro-
coul be a potential role for MRI to assess extent of isease, papillary, papillary, cribriform, soli, an comeo subtypes.
but having “C” as an option woul obviate “D” as a choice. The former three being consiere less aggressive than
MRI oes not have a role in the evaluation of breast pain (B). the latter two. The comeo subtype is consiere the most
aggressive, an because cells turn over more quickly, they
31. C. LCIS is a lobular neoplasia that is noninvasive an can quickly outgrow their bloo supply an the center of the
originates from the terminal lobular region of the lactiferous uct may become plugge with ea cellular ebris, often
ucts. LCIS foun on core-neele biopsy requires lumpec- referre to as comeo necrosis (B–E). Comeo DCIS tens to
tomy or wie-excision to rule out concurrent invasive cancer. also have a higher cytologic grae an is more likely to pro-
Unlike uctal carcinoma in situ, it is often not associate with uce microcalcications that eposit aroun necrotic tissue.
calcications an is instea most often an inciental ning Reference: Nakhlis F, Morrow M. Ductal carcinoma in situ. Surg
on biopsy. There are two subtypes, classic an pleomorphic. Clin North Am. 003;83(4):81–839.
LCIS is not consiere to be a premalignant lesion (i.e., oes
not itself progress to cancer), an therefore wie excision 34. C. Consensus guielines in 016 recommen a -mm
with negative margins is not necessary for classic LCIS (C) margin as aequate for DCIS treate with whole-breast
an neither is raiotherapy. However, negative margins are raiation. This is in contrast to invasive uctal carcinoma
require for pleomorphic LCIS as this form is consiere to where “no ink on tumor” is consiere satisfactory for
be much more aggressive (B). It is a noninvasive lesion an an oncologic resection. Raiotherapy alone or tamoxifen
oes not require noal evaluation (D). It is, however, a marker alone woul be inaequate to treat the above patient (A, B).
for the subsequent evelopment of cancer, most often inva- Sentinel noe woul only be inicate if the lumpectomy
sive, in either the ipsilateral or contralateral breasts. This risk emonstrate invasive cancer or if the patient ha opte
is reportely 7 to 10 times the average woman’s risk, but not for a mastectomy for the inex operation (D). Mastectomy
high enough to warrant bilateral prophylactic mastectomy for positive margins is reserve for cases where multiple
(A). Management of LCIS after excision entails close surveil- margins are positive or when margins remain positive after
lance. Tamoxifen or raloxifene will be offere for 5 years (but multiple reexcisions (E).
not lifelong), but many patients ecline given sie effects (E). Reference: Morrow M, Van Zee KJ, Solin LJ, et al. Society of Sur-
References: Frykberg ER. Lobular carcinoma in situ of the gical Oncology–American Society for Raiation Oncology–Ameri-
breast. Breast J. 1999;5(5):96–303. can Society of Clinical Oncology consensus guieline on margins for
Sonnenfel MR, Frenna TH, Weiner N, Meyer JE. Lobular car- breast-conserving surgery with whole-breast irraiation in uctal
cinoma in situ: mammographic-pathologic correlation of results of carcinoma in situ. Pract Radiat Oncol. 016;6(5):87–95.
neele-irecte biopsy. Radiology. 1991;181():363–367.
35. B. All of the choices provie are inherite isorers
32. E. The most common risk factors for breast cancer are that carry an increase lifetime risk of eveloping breast
female sex, age, family history of breast cancer (specically cancer. Cowen synrome is cause by a mutation in PTEN
a primary relative), genetic mutations (BRCA genes, PALB2, an is characterize by multiple hamartomatous lesions as
p53), personal history of breast cancer, receiving therapeu- well as cancer of the breast, enometrium, kiney, an thy-
tic ose of raiation to chest wall before age 30, prior breast roi (A). Li-Fraumeni synrome is cause by mutations in
biopsy showing uctal or lobular atypia or lobular carci- p53 an is associate with breast cancer, sarcomas, glioblas-
noma-in-situ, obesity, rst pregnancy after age 30, menses toma, an arenocortical cancers. Peutz-Jeghers synrome is
beginning before age 1 or ening after age 55, aily alcohol cause by mutations in STK11 gene an classically is associ-
intake of two rinks or more, smoking, physical inactivity, ate with the presence of hyperpigmente mucocutaneous
an having ense breast tissue on mammography. The high- spots, bowel hamartomas, an cancers of the gastrointestinal
est risk is associate with gene mutation carriers, where life- tract, pancreas, liver, breast, enometrium, an ovary (C).
time risk can be upwars of 80%. In the scenario presente Ataxia-telangiectasia is cause by mutation of the ATM gene
above, the patient’s age oes not particularly put her at risk, an, along with neurologic an vasculocutaneous nings
since most breast cancers occur after the age of 60. A woman for which this isorer is name, it carries an increase risk
in her 40s has a breast cancer risk of 1 in 69 compare to of breast cancer, lymphoma, an leukemia (D). BRCA2 muta-
1 in 9 for a woman in her 60s (A). Having a primary rela- tions are associate with breast, ovarian, fallopian tube, pan-
tive with breast cancer (without associate gene mutation) creas, prostate, an skin (melanoma) cancers (E).
elevates a woman’s personal risk by a factor of  (B). Daily References: Blan KL, Beenken SW, Copelan EM. Breast. In:
alcohol intake of 3 rinks or more increases a woman’s risk Brunicari FC, Anersen DK, Billiar TR, etal., es. Schwartz’s princi-
by 1.5 times (C). The effect of smoking on breast cancer risk ples of surgery. 8th e. New York: McGraw-Hill; 005:453–500.
CHAPtEr 12 Breast 173

Iglehart S, Kaelin C. Breast. In: Townsen CM, Jr, Beauchamp RD, there is insufcient breast tissue to allow for two separate
Evers BM, Mattox KL, es. Sabiston textbook of surgery: the biological wie excisions (E). It is important to note that multifocal can-
basis of modern surgical practice. 17th e. Philaelphia: W.B. Sauners; cers refer to multiple foci of breast cancer in the same qua-
004:867–98. rant an are amenable to breast-conserving therapy. The best
choice is the woman with ipsilateral involve noes because
36. A. Hereitary breast cancers (cause by mutations axillary issection is performe separately an oes not limit
in BRCA, PTEN, ATM, STK11, PALB2, an p53 genes) col-
the ability to perform breast conservation.
lectively account for only 10% of all breast cancers (C),
Reference: Morrow M, Strom EA, Bassett LW, et al. Stanar for
with BRCA mutations accounting for 5% of all hereitary breast conservation therapy in the management of invasive breast
breast cancers. The mutations result in “loss of function” of carcinoma. CA Cancer J Clin. 00;5(5):77–300.
the tumor suppression that BRCA genes normally provie
(B). BRCA1 mutations confer a 55% to 65% lifetime risk for 39. C. Large tumor size, poor histologic grae, an estro-
breast cancer an a 35% to 45% lifetime risk for ovarian can- gen-receptor status can certainly enote a poorer prognosis
cer. BRCA2 mutations confer a lifetime risk of 40% to 55% for but the strongest preictor is the presence of regional meta-
breast cancer an a 15% to 5% lifetime risk for ovarian cancer static isease (A, B, D). Younger patients also ten to have
(D). Breast cancers in women with BRCA1 mutations ten to more aggressive, higher grae, receptor-negative breast
be hormone receptor-negative an are often triple-negative. cancers, but noal status is still more preictive (E). In more
Therefore, the use of tamoxifen as chemoprevention is not recent years, gene expression proles have surpasse noal
generally recommene to reuce risk (D). Prophylactic status in early-stage breast cancer in the ability to preict
bilateral mastectomy is the risk-reucing strategy most rec- cancer recurrence (i.e., nee for systemic therapy).
ommene, resulting in 90% overall risk reuction. Male
breast cancer risk is typically elevate to 100-fol risk in ini- 40. D. Inammatory breast cancer comprises only 1% of
viuals who have BRCA2 mutations, not BRCA1 (E). all breast cancers an is characterize by erythema an skin
Reference: Fosslan VS, Stroop JB, Schwartz RC, Kurtzman SH. eema (calle peau ’orange) that result from malignant
Genetic issues in patients with breast cancer. Surg Oncol Clin N Am. obstruction of subermal lymphatics. It is often mistaken
009;18(1):53–71. initially with mastitis, an failure to respon to conventional
antibiotic therapy is an inication to obtain tissue for analy-
37. E. This question aresses the appropriate ajuvant sis. Absence of a palpable mass is common; therefore, biopsy
therapy for early-stage triple-negative breast cancer. First, shoul be performe of the abnormal skin an abnormal
all patients for whom lumpectomy is performe for invasive lymph noes to conrm the iagnosis. The best prognosis
cancer shoul unergo raiotherapy to reuce local recur- results from early treatment with systemic chemotherapy
rence an achieve a similar survival outcome to mastectomy. (i.e., neoajuvant chemotherapy) followe by either surgery
Secon, hormonal therapy is not inicate in patients who or raiotherapy epening on resectability (A, C). The surgi-
are hormone receptor negative. Chemotherapy is inicate cal therapy of choice is moie raical mastectomy because
for all triple-negative breast cancer because of the more there is no role for sentinel noe biopsy in inammatory can-
aggressive nature of the isease an lack of other systemic cer or in patients with clinically positive noes (B). Concur-
therapy options. Taken together, this patient requires the rent chemotherapy an raiotherapy have been shown to be
last option of raiotherapy an chemotherapy, without hor- inferior to sequential therapy. There is no role to continue
monal therapy (A–D). For triple-negative cancers that are antibiotics because the erythema is ue to inammatory can-
 cm or greater or noe-positive, consieration shoul be cer, not ongoing infection (E). The 5-year survival rate is still
given to neoajuvant chemotherapy. only 30% to 50%.
Reference: Cristofanilli M, Buzar AU, Hortobágyi GN. Upate
38. C. Breast conservation, by way of lumpectomy, can on the management of inammatory breast cancer. Oncologist.
be aequately performe to treat T1 an smaller T breast 003;8():141–148.
cancers as long as ajuvant raiotherapy is aministere
to reuce the risk of local recurrence. Contrainications to 41. B. Batson plexus is a venous network that runs in the
receiving raiotherapy therefore rive the contrainications paravertebral space an rains abominopelvic an thoracic
of having a lumpectomy. Raiotherapy cannot be safely regions. The veins are valveless an therefore have been
aministere in pregnancy, so women who are iagnose implicate in the metastatic sprea of prostate, breast, an
in their rst  semesters are often recommene mastec- colon malignancies to bone, particularly the pelvis, vertebral
tomy (A). Women in their thir trimester can often unergo boies, an skull. The Batson venous plexus also explains
operative therapy an wait until after chilbirth to initiate why patients may have bone metastases without rst having
raiotherapy. Women with locally recurrent breast cancer, pulmonary metastases because tumor cells enter the plexus
where raiation therapy was previously complete, are also an eposit in the vertebrae without rst passing through
typically avise to unergo mastectomy because a secon the lungs (A, C–E).
roun of raiotherapy to the same breast woul excee Reference: Muny GR. Mechanisms of bone metastasis. Cancer.
maximal recommene oses (B). Active connective tissue 1997;80(S8):1546–1556.
isorers, such as scleroerma, may lea to increases in
raiotherapy-relate complications an therefore are con- 42. D. Purple noular lesions occurring on an arm with
siere relative to absolute contrainications to raiation long-staning lympheema present is angiosarcoma, or
(D). Multicentric cancers (being locate in separate quarants lymphangiosarcoma, otherwise referre to as Stewart-Treves
of the breast) can also preclue lumpectomy, particularly if synrome. Classically, the patient has unergone axillary
174 PArt i Patient Care

issection an raiotherapy for cancer treatment an evel- ischarge is rarely present, an cytologic evaluation of uc-
ops lympheema. The local immune response is impaire, tal ui has no role in the iagnostic evaluation since the
allowing for evelopment of this aggressive malignancy Paget cells are within the epithelial layer (C). Paget isease
within the breast or ipsilateral arm. The iagnosis is estab- is a cancerous lesion an is often simultaneously associate
lishe via open biopsy because ne-neele aspiration alone with extensive uctal carcinoma in situ or invasive uctal
may not be sufcient. Characteristic features inclue pleo- cancer extening eep to the visible lesion (D). Paget isease
morphic nuclei, frequent mitosis, necrosis, an stacking up is most commonly treate with mastectomy given the extent
of the enothelial cells lining neoplastic vessels (particularly of the unerlying cancer (E).
with high-grae lesions). The tumor is highly aggressive with Reference: Kollmorgen DR, Varanasi JS, Ege SB, Carson WE
a propensity for early metastasis to the lungs. Treatment con- III. Paget’s isease of the breast: a 33-year experience. J Am Coll Surg.
sists of early wie surgical ebriement, which may require 1998;187():171–177.
amputation of the limb. Prognosis is poor with most patients
surviving less than  years. 45. D. Breast lymphoma is a rare isease. The majority of
References: Heitmann C, Ingianni G. Stewart-Treves syn- cases are B-cell lymphomas, an the most common type is
rome: lymphangiosarcoma following mastectomy. Ann Plast Surg. iffuse large B-cell lymphoma (40%–70%) (A). Breast lym-
000;44(1):7–75. phomas are equally ivie into primary an seconary (B).
Sher T, Hennessy BT, Valero V, et al. Primary angiosarcomas of Treatment epens on whether the lesion is localize or if-
the breast. Cancer. 007;110(1):173–178.
fuse as well as on the grae of lymphoma. With localize an
Vorburger SA, Xing Y, Hunt KK, et al. Angiosarcoma of the
low-grae lymphomas, primary excision may be all that is
breast: angiosarcoma of the breast. Cancer. 005;104(1):68–688.
necessary, while stanar combination therapy with CHOP
43. E. In breast cancer local recurrence, the receipt of prior (cyclophosphamie, oxorubicin, vincristine, prenisone)
raiotherapy preclues the option of breast conservation the along with raiation therapy is recommene for interme-
secon time aroun (B). Therefore, mastectomy is routinely iate- or high-grae lymphoma (C, E). Several stuies have
inicate in local recurrence after lumpectomy an whole note an unusual preilection for istant issemination for
breast raiation. Repeat sentinel noe biopsy has been breast lymphoma to the central nervous system.
shown to have aequate ientication an false-negative References: Brogi E, Harris NL. Lymphomas of the breast:
pathology an clinical behavior. Semin Oncol. 1999;6(3):357–364.
rates, so axillary issection can usually be avoie in this
Wong WW, Schil SE, Halyar MY, Schomberg PJ. Primary
group if the lymph noes remain clinically negative (A). non-Hogkin lymphoma of the breast: the Mayo Clinic Experience. J
Breast cancer recurrence is a ifferent issue than having Surg Oncol. 00;80(1):19–5.
two separate primary breast cancers in one’s lifetime. Typ-
ically, occurrence of cancer within a prior surgical incision 46. C. Herceptin (trastuzumab) is a humanize IgG1 kappa
is inicative of local recurrence, which is more common monoclonal antiboy that selectively bins with high afn-
in BRCA-positive patients. However, bilateral mastectomy ity to the epiermal growth factor receptor  (HER2) protein.
is not inicate but may be consiere if BRCA positive Overexpression of HER2/neu (foun in approximately 15%–
(D). Lastly, the local recurrence occurre while the patient 0% of breast cancers) is associate with a worse prognosis
was taking tamoxifen, so the patient shoul preferably be an an increase risk of recurrence but provies a specic
switche to a ifferent agent. Given her postmenopausal sta- target for the treatment of breast cancer. Trastuzumab is asso-
tus, aromatase inhibitor woul be a better option (C). ciate with cariac failure manifesting as a ecrease left
ventricular ejection fraction (LVEF). Thus, serial assessments
44. B. Paget isease of the breast is a rare cancer of the nip- of the LVEF nee to be performe while aministering trastu-
ple involving intraepithelial invasion of Paget cells (large, zumab, particularly if given in conjunction with other agents
pale vacuolate cells) at the nipple surface causing an eczem- that are cariotoxic (such as anthracyclines) (A, B, D, E).
atoi appearance that is often confuse for a contact erma- Reference: Seiman A, Huis C, Pierri MK, et al. Cariac ys-
titis (A). Failure to respon to topical treatments shoul raise function in the trastuzumab clinical trials experience. J Clin Oncol.
concern for Paget isease, an full-thickness skin punch 00;0(5):115–11.
biopsy (not shave) is the biopsy metho of choice. Nipple
Endocrine Surgery
MICHAEL A. MEDEROS AND JAMES WU 13
ABSITE 99th Percentile High-Yields
I. Thyroi
A. Workup of new thyroi noule
1. Initial workup:
a) TSH (hot noules rarely cancerous)
b) Neck ultrasoun (risk of malignancy estimate by TIRADS or ATA scoring systems)
. Inications for thyroi ne-neele aspirate (FNA) base on size + estimate risk on ultrasoun
a) Do not biopsy lesions less than 1 cm
3. FNAB categorize by Bethesa category
a) Bethesa III & IV are ineterminate thyroi noules; options inclue repeat FNA, observation,
iagnostic lobectomy, or genomic classier testing (Arma, Thyroseq)
b) Genomic classier testing: high NPV, meiocre PPV
B. Papillary an follicular carcinoma consierations
1. Extent of surgery base on size (<1 cm: lobectomy, 1–4 cm: lobe vs total, >4 cm: total)
a) Total thyroiectomy inicate if there are noal metastases, gross extrathyroial extension
. Postoperative raioactive ioine only neee for high-risk isease after total thyroiectomy (noal
metastases, extrathyroial extension)
3. Follicular thyroi carcinoma spreas hematogenously (most common in bone an lung)
C. Meullary thyroi carcinoma consierations
1. Workup inclues: calcitonin, CEA, RET mutation testing
. Rule out pheochromocytoma prior to surgery, check calcium for hyperparathyroiism (MEN A/MEN B)
3. Extent of surgery shoul inclue prophylactic bilateral central neck issection
4. Does not take up raioactive ioine, so raiation therapy an chemotherapy are ineffective
D. Graves isease
1. Treatment options: antithyroi meication, raioactive ioine, total thyroiectomy
. Surgery favore: in association with thyroi noules, or ophthalmic isease (raioactive ioine will worsen
eye isease)
3. Lugol solution or potassium ioie given one week before surgery will ecrease thyroi vascularity
4. Patients can experience postthyroiectomy thyrotoxicosis, treat with beta-blockae preoperatively

II. Parathyroi
A. Hyperparathyroiism
1. Inications for parathyroiectomy for primary hyperparathyroiism
a) Symptomatic isease: osteoporosis or fragility fracture, or kiney stones
b) Asymptomatic isease: age <50, serum Ca >1 mg/L above normal range, vertebral compression
fractures, osteoporosis, silent nephrolithiasis, nephrocalcinosis, urine Ca >400 μg/4 hours AND
high stone risk prole, CrCl less than 60 ml/min
. Shoul rule out familial hypocalciuric hypercalcemia (FHH) with 4-hour urine calcium (high urine
Ca rules this out)
175
176 PArt i Patient Care

3. A chlorie/phosphate ratio of more than 33 an evience of signicant hypercalciuria bolster the iagnosis
4. MEN 1 an MEN A shoul have a 3.5 glan parathyroiectomy with bilateral thymectomy
ue to greater incience of supernumerary an ectopic glans, or total parathyroiectomy with
autotransplantation into forearm
B. Hypercalcemic crisis
1. First-line treatment is IV normal saline ui resuscitation followe by bisphosphonates
. Calcium of 14–15 mg/L (in association with high PTH an palpable neck mass) is highly concerning
for parathyroi carcinoma (rare)
C. Intraoperative consierations
1. Miami criterion for successful parathyroiectomy: >50% rop in PTH at 10 minutes post excision
(check again 0 minutes post excision if criterion not met)
. Location of ectopic superior glans: tracheoesophageal groove & retroesophageal > intrathyroial,
caroti sheath, in cervical thymus
3. Ectopic inferior glans (more variable): thymus > tracheoesophageal groove, intrathyroial
a) Perform cervical thymectomy if unable to locate inferior glan
4. If unable to locate parathyroi, complete operation an perform postoperative localization stuies
5. Correcting hyperparathyroiism oesn’t improve osteoporosis t-score but it ecreases the rate at
which it rops
III. Arenal
A. Incientaloma
1. Majority are benign, nonfunctioning (<10 Hounsel units)
. Perform biochemical workup to rule out functional tumor
a) Cushing: low-ose examethasone suppression test, late-night salivary cortisol, or serum DHEA-S
b) Alosteronoma: plasma alosterone concentration, plasma renin activity (renin-alosterone ratio
>0), BMP to check potassium
c) Pheochromocytoma: 4-hour urine metanephrines
3. If nonfunctional, rule out malignancy
a) Higher malignancy risk: size >4 cm, >30 Hounsel units, heterogeneity, >50% washout
b) Role of FNA for arenal mass is very limite
(1) If (+) cancer history, consier biopsy to rule out arenal metastasis
() Primary arenal cortical carcinoma cannot be iagnose with FNA
(3) FNA of unsuspecte pheochromocytoma can trigger catecholamine surge
c) Consier arenalectomy for lesions >4 to 5 cm (except myelolipoma) or lesions with suspicious
raiographic features regarless of size
B. Cushing isease versus primary hypercortisolism
1. Twenty-four-hour urine cortisol, ACTH level, high-ose examethasone suppression test
. Patients with subclinical hypercortisolism may have normal 4-hour cortisol levels; if there is high
suspicion, procee with low-ose examethasone suppression testing
C. Pheochromocytoma
1. Associate synromes: von Hippel-Linau, MEN , neurobromatosis 1 (von Recklinghausen isease)
. Alpha-blockae prior to initiating beta-blockers
D. Hyperalosteronism (Conn synrome)
1. Hypertension, hypokalemia, alkalosis
. Alosterone to renin ratio ≥30 (90% sensitive), plasma alosterone concentration >10
3. Unilateral aenoma > bilateral arenal hyperplasia; aenomas are usually small
E. Arenal cortical carcinoma consierations
1. High attenuation (>0 Hounsel units), >4 cm, heterogenous appearance on CT
. Often are functional: hypercortisolism, hyperalosteronism, hyperanrogenism
3. Open arenalectomy; laparoscopic arenalectomy is currently contrainicate (higher rate of local
recurrence, poorer isease-free survival)
4. Mitotane for positive margins, vascular or capsular invasion, rupture/spillage, unresectable/
recurrent/metastatic isease (most commonly liver & lung)
5. Associate synrome: Li-Fraumeni (p53)
F. Arenal metastases
1. Lung, kiney, melanoma, breast most common; often bilateral
. Bilateral arenalectomy may benet select patients
a) Evaluate for an correct arenal insufciency prior to bilateral arenalectomy to prevent
perioperative arenal crisis; 30% of patients have entire glan replace with tumor
CHAPtEr 13 Endocrine Surgery 177

IV. Thyroiitis

Thyroiditis Hyper-/hypothyroid Cause & associations Treatment


Hashimoto (chronic Hypothyroid • Autoimmune, (HLA) -DR3, • Levothyroxine
or lymphocytic) May have thyrotoxicosis with -DR5, & -B8 • Thyroidectomy reserved
thyroiditis transient hyperthyroidism • Antithyroid peroxidase for large goiters with
due to gland destruction (anti-TPO) antibodies compressive symptoms
(90%); antithyroglobulin
antibodies (20%–50%)
• Female > male, 30–60 years
old, painless goiter
• Risk of primary thyroid
lymphoma
Postpartum Hyperthyroid with decreased • Autoimmune, may have • No treatment unless
thyroiditis uptake on RAI scan higher antithyroid titers symptoms are severe
due to follicular cell • Painless • Hyperthyroid—beta-
destruction → euthyroid blockers (propranolol if
period → transient breastfeeding)
hypothyroidism → euthyroid • Hypothyroid—
(recovery) levothyroxine
Acute (suppurative) Euthyroid • Bacterial (Staphylococcus • Ultrasound-guided ęne-
thyroiditis aureus and Streptococcus needle aspiration with
pyogenes) > fungal > Gram stain and culture
parasitic to determine etiology,
• Painful antibiotics
Subacute Hyperthyroid with • Preceded by viral URI • Supportive care, NSAIDs,
granulomatous decreased uptake on • Female > male, painful steroids
(de Quervain) RAI scan due to gland goiter, fatigue, weight loss
thyroiditis destruction → euthyroid
period → transient
hypothyroidism → euthyroid
(recovery)
Riedel (ębrous) Hypothyroid • Firm, nontender, extensive • Glucocorticoids,
thyroiditis ębrosis that may compress tamoxifen, levothyroxine
adjacent structures • Surgery reserved for
• May have elevated serum severe compressive
IgG4 symptoms

V. Bethesa Classication

Category Cytopathology Malignancy rate Management


I Nondiagnostic/inadequate 0%–20% Repeat FNA with ultrasound
guidance
II Benign 0%–3% Clinical and sonographic follow-
up
III Atypia of undetermined signięcance/ 5%–15% Repeat FNA, molecular testing,
follicular lesion of undetermined or lobectomy
signięcance (FLUS)
IV Follicular neoplasm/suspicious for 15%–30% Molecular testing and lobectomy
follicular neoplasm,
e.g., Hürthle cell (oncocytic) type
V Suspicious for malignancy 60%–75% Near-total thyroidectomy or
lobectomy
VI Malignant 97%–99% Near-total thyroidectomy or
lobectomy
178 PArt i Patient Care

IV. Hyperparathyroiism

Type Cause Serum Ca/Phos Urine Ca Treatment


Primary Parathyroid adenoma ↑/↓ ↑ (calcium Adenoma—parathyroidectomy of
(85%) or hyperplasia creatinine aěected gland
Lithium-induced clearance ratio Hyperplasia—3.5-gland excision
parathyroid >0.02 suggestive) or 4-gland excision with
hyperplasia autotransplantation
Secondary Chronic renal failure, ↓/↑ Phosphate binders & dietary
hyperphosphatemia modięcation, calcium
Vitamin D deęciency supplementation, vitamin D,
cinacalcet
3.5-gland parathyroidectomy
for calciphylaxis, bone pain,
intractable pruritus, persistent
anemia, or pathologic fractures
Tertiary Persistent ↑/↓ ↑ 3.5-gland parathyroidectomy
hyperfunctioning
parathyroid glands
after kidney
transplantation
Parathyroid ↑↑/↓ ↑ Parathyroidectomy with en bloc
carcinoma ipsilateral thyroidectomy +
isthmusectomy & ipsilateral
central neck dissection (lateral neck
dissection only if evidence of gross
LN involvement). Repeat neck
exploration for recurrent disease.

Fig 13.1 Location of Lost Parathyroi Glans.


CHAPtEr 13 Endocrine Surgery 179

Questions
1. A 55-year-ol male has recalcitrant hypertension 5. Following total thyroiectomy for follicular
espite taking three antihypertensive meications. cancer, a 65-year-ol female presents to the
He is subsequently foun to have an alosterone emergency epartment 4 ays later complaining
to renin ratio greater than 30. A CT reveals a 1-cm of circumoral numbness an tingling of her
left arenal lesion an a -cm right arenal lesion. ngers. Phosphate level is normal. Which of the
What is the next best step in management? following is true about this conition?
A. Right arenalectomy A. It likely represents hungry bone synrome
B. Left arenalectomy (HBS)
C. Bilateral arenalectomy B. It may lea to a shortene QT on ECG
D. Selective venous sampling C. The risk can be reuce by routine
E. 11C-metomiate scan postoperative calcium an vitamin D
supplementation
2. A 5-year-ol obese male who unerwent CT D. Most patients are symptomatic
scan  months ago following a car collision E. It is more common with thyroiectomy for
was incientally foun to have a 3-cm left benign lesions
arenal mass. Which of the following is the next
appropriate step? 6. A 45-year-ol man with episoic severe
A. Laparoscopic left arenalectomy hypertension is foun to have an elevate plasma
B. Surveillance CT performe at 6 months an metanephrine level an a serum calcium level of
then annually for 1 to  years 11.5 mg/L. Which of the following woul be
C. No further testing necessary inicate in the workup?
D. PET CT A. CT scan of the sella turcica
E. Overnight low-ose (1 mg) examethasone B. Calcitonin level
suppression test C. Serum gastrin level
D. Serum prolactin level
3. A 40-year-ol female presents with incientally E. A 4-hour urine cortisol
iscovere mil elevation in serum calcium. She
is otherwise healthy. A PTH level is elevate as 7. Which of the following laboratory nings
well. Both ultrasoun an sestamibi scan of the is characteristically associate with primary
neck are negative. Which of the following is true hyperparathyroiism?
about this conition? A. Elevate serum phosphate
A. It may represent tertiary hyperparathyroiism B. Increase serum chlorie
B. A 4-hour urine calcium is inicate C. Decrease urinary calcium
C. She shoul procee to neck exploration D. Metabolic alkalosis
D. It shoul be treate with cinacalcet E. Elevate calcium with a ecrease PTH
E. Selective venous sampling is inicate
8. A 60-year-ol woman presents with fatigue,
4. A 40-year-ol female presents with incientally weakness an confusion. She has history of
iscovere hypercalcemia to 11.7 mg/L. She is kiney stones an pathologic fractures. On
otherwise healthy. A PTH level is elevate as well. physical she has a palpable neck mass. Her serum
Both ultrasoun an sestamibi scan of the neck calcium level is 14.8 mg/L. The most likely
are negative. Urine calcium is elevate. What is iagnosis is:
the most appropriate next step? A. Parathyroi aenoma
A. Procee to neck exploration B. Parathyroi hyperplasia
B. Treatment with cinacalcet C. Parathyroi cancer
C. Selective venous sampling D. Breast cancer with bone metastasis
D. MRI of the neck E. Seconary hyperparathyroiism
E. Observation
180 PArt i Patient Care

9. During neck exploration for primary 14. The most common pituitary neoplasm associate
hyperparathyroiism, only three parathyroi with MEN 1 secretes:
glans are ientie, all of which appear A. ACTH
normal in size. Which of the following woul be B. Prolactin
appropriate? C. Growth hormone
A. Perform a transcervical thymectomy D. Thyroi-stimulating hormone
B. Remove all three glans an reimplant one in E. Follicle-stimulating hormone
the forearm
C. Remove two an a half glans an then close 15. Which of the following features of Graves isease
D. Perform meian sternotomy to look for ectopic oes not improve with antithyroi therapy?
parathyroi A. Tremor
E. Obtain biopsy samples of all three parathyroi B. Anxiety
glans an then close C. Graves ermopathy
D. Gastrointestinal isturbance
10. After total thyroiectomy an postoperative E. Exophthalmos
ioine ablation for a 5-cm follicular thyroi
cancer, the best test to monitor for recurrent 16. A 56-year-ol male presents with refractory
isease is: hypertension espite being starte on
A. Serum thyroi-stimulating hormone (TSH) hyrochlorothiazie an lisinopril by his primary
B. Serum calcitonin care physician. His bloo pressure is 18/9 mmHg.
C. Serum thyroglobulin Laboratory stuies are remarkable for an
D. 131I scan alosterone-renin ratio of 5 an hypokalemia.
E. Cross-sectional CT or MRI Which of the following is the next best step?
A. Triamterene
11. Which of the following is true regaring arenal B. Amilorie
cortical carcinoma? C. Spironolactone
A. Associate evience of hormonal excess is D. Phenoxybenzamine
common E. Eplerenone
B. The iagnosis is generally mae by CT-guie
neele biopsy 17. A 40-year-ol female presents with a 4 cm thyroi
C. Staging is base on tumor histology noule that is biopsy-proven papillary thyroi
D. Because of malignant potential, arenal carcinoma. The patient is taken to surgery,
masses larger than 3 cm shoul be excise an nal pathologic evaluation reveals a 4-cm
E. Laparoscopic arenalectomy is the preferre papillary thyroi carcinoma with microscopic
approach for surgical resection invasion of the perithyroial tissue, but no
vascular invasion. A 1-cm lymph noe in the
12. Malignancy within a thyroglossal uct cyst is lateral neck is positive. Which of the following
typically: answer choices correctly pairs this patient’s
A. Follicular thyroi American Thyroi Association (ATA) risk
B. Papillary thyroi stratication with her pathologic nings?
C. Squamous cell A. Low risk; no vascular invasion
D. Anaplastic thyroi B. Intermeiate risk; tumor 4 cm
E. Hürthle cells C. Intermeiate risk; positive lymph noe <3 cm
D. High risk; tumor 4 cm
13. After a total thyroiectomy, the right vocal cor is E. High risk; microscopic invasion of
note to be xe in a parameian position. This perithyroial tissue
most likely represents:
A. Injury to the recurrent laryngeal nerve (RLN) 18. A 46-year-ol female with a 3-cm palpable
B. Injury to the internal branch of the superior right-sie thyroi noule has a ne-neele
laryngeal nerve aspirate (FNA) performe, which is reporte as
C. Injury to the external branch of the superior noniagnostic. What is the best next step?
laryngeal nerve A. Repeat FNA
D. Trauma from enotracheal intubation B. Core neele biopsy
E. Compression from hematoma C. Right thyroi lobectomy
D. Total thyroiectomy
E. Ultrasoun in 6 months
CHAPtEr 13 Endocrine Surgery 181

19. A 51-year-ol male with a -cm palpable right- 24. A 45-year-ol male presents with a -ay history
sie thyroi noule has an FNA performe, of nausea, vomiting, an marke abominal
which is reporte as follicular lesion of istention. He has no prior surgical history. Before
unetermine signicance (FLUS). Which of the this, he’s ha watery iarrhea for about a month.
following is true about this conition? On exam, he has iffuse tenerness without
A. Repeat FNA is not recommene reboun or guaring. Compute tomography
B. Molecular testing oes not inuence (CT) scan emonstrates markely ilate loops
management of small bowel with an abrupt transition in the
C. Right thyroi lobectomy is an acceptable mi jejunum with istal collapse. In aition,
option there are two, -cm soli masses in the right
D. Total thyroiectomy is the next best step lobe of the liver. At surgery, at the point of
E. Ultrasoun follow-up in 6 months is the best obstruction, there is a small mass in the mi ileum
option with surrouning brosis, causing tethering
an kinking of the small bowel mesentery. The
20. The thyroi glan is erive from which two lesions in the right lobe of the liver are not
embryologic structure? palpable. A segmental small bowel resection is
A. First pharyngeal arch performe. Aitionally, which of the following
B. Thir pharyngeal pouch is recommene?
C. Thir pharyngeal arch A. Cholecystectomy
D. Fourth pharyngeal pouch B. Ultrasoun-guie liver biopsy
E. Fourth pharyngeal arch C. Liver resection
D. Appenectomy
21. Which of the following cancers most commonly E. No aitional proceure
metastasizes to the thyroi?
A. Parathyroi glan 25. Which of the following is true regaring Hürthle
B. Kiney cell carcinoma?
C. Lung A. It is consiere a subtype of follicular
D. Breast carcinoma
E. Esophagus cancer B. Lymph noe metastasis is exceeingly rare
C. Diagnosis of malignancy is usually mae by
22. Two patients are iagnose with FNA
pheochromocytoma. In one patient, the mass is D. Resiual isease is effectively treate with
locate in the arenal glan an in the other, the ioine 131 (131I)
mass is localize to the organ of Zuckerkanl. E. Histologically they emonstrate Orphan Annie
Which enzyme accounts for the ifference in the cells
serum levels of epinephrine in the two patients?
A. Tyrosine hyroxylase 26. A patient presents with fatigue an bone
B. Dopamine-beta-hyroxylase pain. Serum calcium level is 11.1 mg/L an
C. Phenylethanolamine N-methyltransferase parathyroi hormone (PTH) is elevate. Which is
(PNMT) the least acceptable metho of localization?
D. Dihyroxyphenylalanine (DOPA)- A. Operative exploration
ecarboxylase B. CT scan
E. Catechol-O-methyltransferase (COMT) C. Technetium-99m sestamibi imaging
D. Magnetic resonance imaging (MRI)
23. A 45-year-ol woman with rheumatoi arthritis E. Ultrasoun scan
on chronic sterois has not been able to get a
rell on her meications incluing atenolol, 27. Which of the following is true regaring follicular
methotrexate, an prenisone. She arrives at the thyroi cancer?
emergency epartment with a fever, hypotension, A. It is the most common thyroi malignancy
nausea, an izziness. The next best step is: B. It most commonly spreas via a hematogenous
A. Intravenous (IV) antibiotics route
B. IV hyrocortisone C. Prophylactic noal issection is recommene
C. IV uis D. It is best manage by hemithyroiectomy
D. Aminister oral methotrexate E. Multicentricity is common
E. Complete bloo count, basic metabolic panel,
an cortisol level
182 PArt i Patient Care

28. A 45-year-ol woman presents with symptomatic 33. Which of the following is not an inication
primary hyperparathyroiism. During surgery, for parathyroiectomy in a patient with
it is note that all four glans are markely asymptomatic primary hyperparathyroiism?
enlarge. Which of the following is the best A. Serum calcium of 11.6 mg/L
recommenation? B. Creatinine clearance less than 60 mL/min
A. Removal of three an a half glans for C. Age younger than 50 years
parathyroi hyperplasia, leaving half of a D. Bone ensity at the hip of 1.5 stanar
glan in place eviations below matche controls
B. Removal of all four glans E. High risk of forming kiney stones
C. Terminate the surgery an treat with meical
management 34. A 45-year-ol-woman presents with truncal
D. Biopsy all four glans obesity an hypertension. A 4-hour urine-free
E. Remove one glan, an biopsy the other three cortisol level is markely elevate an a low-
ose examethasone suppression test fails to
29. Which of the following is true regaring suppress the elevate plasma cortisol levels.
laparoscopic arenalectomy? Plasma arenocorticotropic hormone (ACTH)
A. It is the proceure of choice for small levels are also markely elevate. A high-ose
functional aenomas examethasone suppression test also fails to
B. It is contrainicate for pheochromocytoma suppress the urinary-free cortisol level. Which of
C. It is contrainicate for bilateral the following woul most likely emonstrate the
pheochromocytoma cause of her symptoms?
D. It is contrainicate for pheochromocytomas A. CT scan of the sella turcica
larger than 5 cm B. Petrosal sinus sampling for ACTH
E. It is a well-establishe option for malignant C. Chest CT
tumors D. MRI of the sella turcica
E. CT scan of the abomen
30. A 65-year-ol woman with a history of Hashimoto
thyroiitis presents with fever, ysphagia, an 35. Calcie clumps of cells on histology are
a painless thyroi mass that has enlarge over a consistent with:
short perio of time. This most likely represents: A. Papillary cancer
A. Lymphoma B. Hürthle cell cancer
B. Follicular cancer C. Follicular cancer
C. Anaplastic thyroi cancer D. Meullary thyroi cancer
D. Acute suppurative thyroiitis E. Anaplastic cancer
E. Meullary thyroi cancer (MTC)
36. Which of the following is true regaring
31. Which of the following is LEAST likely associate pheochromocytoma?
with hyperparathyroiism? A. Risk of malignancy is higher in patients with
A. Cholelithiasis familial tumors
B. Pancreatitis B. Malignancy is etermine histologically by the
C. Osteoclastomas number of mitoses
D. Diarrhea C. Familial tumors are more likely to be
E. Peptic ulcer isease unilateral
D. Metaioobenzylguaniine scanning
32. Which of the following is true regaring the is useful for localizing extra arenal
parathyroi glans, an/or the location of ectopic pheochromocytomas
superior/inferior glans? E. Urine metanephrine has the highest
A. The inferior glans arise from the fourth sensitivity
branchial pouch an the superior ones from
the thir pouch 37. The most common type of thyroi cancer in
B. The superior glans are more likely to be chilren is:
foun in an ectopic position A. Papillary
C. The superior glans are more likely to be B. Follicular
foun in the thymus C. Meullary
D. Three glans are more common than ve glans D. Hürthle cell
E. Ectopic superior glans are more likely to be E. Anaplastic
foun in the retro- or paraesophageal space
CHAPtEr 13 Endocrine Surgery 183

38. Which of the following is true regaring 43. A 45-year-ol woman with a history of a goiter
seconary hyperparathyroiism? presents to the emergency epartment with a
A. Serum calcium levels are markely increase high fever, heart rate of 130 beats per minute,
B. It is usually associate with a parathyroi tremors, sweating, an exophthalmos. Which of
aenoma the following can exacerbate symptoms?
C. PTH levels are typically normal A. Aspirin
D. Can be cause by severe vitamin D eciency B. Propylthiouracil
E. Most patients will eventually require C. Beta-blocker
parathyroiectomy D. Methimazole
E. Sterois
39. Which of the following is true regaring tertiary
hyperparathyroiism? 44. Which of the following is true regaring
A. It is usually ue to an unerlying parathyroi substernal goiter?
carcinoma A. Surgical resection shoul be reserve for
B. It is most commonly seen after successful patients with tracheal eviation
kiney transplantation B. Most are primary meiastinal goiters with
C. The serum calcium level is usually normal or a bloo supply arising from intrathoracic
low vessels
D. Distinguishing between seconary an tertiary C. Most can be resecte by a cervical incision
hyperparathyroiism is essential because the D. Most are highly responsive to prolonge
management iffers thyroi suppression
E. It only occurs in patients with chronic renal E. Because of the risk of tracheomalacia,
insufciency most patients shoul have a prophylactic
tracheostomy at the time of resection
40. The most common extra arenal site of
pheochromocytoma is the: 45. The most common cause of primary arenal
A. Rectum insufciency in the Unite States is:
B. Blaer A. Autoimmune
C. Neck B. Tuberculosis
D. Organ of Zuckerkanl C. Metastatic isease
E. Sacrum D. Arenal hemorrhage
E. Exogenous steroi use
41. Which of the following is true regaring
neuroblastoma? 46. The most common cause of Cushing
A. It is the thir most common abominal synrome asie from exogenous corticosteroi
malignancy in chilren aministration is:
B. Prognosis is better for oler chilren than A. Arenal cortical carcinoma
those iagnose before 1 year of age B. Arenal aenoma
C. It is associate with aniriia an C. Corticotropin (ACTH)-proucing pituitary
hemihypertrophy aenoma
D. In the meiastinum, they are most often D. Ectopic ACTH synrome
locate anteriorly E. Ectopic corticotropin-releasing hormone
E. Amplication of the N-myc oncogene has an synrome
unfavorable prognosis
47. Which of the following is true regaring the
42. During thyroiectomy, the superior thyroi renin-angiotensin system?
arteries were ligate a centimeter away from A. The juxtaglomerular cells are locate within
the thyroi capsule as oppose to immeiately the renal efferent arteriole
ajacent to it. This technical error woul B. The juxtaglomerular cells secrete alosterone
most likely result in which of the following in response to ecrease bloo pressure
complications? C. The juxtaglomerular cells etect changes in
A. Loss of voice projection chlorie concentration in the renal tubule
B. Loss of airway D. Renin catalyzes the conversion of
C. Hoarseness angiotensinogen to angiotensin I
D. Aspiration E. Angiotensin I irectly stimulates the
E. Ineffective cough prouction of alosterone
184 PArt i Patient Care

48. Which of the following is true regaring the 51. A 38-year-ol female with stage  chronic
anatomy/bloo supply to the arenal glans? kiney isease is iagnose with primary
A. Venous rainage has more anatomic variability hyperparathyroiism. Preoperative localization
than arterial bloo supply stuies inicate a single enlarge left inferior
B. Catheter-base venous hormonal sampling is parathyroi glan. She unergoes minimally
easier to perform on the right arenal vein invasive single glan parathyroiectomy uner
C. On the right, the arenal vein rains into the local anesthesia. An enlarge glan is ientie
right renal vein an remove. Intraoperative PTH levels are sent
D. Right arenalectomy is more likely to lea 10 minutes later, an a 40% rop in PTH from
to life-threatening hemorrhage than left baseline is note. Which of the following is true?
arenalectomy A. One shoul procee to four-glan exploration
E. The majority of the arterial bloo supply arises B. Repeat PTH level shoul be obtaine at
from the celiac trunk 0minutes
C. It is acceptable to close the woun
49. A 70-year-ol man is foun to have an inciental D. The vein from where the PTH was sample
mass in his right arenal glan on CT scan. He oes not affect PTH ecline
has no history of malignancy an has a normal E. The PTH ecline is affecte by the patient’s
bloo pressure. The nings of the remainer kiney isease
of the history an physical examination are
negative. Plasma free metanephrines are negative. 52. The hallmark of multiple enocrine neoplasia
The serum potassium level is normal. Urinary type  (MEN ) is:
free cortisol is normal, an a 1-mg overnight A. Unilateral pheochromocytoma
examethasone suppression test shows a low B. Bilateral pheochromocytoma
cortisol level (1.5 μg/L) the following morning. C. Meullary carcinoma of the thyroi
The mass is 4.5 cm on CT scan, has smooth D. Menin mutation
borers, an has a low attenuation value. Which E. Four-glan parathyroi hyperplasia
of the following is true regaring this conition?
A. The patient shoul unergo a CT-guie 53. The most common cause of congenital arenal
neele biopsy hyperplasia is:
B. The patient shoul unergo a laparoscopic A. 11β-Hyroxylase eciency
arenalectomy B. 3-Hyroxyehyrogenase eciency
C. The patient shoul unergo an open C. 1-Hyroxylase eciency
arenalectomy D. 17-Hyroxylase eciency
D. A repeat CT scan shoul be performe in E. Congenital arenal lipoi hyperplasia
6months
E. The mass is most likely malignant 54. A 45-year-ol man with a history of primary
hyperparathyroiism presents for an enlarge
50. Which of the following is true regaring the thyroi noule. Further workup reveals an
histology of the arenal glan? elevate calcitonin level. Which of the following
A. The zona glomerulosa is the inner layer of the is true regaring the most likely conition?
arenal cortex A. Bilateral prophylactic central noe issection
B. Cells in the zona fasciculata prouce cortisol is inicate in aition to total thyroiectomy
C. Cells in the zona reticularis prouce B. Raiotherapy is an effective treatment
alosterone moality
D. Meullary cells are chromafn negative C. Plasma or urine metanephrines o not nee to
E. The zona reticularis is the mile layer of the be checke prior to intervention
arenal cortex D. The likelihoo of noal metastases is low
E. Chemotherapy is effective for resiual isease

55. The most accurate test for hyperthyroiism is:


A. Free thyroxine (T4)
B. Total T4
C. Total triioothyronine (T3)
D. Thyroi-stimulating hormone (TSH)
E. Thyroi scan
CHAPtEr 13 Endocrine Surgery 185

56. Which of the following is true regaring the bloo 59. Which of the following is a irect effect of
supply to the thyroi/parathyroi glans? parathyroi hormone?
A. The parathyroi glans are usually supplie A. Stimulates hyroxylation of cholecalciferol in
by the superior thyroi arteries the kiney
B. The inferior thyroi artery is the rst branch of B. Stimulates reabsorption of phosphate by the
the external caroti artery kiney
C. The RLNs are at risk of injury uring ligation C. Stimulates reabsorption of bicarbonate by the
of the superior thyroi arteries kiney
D. The external branch of the superior laryngeal D. Stimulates absorption of calcium by the small
nerve is at risk of injury when the inferior intestine
laryngeal arteries are ligate E. Stimulates hyroxylation of
E. The thyroi ima artery usually arises from the 5-hyroxyvitamin D in the kiney
aorta
60. Lateral aberrant thyroi in most instances
57. Which of the following is true regaring the represents:
laryngeal nerves? A. Metastatic papillary carcinoma
A. The external branch of the superior laryngeal B. Metastatic follicular carcinoma
nerve provies sensation to the larynx C. Metastatic Hürthle cell carcinoma
B. Bilateral injury to the superior laryngeal nerves D. A congenital lesion relate to thyroi escent
often results in acute airway obstruction E. An extension of a thyroglossal uct cyst
C. The right RLN separates from the vagus after
crossing the subclavian artery 61. A 45-year-ol woman presents with a 1.5-cm right
D. The recurrent laryngeal nerve is both motor thyroi noule. FNA nings are consistent with
an sensory to the larynx papillary carcinoma. Her history is signicant
E. The RLNs provie motor function to the for raiation therapy for lymphoma as a chil.
cricothyroi Optimal management of this patient woul
consist of:
58. A nonrecurrent laryngeal nerve: A. Right hemithyroiectomy
A. Does not exist B. Right hemithyroiectomy plus central lymph
B. Is more common on the left noe issection
C. Can occur in conjunction with a recurrent C. Total thyroiectomy
nerve on the right D. Total thyroiectomy plus right moie
D. Loops aroun the aorta on the right sie raical neck issection
E. Is less prone to injury uring surgery than a E. Total thyroiectomy with postoperative 131I
recurrent nerve

Answers
1. D. Primary hyperalosteronism is seconary to the arenalectomy without further attempts at localizing the
release of excess alosterone from one or both arenal source (C). Functional nuclear meicine stuies can also
glans. The iagnosis is mae biochemically, ieally after ai with lateralization but is typically performe if venous
iscontinuation of antihypertensives, with an alosterone sampling is unsuccessful (E). Further, if this patient’s hyper-
to renin ration of 0 to 30. Once a biochemical iagnosis is alosteronism is ue to hyperplasia, it woul be treate
mae, a thin-cut arenal CT shoul be the initial metho of meically.
localization. In the case of a unilateral arenal lesion, some Reference: Yeh MW, Livhits MJ, Duh Q. The arenal glans. In:
surgeons avocate proceeing with arenalectomy if the Townsen CM Jr, Beauchamp RD, Evers BM, Mattox KL. Sabiston
lesion is >1 cm an the contralateral arenal glan is nor- textbook of surgery: the biological basis of modern surgical practice. 0th e.
mal on CT. Alternatively, some surgeons recommen rou- Elsevier; 016:963–995.
tine arenal venous sampling in most patients, especially
those oler than 40 years ol as they are more likely to have 2. E. Incientalomas are iscovere in 1% to 4% of imaging
nonfunctioning arenal aenomas. In the setting of bilateral stuies that are evaluating an unrelate issue. The majority
aenomas on CT, arenal venous sampling shoul be per- of incientalomas are nonfunctioning aenomas (60%). The
forme (A, B). It woul be inappropriate to perform bilateral remaining tumors in a patient that o not have a history of
186 PArt i Patient Care

malignancy inclue pheochromocytoma, cortisol-proucing neck exploration for primary hyperthyroiism an have
aenoma, alosteronoma, arenocortical carcinoma, an persistent or recurrent hyperparathyroiism pose a more
myelolipoma. There shoul be a high level of suspicion for ifcult scenario. These patients may benet from invasive
arenal metastasis in a patient with a history of malignancy localization via venous sampling prior to remeial neck
an/or bilateral lesions. All arenal incientalomas shoul exploration (C). MRI is rarely inicate as primary imaging
unergo biochemical testing to evaluate for subclinical for localization. It is reserve patients who are not cani-
Cushing synrome, pheochromocytoma, an alosteronoma ates for other imaging (e.g., pregnant patients). It may be
(E). A functional incientaloma is an inication for arenal- useful in the setting of a surgical reexploration of the neck.
ectomy. For patients with negative biochemical testing an However, this patient has not ha a previous exploration
size <3 cm can effectively be monitore with surveillance an alreay has negative imaging with moalities that carry
cross sectional imaging to evaluate for growth (B). Annual greater sensitivity an specicity than MRI (D). Cinacalcet is
biochemical testing is often performe as well for up to 5 inicate for patients with seconary hyperparathyroiism
years. Inications for arenalectomy for nonfunctioning inci- ue to chronic kiney isease (B).
entalomas inclue size >5 cm. For patients with 3 to 5 cm Reference: Wilhelm SM, Wang TS, Ruan DT, et al. The Amer-
nonfunctioning incientalomas, arenalectomy can be con- ican Association of Enocrine Surgeons guielines for eni-
siere for patients with few surgical risk factors an those tive management of primary hyperparathyroiism. JAMA Surg.
with concerning raiographic features (irregular borers, 016;151(10):959–968.
central necrosis, high vascularity, an internal calcications)
(A). PET CT is not part of the initial workup (D). No further 5. C. Transient hypocalcemia following thyroiectomy is a
testing woul be incorrect (C). known complication an can occur in % to 53% of patients
Reference: Yeh MW, Livhits MJ, Duh Q. The arenal glans. In: unergoing total thyroiectomy. The etiology is likely mul-
Townsen CM Jr, Beauchamp RD, Evers BM, Mattox KL. Sabiston tifactorial an inclues reversible ischemia to the parathy-
textbook of surgery: the biological basis of modern surgical practice. 0th e. roi glans, hypothermia to the glans, an enothelin-1
Elsevier; 016:963–995. release (known to suppress PTH prouction). Aitionally,
iatrogenic removal of one or several parathyroi glans is
3. B. Surgery is inicate in asymptomatic patients uner possible uring thyroiectomy an can contribute to postop-
the age of 50 that are suspecte to have primary hyperpara- erative hypocalcemia. Patients with hypocalcemia can pres-
thyroiism. Familial hypocalciuric hypercalcemia (FHH) ent with neuromuscular excitability, tetany (Chvostek sign),
causes mil increase in serum calcium an can initially circumoral paresthesia, seizures, QT prolongation on ECG,
be misiagnose as primary hyperparathyroiism. It is a an cariac arrest (B). However, most patients with transient
benign conition ue to mutations in CASR, which encoes hypocalcemia following thyroi surgery are asymptomatic
a calcium receptor. The lack of calcium signal increases the (D). Inepenent preictors of hypocalcemia following thy-
PTH level, which increases renal calcium reabsorption. Thus, roiectomy inclue low postoperative PTH level, female
part of the workup of primary hyperparathyroiism is to gener, an patients with a malignant neoplasm (E). Several
obtain a 4-hour urine calcium. Hypercalciuria with a high stuies have emonstrate that the routine use of postoper-
PTH level an high serum calcium level conrms primary ative aministration of calcium an vitamin D can reuce
hyperparathyroiism. A low urine calcium level suggests the incience an/or severity of hypocalcemia. HBS is
FHH. Once FHH is rule out, four-glan neck exploration extremely rare. It has also been propose as a possible con-
can be performe without the nee for further imaging (C). tributing factor but occurs more frequently after parathyroi
Tertiary hyperparathyroiism typically occurs in patients surgery. However, similar to PTH, thyroi hormone can also
with renal failure, most of whom have unergone kiney provie a stimulus to break own bone, an once this stim-
transplantation (A). Cinacalcet is inicate for patients with ulus is remove, the bones attempt to eplete their calcium
seconary hyperparathyroiism (D). Selective venous sam- by removing it from serum, which can lea to HBS. This typ-
pling is an invasive proceure that is inicate in patients ically presents with hypophosphatemia an hypomagnese-
with recurrent hyperparathyroiism, when other forms of mia an is usually seen in patients with severe preoperative
imaging fail to ientify the abnormal glan (E). bone isease (A).
References: Alhefhi A, Mazeh H, Chen H. Role of postoper-
ative vitamin D an/or calcium routine supplementation in pre-
4. A. Noninvasive localization stuies shoul always be
venting hypocalcemia after thyroiectomy: a systematic review an
employe before taking a patient to surgery for primary
meta-analysis. Oncologist. 013;18(5):533–54.
hyperparathyroiism. Inications for parathyroiectomy in Groski S, Serpell J. Evience for the role of perioperative PTH
the asymptomatic patient inclue serum calcium >1 mg/L measurement after total thyroiectomy as a preictor of hypocalce-
above normal, age <50, evience of en-organ ysfunction mia. World J Surg. 008;3(7):1367–1373.
(ecrease creatinine clearance or low bone ensity). In this
young patient, with hypercalcemia greater than 1 mg/L 6. B. The elevate plasma metanephrine inicates a high
above normal, surgery shoul be offere (E). However, it suspicion for pheochromocytoma. Further workup for this
is not uncommon for patients to have negative noninvasive shoul inclue a CT or MRI scan of the abomen to etect an
localization stuies. In a patient who has not ha previous arenal mass. The elevate calcium suggests hyperparathy-
neck exploration, he/she can be taken to surgery for para- roiism. The patient shoul have a PTH level measure an,
thyroi exploration. Ultimately, patients with negative if it is elevate, shoul unergo a sestamibi scan. Given these
imaging remain caniates for parathyroiectomy given the nings, the patient most likely has MEN type , which is
high rate of false-negative imaging. Patients with negative characterize by pheochromocytoma, hyperparathyroiism,
noninvasive localization stuies who have ha a previous an MTC. Screening for MTC involves measuring the serum
CHAPtEr 13 Endocrine Surgery 187

calcitonin level. MEN type 1 is characterize by hyperpara- this setting is intraoperative gamma probe etection. Like-
thyroiism, pituitary tumor, an pancreatic tumors. CT of wise, intraoperative PTH assays can assist in etermining
the sella turcica may be use to look for a pituitary tumor whether the pathologic glan has been remove. Ectopic
such as prolactinoma (A). An elevate prolactin level will parathyroi glans are only rarely foun in the meiasti-
also support a iagnosis of prolactinoma (D). Elevate gas- num, so a meian sternotomy is not recommene unless all
trin level is associate with gastrinoma (C). A 4-hour urine other options are explore (D). Biopsy may result in ischemia
cortisol level can be use in the workup for Cushing syn- of the parathyroi glans (E).
rome (E).
10. C. Serum thyroglobulin levels are the most useful
7. B. PTH inhibits phosphate reabsorption at the proximal moality to monitor patients for recurrence of ifferentiate
convolute tubule, thereby lowering phosphate levels (A). It thyroi cancer (papillary an follicular) after total thyroi-
also inhibits the Na+/H+ antiporter. This leas to an inhibi- ectomy an raioactive ioine ablation. Thyroglobulin is a
tion of bicarbonate excretion in the urine, resulting in a mil glycoprotein that is the primary component of colloi matrix
metabolic aciosis an corresponing hyperchloremia (D). within the thyroi follicle. Thyroglobulin levels in patients
This subsequently results in an elevate chlorie-to-phos- who have unergone total thyroiectomy shoul be 3 ng/mL
phate ratio (>33). PTH levels are increase (E). Hypercalce- or less when the patient is receiving thyroi hormone replace-
mia typically results in hypercalciuria, with the exception ment therapy an less than 5 ng/mL when thyroi hormone
being in patients with familial hypocalciuric hypercalcemia supplementation is withhel. Serum thyroglobulin levels
(C). seem to be most preictive of recurrence when patients are
hypothyroi as ocumente by a high TSH level (A). An
8. C. Parathyroi carcinoma is extremely rare an accounts increase above these levels is highly suggestive of meta-
for less than 1% of cases of primary hyperparathyroiism. static isease. The recommenation after thyroiectomy is to
It shoul be suspecte in the setting of severe symptoms of check thyroglobulin levels initially at 6-month intervals after
hypercalcemia, in association with very high serum calcium surgery. If the thyroglobulin levels are elevate, an 131I scan is
(usually 14.6–15.0 mg/L) an PTH, history of kiney stones recommene (D). Recurrence of MTC is etermine by cal-
an pathologic fractures, an a palpable neck mass (A, B). citonin levels (B). Routine cross-sectional surveillance imag-
Benign causes of hyperparathyroiism very rarely result in a ing via CT or MRI is not currently recommene. Perioic
palpable neck mass an are less likely to cause a hypercalce- ultrasoun in aition to thyroglobulin is recommene by
mic crisis. Determination of malignancy is ifcult because, the NCCN for select patients with a high risk for recurrence
similar to other enocrine malignancies, there are not any (E).
classic histologic features that reliably istinguish parathy- References: Bauin E, Do Cao C, Cailleux AF, et al. Positive
roi malignancy from benign isease. Thus, one must look preictive value of serum thyroglobulin levels, measure uring the
for evience of local invasion at the time of surgery as well rst year of follow-up after thyroi hormone withrawal, in thyroi
as enlarge lymph noes. Treatment is surgical an involves cancer patients. J Clin Endocrinol Metab. 003;88(3):1107–1111.
Duren M, Siperstein AE, Shen W, et al. Value of stimulate
en bloc resection of the parathyroi tumor with the ipsilat-
serum thyroglobulin levels for etecting persistent or recurrent if-
eral thyroi glan, as well as a moie raical lymph noe
ferentiate thyroi cancer in high- an low-risk patients. Surgery.
issection if noal metastasis is present. Recently, cinacalcet 1999;16(1):13–19.
was approve by the US Foo an Drug Aministration an Lal G, Clark OH. Thyroi, parathyroi an arenal. In: Bruni-
is effective in controlling the hypercalcemia associate with cari FC, Anersen DK, Billiar TR, etal., es. Schwartz’s principles of
parathyroi carcinoma. Breast cancer with bone metastasis surgery. 8th e. New York: McGraw-Hill; 005:1395–1470.
may be associate with a paraneoplastic synrome in which
a high level of PTH-relate protein is foun. This is unlikely 11. A. Arenocortical carcinomas are rare. They shoul
to present with a palpable neck mass (D). Seconary hyper- be suspecte in the presence of large tumors (>5–6 cm) or
parathyroiism is associate with a low level of serum cal- if the CT scan shows evience of necrosis, hemorrhage, or
cium (E). local invasion. Approximately 60% of patients with areno-
Reference: Shane E. Parathyroi carcinoma. J Clin Endocrinol cortical carcinoma present with hormonal excess, incluing
Metab. 001;86():485–493. Cushing synrome an virilization. There are no istinctive
Sharretts JM, Kebebew E, Simons WF. Parathyroi cancer. Semin histologic or cytologic features that istinguish arenocorti-
Oncol. 010;37(6):580–590. cal carcinoma from an aenoma (C). Thus, one must rely on
evience of local invasion, lymph noe metastasis, or istant
9. A. On occasion, espite careful neck exploration, only metastasis. CT-guie neele biopsy is not recommene
three parathyroi glans will be encountere. A careful (B). The best chance for cure is surgical resection. Open are-
search for the ectopic glan shoul be conucte (B, C). The nalectomy is the stanar of care for surgical resection for
inferior glans are more likely to be ectopic than the superior arenal cortical carcinoma as the laparoscopic approach is
ones. Most inferior glans are to be foun within  cm of the associate with higher local recurrence rates an poorer is-
inferior thyroi pole. If not foun, the next step is to perform ease-free survival (E). Arenal masses that are hormonally
a cervical thymectomy an sen the tissue for frozen section. active shoul be excise. In the absence of hormonal activity
If still glans are not foun, the caroti sheath shoul be an in the absence of CT scan features suggestive of malig-
opene. Intraoperative ultrasonography shoul then be use nancy, resection is recommene for asymptomatic masses if
to etermine whether there is an intrathyroial parathyroi they are larger than 5 to 6 cm (D).
glan. If ultrasonography is not available, ipsilateral thyroi Reference: Ng L, Libertino JM. Arenocortical carcinoma: iag-
lobectomy shoul be consiere. Another useful moality in nosis, evaluation an treatment. J Urol. Publishe online 003:5–11.
188 PArt i Patient Care

12. B. The frequency of thyroi carcinoma among patients hyperalosteronism results from autonomous alosterone
with a surgically remove thyroglossal uct cyst in one large secretion, which, in turn, leas to suppression of renin secre-
series was 0.7%. The majority is papillary cancer that is foun tion. The iagnosis is mae by emonstrating a combination
incientally after a Sistrunk proceure (performe for the of inappropriate potassium excretion in the urine (kaliuresis),
cyst) (A, C–E). If iscovere incientally, the patient shoul low plasma renin, an a high alosterone-to-renin ratio (>0).
subsequently unergo a total thyroiectomy because ai- While it was previously believe that an arenal aenoma
tional cancer is usually foun within the thyroi glan as well. (Conn synrome) was the most common cause of primary
Reference: Heshmati HM, Fatourechi V, van Heeren JA, Hay hyperalosteronism, we now know that nearly 60% of cases
ID, Goellner JR. Thyroglossal uct carcinoma: report of 1 cases. are ue to iiopathic bilateral arenal hyperplasia (IBAH).
Mayo Clin Proc. 1997;7(4):315–319. It is important to clearly establish the etiology because the
management is ifferent. An arenal aenoma shoul be
13. A. The RLN innervates the intrinsic muscles of the lar- remove with a unilateral arenalectomy but IBAH is man-
ynx, except the cricothyroi muscles, which are innervate age with meical therapy alone using a mineralocorticoi
by the external branch of the superior laryngeal nerve (C). replacement such as spironolactone or eplerenone. Ami-
The internal branch of the superior laryngeal nerve provies lorie an triamterene are also potassium-sparing iuret-
sensory input for the pharynx (B). Injury to one RLN leas to ics but are less optimal (A, B). A ouble-blin ranomize
paralysis of the ipsilateral vocal cor. The cor becomes xe controlle stuy emonstrate the superiority of spironolac-
in either the parameian position or the abucte position. tone in controlling hypertension compare with eplerenone
If the cor becomes xe in the parameian position, the (E). Bilateral arenalectomy is consiere in cases of severe
patient will have a weak voice, whereas if it becomes xe refractory hypertension. However, this has a high risk of
in the abucte position, the patient will have a hoarse voice complications an will subject the patient to lifelong epen-
an an ineffective cough. If both RLNs are injure, an airway ence of mineralocorticois (urocortisone) an sterois.
obstruction may evelop acutely in the patient. Trauma from Phenoxybenzamine is an alpha-1 receptor antagonist use
enotracheal intubation or compression from hematoma in the preoperative management of pheochromocytoma (D).
oes not typically cause vocal cor paralysis (D, E). References: Kaplan NM. The current epiemic of primary alo-
steronism: causes an consequences. J Hypertens. 004;(5):863–869.
14. B. Pituitary tumors are the thir most common tumors Stowasser M. Upate in primary alosteronism. J Clin Endocrinol
in MEN 1. The majority are prolactinomas (A, C–E). They Metab. 009;94(10):363–3630.
may cause bitemporal hemianopsia ue to local compres- Parthasarathy HK, Ménar J, White WB, et al. A ouble-blin,
sion of the optic chiasm resulting in loss of peripheral vision ranomize stuy comparing the antihypertensive effect of eplere-
or may lea to amenorrhea an galactorrhea in women or none an spironolactone in patients with hypertension an evience
hypogonaism in men. Women are more likely to present of primary alosteronism. J Hypertens. 011;9(5):980–990.
early in the course of the isease as they are more likely to
have hormonal symptoms. Men typically present later with 17. C. The AJCC/TNM staging system oes not aequately
mass-effect of the tumor (visual changes, heaaches, etc.). preict the risk of recurrence in ifferentiate thyroi can-
cer. Thus, the ATA evelope a 3-tiere clinic-pathologic risk
15. E. Graves isease is the most common cause of hyper- stratication for recurrence in 009 with moications in 015.
thyroiism in the Unite States an is ue to antiboies For papillary thyroi carcinoma, low-risk patients inclue
targeting thyrotropin receptors, which increase prouction those having intrathyroial tumors without extrathyroial
of thyroi hormone. Patients present with anxiety, rapi or extension, vascular invasion, metastases, aggressive histology,
irregular heart rate, heat intolerance, weight loss, thinning an clinical N0 or ≤5 N1 micrometastases (<0. cm in largest
hair, ecrease libio, iarrhea, thick an shiny skin (Graves imension). Intermeiate-risk patients inclue those with
ermopathy), an exophthalmos. The preferre therapy is microscopic invasion into the perithyroial tissue, aggressive
raioactive ioine ablation, but meical therapy with prop- histology, ascular invasion, an clinical N1 or >5 pathologic
ylthiouracil (PTU) or methimazole is also available. Exoph- N1 noes with all involve noes <3 cm in largest imen-
thalmos evelops in about 10% of patients an is the only sion. High-risk patients are those with macroscopic invasion
symptom that is resistant to antithyroi therapy an even of perithyroial tissue, incomplete tumor resection, istant
worsens after raioactive ioine ablation (A–D). Some stu- metastases, an pathologic N1 isease with any noe >3 cm
ies suggest that the use of prenisone before antithyroi in largest imension. This patient has intermeiate isease
therapy can help improve exophthalmos. base on  factors: microinvasion into perithyroial tissue an
References: Bartalena L, Marcocci C, Bogazzi F, et al. Relation a metastatic lymph noe <3 cm (C). Macroinvasion or incom-
between therapy for hyperthyroiism an the course of Graves’ plete resection is high-risk, not microinvasion (E). Tumor size
ophthalmopathy. N Engl J Med. 1998;338():73–78. is not a component of the ATA risk stratication system (B, D).
Shiber S, Stiebel-Kalish H, Shimon I, Grossman A, Robenshtok E. The absence of vascular invasion is a low-risk feature, but this
Glucocorticoi regimens for prevention of Graves’ ophthalmopathy patient ha other factors that make her intermeiate risk (A).
progression following raioioine treatment: systematic review an Reference: Haugen BR, Alexaner EK, Bible KC, et al. 015
meta-analysis. Thyroid. 014;4(10):1515–153. American Thyroi Association management guielines for ault
Stein JD, Chilers D, Gupta S. Risk factors for eveloping thy- patients with thyroi noules an ifferentiate thyroi cancer: the
roi-associate ophthalmopathy among iniviuals with Graves’ American Thyroi Association Guielines Task Force on Thyroi
isease. JAMA. 015;133(3):90–96. Noules an Differentiate Thyroi Cancer. Thyroid. 016;6(1):1–133.

16. C. Primary hyperalosteronism shoul be suspecte 18. A. The most important test in the evaluation of a sol-
in patients with hypertension an hypokalemia. Primary itary thyroi noule is FNA. This can be performe with
CHAPtEr 13 Endocrine Surgery 189

ultrasoun guiance if the lesion is ifcult to palpate. References: Nakhjavani MK, Gharib H, Goellner JR, van
Before the routine use of FNA, there was a high rate of Heeren JA. Metastasis to the thyroi glan. A report of 43 cases.
benign thyroi surgical resections. With current practice, the Cancer. 1997;79(3):574–578.
percentage of thyroi noules resecte that are foun to be Stevens TM, Richars AT, Bewtra C, Sharma P. Tumors metastatic
to thyroi neoplasms: a case report an review of the literature.
malignant is over 50%. The Bethesa system for reporting
Patholog Res Int. 011;011:38693.
thyroi cytopathology classies noules into six groups:
(1) noniagnostic or unsatisfactory, () benign, (3) atypia of
22. D. The synthesis of catecholamines is a complex process
unetermine signicance or follicular lesion of uneter-
an is governe by various enzymes. Tyrosine hyroxylase
mine signicance, (4) follicular neoplasm or suspicious for
is consiere the rate-limiting step an converts L-tyrosine
a follicular neoplasm, (5) suspicious for malignancy, an (6)
to L-opa, which is then converte to opamine by opa-e-
malignant. Patients with a noniagnostic or unsatisfactory
carboxylase (A). Dopamine is converte to norepinephrine
FNA shoul have a repeat FNA performe (B–E).
by opamine-beta-hyroxylase an norepinephrine is con-
References: Cibas ES, Ali SZ, NCI Thyroi FNA State of the Sci-
verte to epinephrine by PNMT (B, C). COMT metabolizes
ence Conference. The Bethesa System for Reporting Thyroi Cyto-
pathology. Am J Clin Pathol. 009;13(5):658–665.
both norepinephrine an epinephrine (E). With the excep-
Yassa L, Cibas ES, Benson CB, et al. Long-term assessment of a tion of PNMT, all the other enzymes have the name of the
multiisciplinary approach to thyroi noule iagnostic evaluation. precursor as part of their nomenclature, which allows for an
Cancer. 007;111(6):508–516. easy way to remember the key steps. PNMT is rarely pres-
ent outsie of the arenal meulla, which accounts for why
19. C. FNA results are classie into six ifferent groups extra-arenal pheochromocytomas o not synthesize a high
base on the Bethesa criteria. The management of FNA that level of norepinephrine. The brain stem, retina, an cariac
is reporte as FLUS is somewhat controversial. The current tissue may also contain PNMT.
recommenation is to perform a repeat FNA (A). The risk Reference: Ziegler MG, Bao X, Kenney BP, Joyner A, Enns
of malignancy of FLUS has historically been aroun 5% to R. Location, evelopment, control, an function of extraare-
15%. However, more recent series have foun a malignancy nal phenylethanolamine N-methyltransferase. Ann N Y Acad Sci.
rate closer to 30%. These authors recommen proceeing to 00;971(1):76–8.
thyroi lobectomy. Thus, the ecision as to whether to repeat
the FNA or procee to thyroi lobectomy epens on patient 23. C. Fever, hypotension, nausea, an izziness in a
risk factors for malignancy; the institutional rate of malig- patient taking chronic sterois that suenly stoppe taking
nancy with FLUS; ultrasoun features of the lesion; an all meications shoul raise concern for acute arenal insuf-
more recently, molecular testing (not always available an ciency. When the iagnosis is suspecte, treatment shoul
expensive) (B, E). Follicular neoplasms will require a surgical begin immeiately before conrmatory tests become avail-
lobectomy, an FNA emonstrating malignancy or suspicion able (E). Initial treatment consists of IV normal saline volume
for a malignant process will require a total thyroiectomy resuscitation. This is then followe by either aministration
(D). Core neele biopsy has been propose as an aitional of 4 mg of examethasone or 100 mg of hyrocortisone (B).
ajunctive tool, particularly in cases of noniagnostic FNA Dexamethasone is preferre because it will not interfere with
but there have not been any conclusive stuies to emon- cosyntropin stimulation testing, which shoul be one the
strate its usefulness, nor is it consiere the current stanar next morning to conrm the iagnosis. IV antibiotics are not
of care. It may be consiere for patients that are hesitant to use in acute arenal insufciency (A). Cessation of metho-
procee with surgical resection. trexate oes not present with the aforementione symptoms
References: Cibas ES, Ali SZ, NCI Thyroi FNA State of the Sci- (D).
ence Conference. The Bethesa System for Reporting Thyroi Cyto-
pathology. Am J Clin Pathol. 009;13(5):658–665. 24. A. This patient has metastatic migut neuroenocrine
Yoon JH, Kim EK, Kwak JY, Moon HJ. Effectiveness an lim- tumor (NET). The ning of brosis an tethering of the
itations of core neele biopsy in the iagnosis of thyroi noules: mesentery is highly suggestive of a carcinoi tumor. The
review of current literature. J Pathol Transl Med. 015;49(3):30–35. accompanying iarrhea, combine with likely liver metasta-
sis, is highly suggestive of carcinoi synrome. Compare to
20. A. The thyroi glan is one of the earliest enocrine the foregut, migut, an hingut, NETs have a greater 5-year
glans to evelop. It arises from the rst an secon pha- survival rate. Chemotherapy has not been shown to have a
ryngeal arches. The superior parathyroi glan evelops signicant role in increasing isease-free survival. Symptom
from the fourth pharyngeal pouch while the inferior para- control is achieve with somatostatin analogs such as oct-
thyroi glan evelops from the thir pharyngeal pouch (B, reotie. Some of the few accepte lifelong inications for
D). An easy way to remember this is that the “Parathyroi the use of octreotie, enorse by the American Association
erives from the Pouch.” The thir pharyngeal arch helps in of Oncology, inclue patients with peptie/amine-inuce
the evelopment of the stylopharyngeus muscle while the synromes with clinical symptoms an for patients with
fourth pharyngeal arch allows for the evelopment of the progression of metastatic isease even without a synrome.
cricothyroi muscle (C, E). This patient will require postoperative octreotie, given
his history of watery iarrhea. Octreotie promotes biliary
21. B. The most common primary tumor to metastasize to sluging an leas to a high rate of symptomatic cholelithia-
the thyroi is renal cell carcinoma. Other primary cancers that sis an as such, cholecystectomy is recommene at the time
metastasize to the thyroi glan, in escening orer, inclue of surgery (E). This inication becomes stronger in patients
lung, breast, an esophageal cancer (C–E). Parathyroi glan that are planne to unergo hepatic artery embolization sec-
carcinoma oes not metastasize to the thyroi glan (A). onary to metastasis to the liver. Liver biopsy or resection is
190 PArt i Patient Care

not appropriate uring an emergency surgery (particularly of capsular or vascular invasion on histology is necessary.
when the lesion is not reaily palpable) an his isease is Thus, if FNA emonstrates a follicular neoplasm, the patient
likely amenable to less morbi proceures such as raiofre- shoul unergo a thyroi lobectomy to etermine malig-
quency ablation an/or hepatic artery embolization (B, C). nancy. Once histologic conrmation of malignancy is mae,
There is no inication to perform an appenectomy in the total thyroiectomy is recommene with or without post-
above patient (D). operative 131I. Total thyroiectomy also permits the etection
Reference: Öberg K, Kvols L, Caplin M, et al. Consensus report of subsequent metastasis using nuclear scanning (D). Post-
on the use of somatostatin analogs for the management of neuro- operative raioactive ioine following total thyroiectomy
enocrine tumors of the gastroenteropancreatic system. Ann Oncol. is inicate for all tumors larger than 4 cm, gross extrathy-
004;15(6):966–973. roial extension of the tumor regarless of size, lymph noe
metastases, an for high-risk features incluing tall-cell or
25. A. Hürthle cell carcinoma accounts for less than 10% of columnar-cell variant (D). An ae avantage of postoper-
thyroi malignancies an is consiere a subtype of follicu- ative raiation is that it allows for continue monitoring for
lar cancer. Like follicular cancer, the presence of malignancy recurrence with thyroglobulin. Prophylactic noal issection
is establishe by the emonstration of vascular or capsular is not require (C).
invasion. FNA an frozen section o not reliably establish
malignancy (C). The tumors contain sheets of eosinophilic
28. A. Surgical management of a solitary parathyroi
cells packe with mitochonria, which are erive from
aenoma consists of resection of the single enlarge glan.
oncocytic or oxyphilic cells of the thyroi glan. Hürthle cell
After resection of the caniate parathyroi thyroi glan
carcinomas iffer from follicular cell carcinomas in that they
for aenoma, intraoperative parathyroi hormone (PTH) is
are often multifocal an bilateral, are more likely to metasta-
routinely measure to ensure an appropriate rop post
size to local noes an istant sites, an are associate with
resection. On rare occasions, ouble aenomas are present.
a higher mortality rate (B). Resiual isease is not effectively
For four-glan hyperplasia, resection of 3.5 glans is rec-
treate with raioactive ioine because Hürthle cell carci-
ommene. Alternatively, resection of all four glans with
nomas o not take up raioactive ioine (D). Orphan Annie
reimplantation of half of one glan into the brachioraialis
cells are a hallmark of papillary carcinoma (E). Unlike if-
muscle in the forearm can be performe. Removing all four
ferentiate thyroi cancer, noal metastases preict a worse
glans without reimplantation increases the risk for hypo-
outcome in wiely invasive Hürthle cell carcinoma, as oes
parathyroiism (B). Meical management is not appropriate
extrathyroial extension.
for primary hyperparathyroiism (C). On occasion, istin-
Reference: Stojainovic A, Ghossein RA, Hoos A, et al. Hürthle
guishing between aenoma an hyperplasia may be ifcult
cell carcinoma: a critical histopathologic appraisal. J Clin Oncol.
001;19(10):616–65.
if two glans are enlarge an the other two appear normal
or slightly enlarge. In this circumstance, removal of the two
26. D. Technetium-99m sestamibi imaging is the most enlarge glans an biopsy of an aitional glan may be
wiely use an accurate moality, with sensitivity greater performe to rule out four-glan hyperplasia. However, in
than 80% for etection of parathyroi aenomas. High-res- the presence of one enlarge glan, there is no role for biopsy
olution ultrasonography in particular is complementary. of the other three glans because this may result in ischemia
The other imaging techniques are thought to be more use- of the remaining parathyroi glans (D, E). Another frequent
ful when sestamibi scanning fails to ientify the parathyroi ilemma occurs when only three glans are foun, an all
pathology, for the workup of recurrent hyperparathyroi- appear normal. If an stuies unless other methos are contrain-
ism, or when surgical exploration fails to ientify the para- icate inferior one is missing, it may be foun in the thymus,
thyroi lesion (A, C). Ultrasonography has an overall lower angle of the manible, at the skull base, superior to the supe-
sensitivity, although it may be most useful in ientifying rior parathyroi glans, or, rarely, within the thyroi glan.
intrathyroial parathyrois (E). Some institutions utilize 4-D If the ectopic glan is not foun, transcervical thymectomy
parathyroi protocol CT, which has emonstrate higher is recommene. If the superior glan is missing, it may be
sensitivities than sestamibi an neck ultrasoun (B). The foun within the thyroi glan, in the paraesophageal or retro-
combination of 4-D CT an ultrasoun has emonstrate a esophageal grooves, or caual to the inferior glans. Although
sensitivity of 94% an specicity of 96% for localizing hyper- ectopic glans are foun in the meiastinum on rare occasion,
functioning parathyroi glans. MRI generally not use in meian sternotomy is not recommene at initial exploration.
localization stuies unless other methos are contraini- Reference: Quinn CE, Uelsman R. The parathyroi glans. In:
Townsen CM Jr, Beauchamp RD, Evers BM, Mattox KL, es. Sabis-
cate (e.g., pregnancy).
ton textbook of surgery: the biological basis of modern surgical practice.
Reference: Quinn CE, Uelsman R. The parathyroi glans. In:
0th e. Elsevier; 016:45–55.
Townsen CM Jr, Beauchamp RD, Evers BM, Mattox KL, es. Sabis-
ton textbook of surgery: the biological basis of modern surgical practice.
0th e. Elsevier; 016:1–136. 29. A. Laparoscopic arenalectomy has become the pro-
ceure of choice for small- an meium-size functional
27. B. Follicular cancer is the secon most common thyroi an benign arenal tumors. Pheochromocytoma is not a
cancer, an it spreas primarily via a hematogenous route contrainication to the laparoscopic approach an may be
with the lung as its primary site of metastasis (A). Multi- use successfully for unilateral or bilateral tumors (B, C).
centricity is uncommon (E). Unlike papillary carcinoma, accu- Tumor size alone is not a contrainication to the laparoscopic
rate iagnosis using FNA is not possible because cytologic approach. For a large tumor that is clearly malignant base
features cannot istinguish a benign follicular lesion from a on CT scan evience of local invasion or lymph noe metas-
follicular carcinoma. To establish malignancy, emonstration tasis, the laparoscopic approach is contrainicate (E). Open
CHAPtEr 13 Endocrine Surgery 191

arenalectomy is preferre for pheochromocytomas larger the superior glans are more likely to be foun in the retro-
than 6 cm. (D). or paraesophageal position (B, C). Given the longer escent
References: Assalia A, Gagner M. Laparoscopic arenalectomy. of the inferior glans, they are overall much more likely to be
Br J Surg. 004;91(10):159–174. in an ectopic position.
Brunt LM, Moley JF. In: Townsen CM, Jr, Beauchamp RD, Evers
BM, Mattox KL, es. Sabiston textbook of surgery: the biological basis 33. D. Patients with symptomatic hyperparathyroiism
of modern surgical practice. 17th e. Philaelphia: W.B. Sauners; shoul unergo surgery. Symptoms are ene as having
004:103–1070.
evience of kiney stones; neuromuscular, neuropsycho-
Laimore TC, Moley JF. The multiple enocrine neoplasia syn-
logical, or bone symptoms; hypercalcemic crisis; or a history
romes. In: Townsen CM, Jr, Beauchamp RD, Evers BM, Mattox
KL, es. Sabiston textbook of surgery: the biological basis of modern sur-
of pancreatitis or peptic ulcer (E). Conversely, controversy
gical practice. 17th e. Philaelphia: W.B. Sauners; 004:1071–1090. exists as to whether every patient who is asymptomatic
Lal G, Clark OH. Thyroi, parathyroi an arenal. In: Bruni- shoul unergo parathyroiectomy. Natural history stuies
cari FC, Anersen DK, Billiar TR, etal., es. Schwartz’s principles of of patients with asymptomatic hyperparathyroiism inicate
surgery. 8th e. New York: McGraw-Hill; 005:1395–1470. that one-fourth to one-thir of patients without symptoms
will progress to the evelopment of symptoms over 15 years.
30. A. Hashimoto thyroiitis is an autoimmune isorer Current guielines for surgery in asymptomatic patients
that leas to estruction of thyroi follicles by both cell- an inclue at initial evaluation: age less than 50, serum calcium
antiboy-meiate immune processes, incluing activation level more than 1 mg/L above the upper limit of reference
of helper lymphocytes an antiboy formation against thy- value, reuce creatinine clearance (<60 mL/min), evience
roglobulin an thyroi peroxiase. It is the leaing cause of renal stones or nephrocalcinosis, evience of bone mass
of hypothyroiism an most commonly affects young reuction more than .5 stanar eviations below matche
females. It results in a lymphocytic inltration. Treatment of controls, fragility fractures, an unwillingness or inability to
Hashimoto thyroiitis is with thyroi hormone replacement. unergo continue follow-up (A–C, E).
Hashimoto thyroiitis is associate with primary thyroi Reference: Quinn CE, Uelsman R. The parathyroi glans. In:
lymphoma. The chronic antigenic stimulation couple with Townsen CM Jr, Beauchamp RD, Evers BM, Mattox KL, es. Sabis-
a chronic proliferation of lymphoi tissue in the thyroi is ton textbook of surgery: the biological basis of modern surgical practice.
thought to lea to the evelopment of lymphocytic transfor- 0th e. Elsevier; 016:78–9.
mation. In a patient with Hashimoto thyroiitis, lymphoma
shoul be suspecte in the setting of a rapily enlarging 34. C. Measurement of elevate 4-hour urinary cortisol
thyroi mass (B–E). Patients aitionally may report fever, levels is a very sensitive (95%–100%) an specic (98%)
cervical lymphaenopathy, ysphagia, an hoarseness. moality for iagnosing Cushing synrome, an as such
FNA may suggest the iagnosis, but ow cytometry (with it shoul be the rst test use to establish the iagnosis of
core neele biopsy) is require to conrm the iagnosis. The Cushing synrome. If the level is elevate, a low-ose exa-
treatment recommenation is chemotherapy using CHOP methasone suppression test shoul be performe. Suppres-
(cyclophosphamie, oxorubicin, vincristine, an preni- sion rules out Cushing synrome. Failure to suppress cortisol
sone) an raiation therapy in most cases of thyroi lym- levels establishes the iagnosis of Cushing synrome. ACTH
phoma. Hashimoto thyroiitis also oes increase the risk of levels shoul then be measure. Low ACTH levels ini-
thyroi cancer. cate a primary arenal source of cortisol, an thus the next
Reference: Ansell SM, Grant CS, Habermann TM. Primary thy- step woul be to obtain an abominal CT scan (E). A high
roi lymphoma. Semin Oncol. 1999;6(3):316–33. ACTH level suggests either a pituitary or ectopic source of
ACTH prouction. A high-ose examethasone suppression
31. D. Hyperparathyroiism is classically associate with test shoul then be performe because a pituitary source of
“stones (calcium phosphate or oxalate kiney stones), moans ACTH will result in some ACTH an cortisol suppression. If
(not feeling well), groans (vague abominal pain, peptic cortisol prouction is suppresse, pituitary MRI shoul be
ulcer isease, pancreatitis, gallstones, an constipation), performe (D). CT scan is less sensitive in emonstrating a
bones (bone pain, osteoporosis, osteitis brosa cystica, brown pituitary mass (A). Failure to suppress cortisol prouction
[osteoclastic] tumors), an psychiatric overtones (epres- with high-ose examethasone suggests an ectopic ACTH
sion, fatigue).” Pancreatitis tens to occur in patients with a tumor. The most common causes of ectopic ACTH prouc-
very high serum calcium level (>1.5 mg/L). The increase tion are bronchial tumors an small cell lung cancer. Thus,
incience of cholelithiasis is ue to increase biliary calcium, the stuy of choice woul be a chest CT scan. Petrosal sinus
leaing to formation of calcium bilirubinate stones. Diarrhea sampling of ACTH is an invasive proceure to etermine
is not typically associate with hyperparathyroiism but which sie of the pituitary glan an ACTH-proucing tumor
constipation is (A–C, E). is locate (B).

32. E. In one large autopsy stuy, 84% of patients ha 35. A. Psammoma boies are calcie eposits represent-
four parathyroi glans, 13% ha more than four glans, ing clumps of sloughe cells. It is consiere iagnostic of
an only 3% ha three glans (D). The superior parathy- papillary carcinoma (B–E). Another histologic characteristic
roi glans are erive from the fourth branchial pouch, of papillary carcinoma is Orphan Annie nuclei.
which also gives rise to the thyroi glan. The thir bran-
chial pouch gives rise to the inferior parathyroi glans an 36. D. Pheochromocytomas occur either sporaically,
the thymus (A). Ectopic inferior glans are more likely to be as part of multiple enocrine neoplasia (MEN) type A
foun within the thymus than the superior glans, whereas an MEN type B, in association with von Hippel-Linau
192 PArt i Patient Care

isease, an with von Recklinghausen isease. The iagno- removal of all four glans with autoimplantation of parathy-
sis of a benign or a malignant pheochromocytoma cannot roi tissue in the forearm muscle or removal of three an a
be accurately etermine by the histologic appearance but half glans.
rather is base on evience of local invasion or the pres- References: Block GA, Martin KJ, e Francisco ALM, et al.
ence or absence of metastasis (B). The risk of malignancy is Cinacalcet for seconary hyperparathyroiism in patients receiving
lower in patients with familial tumors than in patients with hemoialysis. N Engl J Med. 004;350(15):1516–155.
sporaic tumors, although familial tumors are more likely Linberg JS, Culleton B, Wong G, et al. Cinacalcet HCl, an oral
calcimimetic agent for the treatment of seconary hyperparathy-
to be bilateral (A, C). The iagnosis of pheochromocytoma
roiism in hemoialysis an peritoneal ialysis: a ranomize, ou-
is establishe by emonstrating an increase level of cate-
ble-blin, multicenter stuy. J Am Soc Nephrol. 005;16(3):800–807.
cholamines an their metabolites in the plasma an urine. Shoback DM, Bilezikian JP, Turner SA, McCary LC, Guo MD,
Plasma metanephrine levels have the highest sensitivity for Peacock M. The calcimimetic cinacalcet normalizes serum calcium in
pheochromocytoma (99% sensitivity) an are use by most subjects with primary hyperparathyroiism. J Clin Endocrinol Metab.
as the initial screening test (E). 003;88(1):5644–5649.
Slatopolsky E, Brown A, Dusso A. Pathogenesis of seconary
37. A. Papillary cancer is the most common thyroi malig- hyperparathyroiism. Kidney Int Suppl. 1999;73:S14–S19.
nancy in aults an chilren (B–E). The rate of malignancy
in thyroi noules is higher in chilren. In aults, approxi- 39. B. Tertiary hyperparathyroiism most commonly
mately 5% of thyroi noules are malignant, whereas in chil- occurs in the setting of a patient who has ha long-stan-
ren, the rate is approximately 5%. Prognosis in chilren ing seconary hyperparathyroiism in whom subsequently
overall is excellent. autonomously functioning parathyroi glans evelop that
References: Gauger PG, Doherty GM. The parathyroi glan. continue secreting PTH espite high serum calcium levels
In: Townsen CM, Jr, Beauchamp RD, Evers BM, Mattox KL, es. (C). The most common clinical scenario in which it evelops
Sabiston textbook of surgery: the biological basis of modern surgical prac- is the patient who has unergone renal transplantation (A,
tice. 17th e. Philaelphia: W.B. Sauners; 004:985–1000.
E). Distinguishing between seconary an tertiary hyper-
Hanks JB. The thyroi. In: Townsen CM, Jr, Beauchamp RD,
parathyroiism is not critical because the initial management
Evers BM, Mattox KL, es. Sabiston textbook of surgery: the biological
basis of modern surgical practice. 17th e. Philaelphia: W.B. Sauners;
is meical, an surgery is inicate for failure of meical
004:947–984. management (D). Surgical treatment consists of removal of
Lal G, Clark OH. Thyroi, parathyroi an arenal. In: Bruni- 3½ glans rather than all 4 glans with autoimplantation of
cari FC, Anersen DK, Billiar TR, etal., es. Schwartz’s principles of parathyroi tissue in the forearm muscle in cases in which all
surgery. 8th e. New York: McGraw-Hill; 005:1395–1470. four glans are enlarge.
Reference: Kebebew E, Duh QY, Clark OH. Tertiary hyper-
38. D. Seconary hyperparathyroiism is seen in the parathyroiism: histologic patterns of isease an results of
majority of cases in association with chronic renal failure (B). parathyroiectomy: histologic patterns of isease an results of
Rarely, it occurs seconary to intestinal malabsorption of cal- parathyroiectomy. Arch Surg. 004;139(9):974–977.
cium an vitamin D in the absence of kiney failure (D). The
unerlying etiology is a chronic overstimulation of the para- 40. D. The “rule of tens” regaring pheochromocytoma
thyroi glans. Renal failure leas to a ecrease level of cal- (10% bilateral, 10% extra arenal, 10% familial, 10% multifo-
citriol (vitamin D3), an elevation in phosphate, an a rop in cal, 10% malignant) was taught for generations. It was ulti-
serum calcium levels (A). This leas to increase PTH secre- mately isprove in the year 000 after a series of reports
tion. PTH levels are typically very high, ranging from 500 to escribe novel germline mutations causing pheochromocy-
1500 pg/mL (normal is ≤65 pg/mL) (C). As renal failure pro- toma. We now recognize that 0% to 40 % of pheochromocy-
gresses, there is a ecrease in vitamin D an calcium recep- tomas arise as a result of an unerlying familial synrome
tors, leaing to parathyroi glan resistance to calcitriol an an that clear genotype-phenotype correlations exist. The
calcium. This vicious cycle worsens as renal failure worsens. organ of Zuckerkanl is a para-aortic structure locate at
Patients with seconary hyperparathyroiism are generally the take-off of the inferior mesenteric artery or at the aortic
hypocalcemic or normocalcemic. The typical parathyroi bifurcation. It consists of a small mass of chromafn cells that
glan pathology is four-glan hyperplasia. Meical man- are erive from the neural crest. In the fetal circulation, it is
agement has historically consiste of a low-phosphate iet, important in the regulation of bloo pressure via the secre-
phosphate biners, an oral supplementation with calcium tion of catecholamines but then regresses. Pheochromocy-
an vitamin D. More recently, cinacalcet has been approve toma may rarely be foun in the blaer an can present
by the US Foo an Drug Aministration for the treatment with symptoms uring voiing (B). The remaining choices
of seconary hyperparathyroiism ue to chronic renal are very rare locations for pheochromocytoma (A, C, E).
failure. Cinacalcet is a calcimimetic agent. It increases the Reference: Disick GIS, Palese MA. Extra-arenal pheochromo-
sensitivity of the calcium-sensing receptor to activation by cytoma: iagnosis an management. Curr Urol Rep. 007;8(1):83–88.
extracellular calcium an thus irectly lowers PTH levels.
The majority of patients with seconary hyperparathyroi- 41. E. Neuroblastoma is the most common abominal
ism can be manage meically. The recent introuction of malignancy in chilren an the thir most common overall
cinacalcet will likely lea to an even further reuction in the an is of neural crest origin (A). It most often presents as an
nee for surgical management. In general, surgery is ini- abominal mass, an most patients have avance isease
cate for faile meical management (E). Inications inclue at presentation. For stage I isease, surgical resection is the
intractable bone pain, severe pruritus, calciphylaxis, an pro- best treatment. The overall survival rate is less than 30% (C).
gressive renal osteoystrophy. Surgical treatment consists of The tumor may cross the miline, an a majority of patients
CHAPtEr 13 Endocrine Surgery 193

show signs of metastatic isease at presentation. Because larger amounts. T4 is converte to the more active form of T3
these tumors are erive from the sympathetic nervous sys- in the liver, kineys, pituitary, an other tissues. Thus, treat-
tem, catecholamines an their metabolites will be prouce ment of thyroi storm involves inhibiting several steps: (1)
at increase levels. Prognosis is base on age at presentation aressing the ABCs by etermining whether an airway is
(oler or younger than 1 year of age), tumor biology, an neee, aministering 100% oxygen, an starting aggressive
tumor histology. Chilren less than 1 year of age have more ui hyration; () ecreasing new hormone synthesis; (3)
avance isease (B). Amplication of the N-myc oncogene inhibiting the release of thyroi hormone; an (4) blocking
has an unfavorable prognosis. High-risk groups have only a the peripheral effects of thyroi hormone. Propylthiouracil
0% long-term survival rate. In infants, spontaneous regres- an methimazole both inhibit oxiation of ioie to ioine
sion has been well escribe. In the meiastinum, they most an inhibit the thyroi peroxiase–meiate coupling of
often present in the posterior meiastinum (the most common iootyrosines (D). Propylthiouracil also inhibits the conver-
location for neurogenic meiastinal tumors) (D). Neuroblas- sion of T4 to T3 (B). Beta-blockers such as propranolol are use-
toma is associate with many ifferent synromes, inclu- ful in controlling the arenergic response to thyroi storm
ing ancing eyes–ancing feet synrome (cerebellar ataxia, (C). Propranolol also inhibits peripheral conversion of T4
nystagmus, an involuntary movements), catecholamine to T3. Sterois also inhibit the conversion of T4 to T3 in the
release, periorbital metastasis leaing to proptosis an peri- periphery (E). Aspirin is contrainicate in thyroi storm
orbital ecchymosis, skin metastasis that gives the appearance because it is thought to ecrease protein bining of thyroi
of a blueberry mufn, an severe iarrhea (ue to release of hormones. Thus, it may increase the levels of unboun T3
vasoactive intestinal polypeptie). Aniriia an hemihyper- an T4.
trophy, however, are associate with Wilms tumor. Reference: Nayak B, Burman K. Thyrotoxicosis an thyroi
Reference: Meitar D, Crawfor SE, Raemaker AW, Cohn SL. storm. Endocrinol Metab Clin North Am. 006;35(4):663–686.
Tumor angiogenesis correlates with metastatic isease, N-myc
amplication, an poor outcome in human neuroblastoma. J Clin 44. C. Substernal goiter is ivie into primary an secon-
Oncol. 1996;14():405–414. ary forms. Primary forms, ene as ones that originate in
the meiastinum with bloo supply from intrathoracic ves-
42. A. The external branch of the superior laryngeal nerve sels, are very rare (B). Most substernal goiters are extensions
lies on the inferior pharyngeal constrictor muscle an from cervical goiters. Most surgeons recommen resection
escens alongsie the superior thyroi artery before inner- for the mere presence of a substernal goiter because most are
vating the cricothyroi muscle. Injury to the external superior symptomatic, an those that are not can cause progressive
laryngeal nerve results in an inability to tense the ipsilateral compression of the trachea (A). In aition, they may harbor
vocal cor an ifculty hitting high notes, projecting the an unsuspecte malignancy. The majority can be successfully
voice, an voice fatigue uring a prolonge speech. Injury remove with a cervical collar incision. Sternotomy is very
to the internal branch of the superior laryngeal nerve results rarely neee nor is tracheostomy because most can be intu-
in loss of sensory input from the pharynx an subsequent bate, even in the face of tracheal compression, with a pei-
ineffective cough an/or aspiration (D, E). Injury to the atric enotracheal tube (E). They are not typically responsive
recurrent laryngeal nerve can cause vocal cor collapse an to prolonge thyroi suppression (D).
hoarseness (C). Bilateral recurrent laryngeal nerve can result References: Gauger PG, Doherty GM. The parathyroi glan.
in loss of airway (B). In: Townsen CM, Jr, Beauchamp RD, Evers BM, Mattox KL, es.
References: Gauger PG, Doherty GM. The parathyroi glan. Sabiston textbook of surgery: the biological basis of modern surgical prac-
In: Townsen CM, Jr, Beauchamp RD, Evers BM, Mattox KL, es. tice. 17th e. Philaelphia: W.B. Sauners; 004:985–1000.
Sabiston textbook of surgery: the biological basis of modern surgical prac- Hanks JB. The thyroi. In: Townsen CM, Jr, Beauchamp RD,
tice. 17th e. Philaelphia: W.B. Sauners; 004:985–1000. Evers BM, Mattox KL, es. Sabiston textbook of surgery: the biological
Hanks JB. The thyroi. In: Townsen CM, Jr, Beauchamp RD, basis of modern surgical practice. 17th e. Philaelphia: W.B. Sauners;
Evers BM, Mattox KL, es. Sabiston textbook of surgery: the biological 004:947–984.
basis of modern surgical practice. 17th e. Philaelphia: W.B. Sauners; Heayati N, McHenry CR. The clinical presentation an opera-
004:947–984. tive management of noular an iffuse substernal thyroi isease.
Lal G, Clark OH. Thyroi, parathyroi an arenal. In: Bruni- Am Surg. 00;68(3):45–51; iscussion 51–5.
cari FC, Anersen DK, Billiar TR, etal., es. Schwartz’s principles of Lal G, Clark OH. Thyroi, parathyroi an arenal. In: Bruni-
surgery. 8th e. New York: McGraw-Hill; 005:1395–1470. cari FC, Anersen DK, Billiar TR, etal., es. Schwartz’s principles of
surgery. 8th e. New York: McGraw-Hill; 005:1395–1470.
43. A. In the follicular cell, inorganic ioie is trappe an
transporte across the basement membrane. Ioie is oxi- 45. A. Arenal insufciency has primary an seconary
ize to ioine. It is then couple with tyrosine moieties. causes. The most common cause of primary arenal insuf-
This leas to the formation of monoiootyrosine or iiooty- ciency in the Unite States is autoimmune arenal atro-
rosine, catalyze by thyroi peroxiase. Two iiootyrosine phy. The most common cause worlwie is tuberculosis
molecules couple to form T4, an one monoiootyrosine (B). Other less common causes inclue infections (fungal
an one iiootyrosine combine to form T3, both of which cytomegalovirus, human immunoeciency virus), arenal
are boun to thyroglobulin. In the periphery, approximately hemorrhage, metastases, an inltrative isorers (amyloi-
70% to 75% of T3 an T4 is boun to thyroi-bining glob- osis) (C, D). The most common cause of seconary arenal
ulins (not to be confuse with thyroglobulin), an most of insufciency is exogenous glucocorticoi therapy, followe
the remainer is boun to thyroi-bining prealbumin an by bilateral arenal resection an pituitary tumors (E).
albumin, leaving only a small amount of unboun or active Symptoms an signs of acute arenal insufciency inclue
thyroi hormone. T4 is relatively inactive but is present in fever, nausea an vomiting, abominal pain, hypotension,
194 PArt i Patient Care

hyponatremia, an hyperkalemia. As such, it can reaily be 49. D. Incientally iscovere arenal masses are quite
confuse with septic shock. The most specic test for arenal common an are terme adrenal incidentalomas. Most are
insufciency is the ACTH stimulation test. Cortisol levels are nonfunctioning cortical aenomas. The ifferential iagno-
measure at 1, 30, an 60 minutes. Bloo an urine cortisol sis inclues a functional tumor (pheochromocytoma, alo-
levels normally rise with ACTH; failure to rise is inicative steronoma, cortisol proucing), metastatic cancer (from lung,
of arenal insufciency. breast, melanoma), an arenocortical carcinoma. A careful
Reference: Arlt W, Allolio B. Arenal insufciency. Lancet. history an physical examination shoul be performe to
003;361(937):1881–1893. etect evience of hormonal excess (hypertension, viriliza-
tion, Cushing isease). If the patient has hypertension an
46. C. Progressive truncal obesity is the most common a low potassium level, plasma alosterone, an renin lev-
symptom of Cushing synrome, but it is not specic. Rela- els shoul be obtaine. If there is no evience of hormonal
tively specic nings inclue proximal muscle weakness, excess, the following stuies shoul still be obtaine to rule
wie purple striae, spontaneous ecchymoses, an hypoka- out a functional tumor: plasma free metanephrines to rule
lemic metabolic alkalosis. Hirsutism an acne are also asso- out pheochromocytoma an a 1-mg overnight exametha-
ciate with Cushing synrome but are not specic. Cushing sone suppression test to rule out a cortisol-proucing tumor
synrome is most often ue to exogenous corticosteroi (in normal patients, this will markely suppress enogenous
aministration. The most common pathology associate cortisol prouction to a level <1.8 μg/L). Characteristics on
with Cushing synrome is an ACTH-proucing pituitary the CT scan shoul also be etermine. A mass with smooth
aenoma, which is referre to as Cushing disease. Causes borers, that is, homogeneous, an low attenuation (using
of Cushing synrome are ivie into ACTH epenent Hounsel units) is very likely benign, whereas an irregular
(ACTH-proucing pituitary aenoma, ectopic ACTH syn- mass with evience of local invasion, that is, inhomogeneous,
rome, an ectopic corticotropin-releasing hormone syn- an a high attenuation score is of much more concern for
rome) an ACTH inepenent (arenal carcinoma, arenal malignancy (E). Fine-neele aspiration biopsy is not helpful
aenoma, an arenal hyperplasia) (A, B, D, E). in istinguishing a benign arenal aenoma from a malig-
nant arenocortical carcinoma because it is even ifcult to
47. D. The juxtaglomerular cells are moie smooth mus- istinguish the two on histologic examination. Fine-neele
cle cells locate in the afferent arteriole of each glomerulus aspiration biopsy woul only be useful in the patient with a
(A). They synthesize the precursor prorenin, which is cleave history of malignancy to rule out an arenal metastasis (A).
into the active proteolytic enzyme renin. Renal hypoperfu- Surgery is generally recommene for functional arenal
sion, ecrease plasma soium, an increase sympathetic aenomas, pheochromocytomas, masses that have CT scan
activity are the major stimuli for renin secretion (B, C). Renin features suggestive of malignancy, an masses larger than 5
initiates a sequence of steps that begins with cleavage of cm. Once surgery is inicate, laparoscopic arenalectomy
angiotensinogen (a protein prouce in the liver) to form has replace open arenalectomy for most inications. Open
angiotensin I. Angiotensin I is then converte to angiotensin arenalectomy is still preferre for very large tumors (>6 cm)
II by angiotensin-converting enzyme, foun primarily in the an, in particular, when malignancy is suspecte (B, C). For
lung. Angiotensin II causes systemic vasoconstriction an nonfunctional arenal aenomas that o not t the above cri-
stimulates alosterone synthesis an release by the arenal teria, repeat CT scan in 6 months may be performe.
glan, leaing to soium an water retention an expansion Reference: Grumbach MM, Biller BMK, Braunstein GD, et al.
of the plasma volume (E). In the glomerulus, it leas to vaso- Management of the clinically inapparent arenal mass (“incien-
constriction of the efferent arteriole. This leas to increase taloma”). Ann Intern Med. 003;138(5):44–49.
glomerular pressure in an attempt to maintain the glomeru-
lar ltration rate espite systemic hypoperfusion. 50. B. The arenal glan is ivie into the outer cortex an
the inner meulla. The cortex is further subivie into three
48. D. The arterial bloo supply to the arenal glans is layers (“GFR”: glomerulosa, fasciculata, reticularis). The
highly variable, whereas the venous rainage is more con- zona glomerulosa is the outermost layer an is responsible
stant (A). The arenal glans are supplie by three primary for alosterone prouction (A). The mile layer, the zona
sources: the inferior phrenic artery, arenal branches irectly fasciculata, prouces glucocorticois. The zona reticularis is
off the aorta, an branches from the renal artery (E). Aitional the inner layer of the arenal cortex (E). Arenal anrogens
branches may arise from the intercostal an gonaal arteries. are prouce by the eepest cortical layer, the zona reticu-
A single left arenal vein empties into the left renal vein an laris (C). Cells of the arenal meulla prouce epinephrine
is a relatively longer vein than the single right arenal vein, (80%) an norepinephrine (0%). Meullary cells are chro-
which is very short an enters the posterior aspect of the infe- mafn positive (D).
rior vena cava (C). Arenalectomy (open an laparoscopic) is
more challenging on the right sie because of (1) the nee to 51. B. In patients with primary hyperparathyroiism sec-
retract the liver (for a laparoscopic approach), () the nee to onary to a single aenoma, removal of the enlarge glan
mobilize the uoenum, an (3) the short, posteriorly locate is consiere the preferre treatment an biochemical cure
arenal vein that rains into the inferior vena cava, posing a is typically conrme intraoperatively. The Miami criteria
risk of inferior vena cava hemorrhage. Likewise, venous sam- outlines targete PTH values after glan resection, an the
pling of the right arenal vein is more challenging (B). criterion to conclue surgery is a greater than 50% rop in
Reference: Corcione F, Esposito C, Cuccurullo D, et al. Vena PTH level after glan removal. Serum PTH has a half-life
cava injury. A serious complication uring laparoscopic right are- estimate to be 3 minutes. PTH sampling shoul rst be
nalectomy. Surg Endosc. 001;15():18. performe at 10 minutes after glan removal an can be
CHAPtEr 13 Endocrine Surgery 195

repeate after 0 minutes if the PTH level oes not ecrease are sporaic. Sporaic cases are less likely to be multicentric
by more than 50%. Previously, it was thought that oler age, than those associate with MEN . Microscopically, a char-
high boy mass inex, an poor renal function can lea to an acteristic feature of MTC is the ning of abunant collagen
insufcient ecline in PTH level uring surgical resection, an amyloi. Prior to aressing the MTC in a patient with
but a recent JAMA Surgery stuy emonstrate that these suspecte MEN , pheochromocytoma must be exclue/
factors i not have a signicant impact on PTH half-life, manage prior to prevent hypertensive crisis (C).
an as such the Miami Criteria can be use in these patients References: Kebebew E, Ituarte PH, Siperstein AE, Duh QY,
as well (E). It woul be inappropriate to procee to a four- Clark OH. Meullary thyroi carcinoma: clinical characteristics,
glan exploration or to close the woun without conrming treatment, prognostic factors, an a comparison of staging systems.
biochemical cure (A, C). If the baseline PTH level is sample Cancer. 000;88(5):1139–1148.
Moigliani E, Cohen R, Campos JM, et al. Prognostic factors for
from the internal jugular vein ipsilateral to a single aenoma,
survival an for biochemical cure in meullary thyroi carcinoma:
then the PTH level can take longer to rop; therefore, longer
results in 899 patients. The GETC Stuy Group. Groupe ’étue es
wait times may be appropriate in this setting (D). tumeurs à calcitonine. Clin Endocrinol (Oxf). 1998;48(3):65–73.
References: Calò PG, Pisano G, Loi G, et al. Intraoperative
parathyroi hormone assay uring focuse parathyroiectomy: the
55. D. TSH is the most accurate test in hyperthyroiism,
importance of 0 minutes measurement. BMC Surg. 013;13(1):36.
with signicant suppression in hyperthyroi states. In most
Leiker AJ, Yen TWF, Eastwoo DC, et al. Factors that inuence
parathyroi hormone half-life: etermining if new intraoperative
states of hyperthyroiism, free T4, total T4, an total T3 are ele-
criteria are neee. JAMA Surg. 013;148(7):60–606. vate (A–C). Thyroi scan is not use in the initial workup
for hyperthyroiism (E).
52. C. The hallmark of MEN  is MTC. Eventually, nearly
100% of patients with MEN  evelop MTC, whereas only 56. E. The thyroi glan is supplie by paire superior
approximately 40% evelop pheochromocytoma an one- thyroi arteries from the external caroti arteries an the
thir have parathyroi hyperplasia (A, B, E). MTC is charac- inferior thyroi arteries from the thyrocervical trunk. The
teristically multifocal an bilateral an presents at a young superior thyroi artery is the rst branch of the external
age. MTC is associate with C-cell hyperplasia. It is cause caroti artery (B). During thyroiectomy, care must be taken
by mutations in the RET protooncogene (not menin) that are when ligating the superior thyroi arteries to avoi injury to
present in all thyroi C cells an thus lea to multifocal MTC the external branch of the superior laryngeal nerve (D). To
(D). avoi injury, ligating the artery an vein separately an close
to the thyroi glan is recommene. In approximately 3%
53. C. Congenital arenal hyperplasia results from inher- of iniviuals, a thyroiea ima artery also provies bloo
ite enzyme eciencies that can lea to ambiguous genita- to the thyroi glan an arises either from the aorta or the
lia, postnatal virilization, an problems with salt metabolism. innominate artery. When ligating the inferior thyroi arter-
The most common enzyme efect is 1-hyroxylase e- ies, care must be taken to avoi injury to the RLNs (C). The
ciency (>90% of cases). In the complete form, the eciency inferior thyroi arteries usually supply the parathyroi
leas to a ecrease in both cortisol an alosterone. This glans (A). Ligation of the main trunk of the inferior thyroi
leas to ambiguous genitalia in females (ue to anrogen arteries uring total thyroiectomy can lea to parathyroi
excess), salt wasting with hypernatremia, an hypokalemia. glan ischemia. There are three main pairs of veins raining
The remaining answer choices can also cause congenital the thyroi glan: the superior, mile, an inferior thyroi
arenal hyperplasia but are less commonly foun (A, B, D, veins. The mile veins are the least constant. The supe-
E). rior an mile veins rain into the internal jugular veins,
whereas the inferior veins rain into the brachiocephalic
54. A. A patient with a history of primary hyperparathy- veins.
roiism, newly enlarging thyroi noule, an elevate calci-
tonin level likely has multiple enocrine neoplasm-A. These 57. C. The superior laryngeal nerve an RLN arise from
patients are at risk for eveloping meullary thyroi carci- the vagus nerve. The superior laryngeal nerve ivies into
noma (MTC). The characteristics of MTC that affect surgical two branches an is both motor an sensory to the larynx
approach inclue the following: (1) MTC is more aggressive (D). The internal branch is sensory to the supraglottic larynx,
than other thyroi cancers with higher recurrence an mor- an, although rare, injury uring thyroi surgery woul
tality rates. () MTC oes not take up raioactive ioine, an lea to aspiration (A). The external branch innervates the cri-
raiation therapy an chemotherapy are ineffective (B, E). (3) cothyroi muscle. Injury to the external superior laryngeal
MTC is multicentric in 90% of MEN  patients. (4) In patients nerve causes an inability to tense the ipsilateral vocal cor.
with palpable isease, more than 70% have noal metasta- This oes not cause hoarseness, but rather results in voice
ses (D). (5) The ability to measure postoperative stimulate fatigue, an in singers creates ifculty in hitting high notes.
calcitonin levels has allowe assessment of the aequacy of It has been referre to as the nerve of Amelita Galli-Curci
surgical extraction. The two main factors affecting survival or “high note” nerve after the opera singer who unerwent
are stage an age at iagnosis (D). A key factor in survival is thyroi goiter surgery in the 1930s an lost her ability to sing
early etection via calcitonin screening in at-risk patients. In afterwar. The left RLN loops aroun the aorta at the liga-
one large stuy, biochemical cure preicte a survival rate mentum arteriosum. The right RLN loops aroun the right
of 97.7% at 10 years. Management of MTC inclues total subclavian artery. The RLN innervates the intrinsic muscles
thyroiectomy with routine central noe issection (A). It of the larynx with the exception of the cricothyroi mus-
shoul be note that MEN A is rare, an in fact, most MTCs cle, which is innervate by the external laryngeal nerve (E).
196 PArt i Patient Care

Injury to one RLN leas to paralysis of the ipsilateral vocal 60. A. Lateral aberrant thyroi is a term use to enote
cor, which becomes xe in the parameian or abucte what appears to be ectopic thyroi tissue foun within the
position. Bilateral RLN injury may lea to airway obstruc- neck. In most instances, it actually represents metastatic thy-
tion an complete loss of the voice (B). roi cancer within a lymph noe, most often of the papillary
type. It is not typically associate with the remaining answer
58. C. A nonrecurrent laryngeal nerve is rare an occurs choices (B–E).
much more commonly on the right (A, B). It branches off the Reference: Jong D, Demeter S, Jarosz J. Primary papillary thyroi
vagus nerve in the neck an heas irectly to the larynx, as carcinoma presenting as cervical lymphaenopathy: the operative
oppose to arising from the vagus after passing the subcla- approach to the “lateral aberrant thyroi. Am Surg. 1993;59:17–176.
vian artery (D). The anomalous location, as oppose to its
normal position in the tracheoesophageal groove, makes it 61. C. The accepte management of low-risk papillary thy-
more prone to injury (E). On the right, a patient can have roi cancer is either right hemithyroiectomy or total thy-
both a nonrecurrent nerve an a recurrent nerve. Nonre- roiectomy with or without postoperative 131I. In patients
current left laryngeal nerves have been reporte but are with papillary carcinoma with a history of raiation expo-
extremely rare. The recurrent laryngeal nerve is most vulner- sure, there is a higher rate of multicentricity. As such, total
able to injury uring the last  to 3 cm of its course but also thyroiectomy is the recommene proceure (A, B). Post-
can be amage if the surgeon is not alert to the possibility operative raioactive ioine following total thyroiectomy
of nerve branches an nonrecurrent nerves, particularly on is inicate for tumors larger than 4 cm, gross extrathyroi-
the right sie. al extension of the tumor regarless of size, lymph noe
metastases, an for high-risk features incluing tall-cell or
59. E. PTH increases the bone resorption by stimulating columnar-cell variant (E). An ae avantage of postoper-
osteoclasts an inhibiting osteoblasts, leaing to the release ative raiation is that it allows for the continue monitoring
of calcium an phosphate into the circulation. At the kiney, for recurrence with thyroglobulin. Prophylactic central neck
PTH limits calcium excretion at the istal convolute tubule noe issection is gaining popularity as well. Moie rai-
via an active transport mechanism an inhibits phosphate cal neck issection woul not be inicate unless there were
an bicarbonate reabsorption, the latter leaing to a mil obvious lateral neck noes (D).
metabolic aciosis (B, C). PTH also enhances hyroxylation References: Guerrero MA, Clark OH. Controversies in the manage-
of 5-hyroxyvitamin D to 1,5-hyroxyvitamin D in the ment of papillary thyroi cancer revisite. ISRN Oncol. 011;011:30318.
kiney, which in turn irectly increases intestinal calcium Hay ID, Thompson GB, Grant CS, et al. Papillary thyroi carci-
absorption (not a irect effect of PTH) (D). Cholecalciferol is noma manage at the Mayo Clinic uring six ecaes (1940–1999):
hyroxylate to 5-hyroxyvitamin D in the liver. This is not temporal trens in initial therapy an long-term outcome in 444
regulate by PTH (A). consecutively treate patients. World J Surg. 00;6(8):879–885.
Skin and Soft Tissue
ERIC O. YEATES, AREG GRIGORIAN, AND CHRISTIAN DE VIRGILIO 14
ABSITE 99th Percentile High-Yields
I. Most Common Skin Cancers
A. Basal cell carcinoma: most common skin cancer an overall cancer
1. Majority foun on hea an neck, more commonly on upper lip
. Typically appears as a shiny, pearly skin noule with rolle borers
3. Treatment
1. Excision with 4 to 5 mm margins for low risk, an 1 to  cm for high risk
. Low risk is trunk an extremity lesions < cm an hea an neck lesions <1cm; high risk is not
meeting low risk criteria, immunocompromise, perineural invasion, morpheaform sclerosing, or
micronoular
3. Mohs micrographic surgery can be use for cosmetically sensitive areas (i.e., face)
B. Squamous cell carcinoma (SCC): n most common skin cancer (most common in transplant patients)
1. Foun on sunexpose skin, more likely on bottom lip
. UV an immunosuppression are risk factors
3. Appears as ulcer or re/brown skin plaque
4. Similar iagnoses: Bowen isease is SCC in situ, Marjolin ulcer is aggressive SCC originating from
burns, scars, or chronic wouns
5. Higher risk of metastases than BCC
6. Treatment same as BCC
C. Melanoma: thir most common skin cancer, majority of skin cancer eaths
1. UV exposure an congenital nevi are most common risk factors
. Uncommon locations: intraocular, anal
3. Four main subtypes (all melanomas arise from melanocytes in epiermis)
1. Supercial spreaing: most common, moerately aggressive
. Noular melanoma: secon most common, very aggressive, vertical growth, often metastasize at
iagnosis
3. Lentigo maligna: least aggressive
4. Acral lentiginous: very aggressive, more common in people of color, foun on palms, soles of feet,
an subungual
4. Treatment
a) For stages 1 to , no preoperative workup neee after history an exam
b) For melanoma in situ of face: treat with MOHS an 5 mm margins
c) Wie local excision: margins base on thickness (0.5 cm for in situ, 1 cm for ≤1 mm, 1 to  cm for
1 to  mm,  cm for ≥ mm)
) Sentinel lymph noe biopsy (SLNB) if ≥1 mm thick or ≥ 0.75 mm with ulceration
e) Multicenter Selective Lymphaenopathy Trial (MSLT-1): SLNB le to ecrease recurrence of
melanoma compare to WLE alone
f) MSLT-: completion lymph noe issection ha less isease in regional noes at 3 years but i
not improve melanoma-specic survival in patients with sentinel lymph noe metastases
197
198 PArt i Patient Care

g) Targete therapies: abrafenib or vemurafenib for unresectable or metastatic melanoma in


patients with BRAF mutations (increases risk of skin SCC); ipilimumab is a monoclonal antiboy
to CTLA-4 use in metastatic melanoma; both classes improve survival in stage-4 isease

II. Soft-Tissue Sarcomas: Over 50 Different Subtypes


A. Most common subtypes (most common presentation is asymptomatic mass)
1. Aults: Malignant brous histiocytoma (#1), liposarcoma, leiomyosarcoma
. Chilren: Rhabomyosarcoma (#1), osteosarcoma, Ewing sarcoma
3. Most sarcomas sprea hematogenous
4. Select sarcomas sprea via lymphatics: Synovial, Clear cell, Angiosarcoma, Rhabomyosarcoma,
Epithelioi (SCARE)
5. Diagnosis
a) MRI (use T an gaolinium) an x-ray in extremity, CT for retroperitoneum
b) Core neele biopsy (CNB) is rst-line moality to work-up
c) If CNB ineterminate: incisional (if >3 cm) or excisional biopsy (if <3 cm); excisional biopsy
shoul inclue an elliptical incision along longituinal axis of extremity
6. 5-year survival of all stages is 65%
7. Prognosis an staging base on mitotic inex, necrosis, an tumor grae
8. Staging shoul inclue CXR to rule out lung metastases (most common site for metastasis)
9. Treatment
a) Resection with  to 3 cm margins if possible; resection alone (w/o raiation) is OK for low-grae
sarcomas, <5 cm with clear 1 to  cm margins
b) Consier neoajuvant chemo-XRT if it coul allow limb salvage
c) Consier ajuvant raiation for high grae, positive margins, large

III. Other Commonly Teste Skin an Soft Tissue Tumors

Tumor Description Treatment


Merkel cell carcinoma Neuroendocrine origin, red or blue nodule, most WLE with 1–2 cm margins with SLNB,
infected by Merkel cell polyomavirus, UV adjuvant radiation
exposure and immunocompromised states also
risk factors, highly aggressive
Dermatoębrosarcoma Firm, Ěesh-colored, red plaque often mistaken WLE with 2–3 cm margins, can
protuberans for keloid or hypertrophic scar; histologically consider Mohs for face, radiation if
identięed by ęnger-like projections of spindle negative margins not possible
cells
Kaposi sarcoma Vascular sarcoma caused by HHV8, occurs in Not curable, HAART for control
immunocompromised hosts (i.e., AIDS)
Desmoid tumors Arise from ębroblasts, similar to very low-grade Surgery or active surveillance (now
ębrosarcoma, most common in women in controversial with surveillance
30s, can occur after surgery or trauma, locally preferred for non–favorable
aggressive; associated with familial adenomatous locations), neoadjuvant or adjuvant
polyposis (FAP) and Gardner syndrome; in chemotherapy can be considered
sporadic forms, it has a female predominance in
the postpartum period
CHAPtEr 14 Skin and Soft Tissue 199

Questions
1. Which of the following is true regaring 5. Which of the following is true regaring sarcoma?
necrotizing soft-tissue infections (NSTI)? A. Kaposi sarcoma is a common cause of eath in
A. Type I necrotizing fasciitis is polymicrobial patients with AIDS
B. Mortality rates are higher in peiatric cases B. Embryonal subtype is a rare chilhoo
C. The Laboratory Risk Inicator for Necrotizing rhabomyosarcoma
Fasciitis (LRINEC) score is highly sensitive for C. Embryonal subtype has the worst prognosis in
NSTI chilhoo rhabomyosarcoma
D. Clostriium species is the most common D. Osteosarcoma arises from stromal cells
pathogen ientie E. Osteosarcoma is one of the rarest malignant
E. Crepitus an bullae are early skin nings bone tumors

2. A 43-year-ol male presents with a  cm lesion on 6. A 19-year-ol male presents with severe pain in
her upper lip, just above the vermillion borer. A the secon igit of the right han. He has a fever
biopsy reveals a common skin cancer. Which of of 103°F. He has recently been biting his nails. On
the following is true regaring her iagnosis? exam, he is tener lateral to the nail fol of the
A. This is most likely a squamous cell carcinoma igit, an it appears swollen an re. Which of
given the location the following is the best management?
B. Wie local excision with 5-mm margins shoul A. Warm compresses an oral antibiotic coverage
be performe for skin ora
C. XRT shoul be performe given the size an B. Incision an rainage at the mi-igital pulp
location of the lesion C. Incision at lateral nail fol
D. An ultrasoun of the neck shoul be D. Incision at lateral nail fol plus oral antibiotic
performe to rule out lymph noe metastases coverage for skin ora
E. This patient woul be a goo caniate for E. Incision at lateral nail fol plus oral antibiotic
Mohs micrographic surgery coverage for skin ora an anaerobic bacteria

3. A 4-year-ol female presents with a painless 7. A 45-year-ol male with human papillomavirus
growing mass in her left inner thigh. An MRI (HPV) presents to clinic to iscuss his care after
reveals a 6 × 5 cm mass that is concerning for being iagnose with Bowen isease of the anus.
a soft-tissue sarcoma. A core-neele biopsy is Which of the following is true regaring his
ineterminate. Which of the following is the next conition?
best step in management? A. This is consiere an invasive cancer
A. Excisional biopsy B. Wie local excision shoul be performe
B. Resection with  cm margins C. It can be manage initially with imiquimo
C. Incisional biopsy through longituinal incision D. HPV 6 an 11 are the most common subtypes
D. Incisional biopsy at the tumor ege leaing up to this conition
E. Repeat imaging in 6 months E. Negative margins prevent local recurrence

4. Which of the following is least likely to be


associate with lympheema?
A. Recurrent episoes of cellulitis
B. “Buffalo hump” appearance of the orsum of
the foot
C. Hyperpigmentation of the skin
D. Peau 'orange appearance of the skin
E. Thickening an squaring of toes

AL GRAWANY
200 PArt i Patient Care

8. A 65-year-ol female presents with a 5-cm rubber- 11. A 9-year-ol male presents with left wrist
like mass locate on the right sie of her back that pain. He has a mass at the volar wrist that has
has recently been causing pain. It has been slowly been growing in size for the past 4 months an
growing for the past year. On imaging, she has an recently starte causing him pain. The mass is
unencapsulate mass with a lenticular shape. It compressible, freely moving but tethere in place,
has alternating streaks of brous an fatty tissue an transilluminates. Which of the following is
an is locate between the subscapular region at true regaring this conition?
the inferior pole of the scapula an the serratus A. It affects the volar wrist more commonly than
anterior muscle over the thoracic rib cage. Which it oes the orsal wrist
of the following is true regaring this conition? B. It is unlikely to resolve without intervention
A. This is a malignant conition C. Ligation of the tethering peicle is require to
B. It is a benign tumor compose of aipose achieve the lowest recurrence rate
tissue D. Simple aspiration is the preferre treatment
C. Biopsy is necessary even when raiologic option
nings are typical E. The pain is likely seconary to compression
D. Simple excision shoul be performe of the terminal branches of the posterior
E. Wie local excision shoul be performe interosseous nerve

9. A 1-year-ol male presents to the emergency 12. A 76-year-ol female with a history of chronic
epartment (ED) with pain in his upper buttock. lymphocytic leukemia (CLL) presents with
On exam, he has a tener mass at the intergluteal a painless blue, rm noule on the right
region overlying the natal cleft with a sinus tract shouler. It rst appeare several weeks prior
raining purulent ui. A single stran of hair is an was pink in color. It now has overlying
seen protruing from the tract. He reports that he ulceration an measures  cm in iameter.
has been treate for this conition several times. Immunohistochemistry analysis of a skin
Which of the following is most correct? sample emonstrates polyomavirus genome.
A. Control of hair growth at the intergluteal cleft Which of the following is the best next step in
is unlikely to prevent recurrence management?
B. Incision an rainage shoul be performe in A. Expectant management
the ED B. Wie local excision with 1-cm margin an
C. Surgical excision of the sinus tract an ajuvant raiation
marsupialization of the woun shoul be C. Wie local excision with 1-cm margin, sentinel
performe in the OR lymph noe biopsy (SLNB), an ajuvant
D. The pathogenesis likely involves apocrine raiation
glans D. Wie local excision with -cm margin an
E. CT scan of the pelvis shoul be performe ajuvant chemoraiation
E. Neoajuvant chemoraiation followe by
10. The most common cause of primary lympheema wie local excision with -cm margin
is:
A. Congenital lympheema 13. Which of the following is true regaring
B. Lympheema praecox ermatobrosarcoma protuberans (DFSP)?
C. Lympheema tara A. Gross clinical margins are helpful in guiing
D. Filariasis with of excision
E. Malignancy B. The tumor is not raiosensitive
C. If it occurs on the neck, wie local excision is
the surgical treatment of choice
D. Local recurrence rate is lower with Mohs
micrographic surgery compare with wie
local excision
E. Sentinel lymph noe biopsy shoul be
performe
CHAPtEr 14 Skin and Soft Tissue 201

14. Which of the following is true regaring SLNB in D. Blue ye use for lymph noe mapping
melanoma? shoul be injecte outsie of the planne wie
A. A 0.5-mm eep melanoma with ulceration local excision
oes not require SLNB E. All noes whose raioactivity count is greater
B. SLNB is unnecessary for melanoma that has than or equal to 10% of that of the hottest noe
more than a 4-mm thickness shoul be remove
C. There is a survival benet for completion
lymphaenectomy following a positive SLNB

Answers
1. A. There are two types of necrotizing fasciitis (NF). Type I lower lip (A) an involve the vermillion. Although lip can-
NF is the most common type an is polymicrobial with both cers in general have a male preponerance, upper lip cancers
aerobic an anaerobic organisms. It most commonly affects are similar for both sexes or even more common in women.
the elerly, iabetics, an the immunocompromise. Type Though wie local excision is the correct management for
II NF is monomicrobial, with the most common pathogens many BCC, this lesion is in a cosmetically sensitive area an
being group A strep, followe by Staphylococcu aureus. Clos- maximal tissue preservation shoul be attempte (B, C).
tridium is now consiere a rare pathogen in NSTI (D). Type Mohs micrographic surgery utilizes multiple frozen sec-
II NF typically occurs in younger, healthier patients that tions of tissue to achieve complete resection with improve
may have a history of IV rug use, trauma, or recent sur- cosmetic results. BCC rarely metastasizes an imaging of
gery. Though many NSTIs start with a breach in the skin/ regional lymph noe basins is not routinely performe (D).
mucosa which facilitates organism entry into soft tissues, Reference: Murray C, Sivajohanathan D, Hanna TP, et al. Patient
this is not always the case. A nonpenetrating injury can cause inications for Mohs micrographic surgery: a systematic review.
a strain or hematoma leaing to an inammatory response J Cutan Med Surg. 019;3(1):75–90.
with inux of leukocytes. In a susceptible host with transient
bacteremia, organisms can be introuce at the injury site. 3. C. Diagnosis of a soft-tissue sarcoma in an extremity
NSTIs can be ifcult to iagnose ue to their wie range shoul start with an MRI to rule out vascular involvement
of presentations an symptoms. Early nings are nonspe- prior to a biopsy. Core neele biopsy is highly accurate an
cic but may inclue pain out of proportion to exam, ecchy- is the preferre metho for tissue iagnosis. If core neele
mosis, an erythema. Crepitus, bullae, an necrosis are late biopsy is ineterminate an there is still a high suspicion for
skin nings (E). NSTI is a clinical iagnosis that is aie by a sarcoma, an excisional biopsy (if less <3 cm) or incisional
laboratory values an imaging. The Laboratory Risk Inica- biopsy (if >3 cm) shoul be performe (A, B, E). A longitu-
tor for Necrotizing Fasciitis (LRINEC) score, which inclues inal elliptical incision along the long axis of the extremity
WBC, soium, glucose, hemoglobin, CRP, an creatinine, is shoul be use so that it can later be inclue uring the
wiely; use but not a perfect test. In a recent metaanalysis, formal resection (D). This is true for either incisional or exci-
an LRINEC score ≥6 ha sensitivity of 68% an specicity sional biopsies.
of 85% (C). CT scan has been shown to be relatively sensi- Reference: Okaa K. Points to notice uring the iagnosis of
soft tissue tumors accoring to the “Clinical Practice Guieline on
tive an specic an notably superior to plain raiography.
the Diagnosis an Treatment of Soft Tissue Tumors.” J Orthop Sci.
In suspecte cases of NSTI, early surgical intervention with
016;1(6):705–71.
ebriement of all necrotic tissue is the single most import-
ant treatment to reuce mortality. An early secon look oper- 4. C. Distinguishing between chronic venous stasis an
ation is also recommene. The mortality for NSTI has been lympheema on physical examination can be ifcult, partic-
unchange for the last 100 years (5%–30%) but it is consis- ularly early in their course. Lymphoscintigraphy is the iag-
tently lower in peiatric patients compare to aults (B). nostic test of choice for lympheema. Both patient groups
References: Fernano SM, Tran A, Cheng W, et al. Necrotizing will report heaviness an fatigue in the limb, which tens
soft tissue infection: iagnostic accuracy of physical examination,
to worsen at the en of a ay of prolonge staning. Venous
imaging, an LRINEC score: a systematic review an meta-analysis.
Ann Surg. 019;69(1):58–65.
stasis tens to be more pitting an lympheema nonpit-
Stevens DL, Bryant AE. Necrotizing soft-tissue infections. N Engl ting. Venous stasis tens to spare the foot an toes, whereas
J Med. 017;377(3):53–65. lympheema involves them. The swollen orsum of the
foot has a buffalo hump appearance, an toes look square
2. E. Given the lesion is locate on the upper lip, just above off (B, E). Recurrent cellulitis is a common complication of
the vermillion borer, this is most likely a basal cell carci- lympheema (A). In avance lympheema, the skin evel-
noma (BCC), the most common malignancy in the Unite ops a peau 'orange appearance (similar to inammatory
States. Squamous cell carcinoma is more commonly on the isease of the breast), lichenication, an hyperkeratosis (D).
202 PArt i Patient Care

Hyperpigmentation of the skin, ue to hemosierin epo- patients with HPV (E). This is likely ue to the fact that the
sition, is seen in venous insufciency an not usually with remaining perianal skin continues to harbor HPV leaing to
lympheema. continue transformation of normal cells. Initial treatment of
Bowen isease inclues imiquimo or topical 5-FU. Surgical
5. D. Kaposi sarcoma is consiere the most common excision can be consiere for patients with severe symp-
malignancy in AIDS but is rarely a cause of eath (A). It is tomatic isease such as refractory pruritus. Patients shoul
a vascular an cutaneous sarcoma most commonly occur- receive frequent biopsies to look for invasive cancer.
ring in the oral an pharyngeal mucosa an often presents References: Brown SR, Skinner P, Tiy J, Smith JH, Sharp F,
with hemoptysis an ysphagia. Rhabomyosarcoma is Hosie KB. Outcome after surgical resection for high-grae anal
the most common soft-tissue sarcoma in chilhoo with the intraepithelial neoplasia (Bowens isease): surgical resection of high-
embryonal subtype being the most common an with a goo grae anal intraepithelial neoplasia. Br J Surg. 1999;86(8):1063–1066.
prognosis (B). Alveolar subtype has the worst prognosis (C). Goron PH, Nivatvongs S. Principles and practice of surgery for the
Osteosarcoma is erive from mesenchymal stromal cells colon, rectum, and anus. 3r e. CRC Press; 007.
an is consiere the most common malignant bone tumor
in aults (E). 8. D. Elastobroma orsi is a benign, slow-growing process
Reference: Ottaviani G, Jaffe N. The epiemiology of osteo- that is often mistaken for a soft-tissue sarcoma. Some consier
sarcoma. In: Jaffe N, Brulan O, Bielack S., es. Pediatric and adoles- it to be a reactive process; therefore, it is sometimes terme
cent osteosarcoma. Cancer treatment and research. Vol. 15. Springer; a pseuotumor. There has never been a report of malignant
009:3–13. transformation (A). They are almost exclusively foun in
the subscapular or infrascapular region between the scapula
6. E. This patient has acute paronychia, which is an inam- an rib cage. Elastobroma orsi occurs more commonly in
mation involving the proximal or lateral ngernail fols. It women oler than 55. They are frequently right sie, often
presents with suen onset of pain at the nail fol with ery- unilateral, an typically asymptomatic. The pathogenesis is
thema an swelling. Acute paronychia is a clinical iagnosis thought to be ue to repetitive microtrauma, but this has not
but must be ifferentiate from a felon, which can have last- been proven conclusively. Biopsy is unnecessary when raio-
ing consequences if not manage early. A felon is an abscess logic nings are typical (C). MRI is the preferre imaging
of the igital pulp an oes not involve the nail be. The moality an will emonstrate a mass with streaks of brous
appropriate management for a felon is an incision an rain- an fatty tissue locate beneath the scapula. Patients with
age of the igital pulp at the miline to avoi injuring igital asymptomatic lesions o not require intervention. Symptom-
nerves (two sensory nerves meially an two sensory nerves atic patients shoul unergo simple excision (not wie local
laterally) (B). In contrast, most cases of acute paronychia are excision) (E). Local recurrence oes not occur. A lipoma is a
treate with warm compresses (A). In more severe cases (e.g., benign tumor compose of aipose tissue (B).
fever of 103°F), incision an rainage shoul be performe References: Vastamäki M. Elastobroma scapulae. Clin Orthop
by placing a surgical blae uner the cuticle margin an Relat Res. 001;39(39):404–408.
extening it laterally along the sie of the affecte nail fol. Daigeler A, Vogt PM, Busch K, et al. Elastobroma orsi-ifferen-
Oral antibiotics shoul be given for 5 ays after rainage an tial iagnosis in chest wall tumours. World J Surg Oncol. 007;5(1):15.
shoul inclue coverage for skin ora, particularly with the Muratori F, Esposito M, Rosa F, et al. Elastobroma orsi: 8 case
reports an a literature review. J Orthop Traumatol. 008;9(1):33–37.
use of an antistaphylococcal agent (C). However, in a patient
with a history of nail-biting or in a patient with han trauma
9. B. This patient has a pilonial cyst with recurrent inter-
an oral contact (e.g., punching the face), antibiotics shoul
gluteal abscess formation. Pilonial cysts occur most com-
also cover oral ora incluing anaerobic bacteria (D).
monly at the upper borer of the intergluteal cleft an most
References: Brook I. Paronychia: a mixe infection. Microbiol-
ogy an management. J Hand Surg Br. 1993;18(3):358–359.
commonly in young males. The pathophysiology is unclear
Clark DC. Common acute han infections. Am Fam Physician. but likely has to o with clogge hair follicles (D). Occa-
003;68(11):167–176. sionally, hair may be seen protruing from the sinus tract.
Iname apocrine glans are thought to be the culprit in
7. C. Bowen isease is squamous cell carcinoma in situ patients with hiraenitis suppurativa. The iagnosis is
(not invasive) of the perianal margin an is most commonly mae clinically an not with imaging or laboratory stuies
cause by HPV-16 an 18 (A, D). High-grae lesions are more (E). Patients with an acute infection will present with a ten-
likely to be symptomatic an present as a scaly, erythematous, er abscess raining purulent ui at the pilonial cyst site.
pigmente plaque that may have a moist surface. Ulceration This shoul be manage as all other cutaneous abscesses are
is suggestive of malignant transformation. Patients with treate, with incision an rainage (C). This will most likely
known HPV infection shoul unergo screening for anal recur, so the patient shoul have a referral to see a colorectal
intraepithelial neoplasia (AIN). Some regar high-grae AIN surgeon to iscuss enitive repair after the acute conition
as Bowen isease. Screening is often one in the operating has resolve. Although there is not a “gol stanar” for
room (OR) using Lugol solution, which is selectively taken chronic pilonial cyst management, the preferre treatment
up by normal perianal tissue but not by AIN because it lacks option epens on if the pilonial cyst is simple or complex.
glycogen, giving it a characteristic tanne appearance an Excision with primary closure off the miline for a simple,
allowing for tissue biopsy. Previously, it was stanar for all noninfecte pilonial cyst is the most appropriate treatment
patients with high-grae AIN or Bowen isease to unergo option. Complex pilonial cysts will require an en bloc exci-
wie local excision (B). However, this has come uner scru- sion of the sinus tract with a ap reconstruction. A rhomboi
tiny as several reports have reporte a high rate of recurrence ap is the favore approach. Interestingly, there have been
(up to 40%) even with negative margins an particularly in several stuies emonstrating that control of intergluteal
CHAPtEr 14 Skin and Soft Tissue 203

hair growth, either with clippers or laser treatment, will lea appears as a pink noule an progresses to a violaceous blue
to ecrease recurrence of isease (A). color with or without ulceration. About 80% of patients with
References: Humphries AE, Duncan JE. Evaluation an man- MCC have Merkel cell polyomavirus genome foun in tissue
agement of pilonial isease. Surg Clin North Am. 010;90(1):113–14. samples. It is unclear how this leas to the progression of
Khan MAA, Jave AA, Govinan KS, et al. Control of hair MCC as Merkel cell polyomavirus is ubiquitous an foun
growth using long-pulse alexanrite laser is an efcient an cost on most human skin. Wie local excision with 1- to -cm neg-
effective therapy for patients suffering from recurrent pilonial is-
ative margins is the mainstay of treatment (A). Because there
ease. Lasers Med Sci. 016;31(5):857–86.
is a high propensity of lymph noe sprea, patients (with the
Khanna A, Rombeau JL. Pilonial isease. Clin Colon Rectal Surg.
011;4(1):46–53.
exception of hea an neck MCC) without palpable lymph-
aenopathy shoul have SLNB performe at the time of sur-
10. B. Lympheema is ivie into primary (with no cause) gery (B). Aitionally, all patients shoul receive ajuvant
an seconary (there is a known cause). Primary lymph- raiation to control local recurrence (10% recurrence rate
eema is subivie into three types: congenital, praecox, with raiation an 50% without) (E). Chemotherapy is likely
an tara. Congenital lympheema is present at birth (A). A going to play an important role in the future, but as of yet,
familial version of congenital lympheema is calle Milroy there are no conclusive stuies to recommen this as a stan-
isease. Lympheema praecox evelops uring chilhoo ar treatment moality for all patients with MCC (D).
or teenage years an accounts for 80% to 90% of cases of pri- References: Meina-Franco H, Urist MM, Fiveash J, Heslin
MJ, Blan KI, Beenken SW. Multimoality treatment of Merkel cell
mary lympheema an is 10 times more common in women
carcinoma: case series an literature review of 104 cases. Ann Surg
(praecox is primary). It starts usually in the foot or lower Oncol. 001;8(3):04–08.
leg. Lympheema tara is ene as starting after age 35 Heath M, Jaimes N, Lemos B, et al. Clinical characteristics of
(C). Seconary lympheema is more common than primary Merkel cell carcinoma at iagnosis in 195 patients: the AEIOU fea-
lympheema. Worlwie infestation by Wuchereria ban- tures. J Am Acad Dermatol. 008;58(3):375–381.
crofti (lariasis) is the most common cause, whereas in the Santos-Juanes J, Fernánez-Vega I, Fuentes N, et al. Merkel cell
Unite States, the most common cause is post–axillary noe carcinoma an Merkel cell polyomavirus: a systematic review an
issection typically one for unerlying breast cancer (D, E). meta-analysis. Br J Dermatol. 015;173(1):4–49.

11. C. This patient has a ganglion cyst, which is also collo- 13. D. DFSP is consiere the secon most common cuta-
quially known as a “Bible cyst” because they were histori- neous soft-tissue sarcoma following Kaposi sarcoma. It is a
cally manage by slamming a book (the Bible) on the cyst locally aggressive cancer with low metastatic potential. The
allowing for ecompression. The etiology has not been elu- majority of patients have a unique chromosomal transloca-
ciate but is likely multifactorial. The leaing theory is a tion (t:17;), leaing to overexpression of PDGFB, a tyrosine
simple herniation of the joint capsule. It consists of connec- kinase. It can occur at any age but most commonly presents in
tive tissue from the synovial membrane of the joint or ten- the fourth ecae of life. DFSP rst appears as a rm noule
on sheath an most commonly affects the orsal wrist (A). that slowly enlarges an most commonly affects the trunk.
Most patients are asymptomatic but pain, iscomfort, an Core neele biopsy is use for tissue iagnosis. The mainstay
paresthesia can occur. Compression of the terminal branches of treatment is wie local excision. Since it has an inltrat-
of the posterior interosseous nerve may be responsible for ing growth pattern, extension beyon the clinical margins is
pain in the case of orsal ganglion cysts while compression common; thus goo clinical margins are not helpful (A). This
of the branches of the meian or ulnar nerve contributes to may help explain the high rate of local recurrence following
the paresthesia experience by patients with volar ganglion surgery. DFSP occurring in the hea an neck is better serve
cysts (E). About 50% of cases resolve spontaneously within with Mohs microscopic surgery to achieve superior cosmesis
several months to  years (B). Intervention is inicate for (C). Like most sarcomas, DFSP is raiosensitive an raia-
patients that have pain or that are bothere by the cosmetic tion therapy has been emonstrate to ecrease local recur-
appearance. Simple aspiration or surgical excision alone has rence (B). However both systemic an local metastases are
a high recurrence rate (up to 50%). To achieve a recurrence rare an thus, sentinel lymph noe biopsy is not necessary
rate less than 10%, surgical excision with ligation of the pe- (E). A recent metaanalysis emonstrate a lower recurrence
icle is require an is now consiere the gol stanar in rate with Mohs microscopic surgery compare with wie
the treatment of a ganglion cyst. local excision (1.1% versus 6.3%). The prognosis of DFSP is
References: Meena S, Gupta A. Dorsal wrist ganglion: current excellent, with a 10-year survival close to 100%.
review of literature. J Clin Orthop Trauma. 014;5():59–64. References: Gloster HM Jr. Dermatobrosarcoma protuberans.
Rizzo M, Berger RA, Steinmann SP, Bishop AT. Arthroscopic J Am Acad Dermatol. 1996;35(3):355–374.
resection in the management of orsal wrist ganglions: results with a Foroozan M, Sei JF, Amini M, Beauchet A, Saiag P. Efcacy of
minimum -year follow-up perio. J Hand Surg Am. 004;9(1):59–6. Mohs micrographic surgery for the treatment of ermatobrosar-
coma protuberans: systematic review: Systematic review. Arch Der-
matol. 01;148(9):1055–1063.
12. C. Merkel cell carcinoma (MCC) is a rare but aggressive
Kreicher KL, Kurlaner DE, Gittleman HR, Barnholtz-Sloan JS,
skin cancer of neuroenocrine origin arising from specialize Boreaux JS. Incience an survival of primary ermatobrosar-
touch receptor cells in the epiermis of the skin. It occurs in coma protuberans in the Unite States. Dermatol Surg. 016;4 Suppl
elerly, light-skinne patients an those with a history of sun 1:S4–31.
exposure or immunosuppression, particularly CLL. The clin-
ical features can be remembere by the mnemonic “AEIOU”: 14. E. Lymph noe metastases are not uncommon in mela-
Asymptomatic, Expaning rapily, Immunosuppression, noma. SLNB can provie accurate staging in melanoma an
Oler than 50 years ol, an UV-expose area. It often rst is recommene for all melanoma larger than 1 mm eep or
204 PArt i Patient Care

for those with overlying ulceration regarless of epth (A, trial showe that there was no melanoma-specic survival
B). Most surgeons perform SLNB using a raioactive tracer, benet in patients with positive SLNB who unerwent com-
blue ye, or both. There has not been any conclusive ata to pletion lymphaenectomy compare with those that were
show that any one particular agent is better than the other. observe, though there was improve regional control at 3
Ironically, the raioactive tracer is consiere to be safe in years (C). Palpable lymph noes will require a therapeutic
pregnancy but the blue ye is not. The raioactive tracer can lymph noe issection. However, this shoul rst be con-
be mappe with a Geiger counter, an the lymph noe that rme with a ne neele aspiration (FNA) biopsy.
takes up the largest amount of tracer (hot noe) is assume References: Bilimoria KY, Balch CM, Bentrem DJ, et al. Com-
to be the sentinel lymph noe. All noes whose raioactivity plete lymph noe issection for sentinel noe-positive melanoma:
count is greater than or equal to 10% of that of the hottest assessment of practice patterns in the Unite States. Ann Surg Oncol.
noe shoul be remove because it is possible to have more 008;15(6):1566–1576.
Coit DG, Antbacka R, Anker CJ, et al. Melanoma, version .013:
than one sentinel lymph noe. The most common blue ye
feature upates to the NCCN guielines. J Natl Compr Canc Netw.
use is isosulfan blue. Since the ye can stay aroun the skin
013;11(4):395–407.
for several months, it is recommene that the ye be injecte Morton DL, Thompson JF, Cochran AJ, et al. Final trial report of
within the bounary of the planne wie local excision, so it sentinel-noe biopsy versus noal observation in melanoma. N Engl
is also remove with the specimen (D). Rarely, isosulfan blue J Med. 014;370(7):599–609.
ye has been associate with a severe anaphylactic reaction. Raut CP, Hunt KK, Akins JS, et al. Incience of anaphylactoi
Aitionally, all grossly suspicious lymph noes shoul be reactions to isosulfan blue ye uring breast carcinoma lym-
remove as well. Although the role of SLNB has been rmly phatic mapping in patients treate with preoperative prophylaxis:
establishe in current practice, completion lymphaenec- results of a surgical prospective clinical practice protocol. Cancer.
tomy is a point of ebate in the surgical community. Recently, 005;104(4):69–699.
the Multicenter Selective Lymphaenectomy Trial (MSLT-)
Surgical Critical Care
ERIC O. YEATES AND DENNIS KIM 15
ABSITE 99 Percentile High-Yields
I. Acute Respiratory Distress Synrome
A. 01 Berlin enition: replaces criteria from American-European Consensus Conference (AECC)
1. Onset within 7 ays of insult
. Bilateral opacities on chest x-ray or chest CT
3. PaO/FiO (PF) <300 with minimum PEEP 5 (Mil 00–300, Moerate 100–00, Severe <100)
4. Must not be fully explaine by cariac failure or ui overloa (physician’s best estimation); no nee
for invasive pulmonary artery catheter but can use echocariogram
B. Management consierations
1. Tial volume
a) 6 ml/kg of ieal boy weight
b) Plateau pressure <30 cm of water
c) Allow permissive hypercapnia
. Minimal PEEP an FiO to match an acceptable arterial oxygenation (55–80mmHg)
3. Prone positioning
a) Improves oxygenation an reuces ventilator-associate lung injury
b) Improves mortality in moerate to severe ARDS
c) Absolute contrainication: unstable spine fracture
) Relative contrainications: elevate ICP, severe facial trauma, hemoynamic instability, pregnancy,
single anterior chest tube with air leak, unstable femur or pelvis
4. Paralysis
a) Consier in patients with PF <150
b) Cisatracurium for 48hours reuces mortality an barotrauma an increases the number of
ventilator-free ays
5. Inhale nitric oxie (iNO): improves oxygenation, but oes not reuce mortality, may increase the
risk of renal impairment
6. Conservative ui management: along with use of furosemie can ecrease uration of mechanical
ventilation, no improvement in mortality
7. Continuous high-volume hemoltration: may improve oxygenation, reuce uration of mechanical
ventilation, an improve survival
8. Early glucocorticois: may ecrease uration of mechanical ventilation an reuce mortality, but still
controversial

II. Central Line Consierations


A. Infection an DVT rates by location
1. Femoral vein > internal jugular vein > subclavian vein
. Subclavian highest pneumothorax an failure rate
3. If coagulopathic, use compressible site, avoi subclavian an IJ if possible

205
206 PArt i Patient Care

4. Avoi subclavian for patients that may nee hemoialysis access in the future (central venous
stenosis)
5. Air embolism
6. Prevention: trenelenburg uring placement to increase CVP an ecrease pressure graient, avoi
placing uring inspiration, avoi short subcutaneous path to ecrease risk uring removal
B. Management: Durant maneuver -> left lateral ecubitus an Trenelenburg, encourages air bubble to
move from right ventricular outow tract to right atrium, can attempt aspiration

III. Commonly Use Seation an Analgesic Meication in the ICU

Medication Mechanism Pharmacokinetics Side eěects Miscellaneous


Lorazepam GABA agonist Metabolized by Delirium, renal Safer than other
(Ativan) liver, no active dysfunction benzodiazepines in liver
metabolites (propylene glycol) disease
Midazolam GABA agonist Metabolized by liver, Delirium Fastest onset of
(Versed) active metabolites benzodiazepines
(renal excretion)
Diazepam (Valium) GABA agonist Metabolized by liver, Delirium Longest acting of
active metabolites benzodiazepines
(renal excretion)
Propofol GABA agonist Metabolized by liver, Hypotension, PRIS: acute bradycardia
(Diprivan) redistributes in fat propofol infusion with metabolic acidosis,
syndrome (PRIS) rhabdomyolysis,
hyperlipidemia, faĴy liver
Dexmedetomidine α2 agonist Metabolized by liver Hypotension, No respiratory depression,
(Precedex) bradycardia, has analgesic eěects, less
hypertension delirium
Fentanyl µ agonist Highly fat soluble, Respiratory Accumulates with prolonged
(Sublimaze) metabolized by liver depression, infusion, most potent
constipation
Morphine µ agonist Metabolized by liver, Respiratory Least potent, morphine-6-
(Roxanol) active metabolites depression, glucuronide more potent
(renal excretion) constipation than morphine
Hydromorphone µ agonist Metabolized by liver Respiratory Can use if patient develops
(Dilaudid) depression, tachyphylaxis to fentanyl
constipation
Ketamine NDMA Metabolized by liver Hallucinations, When used for anesthesia,
antagonist cystitis, liver stimulates circulatory
toxicity system, protective airway
reĚexes preserved

IV. Commonly Use Vasopressors in ICU

Medication α β1 β2 DA V Uses
Epinephrine Cardiogenic shock, septic shock,
anaphylaxis
Low dose + ++ ++
High dose ++ + +
Norepinephrine ++ + Septic shock (1st line), most other
types of shock
Vasopressin + Septic shock (2nd line),
pulmonary hypertension
Phenylephrine + Neurogenic shock
Dobutamine + Cardiogenic shock
CHAPtEr 15 Surgical Critical Care 207

Medication α β1 β2 DA V Uses
Dopamine Neonatal hypotension, formerly
for AKI
Low dose + ++
Medium dose + ++ ++
High dose ++ + +
Isoproterenol + + Bradycardia, formerly to treat
asthma

V. Types of Shock an Relate Physiologic Parameters

Physiologic state CI CVP/PCWP SVR Exam ęndings


Normal 2.4–4.0 L/min 0–5/6–12 mmHg 800–1200 dyn*sec/cm5

Hypovolemic shock ↓ ↓ ↑ Cool extremities, dry


(hemorrhagic, mucous membranes,
hypovolemic) tachycardia
Cardiogenic shock (MI, ↓ ↑ ↑ Cool extremities, lung
arrhythmia, heart failure) crackles
Obstructive shock (PE, ↓ ↑ ↑ Cool extremities,
tension pneumothorax, decreased breath
tamponade sounds, tracheal
deviation, distended
neck veins
Septic shock ↑ ↓ ↓ Warm extremities, febrile
Neurogenic shock ↓ ↓ ↓ Warm extremities,
bradycardia, spinal
trauma
208 PArt i Patient Care

Questions
1. A 75-year-ol male unergoes an emergent 4. A 68-year-ol (70-kg) male nursing home resient
exploratory laparotomy an bowel resection is amitte for an altere mental status. His
for a small bowel obstruction. Two ays later, vital signs emonstrate orthostatic hypotension.
he remains in the ICU intubate for hypoxic Laboratory stuies reveal a serum soium level of
respiratory failure. On rouns, his nurse reports 168 mEq/L, a serum potassium level of 4.0 mEq/L,
that he is Confusion Assessment Metho a serum chlorie level of 118 mEq/L, an HCO3
(CAM)—ICU positive. Which of the following is level of 8 mEq/L, a bloo urea nitrogen (BUN)
true regaring his iagnosis? of 30 mg/L, an a serum creatinine level of 1.6
A. This conition will likely not affect his mortality mg/L. His free water ecit is:
B. Benzoiazepines an exmeetomiine have A. 3 L an all of it shoul be replace over the
a similar risk of contributing to this conition next 1 hours
C. Quetiapine may improve his conition B. 4 L an all of it shoul be replace over the
D. CAM-ICU may be use on patients with any next 4 hours
Richmon Agitation Seation Scale (RASS) score C. 5 L an .5 L shoul be replace over the next
E. Haloperiol is contrainicate for this conition 4 hours
D. 7 L an 3.5 L shoul be replace over the next
2. Which of the following is true regaring the 4 hours
intraaortic balloon pump (IABP)? E. 10 L an 5 L shoul be replace over the next
A. A suen rop in urine output in a patient 4 hours
with an IABP shoul prompt an immeiate
chest x-ray 5. Which of the following electrocariographic
B. The balloon inates uring early systole an changes is least likely to occur with hypokalemia?
eates uring iastole A. ST segment epression
C. Coronary bloo ow is unchange B. T-wave inversion
D. IABP can be remove regarless of the platelet C. Secon- or thir-egree atrioventricular block
count D. Premature ventricular complexes
E. Heparinization is require at all times with an E. U waves
IABP in place ue to thrombotic risk
6. A 4-year-ol woman with metastatic breast
3. A 55-year-ol male sustains severe multisystem cancer is lethargic an has mental status changes.
injuries, incluing multiple rib fracture an a Her serum calcium is 14.5 mg/L, serum alkaline
traumatic brain injury with subural hematoma, phosphatase is 000 IU/L, BUN is 4 mg/L, an
after a motor vehicle collision. He remains intubate serum creatinine is 1.1 mg/L. Initial treatment of
for multiple ays an evelops ventilator-associate the hypercalcemia shoul be:
pneumonia, acute renal failure, an septic A. Mithramycin
shock an requires placement of a nontunnele B. Bisphosphonates
hemoialysis catheter at besie. His INR is .4. C. Loop iuretics
Which of the following is true regaring central D. Thiazie iuretics
line placement in this patient? E. Normal saline infusion
A. The use of prophylactic antibiotics reuces
catheter-associate infection
B. Placement in the subclavian vein placement is
ieal in this patient
C. Placement of a central line in the internal
jugular vein is contrainicate in a patient
with intracranial hemorrhage
D. Placement in the subclavian vein has the
highest risk of pneumothorax
E. Placement in the femoral vein has a lower
risk of central line-associate bloostream
infection (CLABSI) compare to subclavian
vein placement
CHAPtEr 15 Surgical Critical Care 209

7. A 75-year-ol man becomes hypotensive (systolic 12. Argatroban:


bloo pressure of 70 mmHg) after repair of A. Activates antithrombin
an inguinal hernia. Urine output is low, an B. Is cleare by the kineys
he is unresponsive to ui aministration. A C. Is reverse with fresh-frozen plasma
pulmonary artery catheter is inserte. Cariac D. Can be monitore by the activate partial
output is 3 L/min; systemic vascular resistance thromboplastin time
(SVR) is 140 ynes/sec × cm; SvO is 55%; E. Has a 3-hour half-life
pulmonary capillary wege pressure (PCWP) is
4 mmHg. Which of the following is most likely 13. A 54-year-ol man who weighs 100 lbs comes to
to elevate the systolic bloo pressure? the ED after vomiting for 3 ays an losing 10lb.
A. 500 mL of Ringer’s lactate to improve preloa His serum electrolytes are as follows: soium
B. Lasix (furosemie) 0 mg intravenously to 136 mEq/L, potassium 3.1 mEq/L, chlorie
improve urine output 88mEq/L, an carbon ioxie 37 mEq/L. Which
C. Nitroprussie at 0.5 μg/kg/L/min to ecrease one of the following woul be most helpful in
the SVR etermining the cause of his aci-base isorer?
D. Dobutamine at 5 to 10 μg/min for inotropic A. Urine soium
support B. Urine creatinine
E. Neo-Synephrine (phenylephrine) at 1 μg/L per C. Urine chlorie
min to increase bloo pressure D. Urine pH
E. Urine potassium
8. Nosocomial pneumonia among intensive care
unit patients: 14. Which of the following is true regaring
A. Has the same mortality rate as oes meications commonly use for seation in the
community-acquire pneumonia (CAP) intensive care unit?
B. Is the most common nosocomial infection A. Propofol is associate with tachycaria
C. Can be avoie by early tracheostomy B. Miazolam can lea to propylene glycol
D. Is irectly relate to the uration of intubation toxicity when given as a continuous infusion
E. Can be prevente by early institution of C. Dexmeetomiine has a similar metho of
prophylactic IV antibiotics action to cloniine
D. Lorazepam has active metabolites leaing to a
9. Bloo samples for the etermination of mixe longer uration of action
venous oxygen saturation (SvO) are ieally E. Ketamine inhibits protective airway reexes
obtaine from:
A. Right atrium 15. A 70-year-ol male with chronic obstructive
B. Pulmonary artery pulmonary isease (COPD) presents to the
C. Pulmonary vein emergency epartment (ED) after a stab woun
D. Two peripheral veins mixe together to the left anterior thoracoabominal region.
E. Central venous pressure line His vitals are stable. The patient evelops
progressively worsening shortness of breath an
10. Which of the following variables has the least noninvasive positive pressure ventilation (NPPV)
inuence on oxygen elivery? with bilevel positive airway pressure (BiPAP) is
A. Hemoglobin starte for a presume COPD exacerbation. The
B. Cariac contractility patient’s bloo pressure then abruptly rops to
C. Heart rate 70/40 mmHg. Which of the following statements
D. Carbon monoxie concentration is true?
E. Partial pressure of issolve oxygen in the A. This complication coul have been prevente
bloo with a higher expiratory pressure on BiPAP
B. The cariopulmonary compromise is likely
11. Pulmonary artery occlusion pressure (PAOP) seconary to obstructive shock
reects which of the following physiologic C. Immeiate enotracheal intubation is
variables? manatory
A. Cariac output D. BiPAP is contrainicate with COPD
B. Pulmonary arterial pressure E. A nasogastric tube shoul have been place
C. Left atrial pressure before initiation of BiPAP
D. Pulmonary compliance
E. Systemic vascular resistance (SVR)
210 PArt i Patient Care

16. A 4-year-ol obese male with a traumatic 19. A 68-year-ol male has new onset of an irregular,
brain injury is 6 hours post proceure from a narrow complex tachycaria with a ventricular
percutaneous ilatational tracheostomy tube rate of 15 beats per minute. A single ose of
placement. The nurse calls to state that the metoprolol is aministere with minimal affect.
tracheostomy tube was accientally isloge. The patient subsequently becomes iaphoretic
Which of the following is recommene? an the bloo pressure rops to 7/35 mmHg.
A. Immeiately reinsert the tube What is the next best step in management?
B. Immeiately reinsert the tube using A. Unsynchronize carioversion
ultrasoun guiance B. Synchronize carioversion
C. Bag patient an urgently transport to the C. Debrillation
operating room for open reinsertion D. Amioarone push followe by a continuous
D. Perform besie cricothyroiotomy rip
E. Enotracheal intubation E. Intravenous (IV) push of aenosine

17. A 50-year-ol female was amitte to the 20. Which of the following is true regaring septic
intensive care unit (ICU) 36 hours ago with shock?
worsening hypoxic respiratory failure seconary A. It is characterize by poor perfusion of en
to pulmonary contusion following a motor vehicle organs
collision. The most recent chest raiograph shows B. Maintaining hemoglobin level greater than
new bilateral pulmonary inltrates. Current arterial 10g/L is recommene
bloo gas shows a PaO of 70 mmHg. Current C. In early septic shock, whole boy oxygen
ventilator settings inclue a FiO of 60% an a consumption is ecrease
positive en-expiratory pressure (PEEP) of 8 cm D. Positive ui balance is associate with
HO. She has no history of heart isease. Which of increase mortality
the following is true regaring this conition? E. The liver can serve as a continue source of
A. An objective surrogate for pulmonary artery inammatory proucts
capillary wege pressure (PAWP) is necessary
to make a iagnosis of ARDS 21. A 7-year-ol male (75 kg) with severe peritonitis
B. Prone ventilation shoul be initiate ue to perforate appenicitis evelops
C. Inhale nitric oxie will confer a mortality hypotension requiring pressors following
benet laparotomy. He has low systemic vascular
D. High-frequency oscillatory ventilation (HFOV) resistance an high cariac output. Over the
is associate with improve survival past 1 hours his urine output roppe to less
E. Neuromuscular blockae is associate with an than 10 cc/hour espite receiving aequate IV
increase in ventilator ays uis. His creatinine increase from a baseline of
0.9 mg/L on amission to . mg/L. He is not
18. A 5-year-ol male is preamitte for a coronary aciotic nor hyperkalemic an oes not appear to
artery bypass graft for three-vessel isease. While be volume overloae. Which of the following is
attempting to obtain a pulmonary artery capillary true for this patient?
wege pressure with the balloon inate, the A. Hemoialysis (HD) shoul be initiate
patient begins to cough an has a small amount B. HD is better than CRRT at removing
of hemoptysis. However, this resolves quickly, inammatory meiators
an the patient shows no other signs of istress. C. Early initiation of continuous renal
Which of the following is the next best step in replacement therapy (CRRT) will improve
management? survival
A. Deate the balloon, withraw the catheter D. CRRT will require vascular access that iffers
into the right ventricle an reoat into the from that of HD
pulmonary artery E. The timing of initiating CRRT oes not change
B. Deate the balloon an remove the pulmonary the length of hospital stay
artery catheter entirely
C. Leave the balloon inate an prepare the
patient for a catheter-base angiography
D. Take immeiately to the operating room for
emergent thoracotomy
E. Hyperinate the balloon an avance the
catheter as much as possible
CHAPtEr 15 Surgical Critical Care 211

22. A 1-year-ol man who was the river in a hea- 27. A morbily obese 48-year-ol male is amitte
on collision has a pulse of 140 beats per minute, to the ICU following an open cholecystectomy
respiratory rate of 36 breaths per minute, an via a miline incision. The patient’s PaO is 50
systolic bloo pressure of 70 mmHg. His trachea mmHg on a FiO of 60% an PEEP of 5 cm HO.
is eviate to the left, with palpable subcutaneous After increasing PEEP to 10 cm HO, which of the
emphysema an absent breath souns over following parameters is likely to increase?
the right hemithorax. The next best step in the A. Arterial partial pressure of carbon ioxie
management of this patient is: (PaCO)
A. Resuscitative thoracotomy B. Cariac output
B. Ultrasonography or chest raiograph to C. Functional resiual capacity (FRC)
conrm iagnosis D. Left ventricular en-systolic volume
C. Intubation an ventilation E. Pulmonary eema
D. Tube thoracostomy
E. Neele thoracostomy 28. Which of the following is most commonly
associate with transfusion-transmitte bacterial
23. In hemorrhagic shock, which of the following infection?
is the most accurate sign of aequate ui A. Staphylococcus aureus
resuscitation? B. Staphylococcus epidermidis
A. An increase in bloo pressure C. β-Hemolytic streptococcus
B. An increase in urine output D. Bacillus fragilis
C. An increase in arterial oxygenation E. Gram-negative organisms
D. A ecrease in thirst
E. A ecrease in tachycaria 29. Intraabominal hypertension is ene as
intraabominal pressures that excee:
24. A 50-year-ol woman who is septic from A. 1 cm HO
ascening cholangitis is transferre to the B. 16 cm HO
surgical ICU. She unergoes cholecystectomy C. 0 cm HO
an common bile uct exploration after a D. 5 cm HO
faile enoscopic sphincterotomy. Because of E. 30 cm HO
hypotension an marginal urine output, a Swan-
Ganz catheter is place. Which of the following 30. After an elective low anterior resection for rectal
reaings is least consistent with the patient’s cancer, palpitations evelop in a 59-year-ol
clinical course? man with a history of congestive heart failure
A. Central venous pressure 5 cm HO an an ejection fraction of 0% in the ICU. On
B. SVR 300 ynes × sec × cm the electrocariogram, he is note to be in a
C. Cariac inex .0 L/min/cm ventricular tachycaria (VT) at a rate of 150 beats
D. Pulmonary capillary wege pressure 10 cm per minute. On evaluation, he has altere mental
HO status an his bloo pressure is 95/65 mmHg.
E. SvO 86% The best initial treatment for this arrhythmia
woul be:
25. Prolonge QT intervals are seen in association A. Epinephrine 1 mg IV push
with: B. Amioarone 150 mg IV over 10 minutes
A. Hypomagnesemia C. Immeiate ebrillation with 360 J
B. Hypercalcemia D. Synchronize carioversion with 150 J
C. Hyperphosphatemia E. Diltiazem 15 mg IV over  minutes
D. Hyperkalemia
E. Hypokalemia 31. In patients with acute kiney injury, the most
immeiate threat to the patient is:
26. Acute symptoms of hypermagnesemia are treate A. Aciosis
by: B. Hyperkalemia
A. Flui hyration with normal saline C. Platelet ysfunction
B. IV insulin D. Flui overloa
C. Calcium chlorie E. Malnutrition
D. Dextrose
E. Dialysis
212 PArt i Patient Care

32. Which of the following is true regaring 33. A 19-year-ol man presents to the ED after
hepatorenal synrome? a motor vehicle collision. The patient is alert
A. Type II is rapily progressive with a poor an oriente but is unable to move his arms
prognosis an legs. Results of a focuse assessment with
B. It is associate with intense renal vasoilation sonography for trauma (FAST) scan an chest an
C. It is associate with splanchnic pelvic raiographs are all negative. On physical
vasoconstriction examination, the patient has a bloo pressure
D. The urine soium is typically less than of 80/60 mmHg an a heart rate of 70 beats per
10 mEq/L minute. His feet are warm an pink, an he is
E. Type I is relatively stable note to have priapism. Which of the following is
the next best step in management?
A. Phenylephrine
B. Intravenous ui aministration
C. Dobutamine
D. Epinephrine
E. Norepinephrine

Answers
1. C. The CAM-ICU is one of the most commonly use tools ow (C). Contrainications inclue moerate/severe aor-
for assessing for elirium in the ICU an can be use on intu- tic regurgitation an aortic issection. There are a number
bate an nonintubate patients. The CAM-ICU algorithm of known complications with IABPs, incluing hemolysis
rst requires the patient to be sufciently awake (RASS ≥ –3) ue to mechanical amage to re bloo cells. Aitionally,
(D). Next, it tests for inattention an isorganize thinking. IABP migration or malpositioning can occur an cover the
Delirium is common in the ICU an has been shown to be an renal vessels, ecreasing urine output. Though hepariniza-
inepenent preictor of mortality (A). Prevention strategies tion is recommene when possible to prevent thrombotic
inclue avoiing certain meications an early mobiliza- complications, the risk of bleeing shoul be weighe,
tion uring interruptions in seation. For example, patients especially in patients who recently unerwent major car-
receiving exmeetomiine over benzoiazepines ha a iac surgery (E). Before removal of IABP, the platelet count,
lower incience of elirium (B). There is no meication that PT, an PTT shoul be normal (D).
has conclusively been emonstrate to ecrease the risk of References: Krishna M, Zacharowski K. Principles of intra-
the evelopment of elirium in ICU patients. However, anti- aortic balloon pump counterpulsation. Contin Educ Anaesth Crit Care
psychotics incluing haloperiol an quetiapine can be use Pain. 009;9(1):4–8.
to treat the symptoms of hyperactive elirium (E). Pucher PH, Cummings IG, Shipolini AR, et al. Is heparin neee
for patients with an intra-aortic balloon pump? Interact Cardiovasc
References: Ely EW, Shintani A, Truman B, et al. Delirium as
Thorac Surg. 01;15(1):136–139.
a preictor of mortality in mechanically ventilate patients in the
intensive care unit. JAMA. 004;91(14):1753–176.
Reae MC, Finfer S. Seation an elirium in the intensive care
3. D. The choice of central line placement site is important
unit. N Engl J Med. 014;370(5):444–454. in complex ICU patients that may have various contrainica-
tions. In a ranomize control trial comparing complications
2. A. The IABP is a circulatory assist evice inicate between central line placement sites, subclavian placement
for use in cariogenic shock. Preoperatively, IABP is ini- ha a lower risk of bloostream infection an thrombosis
cate for low cariac output (CO) states incluing unsta- compare to the internal jugular vein an femoral vein but
ble angina refractory to meical therapy. Intraoperatively, ha a higher rate of pneumothorax (E). In contrast, a Cochrane
it can be use to permit weaning from cariopulmonary analysis foun no ifference in the risk of CLABSI between
bypass when inotropic agents alone are not sufcient. Post- femoral, subclavian, an internal jugular vein site insertions
operatively, IABP is use primarily for low CO states refrac- (E). Subclavian placement shoul also be avoie in patients
tory to meical management. IABPs are most commonly that may nee hemoialysis access in the future an those
inserte into the femoral artery an the raiopaque tip of with coagulopathy, as it is ifcult to control bleeing with
the balloon is positione just below the aortic knob an just irect pressure in this area (B). Intracranial hemorrhage an/
istal to left subclavian artery. IABPs work by inating in or elevate intracranial pressures are not a contrainication
iastole an eating in early systole (B). This inirectly to catheterization of the internal jugular veins (C). There is
assists the heart by ecreasing afterloa an augmenting also no high-level evience supporting prophylactic antibi-
iastolic aortic pressure an increasing coronary bloo otics prevents catheter-associate infection (A).
CHAPtEr 15 Surgical Critical Care 213

Reference: Marik PE, Flemmer M, Harrison W. The risk of (Neo-Synephrine) is an α1-agonist that will increase SVR
catheter-relate bloostream infection with femoral venous cathe- (afterloa) an can increase bloo pressure. However, the
ters as compare to subclavian an internal jugular venous cathe- patient is alreay maximally vasoconstricte as you woul
ters: a systematic review of the literature an meta-analysis. Crit Care expect in cariogenic shock an as evience by the high
Med. 01;40(8):479–485.
SVR. Phenylephrine woul be a poor choice in a patient with
4. D. Free water ecit = (serum soium − 140)/(140) × total cariogenic shock (E).
boy water. Total boy water is 50% of lean boy mass in
8. D. Nosocomial pneumonia is the secon most com-
men an 40% in women. The free water ecit calculates to
mon nosocomial infection (the most common is urinary
be 10 L, half of which shoul be replace over the next
tract infection) an the most common nosocomial infection
4 hours. The correction must be mae slowly to avoi neu-
among ventilate patients (B). The risk of ventilator-associ-
rologic complications such as cerebral eema.
ate pneumonia increases 5% per ay an is as high as 70%
5. C. Electrocariographic changes associate with hypo- at 30 ays. The 30-ay mortality rate from nosocomial pneu-
kalemia inclue U waves, T-wave attening, ST-segment monia can be as high as 40%, which is signicantly higher
changes, an arrhythmias (A, B, D, E). Atrioventricular block than CAP (A). Nosocomial pneumonias are frequently poly-
is more common with hypercalcemia an hyperkalemia. microbial, an gram-negative ros are the preominant
Hypokalemia is a common electrolyte abnormality in surgi- organisms. The criteria for iagnosis inclue fever; cough;
cal patients, occurring because of inaequate supplementa- evelopment of purulent sputum in conjunction with raio-
tion with total parenteral nutrition an excessive IV uis. logic evience of an inltrate; suggestive Gram stain n-
ings; an positive sputum, tracheal aspirate, pleural ui, or
6. E. The treatment of hypercalcemia of malignancy shoul bloo cultures. Prophylactic use of IV antibiotic has not been
begin rst with inucing calciuresis. This is accomplishe by shown to reuce rates of nosocomial infection or to improve
saline volume expansion. Once volume has been expane, survival (E). However, there are some ata to suggest that
the next step is to aminister loop iuretics because this oral econtaminant regimens (gentamicin/colistin/vanco-
similarly inuces calciuresis (loops lose calcium) (C). Thi- mycin % in Orabase gel every 6 hours) can reuce the rate
azie iuretics will have the opposite effect (D). Mithramy- of ventilator-associate pneumonia. However, this is not
cin acts irectly on bones, lowering calcium levels, but the yet the stanar of care. Although early tracheostomy can
effect takes more than 4 hours (A). In contrast, bisphospho- reuce the number of ays on the ventilator, it oes not lea
nate rugs are inicate in aition to IV hyration an to reuce rates of pneumonia (C).
loop iuretics in patients with cancer. This class inclues Reference: Bergmans DC, Bonten MJ, Gaillar CA, et al. Pre-
zoleronic aci (superior) an pamironate, which inhib- vention of ventilator-associate pneumonia by oral econtamina-
tion: a prospective, ranomize, ouble-blin, placebo-controlle
its osteoclast activity, resulting in lower calcium levels in
stuy. Am J Respir Crit Care Med. 001;164(3):38–388.
patients with bony metastasis. However, these agents may
take 48 to 7 hours before reaching full therapeutic effect
9. B. SvO is an inirect measurement of oxygen elivery. It
(B). Aitionally, calcitonin lowers serum calcium levels
is measure from a bloo sample obtaine from the pulmo-
within hours by inhibiting bone resorption that is occurring
nary artery. A true SvO inclues bloo from the vena cava
from metastatic isease; calcitonin is inicate in hypercal-
an the coronary sinus (A, C–E). SvO is a marker for ae-
cemic crises. Corticosterois are most useful in hypercalce-
quacy of resuscitation an reversal of hypoxemia. However,
mia relate to sarcoiosis an multiple myeloma. They may
mixe venous gas is most commonly sample from the supe-
be useful in patients with bony metastasis, but they take as
rior vena cava using a central venous catheter.
long as 1 week to work.
Reference: Major P, Lortholary A, Hon J, et al. Zoleronic aci
is superior to pamironate in the treatment of hypercalcemia of
10. E. Oxygen elivery (DO) is etermine solely by the
malignancy: a poole analysis of two ranomize, controlle clin- cariac output an the oxygen content of bloo (CaO), so
ical trials. J Clin Oncol. 001;19():558–567. anything that affects these two variables is going to have
a irect effect on the DO. Because cariac output is eter-
7. D. The patient is in cariogenic shock as evience by mine by the stroke volume an heart rate, a change in
low cariac output, elevate SVR, an elevate PCWP. He cariac contractility will irectly inuence DO (B, C). The
has alreay shown to not have a persistent response to uis oxygen content of bloo can be ene by the equation
so an aitional bolus is unlikely to be helpful (A). Inotropic CaO = (Hg × 1.34 × SaO) + (PaO × 0.003), where Hg is the
support in the form of obutamine is inicate to improve concentration of hemoglobin, SaO is the percent saturation
cariac contractility an cariac output, while ecreasing of hemoglobin, 1.34 is the oxygen-carrying capacity of 1 g
afterloa. Alternative inotropes (vasoactive agents) inclue of hemoglobin, PaO is the partial pressure of oxygen is-
epinephrine an a phosphoiesterase inhibitor, such as mil- solve in the bloo, an 0.003 is the measure of O (in mL)
rinone. Given the patient’s elevate PCWP, it is unlikely issolve in 1 L of bloo per mmHg of pressure. So hemo-
that further ui resuscitation to increase preloa is neces- globin an carbon monoxie (which ecreases the percent
sary. Furosemie (Lasix) is not a goo option because his saturation of hemoglobin by oxygen) will both affect oxy-
low urine output reects poor forwar ow versus volume gen elivery as well (A, D). The PaO only contributes 1% to
overloa (B). Nitroprussie is a vasoilator an coul poten- % of the total oxygen content (E). However, it is important
tially improve cariac output, but bloo pressure is unlikely to keep in min that the SaO is partly reliant on the PaO
to improve without ionotropic agents (C). It shoul not be as can be emonstrate by the oxyhemoglobin issociation
use as the next step in treating this patient. Phenylephrine curve.
214 PArt i Patient Care

11. C. PAOP or pulmonary artery wege pressure (PAWP) of effect lengthens. Miazolam, in particular, also has active
provies an inirect estimate of both left atrial an left ven- metabolites, which further prolong its uration; this effect is
tricular iastolic pressure (A–B, D–E). These pressures can be worsene by hepatic or renal failure. In contrast, lorazepam
measure using a Swan-Ganz or pulmonary artery catheter, has no active metabolites, but mobilization to an from the
which is a exible balloon-tippe catheter that is inserte into peripheral tissues is much slower (D). With prolonge usage
the pulmonary artery an inate, thereby occluing pulmo- at high oses, lorazepam can lea to propylene glycol tox-
nary artery pressures an reecting left heart pressures. In icity because it is inclue in the iluent. However, this is
iastole, there are no valves between the open mitral valve not use in the formulation of miazolam (B). For the above
an close pulmonic valves. In this unobstructe pathway reasons, benzoiazepines are poor choices for prolonge
between the left ventricle an the close pulmonic valves, as seation in the ICU. Propofol, which is believe to work on
well as the relatively low compliance of the pulmonary artery the GABA receptor, is a highly lipophilic anesthetic with a
circulation, there exists a column of bloo from the catheter very quick onset an short uration. It quickly istributes to
tip in the pulmonary artery through the pulmonary capillary tissues an is rapily metabolize by the liver, leaing to a
be, pulmonary vein, left atrium, an left ventricle. The mea- short uration of action. However, it has signicant cario-
sure pressure approximates the pressure in the LV uring vascular effects incluing hypotension an braycaria (A).
en-iastole an is use as a measure of LV preloa. With Ketamine is a potent seative that blocks glutamate NMDA
the balloon wege an the ventricles in systole, PAWP now receptors within sensory nerve enings. It creates a isso-
measures pressures reecte by the LA provie there are ciative anesthetic effect where patients remain conscious
no signicant mitral valve abnormalities. Swan-Ganz cathe- without inhibition of respiratory rive or protective airway
ters provie both irect an inirect measurements of cariac reexes (E). Unlike propofol, ketamine is consiere to have
performance an these measurements are contingent upon analgesic properties. However, because of signicant psy-
when uring the cycle the measurements are taken as well as choactive effects, its use is limite. Dexmeetomiine an
whether or not the catheter is wege. cloniine are both selective alpha- receptor antagonists,
though the former has a much higher afnity for alpha-
12. D. Argatroban an lepiruin are both irect thrombin receptors than cloniine. Dexmeetomiine is a seative
inhibitors an use for heparin-inuce thrombocytopenia with anxiolytic an analgesic properties without signicant
(HIT) an thrombosis (A–C, E). Both can be monitore by respiratory epression. Patients transition easily from unis-
the activate partial thromboplastin time an both have rela- turbe seation to being arouse with stimuli.
tively short half-lives (60–90 minutes for lepiruin an 40–50 References: Mihic S, Harris R. Hypnotics an seatives. In:
minutes for argatroban). Neither can be reverse an neither Brunton LL, Chabner BA, Knollmann BC, es. Goodman & Gil-
requires the presence of antithrombin to be effective. Arga- man’s: the pharmacological basis of therapeutics. 1th e. McGraw-Hill;
troban is cleare by hepatic metabolism, whereas lepiruin 015:101–119.
is cleare by the kineys. In aition to being use for HIT, Sokol S, Patel BK, Lat I, Kress JP. Pain control, seation, an use
argatroban is approve in patients with or at risk of HIT who of muscle relaxants. In: Hall JB, Schmit GA, Kress JP, es. Principles
of critical care. 4th e. McGraw-Hill; 014:76–84.
are unergoing percutaneous coronary intervention. Arg-
atroban has a short half-life (40–50 minutes) an reaches a
steay state with IV infusion at 1 to 3 hours. Because it is 15. B. Traumatic pneumothorax shoul be suspecte in
cleare by hepatic metabolism, it is the rug of choice for all patients with a penetrating thoracoabominal injury.
patients with HIT an renal insufciency. The aition of NPPV increases intrathoracic pressure an
may convert a pneumothorax into a tension pneumothorax,
13. C. With a bicarbonate level of 37, this patient has a meta- which can subsequently ecrease venous return an result
bolic alkalosis. The cause of metabolic alkalosis can be eter- in obstructive shock (as emonstrate in the above case).
mine by whether it is chlorie responsive or resistant (A–B, Similarly, enotracheal intubation also increases intratho-
D–E). Chlorie-responsive cases (urine chlorie <15 mEq/L) racic pressure an can result in shock (C). In a stable patient
are much more common in surgical patients an result from without classic exam nings for pneumothorax (ecrease
vomiting (gastrointestinal loss of hyrogen ions), iuretics breath souns, tympanic chest), a chest x-ray can be per-
(genitourinary loss of chlorie), an volume epletion (alo- forme to look for the collapse lung. This can be followe
sterone-stimulate hyrogen ion loss in urine). Conversely, by insertion of a tube thoracostomy (chest tube). NPPV can
chlorie-resistant types (urine chlorie >5 mEq/L) result be elivere using continuous positive airway pressure
from mineralocorticoi excess or potassium epletion. (CPAP) or BiPAP, with the former proviing continuous
positive pressure support on a single setting an the latter
14. C. Benzoiazepines (particularly lorazepam an mi- proviing ifferent amounts of pressure support uring the
azolam) are commonly use meications for long-term expiratory an inspiratory phases. The theoretical benet of
seation in the intensive care unit (ICU). While they are BiPAP is that it allows for a lower amount of pressure sup-
equally efcacious when aministere in oses of equiva- port uring the expiratory phase, which can help patients
lent potency, they iffer in onset of action an uration. Mi- blow off carbon ioxie. In patients with COPD, this is par-
azolam is highly lipophilic an has a quick onset of action ticularly useful because they are at increase risk for hyper-
when compare with lorazepam. Duration of action is much capnia (D). NPPV is a useful ajunct for respiratory failure
more multifactorial. Initially, it is also etermine heavily by if use in select patients without contrainications. It is cur-
lipophilicity because of rapi reistribution from the central rently recommene for rst-line treatment of acute respira-
nervous system to the peripheral tissues. However, as tis- tory failure from COPD an congestive heart failure (CHF)
sue levels buil up with continuous infusion, the uration with pulmonary eema (A). There are some ata to suggest
CHAPtEr 15 Surgical Critical Care 215

a trial of NPPV shoul be attempte rst in select patients the PAWP, clinical suspicion an a known inciting factor of
with acute hypoxic respiratory failure as this may prevent ARDS in the last 7 ays are sufcient for inclusion in the
intubation. However, failure to improve within the rst 1 to  enition (A). Base on these criteria, this patient woul fall
hours of treatment shoul prompt conversion to intubation. into moerate ARDS (PaO/FiO = 117 mmHg). A lung-pro-
Contrainications to NPPV inclue: cariac or respiratory tective strategy of ventilation using <6 mL/kg of tial vol-
arrest, inability to cooperate an protect the airway or clear ume an higher levels of PEEP continues to be the mainstay
secretions, severely impaire consciousness, nonrespiratory of treatment. In aition, prone ventilation for more than
organ failure, facial trauma or eformity, high risk of aspira- 1 hours per ay has also shown a mortality benet when
tion, recent upper gastrointestinal (GI) anastomosis, antici- institute early in moerate to severe ARDS but not in mil
pate prolonge uration of mechanical ventilation, bullous ARDS. While inhale nitric oxie will improve a patient’s
lung isease such as emphysema (can result in pneumotho- oxygenation, there have been no stuies to ate that have
rax), or hypotension (intrathoracic pressure can ecrease enitively proven that it confers a mortality benet (C).
venous return an thus cariac output). Placement of a naso- Also, there are some ata to inicate that its use coul poten-
gastric tube can potentially complicate NPPV by impairing tially increase the risk of renal impairment. Similarly, HOFV
its ability to form an effective seal an increasing the risk has come uner scrutiny after the OSCILLATE trial, which
of pressure ulcer formation, an it has not been enitively showe a tren towar increase mortality with early initi-
shown to ecrease risk of aspiration (E). ation of HFOV (D). Though the ata are conicte in regar
References: Garpesta E, Brennan J, Hill NS. Noninvasive ven- to the utility of neuromuscular blockae, there is some evi-
tilation for critical care. Chest. 007;13():711–70. ence to suggest that cisatracurium may ecrease the num-
Keenan SP, Sinuff T, Burns KEA, et al. Clinical practice guielines ber of ventilator an ICU ays, as well as potentially provie
for the use of noninvasive positive-pressure ventilation an nonin- a mortality benet (E).
vasive continuous positive airway pressure in the acute care setting.
References: Bein T, Grasso S, Moerer O, et al. The stanar of
CMAJ. 011;183(3):E195–14.
care of patients with ARDS: ventilatory settings an rescue therapies
Siiqui FM, Felton T, Stevens A, Slater R. An unusual contrain-
for refractory hypoxemia. Intensive Care Med. 016;4(5):699–711.
ication to the use of non-invasive ventilation in A&E. Emerg Med J.
Ferguson ND, Fan E, Camporota L, et al. The Berlin enition
010;7(8):615.
of ARDS: an expane rationale, justication, an supplementary
material. Intensive Care Med. 01;38(10):1573–158.
16. E. The percutaneous metho of tracheostomy place- Ferguson ND, Cook DJ, Guyatt GH, et al. High-frequency oscil-
ment has become wiely use in critically ill patients lation in early acute respiratory istress synrome. N Engl J Med.
because there is no nee to transport the patient; the com- 013;368(9):795–805.
plications seem to be equivalent or lower to open trache-
ostomy, an the cost of the proceure is reuce. Initially, 18. C. Pulmonary artery rupture is one of the most reae
there was concern about the safety of this relatively novel complications of pulmonary artery catheter placement. The
metho, especially in obese patients. However, a single-cen- most common etiologies are a balloon that is inate too is-
ter stuy with over 3000 patients emonstrate that percu- tal into the pulmonary system or too much force is use to
taneous tracheostomy placement ha a lower complication obtain a wege pressure. Most of the time, the rupture of the
rate. The stuy also inicate no higher complication rates artery is herale by an initial small volume hemoptysis as
for obese patients, especially with the avent of longer tra- the injury is initially containe within a pseuoaneurysm.
cheostomy tubes. Early tracheostomy islogment is a rela- In suspecte cases, the balloon shoul be left inate an
tively rare but potentially fatal complication associate with the patient taken for catheter-base angiography. By leaving
tracheostomy tube placement. Before the evelopment of a the catheter in place, this allows for an immeiate route of
mature tract, it is possible to inavertently place the trache- access for angiography. Aitionally, the balloon may stop
ostomy into the subcutaneous tissue, which woul manifest further bleeing (B). Overination of the balloon or repeate
with subcutaneous emphysema an oxygen esaturation. attempts at placement have the potential to worsen the ini-
While replacement uring this time perio is possible in tial injury an shoul be avoie (A, E). Open repair in the
experience hans, immeiate enotracheal intubation is operating room is technically possible, but exposure of the
the recommene management (A–C, D). Although ultraso- pulmonary artery branch responsible is a morbi proceure
nography or a tracheostomy obturator oes facilitate easier an time consuming (D). Embolization of the pseuoaneu-
placement, it is still possible to place the tracheostomy tube rysm before full rupture is the preferre treatment moality.
in false tissue tracts (B).
Reference: Dennis BM, Eckert MJ, Gunter OL, Morris JA Jr, 19. B. The patient’s new-onset irregular, narrow complex
May AK. Safety of besie percutaneous tracheostomy in the crit- tachycaria is likely atrial brillation with a rapi ventric-
ically ill: evaluation of more than 3,000 proceures. J Am Coll Surg. ular response. While it is reasonable to attempt meical
013;16(4):858–865. carioversion in a stable patient, conversion to unstable
tachycaria requires immeiate electronic synchronize
17. B. Acute respiratory istress synrome (ARDS) was carioversion (as outline in ACLS). The “synchronization”
reene in 01 uner the Berlin Denition into a three- refers to elivering a low energy shock at the peak of the QRS
tiere graing system consisting of mil (PaO/FiO = 00– complex. This explains why there is a brief pause between
300 mmHg), moerate (PaO/FiO = 100–00 mmHg), an pressing the shock button an elivery of the shock. The
severe (PaO/FiO < 100 mmHg). The purpose of the con- theoretical benet of synchronize carioversion is avoi-
sensus meeting was to correlate a new naming system with ance of the shock uring cariac repolarization, which may
preicte mortality an to remove some outate require- precipitate ventricular brillation. In contrast, unsynchro-
ments (inclusion of the PAWP in the enition). In place of nize carioversion, also known as ebrillation, elivers
216 PArt i Patient Care

a high-energy shock as soon as the button is presse. This Meakins JL, Marshall JC.The gastrointestinal tract: the “motor” of
is reserve for pulseless ventricular tachycaria/brillation MOF. Arch Surg.1986;11():197–01.
in which any elay in shock elivery leas to poorer out-
comes (A, C). Amioarone shoul be use with caution if 21. C. The rst step in the management of oliguria is often
a patient has paroxysmal atrial brillation or the chronicity a ui challenge; the hypovolemic patient will respon with
is unknown because this can potentially chemically convert a corresponent increase in urine output. In a septic patient
the patient to a sinus rhythm an embolize any clot that has (as in this case), oliguria is a result of intrinsic renal ys-
forme (D). Aenosine is typically reserve for monomor- function seconary to wiesprea inammation an thus a
phic narrow complex tachycaria consistent with supraven- ui bolus is unlikely to result in improve urine output. In
tricular tachycaria (E). the surgical ICU, the emphasis is place on “preventative”
critical care management, when appropriate (e.g., gastroin-
20. D. The essential management of the septic patient testinal [GI] prophylaxis to prevent ulcers). In this case, this
inclues early recognition, broa-spectrum IV antibiotics, translates into proviing RRT before the overt presence of
pressors (norepinephrine rst, then vasopressin), an ui renal ysfunction: aciosis, hyperkalemia, volume over-
resuscitation. There is a notable absence of large ranom- loa, an azotemia. RRT is provie by either HD or CRRT
ize, controlle trials emonstrating improve survival of in patients with acute kiney injury an/or renal failure.
ajunctive treatment options asie from the above essentials. Both can be starte using a nontunnele multilumen ialysis
Sepsis is an inammatory cascae that is triggere by injury catheter (D). HD allows for rapi ui an solute removal
or bacterial invasion in an attempt to control the noxious in a 4- to 5-hour time perio, which can result in hypoten-
stimuli. The location an type of pathogen or injury are irrel- sion uring ialysis. In contrast, CRRT works continuously
evant an o not inuence outcomes or survival. The core with a slower unloaing of ui an solutes; this slow but
problem of septic shock is the poor utilization of oxygen, continuous ltration allows for a larger overall amount of
not a lack of perfusion. Bloo elivery (an perfusion to en ui remove. The primary moality for RRT in the septic
organs) is not signicantly impaire an as such maintain- patient is CRRT because HD is relatively contrainicate in
ing the hemoglobin above a certain threshol provies little hypotensive patients requiring pressors owing to the large
benet (A, B). Inammatory meiators chiey impair mito- ui shifts that can occur with HD (A). Aitionally, since
chonrial oxiation by inhibiting pyruvate ehyrogenase septic shock is largely a result of wiesprea inammatory
an cytochrome oxiase an thus estroy the cell’s ability to meiators, CRRT can be use in an ultraltration moe to
prouce its energy currency, aenosine triphosphate (ATP) lower plasma concentrations of inammatory meiators an
(this is terme cytopathic hypoxemia). Aitionally, whole thus ecrease the risk of multiorgan failure. The less porous
oxygen boy consumption is actually increase in early sep- interface use in the ltration membrane of HD is inferior
tic shock as inammatory meiators inuce prouction of to CRRT in removing inammatory meiators (B). However,
toxic oxygen free raicals (respiratory burst) in an attempt this benet is theoretically lost if CRRT is not employe early
to break own bacterial cell membranes, enature proteins, in the course of septic shock before wiesprea multiorgan
an estroy DNA (C). Although enogenous antioxiants amage an/or failure. Until recently, the timing of CRRT
are plentiful in homeostasis an prevent free raicals from was up for ebate because there were no large ranomize
causing havoc on normal functioning cells, the septic patient stuies emonstrating improve survival. However, in 016,
has an exaggerate, large, an wiesprea prouction of the ELAIN trial publishe in JAMA was the rst large ran-
free raicals that excee enogenous antioxiant protection; omize controlle trial emonstrating improve 90-ay
this is known as oxiant stress. Aitionally, inammatory mortality (39% versus 54%) with the initiation of early CRRT
meiators inuce prouction of nitrous oxie resulting in (ene as within 8 hours of acute kiney injury onset). The
systemic vascular ilation an high cariac output (from uration of renal replacement therapy an length of hospital
increase heart rate). However, ue to the increase stress stay (but not ICU stay) were signicantly shorter in the early
put on the cariovascular system, cariac output begins to group versus the elaye group (E). However, there was
fall late in untreate septic shock an portens a poor prog- no ifference in the rate of requirement of RRT after ay 90
nosis. The systemic venoilation leaves the majority of the between the two groups. Patients with an increase level of
intravascular volume ormant in the venous system, which IL-8 ha an increase risk of RRT epenence after hospital
is the basis of why ui resuscitation is essential early in ischarge.
septic shock. However, a large positive ui balance shoul References: Honore PM, Jamez J, Wauthier M, et al. Prospective
be avoie as it is associate with increase mortality. An evaluation of short-term, high-volume isovolemic hemoltration
lastly, although there is a systemic vascular ilation, there on the hemoynamic course an outcome in patients with intrac-
is relative splanchnic vasoconstriction resulting in gut isch- table circulatory failure resulting from septic shock. Crit Care Med.
emia an mucosal injury. This allows for enteric pathogen 000;8(11):3581–3587.
translocation an aitional subsequent inammatory Zarbock A, Kellum JA, Schmit C, et al. Effect of early vs elaye
meiators resulting in further splanchnic vasoconstriction initiation of renal replacement therapy on mortality in critically ill
an mucosal injury; this self-sustaining process of contin- patients with acute kiney injury: the ELAIN ranomize clinical
trial. JAMA. 016;315(0):190–199.
ue inammation is known as the “motor” of multiorgan
failure (E).
References: Abraham E, Singer M. Mechanisms of sepsis- 22. E. As with all trauma patients, the primary survey
inuce organ ysfunction. Crit Care Med. 007;35(10):408–416. begins with checking airway an breathing. The patient is
Babior BM. The respiratory burst of phagocytes. J Clin Invest. exhibiting signs of a tension pneumothorax with evience
1984;73(3):599–601. of hypotension, tracheal eviation, an ecrease breath
CHAPtEr 15 Surgical Critical Care 217

souns over the right hemithorax. Tension pneumothorax inclue hyperactive reexes, muscle tremors, an tetany
is a clinical iagnosis an oes not require raiographic with a positive Chvostek sign. Severe eciencies can lea to
conrmation before instituting therapy (B). Treatment elirium an seizures. Electrocariographic changes inclu-
options inclue neele thoracostomy rst followe by chest ing prolonge QT an PR intervals, ST-segment epression,
tube insertion (D). Intubation an application of positive attening or inversion of P waves, torsae e pointes, an
pressure ventilation shoul not occur before ecompress- arrhythmias can also be seen (B–E). When hypokalemia or
ing a tension pneumothorax because this will worsen the hypocalcemia coexists with hypomagnesemia, magnesium
tension physiology an further impee preloa an cariac shoul be aggressively replace to assist in restoring potas-
output (C). If the patient continues to ecompensate after sium or calcium homeostasis.
tube thoracostomy, intubation can be consiere. Resusci-
tative thoracotomy or tracheostomy is not inicate in this 26. C. Treatment of hypermagnesemia inclues withhol-
patient (A). ing exogenous sources of magnesium, correcting volume
ecits, an correcting aciosis if present. To manage acute
23. B. Hemorrhagic shock is a form of hypovolemic shock symptoms, calcium chlorie shoul be aministere to
an the most common cause of shock in trauma patients. In antagonize the cariovascular effects (C). If elevate levels or
response to hypovolemia, the sympathetic an cariovascu- symptoms persist, ialysis is inicate (E). Insulin, extrose,
lar systems increase the heart rate, myocarial contractility, an ialysis are typically use in the treatment of hyperka-
an SVR to maintain bloo pressure. This response occurs lemia (A, B, D).
seconary to an increase in norepinephrine secretion an
a ecrease in vagal tone. The cariovascular system also 27. C. PEEP increases intrathoracic pressure, which may
reistributes bloo ow to the brain, heart, an kineys an result in a ecrease cariac output via ecrease preloa,
shunts it away from the skin, muscle, an gastrointestinal particularly in patients who are hypovolemic (B). PEEP
tract. The kineys respon to hemorrhagic shock by increas- oes not ecrease lung water, reuce vascular permeability,
ing reabsorption of soium an water, which results in a or hasten the resolution of pulmonary eema. PEEP may
small volume of concentrate urine. When a patient is ae- shift some eema ui from the alveolar to the extraalveo-
quately resuscitate, the rst sign is an improvement in urine lar interstitial space, but PEEP oes not reuce the overall
output (A, C–E). There are 4 classes of hemorrhagic shock: egree of pulmonary eema (E). PEEP is often an effective
Class 1 (up to 750 cc or <15% of total bloo volume loss) oes way of increasing arterial oxygen content by increasing FRC
not have any hemoynamic changes; Class  (750–1500 cc or through the recruitment of collapse or atelectatic alveoli
15%–30%) can have tachycaria, ecrease pulse pressure, in patients who have ecrease lung compliance, thereby
an typically normal bloo pressure; Class 3 (1500–000 cc improving SaO. PEEP oes not affect PCO nor oes it alter
or 30%–40%) can have tachycaria, ecrease pulse pressure, cariac contractility (A). PEEP can improve cariac output
an ecrease bloo pressure; Class 4 (>000 cc or >40%) can by reucing left ventricular (LV) afterloa an is a useful
have tachycaria, ecrease pulse pressure, an signicantly ajunct in patients with CHF exacerbations (D). It is import-
ecrease bloo pressure, which may be incompatible with ant to keep in min, however, that changing the PEEP will
life. not have an immeiate effect on oxygenation because it takes
time to increase the FRC.
24. C. Sepsis prouces high-output cariac failure with ele-
28. E. Although rare, the incience of bacterial contami-
vate cariac inex. If this goes untreate, the cariac inex
nation of infuse bloo is higher than the incience of viral
will eventually ecrease. SVR is ecrease ue to toxins that
infection transmission an can be acquire as a result of envi-
prouce vasoilation (B). This is reecte in a low systemic
ronmental contamination (collection bags or contaminate
bloo pressure. Central venous pressures are low from the
water baths) or from the onor’s skin, bloo, or phleboto-
loss of intravascular volume ue to increase capillary per-
mist’s skin. Gram-negative organisms, especially Yersinia
meability (A). Wege pressures are generally unaffecte (D).
enterocolitica an Pseudomonas species, which are capable of
SvO will be high because the tissues are unable to extract
growth at 4°C (39.°F), are the most common cause (A–D).
oxygen from the bloo for consumption (E).
Gram-positive organisms are more frequently encountere as
platelet contaminants. Clinical manifestations inclue fever,
25. A. Magnesium epletion is a common problem in hos-
chills, abominal cramps, vomiting, an iarrhea. There
pitalize patients, particularly in the intensive care unit. The
may be hemorrhagic manifestations an increase bleeing.
kiney is primarily responsible for magnesium homeosta-
If the iagnosis is suspecte, the transfusion shoul be is-
sis through regulation by calcium/magnesium receptors
continue an resuscitative efforts initiate. Bloo shoul be
on renal tubular cells that sense serum magnesium levels.
culture an a workup for a transfusion reaction shoul be
Hypomagnesemia results from a variety of causes ranging
performe. Emergency treatment inclues oxygen, arener-
from poor intake (starvation, alcoholism, prolonge amin-
gic blocking agents, an the aministration of broa-spec-
istration of IV uis, an total parenteral nutrition with
trum antibiotics.
inaequate supplementation of magnesium), increase
References: Mullins R. Shock, electrolytes, an ui. In:
renal excretion (alcohol, most iuretics, an amphotericin
Townsen CM, Jr, Beauchamp RD, Evers BM, Mattox KL, es. Sabis-
B), gastrointestinal losses (iarrhea), malabsorption, acute ton textbook of surgery: the biological basis of modern surgical practice.
pancreatitis, iabetic ketoaciosis, an primary aloste- 17th e. Philaelphia: W.B. Sauners; 004:67–11.
ronism. Magnesium epletion is characterize by neuro- Peitzman A. Shock, electrolytes an ui. In: Brunicari FC,
muscular an central nervous system hyperactivity, an Anersen DK, Billiar TR, etal., es. Schwartz’s principles of surgery.
symptoms are similar to those of calcium eciency. Signs 8th e. New York: McGraw-Hill; 005:85–108.
218 PArt i Patient Care

29. A. Intraabominal hypertension is ene as a sustaine 32. D. Hepatorenal synrome is a functional renal problem
increase in intraabominal pressures greater than 1 mmHg. that likely results from relative hypovolemia, splanchnic
The increase pressure may be acute, subacute, or chronic. an peripheral arterial vasoilation, an intense vasocon-
Abominal compartment synrome (ACS) is ene as sus- striction of the renal circulation (B, C). The synrome is
taine intraabominal pressures greater than 0 mmHg asso- probably the nal consequence of extreme unerlling of
ciate with new organ ysfunction. ACS occurs in patients the arterial circulation seconary to arterial vasoilation
who have sustaine multiple traumas, severe burns, or retro- in the splanchnic vascular be. It is characterize by azo-
peritoneal injuries; have unergone an operation for massive temia, oliguria, very low urinary soium (<10 mEq/ay),
intraabominal infection; or have unergone a complicate, an a high urine osmolarity. The prognosis is poor. Type
prolonge abominal operation. Massive IV ui resuscita- I is mainly associate with acute liver failure or alcoholic
tion with resultant thir spacing of ui an marke bowel cirrhosis, but it can evelop in any other form of liver fail-
wall eema places patients at high risk of the evelopment of ure. It is characterize by rapi eterioration of renal func-
this complication. The symptoms an signs inclue progres- tion, with a marke increase in serum creatinine an bloo
sive abominal istention, increasing peak airway pressure, urea nitrogen over a short perio of time (E). The optimal
ecrease cariac output, an oliguria. These complications treatment is liver transplantation, but the patients may not
are the result of the abominal pressure ecreasing venous receive the transplant in time. Hyponatremia an hyper-
return from the inferior vena cava an renal veins an from kalemia are typical. Type II is a more stable form (A). The
ecrease pulmonary compliance. Renal failure, severe pul- ecrease in the glomerular ltration rate an the increase in
monary compromise, an intracranial hypertension can creatinine are moerate. It occurs mostly in patients with a
eventually evelop in patients. Intraabominal pressures relatively preserve hepatic function. In one stuy, a combi-
transuce from the blaer can be reaily measure by nation of miorine, an α-agonist, an octreotie improve
instilling 5 mL of saline into the aspiration port of a Foley 30-ay survival.
catheter with the rainage tube clampe. An 18-gauge nee- Reference: Ginès P, Guevara M, Arroyo V, Roés J. Hepatorenal
le attache to a pressure transucer may then be inserte synrome. Lancet. 003;36(9398):1819–187.
into the aspiration port at which point the system shoul be
zeroe at the level of the miaxillary line. A pressure of greater 33. B. The presentation is consistent with neurogenic
than 0 mmHg with evience of physiologic compromise as shock. Finings suggestive of neurogenic shock inclue
manifeste by renal, respiratory, or neurologic compromise hypotension with relative braycaria, warm, well-per-
is consiere iagnostic. Treatment consists of opening the fuse extremities reflecting loss of sympathetic tone,
abomen or paracentesis in select cases. evience of a high spinal cor injury, an priapism (sus-
taine erection ue to unoppose parasympathetic stim-
30. D. Patients with unerlying cariac isease are at ulation). In a patient with a high cervical spine injury an
increase risk of arrhythmias, seeming to be more sensitive evience of hypercarbic or ventilatory failure, the first
to hypoxia, hypercarbia, an electrolyte abnormalities than step is to secure an airway. The phrenic nerve is supplie
patients without heart isease. VT is a serious wie-complex by the C3 to C5 nerve roots. Thus, patients with an injury
tachycaria that warrants immeiate treatment because it above C5 will routinely require ventilatory support.
may progress to unstable ventricular rhythms. Management After the airway is secure an ventilation is aequate,
of VT is epenent on the stability of the patient. For those flui resuscitation an restoration of intravascular vol-
without hypotension, altere mental status, signs of shock, ume will often improve perfusion in neurogenic shock.
chest pain, or acute heart failure, pharmacologic treatment Most patients with neurogenic shock will respon to the
with antiarrhythmic infusions is inicate. Amioarone is restoration of intravascular volume alone, with satisfac-
the rug of choice, although procainamie an sotalol are tory improvement in perfusion an resolution of hypo-
also acceptable provie that the QT interval is not pro- tension. It is always important to rule out hypovolemia
longe (B). If the patient exhibits altere mental status an/ ue to hemorrhage in the trauma setting. In aition,
or hypotension, immeiate synchronize carioversion is one must always be aware that in the presence of spinal
inicate with an initial recommene ose of 100 J (D). cor injury, one cannot rely on the abominal examina-
Consieration shoul be given to the aministration of sea- tion. Thus, an abominal an pelvic CT scan woul be
tion or analgesia before carioversion, if possible. Debril- inicate to rule out visceral injury. If the patient oes
laton an epinephrine are inicate in pulseless VT (A, C). not respon to fluis, aministration of vasoconstrictors
Diltiazem is useful in atrial tachycaria but has no place in will improve peripheral vascular tone, ecrease vascular
VT (E). Most importantly, a search for an correction of any capacitance, an increase venous return but shoul only
reversible causes shoul be unertaken. be consiere once hypovolemia is exclue as the cause
of the hypotension an the iagnosis of neurogenic shock
31. B. All are consequences of acute kiney injury; however, is establishe (A, C–E). Restoration of bloo pressure an
hyperkalemia is generally the most immeiately life-threat- circulatory perfusion is also important to improve perfu-
ening complication an can preispose the patient to ventric- sion to the spinal cor, prevent progressive spinal cor
ular tachycaria an brillation (A, C–E). ischemia, an minimize seconary cor injury.
Trauma
NAVEEN BALAN, CAITLYN BRASCHI, AND DENNIS KIM 16
ABSITE 99th Percentile High-Yields
I. Traumatic Brain Injury (TBI)
A. Subural hematoma (SDH): craniectomy if, >10 mm in size or >5 mm miline shift; more consistent with
nonacciental trauma
B. Epiural hematoma (EDH): craniectomy if >15 mm in size or >5 mm miline shift
C. Massive subarachnoi hemorrhage: CTA hea to evaluate for rupture aneurysm or arteriovenous
malformation
D Start VTE chemoprophylaxis 48 hours from most recent stable CT; low-molecular weight heparin
preferre in TBI

II. Spinal Cor Injury (SCI)


A. Unstable spine injury: isruption of /3 of the longituinal spinal columns (requires surgery)
B. High SCI (above T6), concern for neurogenic shock acutely (hypotension, vasoilation with warm skin,
braycaria, or inappropriately normal heart rate for trauma setting); long term concern (months later)
for autonomic ysreexia with braycaria, iaphoresis, an uncontrolle hypertension

Injury Location Mechanism Management


Hangman’s fracture Cervical Hyperextension of the neck, Traction and external
(bilateral pedicles) caused by hanging, diving immobilization (Halo) vs spinal
fusion
Dens fracture C2 (odontoid process) Hyperextension of the neck, Type I is above the base and
falls in the elderly considered a stable fracture.
Type II (which extends to the
base of dens) is unstable and
need surgical ęxation. Type
III fractures extend into the
C2 vertebral body—these tend
to have a beĴer healing rate
than type II and rarely require
surgery.
Chance fracture Thoracolumbar Flexion-distraction injury Orthotic brace for low-grade
(horizontal disruption from rapid deceleration injury, surgery if neurologic
of all columns) during blunt trauma deęcits or ligamentous injury;
also higher risk of hollow
viscous injury

219
220 PArt i Patient Care

III. Neck Trauma


A. Screening for blunt cerebrovascular injury (caroti or vertebral artery): use expane Denver criteria

Risk factors for BCVI Signs/symptoms of BCVI


LeFort II/III or mandible fracture Pulsatile bleeding from neck or nasal/oral cavity
Complex skull fracture or basilar skull fracture Carotid bruit
Traumatic brain injury (GCS <6) Expanding hematoma
Cervical spine injury (subluxation, ligamentous injury, Focal neurologic deęcit: hemiparesis, Horner syndrome
transverse foramen fracture, vertebral body fracture, any (ptosis, miosis, anhidrosis)
C1-C3 fracture
Near hanging with anoxic brain injury Stroke on CT HEAD or MRI BRAIN
Clothesline injury with neck seat belt sign Neurologic deęcit not explained by CT HEAD
Scalp degloving

B. Esophagus (ysphagia, rooling, oynophagia, pneumomeiastinum): start with water-soluble contrast


stuy; only 1% of pneumomeiastinum after blunt trauma have an esophageal injury, most cases are
from air issecting along pulmonary vasculature in pneumothorax (Macklin effect)
1. Small, containe injury: NPO with broa-spectrum antibiotics (incluing antifungal)
. Free-perforation injury, stable: enlarge efect to easily view mucosal injury extent, close in  layers,
an use muscle ap (strap muscle, intercostal muscle) to buttress repair
3. Devitalization or > cm segmental loss: esophagectomy, wie rainage, an reconstruction if stable or
iversion with esophagostomy (spit stula) if unstable
4. Surgical access: if cervical esophagus use left neck incision, if upper /3 of esophagus use right
posterolateral thoracotomy, if istal 1/3 esophagus use left posterolateral thoracotomy
C. Tracheal injury: ebrie, repair with primary closure in one layer (two layers coul lea to stenosis)
using absorbable suture an buttress with strap muscles
D. Surgical airway: cricothyroiotomy preferre over tracheostomy; chilren (age <1- years) avoi surgical
cricothyroiotomy (risk of subglottic stenosis) an consier neele cricothyroiotomy with jet-neele
insufation

IV. Thoracic Trauma


A. Cariac injuries
1. Blunt cariac injury (BCI) screening
a) ECG, troponin in all patients suspecte of sustaining BCI (normal rules out injury)
b) Any abnormality on EKG or elevate troponin: amit for cariac monitoring
c) Hemoynamic instability or new persistent arrhythmia: obtain ECHO
B. Rib fractures are the most common blunt thoracic trauma injury
1. Meical management: aggressive pulmonary toilet, multimoal pain control, serratus anterior,
paravertebral, or intercostal nerve block or epiural (latter improves mortality in elerly)
. Rib plating: inicate in patients with severe ail chest either causing respiratory istress (usually
hypoventilation) or failure to extubate; compare to nonop management, rib xation increases
spirometry volumes, increases hospital costs, an ecreases pneumonia rate, however, it oes not
ecrease mortality rate or narcotic requirements
C. Diaphragmatic injury: repair after life-threatening injuries is manage, transabominal approach favore
over transthoracic to evaluate bowel, use nonabsorbable permanent suture; if penetrating trauma to left
thoracoabominal area in an asymptomatic patient, observe for 8 hours for evience of peritonitis, if
none, then perform iagnostic laparoscopy to rule out iaphragm injury

V. Abominal Trauma
A. Pancreas injuries (most commonly after penetrating trauma)
1. No uctal involvement (graes I–II): wie rainage
. Distal injury with uctal involvement (grae III): istal pancreatectomy with splenic preservation (if
hemoynamically stable)
CHAPtEr 16 Trauma 221

3. Pancreatic hea transection or massive isruption (graes IV–V): with rainage or amage control
operation, consier stage Whipple
4. If uct involvement is unclear intraoperatively, consier intraoperative cholangiogram or perform
wie rainage with postoperative ERCP/MRCP
B. Duoenal injuries
1. Hematoma (grae I): trial nonoperative (NPO, TPN) for  weeks, exploration if nonop fails
. Lacerations (II–IV) of < 50% circumference: favor two-layer primary repair
3. Lacerations (II–IV) of >50% circumference: segmental resection with primary anastomosis for all
segments except D
C. Colorectal injuries
1. Colon an intraperitoneal rectal:
a) Nonestructive (<50% circumference): ebriement an primary repair
b) Destructive (>50% circumference): segmental resection with 1° anastomosis if stable, segmental
resection left in iscontinuity with planne n look operation if unstable
. Extraperitoneal rectum: proximal iversion alone (no presacral rainage or washout)

VI. Retroperitoneal hematomas


A. Zone I: explore all hematomas
B. Zone II: selective exploration for expaning hematoma (blunt or penetrating)
C. Zone III: explore penetrating, o not explore blunt

VII. Orthopeic an Neurovascular Injury Patterns

Associated neurovascular
Fracture/Injury injury Complication/deęcit
Anterior (more common) shoulder Axillary nerve injury Weak shoulder abduction
dislocation
Posterior shoulder dislocation (e.g., seizures) Axillary artery injury
Humeral shaft fracture Radial nerve palsy Wrist-drop
Supracondylar humerus fracture Brachial artery injury Forearm compartment syndrome,
Volkmann ischemic contracture
Colles fracture (distal radius) Median nerve compression Pain, paresthesias in digits 1–3 ½
Scaphoid fracture Snuġox tenderness, avascular
necrosis; often normal initial XR
Posterior (more common) hip dislocation Sciatic nerve injury (peroneal
(adducted and internally rotated) branch)
Posterior knee dislocation Popliteal artery injury
Fibula head fracture (or prolonged Peroneal nerve injury Foot drop
lithotomy)

Exposure/maneuver Location
Right posterolateral thoracotomy Mid esophagus
Left posterolateral thoracotomy Distal esophagus, descending aorta (distal to left subclavian
takeoě)
Left anterolateral thoracotomy Left distal subclavian artery
Median sternotomy Ascending and arch of aorta, innominate artery, bilateral
common carotid artery, superior vena cava, proximal right
and left subclavian artery
Left infraclavicular incision Left mid-subclavian artery
Kocher maneuver Head of pancreas, SMV, SMA
Left medial visceral rotation (MaĴox maneuver) Aorta, celiac trunk, SMA, left renal artery, common iliac
arteries
Right medial visceral rotation (CaĴell-Braasch maneuver) Inferior vena cava, right renal vessels, common iliac veins
Pringle maneuver Control intrahepatic liver hemorrhage, max clamping 30–60 min
222 PArt i Patient Care

VIII. Acute Compartment Synrome (ACS)


A. Risk factors: open fractures > close fractures, crush injuries, young, male, long bone fracture (e.g., tibia,
raius), high voltage burns, >6 hours ischemia with reperfusion, combine arterial/venous injury
B. Diagnosis: high clinical suspicion, compartment pressure >30 mmHg, elta P (compartment pressure/
iastolic pressure) <30 mmHg
C. Upper extremity:
1. Forearm compartments: lateral (mobile wa), orsal extensor, volar (contains meian an ulnar
nerves)
. Fasciotomy: S-shape volar incision (often inclues carpal tunnel if neee), linear orsal incision
(3 cm istal to lateral epiconyle towar the miline of the wrist)
D. Lower extremity:
1. Thigh compartments: anterior, posterior, meial
. Leg compartments: anterior (most common site ACS—sensory ecit of rst web space seconary to
eep peroneal nerve in this compartment), lateral, supercial posterior an eep posterior
a) Fasciotomy—Two incision, four compartment technique: Meial incision 1 to  cm meial to tibia
(take soleus off tibial periosteum; posterior compartments), Lateral incision along anterior margin
of bula (anterior an lateral compartments; risk of injury to supercial peroneal nerve—most
common) (Figs. 16.1–16.3).

Fig. 16.1 Pancreatic Injury Graes.


CHAPtEr 16 Trauma 223

Fig. 16.2 The Cattell-Braasch Maneuver.

Fig. 16.3 The Mattox Maneuver.


224 PArt i Patient Care

Questions
1. A 3-year-ol male presents to the emergency 4. A 9-year-ol male presents to the ED following a
epartment with a left parasternal stab woun. high-spee MVC. He complains of neck pain an
His heart rate is 110 beats per minute an he is neurologically intact on examination. Hea
bloo pressure is 130/70 mmHg. FAST shows a CT is negative. He unergoes CT imaging which
pericarial effusion. What is the next best step? reveals an isolate C lateral mass fracture. CT
A. Formal echocariogram angiography of the neck is signicant for a small
B. Subxiphoi pericarial winow pseuoaneurysm of the left vertebral artery. What
C. Left anterolateral thoracotomy is the most appropriate management of this lesion?
D. Meian sternotomy A. Initiate antithrombotic therapy with
E. Pericariocentesis unfractionate heparin an repeat imaging in
7 ays
2. A 36-year-ol male presents with right upper B. Enovascular stent placement
quarant pain, jaunice, an melena. He was C. Operative exploration
recently ischarge for a blunt hepatic injury D. Observation
following a high-spee motor vehicle with injuries E. Thrombin injection
that inclue a liver injury that was manage
nonoperatively. His heart rate is 10 beats per 5. A 3-year-ol woman with no meical history
minute, bloo pressure is 80/60 mmHg. IV uis presents to the ED following a hea-on motor
an bloo transfusion are given with repeat bloo vehicle crash at 45 mph. Her heart rate is 98 beats
pressure of 90/70 mmHg an heart rateof 110beats per minute, bloo pressure is 11/7 mmHg, an
per minute. He is afebrile. Laboratory values SpO is 98% on room air. Pan-CT shows isolate
inclue a hematocrit of 4%, a normal white bloo anterior pneumomeiastinum. She enies
cell count, total bilirubin of 3.5 mg/l, an alkaline symptoms. What is the next best step?
phosphatase of 400 IU/L. What is the next best A. Esophagoscopy, laryngoscopy, bronchoscopy
step? B. Observation
A. Enoscopic retrograe C. Bilateral tube thoracostomy
cholangiopancreaticography (ERCP) D. CT thorax, abomen with oral contrast
B. Angioembolization E. Water-soluble swallow stuy
C. Abominal ultrasoun
D. Enoscopy 6. Which of the following orthopeic injuries is
E. CT abomen with IV contrast correctly paire with its commonly associate
neurovascular ning?
3. A 5-year-ol man presents with a gunshot A. Supraconylar humerus fracture: Volkmann
woun (GSW) to the buttocks. Abominal ischemic contracture
examination is unremarkable, an the patient is B. Distal raius fracture: claw-han eformity
hemoynamically stable. Proctoscopy reveals C. Posterior hip islocation: obturator nerve injury
bloo an stool in the istal rectal vault, but D. Posterior shouler islocation: axillary nerve
an injury cannot be ientie. Compute injury
tomography (CT) scan of the abomen an pelvis E. Scaphoi fracture: wrist rop
with rectal contrast is unremarkable. Which of the
following is the best management option?
A. IV antibiotics with close observation
B. A proximal iverting colostomy
C. Exploratory laparotomy with primary closure
of rectal injury, iverting colostomy, istal
rectal irrigation, an presacral rainage
D. Presacral rainage an IV antibiotics
E. Abominal perineal resection
CHAPtEr 16 Trauma 225

7. A 45-year-ol male presents after blunt trauma to 11. A 35-year-ol male is taken to the ED after being
the neck. He is hemoynamically stable without stabbe in the right abomen. He complains of
respiratory istress. He is afebrile without minimal abominal pain with no reboun or
leukocytosis an enies oynophagia. CT shows guaring. During local woun exploration (LWE),
only subcutaneous emphysema. Bronchoscopy it appears that the anterior fascia is not violate.
emonstrates a 3-cm laceration in the cervical His vital signs are normal. Which of the following
trachea. The surrouning tissue is viable. What is is the most appropriate management?
the best next step in management? A. Amission for 4-hour observation
A. Antibiotics, voice rest, an repeat B. Discharge after 6 hours if abominal exam
bronchoscopy oes not change
B. Primary repair with permanent suture C. Compute tomography (CT) scan of the
C. Primary repair with absorbable suture in one layer abomen an pelvis
D. Primary repair with absorbable suture in two D. Focuse assessment with sonography for
layers trauma (FAST)
E. Tracheostomy through the laceration an E. Discharge home
closure of remaining injure trachea
12. A 45-year-ol male is brought to the ED after a
8. A 45-year-ol man presents with secon- an GSW to the right leg. He is hypotensive in the
thir-egree burns to the anterior surface of both ED, with a large amount of bloo loss at the
arms an entire right leg. He also has supercial scene. Massive transfusion protocol is initiate.
burns to both hans. What is his estimate total Following interposition vein graft for a supercial
boy surface area (TBSA) burne? femoral artery transection, he is amitte to the
A. 5% ICU for observation. The following morning he
B. 7% is foun to be oliguric, has rising peak airway
C. 9% pressures, an has a istene abomen. Which
D. 36% of the following woul be expecte in this patient?
E. 38% A. Increase pulmonary compliance
B. Increase functional reserve capacity (FRC)
9. A -year-ol male presents to the emergency C. Decrease pulmonary vascular resistance
epartment (ED) after a gunshot woun (GSW) D. Increase pulmonary capillary wege pressure
to the abomen. He is hypotensive. Which of E. Increase venous return
the following intravenous (IV) routes is the
most appropriate way to eliver rapi ui 13. Which of the following is true with regars to
resuscitation to this patient? burn injury?
A. 18-gauge peripheral catheter A. Supercial partial-thickness burns o not have
B. 0-gauge peripheral catheter blistering
C. 6-French femoral vein central line catheter B. Full-thickness thir-egree burns can involve
D. 7-French subclavian vein central line catheter unerlying fascia
E. 7-French internal jugular vein central line C. Deep partial-thickness burns have a loss of
catheter hair follicles
D. Deep partial-thickness burns often heal
10. Which of the following is the leaing cause of spontaneously
eath in the trauma patient reaching the hospital E. Supercial partial-thickness burns are not
alive? painful
A. Hea injury
B. Hemorrhagic shock 14. Which of the following is correct regaring
C. Multiorgan failure common topical antimicrobials use in burn care?
D. Sepsis A. Mafenie acetate leas to a respiratory
E. Cariac injury alkalosis
B. Silver sulfaiazine has broa coverage against
Pseudomonas
C. Silver nitrate can be use in patients with a
sulfa allergy
D. Mafenie acetate is not effective in patients
with eschars
E. Silver sulfaiazine can lea to electrolyte
abnormalities
226 PArt i Patient Care

15. Which of the following is an inication to transfer 18. A 33-year-ol alcoholic patient presents to the
a patient to a burn center? ED after a high-spee MVC. She appears to
A. 45-year-ol female with rst-egree burns 30% be inebriate an combative an is promptly
TBSA intubate. Compute tomography (CT)
B. 10-year-ol female with thir-egree burns 4% emonstrates a Chance fracture at L1, an free
TBSA ui in the abomen, but no evience of soli
C. 1-year-ol male with a chemical burn to the organ injury. Vitals are normal an stable. The
right han next step in management is:
D. 30-year-ol female with secon-egree burns A. Magnetic resonance imaging (MRI) of the
18% TBSA spine
E. 71-year-ol female with secon-egree burns B. Amission to the ICU for close monitoring
8% TBSA C. Exploratory laparotomy
D. FAST scan
16. A 8-year-ol male with morbi obesity arrives at E. Repeat CT scan of the abomen in 6 hours
the ED after suffering an electrical shock. He was
working on his car at the time of injury. He has a 19. A 4-year-ol female presents to the ED with
burn mark on his han an his forearm appears abominal pain after an MVC. CT scan shows
swollen. Which of the following is true regaring contrast extravasation in the spleen with a
this patient? signicant hemoperitoneum. Heart rate is
A. The source of the shock was likely a irect presently 10 beats per minute an bloo
current pressure (BP) is 90/70 mmHg. Hemoglobin is
B. Renal failure is the main cause of eath in 7.1 g/L. She is a Jehovah’s Witness an refuses
those who survive the initial injury bloo transfusions. Which of the following is the
C. He likely ha repetitive, tetanic muscle most appropriate next step in management?
contractions at the time of electrocution A. Document refusal of bloo proucts an
D. His boy habitus will likely protect him from aminister normal saline to keep BP above
eep thermal injury 100 mmHg
E. Re urine on amission is suggestive of B. Document refusal of bloo proucts an
blaer injury perform a splenectomy
C. Document refusal of bloo proucts an
17. A 36-year-ol alcoholic female arrives at the ED perform angiography with embolization
uring a winter storm with a frostbite to the right D. Document refusal of bloo proucts an
arm an han. She passe out in a park with her consult the hospital ethics committee
arm expose on a freezing metallic bench. Her E. Aminister  units of packe re bloo cells
right han has several areas of what appear to be given life-threatening situation an perform
hemorrhagic bullae. It has been 7 hours since she splenectomy
was brought to the ED. Which of the following is
true regaring this patient? 20. A 9-year-ol male arrives at the ED after a
A. She likely has a secon-egree frostbite high-spee MVC with a Glasgow Coma Scale
B. She shoul receive early ebriement of (GCS) of 4. He has a cervical collar that was
obviously necrotic tissue place by emergency meical services (EMS).
C. She will likely respon favorably to tissue He is intubate an taken for a CT scan, which
plasminogen activator (tPA) treatment emonstrates a large subural hemorrhage an
D. Reperfusion injury is an important contributor iffuse punctate hemorrhage with no evience of
to the amage seen with her injury cervical spine injury. He is amitte to the ICU.
E. Rewarming in warm water shoul be one With regar to the management of the cervical
graually collar, which of the following is recommene?
A. Remove immeiately
B. MRI cervical spine (c-spine) an remove the
cervical collar if there are no injuries ientie
C. Continue cervical collar until the patient can
be clinically evaluate
D. Exchange the cervical collar place by EMS
with a soft-collar
E. Exchange the cervical collar place by EMS
with a soft-collar an orer MRI c-spine
CHAPtEr 16 Trauma 227

21. Which of the following is true regaring 24. A 8-year-ol male presents to the ED  ays after
pneumothorax in the trauma patient? being involve in a bar ght where he punche
A. A small asymptomatic pneumothorax another patron in the mouth. His right han
ientie on CT scan will resolve within appears to have a soft-tissue infection. Which of
4hours using 100% inspire supplemental the following is the most likely pathogen?
oxygen A. Treponema pallidum
B. A small asymptomatic pneumothorax shoul B. Prevotella spp.
be manage with a tube thoracostomy if the C. Hepatitis C
patient is to unergo general anesthesia D. Propionibacterium spp.
C. A small asymptomatic pneumothorax in a E. Bacteroides
ventilate patient in the ICU, iscovere on
rereview of amission CT, shoul be manage 25. A 45-year-ol male arrives at the ED with a GSW
with a tube thoracostomy to the hea. He has ecline organ onation on
D. A persistent air leak ientie on postinjury his river’s license registration which is several
ay 3 is best manage with VATS years ol. He is eclare to be brain ea the
E. Penetrating injuries leaing to pneumothorax following morning. His parents an sister y in
have concomitant hemothorax less than half of from out of state. His sister has en-stage renal
the time isease an woul like to receive her brother’s
kiney because she states he was teste “an
22. A 31-year-ol female with obesity presents to foun to be a match.” His parents are saene
the ED after a large refrigerator fell on her. She by their son’s passing but agree that their
is complaining of severe pain in her hips. Her aughter shoul receive the kiney an that
hemoglobin is 7.9 g/L. Her heart rate is 18 their son woul have wante this. The treating
beats per minute, an her systolic bloo pressure physician shoul:
is 105 mmHg. She has no evience of extremity A. Arrange for organ harvesting an coorinate
injuries, an istal pulses are normal. She has with a transplant surgeon to perform the
an unstable pelvis, so a pelvic biner is applie. kiney transplant
Massive transfusion protocol is initiate. She is B. Contact an organ onation service to facilitate
rushe to the angiography suite an unergoes a iscussion with the family
embolization, then stabilizes. The following ay, C. Remove the patient from ventilator support
her CK levels rise to 40,000 an her urine turns D. Aminister a lethal ose of morphine sulfate
re-tinge. The most likely source is the muscles E. Consult the hospital ethics committee
of her:
A. Thighs 26. A 45-year-ol male presents to the ED following
B. Buttocks a high-spee MVC with evience of severe facial
C. Abominal wall fractures an bilateral lower extremity eformities.
D. Arms Parameics report a signicant amount of bloo
E. Calves in his airway, an the patient’s respirations are
being assiste with bag-valve mask ventilation.
23. A 46-year-ol woman presents to the ED On exam, the patient is hemoynamically stable
hemoynamically stable after a high-spee with an O saturation of 85% on a nonrebreather
MVC. A CT scan of the abomen an pelvis mask an GCS is 7. Attempts at rapi sequence
reveals a right perinephric hematoma with intubation are unsuccessful because of the inability
a eep laceration in the inferior aspect of the to visualize the airway as a result of ongoing
renal parenchyma with some localize urine bleeing. Attempts at bagging become more
extravasation within the collecting system. ifcult. Which of the following is the next best
Management consists of: step in management?
A. Observation A. Neele cricothyroiotomy
B. Right nephrectomy B. Nasotracheal intubation
C. Attempt at partial nephrectomy C. Surgical cricothyroiotomy
D. Attempt at renal salvage with suture repair of D. Fiberoptic bronchoscopic-assiste intubation
the parenchyma E. Apneic oxygenation
E. Nephrostomy tube
228 PArt i Patient Care

27. A 36-year-ol male is transferre from another 30. An 18-year-ol male is brought to the
hospital to the ED after a high-spee hea-on MVC ED following a motorcycle crash. He is
with signicant front-en amage to the vehicle. hemoynamically stable an complains of severe
The accient occurre over 4 hours ago. On arrival, pelvic pain. Examination reveals bloo at the
the patient is complaining of left-sie chest an urethral meatus, scrotal ecchymosis, an a scrotal
abominal pain. His systolic bloo pressure is hematoma. A pelvic x-ray conrms the presence
80 mm Hg an heart rate is 10 beats per minute. of a pelvic fracture. Which of the following is the
Breath souns are present an equal bilaterally. most appropriate next step in iagnosis?
Abominal exam reveals signicant tenerness to A. Insertion of a Foley catheter
palpation. Plain lm of the chest shows a wiene B. CT abomen with IV contrast
meiastinum (10 cm) without hemothorax. Pelvic C. Retrograe urethrogram (RUG)
x-ray is normal. FAST is positive for free ui in D. Cystogram
the abomen. Following a 1-L crystalloi bolus an E. Intravenous pyelogram
 units of bloo, the patient’s bloo pressure an
heart rate are unchange. Which of the following is 31. A 8-year-ol man sustains a GSW to the right
the most appropriate next step in management? supraclavicular area with no exit woun. On
A. Aminister a bolus of tranexamic aci (TXA) arrival, his systolic bloo pressure is 60 mmHg an
an continue infusion on the way to the OR he is confuse an combative. E-fast emonstrates
for an exploratory laparotomy bilateral lung sliing an there is no evience of
B. CT scan of the chest, abomen, an pelvis active hemorrhage. Which of the following is the
C. Take the patient to the OR for an exploratory best next step in the management?
laparotomy without aministration of TXA A. Immeiate enotracheal intubation
D. Transthoracic echocariography B. Right tube thoracostomy
E. Intraoperative thoracic angiogram for possible C. Transfuse bloo
enovascular repair of thoracic aorta followe D. Resuscitative (ED) thoracotomy
by exploratory laparotomy E. Insert resuscitative enovascular balloon
occlusion of aorta (REBOA)
28. A -year-ol man sustains a GSW to the right
leg below the knee. Vital signs are within normal 32. A 30-year-ol man sustains a GSW to the left
limits. Physical exam reveals a single GSW to the mi-neck. On arrival at the ED, his systolic bloo
lateral leg with minimal swelling an no obvious pressure is 80 mmHg, heart rate is 10 beats per
eformity. Pulse exam reveals iminishe peal minute, an his GCS is 8. There is a moerate,
pulses on the right in comparison to the left. but nonexpaning hematoma in the neck
Which of the following is the most appropriate with no active bleeing or bruit. An airway is
next step in management? immeiately establishe, an bloo is given with
A. CT angiogram repeat systolic bloo pressure of 90 mmHg. The
B. OR an angiogram next most appropriate step in management is:
C. Aministration of IV papaverine A. Hea CT scan
D. Formal angiogram B. CT angiography of the neck
E. Arterial-pressure inex (API) C. Stanar four-vessel arteriography
D. Surgical neck exploration
29. A 60-year-ol male presents to the ED following an E. Triple enoscopy
MVC in which he was a restraine passenger. The
initial systolic bloo pressure was 90 mm Hg but 33. Which of the following is the best inication for
improves to 110 mm Hg after 1 L of normal saline. resuscitative (ED) thoracotomy?
Abominal exam reveals mil iffuse tenerness A. Severe blunt abominal an hea trauma with
without peritonitis. CT of the abomen reveals suen arrest in the ED
an isolate grae III splenic injury with active B. Abominal stab woun with no signs of
extravasation an a low-volume hemoperitoneum. life (SOL) in the el, cariopulmonary
Hemoglobin is stable at 1 g/L. Which of the resuscitation (CPR) en route
following is the best next step in management? C. Blunt trauma with loss of pulse in the el,
A. Laparotomy with splenectomy CPR en route
B. Laparotomy with attempt at splenorrhaphy D. Stab woun to chest with agonal breathing on
C. Angiography with embolization transport, no pulse in ED
D. Serial abominal examinations an E. GSW to abomen with asystole as presenting
hematocrits in the ICU rhythm an no pericarial tamponae on FAST
E. Laparoscopic splenectomy
CHAPtEr 16 Trauma 229

34. An 11-month-ol boy presents to the ED with 38. Which of the following is true regaring blunt
hypotension after being involve in an MVC. He cariac injury (BCI)?
has obvious eformities of both legs below his A. Creatine kinase-myocarial boun (CK-MB)
knees. Numerous attempts are mae to establish enzyme etermination lacks sensitivity
venous access at the antecubital fossa without B. It commonly results in serious ventricular
success. The best option for establishing access for arrhythmias
ui aministration woul be: C. It usually results in traumatic thrombosis of a
A. Internal jugular central line coronary artery branch
B. Distal saphenous vein cutown D. Presence of a sternal fracture preicts the
C. Femoral vein central line presence of BCI
D. Intraosseous (IO) cannulation of the proximal tibia E. It shoul be suspecte in patients with
E. IO cannulation of the istal femur transient sinus tachycaria
35. Which of the following is true regaring the 39. Which of the following surgical maneuvers is
pregnant trauma patient? most correct to access the corresponing bloo
A. Bloo volume increases proportionally less vessel?
than re bloo cell volume A. Left-sie meial visceral rotation or Mattox
B. A pregnant patient tens to have a mil maneuver for the mi inferior vena cava (IVC)
respiratory aciosis B. Transection of the neck of the pancreas for the
C. Use of raiographs is unsafe for the fetus in superior mesenteric artery
the thir trimester C. Right-sie meial visceral rotation or Cattell
D. The ,3-iphosphoglycerate level is increase maneuver for the suprarenal aorta
E. The glomerular ltration rate ecreases D. Kocher maneuver for the celiac axis
E. Division of the right common iliac artery for
36. A 30-year-ol man sustains a GSW to the right the istal vena cava an common iliac vein
chest. His bloo pressure in the emergency bifurcation
epartment is 70/40 mmHg. A chest tube is
place in the right chest with 500 mL of initial 40. In the setting of trauma, ligation is best tolerate
output. A follow-up chest raiograph reveals a for which of the following vessels?
complete whiteout of the right lung. The patient A. Right renal vein
is taken to the operating room an a right B. Left renal vein
thoracotomy is performe. On evaluation of the C. Brachial artery
right lung, there is a through-an-through injury D. Popliteal artery
to the right lower lobe that appears to have an E. Suprarenal IVC
active air leak an ongoing bleeing. Surgical
management shoul consist of: 41. Which of the following is true regaring
A. Formal right lower lobectomy extremity compartment synrome?
B. Pneumonectomy A. The soleus muscle must be etache from
C. Closure of both the anterior an posterior the tibia to ecompress the eep posterior
parenchymal efects with interrupte sutures compartment of the lower leg
D. Pulmonary tractotomy B. A compartment pressure greater than 40 mmHg
E. Ligation of the right lower lobe pulmonary artery is necessary to establish the iagnosis
C. The lateral compartment is the most
37. Which of the following is true regaring ail commonly affecte lower leg compartment
chest? D. An early sign of anterior compartment
A. The initial chest raiograph provies a involvement of the lower leg is numbness on
useful preictor of subsequent pulmonary the plantar aspect of the foot
insufciency E. It oes not occur in the buttocks
B. Respiratory failure is primarily cause by the
paraoxical motion of the chest wall
C. Operative chest wall stabilization in patients
without pulmonary contusion may shorten the
length of intubation
D. Aggressive ui resuscitation is an important
management ajunct
E. Once the iagnosis is establishe, the patient
shoul be intubate
230 PArt i Patient Care

42. A 17-year-ol boy is brought to the ED after being 45. After a motor vehicle accient, a 17-year-ol girl
involve in a high-spee motorcycle collision. He with blunt abominal trauma is foun to have
is hypotensive with a systolic pressure of 60 mmHg. free ui on abominal CT without evience of
A FAST scan is positive. At laparotomy, he is liver or spleen injury. She is hemoynamically
foun to have a large amount of bleeing from stable. Her abomen is iffusely tener. She is
behin the liver. Temporary application of a taken to the operating room. At surgery, she is
Pringle maneuver oes not control the bleeing. foun to have a 75% luminal circumference injury
However, laparotomy packs are place, an the to the rst portion of the uoenum. Surgical
bleeing appears to slow own. The systolic management consists of:
bloo pressure increases to 110 mmHg after A. Pyloric exclusion
aggressive resuscitation. The patient’s pH is 7.06 B. Duoenal iverticulization
an his temperature is 34°C. The next best step in C. Primary uoenal repair
management is: D. Whipple resection
A. Obtain control of the IVC above an below the E. Resection with uoenouoenostomy
liver
B. Perform a meian sternotomy for atriocaval 46. A 9-year-ol man presents with a GSW
shunt placement to the right upper quarant. On physical
C. Damage control closure an transport to ICU examination, the patient has a tener abomen.
D. Damage control closure an transport to At surgery, the patient is foun to have a 500-mL
interventional raiology (IR) suite for hepatic hemoperitoneum with a through-an-through
embolization injury to the right lobe of the liver that is no
E. Obtain control of aorta at the iaphragmatic longer actively bleeing. Further management
hiatus woul consist of:
A. Closing the injury with a liver suture
43. A 55-year-ol man is brought into the ED after a B. Packing the injury with omentum
high-spee MVC. The patient is hemoynamically C. Application of a brin sealant
stable. Gross hematuria is present. CT cystography D. No further management
reveals air in the blaer an an accumulation of E. Drainage with a Penrose rain
contrast in the right paracolic gutter. Which of the
following is the best management option? 47. A 0-year-ol man with morbi obesity sustains
A. Foley catheter rainage a GSW to the abomen. His bloo pressure is
B. Suprapubic cystostomy tube placement 110/70 mmHg an heart rate is 100 beats per
C. Open repair of the intraperitoneal blaer minute. At surgery, he is foun to have a blast
injury with absorbable sutures injury to the sigmoi colon involving 75% of the
D. Obtaining a formal cystogram circumference of the bowel, with a moerate
E. Open repair of the intraperitoneal blaer amount of fecal contamination. Hemoynamics,
injury with silk sutures temperature, an base ecit are normal. Which
of the following is the best option?
44. A 30-year-ol man sustains a GSW to the abomen A. Sigmoi colectomy with primary anastomosis
an presents to the ED with a systolic bloo with a iverting ileostomy
pressure of 60 mmHg. Emergent laparotomy reveals B. Primary repair of the sigmoi colon
a -L hemoperitoneum with an injury to the IVC C. Sigmoi colectomy with primary anastomosis
an right iliac vein. Both injuries are successfully D. Primary repair of the sigmoi colon with
repaire. Further exploration emonstrates a exteriorization of the repair
istal right ureteral injury below the level of the E. Sigmoi colectomy with a proximal colostomy
iliac vessels with a 3-cm efect. After 10 units of an oversewing of the rectal stump
bloo proucts, the patient’s bloo pressure is
80/60 mmHg, his heart rate is 110 beats per minute,
an his temperature is 96°F. Which of the following
is the best management option?
A. Proximal an istal ligation of the ureter
B. Ureteroureterostomy
C. Transureteroureterostomy
D. Psoas hitch
E. Ureteroneocystostomy
CHAPtEr 16 Trauma 231

48. A 46-year-ol female is brought into the 49. A 40-year-ol man with a history of heavy alcohol
emergency epartment following a motorcycle use is amitte to the hospital after being hit
crash. There is an open right tibia fracture. by a car. The patient unerwent pelvic xation,
Plain raiographs of the chest an pelvis are intraperitoneal blaer repair, an splenectomy
normal. Heart rate is 110 beats per minute, bloo 4 ays ago. On rouns, his vitals are normal, his
pressure is 110/70 mmHg, an her SpO is 99% abomen is very istene, an there is increase
on room air. CT imaging shows a small right output from his intraabominal rain. Labs are
pneumothorax. Orthopeic surgery is planning to as follows: Serum creatinine 1.5 mg/L, serum
take the patient to the operating room to place an albumin 3.0 g/L, rain ui creatinine 1.6 mg/L,
external xator. What is the best management for rain ui albumin 1.5 g/L, an rain ui
the pneumothorax? WBC 00. What is the most likely iagnosis?
A. Right tube thoracostomy with 8 French chest A. Hepatic ascites
tube B. Urine leak
B. Small pigtail catheter chest tube C. Abominal compartment synrome
C. Careful intraoperative monitoring of en-tial D. Pancreatic leak
CO E. Bacterial peritonitis
D. Repeat chest CT after orthopeic surgery
E. Neele ecompression
232 PArt i Patient Care

Answers
1. B. A parasternal penetrating injury with evience of peri- via a colostomy is not necessary (C). In aition, by exposing
carial effusion in a trauma patient is concerning for hemo- the extraperitoneal injury to the peritoneal cavity, it effec-
pericarium with unerlying cariac injury. The patient tively reners it an intraperitoneal injury; thus, presacral
being young makes this even more concerning as a chronic rainage woul not be inicate (D). If the extraperitoneal
pericarial effusion woul be highly unlikely. In recent years, injury cannot be ientie an repaire, a proximal ivert-
there has been a shift towar the selective management of ing colostomy has been shown to be effective in allowing
hemopericarium. Performing a meian sternotomy or tho- the injury to heal itself. Distal irrigation of the rectum an
racotomy in all penetrating trauma patients with pericarial routine rainage of the presacral space are not necessary
effusion leas to an unacceptably high nontherapeutic rate an may even contribute to forcing fecal material out from a
which may be as high as 38% (C, D). Aitionally, a ran- rectal laceration. In particular, if the injury is to the anterior
omize controlle trial in stable patients emonstrate that rectum, the rainage will be ineffective. Abominoperineal
penetrating thoracic trauma patients with a positive pericar- resection woul not be inicate (E). IV antibiotics alone are
ial winow after a 4-hour observation perio can safely not appropriate (A). A CT scan is not reliable enough to rule
be manage with just irrigation an no aitional surgery. out a istal rectal injury. As such, the ning of bloo on
However, in the case of obstructive shock (tachycaria, nar- proctoscopy is enough of an inication of an injury to pro-
rowe pulse pressure, lethargy, hypotension), performing a cee with stool iversion.
meian sternotomy woul be the appropriate intervention. References: Bosarge PL, Como JJ, Fox N, et al. Management of
There is no role for pericariocentesis in trauma (E). A for- penetrating extraperitoneal rectal injuries: an Eastern Association
mal echocariogram woul be useful in the case of the stable for the Surgery of Trauma practice management guieline. J Trauma
patient with an equivocal FAST or to screen for pseuoaneu- Acute Care Surg. 016;80(3):546–551.
Demetriaes D, Murray JA, Chan L, et al. Penetrating colon
rysms after cariac repair (A). Left anterolateral thoracotomy
injuries requiring resection: iversion or primary anastomosis? An
woul be the proceure of choice if the patient ha presente
AAST prospective multicenter stuy. J Trauma. 001;50(5):765–775.
as a traumatic arrest to rapily resuscitate the patient (C). Gonzalez RP, Falimirski ME, Holevar MR. The role of presacral
Meian sternotomy provies the best exposure to repair car- rainage in the management of penetrating rectal injuries. J Trauma.
iac injuries. 1998;45(4):656–661.

2. B. Hemobilia is characterize by the tria of upper GI 4. A. Blunt cerebrovascular injury (BCVI) is the collective
bleeing (melena), jaunice, an right upper quarant pain term for blunt injury to the caroti an vertebral arteries.
which may occur ays to weeks after liver injury. The right These injuries are associate with signicant morbiity an
hepatic artery is often involve, an the unerlying lesion is mortality following trauma, specically relate to risk of
an arterial pseuoaneurysm which forms a stulous connec- stroke. BCVI is grae using the Bif classication. Grae I
tion with the biliary tree. In a stable patient, workup woul refers to intimal irregularity with <5% luminal narrowing,
inclue a CT scan with IV contrast to look for a blush. But in grae II is a issection or intramural hematoma with >5%
an unstable patient with high suspicion, the next best step is luminal narrowing, grae III is a pseuoaneurysm, grae IV
angioembolization which can be both iagnostic an thera- a complete occlusion an Grae V is a transection with active
peutic (D). Biloma is another complication of traumatic liver extravasation. Most grae I–II injuries shoul be treate with
injury if a bile uct is isrupte an has an ongoing leak. antithrombotic therapy, either unfractionate heparin or
This can be emonstrate with an abominal ultrasoun antiplatelet agents (B–E). Antithrombotic therapy reuces
an initially manage with percutaneous rainage an pos- the risk of stroke an reuces morality an therefore shoul
sibly ERCP (A, C). Another complication of liver trauma is be initiate as soon as safe. Repeat imaging is often recom-
hepatic necrosis an/or abscess. Injure or necrotic liver mene in 7 to 10 ays to monitor for progression of these
parenchyma can incite a massive inammatory response an lesions.
these patients often have a high fever an leukocytosis with- Reference: Kim DY, Bif W, Bokhari F, et al. Evaluation an
out GI blee. This complication shoul be worke up with a management of blunt cerebrovascular injury: a practice manage-
CT abomen with IV contrast (E). ment guieline from the Eastern Association for the Surgery of
Trauma. J Trauma Acute Care Surg. 00;88(6):875–887.
3. B. The management of a rectal injury epens on whether
it is intra- or extraperitoneal, the egree of tissue estruction, 5. B. Asymptomatic pneumomeiastinum following blunt
an the hemoynamic status of the patient. As a general rule, chest trauma is most often benign. Associate aeroiges-
intraperitoneal injuries can be repaire primarily (they are tive injuries are rare, occurring in less than 1% of cases.
treate like a colon injury). If it is an extraperitoneal injury, Aitional oral contrast stuies have been shown to have
there are two basic options: primary repair of the injury or a no ae benet to ientifying esophageal injuries in the
iverting colostomy. The ecision of whether to o primary absence of high-risk nings such as pleural effusion or
repair relates to its accessibility. Proximal extraperitoneal ysphagia (D, E). Thoracostomy tubes are not inicate in
injuries can be repaire primarily. In general, when primary this patient as there is no evience of hemothorax or pneu-
repair of the extraperitoneal injury is performe, iversion mothorax (C). An associate hemothorax, as well as air in
CHAPtEr 16 Trauma 233

all meiastinal compartments or specically posterior com- Current therapy of trauma and surgical critical care. n e. Elsevier;
partment pneumomeiastinum, are features that have been 016:19–04.
associate with an increase in mortality. These features may
prompt aitional workup. Panenoscopy with esophago- 8. B. Estimate TSBA burne is useful to etermine appro-
scopy, laryngoscopy, an bronchoscopy may be consiere priate ui resuscitation volumes. Each upper extremity
for select patients with penetrating neck injury (A). accounts for 9% of the TBSA (anterior surface woul be half
References: Lee WS, Chong VE, Victorino GP. Compute tomo- that or 4.5%), each lower extremity accounts for 18%, the ante-
graphic nings an mortality in patients with pneumomeiasti- rior an posterior trunk each accounts for 18%, the hea an
num from blunt trauma. JAMA Surg. 015;150(8):757–76. neck account for 9%, hans are 1% each, an the perineum
Matthees NG, Mankin JA, Trahan AM, et al. Pneumomeiasti- accounts for 1%. First-egree burns are not inclue. For this
num in blunt trauma: if aeroigestive injury is not seen on CT, inva- patient, the anterior surface of both arms accounts for 9%,
sive workup is not inicate. Am J Surg. 019;17(6):1047–1050. the entire leg is 18%, an the hans are not counte
Muckart DJJ, Harcastle TC, Skinner DL. Pneumomeiastinum (rst-egree burns), totaling 7% (A, C–E). The most wiely
an pneumopericarium following blunt thoracic trauma: much
use approach to ui resuscitation in a burn patient is the
ao about nothing? Eur J Trauma Emerg Surg. 019;45(5):97–931.
Parklan formula: 4 mL/kg for each percentage of TBSA
burne over the rst 4 hours, with one-half of that amount
6. A. Supraconylar humerus fractures often occur in chil-
aministere in the rst 8 hours an the remaining half
ren who fall onto an outstretche arm. The classic n-
over the next 16 hours. For chilren, some use a moie
ing is a patient with a “pink an pulseless” han that may
Parklan formula with 6 mL/kg. Keep in min that Ringer
improve with close reuction of the fracture. Vascular inter-
lactate is the ui of choice. Normal saline in such large vol-
vention versus watchful waiting is ebate in these cases;
umes will lea to hyperchloremic metabolic aciosis. The
however, the most common recommenation is to attempt
most important enpoint of resuscitation is aequate urine
reuction rst an strongly consier surgical intervention
output (0.5–1 cc/kg/hr).
of the brachial artery if the vascular exam oes not improve
after reuction. Volkmann ischemic contracture evelops in
9. A. In the emergent setting, the fastest way to gain vas-
the setting of ischemia or compartment synrome which is
cular access is by a peripheral catheter, often at the meian
manifeste by a complex eformity of the wrist an han.
antecubital fossa, because this will typically accommoate a
Claw han evelops after an injury to the istal ulnar nerve
large bore IV an is easy to cannulate. Short-wie catheters
(B) which can be seen in lacerations or sports-relate inju-
are use to maximize volume ow for rapi resuscitation.
ries. In the ulnar claw han, the 4th an 5th ngers cannot
The rate of ui ow is proportional to the cross-sectional
be extene. Posterior hip islocations are commonly associ-
area of the catheter an inversely proportional to the fourth
ate with sciatic nerve injuries (C). Anterior shouler islo-
power of its raius. As such, an 18-gauge catheter is pre-
cations are associate with axillary nerve injuries which can
ferre over a 0-gauge catheter since the 18-gauge catheter
be etecte by a “military patch” anesthesia over the eltoi
has a larger iameter (B). Central vein catheterization is
(D). Chronic anterior shouler islocations shoul not be
not the preferre moe of vascular access in the immeiate
reuce seconary to the risk of axillary artery injury. Wrist
trauma setting because it is time consuming an has a high
rop is seen seconary to raial nerve injury an this is com-
rate of complications. These complications are exacerbate
monly in the setting of a humeral shaft fracture (E).
by the urgency of the line placement, central veins often
Reference: Delniotis I, Kteniis K. The pulseless supraconylar
being collapse ue to hypovolemia, an suboptimal use of
humeral fracture: our experience an a 1-year follow-up. J Trauma
Acute Care Surg. 018;85(4):711–716. sterile technique. Central line-associate bloostream infec-
tions alone have a mortality rate as high as 0%. A short but
large central vein coris will allow for a faster route for infu-
7. A. Blunt trauma patients with small (<4 cm) tracheal
sion but is not appropriate in the initial trauma setting for the
lacerations with viable surrouning tissue that are hemoy-
aforementione reasons (C–E).
namically stable without any concerning nings suggestive
Reference: Mermel LA, Allon M, Bouza E, et al. Clinical prac-
of concurrent esophageal trauma (fever, leukocytosis, oy- tice guielines for the iagnosis an management of intravascular
nophagia, ysphagia) can safely be manage nonoperatively catheter-relate infection: 009 Upate by the Infectious Diseases
with antibiotics, voice rest, PPI therapy, an repeat bron- Society of America. Clin Infect Dis. 009;49(1):1–45.
choscopy in 4 to 48 hours. Large (≥4 cm) tracheal injuries
in blunt trauma patients shoul be repaire. In the setting 10. A. Trauma is the leaing cause of eath for iniviuals
of isolate tracheal injuries in the neck requiring operative over the age of 45 years in the Unite States. Although all
repair, a collar incision is appropriate. Primary repair can be the liste choices are causes of eath in the trauma patient,
attempte for wouns that are well-appose after ebrie- traumatic brain injury (TBI) is the single largest contributor
ment of evitalize eges. Permanent suture shoul not be accounting for nearly half of all trauma eaths an is the
use in tracheal repair as it can serve as a nius for infection most common cause of eath in trauma patients reaching the
(B–D). Aitionally, a two-layer closure has a high chance hospital alive (B–E). Hemorrhagic shock is the most common
or tracheal stenosis an so a one-layer closure with absorb- cause of eath in trauma patients within the rst hour. An
able suture is most appropriate. Buttressing the repair with important component to the management of TBI is the pre-
muscle (i.e., hyoi or sternocleiomastoi) is commonly per- vention of seconary injury to the brain by avoiing hypo-
forme. A tracheostomy is rarely neee but if inicate, it tension an hypoxia.
shoul be place one ring-space below the injury (E). References: Baker CC, Oppenheimer L, Stephens B, Lewis
Reference: Fernanez LG, Norwoo SH, Berne JD. Tracheal, FR, Trunkey DD. Epiemiology of trauma eaths. Am J Surg.
laryngeal, an oropharyngeal injuries. In: Asensio J, Trunkey D, es. 1980;140(1):144–150.
234 PArt i Patient Care

National Center for Injury Prevention an Control. Traumatic brain characterize by blistering, pain, blanching, an intact hair
injury in the United States: a report to Congress. Centers for Disease Con- follicles, are limite to the ermal layer, an o not typically
trol an Prevention, US Department of Health & Human Services; 1999. require any skin grafting. () Deep partial-thickness burns
are characterize by blistering, are less sensitive (sometimes
11. E. The anterior abomen is boune by the nipples, painless) an nonblanchable, an involve loss of hair follicles
groin crease, an anterior axillary lines. Stab wouns to this (A, E). Since the hair follicles offer the regenerative capacity
area are ivie into thirs; one-thir o not penetrate the for the skin, eep partial-thickness burns will not heal spon-
peritoneal cavity, one-thir penetrate the peritoneal cavity taneously an will often require intervention such as skin
but on’t cause signicant intraabominal injury, an one- grafting (D). Thir-egree burns are consiere full-thickness
thir penetrate the peritoneal cavity causing signicant because they involve all the layers of the skin an are charac-
intraabominal injury. Immeiate exploratory laparotomy terize by a white leathery appearance. Fourth-egree burns
is manate in the hemoynamically unstable patient or in are also consiere full thickness but also involve either
the presence of iffuse peritonitis. In a hemoynamically unerlying muscle, fascia, or bone an typically lea to is-
stable patient without peritonitis, the surgeon has several gurement (B).
options to choose from. These inclue amission for serial Reference: Tiwari VK. Burn woun: how it iffers from other
abominal exams, CT scan, FAST scan (which has a lower wouns? Indian J Plast Surg. 01;45():364–373.
sensitivity than CT but is quick an inexpensive), an LWE
(C, D). The main avantage of LWE is that if the stuy is 14. C. Prophylactic use of IV antibiotics shoul be is-
negative (anterior fascia has not been penetrate), the patient courage in the burn patient because this will bree mul-
can be ischarge from the ED. If LWE is positive, it oes tirug-resistant organisms. However, several topical
not mean the peritoneum has been violate. Taking all posi- ointments are available that are use wiely in burn care to
tive LWE patients to the operating room (OR) will result in a prevent bacterial colonization. Silver sulfaiazine is consi-
high negative laparotomy rate. As such several options exist: ere a broa-spectrum agent, but it has poor coverage for
procee to CT scan, amission for serial abominal exams, Pseudomonas, has poor eschar penetration, an can lea to
or iagnostic laparoscopy. The ecision of which to perform neutropenia an thrombocytopenia (B). It shoul be avoie
epens on the institution (A, B). in patients with a sulfa allergy. An avantage is its painless
References: Cothren CC, Moore EE, Warren FA, Kashuk JL, Bif application. Silver nitrate, also consiere a broa-spectrum
WL, Johnson JL. Local woun exploration remains a valuable triage agent, oes not work against Pseudomonas, an its application
tool for the evaluation of anterior abominal stab wouns. Am J Surg. is painful. It has poor eschar penetration, causes tissue is-
009;198():3–6.
coloration, an can lea to severe electrolyte erangements
Shanmuganathan K, Mirvis S, Chiu W. Penetrating torso trauma:
triple-contrast helical CT in peritoneal violation an organ injury-a
(epletes Na+, K+, an Cl−) (E). It can be use in patients
prospective stuy in 00 patients. Radiology. 004;31(3):775–784. with a sulfa allergy. Bacitracin an neomycin have a painless
application, limite eschar penetration, an poor gram-neg-
12. D. This patient has receive a large volume of ui ative coverage. Mafenie acetate (Sulfamylon) is consiere
resuscitation that le to abominal compartment synrome, a broa-spectrum agent incluing activity against Pseudomo-
which presents with the tria of oliguria, rise in peak airway nas an Enterococcus spp. an has goo eschar penetration
pressures, an increase intraabominal pressure. Blaer (D). Since it is a carbonic anhyrase inhibitor it can lea to
pressure (as measure via an inwelling Foley) is use as hyperchloremic metabolic aciosis, an thus its use shoul
a surrogate to etermine abominal pressure. Intraabom- be limite to small areas of full-thickness burns (A).
inal hypertension has somewhat arbitrarily been ene as Reference: Dai T, Huang YY, Sharma SK, Hashmi JT, Kurup
a sustaine intraabominal pressure greater than or equal DB, Hamblin MR. Topical antimicrobials for burn woun infections.
Recent Pat Antiinfect Drug Discov. 010;5():14–151.
to 1 mmHg. En-organ amage typically occurs with pres-
sures greater than 0 mmHg. As the pressure in the abomen
15. C. The American College of Surgeons an American
increases, the iaphragm’s ability to contract is compro-
Burn Association have set guielines as to which patients
mise, an this subsequently lessens pulmonary compliance
shoul be transferre to a burn center. These patients have
an FRC (A, B). This translates to an increase intrathoracic
been emonstrate to have improve outcomes an sur-
pressure resulting in ecrease venous return, increase
vival when treate in a nationally recognize burn center
pulmonary vascular resistance, an increase pulmonary
that can approach the burn patient with a multiisciplinary
capillary wege pressure (C, E). Treatment is to perform
approach. Inications for transfer are as follows: (1) secon-
a ecompressive laparotomy, leaving the abomen open
or thir-egree burns greater than 0% TBSA in patients age
(though covere with a protective bag).
10 to 50 years ol; () secon- or thir-egree burns greater
Reference: Papavramiis TS, Marinis AD, Pliakos I, Kesisoglou
I, Papavramiou N. Abominal compartment synrome—Intra-
than 10% TBSA in patients younger than 10 years or oler
abominal hypertension: ening, iagnosing, an managing. J than 50 years; (3) thir-egree burns greater than 5% TBSA
Emerg Trauma Shock. 011;4():79–91. in any age; (4) any secon- or thir-egree burn to hans,
feet, face, eyes, genitalia, perineum, or skin over major joints;
13. C. Burn injuries are classie into ve categories with (5) any electrical or chemical burn; an (6) any concomitant
secon-egree burns having two subclassications. First-e- inhalation injury or multiple trauma. From the available
gree, or supercial, burns only involve the epiermis with answer choices, the only patient that has an inication for
re skin, no blisters, an pain. Sunburns are consiere transfer to a burn center is the 1-year-ol male with both
rst-egree burns. Secon-egree burns are ivie into a han burn an a chemical burn (A, B, D, E). First-egree
two categories: (1) Supercial partial-thickness burns are burns o not nee referral.
CHAPtEr 16 Trauma 235

References: American College of Surgeons. Resources for optimal it is for this reason that tPA has ha an emerging role in the
care of the injured patient; 1993:64. management of frostbite burns. Thrombolytic therapy will
Hospital an prehospital resources for optimal care of patients limit microvascular thrombosis an prevent reperfusion
with burn injury: guielines for evelopment an operation of injury. Preictors of poor response to tPA inclue warm
burn centers. American Burn Association. J Burn Care Rehabil.
ischemia time longer than 6 hours, more than 4 hours of
1990;11():98–104.
col exposure, an multiple freeze-thaw cycles. Because
this patient has ha a warm ischemia time of 7 hours, she
16. A. Electrical shock requires the expertise of a burn cen-
is ineligible for tPA treatment (C). Patients that are eeme
ter, an all patients shoul be transferre as soon as they
appropriate caniates for tPA therapy shoul continue
are stable. The two types of electrical currents are alternat-
until there is evience of tissue reperfusion, 48 hours have
ing an irect. An alternating current will lea to repetitive,
passe, or the treating team feels there is no further thera-
tetanic muscle contractions (C). An example of this is a city
peutic gain from continue infusion.
worker who gets electrocute on a power line that emits an
Reference: Gross EA, Moore JC. Using thrombolytics in frost-
alternating current. Since exor muscle tone is generally
bite injury. J Emerg Trauma Shock. 01;5(3):67–71.
stronger than extensor muscle tone, patients will often grip
the source of electricity leaing to a prolonge exposure. In 18. C. Chance fractures are also calle seat-belt fractures.
contrast, irect current electrocution will often result in a In chilren, they occur when the chil is only wearing a lap
single, large muscle contraction that will throw the patient belt. They are exion-istraction type injuries of the spine.
several feet away from the source. A car battery has a irect There is a signicant association with intraabominal inju-
current, so this patient likely suffere a irect current elec- ries (most commonly hollow viscus an pancreas). Recent
trocution. Aipose tissue has a high resistance to electricity, reports using large-scale trauma registry ata suggest that
which will result in an increase tissue temperature an sub- the rate of intraabominal injury is close to 33% (previously
sequent coagulation; thus, patients with obesity will have a reporte much higher). The presence of a Chance fracture is
higher amount of eep thermal burns (D). The main cause in an of itself not an inication for a laparotomy. However,
of eath in the early post electrocution perio is cariac the patient presente has an unreliable abominal exam-
arrhythmias (B). Other immeiate complications of electro- ination. In aition, the presence of free ui on CT, in the
cution injury inclue posterior shouler islocation an spi- absence of a soli organ injury, shoul raise the suspicion of
nal cor injury. Long-term, patients are at increase risk of a hollow viscus injury. As such laparotomy is inicate. In an
cataracts, polyneuritis, an ototoxicity. The skin burn mark alert an oriente, nonventilate patient, serial abominal
with an electrical injury can vastly unerestimate the sever- examination woul be the initial management (in spite of the
ity of the burn. Often there is severe injury to the unerlying free ui) (B). MRI of the spine will be helpful to etermine
muscle an connective tissue espite a relatively minor outer spinal cor impingement, but this will nee to be performe
skin burn. As such, these patients are susceptible to rhabo- after exploratory laparotomy (A). Until then, the patient
myolysis, which woul be suggeste by the presence of re shoul remain in strict spine precautions. Repeat imaging
urine. Thus, creatine kinase (CK) levels shoul routinely be alone is not appropriate if there is concern for a hollow-vis-
sent (E). These patients shoul be amitte, place on car- cous injury (D, E).
iac monitoring, an resuscitate with IV uis to maintain References: Neugebauer H, Wallenboeck E, Hungerfor M.
high urine output. Seventy cases of injuries of the small intestine cause by blunt
Reference: Wesner ML, Hickie J. Long-term sequelae of electri- abominal trauma: a retrospective stuy from 1970 to 1994. J
cal injury. Can Fam Physician. 013;59(9):935–939. Trauma. 1999;46(1):116–11.
Tyroch AH, McGuire EL, McLean SF, et al. The association
17. D. Frostbite can occur when tissue is expose to tem- between Chance fractures an intra-abominal injuries revisite: a
peratures below −°C or 8°F. The severity of the injury multicenter review. Am Surg. 005;71(5):434–438.
increases proportionally to the uration of exposure. Frost-
bites are classie as follows: (1) First egree are hyperemic 19. B. Ault Jehovah’s Witnesses have the right to refuse
without necrosis an characterize by a yellow plaque; () bloo proucts, even in lifesaving situations. Anemia oes not
secon egree have supercial vesicles with hyperemia rener the patient incapable of making an informe ecision
an partial-thickness necrosis; (3) thir-egree have hem- an giving bloo proucts against the patient’s wishes is a
orrhagic bullae an full-thickness necrosis; an (4) fourth violation of her autonomy, an the physician may be repri-
egree are characterize by frank gangrene with involve- mane by the American Meical Association (E). The patient
ment of unerlying muscle an bone (A). Treatment begins shoul still continue to receive the care she woul otherwise
with rewarming the extremity in a warm water bath between get if she i consent to bloo transfusion (A). With the rela-
40 an 4°C. It shoul be one rapily (E). Because tissue via- tive hemoynamic instability (tachycaria an hypotension),
bility will often take weeks to etermine, early ebriement contrast extravasation in the spleen, anemia, an hemoperi-
an/or amputation shoul be avoie (B). Tissue freezing toneum, there is little margin for error, so the patient shoul
an reperfusion both contribute to the tissue amage seen unergo a splenectomy. The physician shoul ocument the
in frostbite burns. Crystallization of the extracellular space patient’s refusal of bloo proucts in the electronic meical
leas to an increase extracellular oncotic pressure resulting recor because this places her at higher risk for eath given
in cellular ehyration an impaire intracellular metab- her present anemia. Angiography with embolization is con-
olism. An inammatory response ensues ultimately lea- siere an appropriate option for hemoynamically stable
ing to thrombosis, tissue ischemia, an enothelial injury. patients with contrast extravasation (C). In a true emergency
Reperfusion injury occurs when bloo ow is restore, an setting, there is no time to consult the ethics committee (D).
236 PArt i Patient Care

20. A. In the trauma patient, c-spine clearance is accom- also recommens that persistent air leaks on postinjury ay
plishe using the National Emergency X-Raiography Utili- 3 shoul be further evaluate with VATS because this can be
zation Stuy (NEXUS) criteria. Patients that have any one of suggestive of unerlying bronchial injury or bronchopleural
the NEXUS criteria shoul continue with spinal precautions stula.
until a CT scan of the c-spine is performe. The NEXUS cri- References: Mowery NT, Gunter OL, Collier BR, et al. Practice
teria can be remembere by the “NSAID” mnemonic: Neu- management guielines for management of hemothorax an occult
rologic ecit, Spinal (cervical) tenerness, Altere mental pneumothorax. J Trauma. 011;70():510–518.
status, Intoxicate, or Distracting injury. Patients with a neg- Sharma A, Jinal P. Principles of iagnosis an management of
traumatic pneumothorax. J Emerg Trauma Shock. 008;1(1):34–41.
ative CT c-spine can then be clinically cleare an the c-collar
may be remove. This is not possible in an obtune or intu-
22. B. The patient escribe has evience of muscle isch-
bate patient. If the patient is only expecte to be obtune
emia/necrosis an has evelope rhabomyolysis as evi-
or intubate for a short perio of time (e.g., combative runk
ence by the rise in CK. Rhabomyolysis can present with
patient), it is reasonable to keep the c-collar on an assess the
CK levels of 10,000 to 0,0000 u/L; no other conition can
c-spine once the patient is awake. This patient has extensive
cause such an extreme rise in CK (normal is 45–60 u/L).
traumatic brain injury an is likely going to be intubate for
Rhabomyolysis can occur in any setting that causes isch-
a prolonge perio of time. Prolonge application of a har
emia to the muscles (such as hypotension after trauma), or
cervical collar appears to compress the jugular veins, causing
from prolonge pressure on muscle compartments uring
venous outow obstruction, an thus increasing intracranial
surgery. It is likely exacerbate by obesity an improper
pressure (ICP). The collar also creates a nociceptive stimulus,
paing on the OR or proceure table. The ischemia/reper-
which might also contribute to elevate ICP; therefore, keep-
fusion cycle that ensues places the patient at risk of evelop-
ing the c-collar on for a prolonge perio of time increases
ing compartment synrome. A small stuy of patients with
the risk for complications (C). Previously, this patient woul
obesity unergoing Roux-en-Y bypass foun that boy mass
have receive an MRI c-spine an if no injuries were ien-
inex (BMI) was an inepenent risk factor for the evel-
tie, the c-collar woul then be remove. However, the
opment of postoperative rhabomyolysis. In a patient posi-
Eastern Association for the Surgery of Trauma (EAST) has
tione in the supine position, the muscles that woul most
recently recommene that in an obtune ault blunt
likely be compresse are the gluteal ones. In aition, pel-
trauma patient, the c-collar shoul be remove after a nega-
vic embolization, as performe for trauma or enovascular
tive CT c-spine alone. MRI c-spine may no longer have a role
abominal aortic aneurysm (AAA) repair, is a known risk for
in the obtune trauma patient as it has been emonstrate
eveloping buttock clauication. Rarely, it is associate with
that it may lea to a higher complication rate an longer ICU
evastating pelvic ischemia an/or buttock ischemia/necro-
stay as occult injuries that are not clinically relevant may be
sis (A, C–E). So in this patient, in aition to aggressive ui
ientie an acte upon (B). There are no stuies showing
hyration, it woul be imperative to roll the patient over to
improve outcomes in switching to a soft-collar (D, E).
inspect the buttock muscles.
Reference: Patel MB, Humble SS, Cullinane DC, et al. Cervical
spine collar clearance in the obtune ault blunt trauma patient:
References: Benevies ML, Nochi Júnior RJ. Rhabomyolysis
seconary to gluteal compartment synrome after bariatric surgery:
a systematic review an practice management guieline from the
case report. Rev Bras Anestesiol. 006;56(4):408–41.
Eastern Association for the Surgery of Trauma. J Trauma Acute Care
Yasumura K, Ikegami K, Kamohara T, Nohara Y. High inci-
Surg. 015;78():430–441.
ence of ischemic necrosis of the gluteal muscle after transcathe-
ter angiographic embolization for severe pelvic fracture. J Trauma.
21. D. Pneumothorax is a common complication of both 005;58(5):985–990.
penetrating an blunt trauma. It is a clinical iagnosis
that can be mae uring the primary survey. Patients with 23. A. Kiney injuries are grae from I to V, with grae I
ecrease breath souns, trachea eviation, an hypoten- being a contusion or subcapsular, nonexpaning hematoma
sion shoul be suspecte of having a tension pneumothorax an grae V a completely shattere kiney or an avulsion of
an shoul have neele ecompression or tube thoracos- the renal hilum. Grae I an II injuries are consiere minor,
tomy performe immeiately. In equivocal cases, imaging grae III injuries are eep lacerations that o not involve
can be helpful. Occult pneumothorax is one that is not seen the collecting system, whereas grae IV injuries are lacera-
on the initial raiograph but may be emonstrate on CT. tions extening into the collecting system or an injury to the
Pneumothorax as a result of penetrating trauma has a con- main renal artery. The vast majority of blunt renal injuries
comitant hemothorax up to 80% of the time (E). Small pneu- (approximately 90%) can be manage nonoperatively. The
mothoraces ientie on CT can be observe if the patient injury escribe in this patient woul be a grae IV an, in a
is stable. Normally, 1.5% of the pneumothorax volume is stable patient, can be manage nonoperatively (B–D). Grae
absorbe in 4 hours. Aitionally, the use of 100% inspire IV injury from blunt trauma can be manage nonoperatively
supplemental oxygen is controversial because it can result in provie the patient is hemoynamically stable. Most uri-
oxygen toxicity (A). EAST recommens that an occult pneu- nary extravasation resolves. If it persists, or if the patient
mothorax can be safely observe in a stable patient uner- emonstrates evience of sepsis, it shoul be treate using
going general anesthesia. This recommenation was base a combination of enourologic an percutaneous techniques
on two prospective ranomize stuies that supporte the (such as a percutaneous nephrostomy) (E). The ecision to
notion that occult pneumothoraces will likely not progress explore a zone II or perinephric retroperitoneal hematoma
regarless of the presence of positive pressure ventilation at the time of operation an in the absence of preoperative
(B). Similarly, an occult pneumothorax can be observe in imaging has classically been base on the mechanism of
a ventilate patient that remains asymptomatic (C). EAST injury an hemoynamic status of the patient. Following
CHAPtEr 16 Trauma 237

blunt trauma an in the absence of hemoynamic instability onation shoul be properly investigate by an organ ona-
or a rapily expaning or pulsatile perinephric hematoma, tion service (A, C). Aministering a lethal ose of morphine
these perinephric hematomas shoul not be explore. Fol- sulfate, or euthanasia, is only practice in certain states an
lowing penetrating trauma an in the absence of preoper- requires an awake patient to consent (D). Consulting the
ative imaging to assist in the ientication of a renal injury, hospital ethics committee woul not be appropriate in this
the presence of a perinephric retroperitoneal hematoma situation (E).
manates exploration. If inicate, nephrectomy shoul be Reference: Emanuel EJ, Emanuel LL. Proxy ecision making
precee by palpation for a contralateral kiney. Surgery is for incompetent patients: an ethical an empirical analysis. JAMA.
inicate for vascular or renal peicle injuries or in a com- 199;67(15):067–071.
pletely shattere kiney.
References: Kuan JK, Wright JL, Nathens AB, Rivara FP, Wes- 26. C. In the uncommon scenario in which a patient “can-
sells H, American Association for the Surgery of Trauma. American not be intubate nor ventilate,” a surgical cricothyroiot-
Association for the Surgery of Trauma Organ Injury Scale for kiney omy shoul be immeiately unertaken. This is performe
injuries preicts nephrectomy, ialysis, an eath in patients with using an 11 blae via a transverse or vertical incision of the
blunt injury an nephrectomy for penetrating injuries. J Trauma. skin irectly over the cricothyroi membrane followe by
006;60():351–356. a transverse incision through the cricothyroi membrane.
Tinkoff G, Esposito TJ, Ree J, et al. American Association for the A vertical incision is preferre on the skin an subcutane-
Surgery of Trauma Organ Injury Scale I: spleen, liver, an kiney,
ous tissue to avoi injuring the anterior jugular veins. The
valiation base on the National Trauma Data Bank. J Am Coll Surg.
airway shoul be ilate using one’s nger allowing for
008;07(5):646–655.
insertion of an appropriately size enotracheal or trache-
24. B. A patient who has punche another person in the ostomy tube (6 French or smaller). Neele cricothyroiot-
mouth is at risk for a human bite woun. The most com- omy is traitionally reserve for chilren uner the age of
mon organism foun isolate in wouns from infecte 1 years ol because a surgical or open cricothyroiotomy
human bites is Streptococcus followe by Staphylococcus. in this population may result in subglottic stenosis (A). In
Other common organisms inclue Eikenella, Fusobacterium, the absence of a percutaneous neele cricothyroiotomy
Prevotella, an Porphyromonas. T. pallidum is the organism kit, a high-jet insufator is typically require to permit
that causes syphilis an has been reporte to be transmit- temporary oxygenation of patients in whom a neele cri-
te by a human bite, but this is rare (A). Propionibacterium cothyroiotomy has been performe. Nasotracheal intu-
an Bacteroides are anaerobic organisms an are unlikely bation requires that a patient is spontaneously breathing
to be transmitte from a human bite (D, E). Hepatitis C is an is contrainicate in a patient with severe maxillofa-
the leaing cause of eath from liver isease in the Unite cial fractures or in those with the potential for a cribriform
States an the most common etiology leaing to liver trans- plate fracture (B). Due to the signicant amount of blee-
plantation. However, transmission from infecte persons is ing, beroptic bronchoscopy is unlikely to be of benet in
rare (C). Hepatitis B is more likely to be transmitte from a this situation (D). Issues relate to setup, equipment, an
human bite. availability also limit the use of this moality in emergent
References: Stevens DL, Bisno AL, Chambers HF, et al. Practice trauma situations. Apneic oxygenation is a technique of
guielines for the iagnosis an management of skin an soft tissue proviing supplemental high-ow oxygenation via nasal
infections: 014 upate by the Infectious Diseases Society of Amer- cannula in aition to stanar preoxygenation techniques.
ica. Clin Infect Dis. 014;59():e10–e5. This ajunct may ecrease the incience of esaturation in
Talan DA, Abrahamian FM, Moran GJ, et al. Clinical presentation patients unergoing intubation but is not a replacement for
an bacteriologic analysis of infecte human bites in patients present- a enitive airway (E).
ing to emergency epartments. Clin Infect Dis. 003;37(11):1481–1489. Reference: American College of Surgeons Committee on
Trauma. Advanced trauma life support program for doctors. 9th e.
25. B. Brain eath is iagnose by a stanarize set of American College of Surgeons; 01.
tests incluing electroencephalography, nucleotie brain
scan, apnea test, an clinical assessment incluing brain 27. C. This patient is hemoynamically unstable with a
stem reexes. Brain eath is both a meical an legal eter- positive abominal FAST following blunt trauma. In gen-
mination of eath. It is appropriate to support a brain-ea eral, patients in hemorrhagic shock are classie as respon-
patient using a ventilator for a limite perio of time. This ers, transient responers, an nonresponers on the basis
will help the patient’s family come to terms with their loss of whether or not their vital signs improve following a
an will help coorinate possible organ onation. This ui challenge. Transient an nonresponers shoul be
shoul always be facilitate by an organ onation service consiere to have ongoing bloo loss until proven other-
an not by the physician. If the patient is not registere for or wise. Given these nings, the patient shoul be taken to
against organ onation, the ecision regaring organ ona- the operating room for an exploratory laparotomy (B, E).
tion shoul be guie by the stanar of substitute jug- The CRASH- trial emonstrate that the early aministra-
ment. This involves a family member or close frien making tion of TXA (antibrinolytic agent) in blunt trauma reuce
the ecision base on the known wishes or preferences of all-cause mortality. The benet of TXA is best seen if given
the patient at the time of eath. However, this shoul never within the rst hour of trauma an nonexistent after three
be referee by the physician. If a query for organ onation is hours. In fact, TXA given after three hours may increase mor-
initiate from a family member, a thir-party service shoul tality seconary to bleeing (A). The ning of chest pain, a
be mae available to the family to facilitate a iscussion. wiene meiastinum, in conjunction with the high-spee
Any inconsistencies of the patient’s wishes regaring organ eceleration injury is concerning for blunt aortic injury.
238 PArt i Patient Care

Management of traumatic blunt aortic injury typically begins of splenic injuries. Patients shoul have no other inications
with bloo pressure an pain control. Management epens for laparotomy on the basis of physical exam nings (peri-
on injury grae (E). However, it is more that the source of tonitis or hemorrhagic shock) or the results of other iagnos-
the patient’s hemoynamic instability (with the positive tic tests (free air on CT scan of the abomen) an shoul be
FAST scan) is bleeing in the abomen. A transthoracic evaluable (absence of a complete high spinal cor injury or
echocariogram may provie information regaring car- intoxication). The presence of a traumatic brain injury oes
iac function an volume status, but it is not inicate given not preclue NOM, nor oes oler age. An increasing vol-
the patient’s ongoing hemoynamic instability (D). Due the ume of hemoperitoneum is associate with higher failure
patient’s ongoing shock an nonresponsiveness to a crystal- rates of NOM as is an increasing American Association for
loi challenge, transfusion of bloo proucts in conjunction the Surgery of Trauma (AAST) grae of injury. Angiography
with hemorrhage control, shoul be initiate. with embolization shoul be consiere for patients with
References: American College of Surgeons Committee on AAST injury grae of greater than III, presence of a contrast
Trauma. Advanced trauma life support program for doctors. 9th e. blush, moerate hemoperitoneum, evience of ongoing
American College of Surgeons; 01. splenic bleeing (requiring > units of packe re bloo cells
CRASH- Trial Collaborators, Shakur H, Roberts I, et al. Effects of [PRBCs]), presence of a pseuoaneurysm or suspecte arte-
tranexamic aci on eath, vascular occlusive events, an bloo trans-
riovenous stula provie that they are hemoynamically
fusion in trauma patients with signicant haemorrhage (CRASH-):
stable. Serial abominal exams an trening the hematocrit
a ranomise, placebo-controlle trial. Lancet. 010;376(9734):3–3.
Demetriaes D, Velmahos GC, Scalea TM, et al. Blunt traumatic
woul be inappropriate in the presence of active extravasa-
thoracic aortic injuries: early or elaye repair–results of an Amer- tion of contrast (D). If angioembolization is not available, lap-
ican Association for the Surgery of Trauma prospective stuy. J aroscopic an open splenectomy are both reasonable options
Trauma. 009;66(4):967–973. in hemoynamically stable patients that meet the above ini-
Henry DA, Carless PA, Moxey AJ, et al. Anti-brinolytic use for cations for surgery (A, E), whereas in the unstable patient or
minimising perioperative allogeneic bloo transfusion. Cochrane with iffuse peritonitis, open splenectomy is recommene.
Database Syst Rev. 007;(4):CD001886. Once in the operating room, attempts at splenic preservation
via splenorrhaphy are reasonable in hemoynamically stable
28. E. Penetrating extremity trauma may be accompanie patients (B).
by har or soft signs of vascular injury. Har signs inclu- Reference: Stassen NA, Bhullar I, Cheng JD, et al. Selective
ing shock, pulsatile bleeing, expaning or pulsatile hema- nonoperative management of blunt splenic injury: an Eastern Asso-
toma, palpable thrill or bruit, or absent istal pulses warrant ciation for the Surgery of Trauma practice management guieline. J
immeiate operative exploration (B). Soft signs are nings Trauma Acute Care Surg. 01;73(5 Suppl 4):S94–S300.
on the physical exam that are suggestive of a potential vas-
cular injury an require further iagnostic testing. Soft signs 30. C. The physical exam nings are concerning for the
inclue iminishe pulse, proximity of wouns to vessels, presence of a urethral injury. The most common location is
hematomas, an reports of signicant bloo loss. Given the at the prostatic urethra. Genitourinary injuries may occur
absence of a har sign, this patient is stable to unergo fur- in up to 15% of patients with pelvic fractures. Hea injury
ther iagnostic workup an oes not require an immeiate is the most common associate injury seen in patients with
operation. Ankle-brachial inex (ABI) is both sensitive an pelvic fractures. Clinical suspicion of a urethral injury war-
specic for lower extremity vascular injuries. In compari- rants the performance of a RUG to ientify the presence
son to CT angiography or formal angiography, ABI oes not an location of a urethral injury. Blin insertion of a Foley
require ionizing raiation or the aministration of contrast catheter is contrainicate in this patient (A). CT abomen
(A, D). ABI less than 0.9 is suggestive of vascular injury an with IV contrast is helpful for ientifying injuries to the ki-
prompts a CT angiography. Signicant vascular injury can be neys an elaye acquisition images may also ai in the
exclue with a negative preictive value of 99% when ABI ientication of ureteral or blaer injuries (B). A CT cysto-
is >0.9. An alternative to ABI is API an is use in the same gram accurately iagnoses both extraperitoneal an intra-
way. API is the arterial pressure just istal to the injury com- peritoneal blaer injuries (D). Intravenous pyelogram is
pare to the uninvolve contralateral extremity. Although use to ientify renal injuries an is rarely performe (E).
arterial vasospasm may occur following proximity trauma, Management of urethral injuries epens on the location
this iagnosis is usually one of exclusion an oes not war- an severity of injury, as well as presence of associate inju-
rant immeiate treatment with papaverine (C). ries, an surgical expertise.
References: Feliciano DV, Moore EE, West MA, et al. Western Reference: Johnsen NV, Dmochowski RR, Mock S, Reynols
Trauma Association critical ecisions in trauma: evaluation an WS, Milam DF, Kaufman MR. Primary enoscopic realignment
management of peripheral vascular injury, part II. J Trauma Acute of urethral isruption injuries—A ouble-ege swor? J Urol.
Care Surg. 013;75(3):391–397. 015;194(4):10–106.
Feliciano DV, Moore FA, Moore EE, et al. Evaluation an man-
agement of peripheral vascular injury. Part 1. Western Trauma Asso- 31. C. Although A, B, C (Airway, with cervical spine pre-
ciation/critical ecisions in trauma. J Trauma. 011;70(6):1551–1556. cautions; Breathing; Circulation with hemorrhage control)
Johansen K, Lynch K, Paun M, Copass M. Non-invasive vascular has always been the recommene sequence in trauma
tests reliably exclue occult arterial trauma in injure extremities. J patients, recent recommenations are shifting to C, A, B in
Trauma. 1991;31(4):515–5. those with penetrating injuries who are severely hypoten-
sive, as the combination of rapi-sequence intubation an
29. C. Nonoperative management (NOM) of soli organ positive pressure ventilation can worsen hypotension an
injuries is a well-accepte treatment moality. Several crite- lea to cariac arrest (A). Thus, bloo proucts woul be the
ria shoul be consiere when selecting patients for NOM preferre rst step, followe by immeiate transport to the
CHAPtEr 16 Trauma 239

operating room. Some meical centers are now proviing of CPR than long uration, an stab wouns than GSW (A–C,
initial resuscitation with whole bloo for the trauma patient E). Thus, the best scenario for resuscitative thoracotomy woul
in shock. Given the location of the injury (zone I of the neck), be an isolate stab woun to the chest, with SOL (survival
one shoul have a high suspicion for a right subclavian or from poole ata is 1%). Such a patient is much more likely to
innominate artery injury. Once in the operating room (if pos- have arreste ue to cariac tamponae an therefore has not
sible), the patient is preppe an rape prior to intubation. suffere exsanguinating hemorrhage. Conversely, at the other
REBOA (E) is utilize for control of vascular injuries below extreme, for blunt trauma without SOL, survival was only 0.7%.
the iaphragm. Proximal control of such an injury on the The following are consiere SOL: agonal respirations, cariac
right via an open approach is best achieve by a meian ster- electrical activity, palpable pulse, measurable bloo pressure,
notomy. If the same injury were present on the left, proximal spontaneous movement, or pupillary reactivity. Thus, the ben-
control of the left subclavian artery is best achieve via a left et of resuscitative thoracotomy for SOL an penetrating chest
anterolateral thoracotomy. Enovascular balloon occlusion is trauma is clear. Less compelling but still potentially benecial
another option. If bloo is exsanguinating through the bullet inications woul be penetrating chest trauma without SOL,
hole, manual compression in this area is ineffective. Tempo- penetrating extrathoracic injury with or without SOL, an
rary tamponae can be achieve via insertion an ination blunt trauma with SOL. There is no benet for blunt trauma
of a Foley balloon irectly into the woun, permitting rapi with no SOL. For those that survive, a surprising majority sur-
transportation to the operating room. Thoracostomy is ini- vive with favorable neurologic outcomes.
cate for pneumothorax or hemothorax seen on raiograph References: Burlew CC, Moore EE, Moore FA, et al. Western
imaging or after primary survey suggestive of these conitions Trauma Association critical ecisions in trauma: resuscitative tho-
(B). The above patient has not ha a cariopulmonary arrest, racotomy: resuscitative thoracotomy. J Trauma Acute Care Surg.
nor oes he meet any inication for ED thoracotomy (D). 01;73(6):1359–1363.
Seamon MJ, Haut ER, Van Arenonk K, et al. An evience-base
References: American College of Surgeons Committee on
approach to patient selection for emergency epartment tho-
Trauma. Advanced trauma life support program for doctors. 9th e.
racotomy: a practice management guieline from the Eastern
American College of Surgeons; 01.
Association for the Surgery of Trauma. J Trauma Acute Care Surg.
Demetriaes D, Chahwan S, Gomez H, et al. Penetrating
015;79(1):159–173.
injuries to the subclavian an axillary vessels. J Am Coll Surg.
1999;188(3):90–95.
34. E. The preferre access for young chilren an infants
32. D. With penetrating neck trauma, there is concern that following trauma is via the peripheral percutaneous route
bleeing may rapily compress the trachea. As such, the rst (antecubital fossa or saphenous vein at the ankle). After two
step in the management algorithm is to establish an airway, unsuccessful attempts, consieration shoul be given to
particularly in the presence of an expaning hematoma or IO infusion via a bone marrow neele (18 gauge in infants,
epresse level of consciousness. If the patient has a “har 15 gauge in young chilren). IO cannulation of the proxi-
sign” of a vascular injury, such as a rapily expaning or pul- mal tibia provies goo short-term access for resuscitation
satile hematoma, visible exsanguination, palpable thrill or because it targets the noncollapsible veins of the meullary
auible bruit, or ense neurologic ecit (such as this patient sinus. The optimal site of insertion is the anteromeial tibia
with GCS 8), the patient shoul then be transporte irectly to  to 3 cm below the tibial tuberosity, ensuring to angle away
the OR. If the patient is hemoynamic unstable, without har from the growth plates. This can be performe using a bone
signs, the presumption shoul be that the patient exsanguina- marrow neele or an IO vascular access system such as the
te in the el. Thus, shock is another inication for immeiate EZ-IO®. Once the patient has been resuscitate, follow-up
surgical exploration (this patient has a low BP as well). Con- attempts at peripheral access shoul be mae. If a patient
versely, in the absence of har signs, the next step woul be to has obvious eformities in the tibiae (as in this patient), the
obtain CT arteriography of the neck vessels. This historically next location for IO cannulation woul be the istal femur
has been achieve with formal arteriography, because of the just above the femoral conyles (D). In aults, there has been
ease an rapiity of its use (B, C). In aition, an assessment a shift in recent years, an sternal IO access is now consi-
for injuries to the aeroigestive tract (triple enoscopy an/or ere the preferre initial site for cannulation (thinner cortex
esophagography) an cervical spine nees to be performe (E). an abunant re bone marrow) followe by the tibia. The
As a general guie, repairing a caroti artery injury in a patient proximal humerus is an aitional option in aults. It is
with a neurologic ecit is recommene as it may result in also important to note that serum electrolytes, bloo gases,
improve neurologic function, whereas caroti ligation typi- an type an cross can all be performe using bloo from
cally oes not. Repair can be achieve by primary suturing, interosseous access. A istal saphenous vein cutown is
resection with a primary reanastomosis, or interposition graft another option in chilren ages 1 to 6 years, but in a chil
placement (saphenous vein or polytetrauoroethylene) (A). younger than 1 year of age, it woul be challenging an not
appropriate in the setting of obvious leg eformity (B). In
33. D. Resuscitative thoracotomy is a potentially lifesaving hypovolemic peiatric patients younger than 6 years of age,
proceure. Inications an guielines continue to evolve. There percutaneous femoral vein cannulation is another alternative
are many articles in the literature on the topic, with variable but is associate with an increase risk of venous thrombo-
nings an recommenations. However, several overarching sis an woul be much more challenging in a chil younger
themes consistently permeate these stuies. Outcomes are bet- than 1 year (C). Subclavian an internal jugular central lines
ter for those with SOL than those without, penetrating trauma woul be too ifcult to perform in the trauma setting in
than blunt, chest trauma than abominal, isolate injury than such a small chil an woul be associate with an increase
multiple injuries, without hea injury than with, short uration risk of iatrogenic injury (A). The interosseous cannula shoul
240 PArt i Patient Care

be remove expeitiously (within 4 hours) because of the Kim DY, Coimbra R. Thoracic amage control. In: Di Saverio S,
potential risk of infectious complications incluing osteomy- Tugnoli G, Catena F, Ansaloni L, Naioo N, es. Trauma surgery: vol-
elitis. Extremity compartment synrome is another potential ume 2: thoracic and abdominal Trauma. Springer Milan; 014:35–46.
complication of IO infusion. 37. C. Flail chest occurs when two or more ribs are frac-
References: Cullen PM. Intraosseous cannulation in chilren. ture in at least two locations. Paraoxical movement of this
Anaesth Intensive Care Me, 01; 13:8–30. free-oating segment of chest wall is typically not sufcient
Pasley J, Miller CHT, DuBose JJ, et al. Intraosseous infusion rates alone to compromise ventilation (B). Rather, pain an splint-
uner high pressure: a caaveric comparison of anatomic sites. J ing, in conjunction with unerlying pulmonary contusions,
Trauma Acute Care Surg. 015;78():95–99. may result in hypoxemia an hypercarbia ue to shunting
an ineffective ventilation, respectively. Most patients can
35. D. Both bloo volume an re cell volume increase in be manage without intubation (E). Respiratory failure
the pregnant patient, but bloo volume increases more than often oes not occur immeiately, an frequent reevaluation
re cell volume. Bloo volume increases by approximately is warrante. The initial chest raiograph usually uneres-
50% as term approaches, whereas re cell volume increases timates the egree of pulmonary contusion, an the lesion
by approximately 30%, resulting in a functional hemoilu- tens to evolve with time an with ui resuscitation (A).
tion an resultant physiologic anemia of pregnancy (A). Intravenous ui aministration shoul be limite as over-
Thus, pregnant patients are less likely to manifest signs of zealous resuscitation may result in blossoming of pulmo-
bloo loss such as tachycaria an hypotension, an if such nary contusions (D). The most important aspect of treatment
signs are present, they are inicative of an even more severe of ail chest is pain control. Stanar approaches inclue
bloo loss than in the nonpregnant patient (on the orer of the use of patient-controlle analgesia an oral pain mei-
1500–000 mL of bloo loss). The pregnant patient has an cations an the placement of continuous epiural catheters.
increase tial volume an minute ventilation, esigne to Although the treatment of ail chest has historically been
increase oxygen release to the fetus. This results in a mil nonoperative, recent literature inicates that internal xa-
respiratory alkalosis, with a PCO in the 7 to 3 range (B). tion of the chest wall in select patients without pulmonary
Oxygen consumption is increase, an functional resiual contusion ecreases intubation time, ecreases mortality,
capacity is ecrease. In aition, the ,3-iphosphoglyc- shortens uration of mechanical ventilation as well as hospi-
erate level is increase to enhance the release of oxygen to tal stay, ecreases complications, an improves cosmetic an
the fetus. However, these physiologic changes result in less functional results. In the presence of a pulmonary contusion,
pulmonary reserve in an acutely ill pregnant patient. The however, internal xation may not be as benecial. Eastern
use of raiographs is thought to be safe for the fetus after Association for the Surgery of Trauma (EAST) guielines rec-
the 0th week of gestation (C). The glomerular ltration rate ommen ORIF in aults with ail chest after blunt trauma.
increases, resulting in a ecrease in serum creatinine (E). Situations in which internal xation shoul be consiere
Other important aspects to be aware of are that the gravi inclue ail chest in patients who are alreay unergoing
uterus can compress the IVC, resulting in ecrease venous thoracotomy for an intrathoracic injury, ail chest without
return. Therefore, the pregnant patient shoul be place in pulmonary contusion, noticeable paraoxical movement of
the left lateral position at approximately 15 egrees. Preg- a chest wall segment while a patient is being weane from
nant patients are more prone to aspiration, so early NG the respirator, an severe eformity of the chest wall.
tube ecompression is important. Finally, the progressive References: Kasotakis G, Hasenboehler EA, Streib EW, et al.
stretching of the peritoneum leas to esensitization so that Operative xation of rib fractures after blunt trauma: a practice man-
a pregnant patient is less likely to emonstrate peritoneal agement guieline from the Eastern Association for the Surgery of
signs. Trauma. J Trauma Acute Care Surg. 017;8(3):618–66.
Reference: Shah AJ, Kilcline BA. Trauma in pregnancy. Emerg Leinicke JA, Elmore L, Freeman BD, Colitz GA. Operative man-
Med Clin North Am. 003;1(3):615–69. agement of rib fractures in the setting of ail chest: a systematic
review an meta-analysis. Ann Surg. 013;58(6):914–91.
36. D. In the past, the injury escribe woul have been Voggenreiter G, Neueck F, Aufmkolk M, Obertacke U,
ealt with by performing a formal lobectomy (A). However, Schmit-Neuerburg KP. Operative chest wall stabilization in ail
pulmonary tractotomy is now use as a less aggressive alter- chest–outcomes of patients with or without pulmonary contusion. J
Am Coll Surg. 1998;187():130–138.
native. The technique involves using a linear stapling evice
to insert irectly into the injure bullet tract. Two hemostatic
staple lines are create, an the lung is ivie in between. 38. A. BCI shoul be suspecte in anyone with severe blunt
This allows irect access to the bleeing vessels within the chest trauma. Attempts to ientify a BCI an stratify severity
parenchyma as well as any leaking bronchi. Bleeing vessels on the basis of CK-MB, nuclear scans, an echocariography
can then be oversewn with a polypropylene monolament have not been successful because these moalities lack sen-
(C). Lobectomy is a better choice for a completely evascu- sitivity. ECG is the most commonly recommene tool for
larize or estroye lobe. A pneumonectomy is rarely ini- the initial iagnosis of BCI. The presence of a sternal fracture
cate an, in the trauma setting, is associate with an 80% is not a marker for BCI (D). A normal screening ECG has a
mortality rate (B). Similarly, ligation of a lobar pulmonary negative preictive value of 95% (E). Aition of a normal
artery has a high rate of morbiity (E). cariac troponin increases the negative preictive value to
References: Cothren C, Moore EE, Bif WL, et al. Lung- 100%. If a stable patient has an abnormal cariac troponin
sparing techniques are associate with improve outcome level or ECG, he/she shoul be amitte for observation to
compare with anatomic resection for severe lung injuries. J Trauma. a monitore be. However, troponin level oes not correlate
00;53(3):483–487. with risk of cariac complications in BCI. If the patient is
CHAPtEr 16 Trauma 241

unstable, an emergent echocariogram shoul be performe. Ligation of the IVC below the renal veins is better tolerate
If a tamponae is seen, emergent sternotomy shoul be per- than the suprarenal IVC; however, marke leg swelling may
forme for suspecte cariac rupture. Very rarely, BCI can evelop an may require fasciotomies. Ligation of the supe-
lea to coronary artery thrombosis, valvular isruption, or rior mesenteric vein is also fairly well tolerate an better
septal isruption (C). In an unstable patient with BCI with- tolerate than portal vein ligation, although again it is pref-
out an anatomic abnormality on echocariography, invasive erable to repair the superior mesenteric vein if the patient is
bloo pressure monitoring with pressor support shoul be stable an it is technically feasible because there is similarly
institute. Most patients with a iagnosis of myocarial marke bowel eema an risk of bowel infarction as with
contusion have a benign course, with very few eveloping portal vein repair. Arteries for which repair shoul always
arrhythmias or heart failure (B). be attempte inclue the innominate, brachial, superior mes-
References: Clancy K, Velopulos C, Bilaniuk JW, et al. Screening enteric, proper hepatic, iliac, femoral, an popliteal arteries
for blunt cariac injury: an Eastern Association for the Surgery of an the aorta (C, D). If enitive repair is preclue ue to
Trauma practice management guieline. J Trauma Acute Care Surg. hemoynamic instability or if a amage control approach is
01;73(5 Suppl 4):S301–S306. eeme appropriate, perfusion or ow may be maintaine
Velmahos GC, Karaiskakis M, Salim A, et al. Normal electrocari-
via a temporary intravascular shunt. In the forearm, either
ography an serum troponin I levels preclue the presence of clini-
the raial or ulnar artery can be ligate, provie the other
cally signicant blunt cariac injury. J Trauma. 003;54(1):45–50.
vessel is palpable. Similarly, in the lower leg, at least one of
39. E. The Cattell maneuver involves a right meial visceral the two palpable vessels (anterior or posterior tibial artery)
rotation of the cecum an ascening colon. It is achieve by shoul be salvage. Because of the excellent collateraliza-
incising the peritoneal reection at the white line of Tolt. tion aroun the shouler, ligation of the subclavian artery
It is useful for exposing right retroperitoneal structures, is well tolerate. In fact, the artery is often occlue uring
such as the IVC an the right ureter (C). Further cephala, stent-grafting of thoracic aneurysms or aortic transection.
mobilization an meial rotation of the uoenum (Kocher Reference: Rich NM, Mattox KL, Hirshberg A. Vascular trauma.
n e. Elsevier Science; 004.
maneuver) aitionally assists in exposing the suprarenal
IVC below the liver. The Kocher maneuver is not useful for
41. A. Extremity compartment synrome can occur any-
exposing the celiac axis (D). This is best one by combining
where in the extremities, incluing the buttocks, shoulers,
a Mattox maneuver with a ivision of the left crus of the
an hans (E). The mechanisms of compartment synrome
iaphragm an iviing the celiac plexus (A). The Mattox
are numerous an can be ivie into extrinsic an intrinsic
maneuver consists of a left meial rotation of the escen-
causes. Extrinsic causes inclue constriction by a cast, tight
ing colon (again at the line of Tolt), spleen, an/or kiney
circumferential ressings, or eschar from a burn. Intrinsic
towar the miline. Exposure of injuries to the istal IVC
causes are ivie into bleeing, eema, an exogenous ui.
an iliac vein bifurcations can be exceeingly ifcult. On
Bleeing is usually ue to trauma but can also be seen after
occasion, ivision of the right common iliac artery is neee
relatively minor injuries in patients with an unerlying coagu-
to expose an repair an injury of this area. A primary repair
lopathy or those receiving anticoagulants. Eema of the com-
of the iliac artery can then be performe. On rare occasions,
partment is the largest an broaest category. It is most often
with massive bleeing, the junction of the superior mesen-
seen after reperfusion of an ischemic limb, from either an arte-
teric vein (not artery), splenic, an portal veins may nee to
rial embolus or thrombosis or trauma. Ischemia/reperfusion
be expose by ivision of the neck of the pancreas (B).
is also seen in a person with a rug overose or an alcoholic
References: Asensio JA, Chahwan S, Hanpeter D, et al. Opera-
who falls asleep on the limb, in patients with profoun shock
tive management an outcome of 30 abominal vascular injuries.
Am J Surg. 000;180(6):58–533.
in whom iffuse muscle ischemia with subsequent reperfu-
Hoyt DB, Coimbra R, Potenza BM, Rappol JF. Anatomic expo- sion evelops, an after massive iliofemoral eep venous
sures for vascular injuries. Surg Clin North Am. 001;81(6):199–1330. thrombosis. Finally, inavertent infusion of IV ui into the
subcutaneous tissue can lea to compartment synrome.
40. B. Most veins can be safely ligate in the setting of trau- Diagnosis of compartment synrome begins by having a high
matic injury. However, certain veins are less likely to tolerate clinical inex of suspicion an knowing the clinical scenar-
ligation well. These inclue the superior vena cava (because ios in which it occurs. The most common features are severe
it may result in an acute superior vena cava synrome), the pain in the limb typically out of proportion to the physical
renal veins close to the renal parenchyma (because there is exam, pain on passive motion of the limb, an tense eema
then inaequate outow for the kiney), the IVC above the with tenerness on palpation of the compartment. Distal
renal veins (because it will impair outow to both kineys), arterial pulses typically remain palpable with compartment
or just at the iaphragm (because this will cause an acute synrome. The anterior compartment of the leg is usually
Bu-Chiari synrome), an the portal vein (because it sup- the rst compartment to be involve in the lower extrem-
plies 75% of the bloo to the liver) (A, E). An exception to the ity (C). The eep peroneal nerve runs within it so numbness
aforementione is ligation of the left renal vein close to the in the rst web space of the toe is one of the early nings
IVC is well tolerate because rainage can occur via the are- (D). Once the iagnosis is suspecte, conrmation is sought
nal, gonaal, an iliolumbar veins. This is sometimes per- by oing irect pressure measurements of the iniviual
forme uring open abominal aortic aneurysm repair. The compartments. If the pressures are increase more than
portal vein has been ligate successfully, provie aequate 30 mmHg in any of the compartments, then strong consi-
ui is aministere to compensate for the ramatic but eration shoul be given to performing a four-compartment
transient eema that occurs in the bowel, but ligation seems fasciotomy. The use of an absolute value has been questione
to be associate with a higher mortality rate than repair. because the perfusion pressure necessary for oxygenation
242 PArt i Patient Care

is partly epenent on the patients’ bloo pressure an, maneuver. Alternatively, an atriocaval (Schrock) shunt coul
therefore, coul lea to unnecessary fasciotomies (B). The be place or venovenous bypass initiate.
use of ifferential pressure (Δp = iastolic bloo pressure— References: Asensio JA, Demetriaes D, Chahwan S, et al.
intracompartmental pressure), with a propose threshol Approach to the management of complex hepatic injuries. J Trauma.
of 30 mmHg, has been propose to be of greater iagnostic 000;48(1):66–69.
value. It is also important to remember that there is no abso- Kozar RA, Feliciano DV, Moore EE, et al. Western Trauma Asso-
ciation/critical ecisions in trauma: operative management of ault
lute pressure level that rules compartment synrome in or
blunt hepatic trauma. J Trauma. 011;71(1):1–5.
out. The measurements shoul be use in conjunction with
the patient’s clinical examination. The eep posterior com-
43. C. The majority of blaer injuries occur following a
partment is the one that is most commonly inaequately
blunt mechanism of injury, an over 80% of patients with
ecompresse. Because this compartment contains the tibial
a blaer rupture will have a concomitant pelvic fracture.
nerve, missing this compartment can have evastating con-
Blaer injuries are classie as extraperitoneal, intraperi-
sequences. The soleus muscle must be etache from the
toneal, or combine, with extraperitoneal injuries being the
tibia to ecompress the eep posterior compartment. But-
most common (as many as 70%). Extraperitoneal blaer
tock compartment synrome has been escribe in patients
injuries often result from perforation ue to ajacent pelvic
with obesity after prolonge anesthesia as well.
bony fragments or spicules, whereas intraperitoneal injuries
Reference: von Keuell AG, Weaver MJ, Appleton PT, et al.
typically occur ue to a suen increase in pressure when
Diagnosis an treatment of acute extremity compartment synrome.
Lancet. 015;386(10000):199–1310.
a full blaer sustains a irect blow (i.e., MVC following
binge-rinking). These injuries usually result in large tears
involving the ome of the blaer. Hematuria in the pres-
42. C. The management of liver injuries has unergone a ence of a pelvic fracture shoul increase the suspicion for a
major evolution in the past 5 years, from routine laparot- blaer injury. If bloo is visible at the urethral meatus, then
omy in the past to the current application of selective non- a Foley catheter shoul not be inserte until a retrograe
operative management in hemoynamically stable patients,
urethrogram is performe to rule out a urethral injury (A).
liberal use of angiographic embolization, an operative man-
Otherwise, in the presence of hematuria, the iagnosis of a
agement with selective packing an amage control when
blaer injury can usually be mae by stress cystography.
the patient is col an coagulopathic. In a patient who has This may be performe using a stanar raiographic or CT
sustaine blunt trauma an is hemoynamically stable, technique. Avantages of CT cystography inclue the abil-
a CT scan with IV contrast shoul be performe. If a con-
ity to assess other abominal an pelvic injuries. Typically,
trast blush is seen in the liver, the patient shoul be taken to
300 to 400 cc of ioinate contrast is instille into the blaer
angiography for embolization, provie there are no other via the Foley catheter, which is then clampe. When extrav-
injuries that require operative intervention. Conversely, if the asation is seen, it is important to etermine whether it is
patient is hemoynamically unstable (as in this patient), the
intraperitoneal, extraperitoneal, or both. Contrast above the
patient shoul be taken to the operating room an unergo
peritoneal reection is intraperitoneal (the paracolic gutter
packing of all four quarants to obtain temporary hemosta- woul be intraperitoneal). The management of an extraperi-
sis while anesthesia attempts to “catch up” or aequately toneal rupture of the blaer is nonsurgical in most instances
resuscitate the patient. Strong consieration shoul be given
an consists of placing an 18- to 0-French or larger Foley
to activating the institutional massive transfusion protocol
catheter for 7 to 10 ays followe by a repeat cystogram to
in aition to aministering tranexamic aci. Given that this ensure no further extravasation of contrast before catheter
patient ha continue bleeing espite application of a Prin- removal. Intraperitoneal injuries are manage operatively
gle maneuver, he has likely sustaine an injury to the ret-
via a transabominal approach. Before closure of the injury,
rohepatic IVC or hepatic veins. If the bleeing is controlle
palpation an visualization of the interior of the blaer
with packing an, in aition, the patient is col (tempera-
shoul be performe to ensure absence of other injuries.
ture <34 °C), coagulopathic, an with a refractory aciosis Repair is unertaken using absorbable sutures. Silk suture
(as in this patient), the best option woul be to perform a is inappropriate because permanent sutures in the blaer
amage control operation an transfer the patient to the ICU
will increase the risk of ongoing blaer mucosal irritation
for resuscitation (A, B, D, E). If, conversely, the bleeing is not
an are lithogenic (E). A suprapubic cystostomy is generally
controlle, the next step woul be to rapily take own the not require in the absence of very large wouns or the pres-
hepatic ligaments incluing the ligamentum teres, falciform ence of signicant evitalize tissue (B). If CT cystography is
ligament, triangular ligament, an the right coronary liga-
equivocal, a formal cystogram shoul be obtaine; it is oth-
ment, an perform a Kocher maneuver. This allows better
erwise unnecessary (D).
irect compression with packing in the retrohepatic space. A Reference: Myers JB, Taylor MB, Brant WO, et al. Process
ecision must then be mae as to whether to attempt repair improvement in trauma: traumatic blaer injuries an compliance
of a retrohepatic IVC injury. This ecision epens on the with recommene imaging evaluation. J Trauma Acute Care Surg.
experience of the surgeon, the clinical status of the patient, 013;74(1):64–69.
an whether bleeing is controlle. If bleeing has now
stoppe with packing, one option is to take the patient back 44. A. Ureteral injuries are relatively uncommon an most
to the ICU to resuscitate an rewarm. If bleeing persists, often occur following penetrating trauma. Surgical manage-
total vascular exclusion of the liver is now possible because ment is ictate by the patient’s hemoynamics, as well as
control of the IVC just below the iaphragm an just infe- level of injury (upper, mile, or lower thir), egree of ure-
rior to the liver can be performe, combine with the Pringle teral loss, an status of surrouning tissues. Ureteral repairs
CHAPtEr 16 Trauma 243

following trauma are usually repaire over a stent. For upper the luminal circumference, then resection is often require.
an mile thir urethral injuries that have a small ureteral If such an injury is in the rst, thir, or fourth portion of the
segment missing (< cm), a primary repair can often be uoenum, then resection with uoenouoenostomy
one. Reimplantation to the blaer (ureteroneocystostomy) (as in this patient) or uoenojejunostomy may nee to be
is preferre for small segment injuries of the lower thir as performe. Segmental resection of the secon part is chal-
it is technically easier to perform compare with primary lenging because of the presence of the ampulla of Vater an
repair (E). For larger ureteral injuries involving the upper or the common bloo supply with the pancreas, which make
mile ureter, the ieal repair entails ebriing evitalize the mobilization ifcult. In this situation or if a tension-free
tissue, spatulating the two ens, an performing an en-to- anastomosis is not possible, a Roux-en-Y uoenojejunos-
en anastomosis over a ouble J stent (ureteroureterostomy) tomy may be inicate. Alternatively, a irect Roux-en-Y
using an absorbable monolament (B). Some mobilization of loop anastomose over the uoenal efect may be consi-
the ureter is feasible, but mobilization risks interrupting the ere. Pyloric exclusion shoul be consiere in rare cases
bloo supply that runs just ajacent to the ureter. As such, with tenuous repairs (A). Duoenal iverticulization is not
the issection shoul be maintaine approximately 1 cm commonly performe (B). A Whipple resection can be con-
away from the ureter so as not to isrupt its bloo supply. siere for a subsequent surgery in patients with grae V or
A goo guie to the viability of the two ens of the ureter is complexe combine pancreatic an uoenal injuries (D).
whether the cut eges are bleeing. Lower ureteral injuries
may require reimplantation of the ureter into the blaer if 46. D. Various techniques may be employe to control
there is not enough istal ureter for a primary anastomosis. bleeing from the liver. The simplest metho of controlling
When a large segment of ureter has been injure an primary bleeing from the liver is the application of manual com-
reanastomosis is not possible, several options are available. pression with or without the use of topical hemostatic agents
A psoas hitch involves mobilization of the blaer, which is such as microbrillar collagen, oxiize cellulose, an gel-
then suture to the iliopsoas fascia above the iliac vessels, atin matrix thrombin sealants. If these are unsuccessful, a
to perform tension-free reimplantation of the ureter (D). If a Pringle maneuver shoul be performe. Ongoing bleeing
tension-free repair cannot be achieve following mobiliza- following occlusion of the porta hepatis suggests the poten-
tion of the blaer, a Boari or blaer ap may be consi- tial for a hepatic vein or retrohepatic IVC injury. In aition
ere. More complex techniques inclue anastomosing the to packing, several other hemostatic maneuvers can be use
ureter to the contralateral ureter (transureteroureterostomy), in patients with severe parenchymal injury. Liver sutures
ileal-ureteral replacement, an renal autotransplantation can be place, using a chromic suture with a blunt-tippe
(C). In this patient, however, with massive bloo loss an neele. This is best use for relatively supercial lacerations.
hemoynamic instability, a amage control approach shoul Another option is to perform a hepatotomy via a nger frac-
be use. There are two options. The rst is to simply ligate ture technique to access the bleeing site to irectly suture
the ureter proximally an istally followe by placement of it. However, profuse bleeing from a small hole in the liver
a percutaneous nephrostomy once the patient is stabilize. presents a more ifcult ilemma because bleeing may be
The patient can later be taken back for a more elective repair emanating from the center of the liver, an a hepatotomy
of the ureter. The other option is to perform a temporary may not be feasible. In this circumstance, one novel approach
cutaneous ureterostomy over a single J stent, placing a tie that has been well escribe is to fashion a balloon tampon-
aroun the ureter an stent an then bringing the stent up to ae catheter. A catheter with sie holes is place through a
the level of the skin. Given the location of the injury an the Penrose rain, an a tie is place on either en of the Penrose
length of injure ureter, the patient woul eventually likely rain (E). The catheter is avance into the bullet woun,
nee a psoas hitch or other more complex repairs. an air with or without contrast is insufate into the cathe-
Reference: Smith TG 3r, Coburn M. Damage control maneu- ter, effectively inating the Penrose rain an creating a tam-
vers for urologic trauma. Urol Clin North Am. 013;40(3):343–350. ponae effect. In this case, however, because the bleeing has
stoppe, there is no role for any aitional treatment (A–C).
45. E. The current tren in the management of severe uo- Placing liver stitches is unnecessary an oes increase the
enal injuries is “less is better.” Management of uoenal risk of causing liver necrosis. Packing the injury with omen-
injuries epens on location, extent of injury, associate tum is useful in large stellate lesions, but hemostasis is better
pancreatic injury, an clinical status of the patient. Duo- achieve in that setting with packing. The use of rains is
enal injuries are grae from I to V, with grae I being a controversial. For smaller wouns, rains are not recom-
hematoma or partial-thickness injury an grae V being a mene. For larger injuries, close suction rainage is use
massive isruption of the pancreaticouoenal complex or by some surgeons. In general, open rains shoul not be
complete uoenal evascularization. If a simple uoenal employe because of a potentially increase risk of infection.
hematoma is recognize preoperatively, it can be manage Reference: Kozar RA, Feliciano DV, Moore EE, et al. Western
without surgery, with nasogastric ecompression an paren- Trauma Association/critical ecisions in trauma: operative manage-
teral nutrition. If it is foun intraoperatively, it is left alone ment of ault blunt hepatic trauma. J Trauma. 011;71(1):1–5.
if small (< cm). If it is a large hematoma (involving >50%
of the lumen), it is recommene to incise the serosa, rain 47. C. Increasingly, colon injuries are being treate with
the hematoma, an then reclose the serosa. The majority of either primary repair, if feasible, or resection with a pri-
full-thickness lacerations of the uoenum can be repaire mary anastomosis (A, B, D, E). This approach applies to
primarily in a transverse fashion to avoi narrowing the both right- an left-sie colon injuries. Primary repair is
lumen, with or without placement of an overlying omental use when less than 50% of the circumference of the bowel
patch (C). Conversely, if the injury involves more than 50% of is involve, whereas resection is recommene for larger
244 PArt i Patient Care

wouns. Once a resection is performe, a ecision must be pneumothorax worsening in patients unergoing positive
mae as to whether to perform a primary reanastomosis pressure ventilation (urgent or elective surgery with general
or a colostomy. The primary contrainication to attempt- anesthesia) most stuies inicate that patients can safely be
ing a primary reanastomosis is hemoynamic instability. observe without a chest tube (A, B). In recent years, there
In these situations, amage control surgery shoul be per- has been a shift towar the use of small-bore chest tubes as
forme an the ecision to reanastamose or create a colos- several reports have suggeste no ifference in outcomes
tomy can be mae at a subsequent operation when the between large an smaller bore chest tubes (A, D). Neele
patient has stabilize an been fully resuscitate. Factors ecompression is still consiere the rst-line intervention
associate with intraabominal complications in patients for patients with a tension pneumothorax (E).
with severe colon injuries unergoing resection inclue Reference: Mowery NT, Gunter OL, Collier BR, et al. Practice
severe fecal contamination, transfusion of 4 or more units management guielines for management of hemothorax an occult
of bloo in the rst 4 hours, an aministration of sin- pneumothorax. J Trauma. 011;70():510–518.
gle-agent antibiotics. The use of vasopressors at the time
of repair may also be associate with anastomotic leaks, 49. A. This patient has multiple conitions that coul lea
whereas the metho of performing the anastomosis (han- to the evelopment of ascites. His heavy alcohol use coul
sewn versus staple) has not been shown to effect leak lea to cirrhosis an hepatic ascites or spontaneous bacte-
rates. Another important consieration is obesity. Morbi rial peritonitis. His recent blaer repair an splenectomy
obesity makes the creation of a stoma ifcult, preis- raise suspicion for a urinoma or pancreatic injury. In this
poses the stoma to the evelopment of ischemia, an, if case, correctly interpreting the labs is necessary to etermine
this occurs, increases the risk of the evelopment of a nec- the etiology of the ascites. The serum ascites albumin grai-
rotizing soft-tissue infection. It also makes the subsequent ent (SAAG) is calculate with the equation: SAAG = serum
colostomy takeown more challenging. As such, strong albumin—ascites albumin. A SAAG <1.1 occurs with coni-
consieration shoul be given in patients with obesity to a tions where the oncotic pressure of the ascites is elevate,
primary reanastomosis. which raws ui into the peritoneal space. A SAAG>1.1
References: Demetriaes D, Murray JA, Chan LS, et al. Han- is usually associate with high hyrostatic pressure push-
sewn versus staple anastomosis in penetrating colon injuries ing ui into the peritoneal space, but can also be associ-
requiring resection: a multicenter stuy. J Trauma. 00;5(1):117–11. ate with urine leak (urine shoul not contain albumin). In
Naumann DN, Bhangu A, Kelly M, Bowley DM. Staple versus this case, the SAAG is >1.1 (3.0–1.5 = 1.5). A pancreatic leak
hansewn intestinal anastomosis in emergency laparotomy: a sys- woul have a SAAG <1.1 an an amylase 3× the serum amy-
temic review an meta-analysis. Surgery. 015;157(4):609–618. lase (D). Primary bacterial peritonitis is associate with a
SAAG >1.1, however, ui neutrophil count is ≥50 in bacte-
48. C. An occult pneumothorax is ene as one that is not rial peritonitis (E). Urine leak woul have a SAAG >1.1, but
etecte on a chest x-ray but is foun on CT scan. It is rea- ui creatinine woul be much higher than serum creatinine
sonable to closely observe patients with occult pneumotho- (B). Patients with abominal compartment synrome have
races if they are not showing signs of respiratory istress or abominal istention, oliguria, ecrease lung compliance,
hemoynamic instability. These patients shoul be manage an hypotension. This patient with normal vital signs oes
with serial chest x-rays (D). Without oxygen supplementa- not have abominal compartment synrome (C).
tion, a pneumothorax will resolve at a rate of 1% per ay. Reference: Runyon BA, Montano AA, Akriviais EA, et al. The
Some stuies suggest that oxygen therapy may accelerate serum-ascites albumin graient is superior to the exuate-transu-
resolution, but whether this helps with an occult pneumotho- ate concept in the ifferential iagnosis of ascites. Ann Intern Med.
rax is ebatable. Although there is concern about an occult 199;117(3):15–0.
Vascular—Arterial
AMANDA C. PURDY AND NINA M. BOWENS 17
ABSITE 99th Percentile High-Yields
I. Peripheral Arterial Disease (PAD)
A. Normal ankle-brachial inex (ABI) ranges from 1 to 1.; ABI <0.9 suggests PAD; ABI ≤0.4 consistent with
rest pain; iabetes an renal isease cause calcication of tibial vessels falsely elevating ABI
B. Initial management of clauication is meical: walking program (most effective), smoking cessation,
aspirin, statin
C. Inications for surgical intervention: rest pain, tissue loss (non healing ulcer or gangrene), life-limiting
clauication refractory to meical management (relative)
D. Surgical options inclue open bypass an enovascular angioplasty +/− stent
E. Consierations for lower extremity bypass:
1. Nee an aequate inow artery, conuit, an istal target
. Best choice for conuit is ipsilateral autologous vein (greater saphenous) ≥3 mm iameter an no
history of thrombosis
3. Prosthetic grafts have acceptable patency rates only if istal target is above the knee
F. Bypass graft failure:
1. Early (<30 ays), ue to a technical error
. Intermeiate (30 ays to  years), ue to intimal hyperplasia
3. Late (> years), ue to native isease (atherosclerosis)
G. Monitor grafts with uplex scans (every 3 months initially, then annually) to etect stenoses

II. Acute Limb Ischemia (ue to embolism, thrombosis, issection, or trauma)


A. Nerve ies rst (within 4 hours), then muscle (within 6 hours)
B. Presents with the 6 Ps: pain, pulselessness, pallor, poikilothermia, paresthesia, paralysis (paralysis occurs
late an is inicative of poor prognosis)
C. Initial management: IV uis, IV heparin, place limb in epenent position
D. Rutherfor classication helps guie management; class 1 can sometimes be manage with heparin
alone; class  a can attempt lytic therapy rst; class b requires emergent surgical intervention; class 3
will not benet from revascularization (nees amputation)
E. Also perform fasciotomy if suspect limb ischemia for ≥6 hours
F. Rutherfor Classication of Acute Limb Ischemia

245
246 PArt i Patient Care

Findings Doppler
Category Prognosis Sensory loss Muscle weakness Arterial Venous
1 Not immediately None None Audible Audible
Viable threatened
2a Salvageable if None-Minimal None Inaudible Audible
Marginally promptly treated (only in toes)
Threatened
2b Salvageable with Yes Mild, Moderate Inaudible Audible
Immediately immediate (more in toes)
Threatened revascularization
3 Major tissue loss or Profound, Profound, complete Inaudible Inaudible
Irreversible permanent nerve complete paralysis
damage inevitable anesthesia

III. Caroti Stenosis (causes strokes or transient ischemic attacks from embolization of plaque)
A. Symptomatic 70% to 99% caroti stenosis benets the most from surgical intervention
B. Caroti enarterectomy (CEA) is the preferre surgical option, but caroti artery stenting (CES)
preferre in symptomatic patient with:
1. Severe cariac or lung isease
. Hostile neck: previous lateral neck issection or raiation, contralateral vocal cor paralysis
3. Extremely proximal or istal plaque (ifcult to access with open surgery)
C. If bilateral caroti arteries require surgical intervention, operate on symptomatic sie rst
D. Nerve injuries after CEA: hypoglossal (can be uner facial vein an tethere by artery to
sternocleiomastoi, most common) (→tongue eviates towars the sie of surgery), vagus [recurrent
laryngeal nerve) (→hoarseness), marginal manibular branch of the facial nerve (→ipsilateral mouth
roop, traction injury), glossopharyngeal (→ifculty swallowing, rare, mostly with high issections)
E. Management of Caroti Stenosis

Symptoms Degree of stenosis Management


Symptomatic 70%–99% CEA
50%–69% CEA (greater beneęt in men)
<50% Medical management
Asymptomatic 80%–99% CEA
60%–79% Controversial, consider CEA (greater beneęt in men)
<60% Medical management

IV. Abominal Aortic Aneurysm (AAA)


A. Smoking has the greatest impact on evelopment an growth of AAA; egeneration of the walls in
AAA is ue to increase matrix metalloproteinases an ecrease elastic & smooth muscle bers; more
common in white than black men
B. Aneurysm: a focal ilation of all arterial wall layers to >1.5× the iameter of the normal artery
C. Men age 65 to 75 who have ever smoke shoul have one-time ultrasoun screening for AAA
D. Inications to repair AAA: symptomatic, iameter of >5 cm in females, iameter of >5.5 cm in males,
growth of >0.5 cm in 6 months or >1 cm in 1 year
CHAPtEr 17 Vascular—Arterial 247

E. Surgical options inclue open AAA repair an enovascular repair (EVAR)
F. Compare to open AAA repair, EVAR has a lower perioperative (30-ay) mortality rate but similar long-
term mortality
G. Complications of AAA repair:
1. MI is the most common cause of in-hospital eath after AAA repair
. Kiney injury: increase incience if intraoperative hypotension or suprarenal aortic cross-clamp
3. Ischemic colitis: risk factors (coverage of the IMA or internal iliac, rupture AAA, intraoperative
hypotension; can present with abominal pain an iarrhea (sometimes blooy); iagnose with exible
sigmoioscopy; treat with antibiotics, NPO, an resuscitation, if patient eteriorates or evelops
peritonitis will require surgery
4. Aortoenteric stula (after open or enovascular repair): upper GI blee usually >6 months after
surgery; ue to infecte graft that eroes into the uoenum; upper enoscopy (rst step in workup)
usually negative, CT shows ui/air aroun aortic graft/sac; management is graft excision, close
the uoenum, an revascularization either with in situ human aortic homograft, neoaortoiliac
proceure (NAIS) or extra anatomic axillobifemoral bypass

V. Mesenteric Ischemia
A. Acute mesenteric ischemia (AMI) etiologies: embolism (to SMA, most common), thrombosis, low ow
state (non occlusive), venous thrombosis
B. AMI presentation: writhing aroun complaining of severe abominal pain, but not signicantly tener on
exam (pain out of proportion to physical exam); x with CTA an have preop iscussion regaring bowel
viability, quality of life, an possible bowel resection
C. Surgery for AMI ue to embolism: resect frankly necrotic bowel, open SMA embolectomy
1. Transverse arteriotomy proximal to the mile colic artery, then embolectomy
. If any bowel with questionable viability: o not resect, leave abomen open, plan for n look within
4 to 48 hours
D. Chronic mesenteric ischemia (CMI) from atherosclerosis of the celiac, superior mesenteric, an/or
inferior mesenteric arteries.
E. CMI presentation: severe abominal pain about 30 to 60 minutes after eating, often leaing to “foo fear”
(intestinal angina) an weight loss
F. First-line treatment for CMI is angioplasty an stenting; if bypass is require in high-risk or sick patient,
perform retrograe mesenteric bypass from common iliac artery or infrarenal aorta to avoi supraceliac
clamping
G. Aggressive ui resuscitation shoul be use with caution in non occlusive mesenteric ischemia in the
setting of ecompensate congestive heart failure, use irect intraarterial papaverine instea
H. Venous thrombosis usually treate with anticoagulation (unless peritonitis); nee hypercoagulable
workup
248 PArt i Patient Care

Fig. 17.1 Types of Endoleaks


CHAPtEr 17 Vascular—Arterial 249

QUESTIONS
1. A 5-year-ol woman presents to the trauma bay 5. A 60-year-ol woman presents with suen onset
after a motor vehicle accient. A pan-CT scan is of acute abominal pain. On examination, the
negative for any acute injuries but oes show an patient is writhing because of severe pain, yet the
inciental focal ilation of the mi-splenic artery abomen is only milly tener, without guaring
to a iameter of .5 cm. The best management of or reboun. The cariac examination reveals an
this ning is: irregularly irregular rhythm. She enies a history
A. No further management is necessary of abominal pain. The serum lactate level is
B. Elective splenectomy elevate. Serum amylase is slightly elevate.
C. Elective coil embolization Plain abominal raiographs are negative. A
D. Open repair with vein interposition graft compute tomography (CT) scan of the abomen
E. Surveillance with repeat imaging in 6 months reveals iffuse eema of the small bowel wall.
The next step in the management woul be:
2. A 40-year-ol woman with refractory A. Thrombolytic therapy
hypertension unergoes further workup. Her B. Arteriography
plasma an urine metanephrines are normal, C. Intravenous heparin
alosterone to renin ratio is <0, cortisol is D. Exploratory laparotomy
normal, an creatinine is 1.. CT of the abomen E. Duplex ultrasoun scan
shows no arenal lesions, an renal arteries have
a “string-of-beas” appearance. How shoul this 6. A 45-year-ol man presents with a -week history
be manage? of vague, iffuse abominal pain an istention.
A. Aspirin an statin He reports that his mother an granmother both
B. Corticosterois ha leg bloo clots. On examination, he has mil
C. Percutaneous angioplasty iffuse tenerness without guaring or reboun.
D. Percutaneous angioplasty an stent A CT scan reveals thickene loops of small
E. Open bypass bowel an failure of opacication of the superior
mesenteric vein. The best management approach
3. Occlusion of a reverse saphenous vein femoral- woul consist of:
to-popliteal artery bypass 3 weeks after surgery is A. Catheter-irecte thrombolytic therapy
most often ue to: B. Intravenous (IV) heparin followe by 3 months
A. Myointimal hyperplasia of rivaroxaban
B. Progressive atherosclerosis C. IV heparin followe by lifelong apixaban
C. Hypercoagulable state D. Arteriography with papaverine infusion
D. Technical error E. Immeiate operative exploration
E. Persistent valve
7. Clauication symptoms are most improve with
4. A 65-year-ol man presents with a 4-hour history the use of:
of suen onset of left leg pain. He has no pulses A. Pentoxifylline
in his left femoral artery or istally. The calf is B. Aspirin
tener to palpation. The foot is cool an pale with C. Cilostazol
iminishe capillary rell. He has iminishe D. Clopiogrel
extension of his left great toe as well as a sensory E. Coumain (warfarin)
loss of his toes. On the unaffecte sie, the
femoral, popliteal, an istal pulses are normal. 8. Four ays after a left femoral-to-popliteal arterial
ECG shows an irregularly irregular rhythm. bypass with ipsilateral reverse saphenous vein,
After aministration of heparin, the next step in the patient reports swelling in the left leg. This
management woul be: most likely inicates:
A. Diagnostic arteriography A. Deep venous thrombosis
B. Thrombolytic therapy B. Reperfusion eema
C. Transfemoral embolectomy C. Decrease venous return from saphenous vein
D. Echocariogram harvest
E. Below-knee popliteal embolectomy D. Cellulitis
E. Lymphatic isruption
250 PArt i Patient Care

9. A 65-year-ol man with a history of a coronary 13. Accoring to the Asymptomatic Caroti
artery bypass graft  years earlier presents with Atherosclerosis Stuy (ACAS), which of the
recurrent chest pain. He escribes the pain as following is true regaring CEA for asymptomatic
substernal an raiating to his jaw. He works as a internal caroti artery (ICA) stenosis?
carpenter an also states that his left arm tires out A. CEA reuces the 5-year risk of stroke an
easily with use. Bloo pressure in the right arm is eath from 0% to 10% in patients with high-
150/90 mmHg an 100/60 mmHg in the left arm. grae stenosis
Relief of his chest pain is likely best achieve with: B. It is benecial, provie the perioperative
A. Reo coronary artery bypass graft stroke an eath rates are 9% or less
B. Coronary stenting C. The ACAS trial use both aspirin an a
C. Increasing the ose of nitrates lipi-lowering agent in the meical arm of
D. Subclavian artery stenting the trial
E. Increasing beta-blocker ose D. It is inicate for patients with ICA stenosis
ranging from 50% to 100%
10. Which of the following is most appropriate in the E. There is less benet in women
surgical management of bowel ischemia ue to an
embolus to the superior mesenteric artery (SMA)? 14. Which of the following woul provie the
A. Intraoperative angiography greatest benet from CEA?
B. Planne secon-look laparotomy
C. Dopamine Symptom Percentage ICA
D. Longituinal arteriotomy of SMA stenosis
E. Resection of bowel with questionable viability A. Asymptomatic Right 90%
11. At surgery for suspecte acute mesenteric B. Right eye amaurosis fugax Left 60%
ischemia, almost the entire small bowel as well C. Right arm/leg transient Right 80%
as the right colon appears ischemic. However, ischemia aĴack
the proximal jejunum, uoenum, an left colon D. Left eye amaurosis fugax Left 80%
appear healthy. The most likely etiology of these E. Right arm/leg paresis Left 45%
nings is:
A. Thrombosis of the SMA 15. Thirty minutes after arriving in the recovery
B. Embolus to the SMA room after a right CEA, the patient evelops left
C. Superior mesenteric vein thrombosis hemiparesis. The most appropriate next step
D. Portal vein thrombosis woul be:
E. Nonocclusive mesenteric ischemia A. Immeiate operative reexploration of the
caroti artery
12. Which of the following is true regaring
B. Tissue plasminogen activator (tPA) infusion
the timing an/or inications for caroti
C. Cerebral angiography
enarterectomy (CEA) in a patient with a stroke?
D. Caroti uplex ultrasoun scan
A. CEA is inicate even if a patient has complete E. Hea compute tomography (CT)
hemiplegia
B. CEA is best performe within 6 to 8 weeks of 16. Following a right CEA, a 65-year-ol male
the stroke evelops a severe 10/10 right frontal heaache
C. CEA is best performe 3 months after the stroke followe by a seizure. There are no focal
D. CEA shoul be performe urgently neurologic ecits. Which of the following is true
E. CEA is best performe within  weeks of the regaring this conition?
stroke A. It typically presents within 4 hours of surgery
B. It is usually self-limite
C. Postoperative hypertension is a risk factor
D. Vasoilators are useful in the treatment
E. The patient will likely nee a return to the
operating room
CHAPtEr 17 Vascular—Arterial 251

17. A 5-year-ol woman presents with several 20. Four months after CEA, a uplex ultrasoun scan
episoes of izziness, syncope, upper extremity reveals recurrent 70% ICA stenosis. The patient
clauication, an an elevate erythrocyte reports no symptoms. Optimal management
seimentation rate. On examination, she has woul consist of:
no raial, brachial, or caroti pulses. Her bloo A. Repeat CEA
pressure is 70/50 mmHg in her right arm an B. Caroti stenting
60/40 mmHg in her left. Magnetic resonance C. Observation
angiography reveals occlusion of both subclavian D. Interposition saphenous vein bypass
arteries as well as high-grae stenosis of both E. Interposition polytetrauoroethylene bypass
common caroti arteries at their mi portion. Which
of the following is true about this conition? 21. A 35-year-ol woman presents to the emergency
A. Methotrexate is not helpful epartment with right-sie heaache, right
B. Transluminal angioplasty is the treatment of eye ptosis, an suen onset of left arm an leg
choice weakness that lasts 1 hour an then resolves
C. Surgery shoul be performe urgently spontaneously. There is no history of trauma.
D. The isease can involve the pulmonary an Duplex ultrasoun scan of the right caroti artery
coronary arteries reveals a complete occlusion of the ICA. CT
E. Antihypertensive agents are contrainicate angiography conrms a tapering of the ICA with
occlusion approximately  to 3 cm istal to the
18. A 40-year-ol woman presents to the emergency bifurcation. Management consists of:
epartment after a motor vehicle accient with A. CEA
a manible fracture. She is neurologically intact. B. Lytic therapy with tissue plasminogen
She is otherwise hemoynamically stable, activator
alert, an oriente. A CT scan of the hea an C. Caroti stenting
neck is negative for an intracranial blee but D. Anticoagulation
emonstrates an intimal injury of the right E. Fogarty embolectomy
internal caroti artery. She is hemoynamically
stable an will not require operative intervention 22. Thromboangiitis obliterans (Buerger isease) is
for the manible fracture. Which of the following characterize by:
is true about this injury? A. Frequent coronary artery involvement
A. Aspirin is the treatment of choice B. Frequent involvement of aortoiliac arterial
B. Associate Horner synrome is extremely rare segments
C. The injure caroti artery shoul be stente C. Disease limite to peal arteries
D. Complete healing of the caroti artery is rare D. Successful treatment with saphenous vein
E. Urgent surgical intervention is inicate bypass
E. Corkscrew collaterals
19. A 60-year-ol man presents with a right arm an
leg hemiparesis that has persiste for 1 hour. 23. Which of the following is true regaring
He has a history of a left moie raical neck noninvasive hemoynamic assessment?
issection an neck irraiation for cancer 10 A. In normal resting subjects in the supine
years previously. CT angiography reveals a 75% position, the ankle pressure can be lower than
stenosis of the left internal caroti artery just that of the arm
istal to the bifurcation. Which of the following is B. There is poor correlation between ankle-
recommene as the enitive management? brachial inex (ABI) an severity of
A. Aspirin symptoms
B. Aspirin an clopiogrel C. En-stage renal failure can cause a false
C. Caroti enarterectomy elevation of the ABI
D. Resection of the isease caroti artery with D. In iabetic patients, toe pressures are usually
an interposition graft falsely elevate
E. Caroti stenting with a cerebral protection evice E. In iabetic patients, transcutaneous oximetry
is unreliable
252 PArt i Patient Care

24. Which of the following is true regaring the 27. Which of the following is true regaring femoral
use of thrombolytic therapy for arterial limb pseuoaneurysms that occur after arteriography?
ischemia? A. Ultrasoun compression is the proceure of
A. It can safely be use in patients within a week choice
of cataract surgery B. Ultrasoun compression is usually successful
B. Bleeing risk correlates with brinogen levels even if the patient is receiving anticoagulation
C. It is useful in patients with a profoun motor therapy
ecit in the ischemic limb C. Surgical repair typically requires interposition
D. It is highly effective regarless of the length of vein grafting
uration of symptoms D. It can be manage with ultrasoun-guie
E. It can safely be use for as long as 7 hours irect thrombin injection
E. A trial of observation is contrainicate
25. A relatively healthy 60-year-ol iabetic male because of the high risk of bleeing
patient presents with gangrene of his right great
toe. The patient has normal femoral an popliteal 28. One ay after open abominal aortic
pulses but no istal pulses. His ABI is 0.5. An aneurysm (AAA) repair, watery iarrhea an
angiography reveals patent iliac, femoral, an abominal istention evelop in the patient.
popliteal arteries with a long-segment occlusion On examination, the patient has mil lower left
of the trifurcation vessels with reconstitution of quarant tenerness without guaring. WBC
the anterior tibial artery just above the ankle an count is 14,000 cells/µL. Which of the following is
runoff into the orsalis peis artery. Bilateral appropriate for this patient?
saphenous veins are 4 mm in iameter on A. Proctosigmoioscopy
ultrasoun. Which of the following is the best B. CT angiography
option? C. Exploratory laparotomy
A. Common femoral-to-anterior tibial bypass D. Diagnostic laparoscopy
with ipsilateral saphenous vein E. Transfemoral arteriography
B. Common femoral-to-anterior tibial bypass
with contralateral saphenous vein 29. A 69-year-ol man presents to the emergency
C. Popliteal-to-anterior tibial bypass with epartment (ED) with suen onset of left ank
ipsilateral greater saphenous vein an back pain, abominal tenerness, a bloo
D. Enovascular stenting of anterior tibial artery pressure of 100/60 mmHg, heart rate of 100, an
E. Great toe amputation only a tener pulsatile miline abominal mass. He
is awake an alert. Which of the following is
26. A 3-year-ol woman notes that her hans recommene next?
become col an painful when expose to col A. A -liter bolus of normal saline
temperatures. The han changes in color from B. A CT scan of the abomen an pelvis
pale to cyanotic to re. Her meical history is C. Besie ultrasoun
negative, an vascular pulse examination is D. Immeiate transport to operating room
normal. Arterial noninvasive stuies reveal a E. Enotracheal intubation in the ED
marke ecrease in igital bloo pressure with
exposure to col temperatures. Symptoms persist 30. Which of the following is true regaring popliteal
espite wearing gloves an avoiance of col artery aneurysms?
exposure. The next step in management is: A. Observation is recommene for an
A. Upper extremity sympathectomy asymptomatic 3-cm popliteal aneurysm
B. Prostaglanins B. An asymptomatic aneurysm with intraluminal
C. Fluoxetine thrombus shoul be repaire only when it is
D. Arteriography larger than  cm in size
E. Diltiazem C. Bypassing the aneurysm with saphenous vein
with interval ligation is the stanar operative
approach
D. An enovascular stent graft is not
recommene for popliteal aneurysms
E. A posterior approach to the aneurysm is not
technically feasible
CHAPtEr 17 Vascular—Arterial 253

31. One year after open AAA repair, a patient 35. The most common enoleak after an EVAR is
presents to the emergency epartment vomiting type:
bloo. Vital signs are stable. CT scan shows A. I
some staning an a small pocket of air aroun B. II
the aortic graft. Which of the following is true C. III
regaring this conition? D. IV
A. Inammatory changes aroun the graft are E. V
common 1 year after surgery
B. Arteriography is useful in establishing the 36. A 61-year-ol male with en-stage renal isease
iagnosis (ESRD) presents with a col, painful right leg
C. A tagge nuclear white bloo cell scan is of -hour uration. He has an irregular heart
unlikely to ai in the iagnosis rate on exam. CT angiography conrms an
D. Upper enoscopy will have a high sensitivity occlusion of the common femoral artery. He
for establishing the iagnosis is appropriately treate with a heparin rip
E. In situ placement of an aortic homograft will an surgical embolectomy with symptom
likely be neee resolution. Symptoms recur 4 ays later, an the
pulses isappear. He is taken back for a repeat
32. The threshol for elective repair of an embolectomy, at which time a whitish-appearing
asymptomatic common iliac aneurysm is greater clot is remove. Which of the following is true
than: regaring this conition?
A. .0 cm A. He shoul receive bivaliruin
B. .5 cm B. tPA woul have been a goo alternative to
C. 3.5 cm reexploration
D. 4.0 cm C. He likely has antithrombin-III eciency
E. 4.5 cm D. He shoul receive lepiruin
E. He shoul receive argatroban
33. The most common symptom of a popliteal
aneurysm is:
A. Rupture
B. Thrombosis
C. Distal embolization
D. Ajacent nerve compression
E. Ajacent venous compression

34. Which of the following is true regaring AAA


repair?
A. The appropriate AAA iameter threshol for
elective repair for men an women is the same
B. In low operative risk patients, the AAA
iameter threshol for enovascular aneurysm
repair (EVAR) is lower than that for open
repair
C. Women have higher perioperative mortality
rates for both EVAR an open AAA repair as
compare with men
D. Careful surveillance of AAA up to 6.0 cm is
safe
E. In a high–cariac risk patient with a 5.0-cm
AAA, the EVAR approach shoul be use
rather than elaying surgery
254 PArt i Patient Care

ANSWERS
1. C. This patient was incientally foun to have a splenic bypass, are generally cause by myointimal hyperplasia (A).
artery aneurysm, the most common visceral artery aneu- Late graft failures (beyon  years) are cause by progression
rysm. The major concern is rupture. Patients with rupture of atherosclerotic occlusive isease, either within the inow
splenic artery aneurysms classically present with the “ou- or outow vessels (B). A persistent valve woul be a poten-
ble rupture phenomenon,” where they experience acute tial problem with an in situ vein bypass (not with a reverse
abominal pain at rst without hypotension while bloo vein), in which case valves are intentionally cut with a val-
pools in the lesser sac. Then, once the lesser sac ruptures vulotome (E). Young patients may have a more aggressive
through the foramen of Winslow freely into the peritoneal form of atherosclerotic isease (virulent isease), an some
cavity, the patient evelops istention an hemorrhagic have postulate that this may be seconary to an unerlying
shock. The most common management of splenic artery hypercoagulable state (C).
aneurysms is coil embolization. The inications for interven- Reference: McCreay RA, Vincent AE, Schwartz RW, Hye GL,
tion inclue rupture, symptoms, iameter >3 cm in patients Mattingly SS, Griffen WO Jr. Atherosclerosis in the young: a virulent
with low surgical risk, an any size in women of chilbear- isease. Surgery. 1984;96(5):863–869.
ing age. Coil embolization is appropriate for proximal an
4. C. In an acutely ischemic limb, in aition to the neu-
mi-portion aneurysms as the spleen continues to be per-
rovascular exam of the ischemic limb, the most important
fuse by the short-gastric arteries avoiing splenic infarc-
aspects of the physical examination are the cariac exam an
tion. For istal-thir aneurysms, resection with splenectomy
the neurovascular examination of the nonischemic limb. If
is usually performe (B). Open repair with vein interposition
the nonischemic limb has normal pulses an no other evi-
graft has largely fallen out of favor (D). Because this patient
ence of chronic ischemia (e.g., hair loss, thin ry skin), then
is a woman of chilbearing age, it woul be inappropriate to
the ischemia is most likely embolic in nature. Fining an
procee with conservative management an surveillance (A,
irregularly irregular rhythm woul further conrm that the
E). Patients foun to have a splenic artery aneurysm with-
heart is the most likely source of the clot ue to atrial bril-
out inication for repair shoul be followe with annual
lation. With an absent femoral pulse, the embolus has likely
CT angiogram or ultrasoun. Women are at highest risk for
loge in the common femoral artery. Because the patient
splenic artery aneurysm rupture uring the 3r trimester of
escribe has class b ischemia (immeiately threatene),
pregnancy.
heparin shoul be starte, an revascularization shoul
Reference: Chaer RA, Abularrage CJ, Coleman DM, et al. The
be performe without elay (E). In class 1 ischemia (not
Society for Vascular Surgery clinical practice guielines on the man-
agement of visceral aneurysms. J Vasc Surg. 00;7(1 S):3 S–39 S.
threatene; no sensory or motor loss), there is no immei-
ate urgency to going to the operating room. Heparin shoul
2. C. This patient has bromuscular ysplasia (FMD). Renal be starte. It is then useful to obtain imaging to conrm the
artery stenosis is a cause of seconary hypertension an can iagnosis. This can be achieve via an arterial uplex scan or
be ue to atherosclerosis or FMD. FMD is an iiopathic is- CT angiogram, which has replace iagnostic arteriography
ease of the musculature of the arterial walls leaing to steno- as the gol stanar (A). An avantage of CT over angio-
sis of small an meium-size arteries an is most common gram is that it may etect etiologies of acute ischemia that
in women from 30 to 60 years ol. The most commonly woul otherwise be unsuspecte, such as an aortic issec-
involve arteries are the renal, caroti, an vertebral arteries. tion or aneurysm, an one can image the chest an abomen
The “string-of-beas” appearance is a classic imaging n- for possible pathology. Following iagnosis, if the patient is
ing seen in FMD. FMD is noninammatory an there is no not immeiately threatene, they may unergo enitive
role for sterois (B). Patients with renal artery stenosis ue treatment via thrombolytic therapy or open embolectomy
to atherosclerosis shoul receive aspirin an statin; most are (B, C). Native arterial occlusions ue to cariac embolization
manage meically. A renal stent may be consiere in the ten to respon less favorably to thrombolytic therapy. Thus,
case of refractory hypertension or ash pulmonary eema open embolectomy is preferre by some. For the patient in
(A, D). On the other han, patients with renal artery stenosis the vignette, a transfemoral approach is optimal because
ue to FMD are most appropriately treate with percutane- it can be one with the patient uner local anesthesia an
ous angioplasty, as stents have a high rate of fracture when allows selective embolectomy own the supercial femoral
use for FMD renal isease. Open bypass is more invasive an profuna femoral arteries. The below-knee popliteal
an has similar success as angioplasty (E). artery approach to embolectomy is reserve for situations in
Reference: Gornik HL, Persu A, Alam D, et al. First interna- which the patient has normal femoral an popliteal pulses
tional consensus on the iagnosis an management of bromuscular an the embolus is loge in the tibial vessels (E). However,
ysplasia. J Hypertens. 019;37():9–5. such an approach is technically more ifcult. If the limb
is not immeiately threatene, istal clots are better man-
3. D. Early failure (within 30 ays) after surgery generally age by lytic therapy as the tPA can be irecte via cathe-
inicates a technical error. Technical errors inclue anasto- ter irectly into the involve vessel. Echocariogram woul
motic stenosis, a kink or twist within the graft, poor choice eventually be useful to look for a cariac source of thrombus,
of proximal or istal target, an inaequate-caliber saphe- but it woul not be of immeiate help in the management
nous vein. Intermeiate failures, from 30 ays to  years after (D). With the avent of hybri operating rooms, patients
CHAPtEr 17 Vascular—Arterial 255

with more avance ischemia (class ) can be taken irectly ischemia, correcting the unerlying shock is the initial man-
to the operating room where a iagnostic angiography fol- agement. Catheter irecte papaverine may also be useful.
lowe by immeiate intervention can be achieve. There are some case reports in which mesenteric emboli have
Reference: Results of a prospective ranomize trial evaluating been successfully manage with lytic therapy, but this is not
surgery versus thrombolysis for ischemia of the lower extremity. the stanar approach an is not the best option for elevate
The STILE trial: The STILE investigators (appenix A). Ann Surg. lactate suggesting a compromise bowel (A).
1994;0(3):51–68.
6. C. Mesenteric venous thrombosis accounts for approxi-
5. C. This patient’s history an CT scan nings are most mately 10% to 15% of cases of mesenteric ischemia. It tens
consistent with acute mesenteric ischemia. Acute mesenteric to have a slow, insiious onset, as in this case. Risk factors for
ischemia can be ivie into four major causes. Embolization mesenteric venous occlusion inclue hypercoagulable states
from a cariac source is the most common cause (30%–50% such as factor V Leien, antithrombin III eciency, an pro-
of cases), is seen most often in the setting of atrial brillation tein C an S eciency, as well as liver isease with portal
an is the likely etiology in the patient presente. The n- hypertension, pancreatitis, an any intraperitoneal inam-
ing of an irregularly irregular heart rhythm suggests an arte- matory conitions. Venous thrombosis is less ramatic than
rial embolism from atrial brillation. The most common site arterial occlusion. Abominal pain is vague, an tenerness
of mesenteric embolization is the superior mesenteric artery is mil or equivocal. CT may emonstrate a thickene bowel
(SMA) (ue to its angle from the aorta). The embolus typi- wall with elaye passage of IV contrast agent into the por-
cally occlues the SMA just istal to the mile colic artery. tal system an a lack of opacication of the portal or superior
These patients often have sparing of the proximal jejunum mesenteric vein. If the iagnosis is establishe from the CT
an transverse colon because the mile colic artery remains scan, further iagnostic tests are unnecessary. Another use-
patent. Celiac artery embolization is rare, given its take-off at ful iagnostic moality is uplex ultrasoun scanning. Arte-
a right angle to the aorta. The inferior mesenteric artery ori- riography may emonstrate venous congestion an a lack of
ce is so small that a cariac thrombus rarely loges insie. prompt lling of the portal system (D). If the patient is man-
Mesenteric arterial thrombosis is usually ue to unerlying ifesting peritoneal signs, operative exploration is inicate
mesenteric artery atherosclerosis. In this situation, the patient (E). However, in the absence of peritonitis, therapy shoul
will typically have a long-staning history of pain after eat- consist of ui hyration, hemoynamic support, anticoag-
ing, fear of eating, an weight loss, an the physical examina- ulation with heparin, an serial examination. If peritonitis
tion will reveal evience of iffuse atherosclerosis an bruits. subsequently evelops, exploratory laparotomy is appropri-
Mesenteric venous thrombosis is a thir etiology an is most ate to assess bowel viability with segmental bowel resection.
often seen in patients with hypercoagulable states. The acute Surgical thrombectomy of the venous system is not likely to
venous occlusion leas to massive bowel eema with secon- be successful. Fibrinolytic therapy has been use increas-
ary arterial insufciency from bowel wall istention. Patients ingly, but is not yet the stanar treatment of choice, an
with mesenteric venous thrombosis ten to present in a less is ieal when symptoms are of short uration (A). Follow-
ramatic fashion, often with ays or weeks of abominal ing heparin, warfarin or a novel oral anticoagulant, such as
pain. Finally, nonocclusive mesenteric ischemia results from apixaban or rivaroxaban, is recommene for 3 to 6 months
shock that creates hypoperfusion of the bowel, such as with if the hypercoagulable state is provoke or temporary (B).
cariac failure or severe hypovolemia. The classic nings Lifelong warfarin or NOAC is recommene if the venous
in acute mesenteric ischemia are the suen onset of severe thrombosis is unprovoke or associate with a permanent
pain out of proportion to the physical examination nings. thrombophilic state. The family history of venous thrombo-
Elevate serum lactate levels shoul raise the suspicion of sis in this patient is highly suggestive of an inherite hyper-
ischemic bowel, but they are not sensitive enough to etect coagulability an woul warrant lifelong anticoagulation.
early bowel ischemia. A plain abominal raiograph is often Aitionally, any mesenteric arterial embolism requires life-
unremarkable, although it may emonstrate evience of long anticoagulation.
eema in the small bowel wall. If the patient has peritoneal Reference: Kumar S, Sarr MG, Kamath PS. Mesenteric venous
signs on abominal examination, this will inicate that the thrombosis. N Engl J Med. 001;345(3):1683–1688.
bowel has alreay been infarcte. In the absence of perito-
nitis an because the ifferential iagnosis is extensive, CT 7. C. Cilostazol has a number of functions incluing inhib-
provies the greatest iagnostic yiel initially (E). However, iting platelet aggregation an smooth muscle proliferation,
CT scan may not be iagnostic because it may not necessarily increasing vasoilation, an lowering high-ensity lipo-
emonstrate opacication in the mesenteric veins or arteries protein an triglycerie levels. Cilostazol has been shown
(epening on the timing of contrast). The rst step in the to signicantly increase walking istance by 50% to 67% in
management is the aministration of IV heparin. Following patients with clauication in several ranomize trials an
heparin, for an embolus, immeiate surgery offers the best results in improvement in physical functioning an quality
chance of treatment an woul involve an SMA embolectomy of life. This rug is contrainicate in patients with conges-
(D). If the history were suggestive of unerlying mesenteric tive heart failure. This rug is more effective than pentoxifyl-
atherosclerosis (longstaning postpranial abominal pain line in the treatment of clauication (A). Pentoxifylline is a
an weight loss) with thrombosis, arteriography woul be methylxanthine erivative that has hemorrheologic proper-
helpful because the management woul involve an arterial ties. Two meta analyses showe that it improves walking is-
bypass or stenting (B). If the CT scan reveale a thrombus tance, but in some more recent ranomize stuies, it prove
in a mesenteric vein, enitive treatment woul be hepa- to be no better than placebo. Pentoxifylline improves symp-
rin alone, provie there is no peritonitis. For nonocclusive toms of clauication by increasing re bloo cell exibility
256 PArt i Patient Care

an reucing bloo viscosity. Antiplatelet meications 10. B. Initial management of patients with acute mesenteric
such as aspirin are use in the treatment of peripheral vas- ischemia inclues ui resuscitation an systemic anticoag-
cular isease an for cariac an stroke prevention but o ulation with heparin sulfate to prevent further thrombus
not appear to improve walking istance (B). Aspirin has propagation. Signicant metabolic aciosis shoul be cor-
been foun to reuce the vascular eath rate by approxi- recte with soium bicarbonate. A central venous cathe-
mately 5% in patients with any manifestation of athero- ter, peripheral arterial catheter, an Foley catheter shoul
sclerotic isease (e.g., coronary, peripheral). Clopiogrel is be place for ui resuscitation an hemoynamic status
effective in reucing overall acute cariovascular events, monitoring. Appropriate antibiotics are given before surgi-
especially in patients with lower extremity occlusive is- cal exploration. The operative management of acute mesen-
ease, but is much more expensive (D). It oes not seem to teric ischemia is ictate by the cause of the occlusion. For
irectly improve walking istance. Pure vasoilators have an SMA embolus, exposure of the SMA is obtaine via rota-
not been efcacious in the treatment of peripheral vascu- tion of the small bowel to the right an by sharply issecting
lar isease because most patients with such occlusive is- the ligament of Treitz. The SMA will be foun at the root of
ease alreay exhibit marke vasoilation. Anticoagulants the mesentery. The primary goal in the surgical treatment of
also have not been shown to alter the course of peripheral embolic mesenteric ischemia is to restore arterial perfusion
atherosclerosis (E). with removal of the embolus from the vessel. This is one
Reference: Money SR, Her JA, Isaacsohn JL, et al. Effect by performing a Fogarty embolectomy using a transverse
of cilostazol on walking istances in patients with intermittent arteriotomy (longituinal arteriotomy will cause stenosis
clauication cause by peripheral vascular isease. J Vasc Surg. upon closure) (D). It is important to avoi resecting bowel
1998;7():67–74. until perfusion has been restore; that way, bowel viability
can be better establishe. After restoration of SMA ow, an
8. E. Leg eema after femoral-to-popliteal arterial bypass is assessment of the intestinal viability is mae, an nonvia-
common. In most instances, it is ue to lymphatic isrup- ble bowel is resecte. Because the amount of bowel resecte
tion. This isruption occurs at both the groin an popliteal can be extensive an this places the patient at risk of short
incisions as well as from harvesting of the saphenous vein. bowel synrome, bowel that is of borerline viability shoul
Deep venous thrombosis can occur after this proceure but is be left in place with a planne secon-look proceure per-
relatively uncommon (A). Reperfusion eema may be asso- forme 4 to 48 hours later to reassess whether aitional
ciate with compartment synrome an can present with bowel resection is neee (E). Low-ose opamine leas to
the Ps (pain, pallor, paralysis, paresthesia, an poikilother- vasoilatation of mesenteric arteries; however, its benets
mia) (B). It is more likely to present after revascularization are unclear (C). Intraoperative angiography will not provie
ue to acute limb ischemia. The saphenous veins are part of any aitional information that woul assist in the surgical
the supercial venous system, which contributes a minority management of SMA embolus (A).
of the venous rainage in the leg, so swelling seconary to
venous congestion is not expecte after a saphenous vein 11. B. The most common cause of mesenteric ischemia is
harvest (C). Cellulitis woul present with erythema, pain, a cariac embolus to the SMA. The SMA provies bloo to
warmth, an possible systemic signs such as fever or leuko- the bowel from the ligament of Treitz to the mi transverse
cytosis (D). colon. Cariac embolus tens to loge just past the SMA ori-
Reference: AbuRahma AF, Wooruff BA, Lucente FC. Eema gin at a point where the artery begins to narrow, which is
after femoropopliteal bypass surgery: lymphatic an venous theo-
just beyon the rst jejunal branches. These patients often
ries of causation. J Vasc Surg. 1990;11(3):461–467.
have sparing of the proximal jejunum an transverse colon
because the mile colic artery remains patent. Thrombosis
9. D. The patient’s history an examination are most consis- of the SMA, conversely, is usually cause by unerlying ath-
tent with symptoms of coronary-subclavian steal synrome. erosclerotic isease that occurs at the SMA origin an woul
Most patients with a coronary artery bypass graft have thus not spare the proximal jejunum (A). Mesenteric venous
unergone a left internal mammary artery-to-left anterior thrombosis an nonocclusive mesenteric ischemia woul
escening graft. In the setting of subclavian artery stenosis more likely cause patchy areas of ischemia (C–E).
or occlusion proximal to the take-off of the internal mammary Reference: Elrup-Jorgensen J, Hawkins RE, Breenberg
artery, arm exercise leas to vasoilation of the arm vessels CE. Abominal vascular catastrophes. Surg Clin North Am.
an lower resistance. Bloo will travel through the path of 1997;77(6):1305–130.
least resistance an ow in a reverse fashion from the left ante-
rior escening artery into the left internal mammary artery 12. E. The timing of CEA after a stroke is controversial. A
an towar the arm, leaing to the evelopment of angina. elay in surgery increases the risk of recurrent stroke. The
The ifferential bloo pressure in the arms is the clue, as is risk is highest within the rst month. Conversely, operating
the left arm clauication. Treatment involves relieving the too early (within 4 hours) creates a potential risk of a reper-
subclavian artery obstruction. This can be one by subcla- fusion injury, particularly if a large infarction is present on
vian artery stenting but on occasion requires a caroti-to-sub- compute tomography (CT) an if hypertension cannot be
clavian artery bypass (A). Since the problem is not relate to controlle postoperatively. Intracranial bleeing is thought
unerlying cariac isease, caroti stenting, increasing beta- to occur because of altere autoregulation an hyperperfu-
blocker ose, or increasing ose of nitrates will not resolve the sion of ischemic tissue. In the North American Symptomatic
patient’s chest pain with exercise (B, C, E). Caroti Enarterectomy Trial (NASCET), however, postop-
Reference: Bryan F, Allen R, Lumsen A. Coronary subclavian erative intracranial hemorrhage occurre in only 0.% of
steal synrome: report of 5 cases. Ann Vasc Surg. 1995;9(1):115–1. patients. Until recently, CEA was routinely elaye for 4 to
CHAPtEr 17 Vascular—Arterial 257

6 weeks after a stroke. Subsequent analysis of the NASCET is completely occlue (100%) (D). There is no further ow
showe that patients with a stable, nonisabling acute in the artery, thus the embolic risk is eliminate. The benet
stroke, a normal CT scan, an a normal level of conscious- of aggressive meical management (incluing antiplatelet
ness can safely unergo CEA shortly after the iagnosis is agents) is that it can also be protective from coronary events.
mae, the symptoms have stabilize, an preoperative risk The biggest limitation of ACAS is that it i not inclue
assessment is complete. Thus, the operation is not urgent (D). the use of a statin, which, in aition to its lipi-lowering
Delaying the surgery for 6 weeks or more eliminates much response, also has pleiotropic effects such as plaque stabil-
of the benet of CEA because the risk of recurrent stroke is ity, which may prove to be a more important contributor in
greatest early on (B, C). Current treatment guielines from preventing the progression to stroke in caroti isease. The
the American Acaemy of Neurology an from the Ameri- Aggressive Meical Treatment Evaluation for Asymptomatic
can Stroke Association/American Heart Association recom- Caroti Artery Stenosis (AMTEC) trial attempte to compare
men that CEA for patients with nonisabling strokes shoul moern meical management with CEA, but the stuy was
preferably be performe within  weeks of the primary prematurely terminate an the results are not yet available.
stroke. Patients with a large stroke on CT scan or those with Newer stuies are neee to etermine if moern meical
a miline shift may be at higher risk of reperfusion injury, therapy continues to be inferior to surgical intervention in
particularly if they have a epresse level of consciousness. patients with caroti isease. Some authors have suggeste
Operation shoul be elaye until these patients improve that we shift away from using ecrease luminal caliber
an plateau in their clinical recovery, which is usually in the as our primary eterminant of choosing which asymptom-
range of 4 to 6 weeks. If the stroke is completely isabling atic patients to offer surgery. Newer methos of ientifying
(A), there remains little if any motor cortex to protect from high-risk patients such as those with plaque ulceration an
future stroke, so CEA is not inicate. Thus, patients with instability shoul be stuie to either replace or supplement
severe neurologic ecits, without meaningful recovery or existing societal guielines.
with marke alteration of consciousness, are not caniates References: Enarterectomy for asymptomatic caroti artery
for CEA because the goal of CEA is to prevent further am- stenosis. Executive Committee for the Asymptomatic Caroti Ath-
age to the ipsilateral motor cortex. erosclerosis Stuy. JAMA. 1995;73(18):141–148.
References: Henerson RD, Eliasziw M, Fox AJ, Rothwell PM, Halliay A, Mansel A, Marro J, et al. Prevention of isabling
Barnett HJ. Angiographically ene collateral circulation an risk an fatal strokes by successful caroti enarterectomy in patients
of stroke in patients with severe caroti artery stenosis. North Amer- without recent neurological symptoms: ranomise controlle trial.
ican Symptomatic Caroti Enarterectomy Trial (NASCET) Group. Lancet. 004;363(940):1491–150.
Stroke. 000;31(1):18–13. Kolos I, Loukianov M, Dupik N, Boytsov S, Deev A. Optimal
North American Symptomatic Caroti Enarterectomy Trial Col- meical treatment versus caroti enarterectomy: the rationale
laborators, Barnett HJM, Taylor DW, et al. Benecial effect of caroti an esign of the Aggressive Meical Treatment Evaluation for
enarterectomy in symptomatic patients with high-grae caroti Asymptomatic Caroti Artery Stenosis (AMTEC) stuy. Int J Stroke.
stenosis. N Engl J Med. 1991;35(7):445–453. 015;10():69–74.
Sacco RL, Aams R, Albers G, et al. Guielines for prevention Weyer GW, Davis AM. Screening for asymptomatic caroti artery
of stroke in patients with ischemic stroke or transient ischemic stenosis. JAMA. 015;313():19–193.
attack: a statement for healthcare professionals from the American
Heart Association/American Stroke Association Council on Stroke: 14. D. The rst NASCET stuy foun that CEA was of
co-sponsore by the Council on Cariovascular Raiology an benet for symptomatic severe ICA stenosis (70%–99%). A
Intervention: the American Acaemy of Neurology afrms the value symptomatic caroti artery stenosis was ene as a non-
of this guieline. Circ. 006;113(10):e409–e449. isabling stroke, a hemispheric transient ischemic attack,
or a retinal symptom (amaurosis fugax). Life-table estimates
13. E. The ACAS ranomize patients with asymptomatic of the cumulative risk of any ipsilateral stroke at  years were
caroti artery stenosis of 60% to 99% to either CEA an aspi- 6% in the aspirin group an 9% in the aspirin an CEA
rin or aspirin alone (C). The stuy was interrupte because group. In the secon NASCET stuy, there was no bene-
of a signicant benet ientie in patients unergoing t for symptomatic patients with less than 50% stenosis
CEA. A relative reuction in stroke rate by 50%, from 11% (E). For symptomatic patients with stenosis from 50% to
to 5% at 5 years, was observe in patients unergoing CEA 69%, there was a very moest benet: 5-year risk of ipsi-
(A). The Asymptomatic Caroti Surgery Trial conrme lateral stroke was 15.7% in the CEA group an .% in
the ACAS nings that in patients with 60% to 99% steno- the meical group (P = 0.04). The benet was greatest in
sis, the net 5-year risk was 6.4% for all strokes or eath in men, in those with hemispheric symptoms (as oppose to
patients unergoing CEA, versus 11.8% in those not uner- retinal ones), an with recent stroke. Women appeare to
going surgery. This was a net absolute gain of 5.4% (relative have less risk of stroke an also ha higher perioperative
risk reuction, 46%). The trial also showe that patients who mortality than men. ACAS emonstrate the benet of
unerwent CEA were much less likely to have a fatal or is- CEA compare with aspirin for asymptomatic ICA steno-
abling stroke (3.5% in the surgery group versus 6.1% in the sis of 60% to 99%. However, the benet is much less than
no-surgery group). The stuies have foun that there is less for symptomatic high-grae stenosis. Thus in this ques-
or no benet in women (E). The greatest benet was in men tion, choice A woul be benecial but of less benet than
younger than 75 years of age. CEA for asymptomatic steno- choice D (symptomatic). Choice B woul be of no benet
sis will only benet the group as a whole if the combine because the stenosis is moerate, an the symptoms are
stroke an eath rate is less than 3% (B). Keeping this com- on the wrong sie (retinal is ipsilateral). In choice C, the
bine enpoint low is epenent on both patient risk an symptoms are also on the wrong sie with respect to the
surgeon skill (C). There is no benet to CEA once the ICA stenosis.
258 PArt i Patient Care

15. A. New neurologic ecits that present within the her bloo pressure aily for the rst week postoperatively.
rst 1 hours of operation are almost always the result of The heaache may be followe by focal motor seizures that
thromboembolic phenomena stemming from the CEA site. are often ifcult to control. Management consists of con-
Possibilities inclue the evelopment of thrombus on the trolling bloo pressure, ieally with a beta-blocker, with the
enarterectomize arterial surface, a resiual intimal ap in avoiance of vasoilators (as these may increase cerebral
the ICA leaing to occlusion, or a resiual ap in the exter- bloo ow), an use of antiseizure meications (D).
nal caroti artery (ECA) leaing to ECA thrombosis an Reference: Schroeer T, Sillesen H, Sørensen O, Engell HC. Cere-
retrograe embolization of the clot into the ICA. Immeiate bral hyperperfusion following caroti enarterectomy. J Neurosurg.
heparinization an exploration are inicate without the 1987;66(6):84–89.
nee for conrmatory arteriography or noninvasive tests.
On reexploring the woun, the ECA an ICA shoul be pal- 17. D. This patient has Takayasu arteritis, an inammatory
pate for the presence of a pulse. If there is no pulse, this isease of the aorta an its branches, as well as the coronary
inicates thrombosis, an initial on-table arteriography is an pulmonary arteries (A–C, E). It occurs most commonly
not necessary. The artery shoul be reopene an inspecte in young women, with a meian age of 5 years. The clinical
to look for a cause of the thrombosis. Before closing the arte- course has been escribe as beginning with constitutional
riotomy, care shoul be taken to ensure that there is goo symptoms such as fever an malaise. However, a National
back-bleeing from the ICA. Fogarty balloon embolectomy Institutes of Health stuy showe that only one-thir of
of the cephala ICA shoul be avoie because this can lea patients recall such symptoms. Characteristic clinical fea-
to a caroti-cavernous sinus stula. The arteriotomy shoul tures inclue hypertension, retinopathy, aortic regurgitation,
then be reclose with a patch. On-table arteriography shoul cerebrovascular symptoms, angina, congestive heart failure,
then be performe to ensure that the istal ICA is patent abominal pain or gastrointestinal bleeing, pulmonary
an to etermine whether there is an embolus in the mi- hypertension, an extremity clauication. The gol stanar
le cerebral artery. If an embolus is present in the intracra- for iagnosis is arterial imaging, with the emonstration of
nial caroti or mile cerebral artery, local infusion of a lytic occlusive isease in the subclavian arteries. Unlike athero-
agent shoul be consiere (B). If on reopening the woun, sclerosis, which tens to affect the origin of these vessels,
an excellent pulse is present in the ICA an ECA, with nor- Takayasu arteritis affects the miportions of these arteries.
mal signals on han-hel Doppler ultrasonography, on-table Characteristic signs an symptoms inclue pulselessness
arteriography is performe (C, D). If arteriography reveals or bloo pressure ifferential in the arms, upper or lower
an intimal ap or irregular mural thrombus at the enarter- extremity clauication, syncope, amaurosis fugax, blurre
ectomy site, then reopening of the vessel is inicate. Neuro- vision, an palpitations. Treatment initially consists of ste-
logic ecits that evelop 1 to 4 hours after the operation roi therapy with the aition of cytotoxic agents use in
are usually ue to thromboembolic phenomena stemming patients who o not achieve remission. Carotiynia, which is
from the CEA site but may also be cause by a postoperative pain along iname arteries, is pathognomonic for Takayasu
hyperperfusion synrome. These latter conitions may be arteritis. Surgical treatment with arterial bypass is only per-
worsene by immeiate heparinization an reexploration. forme in avance states an in situations in which the
Therefore, ecits occurring 1 to 4 hours after the opera- patient oes not respon to meical therapy. It shoul ie-
tion shoul be promptly investigate with hea CT an CT ally be performe when the isease is not active. Because
arteriography (E). the isease causes transmural arterial inammation with
concentric brosis, there is no role for enarterectomy, an
16. C. The incience of hyperperfusion synrome after a angioplasty has not been met with goo results.
CEA is reportely 0.3% to 1%. It is thought to occur as a result
of impaire autoregulation of cerebral bloo ow an oes 18. A. The most common mechanisms of blunt caroti
not nee to be taken back to the OR (E). The thought is that injury inclue motor vehicle accients, st ghts, hanging,
longstaning, severe caroti stenosis leas to hypoperfusion, an intraoral trauma. However, it has also been reporte
leaing to a compensatory ilation of cerebral vessels istal with relatively minor trauma, such as after chiropractic
to the stenosis as part of the normal autoregulatory response manipulation of the neck an forceful sneezing. Bif et al.
to maintain aequate cerebral bloo ow. After CEA restores have grae blunt caroti injury as follows: grae I: luminal
normal pressure, however, autoregulation is impaire an irregularity or issection with less than 5% luminal narrow-
oes not immeiately ajust to the suen increase in bloo ing; grae II: issection or intramural hematoma with greater
ow. Risk factors associate with cerebral hyperperfusion than or equal 5% luminal narrowing; grae III: pseuoan-
inclue recent stroke, surgery for very tight ICA stenosis, eurysm; grae IV: occlusion; grae V: transection with free
concomitant contralateral ICA occlusion, evience of chronic extravasation. Horner synrome (oculosympathetic paresis)
ipsilateral hypoperfusion, stage bilateral CEA performe is common with this injury an is thought to be relate to the
within  months of each other, an poorly controlle pre- an involvement of the internal part of the pericaroti sympa-
postoperative hypertension. Pathologic changes range from thetic plexus (B). The ecision to perform surgery is base on
mil cerebral eema an petechial hemorrhage to severe (1) injury severity, () presence or absence of symptoms, an
intracerebral hemorrhage an eath, particularly if not (3) surgical accessibility of the lesion (C). In general, there
promptly treate (B). The synrome is herale by an ipsilat- is little role for surgical intervention in patients with grae
eral frontal heaache, most commonly occurring at a meian I or II blunt caroti injury as in this patient (E). Antiplatelet
of the fth postoperative ay (A). By that time, the patient is therapy with aspirin is the best treatment option. However,
alreay at home. Thus, it is imperative to warn patients of some trauma centers chose to use subtherapeutic heparin ini-
this rare synrome an ieally have the patient check his or tially in case patients may require a surgery. Minor (intimal)
CHAPtEr 17 Vascular—Arterial 259

injuries ten to heal themselves (D). Pseuoaneurysms typi- woul be high on the ifferential. The iagnosis is mae by
cally o not an are a relative inication for surgery if acces- uplex scan an/or CT angiography. Duplex scan may be
sible in the neck. iagnostic, if it emonstrates a membrane within the lumen,
References: Bif WL, Moore EE, Offner PJ, Brega KE, Franciose consistent with a issection. The most likely mechanism
RJ, Burch JM. Blunt caroti arterial injuries: implications of a new of acute issection is an intimal tear followe by an acute
graing scale. J Trauma. 1999;47(5):845–853. intimal issection, which prouces luminal occlusion ue
Bromberg WJ, Collier BC, Diebel LN, et al. Blunt cerebrovascular to seconary thrombosis. The occlusion angiographically is
injury practice management guielines: the Eastern Association for
typically  to 3 cm beyon the bifurcation. Autopsy stuies
the Surgery of Trauma. J Trauma. 010;68():471–477.
have shown a sharply emarcate transition between the
normal caroti artery an the issecte segment. Treatment
19. E. The patient has a symptomatic high-grae caroti
is with anticoagulation an, in most cases, results in complete
stenosis, an, as such, an intervention is inicate. With the
resolution within a few months. Stenting may be an option in
history of raiation therapy an neck issection, the patient
symptomatic patients in the absence of occlusion (C). CEA,
has what is terme a “hostile neck.” This increases the risk
Fogarty embolectomy, or lytic therapy is not appropriate for
of caroti enarterectomy, in terms of cranial nerve injury
a spontaneous issection (A, B, E).
an woun healing. The previous neck issection results in
a paucity of tissue coverage between the skin an the caroti
22. E. Thromboangiitis obliterans (Buerger isease) is a
artery. This can lea to the catastrophic complication of
progressive nonatherosclerotic segmental inammatory is-
caroti blow out. The best alternative in this patient woul be
ease that most often affects small- to meium-size arteries,
to perform caroti stenting with a cerebral protection evice
veins, an nerves of the upper an lower extremities (C). The
(A–D). Patients with asymptomatic ICA stenosis in the 50%
typical age at onset is 0 to 50 years, an the isorer is more
to 69% range shoul be starte on meical therapy with an
common in men who smoke. The isease also affects the
antiplatelet agent (for all patients), antihypertensive agent (if
veins, an specically the upper extremities may be affecte
they have hypertension), an the use of a high-intensity sta-
by a migratory supercial thrombophlebitis. Patients initially
tin irrespective of lipi levels (ue to the pleiotropic effect of
present with foot, leg, arm, or han clauication. Progression
plaque stabalization).
of the isease leas to ischemic rest pain an ulcerations of
Reference: Harro-Kim P, Kakhoayan Y, Dereyn CP,
the toes, feet, an ngers. Characteristic angiographic n-
Cross DT 3r, Moran CJ. Outcomes of caroti angioplasty an
stenting for raiation-associate stenosis. AJNR Am J Neuroradiol.
ings may show isease connement to the istal circulation,
005;6(7):1781–1788. usually infrapopliteal an istal to the brachial artery. The
occlusions are segmental an show skip lesions with exten-
20. C. Recurrent caroti stenosis can occur after CEA. The sive collateralization, the so-calle corkscrew collaterals.
risk of more than 50% restenosis is 5.8%, 9.9%, 13.9%, an The iagnosis is ifcult to establish an is a iagnosis of
3.4% at 1, 3, 5, an 10 years, respectively. However, severe exclusion because there are no pathognomonic features. As
(>80%) stenosis evelops in only .1% of patients. Early such, the isease can be confuse with chronic emboliza-
(within 4 weeks) restenosis is usually ue to a technical tion an other iseases. Several criteria have been estab-
error. Recurrent caroti stenosis occurring beyon 1 month lishe to conrm the iagnosis: age younger than 45 years;
but within the rst  years after CEA is usually seconary to current (or recent) smoker; istal extremity ischemia (clau-
myointimal hyperplasia. This type of stenosis tens to have a ication, pain at rest, ischemic ulcers, gangrene); exclusion
benign course (the lesion is smooth an less prone to emboli- of autoimmune iseases, hypercoagulable states, an ia-
zation), with a low risk of recurrent stroke. In aition, reop- betes mellitus; exclusion of a proximal source of emboli by
erative CEA carries a higher risk of cranial nerve injury (7.3% echocariography an arteriography; an characteristic
rate of permanent injury in one series) (A). The patient is arteriographic nings in the involve limbs. The aor-
asymptomatic. If the patient ha a symptomatic recurrence, toiliac segments are typically spare, as are the coronary
the best option woul be caroti stenting (B, D–E). When the arteries (A, B). The mainstay of treatment revolves aroun
recurrent stenosis evelops  or more years after CEA, recur- smoking cessation. In patients who are able to abstain, is-
rent atherosclerosis is the usual cause. ease remission is impressive an amputation avoiance
is increase. The role of surgical intervention is minimal
21. D. Suen occlusion of the ICA in a young patient is because there is usually no acceptable target vessel for
highly suggestive of a spontaneous issection. This is fur- bypass (D). Sympathectomy may result in mil improve-
ther supporte by the tapere occlusion seen on imaging ment of symptoms.
(escribe as “ame-shape”). On the other han, occlusion Reference: Olin JW. Thromboangiitis obliterans (Buerger’s is-
ue to atherosclerosis typically occurs ush with the com- ease). N Engl J Med. 000;343(1):864–869.
mon caroti, an in oler patients ICA issection may occur
either spontaneously or after trauma. Cervical artery issec- 23. C. The ABI normally varies between 1 an 1. because
tion is a signicant cause of stroke in patients younger than the ankle pressure in the supine position can be as much as
40 years. Common presenting symptoms of ICA issection 0% higher than in the arm (A). Peripheral arterial isease
are heaache, transient ischemic attack an/or stroke, an has been ene as a value less than 0.9 an inicates some
Horner synrome (ptosis, miosis, anhyrosis). Risk factors egree of stenosis. Patients with clauication typically have
for issection inclue history of infection (syphilis), smok- an ABI between 0.5 an 0.7, an those with rest pain have
ing, Ehlers-Danlos synrome type IV, cystic meial necrosis, an ABI less than 0.4 (B). Patients with iabetes an en-
Marfan synrome, family history, oral contraceptives, an stage renal isease are at risk of eveloping calcication
atherosclerosis. In a young female, bromuscular ysplasia of the arterial meial layer, known as meial calcinosis, or
260 PArt i Patient Care

Mönckeberg arteriosclerosis. This process makes bloo ves- occlusive isease: review of the clinical literature. The Avisory
sels rigi an ifcult to compress, causing falsely increase Panel. J Vasc Interv Radiol. 000;11():149–161.
pressure reaings. The process tens to affect tibial vessels Results of a prospective ranomize trial evaluating surgery ver-
primarily an spares igital vessels in the toes. As such, toe sus thrombolysis for ischemia of the lower extremity. The STILE trial:
The STILE investigators (appenix A). Ann Surg. 1994;0(3):51–68.
pressures are more reliable, as are other measures of istal
perfusion such as transmetatarsal pulse volume recorings
25. C. The ipsilateral greater saphenous vein is the conuit
an transcutaneous oximetry (D, E).
of choice for lower extremity istal bypass for peripheral
Reference: Belkin M, Whittemore A, Donalson M, et al. Periph-
arterial isease (contralateral vein for trauma). An ieal con-
eral arterial occlusive isease. In: Townsen CM, Jr, Beauchamp RD,
Evers BM, Mattox KL, es. Sabiston textbook of surgery: the biological uit shoul be a minimum of 3 mm (but ieally 4 mm). When
basis of modern surgical practice. 17th e. Philaelphia: W.B. Sauners; the greater saphenous vein is not available, options inclue
004:199. the lesser saphenous an cephalic veins. Ectopic veins (i.e.,
lesser saphenous, arm veins) are generally inferior to a sin-
24. B. Absolute contrainications to thrombolytic therapy gle-segment saphenous vein, although they are still superior
inclue recent stroke or transient ischemic attack, active or to the performance of synthetic grafts. A composite graft,
recent bleeing, an signicant coagulopathy. Relative con- which is a vein graft sewn to a polytetrauoroethylene graft,
trainications inclue patients with recent major surgery has a patency rate similar to that of a prosthetic graft an
(within  weeks, an greatest with recent neurosurgery or tens to evelop neointimal hyperplasia. The bypass shoul
eye surgery), recent trauma, uncontrolle hypertension, intra- be as short as possible (proximal inow from the most istal
cranial tumors, an pregnancy (A). Thrombolytic therapy is normal artery (in this case, popliteal), an istal outow to
most effective in patients with ischemia of less than  weeks’ where the artery reconstitutes most proximally (in this case,
uration (D). The risk of bleeing with thrombolytic therapy above the ankle). Options A an B are suboptimal because it
is increase with the longer uration of therapy an with involves a longer bypass than is necessary given the patent
ecreasing brinogen levels. In most series, thrombolytic ther- femoral artery an normal popliteal pulse an harvesting
apy is use for as long as 48 hours, at which point the blee- contralateral vein. Enovascular approaches (such as angio-
ing risk increases signicantly (E). The causes of acute limb plasty) are options but are less urable, particularly in the
ischemia can be ivie into embolic an thrombotic. The presence of a long segment of occlusion. However, in a rela-
heart is the most common source of emboli leaing to acute tively healthy patient, with a goo saphenous vein an goo
ischemia, most often in the setting of atrial brillation. Other runoff into the foot, a bypass is likely the better option (D).
cariac sources inclue mural thrombus after an acute myo- Amputation of the toe is unlikely to heal in the absence of a
carial infarction, valvular isease, an atrial myxoma. Other palpable peal pulse an such a low ABI (E).
sources of emboli inclue arterial aneurysms an atheroscle- Reference: Gentile AT, Lee RW, Moneta GL, Taylor LM,
rotic plaques. Thrombosis is most often cause by unerlying Ewars JM, Porter JM. Results of bypass to the popliteal an tib-
atherosclerosis in the peripheral arteries, an these patients ial arteries with alternative sources of autogenous vein. J Vasc Surg.
typically have a history of clauication. The severity of acute 1996;3():7–79.
limb ischemia is base primarily on the motor an sensory
examination. Patients shoul be place in four categories:
26. E. This is Raynau isease. First escribe in 186 by
Maurice Raynau, the term Raynau isease applies to a
class 1 (nonthreatene) has normal motor an sensory func-
heterogeneous symptom array associate with peripheral
tion; class  (threatene) inclues  a—sensory ecit only
vasospasm, more commonly occurring in the upper extremi-
an b—(immeiately threatene) both motor an sensory
ties. The characteristically intermittent vasospasm classically
ecit; an class 3 inicates irreversible complete motor an
follows exposure to various stimuli, incluing col tempera-
sensory loss. In aition, consieration shoul be given to the
tures, tobacco, or emotional stress. Formerly, a istinction was
uration of ischemia. As a general rule, patients with class 1
mae between Raynau isease an the Raynau phenom-
ischemia can be treate with multiple options, a trial of hep-
enon for escribing a benign isease occurring in isolation
arin alone, thrombolytic therapy, or operative embolectomy/
or a more severe isease seconary to another unerlying
bypass. Patients with class  ischemia nee prompt restoration
isorer, respectively. However, collagen vascular isor-
of bloo ow, so heparin alone is not acceptable. With class
ers evelop in many patients at some point after the onset
b ischemia, the threat of limb loss is more immeiate. Since
of vasospastic symptoms; the rate of progression to a con-
thrombolytic therapy may require more than 4 to 48 hours to
nective tissue isorer ranges from 11% to 65% in reporte
restore ow, class b ischemia (motor an sensory ecit) is a
series. Characteristic color changes occur in response to the
relative contrainication to thrombolysis (C). Such a patient
arteriolar vasospasm, ranging from intense pallor to cyanosis
shoul be taken to the operating room. Category 3 ischemia
to reness as the vasospasm occurs. The igital vessels then
is consiere irreversible an requires amputation. Irrevers-
relax, eventually leaing to reactive hyperemia. The majority
ible ischemia is conrme by an absence of arterial or venous
of patients are women younger than 40 years of age. As many
Doppler signals, uration of ischemia of more than 6 to 8
as 70% to 90% of reporte patients are women, although
hours, presence of mottling of the skin, absence of capillary
many patients with only mil symptoms may never present
rell, an complete anesthesia an paralysis.
for treatment. Geographic regions locate in cooler, amp
References: Norgren L, Hiatt WR, Dormany JA, et al. Inter-so-
ciety consensus for the management of peripheral arterial isease climates such as the Pacic Northwest an Scaninavian
(TASC II). J Vasc Surg. 007;45 Suppl S:S5–S67. countries have a higher reporte prevalence of the isease.
Semba CP, Murphy TP, Bakal CW, Calis KA, Matalon TA. Throm- Certain occupational groups, such as those that use vibrat-
bolytic therapy with use of alteplase (rt-PA) in peripheral arterial ing tools, may be more preispose to Raynau isease or
CHAPtEr 17 Vascular—Arterial 261

igital ischemia. The exact pathophysiologic mechanism bloo cell count, an lactic aciosis. Diagnosis is conrme
behin the evelopment of such severe vasospasm remains by exible proctosigmoioscopy, which reveals a friable
elusive, an much attention has focuse on increase lev- mucosa. Proctosigmoioscopy may not be able to accurately
els of α-arenergic receptors an their hypersensitivity in istinguish partial ischemia from full-thickness necrosis. Ini-
patients with Raynau isease, as well as abnormalities in tial management is meical an consists of nasogastric tube
the thermoregulatory response, which is governe by the ecompression, IV hyration, placing the patient on NPO,
sympathetic nervous system. There is no cure for Raynau an broa-spectrum antibiotics. Full-thickness necrosis of
isease; thus, all treatments mainly palliate symptoms an the colon shoul be suspecte in patients with evience of
ecrease the severity an perhaps frequency of attacks. Con- peritonitis or unremitting aciosis. In such cases, laparotomy
servative measures preominate, incluing the wearing of with colonic resection an colostomy is inicate (C, D). The
gloves, use of electric or chemically activate han warm- mortality rate after emergent colectomy approaches 50%.
ers, avoiing occupational exposure to vibratory tools, absti- Arteriography woul not typically be helpful because the
nence from tobacco, an relocating to a warmer, rier climate. usual cause is an intene ligation or exclusion of an internal
The majority (90%) of patients will respon to avoiance of iliac artery or IMA (B, E).
col an other stimuli. The remaining 10% of patients with Reference: Becquemin JP, Majewski M, Fermani N, et al. Colon
more persistent or severe synromes can be treate with a ischemia following abominal aortic aneurysm repair in the era of
variety of vasoilatory rugs, albeit with only a 30% to 60% enovascular abominal aortic repair. J Vasc Surg. 008;47():58–63.
response rate. Calcium channel blocking agents such as il-
tiazem an nifeipine are the rugs of choice. The selective 29. B. The presentation is consistent with a rupture AAA.
serotonin reuptake inhibitor uoxetine has been shown to If the patient was hemoynamically unstable, he shoul be
reuce the frequency an uration of vasospastic episoes taken irectly to the operating room (D). If the patient is rela-
but is not the rst-line treatment (C). Intravenous infusions tively stable (as in this case), a CT scan is preferre to conrm
of prostaglanins have been reserve for nonresponers the presence of a rupture AAA an etermine feasibility of
with severe symptoms (B). Upper extremity sympathectomy enovascular repair, provie there is a coorinate mul-
may provie relief in 60% to 70% of patients; however, the tiisciplinary rupture aneurysm team that has immeiate
results are short-live, with a graual recurrence of symp- enovascular capabilities. Although most surgeons woul
toms in 60% within 10 years (A). Cervical sympathectomy approach a rupture AAA via the enovascular approach,
has fallen out of favor an has been replace by localize recent Cochrane analysis emonstrate no ifference in
igital sympathectomy using microsurgery. This involves 30-ay mortality for patients with rupture AAA that were
stripping the aventitia of igital arteries an thus removing treate with an enovascular approach compare to an
sympathetic bers. A col stimulation test or nail fol capil- open approach. Although ultrasonography is useful for
laroscopy may be use to conrm the iagnosis of Raynau etermining the presence of an AAA, it is not accurate for
isease, but there is no role for arteriography (D). etermining the presence of a retroperitoneal rupture (C).
Ultrasonography woul be reasonable to perform in this
27. D. Pseuoaneurysms can manifest with pain, a pulsa- patient was unstable, an no pulsatile mass coul be felt on
tile mass, an/or compression of ajacent structures. Large, physical examination, so as to conrm that an aneurysm was
expaning, painful pseuoaneurysms are at signicant risk present. Once in the operating room, the patient shoul be
of rupture an shoul be repaire urgently. Smaller, stable preppe an rape before anesthesia inuction because
pseuoaneurysms may be observe (E). Duplex ultrasonog- the anesthesia may inuce a precipitous ecrease in bloo
raphy has been the iagnostic proceure of choice because pressure (E). Because of the large retroperitoneal hematoma
it helps ene size, morphology, an location. Pseuoaneu- that is typically foun, proximal control is best achieve by
rysms less than  cm in iameter have a higher likelihoo of clamping the aorta at the iaphragm. Most surgeons woul
spontaneous thrombosis with compression therapy, whereas recommen a policy of “permissive hypotension” en route to
larger ones an those in patients receiving anticoagulation the operating room. Excessive ui aministration an ele-
therapy are likely to persist. However, given the reporte vation of the bloo pressure may further exacerbate bleeing
high failure rates with ultrasoun compression (A, B), ultra- (A).
sonography-guie thrombin injection is the best treatment References: Bager SA, Harkin DW, Blair PH, Ellis PK, Kee F,
option an is the treatment of choice. Surgery is reserve for Forster R. Enovascular repair or open repair for rupture abom-
inal aortic aneurysm: a Cochrane systematic review. BMJ Open.
infecte or rapily expaning pseuoaneurysms (C).
016;6():e008391.
Reference: Wixon CL, Philpott JM, Bogey WM Jr, Powell CS.
Lee WA, Hirneise CM, Tayyarah M, Huber TS, Seeger JM. Impact
Duplex-irecte thrombin injection as a metho to treat femoral
of enovascular repair on early outcomes of rupture abominal
artery pseuoaneurysms. J Am Coll Surg. 1998;187(4):464–466.
aortic aneurysms. J Vasc Surg. 004;40():11–15.
Van Der Vliet JA, Van Aalst DL, Schultze Kool LJ. Hypotensive
28. A. Colonic ischemia is a recognize complication after hemostasis (permissive hypotension) for rupture abominal aortic
AAA repair, whether open or enovascular. It occurs in aneurysm: are we really in control? Vascular. 007;15(4):197–00.
approximately 1% to 3% of cases. It is thought to be ue to
either ligation of the inferior mesenteric artery (IMA) or liga- 30. C. Popliteal aneurysms are the most common periph-
tion or exclusion of internal iliac arteries. The most common eral artery aneurysms (overall, aortic an iliac aneurysms
presentations inclue an unexpectely early return of bowel are more common). They can be suspecte on physical
function manifeste by iarrhea, left lower quarant pain, examination. They are bilateral in 50% of patients. Patients
abominal istention, persistent leukocytosis, elevate white who are foun to have a popliteal aneurysm shoul unergo
262 PArt i Patient Care

screening for an AAA because 30% will have a concomitant blee is foun, an thus one must empirically procee to
AAA. The most frequent complication of popliteal aneu- graft excision (D). The classic operative management con-
rysms is leg ischemia ue to thrombosis an embolization siste of obtaining proximal aortic control of the aorta at the
from the aneurysm. Guielines for repair are controversial. iaphragm, graft excision, closure of the aortic stump in two
Some authors recommen repair for all popliteal aneu- layers, closure of the uoenum, placing omentum in the
rysms. Most woul agree that inications for repair are (1) area of the aortic stump closure, followe by an extra ana-
all aneurysms larger than  cm, () aneurysms with intra- tomic axillobifemoral bypass. Recently, the more accepte
luminal thrombus, regarless of size, or (3) those that are treatment is excision of the aortic graft an in situ placement
symptomatic or have evience of previous embolization (A, of a human aortic homograft.
B). Diagnosis is mae by uplex ultrasonography, which Reference: Berger P, Moll FL. Aortic graft infections: is there
can measure the aneurysm size an etect the presence still a role for axillobifemoral reconstruction? Semin Vasc Surg.
of thrombus. Arteriography assists in operative planning 011;4(4):05–10.
but shoul not be use for iagnosis because it oes not
etect the thrombus nor accurately measure the size. The 32. C. Common iliac aneurysms are usually iagnose inci-
surgical approach to the popliteal artery is either via the entally. In most cases, they are foun in association with
meial approach or the posterior approach (E). The pos- an aortic aneurysm. Rare presentation inclues the evelop-
terior approach is ieal if the aneurysm is just behin the ment of a stula with the ajacent iliac vein or compression
knee joint. Magnetic resonance imaging an CT angiog- of the iliac vein. The natural history of common iliac aneu-
raphy can be use as alternatives for operative planning. rysms is less well ene. In a recent stuy, the expansion
The stanar operative approach involves bypassing the rate of common iliac aneurysms was 0.9 cm per year, an
aneurysm with saphenous vein an interval ligation of the hypertension preicte faster expansion. Because no rup-
popliteal artery. With this approach, the aneurysm sac is ture of a common iliac aneurysm smaller than 3.8 cm was
not opene, an as such, there is a small risk of continue observe, the recommene threshol for elective repair of
aneurysm expansion an compression of ajacent struc- asymptomatic patients was larger than 3.5 cm (A, B, D, E).
tures. Formal enoaneurysmorrhaphy, as is one with an Treatment options inclue open surgical replacement with
open AAA repair, is another alternative. In the setting of prosthetic graft or enovascular stent grafting. In patients
acute thrombosis, lytic therapy is the initial treatment of with suitable anatomy, namely, the presence of proximal an
choice. Enovascular stent grafting is another option, espe- istal laning zones, stent grafting has become the treatment
cially if no suitable vein is available in a high risk patient, of choice. Enovascular repair is associate with fewer com-
but this may have a lower primary patency (D). plications overall but poses a higher risk of creating buttock
References: Ascher E, Markevich N, Schutzer RW, Kallakuri S, clauication ue to occlusion of the internal iliac artery.
Jacob T, Hingorani AP. Small popliteal artery aneurysms: are they Reference: Huang Y, Gloviczki P, Duncan AA, et al. Common
clinically signicant? J Vasc Surg. 003;37(4):755–760. iliac artery aneurysm: expansion rate an results of open surgical
Lowell RC, Gloviczki P, Hallett JW Jr, et al. Popliteal artery an enovascular repair. J Vasc Surg. 008;47(6):103–110.
aneurysms: the risk of nonoperative management. Ann Vasc Surg.
1994;8(1):14–3. 33. B. Popliteal aneurysms rarely rupture (A). Most com-
monly, they cause acute or chronic ischemia. In most series,
31. E. A patient with an upper gastrointestinal blee an a the most common symptom is thrombosis, in as many as 49%,
history of aortic surgery shoul be presume to have an aor- followe by istal embolization. As the aneurysm continues
toenteric stula until proven otherwise. The treatment algo- to grow, less commonly, it can compress ajacent structures,
rithm epens on the hemoynamic stability of the patient. such as the popliteal vein (D, E). Chronic embolization can
If the patient is unable to be stabilize ue to massive hem- lea to occlusions of the infrapopliteal vessels an can com-
orrhage, the patient shoul be taken emergently to the oper- plicate revascularization (C). If they present with acute isch-
ating room, even if a iagnosis has not yet been establishe. emia, thrombolysis is the intervention of choice, followe by
Oftentimes, the patient will have a so-calle heral blee, operative repair. Recently, enovascular stent grafting has
after which the bleeing may temporarily stop, allowing a been use, although long-term patency ata are still lacking.
workup for an aortoenteric stula. The iagnosis can be if- References: Dorigo W, Pulli R, Turini F. Acute leg ischemia from
thrombose popliteal artery aneurysms: role of preoperative throm-
cult to establish. Upper enoscopy is negative surprisingly
bolysis. Eur J Vasc Endovasc Surg. 00;3(3):51–54.
often an has a low sensitivity but shoul be the rst step Shortell CK, DeWeese JA, Ouriel K, Green RM. Popliteal
in the workup. Duoenal graft erosion typically occurs at artery aneurysms: a 5-year surgical experience. J Vasc Surg.
the fourth portion of the uoenum, an nings may be 1991;14(6):771–776.
subtle, such as mil mucosal erosion. CT scan is highly use-
ful, as in the presence of an aortoenteric stula will likely 34. C. Recent stuies have shown that AAAs as large as
emonstrate perigraft ui, air, or inammation, inicative 5.5 cm in iameter can be safely observe (D). Another recent
of a graft infection (though less likely contrast extravasation). ranomize stuy inicate that although the perioperative
Flui an inammatory changes aroun a graft woul be mortality rate of EVAR is lower than that of open repair,
abnormal nings beyon 6 weeks after surgery (A). If the long-term mortality is the same (C). Women have been
CT scan nings are negative, a nuclear-tagge white bloo shown to have higher perioperative mortality rates than men
cell scan may be useful for establishing a graft infection (C). with either EVAR or open repair. EVAR shoul not lower the
Arteriography is of limite benet for the iagnosis of vascu- size threshol for repair in a high-cariac risk patient if the
lar graft infections but can be useful in preoperative planning AAA has not yet reache the 5.5-cm threshol (B). Following
(B). In some instances, no source of an upper gastrointestinal are the guielines for treatment of AAAs as reporte by a
CHAPtEr 17 Vascular—Arterial 263

subcommittee of the Joint Council of the American Associ- late as 7 years after EVAR. Enoleaks are classie into ve
ation for Vascular Surgery an Society for Vascular Surgery: major types (types I–V) base on the source of communica-
1. The arbitrary setting of a single-threshol iameter for tion between the circulation an the aneurysm sac. The most
elective AAA repair that is applicable to all patients is common type of leak after enovascular repair is a type II
not appropriate because the ecision for repair must be leak, which results from retrograe lling of the aneurysm
iniviualize in each case. sac from the lumbar arteries or the IMA. Management of
2. Ranomize trials have shown that the risk of rupture type II leaks is controversial an is base on whether the
of small AAAs is quite low an that a policy of careful aneurysm is enlarging or stable. Options inclue coil embo-
surveillance of those with a iameter of as large as lization of the vessel, laparoscopic ligation, or observation.
5.5 cm is safe, unless there is rapi expansion (>1 cm/yr) Type I leaks occur at the stent–graft attachment sites (either
or symptoms evelop. However, early surgery is at the aorta or at the iliac arteries) (A); type III leaks occur
comparable to surveillance with later surgery, so patient at a stent–stent interface an are also known as moular
preference is important, especially for AAAs 4.5 to 5.5 cm isassociations (C); type IV leaks are irectly through the
in iameter. graft an are ue to graft material porosity (D). They usu-
3. Base on the best available current evience, a iameter ally heal spontaneously. The most angerous type of leak is
of 5.5 cm appears to be an appropriate threshol for a proximal type I leak because there is a failure to achieve a
repair in an average patient. However, subsets of proximal seal, leaing to continue lling of the aneurysm
younger, low-risk patients with a long projecte life sac at systemic pressures. Type I leaks require immeiate
expectancy may prefer early repair. If the surgeon’s treatment when iscovere, typically by eploying another
personal ocumente operative mortality rate is low, stent or, if unsuccessful, by open surgical conversion. Type III
repair may be inicate at smaller sizes if that is the enoleaks represent a true mechanical failure of the enog-
patient’s preference. raft an require repair with an aitional enograft to elim-
4. For women or for AAAs with a greater than average inate systemic ow an pressure in the aneurysm. Type V
rupture risk, 4.5 to 5 cm is an appropriate threshol for leak is also referre to as enotension. This can be consi-
elective repair (A). ere iiopathic because the aneurysmal sac may appear to be
5. For high-risk patients, elay in repair until the iameter enlarging without any evience of a leak site on imaging (E).
is larger is warrante, especially if enovascular aortic Reference: Corriere MA, Feurer ID, Becker SY, et al. Enoleak
repair is not possible (E). following enovascular abominal aortic aneurysm repair: implica-
6. In view of its uncertain long-term urability an tions for uration of screening. Ann Surg. 004;39(6):800–807.
effectiveness as well as the increase surveillance
buren, EVAR is most appropriate for patients at 36. E. This patient most likely has an arterial thrombus
increase risk of conventional open aneurysm repair. seconary to heparin-inuce thrombocytopenia thrombo-
EVAR may be the preferre treatment metho if sis (HITT). The classic laboratory ning is a ecrease in the
anatomy is appropriate for oler high-risk patients, those platelet count of more than 50%. Although thrombocytope-
with a hostile abomen, or other clinical circumstances nia usually increases the risk of bleeing, HITT is paraoxi-
likely to increase the risk of conventional open repair. cally known to cause a hypercoagulable state; it is the secon
7. Use of EVAR in patients with unsuitable anatomy most common acquire hypercoagulable state (smoking is
markely increases the risk of averse outcomes, the the most common). There are two types of HITT with type II
nee for conversion to open repair, or AAA rupture. being more common an responsible for the clinical synrome.
8. At present, there oes not seem to be any justication HITT type II is cause by antiboies to platelet-factor 4 an
that EVAR shoul change the accepte size threshol for heparin sulfate resulting in a prothrombotic state (will
intervention in most patients. appear as a white clot). It typically occurs 3 to 5 ays after
9. In choosing between open repair an EVAR, patient starting heparin. If this is suspecte, heparin shoul be
preference is of great importance. It is essential that the iscontinue, an the patient shoul be starte on a irect
patients be well informe to make such choices. thrombin inhibitor. Argatroban is the recommene agent
References: Brewster DC, Cronenwett JL, Hallett JW Jr, et al. for patients with HITT an renal impairment. Lepiruin
Guielines for the treatment of abominal aortic aneurysms. Report an bivaliruin both unergo renal excretion an shoul be
of a subcommittee of the Joint Council of the American Association avoie in patients with ESRD (A, D). The patient initially
for Vascular Surgery an Society for Vascular Surgery. J Vasc Surg. ha acute limb ischemia seconary to cariac emboli from
003;37(5):1106–1117. atrial brillation. His symptoms resolve with initiation of
Mureebe L, Egorova N, McKinsey JF, Kent KC. Gener trens in heparin so it is unlikely that he has unerlying antithrom-
the repair of rupture abominal aortic aneurysms an outcomes. bin III eciency (C). More stuies are neee to evaluate
J Vasc Surg. 010;51(4 Suppl):9 S–13 S.
the role of tPA in HITT (B).
References: Guzzi LM, McCollum DA, Hursting MJ. Effect of
35. B. Enoleak is a common complication after EVAR renal function on argatroban therapy in heparin-inuce thrombo-
that can lea to aneurysm enlargement an even rupture. cytopenia. J Thromb Thrombolysis. 006;(3):169–176.
Enoleaks occur in as many as 40% of patients after EVAR. Visentin GP, For SE, Scott JP, Aster RH. Antiboies from patients
Most enoleaks are foun in the immeiate postoperative with heparin-inuce thrombocytopenia/thrombosis are specic
perio, but late enoleaks also evelop. For this reason, rou- for platelet factor 4 complexe with heparin or boun to enothelial
tine lifelong postoperative surveillance with CT scanning cells. J Clin Invest. 1994;93(1):81–88.
is recommene. New enoleaks have been ientie as
Vascular—Venous
AMANDA C. PURDY AND JOHN McCALLUM 18
ABSITE 99th Percentile High-Yields
I. Deep Vein Thrombosis (DVT) an Pulmonary Embolism (PE)
A. Can be provoke (known inciting event, such as recent surgery, malignancy) or unprovoke
B. Most common inherite prothrombotic isorer: Factor-V Leien mutation (unable to breakown
Factor-V); higher incience on left sie (May-Thurner synrome)
C. Catheter-associate upper extremity DVTs—etermine if catheter is require, if it is, can keep catheter
an start therapeutic anticoagulation; if not require, start anticoagulation an remove catheter in 3 to 5
ays; in both cases, continue anticoagulation for 3 to 6 months
D. Malignancy-associate DVT/PE best treate with low-molecular-weight heparin (not warfarin)

Situation Duration of anticoagulation


Provoked DVT/PE without associated malignancy 3 months
Provoked DVT/PE in association with malignancy Indeęnite
Unprovoked DVT/PE in patient with low-moderate Indeęnite
bleeding risk
Unprovoked DVT/PE in patient with high bleeding risk 3 months
DVT/PE pregnant woman 3 months OR 6 weeks after delivery (whichever is longer)
Superęcial thrombophlebitis of legs 4–6 weeks of fondaparinux

E. Phlegmasia alba olens


1. Cause by massive DVT, eep venous channels are affecte while sparing collateral veins an
therefore maintaining some egree of venous return; patients have pale blanching of extremity (milky
color), eema, an iscomfort
. Treatment is immeiate anticoagulation an leg elevation; may progress to cerulea olens
F. Phlegmasia cerulea olens
1. Venous outow of the entire lower extremity is affecte; persistent venous obstruction eventually
inhibits arterial inow
. Patients present with pain, swelling, an a blue iscoloration (cyanosis) of the extremity; peal
pulses may be iminishe
3. In aition to anticoagulation, catheter-irecte thrombolysis is also recommene (unless > weeks
uration)
G. Paget-Schroetter synrome (effort thrombosis of axillary or subclavian vein)
1. Thrombosis of the axillary an/or subclavian veins
. Often seen in athletes who overuse the affecte extremity, leaing to recurrent vein compression

265
266 PArt i Patient Care

3. Often relate to thoracic outlet synrome, with compression of the subclavian vein in the
costoclavicular space (between the 1st rib an the clavicle)
4. Diagnose with uplex ultrasoun or venogram
5. Treat with anticoagulation; if moerate-severe symptoms of < weeks uration can consier catheter-
irecte thrombolysis; after acute treatment of the DVT, procee with 1st rib resection to prevent
recurrence
H. PE
1. Signs: shortness of breath, pleuritic chest pain, tachycaria, increase respiratory rate, hypoxemic
respiratory alkalosis
. Chest raiograph usually normal, most common EKG ning is sinus tachycaria
3. Best iagnosis is CT angiogram of the chest
4. Treatment consierations:
a) Most treate with anticoagulation
b) Massive PE (ene as PE + hypotension): treat with systemic thrombolysis (if no
contrainication)
(1) If thrombolysis fails or patient has a contrainication to systemic thrombolysis: catheter-
assiste thrombus removal
() If catheter-assiste thrombus removal fails: surgical pulmonary embolectomy

II. Chronic Venous Disease


A. Can have insufciency in supercial, eep, an/or perforator veins
B. Symptoms: painful varicose veins, leg swelling, an meial malleolar ulcers
C. Hyperpigmentation in venous insufciency cause by breakown of extravascular RBC & subcutaneous
scar tissue (liposclerosis)
D. Diagnosis: uplex ultrasoun (specically for valve incompetence); reux (incompetence) is iagnose if
the ultrasoun shows retrograe ow of >1 secon in the femoral or popliteal veins, or >0.5 secon in the
saphenous, tibial, eep femoral, or perforator veins; also, uplex ultrasoun scan to rule out DVT
E. Mainstay of treatment of venous insufciency is compression therapy
F. If compression fails, can consier surgical intervention if incompetence/reux is in supercial or
perforator veins:
1. For supercial veins (greater or small saphenous veins) or perforator veins—treat with enovenous
ablation
. If have concomitant symptomatic varicose veins: treat unerlying venous incompetence AND can
also perform sclerotherapy or stab phlebectomy of supercial varicose veins
3. Complication of saphenous vein ablation: thrombus at saphenofemoral junction (potential risk for
DVT), start ASA
4. Telangiectasias (spier veins): not always associate with reux—if no reux compression an
ablation will not help; treatment is cosmetic: inclues injection sclerotherapy or transermal laser
therapy
CHAPtEr 18 Vascular—Venous 267

Fig. 18.1 May-Thurner Synrome


268 PArt i Patient Care

Questions
1. Which of the following is the most common risk 5. Which of the following is true regaring the
factor for spontaneous venous thromboembolism? initiation of heparin in a 100-kg patient with a
A. Antithrombin III eciency newly iagnose DVT?
B. Factor V Leien A. A bolus of 10,000 units of heparin shoul be
C. Protein C eciency given before starting the rip
D. Protein S eciency B. Following a bolus a rip shoul be starte at
E. Antiphospholipi synrome 18 units/kg per hour
C. Dosing shoul be ajuste using the
2. A 60-year-ol male presents with pain over his international normalize ratio (INR)
left mi-meial thigh. He ha a similar event D. Activate partial thromboplastin time (aPTT)
in his other thigh a month earlier. He has note shoul be titrate to 100 to 10 secons after
a ecrease appetite. On exam the skin over starting the rip
the meial thigh is re, warm, an tener. He E. Heparin shoul be stoppe if the platelet
has no varicose veins, nor evience of skin count ecreases below 00,000
hyperpigmentation or leg swelling. Duplex scan
shows an 8-cm segment of thrombosis of the 6. A 35-year-ol female presents with left leg
mi saphenous vein, but no DVT. Which of the swelling. There are no precipitating factors.
following is recommene? Ultrasoun conrms a left iliofemoral DVT, an
A. IV heparin followe by warfarin the patient is starte on heparin. Workup reveals
B. Warm compresses, nonsteroial no evience of risk factors for DVT, such as recent
antiinammatory rugs (NSAIDs), an a CT surgery, prolonge immobilization, nor any
of the abomen evience of malignancy. Which of the following is
C. Ligation of the sapheno-femoral junction most likely to be of long-term benet?
D. Fonaparinux an a CT of the abomen A. Low-molecular-weight heparin (LMWH)
E. Warm compresses an NSAIDs B. Long-term (>1 months) anticoagulation
C. Lifelong compression stocking
3. A 5-year-ol male college swimmer presents D. Right-to-left femoral vein bypass
with suen onset of right arm swelling an E. Venous thrombectomy
pain. A uplex ultrasoun scan emonstrates
thrombosis of the axillary-subclavian vein. 7. Which of the following is true regar ing the
The patient is starte on IV heparin. The most management of DVT?
important aitional ajunctive therapy for this A. For patients with proximal DVT of the leg
patient is: an no cancer history, irect Xa inhibitor is
A. First rib resection recommene over warfarin
B. Catheter-guie thrombolysis B. For a leg DVT in association with malignancy,
C. Lifelong anticoagulation warfarin is preferre over LMWH
D. Venous stenting C. For incientally iscovere DVT,
E. Physical therapy anticoagulation is unnecessary
D. In patients with isolate istal (calf) DVT of
4. Which of the following is true regaring venous the leg, anticoagulation therapy is superior to
circulation? serial imaging
A. The perforating veins in the leg irect bloo E. In patients with a secon episoe of
ow from eep to the supercial system DVT, three months of anticoagulation is
B. The common iliac veins have valves recommene
C. In a healthy person, venous pressure increases
with walking
D. The greater saphenous vein joins the femoral
vein to become the common femoral vein
E. Muscle contraction plays no role in venous
return
CHAPtEr 18 Vascular—Venous 269

8. A 58-year-ol male with newly iagnose 13. A 50-year-ol male presents with a meial
metastatic colon cancer presents to the ED with malleolar ulcer that has faile to heal with
a swollen right leg an severe pain that starte 4 weeks of compression ressings. He has
1 ay earlier. On exam, he has massive eema of large varicose veins in the lower leg, eema,
the right leg that is tener to palpation. His foot an hyperpigmentation. There is no eep
appears blue. Duplex scan conrms a DVT. Which vein thrombosis (DVT) ientie on uplex
of the following is true about this conition? ultrasoun. However, there is incompetence of
A. The risk of limb loss is low the supercial, eep, an perforator systems.
B. This occurs more commonly on the right sie Which of the following is the best next step?
C. A pale, white foot carries a worse prognosis A. Vein stripping of the greater saphenous vein
than a blue foot B. Raiofrequency ablation (RFA) of the greater
D. Associate hypotension is usually the result of saphenous vein an ultrasoun-guie
sepsis perforator sclerotherapy
E. Catheter-irecte thrombolysis shoul be C. RFA of the greater saphenous vein an
performe compression stockings
D. Continue with a 3-month course of
9. Which of the following is the best inication for compression ressing treatment
placement of an inferior vena cava (IVC) lter? E. Ultrasoun-guie perforator vein
A. A pregnant patient in the thir trimester sclerotherapy
iagnose with a new DVT
B. A patient with severe pelvic fractures 14. A 44-year-ol male presents to the emergency
C. A patient with a large free-oating vena cava epartment (ED) with a temperature of 103°F.
thrombus He is hypotensive espite a -L ui bolus. He
D. A recurrent DVT in a patient who is alreay is preppe for a right internal jugular 9-French
therapeutic on warfarin central venous line to start pressors while being
E. Before planne thrombolysis of a new DVT worke up for an unerlying cause. Following
placement of the catheter, pulsatile bleeing is
10. The most common electrocariographic change note from the catheter. What is the best next
after pulmonary embolism (PE) is: step?
A. Atrial brillation A. Downsize to a smaller catheter in the ED,
B. Right bunle branch block transfer the patient to the intensive care unit
C. Nonspecic ST an T wave changes (ICU), an remove it in several hours
D. S1, Q3, T3 pattern B. Immeiately remove the catheter an hol
E. Sinus tachycaria pressure for 10 to 15 minutes
C. Immeiately remove the catheter an get a
11. Trauma patients sustaining what type of injury uplex ultrasoun stuy of the neck
are at highest risk of venous thromboembolism? D. Remove the catheter uner irect surgical
A. Hea trauma exposure
B. Femur fracture E. Transfer patient to the ICU, then remove, hol
C. Pelvic fracture pressure, an place a suture in the skin
D. Splenectomy
E. Spinal cor injury 15. A 40-year-ol woman presents with pain an
tenerness at the site of a longstaning varicose
12. A 45-year-ol woman presents with a nonhealing vein in her calf. There is a palpable cor with
ulcer at the meial malleolus associate with leg surrouning erythema. Duplex scan shows
eema an hyperpigmentation but no signs of localize thrombus within the varicose vein, an
infection. First-line management consists of: no DVT. Management consists of:
A. Wet-to-ry ressings A. Intravenous (IV) heparin soium
B. Split-thickness skin grafting B. Subcutaneous low-molecular-weight heparin
C. Subfascial perforator ligation C. Warm compresses an nonsteroial
D. Local woun ebriement followe by antiinammatory rugs
intravenous antibiotics D. Ligation of saphenous vein at saphenofemoral
E. Compression ressings junction
E. IV antibiotics
270 PArt i Patient Care

Answers
1. B. The primary risk factors for spontaneous venous of propagating into the eep system an thus benet from
thromboembolism (VTE) as escribe by Virchow inclue anticoagulation (A). A recent stuy comparing fonaparinux
stasis of bloo ow, enothelial injury, an hypercoagula- with placebo emonstrate a ecrease in DVT, recurrent
bility. In cases of spontaneous VTE, hypercoagulability is thrombophlebitis, an clot progression with fonaparinux.
the most important factor. Factors that contribute to hyper- References: Chengelis DL, Benick PJ, Glover JL, Brown OW,
coagulability inclue factor V Leien, prothrombin gene Ranval TJ. Progression of supercial venous thrombosis to eep vein
mutation, protein C an S eciency, antithrombin III e- thrombosis. J Vasc Surg. 1996;4(5):745–749.
ciency, elevate homocysteine levels, an antiphospholipi Decousus H, Pranoni P, Mismetti P, et al. Fonaparinux for the
treatment of supercial-vein thrombosis in the legs. N Engl J Med.
synrome. In aition, nonacquire causes of VTE inclue
010;363(13):1–13.
smoking (most common), obesity, pregnancy, malignancy,
an use of oral contraceptives. In surgical patients, the cause
3. A. Paget-Schroetter synrome, also known as effort-in-
of VTE is multifactorial because postoperative stasis from
uce thrombosis, is a spontaneous thrombosis of the axil-
prolonge be rest an enothelial injury from trauma or
lary-subclavian vein. It is thought to be, in most instances, a
recent surgery are signicant factors. In trauma patients, spi-
manifestation of thoracic outlet synrome, whereby a hyper-
nal cor injury has the highest risk of VTE. Other risk factors
trophie or aberrant muscle compresses the axillary-subclavian
for VTE inclue history of VTE, avance age, an varicose
vein as it passes between the rst rib an the clavicle. It tens
veins. Factor V Leien is the most common genetic efect
to evelop in young, active patients after vigorous activ-
associate with thrombophilia (A, C–E). Factor V Leien is a
ity (swimming, pitching, weightlifting), although it can also
single-point mutation in the gene that coes for coagulation
occur spontaneously. It usually presents in men more often
factor V. It makes factor V resistant to inactivation by acti-
than women. Seconary axillary/subclavian vein thrombosis
vate protein C (which is a natural anticoagulant protein).
can also present in those with meiastinal tumors, congestive
The mutation is transmitte in an autosomal ominant fash-
heart failure (CHF), an nephrotic synrome. Diagnosis is best
ion an accounts for 9% of cases of anticoagulant protein
establishe via uplex ultrasonography. The patient shoul be
resistance. The mutation is present in 4% to 6% of the gen-
promptly starte on IV heparin. The most important ajunc-
eral population an is associate with a sixfol increase
tive measure to prevent recurrence an long-term swelling is
risk of VTE in heterozygotes. In homozygotes, the risk is
thoracic outlet ecompression via rst rib resection. The timing
80-fol. In patients with their rst VTE, factor V Leien was
is controversial but is not time sensitive. Systemic thromboly-
present in 15% to 0%. There is no stanar guieline for
sis is not inicate. However, the journal CHEST recommens
the uration of anticoagulation therapy in patients with an
catheter-irecte thrombolysis for this conition if the patient
acquire hypercoagulable state. It is believe an iniviu-
has moerate-severe symptoms an presents with less than
alize approach shoul be taken to access each person's risk
 weeks of symptoms. The benet of catheter-irecte throm-
of a recurrent VTE an compare this to their relative risk of a
bolysis in this situation is a ecrease risk of postthrombotic
bleeing event. Interestingly, in one stuy, the risk of recur-
synrome (B). A follow-up venogram is frequently obtaine
rent VTE was similar among carriers of the factor V Leien
to ientify any correctable anatomic abnormalities. Stenting a
gene compare with those without this mutation, suggesting
resiual stenosis in this area without ecompressing the tho-
that they o not nee longer anticoagulation than the stan-
racic outlet is contrainicate because the ongoing compres-
ar recommenation for a rst-time event.
sion will invariably crush the stent an cause further venous
References: Bauer KA. Duration of anticoagulation: applying
amage, making any further intervention even more ifcult.
the guielines an beyon. Hematology Am Soc Hematol Educ Pro-
gram. 010;010(1):10–15.
Resiual venous stenoses can be treate with angioplasty,
Mazza JJ. Hypercoagulability an venous thromboembolism: a although some authors recommen oing this after the rst
review. WMJ. 004;103():41–49. rib resection. A recent metaanalysis emonstrate a signi-
cant improvement in symptoms in those that receive a rst
2. D. Unprovoke SVT, an in particular, recurrent unpro- rib resection compare to those that i not. More than 40%
voke SVT, an more specically recurrent, unprovoke of patients in the control group neee to have a rib resec-
SVT in ifferent limbs (supercial migratory thrombophle- tion ue to recurrent symptoms. In an active athlete, an in
bitis) shoul prompt concern for hypercoagulability an, in particular one who performs repetitive movements with the
particular, malignancy. Supercial migratory thrombophle- arm overhea (which by itself can compress the vein), rst rib
bitis is particularly associate with pancreatic cancer (Trous- resection is the best option (C–E).
seau sign) an, to a lesser egree, stomach an lung cancer. References: Angle N, Gelabert HA, Farooq MM, et al. Safety an
efcacy of early surgical ecompression of the thoracic outlet for
Thus, treatment shoul inclue a targete workup for malig-
Paget-Schroetter synrome. Ann Vasc Surg. 001;15(1):37–4.
nancy (that shoul be tailore to nings on history, review Lee WA, Hill BB, Harris EJ Jr, Semba CP, Olcott C IV. Surgical
of systems, an physical exam) (B, C–E). Given the ecrease intervention is not require for all patients with subclavian vein
appetite, suspicion for GI cancer shoul be high an a CT thrombosis. J Vasc Surg. 000;3(1):57–67.
scan appropriate. SVT within the saphenous vein in the Machleer HI. Evaluation of a new treatment strategy for
upper thigh, within 3 cm of the saphenofemoral junction, Paget-Schroetter synrome: spontaneous thrombosis of the axil-
an those with long segments (>5 cm) have an increase risk lary-subclavian vein. J Vasc Surg. 1993;17():305–315.
CHAPtEr 18 Vascular—Venous 271

Urschel HC Jr, Razzuk MA. Paget-Schroetter synrome: what is DVT. Proximal (iliofemoral) DVTs are more likely to lea to
the best management? Ann Thorac Surg. 000;69(6):1663–1668. massive swelling an long-term sequelae of postphlebitic
Lugo J, Tanious A, Armstrong P, et al. Acute Paget-Schroetter synrome. As such, more consieration shoul be given to
synrome: oes the rst rib routinely nee to be remove after the type of anticoagulant, the uration, use of thrombolyt-
thrombolysis? Ann Vasc Surg. 015;9(6):1073–1077.
ics, an mechanical thromboembolectomy as compare with
istal DVT. LMWH is not as efcacious for proximal DVT
4. D. The lower extremity veins are ivie into supercial,
(A). For a proximal (iliofemoral) DVT, that is unprovoke
perforating, an eep veins. The supercial venous system
(no clear contributing factors), the recommenation is for
consists of the greater saphenous an lesser saphenous veins.
long-term (>1 months) anticoagulation. The benet of
The eep veins follow the course of major arteries. Paire
compression stockings to prevent postphlebitic synrome
veins parallel the anterior an posterior tibial an peroneal
is controversial (C). Most authors recommen  years of
arteries an join to form the popliteal vein. The popliteal vein
compression; lifelong compression has no benet, has poor
becomes the femoral vein as it passes through the auctor
patient compliance, an is associate with signicant costs
hiatus. In the proximal thigh, the greater saphenous vein
of renewing expensive stockings every 6 months. A right-
joins with the femoral vein to become the common femoral
to-left femoral vein bypass (with right leg saphenous vein)
vein. Multiple perforating veins traverse the eep fascia to
is rarely performe an woul be a last resort for chronic
connect the supercial an eep venous systems. The most
venous stasis that is unresponsive to enovascular options
important perforators are the Cockett an Boy perforators.
(D). Thrombolytic therapy is an option for select patients
The Cockett perforators rain the lower part of the leg mei-
with severe iliofemoral DVT, particularly if they present
ally, whereas the Boy perforators connect the greater saphe-
with phlegmasia. Venous thrombectomy is reserve for
nous vein to the eep vein higher up in the meial lower leg,
patients with phlegmasia who have faile thrombolytic
approximately 10 cm below the knee. Bloo ows from the
therapy (E).
supercial to the eep venous system (A). Incompetence of
Reference: Heffner JE. Upate of antithrombotic guielines:
these perforators is a major contributor to the evelopment
meical professionalism an the funnel of knowlege. Chest.
of venous stasis an ulceration. There are no valves in the 016;149():93–94.
portal vein, superior vena cava, inferior vena cava (IVC), or
common iliac vein (B). The calf muscles serve an important
7. A. The American College of Chest Physicians release
function in augmenting venous return by acting as a pump
upate guielines in 016 for the management of DVTs.
to return bloo to the heart (E). For this reason, patients who
One major change is that patients with proximal DVT of
are berien are prone to venous stasis. Venous pressure
the leg an no cancer history shoul now be treate with
rops ramatically with walking because of the action of the
a irect Xa inhibitor (abigatran, rivaroxaban, apixaban, or
calf muscles but increases in patients with venous obstruc-
eoxaban) over warfarin. Aitionally, initial parenteral
tion because this leas to persistent stasis that muscle con-
anticoagulation with a heparin rip is not require when
traction cannot overcome (C). This is why compression
using rivaroxaban an apixaban. However, a heparin rip
stockings are recommene for these patients as an ajunct
shoul be starte before aministering abigatran or eox-
to normal venous return.
aban an overlappe with warfarin therapy. Several reports
have shown the superiority of these novel oral anticoagu-
5. B. If a heparin rip is starte, a bolus of 80 units/kg (8000
lants (NOACS). Aitionally, iarucizumab is now available
units for the above patient) shoul rst be given followe by
as a reversal agent for abigatran allowing NOACs to be
the continue infusion of heparin at 18 units/kg per hour
more commonly prescribe. In patients with a cancer history
(A). In patients with DVT, the aPTT nees to be rawn every
an proximal DVT of the leg, LMWH is recommene over
6 to 1 hours with a goal rate of 60 to 90 secons (D). INR is
warfarin an irect Xa inhibitors (B). This is unchange from
checke in patients on warfarin (C). Heparin can potentially
the prior guielines. In patients with a proximal DVT of the
lea to heparin-inuce thrombocytopenia (HIT). This usu-
leg provoke by surgery, 3 months of anticoagulation ther-
ally happens 5 ays or more after the initiation of heparin
apy is recommene over a longer time-limite perio (6, 9,
an will present as a 50% rop in platelet count (E).
1, or 4 months). This recommenation applies to patients
Reference: Hirsh J, Bauer KA, Donati MB, Goul M, Samama
MM, Weitz JI. Parenteral anticoagulants: American college of chest with both low an high bleeing risks. The management of
physicians evience-base clinical practice guielines (8th eition). isolate calf DVT remains controversial. Anticoagulation is
Chest. 008;133(6 Suppl):141S–159S. recommene for those with severe leg symptoms or those
with risk factors for propagation. In patients with an isolate
6. B. DVT an pulmonary embolism (PE) affect up to istal DVT of the leg an without severe symptoms or risk
900,000 people per year in the Unite States, an their inci- factors for extension, serial imaging of the eep veins for 
ence increases with age. When a patient presents with a weeks is recommene over anticoagulation therapy (D).
DVT, always try to etermine which part of the Virchow There is no consensus on the uration of therapy for patients
tria (stasis, vascular injury, an hypercoagulability) can with a secon episoe of DVT because this epens on the
explain the event. This will serve as a reminer to perform presence of reversible risk factors, unerlying cause, malig-
a careful history an physical examination to assess risk fac- nancy, life expectancy, an the buren of therapy. However,
tors for DVT. In most cases, the causes are multifactorial. The most surgeons woul recommen at least 1 year of antico-
uration an type of anticoagulation epen on whether agulation therapy for patients with a secon episoe of DVT
the DVT is provoke (i.e., malignancy, recent surgery, pro- an lifelong anticoagulation for patients with more than two
longe immobilization) or unprovoke, what the provoking episoes of DVT (E). Incientally iscovere DVTs shoul be
factor is, an on the location (proximal or istal leg) of the treate with anticoagulation (C).
272 PArt i Patient Care

References: Connors JM. Antiote for factor Xa anticoagulants. thrombectomy (to prevent further ecompensation), an the
N Engl J Me. 015;373(5):471–47. presence of a large free-oating thrombus in the IVC. Preg-
Heffner JE. Upate of antithrombotic guielines: meical profes- nant patients iagnose with a new DVT shoul be starte
sionalism an the funnel of knowlege. Chest. 016;149():93–94. on anticoagulation with low-molecular-weight heparin for
the remainer of the pregnancy an up to 6 weeks postpar-
8. E. Massive iliofemoral DVT can lea to impaire arterial tum (A). Warfarin shoul be avoie since it is teratogenic.
bloo ow ue to massive swelling. Early on, the limb turns References: Decousus H, Leizorovicz A, Parent F, etal. A clini-
pale an is referre to as phlegmasia alba (white) olens. cal trial of vena caval lters in the prevention of pulmonary embo-
In a subgroup of patients, this may progress to impening lism in patients with proximal eep-vein thrombosis. N Engl J Med.
gangrene phlegmasia cerulea (blue) olens as in the patient 1998;338(7):409-416.
escribe. When the majority of the eep venous channels Millwar SF, Oliva VL, Bell SD, et al. Günther tulip retrievable vena
are burene with clots, the relatively smaller supercial cava lter: Results from the Registry of the Canaian Interventional
venous channels are taske with raining the entire leg. Raiology Association. J Vasc Interv Radiol. 001;1(9):1053–1058.
Patients evelop a tener, pale, an eematous extremity. Rajasekhar A. Inferior vena cava lters: current best practices. J
This is known as “milk-leg” since the pale extremity appears Thromb Thrombolysis. 015;39(3):315–37.
whitish (alba). As the isease progresses an the supercial
venous channels are also affecte, the entire venous rain-
10. E. The most common ning on electrocariogra-
phy after a PE is sinus tachycaria (present in almost half
age of the leg is compromise, causing massive eema in the
of patients) (A–D). A heart rate greater than 100 beats per
leg. As the swelling continues, arterial malperfusion ensues,
minute with associate tachypnea in the setting of suspecte
leaing to severe ischemia (blue extremity), risking limb loss
PE shoul further raise concern. The classic ning on an
(C). DVT an as an extension, phlegmasia, both occur more
electrocariogram is the S1, Q3, T3 pattern, which consists of
commonly on the left. This is a result of the left iliac vein
a prominent S wave in lea I an a Q wave an inverte T
frequently being compresse by the right iliac artery (known
wave in lea III. This electrocariographic ning inicates
as May-Thurner synrome) (B). Unerlying malignancy is
right ventricular strain from a large PE, but it is not com-
the most common risk factor ientie for phlegmasia. The
monly present. A large PE will lea to an enlargement of the
fastest an safest metho of conrming the iagnosis is with
right ventricle causing the interventricular septum to eviate
uplex ultrasoun. CT angiography is not require unless
to the left. The right bunle branch stretches, leaing to a
history, exam, an ultrasoun nings are equivocal. Initial
right bunle branch block.
treatment is similar to that for an acute DVT, with some qual-
iers. More emphasis shoul be place on leg elevation. Due
11. E. The increase risk of the evelopment of VTE in sur-
to ui sequestration, patients may present with hypovole-
gical patients is multifactorial. Patients will have a perio of
mic shock an thus may nee massive volume resuscitation
activate coagulation, transient epression of brinolysis,
(D). The risks of limb loss, pulmonary embolism, postphle-
an temporary immobilization. In aition, many patients
bitic synrome, an mortality are all high (A). As such,
may have a central venous catheter in place an have con-
thrombolytic therapy has emerge as the treatment of choice.
comitant cariac isease, malignancy, or intrinsic hyperco-
Reference: Chinsakchai K, Ten Duis K, Moll FL, e Borst GJ.
agulable states, all of which increase a patient's chance of
Trens in management of phlegmasia cerulea olens. Vasc Endovas-
cular Surg. 011;45(1):5–14. a VTE. Trauma patients, in particular, have a high risk of
VTE. In trauma patients, spinal cor injury (os ratio, 8.33)
an fracture of the femur or tibia (os ratio, 4.8) were the
9. D. Enthusiasm for the aggressive use of IVC lters is injuries with the greatest risk of VTE (A–D). In one large
iminishing. Filters left in place for long perios of time prospective stuy, other risk factors in trauma patients on
can lea to complications, incluing migration of the lter, multivariate analysis inclue oler age, bloo transfusion,
fracturing of the legs of the lter, vena cava perforation, an an nee for surgery.
the increase risk of a recurrent DVT. In a prospective ran- Reference: Geerts WH, Coe KI, Jay RM, Chen E, Szalai JP. A
omize stuy of patients with DVT, the routine aition prospective stuy of venous thromboembolism after major trauma.
of an IVC lter i not improve mortality compare with N Engl J Med. 1994;331(4):1601–1606.
heparin an warfarin alone. Aitionally, PREPIC  trial has
also emonstrate an increase number of recurrent PEs in 12. E. This patient has classic signs of chronic venous insuf-
the lter group compare to the anticoagulation only group ciency. Venous stasis ulcers are classically locate at the
(3% versus 1.5%). Thus, the majority of IVC lters are now meial malleolus. The precise cause of venous stasis ulcers is
retrievable an shoul optimally be remove within 9 to 1 unclear but seems to be multifactorial. The increase venous
weeks. Consensus opinion from most societies is that the pressure from incompetent valves results in an impeance of
strongest inication for an IVC lter placement is a patient capillary ow, which leas to leukocyte trapping. These leu-
who evelops a venous thromboembolic event (VTE [DVT kocytes release oxygen free raicals an proteolytic enzymes
or PE]) who has a contrainication to anticoagulation (such that lea to local inammation. The increase venous pres-
as active gastrointestinal bleeing). Other inications are a sure also leas to the leakage of proteins such as brinogen,
new VTE that evelops in a patient who is alreay receiv- which act as a barricae to oxygen an growth factors neces-
ing therapeutic anticoagulation, or a patient with a VTE who sary for woun healing. First-line therapy for the treatment
is alreay receiving anticoagulation an in whom a major of venous stasis ulcers is compression therapy (A–D). The
hemorrhage evelops (B, C–E). Relative inications inclue workup for this patient shoul inclue a uplex ultrasoun
prophylaxis in high-risk populations (severe hea, pelvic, or scan of the venous system, specically looking for valvular
spinal cor trauma), massive PE treate with thrombolysis or incompetence of the eep, supercial, an perforating veins.
CHAPtEr 18 Vascular—Venous 273

A popular an effective compression banage is the Unna Meissner MH. What is effective care for varicose veins? Phlebol-
boot, which contains zinc oxie, glycerin, gelatin, an cala- ogy. 016;31(1 Suppl):80–87.
mine lotion. The boot shoul be wrappe starting at the foot, O’Donnell TF Jr, Passman MA, Marston WA, et al. Management
up to just below the knee. It can remain in place for as long of venous leg ulcers: clinical practice guielines of the Society for
Vascular Surgery an the American Venous Forum. J Vasc Surg.
as a week. It shoul not be use in the setting of an active
014;60( Suppl):3S–59S.
infection of the ulcer. In this situation, ebriement an anti-
biotics will be neee rst. 14. D. The right internal jugular vein is the preferre option
for central line placement because it is easily accessible an
13. C. A spectrum of chronic venous isorers, from vari-
has a lower risk of pneumothorax compare to a subclavian
cose veins to venous stasis ulcers, aficts 0% to 5% of the
line. It also has a straight course into the right atrium. In 70%
population. The unerlying etiology is incompetence of the
of iniviuals, the internal jugular vein lies anterolateral to
venous valves in either the eep, supercial (saphenous), or
the caroti artery. However, in some cases, it may lie irectly
perforator veins. Patients with chronic venous isease are
anterior or posterior to the caroti artery, increasing the risk
classie an treate base on the severity of their isease.
of a caroti artery cannulation. If the caroti artery is entere
The CEAP (clinical, etiologic, anatomic, an pathophysio-
with the probe neele (as evience by pulsatile bleeing),
logic) classication is use worlwie to stanarize this
the neele shoul be immeiately remove an pressure
evaluation. It is important when iscussing treatment with
shoul be hel for 10 minutes. If the artery is cannulate with
a patient that he or she unerstans that this is an incur-
a ilator or catheter, then the catheter shoul not be remove
able isease an that the goal of intervention is to minimize
blinly. This coul lea to a potential airway-threatening
symptoms an prevent recurrence. In general, supercial
hemorrhage. It is safer to remove the catheter in the oper-
incompetence is ealt with rst. In a patient with a nonheal-
ating room via irect surgical exposure, followe by suture
ing woun that has incompetent valves in all three venous
repair of the artery (A–C, E).
systems, (supercial, perforator, an eep) an is unrespon-
Reference: Kron I, Ailawai G. Cariovascular monitoring an
sive to compression therapy, the supercial venous incom- support. In: Fischer JE, e. Fischer's mastery of surgery. 6th e. Lippin-
petence is aresse rst by obliterating the saphenous vein cott Williams & Wilkins; 011:45–66.
along with compression therapy. This can be one via saphe-
nous vein stripping, foam sclerotherapy, or RFA. A recent 15. C. The patient has supercial venous thrombosis (SVT)
ranomize stuy emonstrate equal results with all three or thrombophlebitis. This entity is essentially a clotte sur-
approaches. That being sai, RFA is generally preferre an is face vein. A palpable cor is suggestive of the iagnosis, as
the current recommenation of the American Venous Forum, are accompanying pain an erythema. There are a few pit-
ue to its less invasive nature as compare with stripping falls in the iagnosis an management of SVT. Patients with
(A–E). Freeom of reux has been seen in 93% of patients at SVT may have a concomitant DVT (5%–40%); thus, a uplex
 years after ablation therapy. Primary venous insufciency ultrasoun scan of the venous system is essential. Secon,
is a recognize risk factor for the evelopment of DVT, an it SVT can easily be misiagnose as cellulitis, in which case
is important to rule this out before intervention to minimize antibiotics may be inappropriately prescribe an a uplex
treatment failure. If treating the supercial system is not suc- ultrasoun scan not obtaine. SVT is generally best man-
cessful, the next step is to treat the perforator incompetence. age with warm compresses an NSAIDs. IV antibiotics are
This is one via ultrasoun-guie sclerotherapy. There is reserve for septic thrombophlebitis, which is typically asso-
no surgical treatment that is reliably effective for eep sys- ciate with an intravenous line (E). Systemic anticoagulation
tem incompetence. A recent ranomize stuy conrme is reserve for a SVT that is near the eep system (A, B). If
the benet of early ablation of the saphenous vein to pro- anticoagulation is contrainicate, ligation of the saphenous
mote woun healing, as oppose to a longer (6 month) trial vein at the saphenofemoral junction is inicate for a saphe-
of compression therapy. nous vein SVT (D). Varicose veins cause stasis an thus pre-
References: Gohel MS, Heatley F, Liu X, et al. A ranomize trial ispose to SVT.
of early enovenous ablation in venous ulceration. N Engl J Med.
018;378():105–114.
Vascular—Access
LUIS FELIPE CABRERA VARGAS, MARK ARCHIE, AND CHRISTIAN DE VIRGILIO 19
ABSITE 99th Percentile High-Yields
I. Vascular access for hemoialysis (HD) in Patients with En-Stage Renal Disease (ESRD)
A. Catheters
1. Temporary noncuffe, nontunnele ialysis catheter
. Permanent (cuffe) or tunnele-ialysis catheter (TDC)
B. Autogenous surgical access (arteriovenous stula [AVF])
1. Preferre locations
a) Upper > lower extremities; istal > proximal
b) Nonominant > ominant arm
c) Cephalic > basilic > brachial veins > femoral vein of the thigh
. En-to-sie preferre to sie-to-sie anastomosis
C. Nonautogenous surgical access (arteriovenous graft [AVG])
1. Material: prosthetic (PTFE), biologic (bovine, human cryopreserve veins)
. Locations: upper arm, forearm, thigh

II. Comparing Vascular Access for HD

Catheters AVF AVG


1. Ideal for short-term hemodialysis, 1. Best overall option for long-term 1. Ideal for patients who are not
or as a bridging tool (AVF, graft or hemodialysis candidates for AVF
kidney transplantation) 2. Advantages: lower risk of infection 2. Advantages: widely available, short
2. Best avoided for long-term use and thrombosis, highest long-term wait time to access (does not need
3. To reduce infection, often coated patency (70%–80% at 1 year), greatest to mature), if thrombose relatively
with antibiotics, silver, or heparin blood Ěow volume during dialysis, easy to declot.
(reduces bacterial trapping) lowest need for reinterventions 3. Disadvantages: lower long-term
4. Advantages: immediate use, easily 3. Disadvantages: require at least 6 patency (about 50% at 1 year),
removed if infected weeks to mature, some never work higher thrombosis rates (due
5. Disadvantages: highest incidence to intimal hyperplasia), higher
of 1 year mortality, thrombotic infection rate (vs AVF)
complications, central vein stenosis,
and bacterial infections

III. Surgical Planning


A. Age (elerly may be less suitable for AVF), han-ominance (prefer nonominant arm), comorbi
conitions (iabetics may evelop severe raial/ulnar artery atherosclerosis), presence of pacemaker
(leas to stenoses of proximal veins)
B. Duplex ultrasoun vein an artery mapping
1. Veins (with soft tourniquet in place)
a) Cephalic, basilic, brachial; assess patency an size (ieally > 3 mm)
. Arteries
a) Brachial, raial, ulnar; assess size (ieally >  mm); severe meical calcication may be
unclampable, look for ow limiting plaque
275
276 PArt i Patient Care

C. Fistula rst initiative (orer of preference): raio-cephalic (Cimino) > brachio-cephalic > brachio-basilic
(may require  stages, rst construction, secon supercialization) > AVG
D. For istal stulas from raial or ulnar arteries, assess arterial ominance of the han with Allen test

IV. Complications
A. Excessive bleeing from ialysis puncture site
1. Nee to rule out stenosis of outow vein or central venous system (high venous pressure)
. Rule out use of anticoagulants
B. Aneurysm of AVF (may be ue to repeat neele trauma or central stenosis)
1. Most are benign an can be observe
. Repair if rapily expaning, overlying skin thinning, skin ulceration, or excessive bleeing
3. Prior to repair, rule out stenosis of outow vein or central venous system as cause
C. Pseuoaneurysm of AVG (ue to repeat neele trauma or infection)
1. Small or uninfecte ones can be observe
a) Repair if large
. Resect AVG if infecte
D. Thrombosis
E. Steal
1. Stage I: asymptomatic retrograe iastolic ow (US ning alone)
. Stage II: pain on exertion an/or uring HD
3. Stage III: pain at rest
4. Stage IV: ulceration/necrosis/gangrene
F. Ischemic monomelic neuropathy (IMN)
G. High-output cariac failure

Fig. 19.1
V.Surgical Treatments of Steal Syndrome
MILLER Procedure DRIL Procedure
(Minimally Invasive PAI Procedure (Distal
Limited Ligation (Proximalization Revascularization RUDI Procedure RUPI Procedure
Access Banding Endoluminal- of the Arterial with Interval (Revision Using (Revision Using
Ligation Procedure Assisted Revision) InĚow) Ligation) Distal InĚow) Proximal InĚow)
Complete Reduction of the Banding of the vascular Enhances access Cons: complex Ideal for brachial Uses smaller caliber
resolution of access Ěow for access with a Ěow surgery AV access patients graft to increase
steal syndrome high Ěow- nonresorbable suture Ideal for low Ěow- Longer operative with high Ěow resistance
Loss of the associated steal guided with a 4–5 mm associated steal time induced steal or
vascular access syndrome dilatation balloon syndrome Need to harvest cardiac failure
Con: need new Best results if Controlled reduction in suitable vein Creates bypass
AVF with intraoperative the vessel diameter Hand perfusion from distal artery
new risk of Ěow Only for high Ěow- reliant on a bypass to ęstula (while
developing steal measurements associated steal graft ligating original
syndrome are used syndrome anastomosis)
Con: banding
a low-Ěow
vascular access
will lead to
ineĜcient
dialysis and
thrombosis
CHAPtEr 19 Vascular—Access
277
278 PArt i Patient Care

Questions
1. A 63-year-ol man with en-stage renal isease on C. Plicating the stula may help prevent another
ialysis via a left upper arm arteriovenous graft episoe of heart failure
(AVG) presents to the emergency epartment D. She shoul unergo a istal revascularization
with what he escribes as “pulsatile bleeing” an interval ligation
from the area of where ialysis was performe E. The stula shoul be converte to a graft
the ay before. He states the bleeing stoppe
after wrapping his arm. On physical exam, his 4. A 65-year-ol woman unergoes creation of
temperature is 100.5°C an his heart rate is 100. an upper arm arteriovenous (AV) graft for
The AVG has a thrill. Overlying the AVG, at the hemoialysis in the left arm using a 6-mm
site of the blee, there is a small black eschar with polytetrauoroethylene graft. Three weeks later,

(WBC) count is 1,000 × 103/mm3 an a uplex


a small aneurysmal swelling. His white bloo cell the patient reports marke coolness, pallor, an
numbness in the han as well as pain in the han
ultrasoun emonstrates a pseuoaneurysm at at rest. Motor exam is normal. On examination,
the site of eschar. What is the next best step in there is no palpable pulse at the raial artery an
management? only a monophasic Doppler signal. Upon graft
A. Discharge the patient with oral antibiotics compression, a pulse becomes palpable an the
B. Amit the patient for intravenous (IV) signal becomes biphasic. Which of the following
antibiotics is the best management option?
C. Obtain a stulogram A. Distal revascularization an interval ligation
D. Take patient to the operating room for graft of the brachial artery
excision B. Ligation of the AV graft an placing the upper
E. Place a covere stent across the arm graft in same arm
pseuoaneurysm C. Ligation of the AV graft an placing the stula
in the ominant arm
2. A 45-year-ol female returns to the emergency D. Baning of the AV graft ajacent to the arterial
epartment 8 hours after having unergone anastomosis
a left upper arm arteriovenous graft (AVG) E. Baning of the AV graft ajacent to the venous
complaining of severe forearm pain. On anastomosis
examination, the forearm is soft an supple.
The han appears to be pink, with a normal 5. A 45-year-ol male evelops progressive
temperature, an a 1+ raial pulse. There is a en-stage renal isease seconary to severe
goo thrill in the graft. She has both motor an hypertension. His glomerular ltration rate is 19
sensory ecits in the han in both the meian mL/min, an ialysis is anticipate within the
an ulnar nerve istributions. The next step in the next 6 months. He is right-hane an appears to
management is: have goo veins in both arms an normal pulses.
A. Distal revascularization with interval ligation Which of the following is the best management
B. Observation plan for ialysis access?
C. Ligation of the graft A. Delay access until about a month before
D. Obtain nerve conuction stuies anticipate ialysis
E. Forearm fasciotomy B. Procee with left raiocephalic AVF (Cimino
stula)
3. A 65-year-ol female with en-stage renal isease C. Procee with left brachiocephalic AVF
presents with recurrent episoes of congestive D. Procee with left brachiobasilic AVF
heart failure. She is currently ialyze via a left arm E. Procee with right raiocephalic AVF
brachiocephalic arteriovenous stula (AVF). Upon
compression of the stula, her heart rate ecreases
from 80 to 60 beats per minute, an bloo pressure
increases from 10/70 to 140/80 mm-Hg. Which of
the following is true about this conition?
A. The stula shoul be ligate
B. It is unlikely that the stula is contributing to
the patient’s heart failure
CHAPtEr 19 Vascular—Access 279

6. A 45-year-ol female with en-stage renal 9. A 50-year-ol male with longstaning history
isease presents with recent onset of heaaches, of hemoialysis via a left brachiocephalic
hoarseness of her voice, an bilateral arm arteriovenous stula (AVF) presents with an
swelling for  ays. She has a history of aneurysm within the miportion of the AVF. He
multiple proceures in both arms an legs reports that there has recently been excessive
for hemoialysis access. Most recently, she bleeing when the neeles have been pulle
unerwent an arteriovenous graft (AVG) in her out. On physical examination, the aneurysm is
right upper arm  weeks earlier. On examination about 3 cm in size. The overlying skin appears
her neck appears to be engorge an her face supple, without ulceration. The next step in the
swollen. There are numerous visible veins on management consists of:
her chest wall. Which of the following is the best A. Fistulogram
management option? B. Resection/plication of the aneurysm
A. Ligation of the AVG C. Replacement of stula with an AV graft
B. Plication of the AVG D. Ligation of the stula
C. Attempt venoplasty of superior vena cava E. Observation
(SVC)
D. Place stent in SVC 10. In comparing the three moalities use
E. Move AVG to right arm for hemoialysis (central venous catheter
[permacath], arteriovenous [AV] graft, an AV
7. A 45-year-ol male with long-staning iabetes stula), which of the following is true?
an progressive en-stage renal failure presents A. They are equal in terms of 1-year patient
to the emergency epartment (ED) with mortality
progressive shortness of breath, vague abominal B. The primary patency for AV stula an AV
pain, an marke leg eema. Laboratory values graft is similar
are remarkable for metabolic aciosis an C. The seconary patency for AV stula an AV
azotemia but a normal white bloo cell count. graft is similar
Dialysis is urgently neee. Dialysis access woul D. Time to maturation for AV stulas an grafts is
be best institute via: similar
A. Right internal jugular vein tunnele, cuffe E. A permacath is the best ialysis option in the
catheter elerly
B. Right internal jugular vein nontunnele,
uncuffe catheter 11. An intubate patient in the OR evelops an air
C. Left internal jugular vein, tunnele, cuffe embolism after central venous catheter insertion.
catheter Which of the following murmurs are associate
D. Right subclavian vein uncuffe, nontunnele with this conition?
catheter A. Austin-Flint murmur
E. Right femoral vein cuffe tunnele catheter B. Carey Coombs murmur
C. Means-Lerman scratch murmur
8. A left internal jugular vein central line is place. D. Still murmur
Fifteen minutes later, the patient is hypotensive. E. Millwheel murmur
Distene neck veins are note. Breath souns
are clear bilaterally. What is the most likely cause
of the patient’s hypotension?
A. Perforate right atrium
B. Perforate subclavian vein
C. Perforate subclavian artery
D. Tension pneumothorax
E. Perforate right ventricle
280 PArt i Patient Care

Answers
1. D. This patient has a pseuoaneurysm of his AVG that to support hemoialysis is greater than 400 to 500 cc/min.
appears to be infecte, given his fever an elevate WBC. However, when the ow rate excees 000 cc/min or 30%
This requires excision of his graft. Though the bleeing has of the cariac output, there is a risk of high-output cariac
stoppe, an infecte pseuoaneurysm is inherently unsta- failure. These patients, an those with clinically evient epi-
ble an will likely blee again, which coul be catastrophic. soes of cariac failure, shoul unergo intervention aime
Thus, antibiotics alone woul not be aequate (A, B). A s- at reucing ow rates. Surgical plication (narrowing the vein
tulogram is also not inicate as the graft is infecte (C). For just beyon the anastomosis to the artery by suturing or
a large, noninfecte pseuoaneurysm, a covere stent is a baning) reuces the ow rate an can partially reverse the
potential treatment option (E). hemoynamic changes an prevent future episoes of heart
Reference: Muoni A, Cornacchiari M, Gallieni M, et al. Aneu- failure. If heart failure continues to occur after an appropri-
rysms an pseuoaneurysms in ialysis access. Clin Kidney J. ate reuction in ow rates, eventual ligation of the stula is
015;8(4):363–367. inicate (A). Distal revascularization an interval ligation
(DRIL) is use to treat steal synrome, causing ischemic
2. C. This patient has nings of ischemic monomelic neu- steal synrome istal to the stula. The proceure increases
ropathy (IMN), a rare complication after vascular access sur- resistance to the stula an ecreases resistance to the istal
gery. The incience of this complication is less than 1% an extremity but may not effectively reuce stula ow in the
is more common in female an iabetic patients. IMN results setting of cariac failure (D). Converting a native stula to
in pain, numbness (in ngers), paresthesia, an motor weak- a graft woul not help because the large iameter of a graft
ness (intrinsic han muscles) usually shortly after surgery. It woul maintain high ow rates (E).
can be istinguishe from steal synrome by its faster onset References: MacRae JM, Levin A, Belenkie I. The cariovascular
an mil or absent signs of clinical ischemia. The patho- effects of arteriovenous stulas in chronic kiney isease: a cause for
physiology of IMN is poorly unerstoo but is thought to be concern?: cariovascular effects of arteriovenous stulas. Semin Dial.
cause by a loss of bloo ow from istal nerve tissue lea- 006;19(5):349–35.
ing to istal neuropathies. IMN is a clinical iagnosis an High arteriovenous (AV) access ow an cariac complications.
electromyography an nerve conuction stuies are only NKF Task Force on Cariovascular Disease, America Journal of Kidney
neee when the clinical neurologic exam is equivocal (D). Disease, 3(5).
The recommene treatment for IMN is ligation of the newly
create access, which may lea to resolution of neuropathy 4. A. A patient presenting with marke coolness, pallor,
in some patients (B,C). Distal revascularization an interval pain at rest, an han numbness following an AV graft
ligation(DRIL) can be utilize in steal synrome, but not in shoul be suspecte of having steal synrome. Ischemic
IMN (A). This patient shoul not unergo forearm fasciot- steal synrome occurs in approximately % to 4% of patients
omy, as she has a soft an supple forearm, making compart- unergoing AV access for hemoialysis. Risk factors for steal
ment synrome very unlikely (E). synrome inclue females, iabetes, age >60, an use of the
References: Datta S, Mahal S, Govinarajan R. Ischemic monome- brachial artery. Proximal stulas have a higher risk of evel-
lic neuropathy after arteriovenous stula surgery: clinical features, oping steal synrome, while istal wrist stulas (Cimino s-
electroiagnostic nings, an treatment. Cureus. 019;11(7):e5191. tulas) have a very low risk. AV grafts also have a greater risk
Thimmisetty RK, Peavally S, Rossi NF, Fernanes JAM, Fixley of steal compare with native AV stulas (B). This is likely
J. Ischemic monomelic neuropathy: iagnosis, pathophysiology, an ue to the fact that the large iameter of the graft creates
management. Kidney Int Rep. 017;(1):76–79. a low-resistance be. In aition, steal seconary to grafts
tens to occur early after the access placement, whereas steal
3. C. Bloo ow through an AVF is essentially a left-to-right after native AV stulas has a bimoal istribution, with some
shunt, an a portion of the cariac output is stolen by the presenting early an others late after the native vein has
stula (B). Although there is no change in peripheral oxy- unergone ilation with lowere resistance. Some egree of
gen consumption after stula placement, there is a rop in physiologic steal occurs in every patient with an AV stula,
peripheral vascular resistance (PVR). Consequently, a com- but only a small minority manifests severe symptoms. The
pensatory increase in cariac output occurs. The increase in steal synrome is cause by a iversion of bloo ow from
venous return increases cariac preloa an causes rises in the anastomose artery to the low-resistance vein. In ai-
atrial natriuretic peptie (ANP) an brain natriuretic pep- tion, the low-resistance venous anastomosis leas to bloo
tie (BNP). The ecrease in afterloa results in a ecrease owing in a retrograe fashion from the istal circulation
in alosterone an renin levels. This subsequently leas to a into the stula. Mil steal can be manage conservatively
ecrease in afterloa as well as suppression of the renin-al- with exercise. More severe symptoms require intervention.
osterone-angiotensin system, which promotes natriuresis. Although ligation of the AV graft woul have a great chance
Compressing the stula increases PVR an afterloa, leaing of resolving the steal synrome, the patient will require a
to a ecrease in pulse rate an an increase in bloo pressure new access an will again be at risk of eveloping steal (C).
(Nicolaoni-Branham sign). Patients with higher stula ow Several options exist for the management. The most effec-
will exhibit greater hemoynamic changes with stula occlu- tive treatment that maintains stula function is istal revas-
sion. Objectively, the minimum stula ow rate require cularization an interval ligation. The isavantage of this
CHAPtEr 19 Vascular—Access 281

proceure is that it requires creating a new bypass, usually References: Disbrow DE, Cull DL, Carsten CG 3r, Yang SK,
with a saphenous vein, from the native artery proximal to Johnson BL, Keahey GP. Comparison of arteriovenous stulas an
the AV graft to the artery istal to it, with interval ligation arteriovenous grafts in patients with favorable vascular anatomy
of the native artery just proximal to the istal anastomosis. an equivalent access to health care: is a reappraisal of the Fistula
First Initiative inicate? J Am Coll Surg. 013;16(4):679–685; is-
Baning or plicating of the AV graft, ajacent to the arterial
cussion 685–686.
anastomosis, serves to increase the resistance in the graft an
Hakaim AG, Nalbanian M, Scott T. Superior maturation an
reuce steal. The primary isavantage of this approach (for patency of primary brachiocephalic an transpose basilic vein arterio-
grafts) is that inaequate baning leas to persistent steal, an venous stulae in patients with iabetes. J Vasc Surg. 1998;7(1):154–157.
excessive baning causes graft thrombosis (stulas less likely [No authors liste]. NKF-K/DOQI clinical practice guielines for
to thrombose) (D, E). Baning or plication is a more attractive vascular access. Am J Kidney Dis. 006;48(Suppl. 1):S7–S409.
option for steal in an autologous AV stula, such as a brachial
artery cephalic vein stula, because the vein is more resistant 6. D. The patient is presenting with superior vena cava
to thrombosis. This is not yet the stanar approach, however. (SVC) synrome with bilateral arm, neck, an face swelling
References: Walz P, Laowski JS, Hines A. Distal revasculariza- an hoarseness of the voice. The patient likely has a preex-
tion an interval ligation (DRIL) proceure for the treatment of isch- isting central vein stenosis (in the SVC). A high proportion of
emic steal synrome after arm arteriovenous stula. Ann Vasc Surg. patients with en-stage renal isease have central vein stenosis
007;1(4):468–473. (5%–40%) ue to prior central venous access. These stenoses
Yaghoubian A, e Virgilio C. Plication as primary treatment of steal are often asymptomatic, an if SVC synrome oes evelop,
synrome in arteriovenous stulas. Ann Vasc Surg. 009;3(1):103–107.
it is usually insiious in onset. However, placement of an
Yu SH, Cook PR, Canty TG, McGinn RF, Taft PM, Hye RJ. Hemo-
ialysis-relate steal synrome: preictive factors an response to
upper arm AVG access creates a suen, massive increase in
treatment with the istal revascularization-interval ligation proce- venous return that cannot be accommoate by the steno-
ure. Ann Vasc Surg. 008;():10–14. sis, leaing to abrupt venous congestion (E). Central venous
stenosis complicates hemoialysis access because it impairs
5. B. When permanent hemoialysis access is neee, the venous stula outow an can reuce ow rates an reuce
nonominant arm (E) shoul be consiere rst in orer to the likelihoo of maturation in stulas. Further, when access
mitigate the effects of potentially evastating complications, is place ipsilateral to a stenotic lesion, there is a high like-
incluing severe steal synrome, limb ischemia, ischemic lihoo of symptoms ue to the increase venous congestion
monomelic neuropathy, an nerve injury. Once the sie is combine with high venous resistance. Arteriovenous grafts
etermine, the type of AVF must be consiere. Raioce- are more likely to cause symptoms than stulas, an upper
phalic stulas shoul generally be place rst (assuming arm access is more likely to cause symptoms than forearm
aequate artery an vein) because subsequent thrombosis access. When central stenosis is suspecte, either from history
will not preclue the placement of a brachiocephalic or bra- or symptoms, a central venogram shoul be performe to
chiobasilic stula more proximally in the arm. Aitionally, iagnose the lesion. Concomitant enovascular venoplasty is
raiocephalic stulas may cause ilation of the proximal arm a reasonable option an has a high rate of success. However,
veins, allowing higher success rates of more proximal stu- rst-line treatment is now enovascular stenting of the SVC
las in the future. Raiocephalic stulas also rarely require a (C). This is appropriate for both benign an malignant cases
secon-stage supercialization or transposition proceure of SVC synrome. Ligation or plication of the graft is not ini-
because the forearm cephalic vein is close enough to the skin cate because this estroys the access an oes not aress
to be use upon maturation. If raiocephalic is not possible the unerlying pathology (A, B). Open SVC repair via ster-
or has faile, a brachiocephalic shoul be consiere next (C). notomy for a benign lesion is overly invasive an unnecessary
Brachiocephalic stulas allow stulas to form on the orsal given the high initial success rates of enovascular treatment.
surface of the upper arm an allow easier cannulation an References: Jones RG, Willis AP, Jones C, McCafferty IJ, Riley
use uring hemoialysis. Further, epening on boy hab- PL. Long-term results of stent-graft placement to treat central venous
itus, brachiocephalic stulas may also not require a secon stenosis an occlusion in hemoialysis patients with arteriovenous
stulas. J Vasc Interv Radiol. 011;(9):140–145.
stage to supercialize the stula close to the skin. The thir
Rizvi AZ, Kalra M, Bjarnason H, Bower TC, Schleck C, Gloviczki
choice for autogenous stula is the brachiobasilic stula.
P. Benign superior vena cava synrome: stenting is now the rst line
Since the basilic vein is eep, it requires supercialization of of treatment. J Vasc Surg. 008;47():37–380.
the vein. Many surgeons perform this in two stages so as to Trerotola SO, Kothari S, Sammarco TE, Chittams JL. Central venous
allow the vein to mature before supercialization (D). Mat- stenosis is more often symptomatic in hemoialysis patients with
uration of a stula typically requires at least 6 weeks an grafts compare with stulas. J Vasc Interv Radiol. 015;6():40–46.
may require aitional interventions. Waiting until 1 month
before ialysis will result in placement of a temporary ial- 7. A. When hemoialysis is urgently neee, temporary
ysis catheter, which carries high mortality risks (A). Despite rapi vascular access must be establishe with a catheter
the avantages, the raiocephalic stula has a higher early that will support high ow (generally >400 cc/min) via 
failure or nonmaturation rate an may not be a goo option lumens. If long-term ialysis is anticipate, as in this patient,
in iabetics ue to meial calcinosis within the raial artery. a tunnele, cuffe hemoialysis catheter, or permacath, is
Further, when a patient is alreay hemoialysis epenent preferre (B) an place into a central vein an exits the
via tunnele catheter, there is ongoing ebate about whether skin at least 10 cm away via a subcutaneous tract. Tunnele
the ability to rapily cannulate a graft (∼ weeks) shifts the catheters are reay to use immeiately an are less prone to
preferences towar initial graft placement rather than stula infection than a nontunnele, noncuffe catheter (Quinton
rst. A forearm loop graft also has the avantage of ilating catheter). Quinton catheters are preferre in patients nee-
the basilic an upper cephalic veins for future stula creation. ing urgent ialysis for a short term, or for those with sepsis
282 PArt i Patient Care

(as they are remove rapily). The right internal jugular vein pseuo aneurysm. If no lesion is seen on the stulogram, a
is the rst choice because it is the most irect route to the central venogram shoul be performe to rule out a central
right atrium. Left-sie placement is less preferable because stenosis as a cause of high outow pressures. After treatment
it jeoparizes venous patency for future permanent access of the venous stenoses, bleeing may resolve because the
in the left arm (as most patients are right-han ominant). abnormally high pressures within the stula return to nor-
Left-sie catheters also result in lower catheter bloo ow mal. Thinne/atrophic skin, translucent skin, ulceration, sus-
rates an increase the risk of stenosis/thrombosis ue to pecte infection, intraluminal thrombus, high-output cariac
the longer an more tortuous length of contact with central failure, steal synrome, or spontaneous bleeing from the
vein sie-wall (C). The subclavian position is associate with stula prompts consieration for revision by resection an
higher rates of complications (D), namely central vein ste- plication or reanastomosis with a healthy vein (B). The size
nosis an pneumothorax, an in some stuies has a higher of the aneurysm is not an inicator for revision. If no healthy
risk of infection when compare with internal jugular cathe- vein is available, graft implantation is an option (C). If out-
ters. The femoral position carries the highest risk of infection, ow cannot be salvage, the access may require ligation (D).
which is a signicant cause of mortality in patients with tem- Reference: Cronenwett JL, Wayne Johnston K. Rutherford's vas-
porary access catheters (E). Femoral lines may compromise cular surgery. 7th e. New York, NY: Sauners/Elsevier; 010.
a future kiney transplant because it may lea to proximal
iliac vein stenosis/thrombosis. 10. C. Fistulas are superior to grafts, which are superior to
Reference: [No authors liste]. NKF-K/DOQI clinical practice catheters in terms of patient survival, mainly because of the
guielines for vascular access. Am J Kidney Dis. 006;48(Suppl. 1): infection risks of prosthetic material (A–E). Diabetics have
S7–S409. an exaggerate increase in mortality ue to their epresse
immune systems. Interestingly, espite the risk of high-out-
8. A. Clinical signs of cariac tamponae inclue hypo- put cariac failure associate with stula an graft, patients
tension, istene neck veins, an mufe or istant heart with tunnele catheters also have the highest risk of cari-
souns (Beck tria). This patient exhibits two of these signs ac-relate mortality. When comparing patency, stulas are
after an invasive proceure of the chest an likely evel- known to have higher primary patency (intervention-free
ope cariac tamponae as a result of perforation of the patency of 85% at 1 year, 50% at 5 years) compare to grafts
right atrium. Tamponae cause by central venous catheter (60% at 1 year, 10% at 5 years) (B). However, stulas have a
placement is a known complication resulting from puncture higher rate of primary failure (nonmaturation or early throm-
by the wire, introucer, or the catheter itself. Perforation of bosis) of up to 40%. Furthermore, when comparing seconary
the right atrium more often occurs because it has a thinner patency (patency with interventions to maintain or reestablish
wall compare to the right ventricle (E). Placing the catheter ow), stulas an grafts are similar. Grafts o not require mat-
tip at the right tracheobronchial angle helps avoi placing uration because their lumen iameter oes not change (D).
the catheter tip in the right atrium. A perforate subclavian However, healing time of at least 10 ays must be observe
artery or vein woul likely lea to hemothorax rather than after graft placement before cannulation to avoi massive
pericarial tamponae (B, C). A tension pneumothorax is pseuoaneurysm formation. Fistulas require at least 6 weeks
a known complication of line placement an may result in for maturation, uring which time the outow vein uner-
hypotension an istene neck veins, but breath souns goes remoeling seconary to increase ow resulting in an
woul not be clear bilaterally (D). increase in iameter an further increase in ow. Fistulas are
References: Barton JJ, Vanecko R, Gross M. Perforation of right eeme mature if they meet the rule of sixes: at 6 weeks, they
atrium an resultant cariac tamponae: a complication of cath- must be 6 mm in iameter, less than 6 mm from skin surface,
eterization to measure central venous pressure. Obstet Gynecol. support 600 mL/min ow (although a minimum of 400 mL/
1968;3(4):556–560.
min is aequate), an have a 6-inch straight segment for use.
Darling JC, Newell SJ, Mohamee O, Uzun O, Cullinane CJ, Dear
References: [No authors liste]. NKF-K/DOQI clinical practice
PR. Central venous catheter tip in the right atrium: a risk factor for
guielines for vascular access. Am J Kidney Dis. 006;48(Suppl. 1):
neonatal cariac tamponae. J Perinatol. 001;1(7):461–464.
S7–S409.
Hunt R, Hunter TB. Cariac tamponae an eath from perfora-
Lok CE, Sontrop JM, Tomlinson G, et al. Cumulative patency
tion of the right atrium by a central venous catheter. AJR Am J Roent-
of contemporary stulas versus grafts (000–010). Clin J Am Soc
genol. 1988;151(6):150.
Nephrol. 013;8(5):810–818.

9. A. AVF can eventually unergo aneurysmal egeneration 11. E. Intubate patients with an air embolus may have
over time, an intervention is require to prevent rupture
an abrupt increase in en-tial CO followe by a ecrease
an exsanguination (E). High outow resistance is a common
in en-tial CO an hypotension, an auscultation may
cause of aneurysm formation an must be rule out by a s-
reveal a “millwheel” murmur. This is often escribe as a
tulogram. Repeate neele cannulation can cause stenosis,
lou churning soun. An Austin-Flint murmur is associate
resulting in higher pressures istal to the lesion an subse-
with aortic insufciency an is a mi-iastolic rumble hear
quent aneurysm formation. Alternatively, repeate neele
best at the apex (A). Carey Coombs murmur is also a mi-i-
cannulation can also lea to aneurysmal egeneration of the
astolic rumble that is associate with rheumatic fever (B).
vein at the stick site. Therefore, cannulation must be avoie
Means-Lerman scratch murmur souns similar to a pericarial
in areas unergoing aneurysmal change. A stulogram is
rub an may be hear in patients with hyperthyroiism (C).
iagnostic of the stenotic lesion an potentially therapeutic
Still murmur is associate with a small ventral septal efect an
via venoplasty with or without stent placement. Further, the
is escribe as a vibratory systolic ejection rumble (D).
stulogram will also help istinguish between a true an
Transplant
JOSEPH HADAYA, AREG GRIGORIAN, AND CHRISTIAN DE VIRGILIO 20
ABSITE 99th Percentile High-Yields
I. Type of Transplant Rejection
A. Hyperacute: occurs in minutes to hours after transplantation (type- hypersensitivity)
1. Due to the presence of preforme or natural antiboies against major bloo group (ABO) or HLA
antigen (sensitization from prior transplants, pregnancy, transfusions)
. Complement an coagulation cascae is activate causing graft thrombosis an ischemia
3. Requires prompt removal of transplante organ
4. Kiney, heart, pancreas, an lung allografts all are susceptible to hyperacute rejection; however, liver
grafts resist this process, so ABO compatibility is not essential for liver transplantation
B. Acute: occurs in ays to months
1. Cause by cellular (macrophages an T-lymphocytes) or humoral (antiboy-meiate) response an
typically requires biopsy for iagnosis
. Treate with immunosuppressants, sterois, antithymocyte globulin
C. Chronic: occurs in months to years (major cause of graft failure an mortality)
1. Cause by cellular (cytotoxic T-lymphocyte, helper T cell) an antiboy-meiate reactions
. Graual process resulting in brosis an progressive graft ysfunction
3. Treate by increasing immunosuppression, though usually requires retransplantation

II. Liver Transplantation


A. Inications: en-stage liver isease seconary to alcohol (most common, replace hepatitis C),
nonalcoholic steatohepatitis, primary hepatic malignancy, cholangiocarcinoma, fulminant hepatic failure;
most common inication in peiatrics: biliary atresia
1. Milan criteria for hepatocellular carcinoma: single tumor less than 5 cm OR 3 tumors less than 3 cm
each
B. Moel for en-stage liver isease (MELD): utilizes creatinine, INR, bilirubin, an soium
1. Originally evelope for risk assessment for TIPS, now accepte as the score to prioritize organ
allocation for liver transplantation
. Score is 0–40; MELD > 15 is inication for liver transplant
3. Consiere a superior scoring system to Chils-Pugh because all parameters are objective
C. Operative consierations: for living onors, right lobe is utilize in aults, left lateral lobe utilize in
chilren (size comparable)
D. Postoperative complications
1. Infection is the most common cause of eath, accounting for over 50% of mortality
. Bile leak: most common complication, 10% an 30% incience, may present with abominal pain,
bilious rainage, fever; early an large-volume leaks manage surgically, late or small volume leaks,
or strictures, can be manage by biliary stenting or rainage
3. Hepatic artery thrombosis: most common vascular complication, high mortality an graft loss rate
(>50%)

283
284 PArt i Patient Care

a) Early thrombosis presents with transaminitis, hepatic failure, bile leak (ue to breakown of
biliary anastomosis), or primary nonfunction
b) Late thrombosis presents as biliary stricture an/or abscesses
c) Doppler ultrasoun rst-line, may be conrme with angiography, CT scan, or surgical
exploration
) If ientie early (<4 hours), consier thrombectomy an revascularization, but may require
retransplantation
4. Portal vein thrombosis (PVT): rare, early presents as abominal pain, late presents as ascites an
gastrointestinal blee, can result in severe injury or graft loss, treate with thrombectomy if etecte
early; complete or partial recanalization of PVT is associate with better survival rates an, therefore,
anticoagulation is recommene in all patients
a) If fever an rising leukocytosis, consier pylephlebitis (infecte thrombosis); a IV antibiotics

III. Kiney Transplantation


A. Inications: Shoul be referre for transplantation evaluation when estimate GFR <30 or when on
chronic ialysis
B. Operation: graft place in iliac fossa, extraperitoneal (close to blaer an avois possible peritoneal
contamination); most common arterial an venous anastomoses to external iliac artery an vein; orer of
anastomoses: vein rst, artery secon, ureter thir
C. Complications: urine output surrogate for graft function
1. Oliguria: most common cause of elaye graft function is acute tubular necrosis, presents with
graual ecline in urine output, rst step with all cases of postop oliguria is to check for kinking of
the foley catheter followe by Doppler ultrasoun to rule out renal arterial occlusion
. Urine leak: common, presents with fever, pain, swelling, oliguria; evaluate with ultrasoun an
aspiration of ui (compare ui creatinine to serum level) or nuclear renography
3. Lymphocele: most common cause of ui aroun the transplante kiney; often asymptomatic
an foun incientally; can present with pain; if signicant compression of ureter, can reuce urine
output an elevate creatinine levels
4. Renal artery stenosis: presents as graft ysfunction, iagnose on ultrasoun as ow acceleration at
level of stenosis, can be treate with angioplasty an stent
5. Arterial thrombosis: rare (<1%), acutely presents as low or no urine output, evaluate with
ultrasoun; if graft thrombose, kiney salvage possible if segmental vessels involve an successful
thrombectomy, otherwise may nee graft nephrectomy.
D. Living onor: improve posttransplant outcomes, lower elaye graft ysfunction rates, reuce wait
times an time spent on ialysis
E. Simultaneous pancreas an kiney (SPK) transplant or pancreas after kiney (PAK) consiere for those
with en-stage renal isease an iabetes mellitus; improves glucose control, iabetic complications
(nephropathy, neuropathy, gastroparesis, retinopathy), an quality of life

IV. Most Common Posttransplant Malignancies


A. Most common skin cancer: squamous cell carcinoma
B. Kaposi sarcoma: cause by HHV-8, typically presents as cutaneous lesions on legs or mucosal lesions,
treate by reucing immunosuppression
C. Posttransplant lymphoproliferative isorer: proliferation of monoclonal B-cells, result of Epstein-
Barr virus (EBV), check serology; more commonly evelops in chilren; usually presents with vague
abominal pain or obstructive symptoms
1. First-line treatment is to reuce the immunosuppressive regimen; secon-line is rituximab (anti-CD0
monoclonal antiboy); this can be followe by aing on chemotherapeutic regimens; surgery oes
not play a role
CHAPtEr 20 Transplant 285

Fig. 20.1 Orer of Anastomosis in Kiney Transplantation.


286 PArt i Patient Care

Questions
1. A 45-year-ol female unergoes orthotopic 4. A 39-year-ol female is unergoing kiney
liver transplantation for en-stage liver isease transplant. Shortly after performing the arterial
seconary to hepatitis C. Eighteen hours after anastomosis, the surgeon notes that the onor
surgery, she remains intubate an seate, kiney appears soft, abby, mottle, an
has require 1 unit of packe re bloo cells eematous. The patient’s heart rate is 136 beats
since surgery, an a norepinephrine rip to per minute an bloo pressure is 90/60 mm-Hg.
keep her mean arterial pressure >65 mmHg. Her Which of the following is true?
transaminases have ouble since return from A. This is a T-cell meiate response
the operating room an her total bilirubin level B. The patient shoul be starte on pressors an
remains at 10 mg/L. There is minimal output the operation complete
from her surgical rains. What is the next best C. The onor kiney shoul be immeiately
step? remove without further workup
A. Immeiate reexploration an listing for D. Lymphokines are involve in this process
retransplantation E. This complication occurs more commonly in
B. Repeat complete bloo count, liver function liver transplants than with kiney transplants
tests, an prothrombin time/INR in 4 hours
C. Ultrasoun an oppler stuy of graft 5. A 46-year-ol male with en-stage renal isease
D. CT scan of abomen an pelvis with IV seconary to iabetes arrives at clinic to iscuss
contrast his placement in the kiney transplant list. His
E. Flui resuscitation an broa-spectrum panel reactive antiboy (PRA) score is 85%. He
antibiotics ha a faile kiney transplant 5 years ago. Which
of the following is true?
2. A 41-year-ol male with en-stage renal isease A. He has a low risk of rejection
seconary to iabetes is unergoing routine renal B. Given his high PRA, he will be given priority
ultrasoun 1 month after kiney transplantation. on the transplant list
He has a 4-cm ui collection next to the onor C. He will lose points in the kiney allocation
kiney. He has no complaints an he is making algorithm because he ha a previous kiney
aequate urine. Which of the following is the best transplant
next step? D. He will experience a shorter wait time
A. Observation compare to a similar patient with a lower
B. Ultrasoun-guie aspiration for culture an PRA
creatinine E. PRA is calculate using nationally poole ata
C. CT scan
D. Internal rainage in the OR 6. Which of the following patients with
E. External pigtail catheter rainage hepatocellular carcinoma is eligible for liver
transplantation?
3. Which of the following is true regaring A. Single 3-cm tumor in segment  with regional
posttransplant lymphoproliferative isorer lymphaenopathy
(PTLD)? B. Single 6-cm tumor in segment 4 with no
A. It is usually of monoclonal T-cell origin regional lymphaenopathy
B. It occurs more commonly following C. Single -cm tumor in segment 5 with vascular
renal transplantation compare to heart invasion
transplantation D. 1-cm, -cm, an .5-cm tumors in segments 3
C. The risk of eveloping PTLD is lowest in the an 4 with no evience of lymphaenopathy
rst year following transplant E. 1-cm an 3.5-cm tumors both in segment 4
D. Epstein-Barr virus (EBV)-negative patients are with no lymphaenopathy or istant isease
at a lower risk than EBV-positive patients
E. Cytomegalovirus (CMV)-negative patients are
at higher risk once they seroconvert following
transplant
CHAPtEr 20 Transplant 287

7. The most clinically important viral infection in 11. A 35-year-ol brain ea trauma victim is
transplant recipients is: being consiere for kiney onation. Which
A. Varicella-zoster of the following onor conitions woul be a
B. Cytomegalovirus (CMV) contrainication to kiney onation?
C. Epstein-Barr virus A. History of arm melanoma status post wie
D. Hepatitis C virus local resection 10 years ago
E. Herpes simplex B. History of lymphoma as a chil
C. Current urinary tract infection
8. Which of the following is the best inication for D. Recent hospitalization for meningococcemia,
pancreas transplantation in type 1 iabetes? now with negative bloo cultures
A. A 45-year-ol male with stage  chronic kiney E. Open cholecystectomy 4 months ago
isease an recurrent episoes of marke
hyperglycemia 12. A 45-year-ol male arrives at clinic 1 year after
B. A 66-year-ol female with en-stage liver transplantation. He woul like to iscuss
renal isease who unerwent kiney his recent laboratory stuies an the health of his
transplantation 10 years ago liver. Which of the following is the best measure
C. A 41-year-ol male with severe anxiety of the function of his liver?
associate with insulin therapy, refractory A. Aspartate aminotransferase (AST)
gastroparesis, an recurrent episoes of B. Alanine transaminase (ALT)
marke hyperglycemia C. Total bilirubin
D. A 38-year-ol female that was recently D. Serum albumin
hospitalize for metabolic complications E. International normalize ratio (INR)
associate with iabetes
E. A 51-year-ol male with stage 3 chronic kiney 13. A 4-year-ol male with en-stage renal isease
isease an recurrent episoes of marke seconary to glomerulonephritis has been matche
hyperglycemia with a ecease onor kiney an arrives at
the hospital for transplantation. Which of the
9. Which of the following poses the highest risk of following is a guiing principle in this surgery?
eath in a patient awaiting renal transplantation? A. The right peritoneum is the preferre initial
A. Chronic obstructive pulmonary isease implant site
(COPD) B. The left retroperitoneum is the preferre initial
B. Cerebrovascular accient implant site
C. Smoker C. Baseline biopsy of the onor kiney shoul be
D. Black race obtaine at the conclusion of the case
E. Congestive heart failure D. The native kiney shoul not be remove
E. The renal artery shoul be anastomose to the
10. Which of the following is true regaring kiney internal iliac artery
transplant onation?
A. The most common cause of eath 14. A 8-year-ol female with en-stage renal isease
postoperatively for kiney onors is acute seconary to lupus nephritis unergoes a living-
renal failure relate onor kiney transplant an is making
B. The most common postoperative complication appropriate urine at the conclusion of the case.
for kiney onors is acute tubular necrosis On postoperative ay , the surgical intern ns
C. Donors must prove to have a glomerular that her urine output has roppe from 180 cc
ltration rate (GFR) greater than 80 mL/min to the previous hour to only 4 cc in the last hour.
be consiere as appropriate caniates The inwelling Foley is ushing well. Which of
D. The serum creatinine will be persistently the following is the most appropriate next step in
higher following kiney onation management?
E. The rate of live kiney onation has increase A. Take patient to the operating room (OR)
in the past 10 years B. Ultrasoun
C. Magnetic resonance angiography (MRA)
D. Compute topography (CT)
E. Urinalysis
288 PArt i Patient Care

Answers
1. C. Within 1 hours of liver transplantation an graft cell carcinoma of the skin, with most occurring about 8 years
reperfusion, a patient’s hemoynamic status, urine output, after the transplant. The most common type of PTLD is of
an coagulopathy shoul all begin to improve. Laboratory monoclonal B-cell origin (A). It occurs more commonly in
tests, incluing INR an transaminases, may follow with a heart an lung transplants compare to liver an renal trans-
slight elay. A lack of improvement in a patient’s clinical status plants (B). Early iagnosis requires a high inex of suspicion
or signicant worsening of transaminases, bilirubin levels, or because it can present with nonspecic symptoms incluing
INR, shoul prompt a clinician to assess for a major vascular/ fevers (most common), lymphaenopathy, night sweats, an
biliary complication or primary nonfunction. In this particular weight loss. Declining graft function can also be a presenting
case, a oubling of transaminases an persistent vasopressor symptom. Diagnosis begins with checking serum EBV viral
requirement is concerning for a major vascular complication, loa, although EBV-negative patients can also evelop PTLD.
most commonly hepatic artery thrombosis. A large biliary In fact, EBV-negative patients are at higher risk than EBV-pos-
leak, while possible, is less likely with minimal output from itive patients (D). Aitionally, CMV-negative patients are at
the patient’s surgical rains. An ultrasoun an oppler increase risk once they seroconvert following transplant.
stuy of the graft can conrm the iagnosis without requiring References: Opelz G, Henerson R. Incience of non-Ho-
cross-sectional imaging (C, D). Hepatic artery thrombosis, if gkin lymphoma in kiney an heart transplant recipients. Lancet.
etecte early, can be treate with thrombectomy an revision 1993;34(8886–8887):1514–1516.
of the arterial anastomosis an may result in graft salvage. Walker RC, Paya CV, Marshall WF, et al. Pretransplantation
seronegative Epstein-Barr virus status is the primary risk factor for
However, many cases require retransplantation (A). While less
posttransplantation lymphoproliferative isorer in ault heart,
common, portal vein thrombosis may present similarly an,
lung, an other soli organ transplantations. J Heart Lung Transplant.
if foun early, may be treate with thrombectomy to salvage 1995;14():14–1.
the graft. Flui resuscitation, broa-spectrum antibiotics, an Walker RC, Marshall WF, Strickler JG, et al. Pretransplantation
repeating laboratory tests will elay treatment of the unerly- assessment of the risk of lymphoproliferative isorer. Clin Infect
ing conition in this case (B, E). Dis. 1995;0(5):1346–1353.
Reference: Moura MM, Liossis C, Gunson BK, et al. Etiology
an management of hepatic artery thrombosis after ault liver trans- 4. C. This patient is experiencing hyperacute rejection. This
plantation: etiology an management of hepatic artery thrombosis. will present with the onor kiney appearing soft, abby,
Liver Transpl. 014;0(6):713–73. mottle, an eematous an can progress to wiesprea
interstitial hemorrhage an necrosis. This occurs within
2. A. Patients who have unergone kiney transplantation minutes to hours after the arterial anastomosis an is mei-
commonly have ui collections aroun the onor kiney. ate by preforme recipient antiboies to onor HLA anti-
This is frequently an asymptomatic ning an is incien- gens (A). The antiboies bin to the graft enothelium an
tally iscovere uring routine imaging stuies, often in ensue a cascae of events resulting in tissue necrosis. This is
the rst year. If the ui collection is small (<5 cm), it is an uncommon complication, but renal grafts are more com-
unlikely to cause any symptoms, an the patient can initially monly affecte. For reasons that are unclear, liver transplants
be observe with no aitional stuies require (C). Possi- are largely resistant to hyperacute rejection, but it is thought
ble etiologies inclue lymphocele, seroma, urine leak, an to be relate to the enormous size of the liver an its abil-
hematoma. The most common cause is lymphocele, which ity to absorb circulating antiboies (E). The only treatment
occurs seconary to severe lymphatic vessels uring sur- for hyperacute rejection is immeiate removal of the onor
gery. This is a self-limite complication an will resolve with kiney because this can result in hemoynamic instabil-
time. With larger ui collections, patients may evelop ity, multiorgan failure, an eath if left untreate (B). This
oliguria (extrinsic compression of the ureter), graft failure is particularly important in a patient who is alreay hypo-
(extrinsic compression of renal artery or vein), or infection. tensive. Acute rejection is a T-cell-meiate response with
Symptomatic ui collections will nee to be treate with activate monocytes secreting soluble meiators incluing
image-guie rainage or surgical rainage (E). In recurrent lymphokines IL-1 an IL- (D). This typically occurs 1 to 
cases, a peritoneal winow allowing internal rainage can be months after the transplant an shoul be conrme with
performe (D). Aitionally, the ui creatinine level shoul a renal biopsy. Patients will present with oliguria an/or
be compare to the serum level (B). This will help etermine rising creatinine. Treatment involves high-ose sterois.
if the patient has a urine leak. In this case, the patient may Chronic graft rejection is a poorly unerstoo process that
nee to receive a renal stent or nephrostomy tube an, rarely, can occur years after having a well-functioning onor graft.
ureteral reconstruction in the OR. Immunosuppression is largely ineffective in these cases.
Reference: Fuller TF, Kang SM, Hirose R, Feng S, Stock PG, References: Bhowmik DM, Dina AK, Mahanta P, Agarwal SK.
Freise CE. Management of lymphoceles after renal transplantation: The evolution of the Banff classication schema for iagnosing renal
laparoscopic versus open rainage. J Urol. 003;169(6):0–05. allograft rejection an its implications for clinicians. Indian J Nephrol.
010;0(1):–8.
3. E. PTLD is the secon most common cancer affecting Goron RD, Iwatsuki S, Esquivel CO, Tzakis A, Too S, Starzl
patients with soli organ transplants, with the majority occur- TE. Liver transplantation across ABO bloo groups. Surgery.
ring in the rst year (C). The most common cancer is squamous 1986;100():34–348.
CHAPtEr 20 Transplant 289

Ramsey G, Wolfor J, Boczkowski DJ, Cornell FW, Larson P, It may halt progression of iabetes-relate isease such as
Starzl TE. The Lewis bloo group system in liver transplantation. retinopathy an may even reverse isease incluing neurop-
Transplant Proc. 1987;19(6):4591–4594. athy an autonomic ysfunction. It oes not lea to rever-
sal of vascular isease seconary to iabetes. The American
5. B. PRA is performe in all patients that are liste for a Diabetes Association has provie inications for pancreas
kiney transplant. This tests the patient’s bloo against transplantation: (1) iabetic patients with imminent or estab-
bloo from a panel of onors in the same geographic area lishe en-stage renal isease who have ha or plan to have
(E). The panel serves as the HLA makeup of the potential a kiney transplant or () patients meeting all three of the
organs available for onation for the recipient. Patients that following criteria: frequent episoes of metabolic complica-
have a high PRA are consiere to be “highly sensitize” tions relate to iabetes (hypoglycemia, ketoaciosis, hyper-
an will have a higher likelihoo of rejection (A). Patients glycemia), emotional problems with insulin therapy that are
with a PRA greater than 80% will nee to wait much longer severe enough to be incapacitating, an consistent failure of
to match with a compatible onor, so they are given ai- insulin-base management to prevent complications. From
tional points on the kiney allocation algorithm prioritiz- the answer choices provie, the best inication is for the
ing them to the top of the list. However, even though their 41-year-ol male with severe emotional problems associate
names come up frequently as potential matches for newly with insulin therapy, refractory gastroparesis, an recur-
available kineys, they are frequently incompatible, so their rent episoes of marke hyperglycemia (A, D, E). Pancreas
wait times are much longer than patients with lower PRAs transplantation shoul be avoie in patients oler than 45
(D). Having a previous kiney transplant likely contribute to 65 because these patients have poor graft an 5-year sur-
to his high PRA, but this in an of itself oes not factor in the vival (B).
kiney allocation algorithm (C). References: Robertson RP, Davis C, Larsen J, Stratta R, Suther-
Reference: Hart A, Smith JM, Skeans MA, et al. OPTN/SRTR lan DER, American Diabetes Association. Pancreas an islet trans-
015 Annual Data Report: Kiney. Am J Transplant. 017;17(Suppl. 1): plantation in type 1 iabetes. Diabetes Care. 006;9(4):935.
1–116. Siskin E, Maloney C, Akerman M, et al. An analysis of pancreas
transplantation outcomes base on age groupings–an upate of the
6. D. Liver transplantation can be offere to patients with UNOS atabase. Clin Transplant. 014;8(9):990–994.
hepatocellular carcinoma an if appropriate caniates are
selecte, outcomes can be favorable. Mazzaferro an others 9. A. While it is true that the most common cause of eath
emonstrate that patients with certain tumor characteris- in patients with iabetes is cariac-relate, a history of cor-
tics that unergo liver transplantation can achieve a 4-year onary artery isease oes not place patients at the highest
survival of 75%. This is now known as the Milan criteria an risk for eath while awaiting renal transplantation (E). This
is use by UNOS to select appropriate caniates. Milan cri- speaks to the prevalence of heart isease in this patient
teria are as follows: a single tumor 5 cm or smaller or up to population. A large multivariable survival moel analyz-
three tumors with none larger than 3 cm, an no evience of ing over 160,000 patients emonstrate that COPD is the
vascular invasion, regional lymphaenopathy, or istant is- most signicant factor inepenently associate with eath
ease (A–C, E). Tumors limite to a particular liver segment among patients awaiting renal transplantation (ajuste
o not factor into selecting appropriate caniates. hazar ratio of 1.31). This is followe by, in escening
Reference: Mazzaferro V, Regalia E, Doci R, et al. Liver trans- orer, smoker status, nonambulatory status, coronary artery
plantation for the treatment of small hepatocellular carcinomas in isease, peripheral vascular isease, congestive heart fail-
patients with cirrhosis. N Engl J Med. 1996;334(11):693–699. ure, cerebrovascular isease, an hypertension (B, C). Black
patients awaiting kiney transplantation survive longer than
7. B. CMV is a member of the herpesvirus family an is the white patients, but this reverses when black patients receive
most clinically signicant viral infection in transplant recip- kiney transplantation (D). Aitionally, COPD is the most
ients. In healthy, nonimmunosuppresse iniviuals, CMV signicant risk factor associate with poor graft function an
is clinically silent or mil. In immunosuppresse transplant survival following a kiney transplant.
recipients, CMV is associate with increase mortality an References: Kapur A, De Palma R. Mortality after myo-
graft loss. In one large stuy of liver transplant recipients, carial infarction in patients with iabetes mellitus. Heart.
CMV infection was foun to be an inepenent risk factor 007;93(1):1504–1506.
for graft failure. In a cariac transplantation stuy, CMV-neg- van Walraven C, Austin PC, Knoll G. Preicting potential sur-
vival benet of renal transplantation in patients with chronic kiney
ative recipients of CMV-positive onor hearts ha impaire
isease. CMAJ. 010;18(7):666–67.
istal epicarial enothelial function an an increase inci-
ence of cariovascular-relate events an eath uring 10. C. As the incience of iabetes an en-stage renal
follow-up. isease has steaily risen in the past several ecaes, the
References: Burak KW, Kremers WK, Batts KP, et al. Impact of number of patients awaiting kiney transplantation has
cytomegalovirus infection, year of transplantation, an onor age
also been increasing. Due to a multiisciplinary approach
on outcomes after liver transplantation for hepatitis C. Liver Transpl.
00;8(4):36–369. an the concerte efforts of transplant groups such as the
Petrakopoulou P, Kübrich M, Pehlivanli S, et al. Cytomegalovirus Unite Network for Organ Sharing (UNOS), the availability
infection in heart transplant recipients is associate with impaire of ecease kiney onors has risen. However, the rate of
enothelial function. Circulation. 004;110(11 Suppl 1):II07–II1. live kiney onation has roppe in greater numbers, leav-
ing a total ecit in the availability of kiney onors espite
8. C. Pancreas transplantation has been shown to improve the increase in ecease onors (E). There are several soci-
survival an quality of life in patients with type 1 iabetes. etal guielines to etermine the caniacy of live kiney
290 PArt i Patient Care

onors, an one prevailing requirement across all govern- the surgical approach originally escribe has change very
ing boies is the requirement of a GFR greater than 80 mL/ little in moern practice. The peritoneum is a poor choice for
min conrme with a nuclear test or 4-hour urine collec- implantation because it poses a high risk of graft contami-
tion. The most common cause of eath postoperatively for nation an infection. The retroperitoneum an pelvic fossa
kiney onors is pulmonary emboli (A). The most common are the preferre sites (A). Most surgeons prefer the right
complication for kiney onors postoperatively is woun sie because the iliac vessels are longer an more horizontal,
infection (B). Although the serum creatinine may be higher allowing for a technically easier anastomosis (B). However,
in the immeiate postoperative perio, it will eventually go if there are any previous issections or operations involving
back own an the baseline creatinine will remain the same mesh (e.g., herniorrhaphy) on the right sie, the left sie can
or close to the baseline as the onor will continue to have one be chosen. Generally, it is preferable to perform the venous
functioning kiney remaining (D). anastomosis before the arterial anastomosis to avoi vas-
References: Clinical Practice Guielines for Living Kiney cular congestion of the kiney, followe nally by ureteral
Donors. Kiney Disease Improving Global Outcomes; KDIGO, 017. reconstruction. The external iliac vein an artery are the
Najarian JS, Chavers BM, McHugh LE, Matas AJ. 0 years or more preferre targets for the anastomosis (E). This is because is-
of follow-up of living kiney onors. Lancet. 199;340(883):807–810. section of the internal iliac vessels is technically challenging,
Hart A, Smith JM, Skeans MA, et al. OPTN/SRTR 015 Annual
which increases operative time an subjects the patient to
Data Report: Kiney. Am J Transplant. 017;17(Suppl. 1):1–116.
aitional risks such as autonomic plexus injury (e.g., erec-
tile ysfunction). The stanar ureteral reconstruction is a
11. A. Since the availability of kiney onors has been
ureteroneocystostomy because it avois the eep issection
eclining, establishing appropriate guielines for isease
necessary for a ureteroureterostomy. The utility of obtain-
kiney onation is imperative to maximize the scarcity of
ing a baseline biopsy is controversial at best. The argument
available organs. Several absolute contrainications to organ
against it is that it exposes the patient to a biopsy-inuce
onation exist, incluing patients with HIV (unless the recip-
vascular thrombosis, which can compromise the graft (C).
ient also has HIV), hepatitis (unless the recipient also has the
It shoul be note that the native kiney shoul remain in
same hepatitis type), cirrhosis, an active systemic infection
place because it can often continue to have a small role by
with positive bloo cultures. A previous hospitalization for
secreting erythropoietin.
systemic infection is not consiere an absolute contrain-
Reference: Zhao J, Gao Z, Wang K. The transplantation opera-
ication as long as the patient has proven to have negative
tion an its surgical complications. In: Understanding the Complexities
bloo cultures (D). Similarly, urosepsis woul preclue organ of Kidney Transplantation. InTech; 011.
onation, but a urinary tract infection in an of itself woul
not (C). A history of cholecystectomy in a patient without 14. B. Proviing aequate ui resuscitation following ki-
signicant liver isease oes not preclue organ onation ney transplantation is essential in preventing graft failure.
(E). A history of cancer may preclue ecease onors from Although there is no consensus on the optimal postoperative
organ onation. Some exceptions can be mae for patients ui regimen in kiney transplantation, the use of crystal-
with a remote history of low-grae visceral malignancy such lois shoul be the volume replacement of choice, an most
as colorectal cancer or patients with less aggressive cancers transplant surgeons woul agree to aim to achieve a urine out-
such as basal cell carcinoma or chilhoo lymphomas. Simi- put greater than 100 cc per hour. The most common cause of
larly, low-grae primary CNS tumors o not pose a high risk postoperative oliguria is acute tubular necrosis (ATN), which
of transmission (B). Melanoma in particular poses a risk for can be initially worke up with urinalysis (E). However, ATN
transmission even in patients with a remote history, so this will present with a graual ecrease in urine output an will
will prevent the patient from being an eligible onor. frequently respon to a ui bolus. A suen rop in urine
References: Birkelan SA, Storm HH. Risk for tumor an other output or anuria is concerning for graft thrombosis. This coul
isease transmission by transplantation: a population-base stuy
have catastrophic outcomes if not iagnose early. In fact, it
of unrecognize malignancies an other iseases in organ onors.
is consiere the main cause of graft failure in the rst year,
Transplantation. 00;74(10):1409–1413.
Feng S, Buell JF, Cherikh WS, et al. Organ onors with positive
with the majority occurring at 48 hours. It typically involves
viral serology or malignancy: risk of isease transmission by trans- the renal vein, but the renal artery can also be affecte. In any
plantation. Transplantation. Publishe online 00;78:1657–1663. patient with a suen ecrease in urine output, the rst step
is to ush the Foley to ensure there is no kinking preventing
12. E. A liver function test (LFT) measures the levels of urine ow. The next step is to perform a besie ultrasoun to
AST, ALT, an alkaline phosphatase, but oes not reect look for vascular thrombosis. If this is ientie, the next step
the synthetic function of the liver; thus, LFT is a misnomer is to go to the OR for surgical revascularization or intraarterial
(A, B). The best test to etermine the liver’s function is the thrombolytic therapy (A). If ultrasoun nings are equivo-
prothrombin time (PT), or INR. Albumin an PTT are also cal, the next step is to perform an ajunct imaging stuy such
helpful (D). Total bilirubin is inuence by biliary tree as MRA, CT, or renal scintigraphy (C, D).
obstruction, intrinsic hepatic isease, an hemolysis (C). References: Ponticelli C, Moia M, Montagnino G. Renal allograft
thrombosis. Nephrol Dial Transplant. 009;4(5):1388–1393.
13. D. Kiney transplantation has le to improve survival Schnuelle P, Johannes van er Woue F. Perioperative ui man-
an quality of life in patients with en-stage renal isease. agement in renal transplantation: a narrative review of the literature.
Transpl Int. 006;19(1):947–959.
It was rst performe in France by Rene Kuss in 1951, an
Thoracic Surgery
JORDAN M. ROOK AND SHONDA L. REVELS 21
ABSITE 99th Percentile High-Yields
I. Anatomy:
A. Azygous vein: ascens along right thoracic vertebral column an rains into SVC
B. Thoracic uct: Begins in abomen at cisterna chyli (L1), traveling between azygous an aorta until T5,
where it crosses right to left, raining into junction of the left subclavian an internal jugular vein
C. Phrenic nerve: escens anterior to hilum; Vagus nerve: travels posterior to hilum
D. Dual bloo supply to lung:
1. Alveoli: unoxygenate bloo via pulmonary artery (low-pressure system)
. Bronchi: oxygenate bloo via bronchial arteries; originate from thoracic aorta (most common), aortic
arch, or intercostal arteries
E. Cellular anatomy:
1. Type 1 pneumocytes: gas exchange; Type  pneumocytes: surfactant

II. Preoperative Evaluation:


A. No matter the surgery, mortality an morbiity increase if preicte postoperative FEV1<40%
B. Preoperative FEV1 greater than  L—can tolerate pneumonectomy
C. Preoperative FEV1 greater than 1.5 L—can tolerate lobectomy
D. If concern with any of the above, obtain quantitative perfusion lung scan or SPECT/CT to etermine
how much isease lung contributes to FEV1

III. Pathology:
A. Bronchogenic cysts
1. Rare congenital malformations of the tracheobronchial tree
. Many are asymptomatic an incientally foun on imaging
3. Those with symptoms present uring the secon ecae of life with coughing, wheezing, an
pneumonia
4. The stanar of care is surgical excision by partial or total lobectomy
B. Pleural effusion management:
1. Simple (nonloculate): treat unerlying cause, rain, pleuroesis if neee
. Hemothorax (retaine): Vieo-assiste thoracoscopic surgery (VATS) washout; ecortication if lung is
trappe
3. Empyema: requires complete rainage (ifcult ue to loculations) -> attempt brinolytic therapy
(TPA an DNase) -> VATS or thoracotomy for ecortication
C. Chylothorax:
1. Disruption of the thoracic uct (1.5–.5 L/ay)
. Causes: 50% trauma (inclues iatrogenic) an 50% malignancy (most common is lymphoma)
3. Dx: greater than 110 mg/L of triglyceries with lymphocytic preominance. Positive Suan re stain.
4. Treatment: no-fat/low-fat meium-chain fatty aci iet, NPO +TPN, rainage, octreotie
a) If fails: right VATS/thoracotomy an ligation of thoracic uct by surgery or enovascular embolization
291
292 PArt i Patient Care

D. Meiastinal tumors:
1. Most common cause of lymphaenopathy: lymphoma
. Most common tumor in chilren: neurogenic tumors (posterior meiastinum)
3. Most common germ cell tumor: teratoma (anterior meiastinum)
4. Thymoma (anterior meiastinum): all require resection
E. Superior vena cava synrome:
1. Common causes: malignancy most common (#1 small cell; # lymphoma); also, stenosis relate to
prior central venous catheters or pacemaker wires
. Treatment:
a) Malignancy: enitive chemoraiation, enovascular stenting if life-threatening venous
hypertension (airway obstruction, cerebral eema)
b) Venous stenosis: angioplasty an stent placement
F. Hemoptysis:
1. Can “rown” with only 150 mL of bloo; 90% ue to high-pressure bronchial arteries
. Tx: establish airway with mainstem intubation of nonbleeing bronchus with bronchoscopy (rigi
preferre to exible), place patient in lateral ecubitus (bleeing sie own), bronchoscopy versus
selective bronchial artery embolization

IV. Lung Cancer:


A. Screening: 00 guielines recommen annual low-ose CT scan for aults age 50 to 80 with at least a
0-pack year smoking history; must be current smokers or have quit within the past 15 years
1. Nonsmall cell lung cancer: approximately 80% of all cancers; 3 types are aenocarcinoma (30%–50%)
(peripheral location, nonsmokers), squamous cell (0%–35%), an large cell (4%–15%)
. Small cell (oat cell) carcinoma accounts for 0% of all lung cancer, occurs centrally near hilum, almost
exclusively in smokers an not amenable to surgery, typically tx chemoraiation
B. Solitary pulmonary noule (SPN)
C. Paraneoplastic synromes:
1. Squamous cell: hyperparathyroiism (PTHrP)
. Small cell: Cushing isease (ACTH) an SIADH
CHAPtEr 21 Thoracic Surgery 293

Fig. 21.1 Anatomy of the Thoracic Outlet.


294 PArt i Patient Care

Questions
1. A 54-year-ol male presents to clinic for surgical 5. The most common cause of lung abscess is:
evaluation of his recently iagnose TN1M0 A. Aspiration
(Stage II) esophageal cancer. He is scheule B. Bronchial obstruction by tumor
to unergo neoajuvant chemoraiation an C. Pneumococcal pneumonia
presents for surgical planning. His primary D. Pneumocystis jiroveci pneumonia
complaint is ysphagia limiting him to thin E. Mycobacterium tuberculosis pneumonia
liquis. Albumin is .6 g/L an prealbumin is
8 mg/L. What is the next best step? 6. Four months after prolonge intubation after
A. Procee with chemoraiation a motor vehicle accient, a 40-year-ol woman
B. J tube placement presents with strior an yspnea on exertion.
C. Attempt PEG tube placement Enoscopy reveals marke tracheal stenosis 4 cm
D. G-J tube placement in length. Management consists of:
E. TPN supplementation A. Laser ablation
B. Bronchoscopic ilation
2. A 63-year-ol male is unergoing evaluation of a C. Primary resection of all scarre segments with
4-cm left upper lobe mass. CT emonstrates a 1.1-cm primary anastomosis
suspicious para aortic (station 6) lymph noe. D. Primary resection of all scarre segments,
What is your next best step? primary anastomosis, an temporary
A. Cervical meiastinoscopy tracheostomy
B. Anterior meiastinoscopy E. Metal stenting
C. Enobronchial ultrasoun
D. Percutaneous core neele biopsy 7. Which of the following is true regaring Lambert-
E. Esophageal ultrasoun Eaton myasthenic synrome?
A. It is most often associate with squamous cell
3. A 43-year-ol female is unergoing a workup of carcinoma of the lung
an incientally iscovere 3.8 cm meiastinal B. 3,4-Diaminopyriine is not effective in treating
mass. CT emonstrates a small, well- symptoms
circumscribe thymic mass without evience of C. Intravenous (IV) immunoglobulin is effective
invasion into local structures. What is your next in treating symptoms
best step? D. Thymectomy is effective in patients in whom
A. Percutaneous image-guie biopsy meical management fails
B. Interval CT in 6 months E. Patients present with istal muscle weakness
C. Fine neele aspirate
D. Thymectomy 8. A 65-year-ol woman presents with a chronic
E. Referral to an oncologist nonprouctive cough of  months uration. A
chest raiograph reveals a -cm mass in the right
4. A -year-ol male is sent to thoracic surgery upper lobe. A CT scan of the chest conrms the
clinic for evaluation of a bronchogenic cyst presence of the -cm mass corresponing to that
incientally iscovere on chest x-ray an foun on the chest raiograph, which appears
conrme on CT. He enies any symptoms. What to be malignant, along with 5-mm noes in the
is the best treatment? meiastinum. The next step in management
A. VATS lobectomy woul be:
B. Cyst fenestration A. Positron emission tomography (PET) scan
C. Observation B. Abominal CT
D. 6-month interval CT C. Bone scan
E. Enobronchial ultrasoun for sampling of cyst D. Meiastinoscopy
ui E. Brain CT
CHAPtEr 21 Thoracic Surgery 295

9. A 4-year-ol woman presents with recurrent 13. What happens to the partial pressure of arterial
episoes of right-sie pneumothorax requiring oxygen as bloo ows from the pulmonary
chest tube insertion. A iagnosis of a catamenial capillaries to the left atrium?
pneumothorax as the cause of recurrent A. Increase
pneumothorax in this patient woul be supporte B. Decrease
by the ning of: C. Stay the same
A. Pneumocystis D. Depens on cariac output
B. Enometriosis E. Depens on pulmonary vascular resistance
C. Cystic brosis
D. Iiopathic pulmonary brosis 14. The most common primary chest wall malignancy
E. Apical blebs is:
A. Osteochonroma
10. A 35-year-ol male with iffuse axonal injury B. Chonrosarcoma
following a motorcycle collision is recovering in C. Ewing sarcoma
the surgical intensive care unit (ICU). He has been D. Plasmacytoma
intubate for 3 weeks. This morning the patient E. Primitive neuroectoermal tumors
evelope an enotracheal air leak that persiste
even with tube exchange an hyperination. 15. A 60-year-ol male presents to the emergency
His abomen appears istene. Bronchoscopy epartment (ED) with right arm swelling an
is performe an emonstrates yellow-colore pain. He has a 40-pack-per-year smoking history.
secretions in both main stem bronchi. Which of He reports a 0-poun weight loss over the
the following is true? past  months. His exam is notable for pitting
A. The patient shoul be switche to low tial eema to the right upper extremity. Chest x-ray
volume ventilation emonstrates a large mass in the right upper lobe.
B. Early conversion to tracheostomy ecreases Which of the following is the best next step in
the risk for this complication treatment?
C. Nasogastric tube increases the risk for this A. Chemotherapy
complication B. Chemotherapy an raiation
D. Low intracuff pressure contributes to the C. Raiation therapy
evelopment of this complication D. Enovascular stenting
E. CT scan of the abomen shoul be performe E. Thoracotomy

11. A 65-year-ol man presents with anorexia, 16. A 6-year-ol male with esophageal cancer
nausea, lethargy, an hyponatremia. A chest evelops shortness of breath 3 ays status post
raiograph reveals a large right upper lobe mass. an esophagectomy after up-titration of his J-tube
This most likely represents: fees. He is afebrile with a normal white bloo
A. Aenocarcinoma cell (WBC) count. Chest x-ray emonstrates a
B. Small cell carcinoma large right-sie pleural effusion, an a chest
C. Squamous cell carcinoma tube is inserte evacuating one liter of milky
D. Carcinoi white ui. Flui analysis emonstrates elevate
E. Bronchoalveolar carcinoma triglyceries an an exuative effusion with a
lymphocytic preominance. What is the next best
12. Which of the following is true of thoracic step?
anatomy? A. NPO an TPN
A. The left lung has three lobes B. IR embolization of the thoracic uct
B. The azygous vein runs along the left sie C. VATS thoracic uct ligation
raining into the subclavian vein D. Octreotie
C. The vagus nerve runs anterior to the lung E. No fat, elemental tube fee regimen
hilum
D. The sternocleiomastoi muscle is consiere
an accessory muscle to breathing
E. The phrenic nerve runs posterior to the lung
hilum
296 PArt i Patient Care

17. A 45-year-ol male presents to the ED with 00 21. Which of the following is true regaring aortic
mL of hemoptysis. He continues to expectorate stenosis (AS)?
bloo an appears to be in respiratory istress. A. In low-risk patients with severe symptomatic
His bloo pressure is 150/90 mmHg an his heart AS, transcatheter aortic valve replacement is
rate is 130 beats per minute with an SpO of 78% preferre
espite attempts at besie suctioning. A chest B. The most common cause of AS is rheumatic
raiograph reveals bilateral inltrates. What is the fever
next best step in management? C. Symptoms generally evelop when the valve
A. Intubation with a ouble-lumen enotracheal area is less than  cm²
tube D. Swollen legs an elevate brain natriuretic
B. Rigi bronchoscopy peptie porten a poor prognosis
C. Bronchial artery embolization E. Valve repair is preferre to valve replacement
D. Pulmonary arteriography with selective
embolization 22. Which of the following is true regaring
E. Flexible bronchoscopy intraaortic balloon pump (IABP)?
A. It improves cariac function in patients with
18. Which of the following is true regaring cariogenic shock ue to aortic regurgitation
pulmonary sequestration? B. It is benecial in patients with aortic
A. MRI is consiere the iagnostic imaging of issection
choice C. It improves coronary bloo ow uring
B. The most common presentation is recurrent systole
pulmonary infection D. It is only benecial in patients that have
C. It typically communicates with the exhauste coronary autoregulation
tracheobronchial tree E. It is not inicate in acute myocarial
D. Extra lobar pulmonary sequestration remains infarction
within the visceral pleura of the native lung
E. The majority of asymptomatic cases can be 23. A 50-year-ol Central American man presents
observe with a chronic cough an a raining sinus in
his left chest wall. Examination of the rainage
19. A 49-year-ol male has a right-sie perihilar reveals sulfur granules. Which of the following is
mass incientally foun on CT scan performe true regaring this conition?
after a motor vehicle trauma 1 month ago. He has A. Surgical resection is inicate
a 30-pack-per-year smoking history. He reports B. The organism involve is likely Nocardia
his clothes t more loosely. On examination, he asteroids
has purple striae on his abomen an prominent C. The organism involve is an anaerobe
fat on his posterior neck. PET/CT scan conrms D. Optimal treatment consists of trimethoprim-
a 4-cm irregular mass as well as an FDG avi sulfamethoxazole
hilar lymph noe but no evience of metastatic E. Central nervous system involvement is
isease. Which of the following most likely common
represents this patient’s enitive treatment?
A. Raiation therapy alone 24. A 45-year-ol male with aenocarcinoma of the
B. Combination chemotherapy an raiation right lung presents to clinic to iscuss surgical
C. Neoajuvant chemotherapy an resection resection. Which of the following is the most
D. Resection an ajuvant chemotherapy important pulmonary function stuy to orer for
E. Chemotherapy this patient?
A. Arterial bloo gas
20. A rare but well-recognize complication of B. Force expiratory volume 1 (FEV1)
bronchial artery embolization performe for C. Total lung capacity
massive hemoptysis is: D. Minute ventilation
A. Esophageal necrosis E. Diffusing capacity of the lung for carbon
B. Pulmonary infarction monoxie (DLCO)
C. Paraparesis
D. Vocal cor paralysis
E. Tracheal necrosis
CHAPtEr 21 Thoracic Surgery 297

25. The patient in question 4 unergoes pulmonary 29. Rasmussen aneurysms form in association with:
function testing for a planne lobectomy of the A. Aspergillosis
right lung an his FEV1 is 1. L. Which of the B. Mucormycosis
following is true? C. Cryptococcosis
A. The patient is not a caniate for lobectomy D. Tuberculosis
B. Surgery can procee as the plan is for a E. Small cell lung cancer
lobectomy
C. A ventilation-perfusion (VQ) scan shoul be 30. Which one of the following statements is true
performe regaring thymoma?
D. He shoul unergo respiratory muscle A. The primary treatment moality is
training with incentive spirometer chemotherapy
E. Repeat testing shoul be performe following B. Malignancy is etermine by mitotic activity
breathing treatment with albuterol C. The majority of patients with myasthenia
gravis have an associate thymoma
26. Which of the following statements is true D. In patients with myasthenia gravis,
regaring tracheal anatomy? thymectomy results are more favorable in
A. The bloo supply is preominantly from the those without a thymoma than those with one
superior thyroi arteries E. It is not associate with SVC synrome
B. The rich collateral bloo supply allows
circumferential mobilization 31. A woman who ha an osteogenic sarcoma of
C. As much as 50% of the length of the trachea the femur remove  years earlier now presents
can be resecte with a primary anastomosis with two small lesions in the right lung an one
following resection small lesion in the left lung. A metastatic workup
D. A tracheostomy tube is ieally place through reveals no other abnormalities. The treatment of
the rst tracheal ring choice is:
E. The rst complete cartilaginous ring is the A. Bilateral wege resections
thyroi cartilage B. Chemotherapy
C. Raiation therapy
27. Which of the following is consiere a D. Immunotherapy with (bacille Calmette-
contrainication to surgical resection of a primary Guérin) vaccine
(nonsmall cell) carcinoma of the lung? E. Observation
A. Invasion of the chest wall
B. A positive ipsilateral meiastinal lymph noe
C. A malignant pleural effusion
D. Stage 3A isease
E. Invasion of parietal pericarium

28. A 47-year-ol woman presents to the ED with


worsening fatigue an moerate yspnea on
exertion. She is a lifelong nonsmoker. A chest
raiograph emonstrates a 3-cm noule in the
periphery of the left lung with a mil pleural
effusion. She reports night sweats an a 0-poun
weight loss in the past 3 months. Which of the
following is the most likely iagnosis?
A. Squamous cell carcinoma
B. Aenocarcinoma
C. Small cell carcinoma
D. Bronchoalveolar
E. Carcinoi
298 PArt i Patient Care

Answers
1. B. This patient presents with potentially curable esopha- Thus, observation an conservative proceural management
geal cancer (stage ). Any patient with stage  or greater is- (B–E) are incorrect.
ease shoul unergo neoajuvant chemoraiation followe
by esophagectomy. Base on symptoms an laboratory test- 5. A. A lung abscess usually results from an aspiration
ing, this patient is malnourishe ue to ysphagia restrict- event that causes a suppurative bacterial infection, leaing to
ing aequate PO nutrition. To optimize surgical outcomes, localize pulmonary parenchymal necrosis. These abscesses,
it is vitally important to improve nutrition prior to procee- known as primary lung abscesses, have similar risk factors
ing with chemoraiation an esophagectomy (A). Enteral as aspiration pneumonia, incluing history of alcohol abuse,
nutrition is preferre over parenteral nutrition (E). Enteral poor entition or gum isease, an seizure isorer or
access shoul be establishe. Gastric conuits are preferre altere level of consciousness. Seconary lung abscesses, or
in reconstructing the intrathoracic esophagus. As such, all those resulting from a preexisting conition, can result from
attempts shoul be mae to avoi placement of a gastros- bronchial obstruction by tumors, leaing to postobstructive
tomy tube, which may irreparably amage the stomach an pneumonia an hematogenous sprea via septic pulmo-
prevent future creation of a gastric conuit (C, D). nary emboli from infecte inwelling catheters, prosthetic
evices, or enocaritis. Various opportunistic infections
2. B. The patient in this question likely has a new iagno- (Nocaria, M. tuberculosis, etc.) can cause abscesses in the
sis of lung cancer with concern for a para aortic lymph noe immunocompromise host (B–E).
metastasis. Aortopulmonary lymph noe stations 5 an 6 are Reference: Feerman DD, Nabel EG, es. Infectious diseases: the
among the meiastinal N stations. With a biopsy positive clinician’s guide to diagnosis, treatment, and prevention. Decker Publish-
for carcinoma, this patient woul be no less than Stage 3A, ing; 014.
inicating a nee for ownstaging with neoajuvant chemo-
raiation. Cervical meiastinoscopy is the most wiely 6. C. Tracheal stenosis is most commonly ue to trauma
use metho of sampling meiastinal lymph noes, sta- from prolonge enotracheal intubation or tracheostomy.
tions L, R, 4L, 4R, an 7 (A). VATS, anterior meiastinot- The risk of stenosis is greater when tracheostomies are
omy (Chamberlain proceure), or anterior meiastinoscopy place too high (through the rst tracheal ring) or for cri-
(B) are reasonable methos to sample the aortopulmonary cothyroiotomies (the cricothyroi membrane marks the
noes, stations 5 an 6. VATS is aitionally useful for sta- narrowest portion of the trachea). Patients with tracheal ste-
tions , 4R, 8, an 9. Neither enobronchial ultrasoun nor nosis present with strior an yspnea on exertion, which
esophageal ultrasoun offers access to the aortopulmonary can be confuse with asthma, an usually present within 
lymph noe stations (C, E). It is not avise to attempt per- to 1 weeks after ecannulation or extubation. The treatment
cutaneous biopsy of meiastinal lymph noes (D). of tracheal stenosis is resection an primary anastomosis. As
much as 50% of the trachea (average length between 10 an
3. D. Surgical resection is the mainstay of therapy for all 13 cm) can be resecte in most ault patients using laryngeal
thymic masses. This patient has tumor characteristics that release proceures. Most patients can be immeiately extu-
are reassuring for benign thymoma, incluing its size of bate without tracheostomy placement (D). Laser ablation,
less than 5 cm, absence of invasion into local structures, an ilation, an stenting are not enitive treatment options
well-ene capsule. Despite this, she shoul unergo resec- an are not inicate for circumferential scar formation or a
tion to rule out malignancy an to prevent complications of stenotic segment greater than 1 cm (A, B, E).
unimpee growth within the meiastinum. It woul not Reference: George M, Lang F, Pasche P, Monnier P. Surgical
be appropriate to observe with interval CT (B). Percutane- management of laryngotracheal stenosis in aults. Eur Arch Otorhi-
ous image-guie biopsy woul not affect management (A). nolaryngol. 005;6(8):609–615.
Fine neele aspirate has no role in the iagnosis of thymoma
(C). It woul be premature to refer this patient to an oncolo- 7. C. Lambert-Eaton or Eaton-Lambert myasthenic syn-
gist (E). While half of patients with thymoma have concom- rome is a paraneoplastic synrome associate with several
itant myasthenia gravis, EMG woul not be inicate nor malignancies, but in particular with small cell carcinoma. It
help etermine management. presents with proximal muscle weakness an can be con-
fuse with myasthenia gravis (E). More than half (estimate
4. A. Bronchogenic cysts are rare congenital malformations to be as great as 84%) of patients have or will be iscovere
of the tracheobronchial tree. Typically, these patients present to have SCLC (A). The synrome is thought to be cause by
uring the secon ecae of life with coughing, wheezing, antiboies irecte against presynaptic calcium channels in
an pneumonia. Many are asymptomatic an incientally the neuromuscular junction that prevent the release of ace-
foun on imaging. The stanar of care for this conition tylcholine. Treatment is aime at the unerlying malignancy;
is surgical excision by partial or total lobectomy. Even for however, meications shown to improve symptoms inclue
asymptomatic patients, given concern for the evelopment 3,4-iaminopyriine, IV immunoglobulin, an sterois (B).
of symptoms ue to airway compression or infection in Unlike in myasthenia gravis, neostigmine is not helpful an
aition to malignant potential, surgical excision is avise. thymectomy is not effective (D).
CHAPtEr 21 Thoracic Surgery 299

Reference: Maison P, Newsom-Davis J. Treatment for Lam- of this complication (B). Low tial volume ventilator man-
bert-Eaton myasthenic synrome. Cochrane Database Syst Rev. agement is preferre for ault respiratory istress synrome
005;():CD00379. (A). CT scan of the abomen is not require (E).
Reference: Paraschiv M. Tracheoesophageal stula–A compli-
8. A. The recommene sequential workup for a poten- cation of prolonge tracheal intubation. J Med Life. 014;7(4):516–51.
tially resectable lung cancer shoul begin with a CT scan of
the chest, followe by a PET/CT scan. If the CT scan shows 11. B. This patient likely has small cell lung cancer with a
a meiastinal lymph noe larger than 1 cm or if a meiasti- paraneoplastic synrome of inappropriate secretion of anti-
nal lymph noe lights up on PET scan, meiastinoscopy is iuretic hormone (SIADH). This paraneoplastic synrome
inicate (D). PET scanning has replace multiorgan scan- evelops in approximately 10% of patients with SCLC. Over-
ning in the search for istant metastases to the liver, arenal all, 70% of paraneoplastic SIADH is ue to SCLC. The iag-
glans, an bones (B, C, E). If PET scanning etects potential nosis is mae by a combination of hyponatremia, low serum
metastasis, it is important to obtain a tissue iagnosis before osmolality, an high urine soium an osmolality. In mil
enying a possible resection. cases, treatment consists of free water restriction. In more
References: Maaus MA, Lukeitch JD. Chest wall, meiasti- severe cases, treatment consists of aing emeclocycline or
num, an pleura. In: Brunicari FC, Anersen DK, Billiar TR, et al., a vasopressin-receptor antagonist such as tolvaptan. SIADH
es. Schwartz’s principles of surgery. 8th e. New York: McGraw-Hill;
woul be unusual with the other tumors liste (A, C–E).
005:545–610.
Hypercalcemia is associate with squamous cell carcinoma
Silvestri GA, Tanoue LT, Margolis ML, Barker J, Detterbeck
F, American College of Chest Physicians. The noninvasive stag- ue to the prouction of parathyroi hormone (PTH)-relate
ing of non-small cell lung cancer: the guielines. Chest. 003;13(1 protein.
Suppl):147S–156S.
12. D. There are several key anatomic lanmarks in the tho-
9. B. Catamenial pneumothorax is an uncommon cause rax that all surgeons must know. The right lung has three
of pneumothorax in women that occurs aroun the time of lobes, incluing the upper, mile, an lower lobes, while
menstruation. The exact etiology is unclear; however, it is the left lung has two lobes, incluing the upper an lower
associate with enometriosis an enometrial eposits on lobes (A). The left lung also has the lingula, which is consi-
the pleura in most instances. The enometrial eposits lea ere an extension of the upper lobe. The azygous vein runs
to pleural irritation. Given that catamenial pneumothorax is along the right sie, raining into the superior vena cava (B).
ifcult to iagnose prior to surgical intervention, it is often The majority of breathing occurs by using the iaphragm,
treate similarly to other spontaneous pneumothoraces with but accessory muscles can contribute up to 0% of the work
tube thoracostomy for a rst episoe. Similar to sponta- of breathing. These inclue the sternocleiomastoi muscle,
neous pneumothorax, patients with recurrent pneumothorax intercostal muscles, anterior scalene, an oblique muscles.
shoul unergo VATS, blebectomy, an pleuroesis. At the The phrenic nerve runs anterior an the vagus nerve runs
same time, most clinicians suggest ligating all iaphragmatic posterior to the lung hilum (C, E). Of note, the azygous vein
perforations (which allows for the transfer of intraperitoneal is typically ivie in infants uring repair of esophageal
enometrial cells to the thoracic cavity) an resecting visible atresia.
enometrial implants. Treatment with hormonal suppressive
therapy has been effective in preventing recurrent attacks. 13. B. Deoxygenate bloo leaves the right ventricle via
Apical blebs along with the other given choices are also pos- the pulmonary arteries to receive oxygen in the lungs. The
sible etiologies of spontaneous pneumothorax but are less hemoglobin traveling in the pulmonary capillaries partic-
likely to be the cause in a patient iagnose with catamenial ipates in air exchange in the alveolar sac. The newly oxy-
pneumothorax (A, C–E). genate hemoglobin is then carrie by the bloo in the
pulmonary veins to rain into the left atrium. Aitionally,
10. C. This patient has evelope a tracheoesophageal s- bronchial veins carrying eoxygenate bloo use by the
tula (TEF) as a result of prolonge intubation. This is the most lung parenchyma also rain into the pulmonary veins an
common cause of benign TEF, with an incience of up to 3% ultimately the left atrium. This results in bloo in the left
in ventilate patients. Risk factors inclue high cuff pressure ventricle having a partial pressure of arterial oxygen that is 5
(single most important), high airway pressure, excessive mmHg lower than that of bloo in the pulmonary capillary
tube motion, prolonge intubation, esophagitis, hypoten- (A, C). Cariac output an pulmonary vascular resistance o
sion, sterois, an avance age (D). If the enotracheal tube not change the general ow of bloo (D, E).
is place against a rigi nasogastric tube in the esophagus,
it can prouce an ischemic necrosis, resulting in abnormal 14. B. Chonrosarcomas are the most common primary
communication. TEF can also manifest after the patient has malignancy of the chest wall (A, C–E). They usually arise
been extubate an will present with expectoration of foo, anteriorly. They are typically low-grae malignancies an
eglutition followe by cough, an bronchopulmonary sup- are slow-growing, so they are not very sensitive to chemo-
puration. In ventilate patients, TEF is suggeste by per- therapy or raiation. Treatment is raical resection. Those
sistent air leaks even with a hyperinate cuff, abominal with unresectable isease or positive margins shoul be
istention (air entering the stomach through the TEF), an treate with raiation therapy. There is no role for ajuvant
bronchial contamination with foo an bile-colore (e.g., or neoajuvant chemotherapy.
yellow) secretions. Bronchoscopy can often ientify the TEF.
Performing tracheostomy early has not been emonstrate 15. B. This patient likely has compression or invasion of
in any large stuies to prevent or ecrease the evelopment his right subclavian vein an possibly brachial plexus ue
300 PArt i Patient Care

to a superior sulcus tumor (Pancoast tumor). Although not proceures (bronchial artery embolization or pulmonary
present in all patients, such as this one, the constellation of artery catheterization an embolization) without rst estab-
symptoms, incluing ipsilateral shouler an arm pain an lishing a safe airway (C, D). Rigi bronchoscopy is the safest
swelling, paresthesias, paresis, an Horner synrome, is means to ientify the source of bleeing, potentially treat
referre to as Pancoast synrome. Nonsmall cell lung cancers the blee an, most importantly, to establish an airway.
account for up to 85% of Pancoast tumors. Small cell lung In the event that bleeing is ientie istal to the carina,
cancer is rarely associate with this synrome. Currently, rigi bronchoscopy allows for the effective intubation of the
best practice for treatment of these malignancies is neoaju- contralateral mainstem bronchus. If a rigi bronchoscope is
vant chemotherapy an raiation followe by resection. In unavailable, exible bronchoscopy can be utilize, although
the previous century, surgery alone (E) as well as neoaju- these ner scopes offer less effective suctioning, which can
vant raiation followe by surgery (C) were foun to be less be critical with signicant hemorrhage (E). Intubation with
effective than neoajuvant chemo-raiation. Chemotherapy a ouble-lumen tube, by itself, will not offer any therapeutic
alone is not the stanar of care (A). The increasing instru- intervention to this patient (A). Furthermore, ouble-lumen
mentation of central veins for ialysis access an pacemaker enotracheal tubes sometimes preclue therapeutic bron-
insertion has le to an increase in central vein stenosis an choscopic intervention given the smaller iameter of each
obstruction from scarring an brosis, for which enovascu- lumen, as well as the possibility that these types of enotra-
lar intervention can be inicate (D). This patient’s swelling cheal tubes will obscure the source of bleeing. Once stable,
is likely ue to malignancy, an thus enovascular interven- this patient can be consiere for thoracic aortogram an
tion is not inicate. selective bronchial arterial embolization.
Reference: Kozower BD, Larner JM, Detterbeck FC, Jones DR. Reference: Kathuria H, Hollingsworth HM, Vilvenhan R,
Special treatment issues in non-small cell lung cancer: iagnosis an Rearon C. Management of life-threatening hemoptysis. J Intensive
management of lung cancer, 3r e: American College of Chest Phy- Care. 00;8(1):3
sicians evience-base clinical practice guielines. Chest. 013;143(5
Suppl):e369S–e399S. 18. B. Pulmonary sequestration is a rare anomaly of the
lung that is classie into two types: intralobar an extralo-
16. E. Overall, approximately 50% of thoracic uct leaks bar, with the former being more common. The key to the
are ue to trauma, of which iatrogenic trauma is the most iagnosis is that they both have no connection to the tra-
likely. The remainer are ue to neoplastic obstruction (most cheobronchial tree (C), with the intralobar type remaining
common is lymphoma). The thoracic uct originates from within the visceral pleura of the native lung an the extralo-
the cisterna chyli locate posterior to the abominal aorta bar type envelope in a separate pleural lining (D). They
an ascens towar the thorax, entering the aortic hiatus at also have their own arterial supply, with the intralobar type
T-1 traveling to the right of the vertebral column. It crosses most commonly receiving its bloo supply from the thoracic
over to the left thorax at T5-6 an rains at the junction of the aorta, while the extralobar type receives its supply from the
subclavian an internal jugular vein. Injury to the thoracic abominal aorta. For reasons that are unclear, the left sie
uct can result in pleural effusion seconary to chylothorax. an lower lobes are more commonly involve. Men are more
Not all cases present with the white milky color suggestive of commonly affecte in a 3:1 ratio. In the largest case series
the iagnosis. Many patients present with blooy, yellow, or involving 65 patients, the most common presentations
serosanguinous effusion. Pleural ui analysis emonstrat- were prouctive sputum, fever, an hemoptysis. The gol
ing chylomicrons an/or triglyceries is highly suggestive stanar to conrm the iagnosis is pulmonary angiography,
of chylothorax. Initial management of chylothorax is focuse but CT angiography is consiere the iagnostic imaging of
on minimizing lymphatic absorption of ietary fats. In the choice because it is less invasive an has high sensitivity/
postsurgical patient, enteral nutrition is preferre to par- specicity (A). Surgical resection (segmentectomy preferre
enteral nutrition. With a J-tube in place, it is reasonable to over lobectomy) has been an remains the stanar of care
change fees to an elemental no-fat iet to minimize the pro- for most patients, given the potential for recurrent infections
uction of chyle. If this fails, it is reasonable to consier NPO an massive hemoptysis (E). Of note, in recent years, as more
an TPN (A). Surgical intervention is reserve for persistent ault cases are incientally ientie via cross-sectional
high-volume lymph leaks, ene as greater than 1 liter per imaging, nonoperative management is increasingly being
ay (B, C). Octreotie can be consiere as a pharmacologic consiere for iniviuals with small asymptomatic lesions.
ajunct (D). References: Alsumrain M, Ryu JH. Pulmonary sequestration in
aults: a retrospective review of resecte an unresecte cases. BMC
17. B. Up to 14% of people presenting with hemoptysis will Pulm Med. 018;18(1):97.
have life-threatening hemoptysis, also known as massive
hemoptysis (greater than 100 mL/hr or 500 mL/4 hr). The 19. B. This patient most likely has small cell lung can-
estimate anatomic ea space of the upper airways is 150 cer complicate by ectopic Cushing synrome ue to the
mL, a volume that can easily be overcome by bleeing espite tumor’s secretion of ACTH. This paraneoplastic synrome
coughing an mucociliary clearance. Sources of hemoptysis is ientie in 1% to 5% of small cell lung cancers. While
are variable but most commonly (up to 90% of cases) involve pulmonary carcinoi can also cause ectopic Cushing syn-
high-pressure bronchial arteries. Initial management shoul rome, in a patient with a signicant smoking history, small
always follow stanar resuscitation protocols, with ensur- cell cancer is more likely. Small cell carcinoma of the lung
ing a secure airway as the primary concern. This patient is (SCLC) accounts for 0% of all lung cancers an is ene
unstable, with evience of aspiration an hypoxia. As such, by its aggressive course with a ismal 5-year survival of
it woul be premature to attempt interventional raiology 5% to 10%. The term limite SCLC is given to patients with
CHAPtEr 21 Thoracic Surgery 301

locoregional isease an offers the only hope for cure. For 22. D. IABP is being use more frequently in patients
those with stage 1 isease (T1-, N0) with no noal isease, with low cariac output states. The balloon is positione in
resection of the primary tumor an meiastinal sampling the escening thoracic aorta just istal to the left subcla-
followe by ajuvant chemotherapy is inicate (D). There vian artery. The principal use of IABP is to augment coro-
is no role for neoajuvant chemotherapy (C). Most patients nary bloo ow an, thus, myocarial oxygen supply. This
with limite SCLC will present with hilar or meiastinal is accomplishe by the balloon eating at systole, thereby
lymph noe involvement, such as this patient. In this case, reucing left ventricular afterloa, an inating at iastole,
enitive chemoraiation is inicate an presents the best resulting in higher iastolic aortic pressure an higher coro-
chance for long-term survival. Overall, most patients present nary perfusion pressure. Coronary bloo ow is improve
with extensive-stage isease ene as metastatic isease uring iastole (C). The three wiely recognize inications
or extensive noal involvement. These iniviuals are often for IABP inclue high-risk percutaneous coronary interven-
exclue from raiation therapy given the toxicity inuce tion, acute myocarial infarction, an cariogenic shock
by the wie raiation el. Most of these iniviuals will be (E). Its use outsie of these clinical scenarios has le to less-
manage enitively with chemotherapy (E). than-ieal outcomes in several recent large, ranomize tri-
References: Non-Small Cell Lung Cancer Treatment (PDQ)– als, causing some to speculate if there is an ae benet in
Health Professional Version. National Cancer Institute. Upate the use of IABP. The major limiting factor in these stuies
January 19, 01. https://www.cancer.gov/types/lung/hp/ was poor patient selection. IABP only works by improving
non-small-cell-lung-treatment-pq myocarial bloo ow, which it can only o when coronary
autoregulation is exhauste; otherwise, the increase coro-
20. C. Bronchial artery embolization is an effective tool nary perfusion will be counteracte by the increase coro-
for treating patients with hemoptysis because most cases nary vascular resistance, which uner normal physiologic
arise from the bronchial circulation rather than the pulmo- conitions works with high elity to guarantee constant
nary artery circulation. Embolization is highly effective in myocarial bloo ow over a wie range of aortic pressures.
stopping the hemoptysis; however, recurrent bleeing will There are several absolute contrainications to IABP, inclu-
evelop in as many as 50% of patients. In approximately 5% ing aortic regurgitation, because it can worsen the mag-
of patients, the bloo supply to the spine (anterior spinal nitue of regurgitation (A). Aitionally, IABP shoul be
artery) may have a common origin with a bronchial artery, avoie in patients with suspecte aortic issection (because
or the bronchial arteries themselves may contribute to the it can exten into the false lumen) an use with caution
spinal bloo supply. As such, the inavertent embolization in patients with abominal aortic aneurysm (because it can
of the spinal artery can result in paralysis an has been esti- result in rupture) (B).
mate to occur in 1% to 4% of cases. The clinician must be Reference: van Nunen LX, Noc M, Kapur NK, Patel MR, Perera
aware of this rare but potentially evastating complication. D, Pijls NHJ. Usefulness of intra-aortic balloon pump counterpulsa-
Clinically apparent necrosis or infarction of the other struc- tion. Am J Cardiol. 016;117(3):469–476.
tures is not well recognize (A, B, D, E). The most common
overall complications are chest pain an transient yspha- 23. C. Given the raining sinus an sulfur granules, the
gia, which can occur in up to 30% of patients. patient most likely has actinomycosis, a chronic isease
Reference: Kathuria H, Hollingsworth HM, Vilvenhan R, usually cause by Actinomyces israelii that occurs most com-
Rearon C. Management of life-threatening hemoptysis. J Intensive
monly in the hea an neck region. Because of its rarity
Care. 00;8(1):3.
an chronicity, the iagnosis is often elaye an unrec-
ognize. A key to the iagnosis is the ning of chronic
21. D. Aortic stenosis is most commonly ue to senile cal-
sinuses with ischarge of purulent material containing yel-
cic aortic valve isease an becomes symptomatic later
low-brown sulfur granules. The organisms enter the lungs
in life. Since the avent of penicillin, rheumatic fever has
via the oral cavity. The organisms are often not culture out
become an uncommon etiology for this isease (B). The clas-
because they are anaerobes. Lung involvement can present
sic signs of aortic stenosis are angina, syncope, an congestive
with progressive pulmonary brosis. Central nervous sys-
heart failure (CHF), which can present with swollen legs an
tem involvement is not common (E). Prolonge, high-ose
elevate brain natriuretic peptie. Of these 3, CHF portens
penicillin is the treatment of choice (D). Surgery is generally
the worst prognosis, with meian survival as low as  years.
not inicate; however, pulmonary actinomycosis can easily
Patients o not have symptoms until the stenosis is severe,
be confuse with a lung cancer, prompting surgical inter-
which occurs when the aortic valve area ecreases below
vention (A). N. asteroides is a gram-positive ro that mimics
1 cm or the mean graient increases above 40 mmHg (C).
fungi microscopically because of its branche lamentous
Aortic an pulmonary stenosis both present with a systolic
morphology an causes nocariosis in immunocompro-
murmur. Symptomatic patients who are appropriate surgi-
mise patients (B). It is associate with pneumonia, eno-
cal caniates shoul unergo aortic valve replacement. In
caritis, an central nervous system abscess. The treatment
high-risk patients (an some intermeiate-risk patients) with
is trimethoprim-sulfamethoxazole.
severe symptomatic AS, transcatheter aortic valve replace-
Reference: Hsieh MJ, Liu HP, Chang JP, Chang CH. Thoracic
ment is preferre (A). Valve repair is preferre over valve
actinomycosis. Chest. 1993;104():366–370.
replacement in patients with mitral valve isease (E).
Reference: Otto CM, Nishimura RA, Bonow RO, et al. 00
ACC/AHA guieline for the management of patients with valvular 24. B. Pulmonary function stuies are routinely performe
heart isease: executive summary: a report of the American college when any resection greater than a wege resection is planne.
of Cariology/American Heart Association Joint Committee on clin- FEV1 is regare as the best preictor of complications of
ical practice guielines. Circulation. 01;143(5):e35–e71. lung resection in the initial assessment of patients. If the
302 PArt i Patient Care

FEV1 is greater than 80% of what is expecte, the patient can Horner synrome; pericarial involvement; an SVC syn-
tolerate a pneumonectomy. Typically, a preoperative FEV1 rome. Surgery may be inicate for selecte patients with
of .0 liters inicates a patient’s tness to unergo pneumo- stage 3A isease in combination with neoajuvant chemo-
nectomy an 1.5 L a lobectomy. One must bear in min that therapy an raiotherapy. A positive ipsilateral meiastinal
these rough guielines o not factor in such things as the lymph noe is N isease (at minimum stage 3A), a poten-
patient’s age, boy size, an preicte postoperative FEV1. tially resectable lesion. A contralateral meiastinal lymph
If the patient’s preoperative FEV1 is borerline, quantitative noe or supraclavicular noe is at least stage 3B (N3 isease).
perfusion lung scanning or SPECT/CT can be use to obtain Patients with stage 1 have only a 50% 5-year survival rate
a preicte postoperative FEV1. Any postoperative value of with resection. Stage  patients have a 5-year survival rate
less than 40% inicates a higher risk for postoperative mor- after surgery of only 30%, whereas those with stage 3A have
tality an morbiity. Aitionally, preoperative DLCO less a 17% 5-year survival rate. The stage 3B survival rate is 5%,
than 50% of what is preicte is associate with increase an the stage 4 survival rate approaches zero.
complications an mortality with pneumonectomy or lobec- Reference: National Comprehensive Cancer Network. NCCN
tomy. (E) Total arterial bloo gas, lung capacity, an resting Clinical Practice Guielines in Oncology: Non-Small Cell Lung
minute ventilation are not inclue in these preictors (A, Cancer. National Comprehensive Cancer Network, Inc. 015; Ver-
C, D). sion 7.015. https://www.tri-kobe.org/nccn/guieline/archive/
lung015-017/english/non_small.pf
25. C. If pulmonary function testing is within normal lim-
its, no further testing is require, an the patient can be 28. B. Aenocarcinoma is the most common lung can-
scheule for surgery (B). If it is below the accepte limits, cer in nonsmokers (an overall). It is also more common in
further testing is recommene, incluing quantitative VQ women an is most commonly a peripheral lesion. Though
scan or SPECT/CT; this permits calculation of postopera- this coul be many ifferent types of malignancy, given the
tive pulmonary reserve. The minimum acceptable preicte patient’s age, lifelong nonsmoking status, an the nings
postoperative FEV1 is 800 mL. If the esire lobe has mini- on chest raiograph, aenocarcinoma is the most likely iag-
mal contribution to FEV1, then the patient can still tolerate a nosis (A, C–E). Aitionally, the pleural effusion is concern-
resection (A). If the preicte FEV1 is less than 800 mL, the ing for stage 4 isease.
patient shoul then be referre to an oncology physician to Reference: Nason KS, Maaus MA, Luketich JD. Chest wall,
lung, meiastinum, an pleura. In: Brunicari FC, Anersen DK,
iscuss nonsurgical management. Respiratory muscle train-
Schwartz SI, es. Schwartz’s principles of surgery. 9th e. McGraw-
ing with an incentive spirometer has not been emonstrate Hill; 010.
to improve pulmonary function test results (D). Breathing
treatments may have a slight improvement in pulmonary 29. D. Active tuberculosis can lea to massive hemoptysis.
function testing but will not correct the unerlying isease Most hemoptysis is ue to bronchial artery bleeing an is
(E). manage via bronchial artery embolization. Rarely, hemop-
tysis is ue to a Rasmussen aneurysm, which is a pulmonary
26. C. The cricoi cartilage is the rst cartilaginous ring of artery aneurysm ajacent to or within a tuberculous cavity
the airway an consists of an anterior arch an a posterior (A–C, E). Such an aneurysm woul be manage by pulmo-
broa-base plate (E). The tracheal bloo supply is segmen- nary arteriography an selective istal embolization. CT
tal via the inferior thyroi an bronchial arteries (A). Each scanning is useful in hemoptysis to help localize the source
arterial branch supplies a 1- to -cm length of the trachea. an guie interventional management.
Circumferential mobilization will isrupt the bloo sup- Reference: Picar C, Parrot A, Boussau V, et al. Massive
ply (B). The trachea has approximately 18 to  rings an is hemoptysis ue to Rasmussen aneurysm: etection with helicoial
approximately 10 to 13 cm long. As much as 6 cm of length CT angiography an successful steel coil embolization. Intensive Care
can be resecte primarily using laryngeal release proceures. Med. 003;9(10):1837–1839.
A tracheostomy is ieally place between the secon an
thir or thir an fourth tracheal rings; higher placement 30. D. Thymoma is the most common neoplasm of the
increases the risk of tracheal stenosis an lower placement anterior meiastinum. Malignancy is etermine base on
increases the risk of tracheoinnominate stula (D). evience of local invasion of ajacent structures or capsu-
lar invasion, not on cellular or histologic characteristics (B).
27. C. Stage 4 nonsmall cell lung cancer is treate primar- Treatment is by surgical resection (A). Thymomas are raio-
ily with enitive chemotherapy an raiation therapy. sensitive, so raiation therapy is use as an ajunct in locally
There may be a role for surgical intervention for palliation avance cases. As many as 50% of patients with thymomas
of symptoms (e.g., thoracentesis or pleural winow for have symptoms of myasthenia gravis. Conversely, less than
recurrent pleural effusions), but, in general, stage 4 isease 10% of patients with myasthenia gravis are foun to have
is not manage surgically. Of the aforementione nings, a thymoma on imaging (C). Nevertheless, thymectomy
malignant pleural effusion is a marker for stage 4 isease (an improves or resolves symptoms of myasthenia gravis in as
effusion with malignant cells is consiere M1a isease) (A, many as 90% of patients without a thymoma, compare with
B, D, E). Other clinical nings that are iagnostic of stage only approximately 5% of patients with thymomas. Due to
4 isease inclue istant metastases, a positive contralat- their location, large thymomas can present with SVC syn-
eral meiastinal lymph noe, an bilateral enobronchial rome (E).
tumors. Attempts at surgical resection are generally reserve
for stages 1 to 3A. Relative contrainications to surgical 31. A. An increase in overall survival has been achieve
intervention inclue recurrent laryngeal nerve involvement; with the resection of isolate lung metastases (B–E). This
CHAPtEr 21 Thoracic Surgery 303

is especially true of osteogenic sarcoma, but it has been pulmonary metastases ue to osteogenic sarcoma treate
reporte for other malignancies as well. Prior to metasta- with metastasectomy have achieve similar positive results
sectomy, however, several conitions must be met. Ieally, as solitary metastatic lesions. Factors associate with sur-
lung metastases present metachronously, an the primary vival following metastasectomy inclue a isease-free inter-
tumor has alreay been controlle; the metastatic lesion val from primary tumor to initial evience of metastasis,
shoul be completely resectable, an there shoul be no surgical resectability, tumor oubling time, an the number
evience of iffuse carcinomatosis. Pulmonary metastasis of metastases.
occurs in as many as 40% to 60% of all primary sarcomas of Reference: Marulli G, Mammana M, Comacchio G, Rea F.
the limbs within 3 years, an a 30% to 50% 5-year survival Survival an prognostic factors following pulmonary metasta-
rate can be achieve with metastasectomy. In general, soli- sectomy for sarcoma. J Thorac Dis. 017;9(Suppl 1):S1305–S1315.
tary metastases have a better prognosis. However, multiple oi:10.1037/jt.017.03.177
Pediatric Surgery
ALEXANDRA MOORE, VERONICA SULLINS, AND STEVEN L. LEE 22
ABSITE 99th Percentile High-Yields
I. Hernias
A. Inguinal:
1. Etiology is patent processus vaginalis (inirect hernia) in 99%; tx is high ligation, no mesh
. Risk of incarceration is inversely proportional to age (younger patients have higher risk of
incarceration); 5% of patients have contralateral hernia not etecte clinically
B. Umbilical:
1. Repair if symptomatic or at 4 years of age or oler
. If efect is < cm, there is >95% chance of spontaneous resolution
C. Congenital Diaphragmatic Hernia:
1. Most common is Bochalek (posterolateral), on left sie; usually iagnose on prenatal US
. Pulmonary hypertension causes hypoxia an signicant morbiity; pulmonary arteries are
anatomically ifferent an less responsive to pulmonary vasoilators (such as nitric oxie)
3. Pulmonary hypoplasia occurs in both lungs, with ipsilateral lungs, more affecte; pulmonary
hypoplasia will result in hypercapnia
4. Management:
a) Start with NG tube ecompression an respiratory support; intubation with gentle mechanical
ventilation strategy with permissive hypercapnia to minimize barotrauma; may nee ECMO
b) Surgical repair elaye, allowing pulmonary hypertension to improve or stabilize

II. Pyloric Stenosis


A. Nonbilious, projectile vomiting in a 3- to 6-week-ol; may have palpable “olive-like” abominal mass
B. Hypochloremic, hypokalemic metabolic alkalosis; alkalosis ue to vomiting HCl; hypokalemia from
renal loss of potassium (ehyration leas to activation of the renin, angiotensin, alosterone system,
alosterone causes renal loss of potassium)
C. Paraoxical aciuria: Urine is transiently aciic ue to Na conservation with excretion of H (prevents
worsening of hypokalemia)
D. Diagnose with ultrasoun, will show hypertrophie pylorus (>3 mm thick, >15 mm long)
E. Management:
1. Start with resuscitation! Normal saline bolus, maintenance IV uis at 1.5× maintenance rate, a
KCl to maintenance uis once patient urinates
. Goal is for patient to be resuscitate with correction of electrolyte abnormalities prior to surgery;
preop lab goals: pH <7.45, base excess <3.5, bicarb <6, Na >13, K >3.5, Cl >100, glucose >7;
increase risk of postoperative apnea if uncorrecte alkalosis prior to surgery
3. Surgery is Freet-Ramstet pyloromyotomy: partial-thickness longituinal incision in the pylorus
1 to  mm proximal to the uoenum, exten proximally to normal antrum; recurrence most
commonly a result of not extening myotomy far enough proximally to antrum

305
306 PArt i Patient Care

III. Malrotation an Migut Volvulus


A. Presentation is typically bilious emesis ± abominal istention
B. Diagnosis is via UGI series (uoenum oes not cross miline)
C. If peritonitis is present, then avoi UGI an immeiately go to OR
D. La proceure to minimize chance of future volvulus (create complete nonrotation anatomy)
1. De-torse the bowel in a counterclockwise fashion (“turn back the clock”)
. Divie La bans (peritoneal attachments of right colon to paracolic gutter)
3. Straighten the uoenum an x the uoenum in the right upper quarant
4. Mobilize colon to patient’s left an x the cecum in the left lower quarant
5. Wien the base of the mesentery (key component)
6. ± Appenectomy

IV. Esophageal Atresia an Tracheoesophageal Fistula


A. Echocariogram to rule out associate cariac anomalies (right arch in 1%–%)
B. Types:

Type Esophageal atresia Tracheoesophageal ęstula


Type A Present Absent
Type B Present Present; ęstula with proximal esophagus
Type C Present Present; ęstula with distal esophagus
Type D Present Present; ęstula with both proximal and distal esophagus
Type E Absent Present, also called H-type

C. Type C most common; treat with Immeiate repair


D. Stage repair for Type A
1. Initial g-tube placement an allow patient to grow until two ens are close enough for repair

V. Abominal Wall Defects

Factor Omphalocele Gastroschisis


Sac Present Absent
Location of defect Central (through umbilicus) To the right of the umbilicus
Umbilical cord Inserts into sac Normal
Defect Size Large Small
Contents Bowel, liver Bowel, gonads
Bowel Normal MaĴed
Malrotation Present Present
Small abdomen Present Present
GI function Normal Prolonged ileus
Associated anomalies Common (30%–70%) Unusual (atresia 15%)
Associated syndromes Beckwith Wiedemann, Trisomy, Not observed
Cantrell
CHAPtEr 22 Pediatric Surgery 307

Fig. 22.1 Gastroschisis vs. Omphalocele.

Fig. 22.2 The La Proceure.


308 PArt i Patient Care

Questions
1. A 9-year-ol boy is seen in the emergency room 4. A 1-ay-ol ex-7-week premature boy was
with a 1-ay history of right lower quarant previously avancing well on enteral fees. He
abominal pain an low-grae fever. On exam, becomes acutely istene. Initial abominal
he is focally tener in the right lower quarant. raiographs reveal moerate pneumatosis
WBC count is 15,000/mcL an US shows a 9-mm intestinalis an enteral feeings were hel. Three
noncompressible appenix an an appenicolith. hours later, a repeat abominal raiograph
Which of the following is true about this reveals pneumoperitoneum. The patient is
conition? brought emergently to the operating room for
A. If nonoperative management with antibiotics laparotomy where three areas of necrotic bowel
is to be consiere, a CT scan shoul be rst are encountere along with numerous other areas
obtaine of patchy ischemia. What is the next best step in
B. Success of nonoperative management in this management?
patient is anticipate to be very high A. Resection of necrotic bowel only, with primary
C. Nonoperative management tens to result anastomoses
in shorter hospital stay as compare to B. Resection of both necrotic an patchy ischemic
appenectomy bowel with primary anastomosis
D. Appenectomy is preferre in this patient C. Place rains without bowel resection
E. Failure of nonoperative management is likely D. Resection of all necrotic an ischemic bowel
to manifest as peritonitis with primary anastomosis an proximal
iverting stoma
2. A 10-year-ol boy is a restraine passenger in a E. Resection of necrotic bowel only, leave in
high-spee motor vehicle collision. On arrival to iscontinuity, secon look in about 48 hours
the emergency epartment, his heart rate is 140
beats per minute an his systolic bloo pressure 5. A previously healthy -month-ol girl is brought
is 80 mmHg. There is an obvious eformity of to the emergency epartment ue to a -hour
his left thigh. GCS is 13. Pupils are equal an history of intermittent inconsolable crying,
reactive. Abomen is milly tener to palpation. vomiting, an apparent pain. She is not eating.
Focuse assessment with sonography in trauma Her parents brought her to the hospital after she
(FAST) is positive for peritoneal ui. He is passe a loose, maroon-colore stool. There are
aministere 0 mL/kg of crystalloi, an BP no signs of peritonitis on exam. WBC count is
remains 80 mmHg. Which of the following is the normal. Which of the following is recommene?
most appropriate next step? A. CT scan of the abomen
A. CT scan of hea/abomen/pelvis B. Laparoscopy
B. Start bloo prouct transfusion an transport C. Colonoscopy
to the OR for exploratory laparotomy D. Nuclear scan
C. Infuse aitional bolus of isotonic crystalloi E. Abominal ultrasoun
D. Infuse lactate ringers
E. Infuse 3% hypertonic saline 6. A 13-year-ol female presents with severe right
lower quarant pain an emesis. At laparoscopy
3. A 6-month-ol girl is brought to the trauma center an ovarian torsion is foun. The ovary appears
for evaluation of a hea injury. Parents report swollen with a blueish-black iscoloration. It
that the patient rolle off a be. Which of the remains unchange after etorsion. The next step
following injuries suggest abusive hea trauma? in management is:
A. Isolate skull fracture A. Biopsy
B. Hea an neck bruising B. Oophoropexy
C. Subural hematoma C. Oophorectomy
D. Epiural hematoma D. Salpingo-oophorectomy
E. Cortical contusion E. Close an obtain serial ultrasoun
CHAPtEr 22 Pediatric Surgery 309

7. A full-term baby girl has a iagnosis of a right- 11. A full-term, healthy newborn boy is note to have
sie congenital lung malformation ientie imperforate anus. After 4 hours, no meconium
on prenatal imaging. Chest raiograph in the is visualize in the perineal area. The most
newborn nursery shows a cystic lesion in the appropriate management shoul be:
right lower lobe with no meiastinal shift. She is A. Observation for another 4 hours
asymptomatic an on room air. What is the next B. Diverting ileostomy
step in management? C. Sigmoi colostomy
A. CT scan of the chest prior to ischarge D. Primary repair through the perineum
B. Discharge with CT angiogram of the chest E. Laparoscopic primary repair
within 6 months
C. Right lower lobectomy 12. A -month-ol infant has persistent jaunice.
D. Right tube thoracostomy Ultrasonography fails to emonstrate a
E. Inpatient MRI of the chest gallblaer. Technetium-99m hepatobiliary
iminoiacetic aci (HIDA) scanning with
8. During laparoscopy for early acute appenicitis phenobarbital pretreatment reveals uptake in the
in a 5-year-ol boy, you n a large, right-sie liver but not in the intestine. α1-Antitrypsin an
renal mass. You perform an appenectomy an: cystic brosis etermination is normal. The most
A. Close, then obtain further workup appropriate surgical management woul be:
B. Biopsy the mass A. Kasai operation (hepatoportoenterostomy)
C. Right nephroureterectomy B. Liver transplantation
D. Right nephroureterectomy with ipsilateral C. Percutaneous transhepatic liver rainage
lymph noe sampling D. Enoscopic biliary stent placement
E. Right nephroureterectomy with ipsilateral E. Choleochojejunostomy
lymph noe sampling an contralateral renal
biopsy 13. A 1-ay-ol full-term infant presents with bilious
emesis. Abominal x-rays show multiple loops
9. A -week-ol boy presents with constipation an of ilate bowel. A contrast enema shows a
abominal bloating. He faile to pass meconium microcolon. What is the pathophysiology behin
on the rst  ays of life. Contrast enema this obstruction?
emonstrates a slightly ilate sigmoi colon A. A fetal mesenteric vascular accient
with a constricte rectum. What is the next most B. Failure of recanalization of the bowel
appropriate step in management? C. Lack of proper rotation of the bowel
A. Rectal irrigations an IV antibiotics D. Lack of ganglion cells in the bowel
B. Creation of a leveling ostomy E. A uplication of a segment of bowel
C. Suction rectal biopsy
D. Change to an elemental formula 14. A newborn baby is born with an abominal wall
E. Obtain a UGI contrast series with small bowel efect. The efect involves the umbilicus an
follow-through has a membrane associate with it. Which of the
following is true regaring this type of efect?
10. A newborn is in severe respiratory istress an A. This patient requires immeiate surgical closure
has a markely scaphoi abomen. Which of the B. Mortality is most often the result of persistent
following is true regaring this conition? sepsis
A. A chest tube shoul be promptly place C. The etiology is ue to an umbilical vein
B. The patient shoul be ventilate with vascular accient
bag-mask ventilation D. The efect is usually associate with intestinal
C. Severe cases may benet from extracorporeal atresia
membrane oxygenation E. These patients commonly have associate
D. Ventilation with high-frequency oscillation is cariac an genetic abnormalities
contrainicate
E. Urgent thoracotomy is require 15. The most common inication for extracorporeal
membranous oxygenation (ECMO) in neonates is:
A. Congenital iaphragmatic hernia
B. Respiratory istress synrome
C. Meconium aspiration
D. Persistent pulmonary hypertension
E. Congenital cariac abnormalities

AL GRAWANY
310 PArt i Patient Care

16. Which of the following is true regaring 21. Operative management for a patient with
Bochalek type of congenital iaphragmatic malrotation an migut volvulus typically
hernia (CDH)? inclues reuction of the volvulus, ivision of
A. Urgent surgical repair is inicate upon La bans, an which of the following?
iagnosis A. Placement of the small intestine in the left
B. Associate pulmonary hypoplasia leas to lower quarant
hypocarbia B. Cecopexy an gastropexy
C. Most efects are on the right C. Broaen base of the small bowel mesentery
D. Pulmonary hypertension is a prominent D. Placement of the cecum in the right upper
feature quarant
E. The iaphragmatic efect is anteromeial E. Reconstruction of the ligament of Treitz

17. A full-term baby is born with rooling, coughing, 22. A full-term baby boy is note to have facial
an cyanosis after the rst feeing, but these features of trisomy 1 an bilious emesis. The
resolve quickly an spontaneously. The next step rest of his exam is normal. Abominal x-rays
in management shoul be: show a ouble-bubble sign with no istal gas.
A. Immeiate intubation Which of the following is the best next step in
B. Placement of orogastric tube management?
C. Two-view abominal x-ray A. Serial abominal x-rays
D. Two-view chest x-ray B. UGI contrast stuy
E. Upper gastrointestinal (UGI) contrast series C. Contrast enema
D. Operative exploration
18. A patient is iagnose with pyloric stenosis E. Echocariogram
after 3 ays of nonbilious emesis. This patient’s
electrolyte an aci/base balance will result in: 23. A -year-ol chil presents with an abominal
A. Respiratory alkalosis mass, “raccoon eyes,” an “blueberry mufn”
B. Hyperkalemia skin lesions. These most likely represent:
C. Aciuria A. Rhabomyosarcoma
D. Hyperchloremia B. Neuroblastoma
E. Hyponatremia C. Wilms tumor
D. Hepatoblastoma
19. A 900-g premature infant evelops formula E. Teratoma
intolerance with vomiting, abominal istention,
an blooy stools. Labs show an elevate white 24. The most common anomaly associate with
bloo cell (WBC) count an platelets of 100,000/ gastroschisis is:
mcL. Abominal x-rays show ilate loops of A. Cariac
bowel with pneumatosis intestinalis. The most B. Renal
appropriate treatment woul be: C. Limb
A. Bloo an platelet transfusions D. Malrotation
B. Antibiotics an bowel rest/ecompression E. Down synrome
C. Ultrasoun an paracentesis
D. Placement of a besie peritoneal rain 25. A newborn baby is born with a istene
E. Exploratory laparotomy abomen an bilious emesis. Both parents are
carriers for cystic brosis. On examination,
20. A healthy -week-ol girl evelops bilious the patient has a istene but soft abomen.
emesis. On exam, her abomen is nontener Abominal x-rays show ilate loops of bowel
an nonistene. What is the most appropriate with a groun-glass appearance. The most
stuy to make the iagnosis? appropriate initial management is:
A. -view abominal x-ray A. Water-soluble contrast enemas
B. Ultrasoun B. Resection of terminal ileum with stoma
C. UGI series C. Resection of terminal ileum with primary
D. Contrast enema anastomosis
E. Compute tomography (CT) scan of D. UGI with small bowel follow-through
abomen/pelvis E. Small bowel enterotomy with evacuation of
meconium
CHAPtEr 22 Pediatric Surgery 311

26. A 6-month-ol boy presents to the ED crying in 30. A newborn baby with a prenatal iagnosis
pain an has bilious emesis. On exam, he has a of gastroschisis is born with the entire small
istene abomen, an there is a tener mass intestine outsie of the abomen. The bowel
in the right groin. Appropriate management appears ischemic an the abominal wall efect
woul be: is small an tight. The most appropriate next step
A. Ultrasoun of right groin in management is:
B. Besie incision an rainage (I&D) of right A. Place a besie silo
groin B. Primary reuction an closure
C. IV antibiotics C. Open the abominal wall efect
D. Attempt reuction D. Resect the ischemic bowel
E. Operative exploration E. Create a iverting ileostomy

27. A 4-year-ol girl presents with recurrent jaunice. 31. A 1-week-ol full-term baby with abominal
Ultrasoun shows a 5-cm fusiform ilation of the istention, fever, tachycaria, an low urine
common bile uct. During surgery, the posterior output is transferre to the NICU. The patient has
aspect of the cystic mass is rmly aherent to the not passe meconium. He ha a suction rectal
portal vein. The most appropriate management is: biopsy showing aganglionosis. Digital rectal
A. Abort surgery, IV antibiotics, an reoperate in examination shows explosive, foul-smelling
3 months liqui stools. Despite broa-spectrum IV
B. Place a rain into the cyst, IV antibiotics, an antibiotics an rectal irrigation, he is clinically
reoperate in 3 months eteriorating. The next step in management is to:
C. Resect the anterior cyst, mucosectomy of the A. Perform contrast enema
posterior cyst with reconstruction B. Perform loop colostomy
D. Internal rainage of the cyst with a Roux-en-Y C. Perform subtotal colectomy an ileostomy
cystojejunostomy D. Perform abominal ecompression for
E. Resect the cyst an portal vein with abominal compartment synrome
reconstruction of the portal vein an common E. A aitional antifungal coverage
bile uct (CBD)
32. A 4-week-ol infant presents with bilious
28. A -week-ol, ex-5-week premature boy is in vomiting, irritability, abominal wall eema, an
the neonatal ICU (NICU) an is iagnose with erythema. Plain lms reveal proximal ilate
a left inguinal hernia. His current weight is 1 kg bowel, with a paucity of istal bowel gas. Which
an he requires supplemental oxygen. The hernia is true regaring this patient?
is easily reucible. The next appropriate step in A. An urgent UGI series is inicate
management is: B. A trial of nasogastric tube ecompression is
A. Ultrasoun evaluation often helpful
B. Immeiate open operative repair C. Enoscopic ecompression is often benecial
C. Immeiate laparoscopic repair D. A CT scan of the abomen an pelvis shoul
D. Repair just prior to ischarge be obtaine
E. Delay repair until 1 year of age E. Delay in management may lea to a nee for
intestinal transplantation
29. The pathogenesis of necrotizing enterocolitis
(NEC) is thought to be relate to: 33. A neonate is foun to have bilateral unescene
A. A genetic preisposition testes that are not palpable in the inguinal canal.
B. An enzyme eciency Which of the following is true regaring this
C. A perio of intestinal hypoperfusion conition?
D. Preexisting intestinal atresia A. A bilateral orchiopexy shoul be performe by
E. An antibiotic reaction 1 year of age
B. Orchiopexy oes not improve fertility potential
C. It is not associate with prune belly synrome
D. Chorionic gonaotropin oes not ai in
testicular escent
E. The testicular arteries must be preserve
uring operation
312 PArt i Patient Care

Answers
1. D. Appenectomy is the preferre treatment strategy patients who have clearly experience signicant hemor-
for this patient because of the presence of a fecolith. While rhage (C). Hypervolemia shoul also be avoie in patients
appenectomy has been the gol stanar for the treatment with traumatic brain injury as it can result in seconary
of uncomplicate appenicitis, multiple stuies have emon- insults to the brain. Although massive transfusion has been
strate that nonoperative management of uncomplicate variably ene, the most wiely accepte peiatric eni-
appenicitis is safe an effective. Nonoperative management tion is the transfusion of bloo proucts in excess of 40 mL/
consists of initial broa-spectrum IV antibiotics an IV u- kg over the rst 4 hours following injury. This pragmatic
is. Patients can be transitione to oral antibiotics an is- threshol reliably ientie critically injure chilren at
charge when their pain improves, fever resolves, an they risk for 4-hour an in-hospital mortality in a combat-injury
are able to tolerate a iet. The total antibiotic course shoul trauma cohort of patients less than 18 years of age. While
be 7 ays. Nonoperative management is initially successful there is goo evience in aults that the implementation of
in 85% to 9% of patients. Patients manage nonoperatively massive transfusion protocols is associate with improve
have a higher reamission rate within 1 year, primarily ue mortality an morbiity an ecrease total bloo use, the
to recurrent appenicitis. Due to recurrence of appenicitis, evience in chilren is less clear, with no stuies to ate
the 1-year success rate of avoiing appenectomy is 67%. showing reuce mortality or morbiity associate with the
Most woul recommen only offering nonoperative man- implementation of an MTP in a peiatric trauma center.
agement in chilren age 5 to 17 years meeting the following References: Acker SN, Ross JT, Partrick DA, DeWitt P, Bensar
criteria: uncomplicate appenicitis conrme on imaging DD. Injure chilren are resistant to the averse effects of early
(US, CT, or MRI) (A), WBC between 5000 an 18,000, pain high volume crystalloi resuscitation. J Pediatr Surg. 014;49(1):
for <48 hours, localize tenerness, an no appenicolith. 85–1855.
Duchesne JC, Heaney J, Guiry C, et al. Diluting the benets
Appenicolith has been associate with failure of nonopera-
of hemostatic resuscitation: a multi-institutional analysis. J Trauma
tive management of appenicitis (B). Failure of nonoperative
Acute Care Surg. 013;75(1):76–8.
management can manifest as worsening or persistent symp- Leeper CM, McKenna C, Gaines BA. Too little too late: hypo-
toms or as systemic sepsis espite antibiotic treatment; an tension an bloo transfusion in the trauma bay are inepenent
usually oes not manifest as iffuse peritonitis (E). While preictors of eath in injure chilren. J Trauma Acute Care Surg.
some stuies have emonstrate that chilren treate non- 018;85(4):674–678.
operatively for appenicitis may return to school sooner, the Magoteaux SR, Notrica DM, Langlais CS, et al. Hypotension an
hospital length of stay is longer with nonoperative manage- the nee for transfusion in peiatric blunt spleen an liver injury:
ment than with appenectomy (C). an ATOMAC+ prospective stuy. J Pediatr Surg. 017;5(6):979–83.
References: Minneci PC, Sulkowski JP, Nacion KM, et al. Fea- Notrica DM, Eubanks JW, 3r, Tuggle DW, et al. Nonoperative
sibility of a nonoperative management strategy for uncomplicate management of blunt liver an spleen injury in chilren: evaluation
acute appenicitis in chilren. J Am Coll Surg. 014;19():7–79. of the ATOMAC guieline using GRADE. J Trauma Acute Care Surg.
Patkova B, Svenningsson A, Almström M, Eaton S, Wester T, 015;79(4):683–693.
Svensson JF. Nonoperative treatment versus appenectomy for
acute nonperforate appenicitis in chilren: ve-year follow 3. B. The American Acaemy of Peiatrics prefers the term
up of a ranomize controlle pilot trial. Ann Surg. 00;71(6): abusive head trauma (AHT) to “shaken baby synrome.”
1030–1035. In chilren less than  years of age, nonacciental trauma
Shinoh J, Niwa H, Kawai K, et al. Preictive factors for negative accounts for 10% of hea injuries. However, AHT accounts
outcomes in initial non-operative management of suspecte appen- for over half of serious hea injury morbiity an mortal-
icitis. J Gastrointest Surg. 010;14():309–314. ity. Improve iagnosis is important because up to one-thir
of AHT cases are misse on initial presentation leaing to
2. B. Trauma is the most common cause of chilhoo mor- aitional repeate injury. AHT is typically the result of vig-
tality. Compare to aults, chilren remain hemoynami- orous shaking leaing to an acceleration-eceleration force.
cally compensate until 30% to 45% of their bloo volume This type of force causes isruption of cortical veins resulting
is lost. Crystalloi resuscitation shoul be initiate ue to in interhemispheric subural or subarachnoi hemorrhage.
its immeiate availability, although infuse volumes shoul Aitional associate injuries inclue iffuse axonal injury,
be limite to 0 mL/kg in the chil who is a hemoynamic shear injury, white matter tears, an retinal hemorrhage,
nonresponer. In patients who o not respon to a single (C). In contrast, the most frequent acciental hea injuries
0 mL/kg isotonic ui bolus, pRBC transfusion is the next result from impact, proucing linear skull fractures, epiural
most appropriate step in resuscitation (D). Trauma patients hematomas, localize homogenous subural hemorrhages,
with a positive FAST who are hemoynamically unrespon- an cortical contusion (A, D, E). A metaanalysis ientie
sive to ui or bloo resuscitation require operative explora- retinal hemorrhage, lack of aequate history, subural hem-
tion (A). Hypertonic saline may be useful in trauma patients orrhage, an metaphyseal an rib fractures as the most inic-
with close-hea injuries that are suspecte to have elevate ative nings of abusive hea trauma.
intracranial pressure (e.g., blown pupil on exam suggest- Reference: Piteau SJ, War MG, Barrowman NJ, Plint AC.
ing uncal herniation) (E). Excessive crystalloi resuscitation Clinical an raiographic characteristics associate with abu-
leas to the hemoilution of clotting factors, worsening sive an nonabusive hea trauma: a systematic review. Pediatrics.
coagulopathy, an metabolic aciosis shoul be avoie in 01;130():315–33.
CHAPtEr 22 Pediatric Surgery 313

4. E. This infant is presenting with surgical necrotizing be one to conrm the iagnosis (D). Once iagnose, about
enterocolitis (NEC). Very low birthweight infants are at the 80% of patients with intussusception are successfully treate
highest risk for NEC. The etiology of NEC is mucosal com- with pneumatic or hyrostatic reuction, so laparoscopy at
promise in the presence of pathogenic bacteria. NEC is fre- this point woul not be appropriate (B). Repeat cases shoul
quently associate with the introuction of enteral feeings. again unergo pneumatic or hyrostatic reuction as long as
This leas to bowel injury an an inammatory cascae. the patient is stable an without peritonitis. Colonoscopy is
Pneumoperitoneum is an absolute inication for surgical not inicate in the initial workup for intussusception (C).
intervention. Areas of frank necrosis shoul be resecte. References: Columbani PM, Scholz S. Intussusception. In:
In cases with patchy ischemia where the viability of the Coran AG, e. Pediatric surgery. 7th e. Mosby; 01:1093–1110.
bowel is unclear, potentially salvageable bowel shoul not Jiang J, Jiang B, Parashar U, Nguyen T, Bines J, Patel MM. Chil-
be resecte to reuce the risk of short gut synrome. (B, D) hoo intussusception: a literature review. PLoS One. 013;8(7):e6848.
In these cases, the necrotic segments shoul be resecte, left
in iscontinuity, an roppe back into the abomen (“clip 6. E. While the escription of a swollen, bluish-black ovary
an rop”) with plans for reexploration in 4 to 48 hours. even after etorsion may seem inicative of ovarian necrosis,
When areas of ischemia are present, primary anastomo- this appearance is most often ue to vascular an lymphatic
ses shoul not be attempte until the viability of all bowel congestion an not necrosis. Frank ovarian necrosis at the
segments has been establishe (A). In some cases, patients time of surgery is rare an woul appear as a gelatinous or
will evelop total intestinal necrosis with no apparent viable poorly ene structure that falls apart when manipulate.
bowel. These patients shoul be close without resection so The color of the ovary after etorsion is not preictive of
that a thorough iscussion can occur with the family about follicular evelopment or future pregnancy, an stuies of
the implications of potential operative intervention (C). patients unergoing etorsion alone show follicular recov-
A common teaching is that NEC classically presents with ery on follow-up ultrasoun. Furthermore, the pathology
blooy stools after the rst feeing. However, the earliest of ovarian torsion in aolescents is preominantly benign.
signs are nonspecic, incluing apnea, braycaria, lethargy, It is for these reasons that every effort shoul be mae to
an temperature instability. The most common GI symptoms spare the ovary. A follow-up ultrasoun in 3 months shoul
are feeing intolerance an high gastric resiuals, while the be performe to evaluate the ovary for follicles or a mass
most common sign is abominal istention. Grossly blooy that coul not be seen at the time of surgery. Unfortunately, a
stools are infrequently seen. Management is initially con- stuy of a large inpatient nationwie atabase emonstrate
servative with NPO, ui resuscitation, broa-spectrum IV that oophorectomy is performe in nearly 80% of females
antibiotics, TPN, an ecompression with an orogastric tube. less than 18 years ol with ovarian torsion (C). Biopsy or
Surgical intervention is inicate for failure of conservative salpingo-oophorectomy are not inicate (A, D). There is no
management, free air on plain lms or CT, an peritonitis. clear evience to support oophoropexy with a rst episoe
References: Dominguez KM, Moss RL. Necrotizing enteroco- of unilateral ovarian torsion. However, it may be performe
litis. In: Holcomb GW III, Murphy JP, Ostlie DJ, St. Peter SD, es. in recurrent or bilateral ovarian torsion or in a patient who
Ashcraft’s pediatric surgery. 6th e. Sauners; 014:454–473. has previously lost an ovary (B).
Ron O, Davenport M, Patel S, et al. Outcomes of the “clip an References: Geimanaite L, Trainavicius K. Ovarian tor-
rop” technique for multifocal necrotizing enterocolitis. J Pediatr sion in chilren: management an outcomes. J Pediatr Surg.
Surg. 009;44(4):749–754. 013;48(9):1946–1953.
Sola R, Wormer BA, Walters AL, Henifor BT, Schulman AM.
National trens in the surgical treatment of ovarian torsion in chil-
5. E. This patient is presenting with classic signs of intus- ren: an analysis of 041 Peiatric Patients Utilizing the Nationwie
susception. These symptoms inclue colicky abominal pain Inpatient Sample. Am Surg. 015;81(9):844–848.
an “currant jelly” maroon stools. The rst-line imaging Fuchs N, Smorgick N, Tovbin Y, et al. Oophoropexy to prevent
moality is abominal ultrasoun, which will emonstrate a anexal torsion: how, when, an for whom? J Minim Invasive Gyne-
target-sign (A). A patient in this age group an with no other col. 010;17():05–08.
prior history is most likely to have ileocolic intussusception,
with a target-sign visualize in the right lower quarant. 7. B. Management of asymptomatic congenital cystic
The most common etiology is hypertrophy of Peyer patches, lung malformations is somewhat controversial, an if sur-
which are circumferentially locate an more closely space gical resection is performe, it is typically within the rst
in the istal ileum, accounting for the ileocolic intussuscep- 6 months of age. The ata regaring asymptomatic lesions
tion being the most common location. Pathologic lea points becoming symptomatic is variable. However, in raio-
account for roughly 5% of intussusceptions in chilren uner graphically ientiable lesions, up to 85% of patients may
3 to 4 years of age. Pathologic lea points may cause intus- become symptomatic, an there is a 4% risk of malignancy.
susception in locations other than the ileocolic. Examples of Regarless of management strategy, it is important to obtain
pathologic lea points inclue Meckel iverticulum, staple cross-sectional imaging to further characterize the lesion an
line from prior bowel resection, thick stool in cystic bro- inform ecisions. Computerize tomography (CT) angio-
sis, intestinal atresias (seen in the neonatal setting), polyps, gram of the chest (B) can istinguish a congenital pulmonary
appenicitis, intestinal lymphoma, submucosal hemorrhage, airway malformation (CPAM) from a pulmonary seques-
foreign boies, an intestinal uplication. The most com- tration or hybri lesion. Compare to a CPAM, a broncho-
mon pathologic lea point for intussusception in chilren is pulmonary sequestration oes not communicate with the
a Meckel iverticulum. A nuclear scan can be use to iag- native airway an has a systemic feeing artery that often
nose a Meckel iverticulum; however, this presentation is arises from the intraabominal aorta. CT can also ifferen-
more concerning for intussusception, so ultrasoun shoul tiate between subtypes of CPAMs base on the size of the
314 PArt i Patient Care

cystic components (microcystic versus macrocystic). Imme- istention an tenerness an is associate with manifes-
iate postnatal CT scans (A) are often suboptimal, so it is rec- tations of systemic toxicity. Enterocolitis is the most com-
ommene to efer CT scan of asymptomatic patients until mon cause of eath in uncorrecte Hirschsprung isease.
closer to potential intervention. Similarly, MRI (E) is not nec- The initial management of a patient with Hirschsprung-as-
essary in an asymptomatic patient an may require unneces- sociate enterocolitis is rectal irrigation an IV antibiotics
sary seation. Immeiate surgical intervention (C–D) is not (A). The enitive iagnosis of Hirschsprung isease is
typically performe in asymptomatic patients. mae by rectal biopsy at least  cm above the entate line
References: Downar CD, Calkins CM, Williams RF, et al. Treat- to avoi sampling error. A contrast enema is useful because
ment of congenital pulmonary airway malformations: a systematic it will often help localize the transition zone between the
review from the APSA outcomes an evience base practice com- ilate proximal ganglion containing the colon an the nar-
mittee. Pediatr Surg Int. 017;33(9):939–953. rowe aganglionic istal segment, but it is not as helpful in
Durell J, Thakkar H, Goul S, Fowler D, Lakhoo K. Pathology
the immeiate neonatal perio because the proximal seg-
of asymptomatic, prenatally iagnose cystic lung malformations.
ment may not be as markely ilate yet. A small bowel
J Pediatr Surg. 016;51():31–35.
follow-through is not helpful because the obstruction is
8. A. This chil most likely has a Wilms tumor, the most in the colon (E). Multiple surgical operations exist for the
common primary renal tumor in chilren less than 15 years management of Hirschsprung isease. Recently, primary
of age. In this case, only the appenectomy shoul be per- repair with a pull-through proceure without a temporary
forme, an the patient shoul have a proper staging colostomy has been performe. A leveling colostomy may
workup (A). This inclues an ultrasoun of the abomen to be performe as part of a stage proceure. However, this
conrm that the mass is originating from the kiney, eval- is only one after the iagnosis is mae (B). Most patients
uating the contralateral kiney, an assessing for extension with Hirschsprung isease will tolerate breast milk or nor-
into the renal collecting system an inferior vena cava. The mal formulas after surgery an will not require an elemen-
patient shoul also have a CT scan or MRI of the abomen tal formula (D).
an CT of the chest to evaluate for extrarenal or metastatic References: Carcassonne M, Guys JM, Morrison-Lacombe G,
Kreitmann B. Management of Hirschsprung’s isease: curative sur-
isease. Surgical management inclues nephroureterectomy
gery before 3 months of age. J Pediatr Surg. 1989;4(10):103–1034.
with ipsilateral lymph noe sampling (D–E). Lymph noe Langer JC. Chapter 101 - Hirschsprung Disease. In: Coran AG, e.
sampling is a critical part of the evaluation an staging oper- Pediatric Surgery. 7th e. Mosby; 01:165–178. ISBN 97803307557,
ation for peiatric renal tumors. Noal status etermines https://oi.org/10.1016/B978-0-33-0755-7.00101-X.
the overall stage an risk stratication group an therefore
the chemotherapeutic regimen. Unfortunately, in a stuy
10. C. Neonates normally have a protuberant abomen,
evaluating surgical protocol violations over 10 years, lack
so the presence of a scaphoi abomen, combine with
of lymph noe sampling (C) accounte for 67% of all viola-
respiratory istress at birth, shoul raise the suspicion of
tions. Absence of aequate lymph noe sampling manates
congenital iaphragmatic hernia (CDH). Overall survival is
treatment as Stage III isease an exposes the patient to
about 60% to 80%. The abomen is scaphoi because the
potentially unnecessary abominal raiation an aitional
majority of the abominal contents are herniate into the
chemotherapeutic agents. Similarly, performing a biopsy at
chest. In infants with CDH, both lungs are hypoplastic (the
the time of surgery (B) may upstage the isease to stage 3.
ipsilateral lung is worse than the contralateral lung) an
There is no role for biopsy of the contralateral kiney in a
there is ecrease bronchial an pulmonary artery branch-
nonsynromic patient with a unilateral Wilms tumor, an in
ing. Infants are prone to the evelopment of pulmonary
this particular scenario, a proper staging workup shoul be
hypertension. Pulmonary vasculature is istinctly abnor-
complete prior to any surgical management.
mal in that the meial muscular thickness of the arterioles
References: Ehrlich PF, Gow K, Hamilton TE, et al. Surgical pro-
is excessive an extremely sensitive to the multiple local
tocol violations in chilren with renal tumors provies an opportu-
nity to improve peiatric cancer care: a report from the Chilren’s an systemic factors known to trigger vasospasm. Between
Oncology Group. Pediatr Blood Cancer. 016;63(11):1905–1910. 80% an 90% occur on the left sie, an the efect is pos-
Kieran K, Ehrlich PF. Current surgical stanars of care in Wilms terolateral (Bochalek hernia), as oppose to the Morgagni
tumor. Urol Oncol. 016;34(1):13–3. hernia, which is an anteromeial efect. Initial manage-
ment shoul be focuse on respiratory support. Ventilation
9. C. Hirschsprung isease is characterize by an absence with high-frequency oscillation is effective, as is the use of
of ganglion cells in the Auerbach plexus an hypertrophy inhale nitric oxie (D). Refractory cases shoul be place
of associate nerve trunks. The cause is thought to be a on extracorporeal membrane oxygenation (ECMO). Place-
efect in the migration of neural crest cells. The rectosig- ment of a nasogastric tube is also important to prevent
moi junction is affecte in 75% of cases, the splenic exure gastric istention, which may slightly worsen the lung com-
or transverse colon in 17%, an the entire colon with vari- pression, meiastinal shift, an ability to ventilate. Because
able extension into the small bowel in 8%. The presentation of the lung hypoplasia, prompt reuction of the bowel con-
of the isease is characterize as a functional istal intes- tents oes not immeiately improve ventilatory function
tinal obstruction. Similar to ulcerative colitis, the isease (E). Once the patient is stabilize, they shoul be taken to
is always present istally an extens a variable istance the operating room to reuce the bowel, repair the efect
proximally an continuously. In the neonatal perio, the with or without mesh, an run the entire bowel to look for
most common symptoms are abominal istention, failure associate anomalies such as malrotation. Chest tubes are
to pass meconium, an bilious emesis. Infants can also pres- not inicate because these may injure the unerlying lung
ent with enterocolitis, which is characterize by abominal an worsen the prognosis (A). Bag-mask ventilation will
CHAPtEr 22 Pediatric Surgery 315

isten the stomach an GI tract leaing to further lung is biliary atresia, which is an obliterative process of the extra-
compression an worsen the patient’s conition (B). hepatic bile ucts an is associate with hepatic brosis. The
Reference: Stolar CJH, Dillon PW. Chapter 63 - Congenital Dia- infant prouces acholic stools an emonstrates a failure to
phragmatic Hernia an Eventration. In: Coran AG, e. Pediatric Sur- thrive. Left untreate, it will progress to liver failure an por-
gery. 7th e. Mosby; 01:809–84. ISBN 97803307557, https://oi. tal hypertension. Nuclear scanning after pretreatment with
org/10.1016/ B978-0-33-0755-7.00063-5. phenobarbital is a useful stuy. One is specically looking
to see whether the raionuclie appears in the intestine,
11. C. In patients with an imperforate anus, the rectum which woul conrm that the extrahepatic bile ucts are
fails to escen through the external sphincter complex. The patent. This ning exclues biliary atresia. If the raionu-
pathophysiology is thought to be ue to the failure of the clie is normally concentrate in the liver but not excrete
urorectal septum to escen. The rectal pouch ens blinly an the metabolic screen results are normal, this is highly
in the pelvis, above (high lesion) or below (low lesion) the suggestive of biliary atresia. The presence of a gallblaer
levator complex. Sixty percent of males have high lesions oes not exclue the iagnosis of biliary atresia. The iag-
compare with only 30% of females. In most cases, the blin nosis can be conrme with a biopsy emonstrating bile
rectal pouch communicates more istally with the genitouri- plugging an periportal brosis. The most effective initial
nary system or with the perineum through a stulous tract. treatment of biliary atresia is portoenterostomy, as escribe
In male patients with a high imperforate anus, the rectum by Kasai. The proceure involves anastomosing an isolate
usually ens up as a stula somewhere along the urethra. In limb of the jejunum to the transecte ucts at the portal plate
females, a high imperforate anus often occurs in the context of the liver. The likelihoo of surgical success is increase
of a persistent cloaca. Approximately 60% of patients have an if the proceure is performe before the infant reaches the
associate malformation; the most common is a urinary tract age of  months. If the patient remains symptomatic after the
efect. Skeletal efects are also seen, an the sacrum is most Kasai operation, he or she will require liver transplantation
commonly involve. Spinal cor anomalies are common, (B). Inepenent risk factors that preict failure of the pro-
especially with high lesions. Imperforate anus is also associ- ceure inclue briging liver brosis at the time of surgery
ate with VACTERL (vertebral efects, anal atresia, cariac an postoperative cholangitis episoes. Percutaneous rain-
efects, tracheoesophageal stula, renal anomalies, an limb age oes not offer long-term ecompression an oes not
abnormalities) synrome. Evaluation shoul inclue plain aress the lack of enteric bile (C). Options D an E are not
raiographs of the spine as well as an ultrasoun scan of possible because of a lack of extrahepatic biliary tree in this
the spinal cor. A plain chest raiograph an a careful clin- isease.
ical evaluation of the heart shoul be conucte. The most References: Cowles RA. Chapter 105 - The Jaunice Infant:
common efect is an imperforate anus with a stula between Biliary Atresia. In: Coran AG, e. Pediatric Surgery. 7th e. Mosby;
the istal colon an the urethra in boys or to the vestibule 01:131–1330. ISBN 97803307557, https://oi.org/10.1016/
of the vagina in girls. When there is no visible meconium in B978-0-33-0755-7.00105-7.
the perineal area after 4 hours, the patient is consiere to Ohhama Y, Shinkai M, Fujita S, Nishi T, Yamamoto H. Early pre-
have a high imperforate anus malformation. Patients with a iction of long-term survival an the timing of liver transplantation
high lesion shoul unergo primary sigmoi colostomy fol- after the Kasai operation. J Pediatr Surg. 000;35(7):1031–1034.
lowe by a enitive pull-through at 3 to 6 months of life
(B). Waiting an aitional 4 hours may lea to worsening
13. A. This patient has jejunal or ileal atresia. Intestinal
abominal istention an respiratory compromise (A). Low
atresias are cause by in utero mesenteric vascular acci-
lesions can be repaire by a perineal proceure at birth (D).
ents leaing to segmental loss of the intestinal lumen. Due
High lesions may be repaire through a posterior sagittal
to small bowel atresia, the colon has been unuse in utero
approach (PSARP) or a laparoscopic-assiste approach (E).
an is therefore of small iameter. They are classie into
Low lesions have a better prognosis with respect to conti-
four types base on their severity. Infants with jejunal or
nence as the anatomy more closely resembles complete
ileal atresia present soon after birth with bilious vomiting
escent an evelopment.
an progressive abominal istention. More istal obstruc-
References: Georgeson KE, Inge TH, Albanese CT. Laparoscop-
tions prouce more istension on physical exam an raio-
ically assiste anorectal pull-through for high imperforate anus–a
new technique. J Pediatr Surg. 000;35(6):97–931. graphs. In cases in which the iagnosis of complete intestinal
Levitt MA, Peña A. Chapter 103 - Anorectal Malforma- obstruction is ascertaine by the clinical picture an the
tions. In: Coran AG, e. Pediatric Surgery. 7th e. Mosby; presence of staggere air–ui levels on plain abominal
01:189–1309. ISBN 97803307557, https://oi.org/10.1016/ lms, the chil can be brought to the operating room after
B978-0-33-0755-7.00103-3. appropriate resuscitation. In these circumstances, there is lit-
tle extra information that can be gaine by a barium enema.
12. A. Jaunice within the rst 4 hours of life or jaunice When the iagnosis is uncertain, a contrast enema may be
that persists beyon  weeks after birth is generally consi- use. The initial treatment of jejunal atresia is nasogastric
ere pathologic. Pathologic jaunice may be cause by biliary tube ecompression an ui resuscitation. Denitive treat-
obstruction, increase hemoglobin loa, or liver ysfunction. ment involves surgical resection of the atretic loop an pri-
One must rule out obstructive isorers, incluing biliary mary reanastomosis. Failure of recanalization of the bowel is
atresia, choleochal cyst, an inspissate bile synrome; associate with esophageal an uoenal atresias (B). Lack
ABO incompatibility; Rh incompatibility; spherocytosis; of proper 70-egree counterclockwise rotation of the bowel
metabolic isorers; α1-antitrypsin eciency; galactosemia; is a feature of malrotation (C). Lack of ganglion cells in the
an congenital infection incluing syphilis an rubella. The bowel is seen with Hirschsprung isease, whereas a uplica-
most common cause of neonatal jaunice requiring surgery tion woul lea to uplication cysts (D, E).
316 PArt i Patient Care

Reference: Frischer JS, Azizkhan RG. Chapter 8 - Jejunoil- Hirschl RB, Bartlett RH. Chapter 8 - Extracorporeal Life Support
eal Atresia an Stenosis. In: Coran AG, e. Pediatric Surgery. 7th for Cariopulmonary Failure. In: Coran AG, e. Pediatric Surgery.
e. Mosby; 01:1059–1071. ISBN 97803307557, https://oi. 7th e. Mosby; 01:13–13. ISBN 97803307557, https://oi.
org/10.1016/B978-0-33-0755-7.0008-9. org/10.1016/B978-0-33-0755-7.00008-8.

14. E. Omphalocele refers to a congenital efect of the 16. D. Approximately 90% of CDHs occur on the left sie
abominal wall at the miline in which the bowel an soli (C). Rarely, they may be bilateral. The cause of CDH is
viscera are covere by peritoneum an the amniotic mem- unknown, but it is believe that they result from the fail-
brane. The abominal wall efect can measure 4 cm or more ure of normal closure of the pleuroperitoneal canal in the
in iameter an is cause by a lack of complete evelopment eveloping embryo. As a result, the abominal contents
of the abominal wall muscles (C). An omphalocele is less of herniate through the efect in the iaphragm an compress
a surgical emergency than a gastroschisis because the bowel both lungs, with the ipsilateral lung being more severely
is protecte by the covering (A). Conversely, omphalocele affecte. Compression of the eveloping lungs leas to
is associate with many other congenital abnormalities that pulmonary hypoplasia, which is clinically manifeste with
are not seen with gastroschisis. The most common anomalies hypercarbia (B). There is a higher incience of malrotation
associate with omphalocele are cariac an musculoskel- in patients with CDH. A Bochalek hernia is in the postero-
etal. The size of the efect may be small or so large that it lateral location an most commonly on the left sie (E). The
contains most of the abominal viscera. There is an increase most signicant physiologic abnormality in patients with
occurrence of cariac an chromosomal abnormalities. CDH is pulmonary hypertension, which can lea to sig-
Omphalocele is associate with premature an intrauterine nicant hypoxia. Extracorporeal membrane oxygenation
growth retaration, while gastroschisis is associate with (ECMO) may be require in some patients with signicant
intrauterine rupture of the umbilical vein. Immeiate treat- pulmonary hypertension. For this reason, urgent surgical
ment of an infant with omphalocele consists of maintaining intervention is not inicate because reucing the hernia
normal vital signs an boy temperature. The omphalocele will not correct the pulmonary hypertension. In fact, surgical
shoul be covere with saline-soake gauze, an the trunk repair may temporarily worsen pulmonary compliance an
shoul be wrappe circumferentially. No attempt shoul be hypertension. Thus, the infant’s conition shoul be mei-
mae to manually reuce the abominal contents because cally optimize before performing the repair (A). Although
this maneuver may increase the risk of sac rupture or inter- there is no ieal time to repair a CDH, most surgeons will
fere with abominal venous return. Gastroschisis, on the wait until the infant’s pulmonary vascular resistance rops,
other han, may be associate with intestinal atresia (10%– which occurs several ays to weeks after birth. Bochalek
15%) (D). Mortality for omphalocele is largely base on the hernias are istinguishe from Morgagni hernias, which are
unerlying comorbiities an is usually not ue to sepsis another type of congenital hernia an typically of the antero-
(B). Aitionally, omphalocele has a higher mortality rate meial iaphragm. The Morgagni hernia efect is small an
than gastroschisis ue to associate congenital anomalies. asymptomatic an typically presents as a ensity on a chest
References: Wagner JP, Lee SL. Infant born with abominal wall raiograph in aulthoo.
efect. In: e Virgilio C, Frank PN, Grigorian A, es. Surgery: a case Reference: Lally KP, Paranka MS, Roen J, et al. Congenital
based clinical review. 1st e. Springer; 015:349–357. iaphragmatic hernia. Stabilization an repair on ECMO. Ann Surg.
Benjamin B, Wilson GN. Anomalies associate with gastroschisis 199;16(5):569–573.
an omphalocele: analysis of 85 cases from the Texas Birth Defects
Registry. J Pediatr Surg. 014;49(4):514–519. 17. B. Esophageal atresia (EA) an tracheoesophageal
stula (TEF) are congenital interruptions or iscontinu-
15. C. In neonates with respiratory istress synrome, ities of the esophagus, resulting in esophageal obstruction.
management inclues high-frequency ventilation, surfac- Most present at birth with excessive rooling an choking
tant, an inhale nitric oxie. When those interventions fail, or coughing after an attempte fee. There are ve types
ECMO is use. ECMO can be performe by either venove- (A–E). The most common type is type C, in which there is
nous or venoarterial cannulation. The major inications for proximal EA with a istal TEF. The most appropriate next
ECMO inclue meconium aspiration, respiratory istress step is to attempt to place an orogastric tube. In patients with
synrome (B), persistent pulmonary hypertension (D), sep- proximal EA an istal TEF, the tube will not be able to be
sis, an CDH (A). Meconium aspiration is the most common passe into the stomach but will curl in the upper esophageal
inication for neonatal ECMO. ECMO has also been use to pouch. A two-view chest x-ray shoul follow to conrm the
temporize infants with ecompensation ue to a congenital iagnosis (D). An esophagram or UGI series is not neee to
cariac abnormality (E). The most reae complication of make the iagnosis an increases the risk of aspiration (E).
ECMO is intracranial hemorrhage seconary to the heparin An abominal x-ray is obtaine after attempte placement of
require to prevent circuit clotting. Aitionally, premature the orogastric tube (C). The abominal x-ray will help eter-
neonates have an unerevelope cerebral microvasculature mine the presence of a TEF by showing gas in the intestines.
an an intolerance of physiologic insults, further increasing A gasless abomen suggests an isolate EA. Intubation an
the risk of intracranial bleeing. positive pressure ventilation shoul be avoie because they
References: Hall J, Hariman K, Lee S, et al. The American increase the risk of ventilating through the TEF, resulting in
society of colon an rectal surgeons clinical practice guielines for respiratory failure (A).
the treatment of left-sie colonic iverticulitis. Dis Colon Rectum. Reference: Rothenberg S. Esophageal atresia an tracheoesoph-
00;63(6):78–747. ageal stula malformations. In: Holcomb GW III, Murphy JP, Ostlie
Hines MH. ECMO an congenital heart isease. Semin Perinatol. DJ, St. Peter SD, es. Ashcraft’s pediatric surgery. 6th e. Sauners;
005;9(1):34–39. 014:365–384.
CHAPtEr 22 Pediatric Surgery 317

18. C. Pyloric stenosis occurs in 1 in 300 live births. Most a UGI series (D). CT scan of the abomen/pelvis may sug-
often, it occurs in males between 3 an 6 weeks of age. gest malrotation in oler chilren or aults with vague
Infants with pyloric stenosis present with projectile, nonbil- symptoms (E).
ious vomiting. As the isease progresses, an almost complete Reference: Sullins VF, Lee SL. Infant with bilious emesis. In: e
gastric outlet obstruction evelops, an the infant is no lon- Virgilio C, Frank PN, Grigorian A, es. Surgery: a case based clinical
ger able to tolerate even clear liquis. The classic electrolyte review. 1st e. Springer; 015:335–343.
isorer that results from protracte vomiting is a hypo-
chloremic, hypokalemic metabolic alkalosis (A, B, D). The 21. C. After malrotation an migut volvulus is iagnose,
urine pH level is high initially because of the alkalosis but the infant shoul be urgently taken to the operating room
eventually becomes aciic an is known as paraoxic aci- because a elay risks the evelopment of gangrene of the
uria. The explanation for this is that the renal tubule initially entire small bowel. The rst step is to reuce the volvulus.
reabsorbs soium in exchange for potassium. However, gas- The goal of the La proceure is to broaen the narrow
tric juice has a high potassium concentration, an as vom- base of the mesentery to prevent the volvulus from recur-
iting continues, serum potassium levels rop. To conserve ring. The bans between the cecum an abominal wall
potassium as well, the renal tubule switches to reabsorbing an between the uoenum an terminal ileum are sharply
soium in exchange for hyrogen ions in the urine (E). ivie to splay out the superior mesenteric artery an its
References: Fujimoto T, Lane GJ, Segawa O, Esaki S, Miyano T. branches. This brings the uoenum into the right abomen
Laparoscopic extramucosal pyloromyotomy versus open pyloromy- an the cecum into the left lower quarant an anatomically
otomy for infantile hypertrophic pyloric stenosis: which is better? creates a complete nonrotation (A, D, E). The appenix is
JPediatr Surg. 1999;34():370–37. typically remove to avoi iagnostic errors later in life, but
Jabaji Z, Sullins VF, Lee SL. Infant with nonbilious emesis. In: e this is not absolutely require because imaging techniques
Virgilio C, Frank PN, Grigorian A, es. Surgery: a case based clinical an iagnostic capabilities have improve. The cecum an
review. 1st e. Springer; 015:343–349. stomach are not xe to the abominal wall because this will
increase the risk of a twist at these sites (B).
19. B. In all infants suspecte of having necrotizing entero- References: Pracros JP, Sann L, Genin G, et al. Ultrasoun
colitis (NEC), feeings are iscontinue, an orogastric tube iagnosis of migut volvulus: the “whirlpool” sign. Pediatr Radiol.
is place for ecompression, an broa-spectrum parenteral 199;(1):18–0.
antibiotics are given. Staging of NEC can be one with the Sullins VF, Lee SL. Infant with bilious emesis. In: e Virgilio C,
Bell criteria. Patients with Bell stage 1 (suspicious for NEC) Frank PN, Grigorian A, es. Surgery: a case based clinical review. 1st e.
Springer; 015:335–343.
are rule out for NEC an kept NPO an on IV antibiotics
for 3 to 7 ays before enteral nutrition is reinitiate. Patients
22. E. The history an raiograph nings are consistent
with Bell stage  (enite NEC) require close observation for
with uoenal atresia. Duoenal atresia occurs because of
7 to 14 ays. Infants with Bell stage 3 (avance NEC) either
failure of recanalization of the uoenum from its soli core
have enite intestinal perforation or have not respone
state. It is associate with prematurity, Down synrome,
to nonoperative therapy an thus require surgery. These
maternal polyhyramnios, malrotation, annular pancreas,
patients have signs of peritonitis, aciosis, sepsis, an is-
an biliary atresia. In most cases, the uoenal obstruction is
seminate intravascular coagulation, all of which are asso-
istal to the ampulla of Vater, an infants present with bilious
ciate with a high mortality rate. This patient is Bell stage 
emesis in the neonatal perio. The classic raiographic n-
an shoul continue treatment with antibiotics an bowel
ing is the “ouble-bubble sign” (an air-lle stomach, a func-
rest/ecompression. Bloo transfusions shoul be base on
tioning pylorus, an a istene proximal uoenal bulb). If
the patient’s clinical status an hemoglobin/hematocrit. A
there is no istal bowel gas, complete atresia is conrme an
platelet count of 100,000/mcL oes not require transfusion
no further stuies are necessary (A–C). Conversely, if istal
(A). Options C, D, an E are reserve for patients with Bell
air is present, a UGI contrast stuy shoul be one to rule
stage 3. Ultrasoun an paracentesis may guie the ecision
out malrotation an migut volvulus. The ning of istal
to procee with operative intervention. Surgical intervention
air in association with a ouble bubble coul also inicate a
may be with a peritoneal rain or exploratory laparotomy.
uoenal stenosis or web or an annular pancreas that oes
References: Bell MJ, Ternberg JL, Feigin RD, et al. Neonatal
not cause a complete obstruction. Patients may also have
necrotizing enterocolitis. Therapeutic ecisions base upon clinical
staging. Ann Surg. 1978;187(1):1–7. associate cariac malformations for which an echocario-
Dominguez KM, Moss RL. Necrotizing enterocolitis. In: Hol- gram is neee before surgical intervention (D). The treat-
comb GW III, Murphy JP, Ostlie DJ, St. Peter SD, es. Ashcraft’s pedi- ment of uoenal atresia is surgical bypass of the obstruction
atric surgery. 6th e. Sauners; 014:454–473. as either a sie-to-sie or proximal transverse-to-istal lon-
gituinal uoenouoenostomy or a uoenojejunostomy.
20. C. The iagnosis of malrotation with migut volvu- When the proximal uoenum is markely ilate, a taper-
lus shoul be suspecte in an infant presenting with bil- ing uoenoplasty may be performe.
ious vomiting an evience of a bowel obstruction. Plain Reference: Sullins VF, Lee SL. Infant with bilious emesis. In: e
raiographs are likely to be normal or noniagnostic Virgilio C, Frank PN, Grigorian A, es. Surgery: a case based clinical
(A). Some authors have recommene ultrasonography review. 1st e. Springer; 015:335–343.
to look for a sonographic clockwise whirlpool pattern of
the superior mesenteric vein an mesentery aroun the 23. B. Neuroblastoma is the most common soli abominal
superior mesenteric artery; however, the gol stanar for malignancy in chilren < years of age. Wilms tumor is the
iagnosis is a UGI series (B). Historically, contrast enemas most common after  years of age. The presenting symptoms
were use to make the iagnosis but are less accurate than of neuroblastoma epen on the site of the primary tumor,
318 PArt i Patient Care

the presence of metastatic isease, the age of the patient, an an primary anastomosis (B, C, E), provie there is no evi-
the metabolic activity of the tumor. The most common presen- ence of giant cystic meconium peritonitis.
tation is a xe lobular mass extening from the ank towar References: Rescorla FJ, Grosfel JL. Contemporary manage-
the miline. The tumor can also exten into the neural foram- ment of meconium ileus. World J Surg. 1993;17(3):318–35.
ina an cause symptoms of spinal cor compression. It tens Ziegler MM. Chapter 83 - Meconium Ileus. In: Coran AG, e. Pedi-
to metastasize to cortical bones, bone marrow, an the liver, atric Surgery. 7th e. Mosby; 01:1073–1083. ISBN 97803307557,
https://oi.org/10.1016/ B978-0-33-0755-7.00083-0.
an patients may present with localize swelling an tener-
ness, limp, or refusal to walk. Periorbital metastases account
for proptosis an ecchymosis, resulting in “raccoon eyes.” In 26. D. This patient presents with an incarcerate right
infants, liver metastases may expan, causing hepatomegaly. inguinal hernia (RIH). The bilious emesis an abominal
Metastatic lesions to the skin prouce the blueberry mufn istention are highly suggestive of a small bowel obstruc-
appearance. Wilms tumor (C) also presents as an abominal tion. Thus, the most appropriate management is to attempt a
mass, in association with Beckwith-Wieemann synrome reuction of the incarcerate RIH. If reuction of the hernia
(macroglossia, hypoglycemia, gigantism, an visceromegaly), is successful, the patient shoul be amitte for observa-
an as part of the WAGR complex (Wilms tumor, aniriia, tion an repair within 4 to 48 hours. The eema from the
genitourinary abnormalities, an mental retaration). Rhab- incarcerate RIH makes immeiate surgical repair more if-
omyosarcoma is a soft-tissue tumor (A). The most common cult. Repairing the hernia after 4 to 48 hours will allow
primary sites are the hea an neck. Sacrococcygeal teratoma the eema to resolve. Incarcerate hernias shoul be iag-
is the most common type of teratoma (E). It presents as a large nose base on history an physical exam without the nee
mass extening off the sacrum in the newborn perio. Hepa- for ultrasoun (A). Ultrasoun may be useful if testicular
toblastoma, although the most common liver malignancy in torsion is suspecte. Erythematous masses may be misi-
chilren, is a rare soli organ malignancy (D). agnose as abscesses. However, in this case, the history an
Reference: Davioff AM. Neuroblastoma. In: Holcomb GW III, physical exam are consistent with an incarcerate inguinal
Murphy JP, Ostlie DJ, St. Peter SD, es. Ashcraft’s pediatric surgery. 6th hernia. IV antibiotics an possible incision an rainage is
e. Sauners; 014:883–905. the treatment for abscesses (B, C). Operative exploration
is performe if the incarcerate hernia cannot be reuce
24. D. Gastroschisis, unlike omphalocele, is not typically (E). When an incarcerate hernia cannot be reuce, there
associate with systemic or chromosomal abnormalities shoul be a heightene suspicion for the presence of isch-
(A–C, E). There is an abominal wall efect to the right of the emic bowel.
umbilicus an the bowel herniates through without a perito- Reference: Fraser JD, Snyer CL. Inguinal hernias an hyro-
neal covering. Because the bowel is eviscerate an expose, celes. In: Holcomb GW III, Murphy JP, Ostlie DJ, St. Peter SD, es.
this conition is a surgical emergency. The bowel can be Ashcraft’s pediatric surgery. 6th e. Sauners; 014:689–701.
thickene an covere with an exuate. All patients with
gastroschisis will have intestinal malrotation. However, mi- 27. C. Choleochal cysts have been classie into ve
gut volvulus is unlikely ue to the ahesions create from types. The most common is type I, which is a fusiform ila-
the gastroschisis. Intestinal atresia is also seen in 10% to 15% tation of the bile uct. Type II is a iverticulum of the CBD.
of patients with gastroschisis. If the efect cannot be primar- Type III is a choleochocele. Type IV is multiple cysts. Type
ily close, a stage-closure utilizing a silo may be require. V is known as Caroli isease, which involves cysts limite
Reference: Wagner JP, Lee SL. Infant born with abominal wall to the intrahepatic bile ucts. The cysts lea to recurrent
efect. In: e Virgilio C, Frank PN, Grigorian A, es. Surgery: a case bouts of cholangitis an have a risk of malignancy. The treat-
based clinical review. 1st e. Springer; 015:349–357. ment of a type I choleochal cyst is resection of the cyst an
reconstruction with a Roux-en-Y choleochojejunostomy
25. A. Meconium ileus is a result of cystic brosis, in which or a simple choleochouoenostomy. If the cyst is aher-
the meconium becomes thick an viscous ue to ecits in ent to the portal vein, the anterior portion of the cyst shoul
pancreatic enzymes. It creates a small bowel obstruction, an be excise along with mucosectomy of the posterior cyst to
as such, the infant may present with bilious vomiting. In the prevent future malignant egeneration. Antibiotics an/or
most severe forms, it can lea to intestinal perforation. The rainage will not result in a more favorable operation (A, B).
raiograph typically emonstrates a “groun-glass” appear- Cholangitis shoul be treate with antibiotics before eni-
ance, which represents small pockets of gas trappe insie tive surgery. Internal rainage alone will still preispose the
the thickene meconium. The treatment strategy epens on patient to a future risk of malignant egeneration (D). The
whether the patient has complicate or uncomplicate meco- portal vein shoul not be resecte uring this operation (E).
nium ileus. Patients with uncomplicate meconium ileus Reference: Liem NT, Holcomb GW III. Choleochal cyst
can be treate nonoperatively. Aministering a water-solu- an gallblaer isease. In: Holcomb GW III, Murphy JP, Ostlie
ble enema such as ilute gastrogran per rectum allows the DJ, St. Peter SD, es. Ashcraft’s pediatric surgery. 6th e. Sauners;
meconium to soften as it takes on more water. Optimally, the 014:593–606.
contrast shoul reach the ilate portion of the ileum uner
uoroscopic visualization. The enema may be repeate every 28. D. Inguinal hernias result from the processus vagina-
1 hours over several ays as neee. A UGI with SBFT is lis failing to close. Inguinal hernias occur more commonly
not inicate or use in the initial management of meconium in males an premature infants an are more common on
ileus (D). Surgery is require if nonoperative management the right sie. The iagnosis of an inguinal hernia shoul
fails or if the patient alreay has evience of perforation. be base on history an physical examination without the
Complicate cases are usually amenable to bowel resection nee for ultrasoun (A). The timing of herniorrhaphy in
CHAPtEr 22 Pediatric Surgery 319

premature infants is ebatable an base on the clinical sce- the umbilical structures/vessels. After the efect is opene
nario. If the hernia is easily reucible, many surgeons will up, a silo is typically inicate (A). Primary reuction an
repair the hernia at the time of ischarge (rather than imme- closure shoul only be attempte when there is no risk to the
iately). By repairing the hernia before ischarge, the risk of bowel (B). Resection of ischemic bowel shoul be reserve
re-presenting to the ED with an incarcerate inguinal hernia for grossly necrotic bowel because patients with gastroschi-
is eliminate. Some surgeons woul ischarge patients an sis are at risk of eveloping short gut synrome (D). Any
repair the hernia when the postconceptional age (the gesta- questionable bowel shoul be observe with serial exams. A
tional age + age of patient) is aroun 55 weeks (in this infant iverting ileostomy is not inicate in this patient (E).
that woul be 30 weeks after birth). However, waiting until Reference: Wagner JP, Lee SL. Infant born with abominal wall
the infant reaches 1 year of age woul increase the risk of efect. In: e Virgilio C, Frank PN, Grigorian A, es. Surgery: a case
incarceration (E). By waiting, there is a lower anesthetic risk, based clinical review. 1st e. Springer; 015:349–357.
an the operation is not as challenging. Thus, the ecision to
repair shoul be consiere at the time of ischarge. Imme- 31. B. This patient has a iagnosis of Hirschsprung isease
iate repair in premature infants is technically more ifcult (HD). The patient then evelope Hirschsprung-associate
an associate with a higher rate of recurrence an postop- enterocolitis (HAEC). The initial management for HAEC is
erative apnea (B, C). Inguinal hernias in chilren only require IV antibiotics, bowel rest, an rectal irrigations. However,
high ligation of the sac. Mesh is rarely ever require in pei- if the patient eteriorates, then urgent colostomy is neee
atric patients with inguinal hernias. to ecompress the colon an may be lifesaving. A contrast
Reference: Fraser JD, Snyer CL. Inguinal hernias an hyro- enema is contrainicate in patients with active HAEC (A).
celes. In: Holcomb GW III, Murphy JP, Ostlie DJ, St. Peter SD, es. Because the level of HD is not known, a colectomy shoul not
Ashcraft’s pediatric surgery. 6th e. Sauners; 014:689–701. be performe. In aition, patients are often too sick to with-
stan a prolonge operation (C). This patient oes not have
29. C. The pathogenesis of NEC is thought to be intestinal abominal compartment synrome or fungal sepsis (D, E).
hypoperfusion (A, B, D, E). This occurs most frequently in Reference: Langer JC. Meckel iverticulum. In: Holcomb GW
the setting of perinatal stress. The perio of hypoperfusion is III, Murphy JP, Ostlie DJ, St. Peter SD, es. Ashcraft’s pediatric surgery.
followe by a perio of reperfusion, an the combination of 6th e. Sauners; 014:474–491.
ischemia an reperfusion leas to mucosal injury. The am-
age intestinal mucosa barrier becomes susceptible to bacte- 32. E. The infant is exhibiting signs of malrotation with mi-
rial translocation, which initiates an inammatory cascae. gut volvulus. By the time abominal wall eema is evient,
Various proinammatory meiators are release, which in there is a high likelihoo of intestinal gangrene. As such, no
turn lea to further epithelial injury an the systemic man- further stuies are inicate, an the infant requires urgent
ifestations of NEC. It is postulate that maintenance of the laparotomy (A, B). Conrmation with an upper GI series is
gut barrier is essential for the protection of the host against only inicate when the patient is stable an the iagnosis
NEC. It has always been taught that NEC classically pres- is unclear. Enoscopy has no role in iagnosis or treatment
ents with blooy stools after the rst feeing. However, the (C). Resection of extensive ea bowel may result in short-
earliest signs are nonspecic, incluing apnea, braycaria, gut synrome an necessitate intestinal transplantation to
lethargy, an temperature instability. The most common GI avoi long-term parenteral nutrition. La bans exten
symptoms are feeing intolerance an high gastric resiu- from the cecum to the lateral abominal wall (D), crossing
als, while the most common sign is abominal istention. the uoenum, which increases the potential for obstruc-
Grossly blooy stools are infrequently seen. Management is tion. Aitional clues to the presence of avance ischemia
initially conservative with NPO, ui resuscitation, broa- inclue erythema of the abominal wall. Sometimes, gan-
spectrum IV antibiotics, TPN, an ecompression with an grenous loops of bowel may be seen transabominally as a
orogastric tube. Surgical intervention is inicate for failure iscolore mass. If left untreate, the infant will progress to
of conservative management, free air on plain lms or CT, shock an eath. It must be reemphasize that the inex of
an peritonitis. suspicion for this conition must be high because abominal
Reference: Dominguez KM, Moss RL. Necrotizing enterocolitis. signs are minimal in the early states. Abominal lms show
In: Holcomb GW III, Murphy JP, Ostlie DJ, St. Peter SD, es. Ash- a paucity of gas through the intestine with a few scattere
craft’s pediatric surgery. 6th e. Sauners; 014:454–473. air–ui levels. In early cases, the patient oes not appear
ill initially, an the plain lms may suggest partial uoenal
30. C. Management of a newborn with gastroschisis obstruction. Uner these conitions, the patient may have
involves stabilizing the airway, preventing hypothermia, malrotation without volvulus. This is best iagnose by
orogastric ecompression, establishing IV access, an an upper GI series that shows incomplete rotation with the
aministering IV uis an antibiotics. The bowel shoul uoenojejunal junction isplace to the right. When volvu-
also be place in a sterile, clear plastic wrap to prevent fur- lus is suspecte, early surgical intervention is manatory if
ther volume an heat loss. The bowel must also be carefully the ischemic process is to be avoie or reverse. Volvulus
inspecte for signs of ischemia. If the bowel appears isch- occurs clockwise an shoul be untwiste counterclockwise.
emic, the bowel must be inspecte to rule out a simple twist Reference: Sullins VF, Lee SL. Infant with bilious emesis. In: e
or kink in the mesentery. The efect is also examine to be Virgilio C, Frank PN, Grigorian A, es. Surgery: a case based clinical
sure that it is not tight an the cause of the ischemia, as in review. 1st e. Springer; 015:335–343.
this case. If the efect is too tight, it must be opene imme-
iately. The best irection to open the efect woul be to the 33. A. Chilren born with bilateral unescene tes-
patient’s right (away from the umbilicus) in orer to avoi tes have a much higher rate of subsequent infertility. It is

AL GRAWANY
320 PArt i Patient Care

associate with prune belly synrome (a lack of abominal aition to the testicular arteries, the testicles receive collat-
wall muscles) (C). When the testicle is not in the scrotum, it eral bloo from the cremasteric artery, a branch of the infe-
is subjecte to higher temperatures, resulting in ecrease rior epigastric artery, an the artery to the vas (a branch of
spermatogenesis. When the testicles are place in the scro- the superior vesical artery). Thus, ivision of the testicular
tum, fertility is improve but still lower than in those with- artery is usually well tolerate an usually oes not result
out cryptorchiism (B). It is recommene that unescene in testicular necrosis (E). The orchiopexy is then performe
testicles be repositione by 1 year of age to maximize the through the groin approximately 6 months later, after which
chances of improving fertility. The use of chorionic gonao- time collateral ow has increase.
tropin sometimes is effective in achieving escent in patients References: Lee JJ, Shortliffe LMD. Unescene testes an tes-
with bilateral unescene testes, suggesting that they may ticular tumors. In: Holcomb GW III, Murphy JP, Ostlie DJ, St. Peter
have a hormonal eciency (D). If the intraabominal testes SD, es. Ashcraft’s pediatric surgery. 6th e. Sauners; 014:689–701.
cannot be effectively mobilize to reach own into the scro- Chan E, Wayne C, Nasr A, FRCSC for Canaian Association of
Peiatric Surgeon Evience-Base Resource. Ieal timing of orchio-
tum, a -stage Fowler-Stephens proceure is use. In the rst
pexy: a systematic review. Pediatr Surg Int. 014;30(1):87–97.
stage, the testicular vessels are clippe laparoscopically. In
Plastic Surgery
AMANDA C. PURDY AND MYTIEN GOLDBERG 23
ABSITE 99th Percentile High-Yields
I. Skin–Grafts
A. Split-thickness versus full-thickness skin grafts:

Split-thickness skin graft (STSG) Full-thickness skin


graft (FTSG)
All of epidermis & all of
Contains… All of epidermis & part of dermis dermis
Donor site Common sites Thigh, buĴocks, back, abdomen Behind ear, groin, neck
Harvesting Dermatome (0.013–0.018 in thick) + /− Freehand with a scalpel
meshing
Healing Mechanism Epithelialization from dermal hair follicles Needs to be primarily closed
& wound border
Used for… Larger wounds in noncosmetically Small wounds on face or
sensitive areas hands, joints, cosmetically
sensitive areas
Primary contraction Immediate shrinking Less; fewer elastic ębers and dermal MORE; more dermal elastin
after harvesting but before grafting extracellular matrix components
Secondary contraction The degree of MORE Less
shrinking during healing
Chance of graft survival BeĴer Contain less dermis & therefore Worse requires a healthier
have a lower metabolic requirement; wound bed to heal; cannot
meshed STSG have the best survival/ be meshed
engraftment rate; they become
vascularized more rapidly

B. Steps of graft take (engraftment):


1. Aherence (rst ay)—brin strans link collagen an elastin to bring the graft closer to the woun
be
. Plasmatic Imbibition (in the rst 3 ays)—the graft absorbs transuative ui from the recipient site,
serves to nourish the graft until it becomes vascularize
3. Inosculation (in 3–4 ays)—cut ens of graft vessels join with the ens of similar-size recipient
bloo vessels from the woun be
4. Neovascularization (in 5 ays)—the growth of new vessels from the woun be into the graft

321
322 PArt i Patient Care

Questions
1. Which of the following is true regaring 5. A 64-year-ol male with chronic obstructive
component separation for abominal wall pulmonary isease (COPD) presents to the ED
reconstruction? with full-thickness burns to the majority of the
A. A patient with a prior colostomy is not an right arm after his robe caught on re while he
appropriate caniate was cooking. Several ays later, he unergoes a
B. Transverse abominis release (TAR) shoul planne split-thickness skin graft (STSG) using
be routinely performe with component his left anterior thigh as a onor site. Halfway
separation through the anticipate harvest of onor skin
C. Prior eep inferior perforator ap is a relative using the ermatome, the surgeon notes visible
contrainication fat. Which of the following is the best next step?
D. An intraoperative enterotomy requires A. Terminate the proceure an rescheule
aborting the proceure B. Continue harvesting with the ermatome at
E. The semilunaris line is reconstructe the same site while attempting to aim more
supercially to obtain the planne STSG
2. A 1-year-ol female has a ash burn to her face, C. Continue harvesting with the ermatome at
sustaining a 3-cm full-thickness woun to her the same site with no change to the angle of
right cheek. Which of the following woul be the the ermatome in an attempt to now harvest
best skin graft? full-thickness skin graft
A. Full thickness from behin the ear D. Stop the ermatome at the current site, an
B. Full thickness from the waist at the inguinal attempt harvesting at another site
fol E. Stop the ermatome at the current site, suture
C. Full thickness from the wrist fol the skin, an attempt harvesting at another
D. Split thickness from the anterior thigh site
E. Split thickness from the posterior thigh
6. Which of the following is an important reason to
3. The most important reason to avoi split- use meshe split-thickness skin-graft (STSG) as
thickness skin grafts (STSGs) over an extremity oppose to non meshe STSG?
joint is: A. Meshe STSG allows for use in a woun be
A. There is an increase risk of infection over with poor granulation tissue
joints B. Meshe STSG allows for use in an ischemic
B. The rate of contracture over a joint can be woun be
ebilitating C. Meshe STSG allows rainage of ui an
C. There is a higher risk of graft necrosis bloo
D. There is a high rate of seroma formation D. Wiely meshe skin is associate with less
compare with other areas on the boy scarring
E. There is reuce imbibition E. Meshe STSG allows for increase amounts of
anexal structures
4. Which of the following is true regaring skin
grafts? 7. Which of the following is the recommene
A. Full-thickness skin grafts (FTSGs) are more surveillance regimen to etect silent rupture for
amenable to imbibition compare with split- a 45-year-ol woman with silicone gel–lle
thickness skin grafts (STSGs) bilateral breast implants?
B. Allografts will eventually get vascularize A. Annual ultrasoun
C. The most common reason for skin graft loss is B. Ultrasoun as neee for pain/iscomfort
a nonviable woun be C. Magnetic resonance imaging (MRI) 3 years
D. The egree of primary contraction is inversely after implant surgery an then every  years
proportional to the amount of ermis in the for life of the implant
skin graft D. Annual plain lms
E. Seconary contraction is greater with FTSG E. Compute tomography (CT) every 5 years for
the life of the implant
CHAPtEr 23 Plastic Surgery 323

8. A 40-year-ol female who has a esire for 13. Pair the ominant vascular supply of the rectus
reconstructive breast surgery after a mastectomy abominis muscle with the correct feeing vessel:
is offere a eep inferior epigastric perforator A. Supercial epigastric artery from the internal
(DIEP) ap. What is the isavantage to thoracic artery
performing a DIEP ap compare to a stanar B. Inferior epigastric artery from the external iliac
peicle ap in this patient? artery
A. It has a higher rate of ap necrosis C. Supercial epigastric artery from the
B. It has an elevate rate of onor site morbiity intercostal arteries
C. Patients have permanent nerve ysfunction D. Inferior epigastric artery from the internal iliac
D. Patients have increase pain artery
E. It is a longer operation E. Deep circumex iliac artery from the internal
iliac artery
9. What is the most common early postoperative
complication in gynecomastia surgery? 14. Which of the following bones is the most common
A. Woun infection isolate orbital bone fracture?
B. Hematoma A. Ethmoi
C. Uner-resection of tissue B. Frontal
D. Asymmetry of breast tissue C. Maxillary
E. Nipple/areola epression D. Lacrimal
E. Zygomatic
10. Which of the following is the most important
principle in repair of a lip laceration? 15. What is the mainstay of postoperative ap
A. Closure of the mucosal layer monitoring?
B. Primary closure of the muscularis A. Doppler ultrasoun
C. Reapproximation of the vermilion-cutaneous B. Pulse oximetry
junction C. Clinical observation
D. Minimal stitching D. Quantitative uorometry
E. Alignment of the unerlying teeth E. Surface temperature probing

11. Which of the following is a contrainication for 16. Which of the following is an appropriate
negative-pressure woun therapy (NPWT)? caniate for repair of a cleft lip?
A. Newly grafte skin A. 1-year-ol female with hemoglobin of 9 g/L
B. Wouns with a stula B. 6-month-ol male with a boy weight of
C. Diabetic wouns 9pouns
D. Ischemic wouns C. 1-week-ol male with hemoglobin of 11 g/L
E. Venous stasis wouns D. 1-year-ol male with prealbumin less than
3mg/L
12. Which of the following is require along with E. 6-month-ol female with concurrent pulmonic
vitamin C to complete cross-linking of proline stenosis
resiues in collagen?
A. Oxygen
B. Oxygen an vitamin A
C. Iron an alpha-ketoglutarate
D. Oxygen, iron, an alpha-ketoglutarate
E. Oxygen, iron, an penicillamine

Answers
1. C. Complex hernia is a term use to escribe abomi- abut bony lanmarks, an recurrent hernias. A component
nal wall efects that are characterize by loss of abominal separation is a technique utilize for complex hernias with
omain an/or those associate with parastomal hernias, the goal of primarily closing the fascial efect without ten-
enterocutaneous stulas, nonmiline hernias, hernias that sion. Previous violation of the rectus, incluing a prior
324 PArt i Patient Care

colostomy, is not consiere a contrainication (A). Compo- important woun closure mechanism. Myobroblasts, spe-
nent separation may also be a useful technique to consier cialize broblasts characterize by intracellular smooth
in the setting of a contaminate el when prosthetic mesh muscle actin laments, contribute to this. There is no
cannot be use (D). However, this proceure is best reserve increase risk of joint infection with STSG (A). Skin grafts
for patients fully optimize to prevent hernia recurrence over a joint have a higher rate of failure ue to sheer force
after abominal wall reconstruction. This inclues opti- an not because of seroma formation (D). Thus, it is import-
mizing nutrition, complete cessation of smoking an iet/ ant to immobilize the joint uring the healing phase to allow
exercise with weight loss. Component separation was rst for graft take (C, E). Rehabilitation with stretching, exercise,
escribe by Ramirez et al. as a technique to repair large ven- an splinting can minimize contracture evelopment. Sur-
tral abominal wall efects. It begins with: gical release of tight bans may also be necessary to restore
1. Rectus muscle separate from the posterior rectus normal function.
sheath
2. External oblique muscle separate from the internal 4. B. Full-thickness an most eep-partial-thickness
oblique at the linea semilunaris wouns will require skin grafting. This shoul take place after
3. Once the external oblique muscle is release from its the woun has been ebrie an a healthy, viable woun
fascia, the compoun ap of rectus muscle an the be is available. If the woun be is not reay for skin graft-
attache internal oblique/transversus abominis is ing, biologic coverage can be achieve with either allograft/
avance towar the miline for primary closure of homograft (caaver skin) or xenograft (bovine skin). Unlike
the fascia. The compoun ap can be avance at the xenograft, allograft will eventually vascularize. However,
epigastrium: 5 cm, waist: 10 cm, an suprapubic: 3 cm both will be rejecte an are thus only use as a temporary
per sie. measure. The only permanent solution is autograft (using
The main supply of the rectus muscle is the superior an the patient’s own skin). The surface area an location of the
eep inferior epigastric arteries. Therefore, prior eep infe- woun will etermine if STSG (which contains all epiermis
rior epigastric perforator ap is a relative contrainication to an some ermis) or FTSG (which contains all epiermis an
component separation because in the DIEP ap proceure, ermis) will be neee. The FTSG onor site will nee to
the eep inferior epigastric artery is harveste, an thus the be primarily close, an thus FTSG is appropriate only for
health of the rectus muscle may be compromise. In cases small wouns in the face an hans to ensure a cosmetically
where aitional length is require, TAR is performe (B). an functionally soun repair. In the case of STSG, meshing
Posterior component separation with TAR allows for an the harveste skin in a 1:1 to 4:1 ratio will allow for cover-
avancement of 8 to 1 cm per sie. Approximately 0.5 cm age of a larger area. The grafts are subjecte to immeiate
meial to the linea semilunaris, the posterior rectus sheath shrinkage, or primary contraction, as well as seconary con-
is incise, an the transversus abominis muscle bers are traction. Primary contraction is epenent on the recoil of
ivie with electrocautery, afforing aitional length to elastic bers in the ermis; thus, this occurs more frequently
the posterior rectus sheath. The posterior rectus sheath can with FTSG. The egree of seconary contraction is inversely
then be reapproximate with sutures. Mesh shoul be place proportional to the amount of ermis in the skin graft an
in the retrorectus space (on top of the reapproximate poste- thus occurs more commonly with STSGs (D, E). The newly
rior rectus sheath) in a sublay fashion to reinforce the repair. grafte skin survives by three main mechanisms. For the rst
The anterior rectus sheath is then reapproximate on top of 3 ays, the graft passively absorbs nutrients from the woun
the mesh to reconstruct the linea alba (E). be by simple iffusion (imbibition). On ays 3 an 4, ino-
References: Heller L, McNichols CH, Ramirez OM. Component sculation allows for a irect connection of the skin graft to
separations. Semin Plast Surg. 01;6(1):5–8. vessels in the woun be. By ay 5, neovascularization an
Garvey PB, Bailey CM, Baumann DP, Liu J, Butler CE. Viola- angiogenesis have occurre, allowing the graft to survive
tion of the rectus complex is not a contrainication to component with its own bloo supply. STSGs have a higher chance of
separation for abominal wall reconstruction. J Am Coll Surg. survival because the thinner skin makes it easier for imbi-
01;14():131–139. bition an inosculation to occur early in the healing process
(A). Skin grafts fail by four main mechanisms, with the most
2. A. FTSG harveste from the upper eyelis, posterior common being hematoma or seroma formation, preventing
auricular region, or supraclavicular fossa are the ieal onor the necessary contact of the skin graft to the woun be (C).
sites for efects on the face versus split-thickness skin graft Other mechanisms of failure inclue infection, poor woun
(D,E). FTSG harveste behin the ear as oppose to the wrist be, an sheer forces.
an inguinal fol gives the best color match, texture, an Reference: Mathes SJ. Reconstructive surgery: principles, anatomy
thickness (B). Aitionally, FTSGs unergo less seconary and techniques. Elsevier Science; 1997.
woun contracture an thus less istortion of the face once it
is heale. The wrist crease can also be use, but many o not 5. E. Patients with home oxygen are at a higher risk for
like the future appearance of the scar on the wrist (C). burn injuries. This inclues patients with COPD. This patient
suffere a full-thickness burn to the majority of the upper
3. B. STSG has the highest seconary contracture (the extremity an thus will require split-thickness skin grafting.
egree of shrinkage uring woun healing). STSG will yiel Full-thickness skin grafting is not appropriate for a large
a higher rate of contracture over a joint resulting in ebili- woun be. STSGs are aroun 0.015 inches eep an take
tating han function. Therefore, FTSG is the preferre graft about 7 to 14 ays to reepithelialize. The harvesting of skin
over any joints in the extremity. Contraction resulting from is highly epenent on both the user an the ermatome.
centrifugal forces in the center of the woun constitutes an If the angle, set epth, an pressure are not correct, one
CHAPtEr 23 Plastic Surgery 325

risks harvesting skin that is too thin or cutting too eep. Reference: Garvey PB, Buchel EW, Pockaj BA, et al. DIEP an
Seeing visible fat inicates that the graft was harveste as peicle TRAM aps: a comparison of outcomes. Plast Reconstr Surg.
a full-thickness graft. The technical error is either ue to the 006;117(6):1711–1719.
user using too much force or the ermatome being set at an
inappropriate epth. In this case, the best next step is to stop 9. B. Gynecomastia is a conition resulting from the abnor-
the ermatome, suture the skin, an attempt harvesting at an mal benign proliferation of glanular breast tissue in men.
alternative site (A–D). Most patients seek surgical treatments for symptoms such
References: Kim S, Chung SW, Cha IH. Full thickness skin as pain, hypersensitivity of the nipple, an psychologic
grafts from the groin: onor site morbiity an graft survival rate well-being. This can be one with surgical excision, suc-
from 50 cases. J Korean Assoc Oral Maxillofac Surg. 013;39(1):1–6. tion-assiste lipectomy, or ultrasoun-assiste liposuction.
Weber RS, Hankins P, Limitone E, et al. Split-thickness skin If surgical excision is chosen, a periareolar incision is mae
graft onor site management. A ranomize prospective trial com- at the junction of the areola an the skin. Next, a cuff of tis-
paring a hyrophilic polyurethane absorbent foam ressing with
sue 1 to 1.5 cm in thickness is preserve irectly eep in the
a petrolatum gauze ressing. Arch Otolaryngol Head Neck Surg.
nipple/areola complex. This maneuver prevents postoper-
1995;11(10):1145–1149.
ative nipple/areola epression or aherence of the nipple/
6. C. STSG contains epiermis an various amounts of er- areola to the pectoralis major muscle (E). The most common
mis. Meshing of the graft increases the surface area, allow- early complication after gynecomastia surgery is hematoma.
ing for increase tissue coverage as well as rainage of ui The hematoma shoul be evacuate, if possible. Uner-re-
an bloo. However, a wiely meshe skin graft is subject section of tissue is the most common long-term complication
to increase scarring an longer healing times (D). Poor of gynecomastia surgery (C). Postoperative woun infection
granulation tissue an an ischemic or infecte woun be is uncommon because it is a clean operation (A). The use of
are relative an absolute contrainications for skin grafting, prophylactic antibiotics, particularly in liposuction cases,
respectively (A, B). Anexal structures are containe in the may account for the low incience of this complication.
ermis an thus are more abunantly available with FTSG Newer techniques allow for superior cosmesis, an as such,
(E). asymmetry is uncommon (D).
Reference: Thorne C. Techniques an principles in plastic
7. C. Breast implants are not lifetime evices an will surgery. In: Thorne CH, Gurtner GC, Chung KC, et al., es. Grabb
and Smith’s plastic surgery. 7th e. Lippincott Williams an Wilkins;
often nee reoperation for implant removal with or without
013:1–1.
replacement. Common inications for reoperation inclue
capsular contracture, rupture, poor cosmesis, infection, an
pain. MRI is the most sensitive an specic moality avail-
10. C. The reapproximation of the vermilion-cutaneous junc-
tion is the main goal of lip laceration repair. A vermillion bor-
able to etect silent rupture of breast implants. The Foo an
er mismatch of 1 mm is visible to the nake eye. Thus, repair
Drug Aministration issue guielines in 011 recommen-
of the vermillion borer uner loop magnication is para-
ing that all recipients of silicone gel-lle breast implants
mount to recreating the lip borers. This will optimize both
receive MRI screening 3 years after implant surgery an then
cosmesis an function following repair. The vermilion borer
every  years for the life of the implant. CT is less sensitive
is initially close with interrupte sutures. This is followe
for the etection of silent rupture an exposes patients to
by closure of the muscularis an then interrupte absorbable
unnecessary raiation (E). Ultrasoun can also be use but is
sutures in the mucosa (A, B). Because each layer will nee to
not as accurate as MRI (A, B). Plain lms are not use in the
be close, multiple stitches will be use (D). The teeth are not
etection of rupture breast implants (D).
a priority in this situation an can be xe at a later time (E).
Reference: Centers for Devices an Raiological Health. FDA
Update on the Safety of Silicone Gel-Filled Breast Implants. US Foo an
Drug Aministration; June 011. 11. D. NPWT works by multiple mechanisms, incluing
reuction of eema an removal of woun ui rich in
8. E. A DIEP ap is a fasciocutaneous ap, also known as a estructive enzymes that are prouce by both the patient
perforator ap, whereby the skin an subcutaneous fat are an local bacterial contamination. In aition, employing the
remove from a istant or ajacent part of the boy to be cyclic compression moe allows stimulation of the mechano-
use to reconstruct another site. The major avantage of a transuction pathways in the woun, resulting in increase
perforator ap is that it reuces the morbiity at the patient’s growth factor release, matrix prouction, an cellular pro-
onor site, mainly because it oes not require the sacrice of liferation. Common clinical scenarios amenable to NPWT
the fascia or muscle (B). Because the fascia stays intact, so too inclue lymphatic leaks, venous stasis wouns, iabetic
o the nerves innervating the muscle, an thus nerve ys- wouns, wouns with stula, sternal wouns, orthopeic
function is kept to a minimum, an without nerve or muscle wouns, an abominal wouns (B, C, E). Likewise, NPWT
amage, the pain is reuce (C, D). Finally, keeping the fas- is frequently use as an alternative to bolster ressings for
cia intact reuces the risk of hernias from the onor site. The split skin grafts, reucing the risk of a seroma or hematoma
major isavantage is that it must be one by a microsurgery uner the graft (A). There are several contrainications to the
specialist an takes a longer time compare to a stanar use of NPWT, an these inclue the presence of malignancy,
peicle ap. When comparing a DIEP to a peicle transverse use on wouns characterize by ischemia, as well as inae-
rectus abominis musculocutaneous (TRAM) ap, the DIEP quately ebrie or baly infecte wouns. There have been
is associate with a shorter hospital stay, a ecrease rate of reports of extension of the zone of necrosis when use on
onor site hernias, an a statistically signicantly lower rate ischemic wouns. Patients with ischemic wouns shoul be
of fat necrosis (17.7% versus 58.5%) (A). consiere for revascularization before application of NPWT.
326 PArt i Patient Care

Reference: Thorne C. Techniques an principles in plastic 15. C. Flap color, capillary rell, tissue bleeing, an ap
surgery. In: Thorne CH, Gurtner GC, Chung KC, et al., es. Grabb temperature are all assesse to ensure aequate ap perfu-
and Smith’s plastic surgery. 7th e. Lippincott Williams an Wilkins; sion. The gol stanar for assessing the viability of trans-
013:1–1. ferre tissue is clinical examination (A, B, D, E). Ientication
of a failing or insufciently perfuse ap can occasionally
12. D. Oxygen, iron, vitamin C, an alpha-ketoglutarate all be challenging for even the most experience microsurgeon.
participate in the hyroxylation an subsequent cross-linking
A Doppler probe can be a useful ajunct to assess vascular
in collagen (A, C). Vitamin A is essential because it promotes
ow within the peicle an/or specic areas of the ap.
epithelialization in collagen synthesis for woun healing, but
Experience personnel are essential for monitoring a ap
it oes not participate in cross-linking of proline resiues in
postoperatively. Doppler monitoring is, however, subject to
collagen (B). Penicillamine is associate with a reuction in
error (both false-positive an false-negative) an thus shoul
numbers of T-lymphocytes, inhibition of macrophage func-
never replace clinical assessments. A number of clinical signs
tion, ecrease numbers of IL-1, an rheumatoi factor. In
(present either singly or in combination) may suggest malp-
aition, it prevents collagen from cross-linking (E).
erfusion. Pale ap color, reuction in ap temperature, loss
Reference: Thorne C. Techniques an principles in plastic
of capillary rell, an loss of ap turgor may inicate arterial
surgery. In: Thorne CH, Gurtner GC, Chung KC, et al., es. Grabb
and Smith’s plastic surgery. 7th e. Lippincott Williams an Wilkins;
insufciency. Venous insufciency, on the other han, can
013:1–1. result in a purple or blue hue in the ap, congestion, swell-
ing, an rapi capillary rell in the early stages, followe by
13. B. The rectus abominis muscle has a ual-ominant eventual loss of capillary rell. Venous congestion may be
bloo supply. The upper vessel is the superior epigastric aresse by surgical measures as well as the application of
artery, which is one of the terminal branches of the internal meical Hirudo medicinalis leeches or by chemical “leeching,”
thoracic artery (previously known as the internal mammary which is topical heparin combine with ermal punctures.
artery). The lower vessel is the inferior epigastric artery, References: Losee J. E., Gimbel M. L., Rubin J, et al. Plastic
arising from the external iliac artery above the level of the an reconstructive surgery. In: Brunicari F, Anersen DK, Billiar
TR, Dunn DL, Hunter JG, Matthews JB, Pollock RE. es. Schwartz’s
inguinal ligament (D). The supercial (not superior) epigas-
principles of surgery. 10th e. McGraw Hill Eucation; 015.
tric artery arises from the femoral artery (A, C) an oes
Thorne C. Techniques an principles in plastic surgery. In:
not supply the rectus abominis. The eep circumex iliac Thorne CH, Gurtner GC, Chung KC, et al., es. Grabb and Smith’s
artery arises from the external iliac artery an supplies the plastic Surgery. 7th e. Lippincott Williams an Wilkins; 013:1–1.
iliac crest (E).
16. C. Cleft lip is one of the most common congenital efor-
14. C. Most isolate orbital fractures involve the orbital mities. Intervention is aime at restoring facial appearance
oor, which is mae up mainly of the maxillary bone (A, B, an oral function. There is still ebate as to the ieal tim-
D, E). Most pure blowout fractures involve the orbital oor. ing for repair, but the “rule of 10s” is a general guieline to
The most common complication after an orbital oor fracture help select appropriate caniates for repair. This inclues
is entrapment. There are two uncommon complications after a hemoglobin greater than 10 g/L, age above 10 weeks,
orbital bone fracture. Superior orbital ssure syndrome results an a boy weight more than 10 pouns (A, B). Contrain-
from compression of structures containe in the superior ications to repair inclue severe malnutrition an concur-
orbit. These inclue cranial nerves III, IV, an VI. Compres- rent cariac anomalies requiring repair (D, E). Cleft palate
sion of these structures leas to symptoms of eyeli ptosis, involves the har palate anterior to the incisive foramen an
globe proptosis, an paralysis of extraocular muscles. If the repair shoul be elaye until 1 year of age to prevent inter-
optic nerve is also involve, symptoms inclue blinness, ference with maxillofacial growth. Too-early repair risks an
an the synrome is ubbe orbital apex syndrome. Both of increase incience of mile ear infections an resultant
these synromes are meical emergencies, an steroi ther- hearing loss.
apy or surgical compression shoul be consiere.
Genitourinary
AMANDA C. PURDY AND JEREMY M. BLUMBERG 24
ABSITE 99th Percentile High-Yields
I. Anatomy
A. Renal hilum from anterior to posterior: renal Vein, renal Artery, renal Pelvis (VAP)
B. Right renal artery courses posterior to the IVC; left renal vein courses anterior to the aorta; right arenal
vein rains into IVC; left arenal, gonaal, lumbar veins rain into left renal vein

II. Testicular Torsion


A. Usually in infants or uring puberty; presents with acute testicular pain; testicle can be high-riing, lying
horizontally, swollen, tener, an hyperemic; absent cremasteric reex; Prehn sign: elevation of testicle
oes not relieve pain in torsion but oes relieve pain in epiiymitis
B. Workup: ultrasoun with oppler shows ecrease testicular perfusion; imaging is not necessary for
iagnosis an if high clinical suspicion, shoul go to OR
C. Must treat urgently to salvage testicle—if etorsion one within 6 hours, salvage rate is near 100%,
salvage rate ecreases to 50% by 1 hours an 10% if more than 4 hours
D. Management: surgical etorsion an bilateral orchiopexy with miline scrotal incision; if torse testicle
not viable, perform orchiectomy an contralateral orchiopexy

III. Epiiymitis (Inammation of the Epiiymis, Usually Due to Infection)


A. In men <35, most ue to chlamyia an/or gonorrhea; in men >35, most ue to E. coli
B. Presentation: acute scrotal pain, point tenerness to posterior testicle, improvement of pain with
elevation of the testicle
C. Diagnosis is clinical; urinalysis an culture (however, urinalysis often negative an negative test oes not
rule out epiiymitis), nucleic aci amplication test (NAAT) for chlamyia an gonorrhea
D. Management: NSAIDs, scrotal elevation, antibiotics; if suspect chlamyia/gonorrhea, treat with IM
ceftriaxone ×1 ose, an oral oxycycline or azithromycin for 10 ays; if o not suspect STI, treat with
levooxacin for 10 ays

IV. Hyrocele (Asymptomatic Flui Collection Aroun Testicle That Transilluminates)


A. Communicating hyroceles are ue to a patent processus vaginalis with free ow of ui between the
scrotum an peritoneal cavity; more common in infants
B. Size of hyrocele will uctuate, shrink when supine an enlarge when upright (helps ifferentiate from
varicocele which oes not uctuate in size)
C. Noncommunicating hyroceles ue to accumulation of secretions within the tunica vaginalis; this is
more common in aults
D. Presents with a cystic scrotal mass that transilluminates
E. In babies, most will resolve spontaneously by 1 year of age

327
328 PArt i Patient Care

F. Communicating hyroceles come with a risk for inirect inguinal hernia an shoul be repaire if they
persist after a chil is 1 to  years ol; repair is high ligation of the patent processus vaginalis
G. Noncommunicating hyroceles that are symptomatic can be treate with hyrocelectomy; most
common complication of this proceure is postoperative hematoma

V. Cryptorchiism (a Testicle Not Within the Scrotum by 4 Months of Age)


A. Testis shoul escen by 4 months of age; unescene testicle most commonly in the supercial
inguinal ring, followe by the inguinal canal
B. Associate with increase risk of ecrease fertility, testicular cancer (especially seminoma), inguinal
hernia, an torsion
C. Treat with unilateral orchiopexy if contralateral testicle has normally escene (shoul be one after 6
months ol, before 1-year-ol); bilateral orchiopexy consiere if clinical scenario inicates potential for
testicular torsion incluing bell-clapper eformity (long mesenteric attachment of testicle allowing it to
rotate)

VI. Testicular Cancer


A. A soli testicular mass is cancer until proven otherwise
B. Initial workup is with ultrasoun, tumor markers (AFP, beta-hCG, LDH); then get tissue iagnosis
with transinguinal raical orchiectomy; on’t approach via scrotum because scrotal lymphatics will be
violate, which increases local recurrence an alters patterns of metastasis
C. 90% are germ cell tumors, which can be seminomas or nonseminomas; seminomas most common,
raiosensitive, an always have normal AFP; nonseminomas (incluing embryonal carcinoma, yolk sac
tumor, teratoma, an choriocarcinoma) are not raiosensitive an may have elevate AFP
D. First site of metastasis for testicular cancer usually the retroperitoneal lymph noes; choriocarcinoma is
the exception—spreas hematogenously
CHAPtEr 24 Genitourinary 329

Fig. 24.1 Psoas-Hitch

Fig. 24.2 If a psoas hitch cannot aequately brige the gap between the injure ureter an the blaer, a Boari ap may
be use. A wie ap of the anterior blaer wall is create an then tubularize to meet the istal en of the iseases or
injure ureter.

AL GRAWANY
330 PArt i Patient Care

Questions
1. A 5-year-ol man presents for a palpable mass 4. A 35-year-ol woman is in the hospital after
in his right testicle that has been present for a few unergoing elective sigmoiectomy for
months. On ultrasoun, there is a soli -cm mass recurrent iverticulitis. On postoperative ay
in his right testicle that appears well ene, 3, she complains of abominal iscomfort an
homogenous an hypoechoic. AFP, beta-hCG, an her abominal rain puts out 300 mL of clear
LDH are normal. What is the most appropriate ui. Her heart rate is 13 beats per minute.
next step? Analysis of the ui shows signicantly elevate
A. MRI of the abomen an pelvis creatinine compare to serum creatinine. Which
B. Core neele biopsy of the following is true about the most likely
C. Transscrotal orchiectomy complication?
D. Transinguinal orchiectomy A. Ureteral stents place prior to surgery
E. Transinguinal orchiectomy with ecrease the risk of ureteral injury
retroperitoneal lymph noe issection B. Optimal management is to continue
abominal rainage an nephrostomy tube
2. On routine examination of a 1-month-ol boy, placement
it is note that only one testicle is present in the C. If ureteral injury is conrme, the patient
scrotum, on the right. A mass is palpate in the shoul return to the operating room for repair
left inguinal region. What is the most appropriate D. The rate of ureteral injury is higher in
next step? laparoscopic surgery than in open surgery
A. Reexamination in 6 months E. Optimal management is ureteral stent
B. Ultrasoun of the groin an scrotum placement with elaye operative repair
C. Laparoscopic ivision of the testicular vessels
D. Left orchiopexy 5. A 4-year-ol female is about to unergo elective
E. Bilateral orchiopexy surgery for an umbilical hernia. A preoperative
urinalysis is positive for nitrite an leukocyte
3. A -year-ol male presents to the ED with esterase with some bacteria ientie but no
an erection that has laste for 9 hours an squamous cells present. She enies urinary
is becoming exceeingly painful. He enies frequency, urgency, or ysuria. Which of the
genitourinary trauma, rug use, or recreational following is the best next step in management?
use of phosphoiesterase-5 inhibitors. He A. Procee to surgery
also enies any personal or family history of B. Procee to surgery after aministering a single
hematologic iseases. He ha a similar episoe ose of IV antibiotics to cover gram-negatives
8 months ago that resolve spontaneously after C. Delay surgery an aminister a 3-ay course
4 hours, but now he is having severe worsening of oral antibiotics
penile pain. Management of his conition D. Delay surgery, aminister a 3-ay course of
involves: oral antibiotics, then repeat urinalysis
A. Oxygen, IV hyration, an close monitoring E. Repeat urinalysis an procee to surgery only
B. Oral phenylephrine if repeat urinalysis is negative
C. Penile Doppler ultrasoun
D. Corporal aspiration an irrigation with saline 6. Which of the following is true regaring renal
E. Urgent cavernoglanular shunt anatomy?
A. The renal vein is posterior to the renal artery in
the hilum
B. If the left renal vein nees ligation, it is best to
o so near the renal hilum
C. Glisson capsule surrouns the kiney
D. The left renal vein crosses posterior to the
aorta
E. The right renal artery crosses posterior to the
IVC
CHAPtEr 24 Genitourinary 331

7. A 14-year-ol boy presents to the ED with nausea, 10. A 57-year-ol male presents to the ED with
vomiting, an severe left scrotal pain that woke a severe heaache that starte suenly. His
him from sleep 3 hours ago. He enies scrotal systolic bloo pressure is 0 mm Hg which
trauma or recent infections. He ha a similar improves with labetalol. He oes not have any
episoe 6 months ago that resolve within meical problems but has recently reporte
minutes. Physical exam reveals an enlarge, rm, some scrotal iscomfort. On exam, his left testicle
an tener left testicle that appears to be high in has a painless soft mass, external to the testicle,
scrotum with abnormal lie. Stroking the left inner that feels like a “bag of worms.” When he lies
thigh oes not elicit elevation of the hemiscrotum. supine, the mass oes not isappear. Urinalysis
Manual elevation of the scrotum oes not relieve emonstrates 18 re bloo cells per high power
the pain. Which of the following is the best next el. Which of the following is the best next step
step in management? in management?
A. Testicular Duplex ultrasoun A. Compute tomography (CT) of the hea
B. Attempt left testicular etorsion in the ED an, B. Testicular ultrasoun
if successful, amit for close observation C. CT of the abomen/pelvis
C. Attempt left testicular etorsion in the ED D. Renal ultrasoun
followe by left testicular orchiopexy in the E. Reassurance an referral to primary care
OR physician to begin antihypertensives
D. Take to the OR to perform left testicular
etorsion an bilateral orchiopexy 11. An otherwise healthy 6-year-ol male presents
E. Attempt left testicular etorsion in the ED an with pneumaturia, urinary frequency, an
if successful perform testicular Duplex scan ysuria for several weeks. He is afebrile an
hemoynamically stable. Urinalysis is negative
8. A 45-year-ol male presents to clinic with his for bloo. Urine culture grows multiple
wife to iscuss having a vasectomy. Which of the organisms. CT scan shows air in the blaer with
following is true regaring this proceure? colonic iverticulosis. Cystoscopy is negative an
A. It is typically performe by a urologist in the colonoscopy is negative other than iverticula.
operating room (OR) uner general anesthesia Optimal management consists of:
B. It involves ligating the vas eferens A. Total parenteral nutrition, bowel rest, an
C. The patient can safely have intercourse 1 antibiotics
month after the proceure with little risk of B. Colon resection with primary closure of
pregnancy blaer
D. Reversal of vasectomy is associate with a C. Colon resection an excision of cuff of blaer
pregnancy rate of less than 10% D. Eight-week course of oral antibiotics
E. There is an increase risk for testicular cancer E. Fulguration via cystoscopy

9. A 67-year-ol male unergoes an uneventful 12. A 45-year-ol male presents to the ED with
raical prostatectomy for prostate cancer. Eight nausea, vomiting, an a sharp right groin pain
ays later, he has a fever an feculent material is that starte 6 hours ago. He is unable to n
note in his urine. Pelvic CT reveals a 9- × 8-cm a comfortable position an moves aroun
heterogeneous perirectal ui collection. The frequently in the hospital be. He is afebrile an
best course of management is aministering hemoynamically stable. CT abomen/pelvis
parenteral antibiotics, percutaneous rainage of without contrast reveals a 4-mm right-sie stone
the ui collection, an: at the ureterovesical junction (UVJ) with mil
A. Repeat CT in  weeks hyronephrosis an some periureteral straning.
B. Initiation of total parenteral nutrition He has no ysuria an his urinalysis is negative
C. Initiation of enteric feeing via a Dobhoff tube for infection. His pain an nausea improve with
D. Insertion of a suprapubic cystostomy tube meical therapy. Which of the following is the
E. Diverting colostomy creation most appropriate course of management?
A. Meical expulsive therapy (tamsulosin,
nonsteroial antiinammatory rugs
[NSAIDs]) an outpatient follow-up
B. Ureteral stent placement
C. Extracorporeal shock wave lithotripsy (ESWL)
D. Ureteroscopy an laser lithotripsy
E. Percutaneous nephrostomy tube placement
332 PArt i Patient Care

13. An 18-year-ol woman presents to the ED 16. A 3-year-ol male is brought to the ED by
following a motorcycle collision. She is ambulance after a motorcycle accient at 45 mph.
hemoynamically stable but has an obvious Abominal CT scan with contrast emonstrates a
pelvic fracture. On exam, bloo is foun at eep renal laceration with urinary extravasation
the vaginal introitus. CT abomen/pelvis into the retroperitoneum. After observation for
emonstrates a severe pelvic fracture with 10 ays, a repeat CT urogram shows persistent
normal-appearing kineys. The best next step is: urinary extravasation with evelopment of a
A. Urethral catheter small urinoma. There is no hyronephrosis. He is
B. Cystogram hemoynamically stable an afebrile. The most
C. Retrograe urethrogram an cystogram appropriate next step is:
D. Urethroscopy, vaginoscopy, an cystogram A. Continue observation
E. Suprapubic blaer catheter B. Surgical exploration an repair
C. Insertion of a ureteral stent
14. A 36-year-ol female is amitte to the hospital D. Percutaneous nephrostomy rainage
after being struck by an automobile while riing E. Percutaneous perinephric rainage
her motorcycle. Plain lms emonstrate a fracture
at her inferior pubic ramus. Upon urethral 17. A 49-year-ol male unergoes a low anterior
catheter placement, she was foun to have gross resection for rectal cancer. During mobilization
hematuria. CT cystogram reveale contrast an issection of the sigmoi colon, the left ureter
extravasation into the extraperitoneal space with is injure. The injure segment measures 1 cm in
no bony structures within the blaer wall. The length an is locate above the pelvic brim. The
patient is hemoynamically stable. Laboratory patient is hemoynamically stable. Which of the
stuies are unremarkable. The next step is: following is the appropriate management for this
A. Prolonge inwelling urethral catheter ureteral injury?
B. Replace urethral catheter with suprapubic A. Resect injure segment an perform a primary
cystostomy en-to-en ureteral anastomosis over a stent
C. Open operative repair of blaer injury B. Ligate ureter an place a percutaneous
D. Cystoscopy to visualize blaer perforation nephrostomy tube
site C. Mobilize ureter an reimplant into the blaer
E. Bilateral nephrostomy tubes for temporary after performing a psoas hitch
urinary iversion D. Perform a nephropexy an
ureteroureterostomy
15. A 19-year-ol male presents to the ED with E. Perform an ileum interposition
a stab woun to his left lower back. He is
hemoynamically stable an has no evience 18. A 7-year-ol male presents to the ED after
of peritonitis. A CT scan of the abomen an sustaining a gunshot woun to the pelvis. He
pelvis with oral an intravenous (IV) contrast unergoes exploratory laparotomy an is foun
emonstrates a subcapsular hematoma of the to have a left sigmoi colon injury, which is
left kiney an a small posterior left kiney repaire primarily. He is hemoynamically stable.
laceration with no extravasation of contrast or On examination of the left istal ureter, it appears
injury to the collecting system. There is no ui or to be contuse. Intravenous inigo carmine is
free air in the peritoneum. Distal ureters are intact aministere, an no extravasation is seen from
bilaterally. The next best step is: the ureter. Which of the following is the most
A. Observation appropriate next step?
B. Retrograe ureteropyelogram A. Observation
C. IV methylene blue an local exploration of B. Ureteral stent
woun C. Percutaneous nephrostomy
D. Retroperitoneal exploration an renal D. Resect amage ureter an reimplant ureter
reconstruction into blaer
E. Exploratory laparotomy, retroperitoneal E. Resect amage ureter an repair with en-to-
exploration, an renal reconstruction en ureteral anastomosis
CHAPtEr 24 Genitourinary 333

Answers
1. D. A soli testicular mass is cancer until proven otherwise. References: Chan E, Wayne C, Nasr A, FRCSC for Canaian Asso-
Initial workup inclues scrotal ultrasoun an tumor ciation of Peiatric Surgeon Evience-Base Resource. Ieal timing of
markers, incluing AFP, beta-hCG, an LDH. Seminomas orchiopexy: a systematic review. Pediatr Surg Int. 014;30(1):87–97.
are classically well ene, oval, homogenous, an Tasian GE, Copp HL. Diagnostic performance of ultrasoun in
nonpalpable cryptorchiism: a systematic review an meta-analysis.
hypoechoic on ultrasoun, whereas nonseminomas appear
Pediatrics. 011;17(1):119–18.
nonhomogeneous, hyperechoic, with calcications, cystic
areas, an inistinct margins. In the presence of a soli mass, 3. D. This patient is suffering from ischemic priapism, a uro-
the next step is obtaining a tissue iagnosis, which is one logic emergency requiring urgent intervention to prevent per-
via transinguinal orchiectomy. It is important not to violate manent erectile ysfunction. This is ue to ecrease venous
the scrotum while obtaining tissue iagnosis, as this can lea outow from the cavernosa an subsequent increase intra-
to lymphatic isruption (B, C). Imaging to assess for istant cavernosal pressure. This also results in ecrease arterial
metastasis is not necessary until the iagnosis of testicular inow, causing stasis of bloo an resultant local hypoxia an
cancer is conrme (A). Tumor markers shoul be repeate aciosis. On exam, the patient has a fully erect, rigi, an ten-
after orchiectomy. A retroperitoneal lymph noe issection er penis, but the glans an corpus spongiosum are soft (cor-
may be inicate for nonseminomas. However, this woul not pora cavernosa are the involve compartments in priapism).
be one uring the initial operation before a tissue iagnosis is Early intervention is very important. In the ED, the patient
establishe (E). The patient shoul be counsele on testicular shoul unergo a corporal aspiration an irrigation with nor-
prosthesis, which may be place uring the orchiectomy. In mal saline to rain static bloo from the corpora an to ush
aition, the patient shoul be counsele on sperm banking out ol clots; this achieves etumescence. Phenylephrine may
prior to orchiectomy if they have risk factors for infertility be injecte intracorporally as well, but the patient must be on
(atrophic contralateral testis, history of infertility). a cariac monitor before oing so because of the risk of hyper-
Reference: Gilligan T, Lin DW, Aggarwal R, et al. Testicular tension, tachycaria, reex braycaria, an arrhythmia if the
Cancer, version .00, NCCN clinical practice guielines in oncol-
phenylephrine is systemically absorbe. If etumescence is
ogy. J Natl Compr Canc Netw. 019;17(1):159–1554.
not successfully achieve by corporal aspiration/irrigation,
2. D. This patient has cryptorchiism. In the majority of the patient shoul unergo a cavernoglanular shunt proce-
infants, the testicles reach the scrotum by 3 to 4 months ol, ure, though this is more invasive an has a higher risk of
an spontaneous escent into the scrotum after 4 months is permanent erectile ysfunction (E). Oral phenylephrine has
unlikely. In patients with cryptorchiism, the unescene not been shown to be benecial for priapism (B). In sickle cell
testicle(s) are expose to increase temperatures, which patients with priapism, rst-line management is meical ther-
leas to stunte growth, ecrease spermatogenesis, an apy with oxygen, IV hyration, an pain control (A). Bloo
an increase risk for subsequent infertility. These patients exchange transfusions to reuce the concentration of HbS are
also have a higher risk of eveloping testicular cancer an inicate in sickle cell patients if initial meical therapy fails.
torsion. The treatment for this is orchiopexy of the affecte Penile Doppler ultrasoun is not routinely one for priapism,
testicle. Orchiopexy, especially when one early, improves though it may be useful to ifferentiate ischemic from nonisch-
fertility, improves testicular growth, minimizes torsion emic priapism (nonischemic priapism is manage conserva-
risk, an may ecrease testicular cancer risk. While there tively an often resolves with observation) (C). Nonischemic
is no clear consensus about whether orchiopexy ecreases priapism is nontener an partially rigi. It is usually ue to
testicular cancer risk, it at least allows for easier etection penile trauma causing a stula between the corporal tissue
of testicular masses. Orchiopexy shoul be one between an the cavernous artery an is not an emergent conition.
6 an 1 months of age an only nees to be one on the References: Broerick, G., et al. (010). Priapism recommen-
ations. Sexual Meicine: Sexual Dysfunction in Men an Women.
affecte sie (E). Reexamination woul be inappropriate as
Thir International Consultation on Erectile Dysfunction (3r
the chil woul be oler than 1 months (A). Imaging is ICUD). In F. Montorsi, et al., (Es.), Plymouth. Unite Kingom:
not neee prior to surgery in this patient with a palpable Health Publication Lt. https://www.auanet.org/guielines/
unescene testicle (B). The majority of unescene tes- guielines/priapism-guieline
ticles are locate in the supercial inguinal ring (most com- Tay YK, Spernat D, Rzetelski-West K, Appu S, Love C. Acute
mon), or the inguinal canal. Less than 10% of patients will management of priapism in men. BJU Int. 01;109 Suppl 3:15–1.
have an intraabominal testicle or an absent testicle. Even
in the event the patient has a unilateral nonpalpable testicle, 4. C. Iatrogenic ureteral injuries are a rare but well-known
imaging oes not nee to be performe prior to exam uner complication uring gynecologic, urologic, colorectal, an
anesthesia with possible exploratory surgery as it oes not vascular surgeries. The risk of ureteral injury in colorectal
ecrease the nee for eventual surgery. In the event the surgery has been shown to be slightly higher in open sur-
patient is foun to have an intraabominal testicle that can- gery, an lower in laparoscopic surgery (D). Ieally, these
not reach the scrotum, ivision of the testicular vessels may injuries are iscovere intraoperatively, so they can be
be necessary to mobilize the testicle to the scrotum. How- repaire without subjecting the patient to a subsequent oper-
ever, that is unnecessary in this case (C). ation. While ureteral stents have not been shown to ecrease
334 PArt i Patient Care

the risk of ureteral injury, they may help surgeons ientify function or rise in creatinine. The Gerota capsule surrouns
injuries intraoperatively (A). In this case, the injury was the kiney while the Glisson capsule surrouns the liver
not iscovere until the postoperative perio. Patients may (C). The renal vein is the most anterior structure in the renal
present with abominal pain or istension, ileus, oliguria, hilum, the renal pelvis is the most posterior structure, an
fever, tachycaria, leakage of clear ui from their incisions, the renal artery is between the two (A). The right renal vein
an/or increase clear rain output. If a rain is present, is short an rains immeiately into the inferior vena cava
ui analysis can be one. High ui creatinine (higher than (IVC), while the longer left renal vein is joine by collateral
serum creatinine, similar to urine creatinine) supports the vessels before entering the IVC. Since the left kiney has a
iagnosis of a urine leak. The iagnosis shoul be conrme longer renal vein, it is the preferre sie for a onor kiney.
with imaging, such as a CT urogram. The management of The left renal vein is joine by the left arenal vein supe-
a postoperative ureteral injury epens on the timing of riorly, the left gonaal vein inferiorly, an the left lumbar
iagnosis. Those iagnose within the rst 5 to 7 postop- vein posteriorly. The left renal vein can be ligate but shoul
erative ays, an with systemic signs (tachycaria) shoul be performe close to the IVC (B); this still permits venous
return to the operating room for repair. If the iagnosis is rainage via collaterals without irreversible renal amage or
elaye more than 7 ays after surgery, the injury shoul be hyronephrosis. The right renal artery passes posterior to the
temporize an treate with elaye surgical repair. Meth- IVC while the left renal vein passes anterior to the aorta (D).
os to temporize the ureteral injury inclue stent placement Rarely, a retroaortic left renal vein is present. This variant can
for incomplete injuries or a nephrostomy tube for complete present problems uring infrarenal aortic surgery because
transections. Neither of which are appropriate in this case the vein is prone to injury an is ifcult to repair.
because it is only postoperative ay 3 (B, E).
References: Bothwell WN, Bleicher RJ, Dent TL. Prophylactic 7. D. This patient presents with the classic clinical picture
ureteral catheterization in colon surgery. A ve-year review. Dis of testicular torsion. Incience of torsion occurs in a bimoal
Colon Rectum. 1994;37(4):330–334. pattern; infant boys (ue to the tunica vaginalis not yet
Halabi WJ, Jafari MD, Nguyen VQ, et al. Ureteral injuries in secure to the gubernaculum in the scrotum) an aoles-
colorectal surgery: an analysis of trens, outcomes, an risk fac- cent boys (rapily growing testicles uring puberty) are at
tors over a 10-year perio in the Unite States. Dis Colon Rectum.
the highest risk of torsion, though it can occur at any age.
014;57():179–186.
Torsion presents with acute onset of severe testicular pain,
5. A. The above patient has asymptomatic bacteriuria, with or without swelling. Many have associate nausea
which is ene by the presence of bacteria or mark- an vomiting that may initially confuse the iagnosis. This
ers thereof (positive for leukocyte esterase, nitrites) in an patient’s history also suggests possible intermittent torsion
appropriately collecte urinalysis (absence or low number that resolve spontaneously, though this is ifcult to iag-
of squamous cells) an without any signs or symptoms of nose enitively. Physical exam nings inclue a tener
a UTI (e.g., urinary frequency, urgency, ysuria). With the rm testicle, horizontal lie of the testicle, high-riing testi-
exception of pregnancy an those unergoing urologic inter- cle, an an absent cremasteric reex (stroking the inner thigh
vention (e.g., prostatectomy, prostate biopsy), ault patients elicits elevation of the hemiscrotum). In contrast to epiiy-
with asymptomatic bacteriuria o not require any treatment mitis, patients with testicular torsion have a negative Prehn
(B–E). In contrast, all symptomatic patients require treat- sign (manual elevation of the scrotum relieves pain). Torsion
ment. Women are at a higher risk for symptomatic UTIs owing is iagnose clinically, an surgical exploration shoul not
to their shorter an straighter urethra. Aitionally, its close be elaye to perform other imaging stuies if suspicion
proximity to the vaginal orice colonize by bacteria makes is high, as in this case (A). If the iagnosis is questionable,
them vulnerable to infection. Most uncomplicate cases can a scrotal Doppler ultrasoun is a reasonable option. This
be manage with a 3-ay course of nitrofurantoin or tri- woul emonstrate an absence of ow an a more hetero-
methoprim-sulfamethoxazole (TMP-SMX). Due to increasing geneous texture of the testicular parenchyma compare with
microbial resistance to ciprooxacin, it shoul be reserve for the contralateral testis. A torse testicle is usually viable if
emographics with TMP-SMX resistance or in cases where etorse within 6 hours. When the suspicion for torsion is
nitrofurantoin or TMP-SMX cannot be use ue to avail- high, the patient shoul be taken irectly to the operating
ability, allergy, or intolerance. Complicate cases require a room. Attempting etorsion in the ED is an option prior to
7-ay course of oral antibiotics an inclue those with pre- surgery, particularly if there will be an anticipate elay in
vious urinary manipulation, abnormal anatomy, an all male getting to the OR, or if a urologist is unavailable. This can
patients. Given the rarity of symptomatic UTI in young men, be attempte in the ED with proper pain meication but
one shoul suspect abnormal anatomy preisposing him to still necessitates an urgent bilateral orchiopexy in the OR
bacteriuria an subsequent infection. Young males an most (B, C, E). After surgical etorsion, both testes are suture to
women with recurrent infections shoul be referre to a the scrotal artos muscle (orchiopexy) to prevent future tor-
urologist to unergo renal ultrasoun an measurement of sion episoes (contralateral testis has a higher risk of torsion
postvoi resiual blaer volume. as well, necessitating concurrent contralateral orchiopexy). A
Reference: Gallegos Salazar J, O’Brien W, Strymish JM, Itani K, common imitator of testicular torsion is epiiymo orchitis,
Branch-Elliman W, Gupta K. Association of screening an treatment ifferentiate by pain relief with testicular elevation, normal
for preoperative asymptomatic bacteriuria with postoperative out- or increase ow in the testicle or epiiymis on Doppler
comes among US veterans. JAMA Surg. 019;154(3):41–48. ultrasoun (increase ow inicating inammation), an
a urinalysis suggesting bacteriuria. Sexually transmitte
6. E. There are two kineys, but humans can survive with infections must also be rule out if epiiymo orchitis is
just one without a clinically signicant ecrease in renal suspecte. If the testicle is foun to be ischemic an oes
CHAPtEr 24 Genitourinary 335

not recover color an appearance after etorsion, testicular an hematuria, but less than 10% have all three nings. In
infarction has resulte an an orchiectomy is necessary. aition, RCC can initially present with a paraneoplastic
References: DaJusta DG, Granberg CF, Villanueva C, Baker LA. synrome that inclues hypertension from renin secretion
Contemporary review of testicular torsion: new concepts, emerging (likely in the above patient), hypercalcemia from parathy-
technologies an potential therapeutics. J Pediatr Urol. 013;9(6 Pt roi hormone (PTH)-relate peptie secretion, polycythemia
A):73–730. from erythropoietin secretion, hypoglycemia from insulin
Johnston BI, Wiener JS. Intermittent testicular torsion. BJU Int.
secretion, an hepatic ysfunction (Stauffer synrome),
005;95:933–934. Sharp VJ, Kieran K, Arlen AM. Testicular tor-
all of which resolve with treatment of RCC. About 40% of
sion: iagnosis, evaluation, an management. Am Fam Physician.
013;88(1):835–840.
patients with RCC have an elevate renin level. Risk factors
for RCC inclue smoking, alcohol, obesity, cystic isease of
8. B. Vasectomy is a very effective metho for male contra- the kiney, an iabetes. The next best step for this patient
ception. It is less costly, safer, an associate with a shorter is to orer a CT scan of the abomen/pelvis with a urogram
recovery time compare with tubal ligation. However, vasec- phase to evaluate for a renal mass an visualize his upper
tomy is performe less frequently, which is likely relate to urinary tracts (A, B, D, E). Although metastasis accounts for
patient misinformation an public stigma. There are a vari- the majority of renal tumors (typically from breast cancer),
ety of methos to perform a vasectomy, an it can be one by the most common primary renal tumor is RCC. The lung is
urologists as well as general surgeons, typically in the ofce the most frequent site of istant sprea. Tissue iagnosis is
with local anesthesia (A). The guiing principle involves require before surgical intervention. Patients with resect-
ligation of the vas eferens, which can be achieve with two able isease an without istant sprea can unergo partial
small scrotal incisions. Although the success rate excees nephrectomy for smaller tumors, or raical nephrectomy
95%, patients shoul not have unprotecte intercourse for for larger tumors or those with local invasion (such as this
3 months after the proceure an only after conrming patient).
sterility with a semen analysis to look for azoospermia (C). Reference: Palapattu GS, Kristo B, Rajfer J. Paraneoplastic syn-
romes in urologic malignancy: the many faces of renal cell carci-
The probability of obtaining a natural pregnancy following
noma. Rev Urol. 00;4(4):163–170.
vasectomy reversal is 50% (D). There is no evience to sug-
gest that patients who have unergone vasectomy have an
11. B. Denitive treatment of a colovesical stula ue to
increase risk for testicular cancer (E).
iverticulitis involves colon resection with primary closure
References: Arahya KW, Best K, Sokal DC. Recent evelop-
of the blaer (C). Though there are some case reports of
ments in vasectomy. BMJ. 005;330(7486):96–99.
van Dongen J, Tekle FB, van Roijen JH. Pregnancy rate after
nonoperative management, particularly in high-risk patients,
vasectomy reversal in a contemporary series: inuence of smoking, this is not the stanar recommenation (A, D). An exception
semen quality an post-surgical use of assiste reprouctive tech- is in patients with Crohn isease. Due to the chronic relaps-
niques. BJU Int. 01;110(4):56–567. ing nature of the isease, meical management with antibiot-
ics, azathioprine, sterois, an/or iniximab may resolve the
9. E. A rare but feare complication of raical prostatec- stula, obviating the nee for resection of part of the blaer.
tomy is rectal injury, with an incience of 1.5%. If ientie If the colovesical stula were ue to malignancy, then an en
intraoperatively, it may be repaire primarily. If the bowel bloc resection woul be recommene. With operative man-
injury is recognize postoperatively as a vesicorectal stula agement of a colovesical stula, an omental ap is place
(as in this case), conservative management is not appropri- between the repaire blaer an bowel to prevent overlap-
ate (A–D). Since the patient has systemic signs of infection, ping suture lines an provie a well-vascularize surface for
he nees to be starte on parenteral antibiotics an the ui healing. Cystoscopy with fulguration an enoscopic stent-
collection nees to be raine. Aitionally, a large ui ing are not use in the management of colovesical stulas (E).
collection suggests a sizeable rectal injury an perforation; Reference: Zhang W, Zhu W, Li Y, et al. The respective role of
this will nee to be treate with a colostomy to temporarily meical an surgical therapy for enterovesical stula in Crohn’s is-
ivert his stool with the intent to perform a elaye repair. ease. J Clin Gastroenterol. 014;48(8):708–711.
References: Harpster LE, Rommel FM, Sieber PR, et al. The
incience an management of rectal injury associate with ra- 12. A. This patient presents with an obstructing 4-mm right
ical prostatectomy in a community base urology practice. J Urol. UVJ stone causing acute pain. The majority are calcium-oxalate
1995;154(4):1435–1438. stones, which are raiopaque. Uric aci stones account for
Rovner ES. Urinary tract stula. In: Campbell MF, Wein AJ, 10% of all nephroliths an are raiolucent, which is why the
Kavoussi LR, es.Campbell-Walsh urology. 9th e. Sauners Elsevier; initial workup shoul inclue a noncontrast stone protocol
007. CT. In the setting of an obstructing istal ureteral stone
without evience of urinary tract infection, it is reasonable
10. C. Scrotal iscomfort accompanie by a mass that feels to observe an meically treat the patient with tamsulosin
like a “bag of worms” is characteristic of a varicocele, but 0.4 mg aily (relaxes ureteral smooth muscle an facilitates
if it right sie, acute onset, or fails to ecompress while stone passage) an NSAIDs, assuming the patient’s pain is
lying supine, it is concerning for proximal venous obstruc- well controlle an oral intake is aequate. Given the stone’s
tion. A left-sie varicocele that fails to ecompress is con- location, there is greater than a 75% chance of spontaneously
cerning for obstruction at the left renal vein, whereas a passing this stone within 3 weeks. Meical expulsive therapy
right-sie one for IVC compression/obstruction. Renal cell is less successful for stone passage if it is larger than 7 mm or
carcinoma (RCC) is one concerning etiology in this setting. if it is in the proximal ureter. ESWL an ureteroscopy/laser
The classic tria for RCC inclues a ank mass, ank pain, lithotripsy are not initially inicate because this stone has a
336 PArt i Patient Care

high chance of passing with meical management alone, but through a Foley catheter an observe for contrast extrava-
may be inicate later if his symptoms persist (C, D). If the sation. Intraoperative blaer ruptures can be similarly iag-
patient meets criteria for prompt intervention, ureteral stent nose by lling the blaer with colore ye (methylene
placement an percutaneous nephrostomy tube placement blue, inigo carmine) to assess for leakage. Management of
woul be inicate to ecompress the urinary system (B, E). an extraperitoneal blaer rupture is manage by a -week
This inclues the following: (1) high-grae unilateral urinary course of an inwelling Foley catheter an a repeat cysto-
obstruction, () bilateral urinary obstruction, (3) urinary gram to ensure blaer healing. Replacing the Foley catheter
obstruction to solitary kiney, (4) urinary obstruction with with a suprapubic cystostomy is invasive an unnecessary
urinary infection or sepsis, (5) inability to tolerate oral intake (B). However, in the setting of persistent hematuria, concomi-
from nausea/vomiting, an (6) severe pain not controlle by tant pelvic organ injury, blaer foreign boies (bullets, bone
oral analgesics. fragments), persistent urine leak, or penetrating trauma,
References: Coll DM, Varanelli MJ, Smith RC. Relationship operative repair of an extraperitoneal blaer rupture is
of spontaneous passage of ureteral calculi to stone size an loca- inicate. Open surgical repair is also necessary for intra-
tion as reveale by unenhance helical CT. AJR Am J Roentgenol. peritoneal ruptures as soon as feasible to prevent peritonitis.
00;178(1):101–103. Intraperitoneal ruptures typically occur at the blaer ome,
Miller OF, Kane CJ. Time to stone passage for observe ure-
which is line by peritoneum. The blaer is close in  to
teral calculi: a guie for patient eucation. J Urol. 1999;16(3 Part
3 layers with absorbable suture (using nonabsorbable suture
1):688–691.
Parsons JK, Hergan LA, Sakamoto K, Lakin C. Efcacy
for blaer wall closures results in calcication of the suture
of alpha-blockers for the treatment of ureteral stones. J Urol. line an blaer stones) (C). Occasionally, blaer injuries
007;177(3):983–987. are not immeiately etecte, an a urinoma may evelop.
Preminger GM, Tiselius HG, Assimos DG, et al. 007 guieline If there is concern for infecte urinoma, the patient may ben-
for the management of ureteral calculi. J Urol. 007;178(6):418–434. et from interventional raiology (IR) rainage, though ae-
quate blaer rainage is usually sufcient. Cystoscopy is
13. D. Open pelvic fractures are associate with very a iagnostic option for intraoperative blaer perforations.
high impact injuries. It occurs most commonly following However, in the setting of a traumatic blaer rupture, a cys-
high-spee motorcycle accients. They have a high rate of togram is the best iagnostic approach because it is quicker,
signicant bleeing an associate injuries an can lea to less invasive, an can ifferentiate between intraperitoneal
life-threatening pelvic sepsis, particularly if a rectal injury an extraperitoneal perforations (D). In poor operative can-
goes unrecognize. Thus, in the setting of a pelvic fracture, it iates who have persistent urine leakage from the blaer
is essential to examine the perineum for evience of external espite urethral rainage, bilateral nephrostomy tubes may
wouns, as well as for bloo in the rectum or vagina (an help ivert the urine temporarily an allow a better oppor-
never assume that vaginal bleeing is ue to menses). Any tunity for the blaer to heal (E).
external perineal wouns or rectal/vaginal bloo shoul be References: Morey AF, Dugi DD. Genital an lower uri-
presume to be ue to an open pelvic fracture until proven nary tract trauma. In: Campbell MF, Kavoussi LR, Wein AJ, es.
otherwise. Given the bloo foun at the vaginal introitus, Campbell-Walsh urology. 10th e. Elsevier Sauners;01.
this patient is at risk of having sustaine injury to the ure- Muny AR, Anrich DE. Pelvic fracture-relate injuries of the
thra, vagina, blaer, or rectum. She will require an exam blaer neck an prostate: their nature, cause an management. BJU
Int. 010;105(9):130–1308.
uner anesthesia, a urethroscopy, a vaginoscopy, an a cys-
togram to evaluate for vaginal, urethral, or blaer trauma
(B). A retrograe urethrogram is technically ifcult to per-
15. A. Historically, the general recommenation has been
that penetrating trauma to the kiney manates surgical
form in a younger female because of a short urethra (aroun
exploration. However, that algorithm has recently been chal-
4 cm); therefore, it is not use in the iagnosis of female
lenge. In select cases in which the patient is hemoynami-
urethral trauma (C). A suprapubic blaer catheter is not
cally stable, the penetrating injury is ue to a stab woun or
necessary without evaluation of the urethra (E). A urethral
a low-velocity gunshot woun, there are no intraabominal
catheter shoul be elaye until a urethral injury is rule out
injuries, an the renal injury is low grae, a nonoperative
given the gross bloo (A).
approach can be implemente (B–E). The main concern with
References: Kong JPL, Bultitue MF, Royce P, Gruen RL, Cato A,
Corcoran NM. Lower urinary tract injuries following blunt trauma: a
a penetrating ank or back woun woul be a misse colon
review of contemporary management. Rev Urol. 011;13(3):119–130. injury. Renal trauma is grae accoring to the severity of
Morey AF, Dugi DD. Genital an lower urinary tract trauma. In: renal parenchymal injury an isruption of the renal pelvis
Campbell MF, Kavoussi LR, Wein AJ, es. Campbell-Walsh urology. an renal vascularity. Grae I is a subcapsular renal hema-
10th e. Elsevier Sauners;01. toma or renal contusion with no renal laceration. Grae II
is a parenchymal laceration less than 1 cm in epth with the
14. A. Blaer injury is usually cause by a pelvic fracture hematoma containe within the Gerota fascia. Grae III is a
or by blunt trauma to the lower abomen when the blaer laceration larger than 1 cm in epth into the meulla with the
is istene. Blaer injuries inclue blaer contusions hematoma containe in the Gerota fascia. Grae IV is a lacer-
(hematuria without extravasation), extraperitoneal blaer ation into the collecting system or the renal pelvis or a isrup-
rupture, an intraperitoneal blaer rupture. Gross hematu- tion of the ureteropelvic junction (seen oftentimes in chilren).
ria is the most common presenting sign of rupture an can be Injury to a segmental renal artery or vein also qualies as
accompanie by concurrent pelvic fracture an suprapubic Grae IV. A Grae V injury is a isruption of the main renal
iscomfort/tenerness. Blaer rupture is iagnose with a artery or vein or a shattere kiney. If the patient is alreay
cystogram; the blaer is lle with 300 to 400 cc of contrast unergoing a laparotomy for a penetrating abominal injury
CHAPtEr 24 Genitourinary 337

an a renal injury is note, the question arises as to whether pelvic brim, the recommene management is resection of
the renal injury shoul be explore. Similarly, if there is no the injure segment followe by a primary en-to-en uret-
large or expaning hematoma an no active bleeing, such eral anastomosis over a stent. For small injuries to the istal
an injury is now increasingly being observe without explo- thir, the recommene management is reimplantation into
ration. Exploring such wouns requires opening the Gerota the blaer. For larger injuries to the upper an mile thir,
fascia an releasing its tamponae effect, which in turn may nephropexy (anchoring the kiney caua) to bring the ure-
lea to bleeing an a nephrectomy (in other wors, esta- teral ens closer together to create a tension-free en-to-en
bilizing a stable conition). As a general rule, Graes I an ureteral anastomosis (ureteroureterostomy) is an option (D).
II rarely nee operative management. Graes III an IV can For large injuries to the istal thir of the ureter, reimplanta-
be observe if no intraperitoneal injuries are note. Delaye tion into the blaer is recommene. However, with larger
bleeing occurs in 0% of Grae III to IV renal injuries an istal ureteral injuries, the ureter may not reach, so the bla-
can be manage with arteriographic embolization. Grae V er will nee to be mobilize. This can be performe with
injuries shoul be explore in the OR. a psoas hitch maneuver in which the blaer is pulle up
References: Buckley JC, McAninch JW. Revision of current an anchore to the psoas muscle to reach the injure ureter
American Association for the Surgery of Trauma Renal Injury gra- (C). Aitionally, a Boari blaer ap can be performe in
ing system. J Trauma. 011;70(1):35–37. which the blaer is tubularize to create aitional length.
Heyns CF. Renal trauma: inications for imaging an surgical In cases where the patient is unstable, the surgeon can ligate
exploration. BJU Int. 004;93(8):1165–1170.
the ureter an place a percutaneous nephrostomy tube (B).
Santucci R, Wessells H, Bartsch G. Evaluation an management
The patient can be brought back at a later ate for repair,
of renal injuries: consensus statement of the renal trauma committee.
BJU Int. 004;93(7):937–954.
which may inclue ileal interposition (E). Absorbable sutures
shoul always be use to avoi stricturing an calculi for-
16. C. Blunt injuries to the kiney are often manage con- mation an to prevent a nius for infection. Most surgeons
servatively with observation alone, even in the presence of prefer to leave rains for ureteral injuries.
urinary extravasation on early imaging stuies. Extravasa-
tion resolves spontaneously in 85% of renal injuries with- 18. B. Blast injury can cause extensive irect an inirect
out further intervention. However, patients with persistent soft tissue amage. The initial blast can cause immeiate tis-
extravasation shoul be manage by rainage of urine sue amage, but there is oftentimes tissue injury that appears
with an internal ureteral stent. A Foley catheter may also be later. Victims suffer burns from the heat ischarge from
neee to maximally ecompress the blaer an prevent the explosive evice or the blast. Gunshot wouns resem-
urine from reuxing up the stente ureter to allow closure ble such blast injuries. The egree of injury correspons to
of the collecting system injury. Percutaneous nephrostomy the type of weapon, caliber of bullet, an istance from the
tubes are ifcult to place without hyronephrosis an pro- projectile to the victim. Bullet velocity has the greatest effect
vie no avantage over internal stents in the above case (D). on soft tissue amage. The faster the bullet, the larger the
Perinephric rainage is unnecessary without evience of temporary cavity create, inicating a greater extent of soft
infection or large urinoma formation (E). Surgical explora- tissue injury. These blast injuries ten to evolve with time
tion is excessively invasive an may result in more amage an become more wiesprea after several ays. A minor
to the kiney (B). ureteral contusion is manage with ureteral stent placement
Reference: Alsika NF, McAninch JW, Elliott SP, Garcia M. Non- to prevent ureteral narrowing from resultant scar tissue. If
operative management outcomes of isolate urinary extravasation the ureteral amage was greater, the microvascular supply to
following renal lacerations ue to external trauma. J Urol. 006;176(6 the ureter woul be compromise, leaing to ureteral break-
Pt 1):494–497. own or stricture that woul manifest ays to weeks after the
initial injury. This woul necessitate excision of the amage
17. A. Ureteral injury an repair is an important part of gen- ureteral segment an either en-to-en anastomosis of the
eral surgery training because it is a well-known complication remaining ureter (ureteroureterostomy) or reimplantation
uring pelvic issection an mobilization of the iliac arteries of the remaining ureter into the blaer (ureteroneocystos-
(ureters cross anterior to the common iliac vessel bifurca- tomy) (A, D–E). Percutaneous nephrostomy is not inicate
tion). Ureteral repair is ivie into thirs an epens on in this case because ureteral stenting is possible (C).
whether a large (> cm) or small (< cm) segment is missing. Reference: McAninch W, Santucci RA. Renal an ureteral
The upper an mile thirs of the ureter are ene as being trauma. In: Campbell MF, Wein AJ, Kavoussi LR, es.Campbell-Walsh
above the pelvic brim. If a small segment is injure above the urology. 9th e. Sauners Elsevier; 007.
Gynecology
AMANDA C. PURDY AND TAJNOOS YAZDANY 25
ABSITE 99th Percentile High-Yields
1. Ectopic Pregnancy (Associate With Pelvic Inammatory Disease)
A. Presents with lower abominal pain, vaginal bleeing, perio of amenorrhea; only 50% have all three
B. Vast majority locate in the fallopian tube (other locations: ovary, cervix, intraabominal)
C. Diagnose with beta-human chorionic gonaotropin (hCG) an transvaginal ultrasoun
1. Consier if no IUP on ultrasoun with beta-hCG >000
D. Management options:
1. Methotrexate
a) Inicators that methotrexate therapy will be successful: Mil symptoms, beta-hCG <5,000, absent
embryonic cariac activity, gestational sac <4cm
b) Absolute contrainications for methotrexate use: Hemoynamic instability, rupture ectopic
(may see peritoneal free ui), immunocompromise, breastfeeing, renal or hepatic ysfunction,
pulmonary isease, peptic ulcer isease, hematologic abnormalities
c) After methotrexate, nee to follow with serial beta-hCG ays 4 an 7; if it fails to ecline more
than 15%, then n ose is given, follow hCG until normal
. Surgery
a) Laparoscopic surgery usually sufcient; can perform salpingotomy in antimesosalpinx portion of
tube (preferre) or salpingectomy

2. Ovarian Torsion (Most Associate With Benign Ovarian Cysts)


A. Presents with acute lower abominal pain (raiating to groin), nausea, vomiting; fever if ovarian
ischemia evelops, increase risk uring pregnancy
B. Doppler ultrasoun for iagnosis: check ovarian bloo ow an evaluate for ovarian cyst/mass (most
are >5 cm); (normal ovarian bloo ow oes not rule out torsion)
C. Management: emergent laparoscopy, most can be successfully etorse; even if the ovary is “bluish-
black” an enlarge, it is most likely ue to venous congestion an not necrosis; in these cases, still
perform etorsion an not oophorectomy; if large cyst, perform cystectomy; if ovary is grossly necrotic
on exploration (rare) or malignancy is suspecte, then perform salpingo-oophorectomy

3. Pelvic Inammatory Disease (PID)


A. Inammation of the upper genital tract; usually an ascening polymicrobial infection from the vagina/
cervix; chlamyia an gonorrhea are common inciting pathogens
B. Minimum iagnostic criteria: pelvic or lower abominal pain AND tenerness with cervical motion
or over uterus or anexa; other supporting criteria: fever >38.3°C, abnormal ischarge, leukocytes on
microscopy of vaginal secretions, elevate CRP an/or ESR, conrme chlamyia an/or gonorrhea
C. Complications of PID = infertility, tubo-ovarian abscess (TOA), ectopics, Fitz-Hugh-Curtis synrome
(RUQ pain, perihepatic inammation with “violin string ahesions” an mil transaminitis)

339
AL GRAWANY
340 PArt i Patient Care

D. Management of PID: if mil-moerate isease, can treat as outpatient with antibiotics (IM ceftriaxone
×1 an oral oxycycline +/− metroniazole ×14 ays); if severe isease, pregnancy, or TOA, treat as
inpatient with IV antibiotics
E. Management of TOA: in stable premenopausal patients, start with antibiotics (most respon to antibiotics
alone); if no improvement on antibiotics in 48 hours, next step is image-guie rainage; if worsening on
antibiotics, next step is surgical rainage

4. Enometrial Cancer (Most Common Gynecologic Malignancy)


A. Risk factors: unoppose estrogen (hormone replacement, tamoxifen, obesity, polycystic ovarian
synrome), Lynch synrome
B. Can present with postmenopausal uterine bleeing or abnormal perimenopausal bleeing
C. Diagnose with enometrial biopsy
D. Staging is surgical—hysterectomy, bilateral salpingo-oophorectomy, pelvic an paraaortic lymph noe
issection, an peritoneal washings; if metastasis is foun, consier cytoreuction

5. Ovarian Cancer (Secon Most Common Gynecologic Malignancy; Dealiest Gynecologic Malignancy)
A. Risk factors: increase number of ovulations (early menarche, late menopause, nulliparity), iabetes,
BRCA tumor suppressor mutation; BRCA1 has a higher risk for ovarian cancer than BRCA2; smoking is
NOT a risk factor
B. Oral contraceptives are protective (may increase risk of breast cancer)
C. Nonspecic presentation of bloating, abominal iscomfort; usually iagnose late when isease is
avance; may have elevate CA-15; stage 1: cancer limite to one or both ovaries only (no peritoneal
or iaphragmatic metastasis)
D. Initial treatment is usually surgical; consier neoajuvant chemotherapy in patients meically unable to
unergo surgery or for very bulky isease where cytoreuction isn’t possible
E. Most common subtype is papillary serous cystaenocarcinoma, secon is mucinous
F. Staging is surgical: hysterectomy, bilateral salpingo-oophorectomy, pelvic an paraaortic lymph noe
issection, peritoneal washings, omentectomy
G. If metastasis foun uring staging, an it is possible to remove most of the isease (goal is <1cm of
resiual isease): primary cytoreuctive surgery
H. Most patients get ajuvant chemotherapy

6. Cervical Cancer
A. Screening: Pap smears starting at age 1, screen every 3 years
B. Risk factors: HPV (especially types 16 an 18), immunocompromise (HIV), an smoking
C. May present with abnormal uterine bleeing or postcoital bleeing
D. Stage CLINICALLY with physical exam (can inclue colposcopy, hysteroscopy, cystoscopy,
proctoscopy) an plain lms (chest x-ray, IV pyelogram, skeletal x-rays)
E. Most common subtype is squamous
F. For cervical ysplasia only or early-stage microscopic isease (patient esires fertility): consier local
excision only with col knife cone
G. For less avance isease (lesions <4 cm within cervix or into upper two-thirs of the vagina): raical
hysterectomy (total hysterectomy + removal of parametrium an the top portion of the vagina) an
pelvic lymph noe issection OR raiation
H. For more avance isease: cisplatin-base chemoraiation + brachytherapy
CHAPtEr 25 Gynecology 341

Questions
1. A 35-year-ol GP woman presents with 4. A 4-year-ol female presents to her obstetrician
signicant pelvic pain an yspareunia that complaining of heavy menstrual bleeing that
has been occurring for the past few years. The appears to be worsening. She is having signicant
pain is cyclical, mostly occurs a few ays before abominal cramping with her menses an is
menses, an lasts until a few ays after menses. having trouble with urinary frequency an
On bimanual exam, there are palpable noules on urgency. Which of the following is true regaring
the uterosacral ligament. Which of the following the most likely conition?
is true about her conition? A. The conition tens to improve uring
A. Transvaginal ultrasoun is the gol stanar pregnancy
for iagnosis B. MRI is most often require to conrm the
B. Diagnosis requires image-guie biopsy iagnosis
C. This conition can present with C. Most cases are associate with vaginal bleeing
pneumothorax D. This is most likely a benign conition
D. CA-15 is most commonly normal E. Uterine artery embolization is preferre in
E. Meical management improves infertility younger women

2. A 45-year-ol woman with both a personal an 5. Which of the following is true regaring anexal
family history of breast cancer ecies to unergo torsion?
BRCA mutation testing. Which of the following is A. Anexal torsion is most commonly ue to an
true? ovarian malignancy
A. Patients with either a BRCA1 or BRCA2 B. Doppler ultrasoun may emonstrate vascular
mutation shoul be offere prophylactic compromise cause by torsion
bilateral salpingo-oophorectomy (BSO) C. CT imaging is the preferre metho to conrm
B. Only those with a BRCA1 shoul be offere iagnosis
prophylactic BSO D. If the ovary is frankly necrotic, oophorectomy
C. The risk of ovarian cancer is higher with with pexy of the contralateral ovary is the
BRCA2 than BRCA1. recommene treatment
D. BRCA mutations are autosomal recessive E. The majority will etorse on their own
E. BRCA2 is an oncogene
6. Which of the following is true in regar to ovarian
3. A 38-year-ol G1P1 female presents with cancer?
abnormal vaginal bleeing. She reports having A. It is the most common malignant tumor in the
intermittent spotting throughout the month with female genital tract
some pelvic iscomfort. This has persiste for the B. It is stage similarly to cervical cancer
past several months. She enies any recent sexual C. Bilateral ovary involvement is consiere
activity. Serum beta-hCG is negative. Which of stage 4 isease
the following is the most important stuy or D. Krukenberg tumor classically emonstrates
proceure for this patient? signet ring cells
A. CT scan of the abomen/pelvis E. Oral contraceptive pills increase the risk of
B. Magnetic resonance imaging (MRI) abomen/ ovarian cancer
pelvis
C. Enometrial biopsy 7. A 3-year-ol female has ha two Pap smears
D. Transvaginal ultrasoun over the last 4 months, showing atypical
E. Pelvic examination squamous cells of unetermine signicance
(ASC-US). On subsequent cervical biopsy, she
is foun to have mil ysplasia. Which of the
following is the most appropriate treatment?
A. Pap smear in 1 year
B. Pap smear in 6 months
C. Cryoablation
D. Loop electrosurgical excision proceure (LEEP)
E. Col knife conization
342 PArt i Patient Care

8. A 8-year-ol female woul like to know if she 10. A 3-year-ol female presents to the ED 1 week
is currently pregnant. Which of the following after vaginal elivery of her rst chil. She has
combinations of imaging an lab threshol persistent right lower quarant abominal pain,
is most likely to accurately emonstrate an nausea, an leukocytosis. Pelvic examination
intrauterine gestational sac the earliest? is unremarkable. A Duplex ultrasoun
A. Transabominal ultrasoun with a serum beta- emonstrates a tubular hypoechoic structure that
hCG of 3500 mIU/mL extens superiorly from the anexa, with absence
B. Transvaginal ultrasoun with a urine beta- of ow on Doppler. Which of the following is true
hCG of 1500 mIU/mL about this conition?
C. Transvaginal ultrasoun with a serum beta- A. MRI is generally not helpful in establishing the
hCG of 000 mIU/mL iagnosis
D. Transvaginal ultrasoun with a urine beta- B. Therapeutic anticoagulation an IV antibiotics
hCG of 500 mIU/mL shoul be starte
E. Transabominal ultrasoun with a serum beta- C. Exploratory laparotomy shoul be performe
hCG of 5500 mIU/mL D. Diagnostic laparoscopy shoul be performe
E. She likely has retaine proucts of
9. A 35-year-ol woman presents to the emergency contraception
epartment (ED) complaining of abominal
pain an irregular vaginal spotting. Her last 11. A 33-year-ol female who is 18 weeks pregnant
menstruation was 8 weeks ago. On physical presents to the ED with hypotension, altere
exam, she has tenerness in her right anexa. mental status, an tachycaria. The parameics
Laboratory ata emonstrates leukocytosis of report that she was in a car accient earlier in the
18,000 cells/mL an beta-hCG of 3,000 mIU/mL. ay an that they were calle when she became
She is hemoynamically stable. Which of the altere. She has obvious vaginal bleeing, an
following is true regaring the most likely the besie nurse states that she is persistently
conition? bleeing from her peripheral IV site. Which of the
A. This is most commonly seen in women after following is true about this conition?
HPV infection A. Low brinogen levels are rare
B. Intrauterine evices (IUDs) increase one’s risk B. Transfusion of bloo proucts is the
of this conition cornerstone of management
C. Immeiate laparotomy is warrante C. This conition can be exclue in cases with
D. Immeiate laparoscopy is warrante no vaginal bleeing
E. Methotrexate can successfully treat this D. Ultrasoun is the best initial screening test
conition E. Delivery of the fetus shoul be performe

Answers
1. C. This patient has enometriosis characterize by iagnosis (A, B). Enometriosis is associate with infertility
enometrial glans an stroma foun outsie of the uter- ue to pelvic ahesions, istorte pelvic anatomy, an bilat-
ine cavity. Patients often present with chronic cyclical pelvic eral tubal blockage. Meical management oes not improve
pain an may report yspareunia, ysuria, an yschezia, infertility, but surgery may improve the spontaneous preg-
epening on where the implants are locate. Catamenial nancy rate (E). Although not sensitive or specic, CA-15 is
pneumothorax (usually on the right) occurs in temporal rela- often elevate in patients with enometriosis (D). First-line
tion to menstruation an is cause by enometrial implants therapy is meical management with NSAIDs an combine
foun in the visceral lung pleura or abnormal iaphragm oral contraceptives (C). The goal of meical management is
fenestrations. A physical exam is usually normal, but some- to improve symptoms. If patients fail meical management,
times there can be palpable noules on the uterosacral liga- surgery can be consiere. The options for surgery inclue
ment or rectovaginal septum. The iagnosis of enometriosis laparoscopic excision or ablation of the enometrial lesions
can be empirically mae if symptoms are ameliorate after a with or without hysterectomy. The aition of a hysterec-
short 3-month trial of Gonaotropin-releasing hormone ago- tomy signicantly ecreases the recurrence of symptoms an
nist therapy; however, laparoscopy is the gol stanar for the nee for reoperation. However, laparoscopic excision/
CHAPtEr 25 Gynecology 343

ablation is still effective an shoul be offere to women submucosal an intrauterine myomas (B). Conservative
unergoing surgery for enometriosis who want to preserve management inclues oral contraceptive pills, meroxypro-
their fertility. gesterone acetate, gonaotropin-releasing hormone (GnRH)
Reference: Shakiba K, Bena JF, McGill KM, Minger J, Falcone agonists, uterine artery embolization, an myomectomy.
T. Surgical treatment of enometriosis: a 7-year follow-up on the Uterine artery embolization is contrainicate in patients
requirement for further surgery. Obstet Gynecol. 008;111(6):185–19. esiring fertility (E). GnRH shoul be given for 3 months
Erratum in: Obstet Gynecol. 008 Sep;11(3):710. before surgery to reuce bloo loss an assist in normalizing
the hematocrit.
2. A. BRCA1 an BRCA2 are autosomal ominant mutations
in tumor suppressor genes that increase the carrier’s risk of 5. B. Anexal torsion occurs when the ovary an/or fallo-
cancer, especially breast an ovarian cancers (D, E). The risk pian tubes become twiste an the vascular supply becomes
of ovarian cancer is greater in patients with a BRCA1 muta- compromise. Anexal torsion is generally a isease of
tion than in those with a BRCA2 mutation (C). In one stuy, reprouctive-age women. Torsion is commonly relate to
44% of women with BRCA1 an 17% of women with BRCA2 ovarian or tubal enlargement, incluing benign neoplasms
evelope ovarian cancer by the age of 80. Although ovarian (benign cystic teratoma, paraovarian cyst, cystaenoma,
cancer is more common in patients with BRCA1 mutations, broma) an pregnancy-relate changes (corpus luteum
risk-reucing BSO shoul be offere to patients with both cyst, ovarian enlargement from ovulation inuction). It
BRCA1 an BRCA2 mutations (B). It is recommene that is rarely relate to an ovarian malignancy (A). While CT
patients consier prophylactic BSO when chilbearing is n- imaging can be use to assist with the iagnosis of anexal
ishe, by the age of 35 to 40. torsion (C), ultrasoun is the preferre metho of imaging.
Reference: Kuchenbaecker KB, Hopper JL, Barnes DR, et al. In patients with signs of ovarian necrosis intraoperatively,
Risks of breast, ovarian, an contralateral breast cancer for BRCA1
anexectomy is the treatment of choice (D), without inter-
an BRCA2 mutation carriers. JAMA. 017;317(3):40–416.
vention for the contralateral ovary. Once a iagnosis has
been conrme, the patient nees to be taken to the operat-
3. C. There is a large ifferential iagnosis in a patient
ing room immeiately to etermine viability of the anexa
with abnormal vaginal bleeing incluing intrauterine
(E). Laparoscopic etorsion can usually be performe in the
pregnancy, ectopic pregnancy, enometriosis, aenomyosis,
majority of patients.
brois, an malignancy. The American Congress of Obste-
References: Chang KH, Hwang KJ, Kwon HC, et al. Conserva-
tricians an Gynecologists (ACOG) recommens that all
tive therapy of anexal torsion employing color Doppler sonogra-
women with abnormal vaginal bleeing receive a full his- phy. J Am Assoc Gynecol Laparosc. 1998;5(1):13–17.
tory an physical examination, incluing pelvic exam, an Jung SI, Park HS, Jeon HJ, et al. CT preictors for selecting con-
bloo work incluing a pregnancy test with serum beta- servative surgery or anexectomy to treat anexal torsion. Clin Imag-
hCG (initial laboratory stuy in the workup) (A, E). This ing. 016;40(4):816–80.
shoul be followe by iagnostic imaging such as a trans- Sommerville M, Grimes DA, Koonings PP, Campbell K. Ovar-
vaginal ultrasoun (D). Aitionally, all patients younger ian neoplasms an the risk of anexal torsion. Am J Obstet Gynecol.
than 45 years of age with persistent abnormal uterine blee- 1991;164():577–578.
ing, or those with unoppose estrogen exposure, shoul
unergo enometrial biopsy to rule out enometrial cancer. 6. D. Although ovarian cancer is consiere the leaing
Aitionally, all women over 45 years of age with abnor- gynecologic cause of eath, the most common malignant
mal uterine bleeing shoul unergo enometrial biopsy. tumor in the female genital tract is enometrial cancer (A).
MRI, while potentially useful to ientify a mass, woul not A woman’s lifetime risk of being iagnose with ovarian
be neee uring the initial examination in a patient with cancer is 1.5%. Since most women with early-stage ovar-
abnormal bleeing (B). ian cancer have very few symptoms, nearly two-thirs of
Reference: Sweet MG, Schmit-Dalton TA, Weiss PM, Masen cases are iagnose in the later stages. Risk factors inclue
KP. Evaluation an management of abnormal uterine bleeing in early menarche, nulliparity, an late menopause; all of these
premenopausal women. Am Fam Physician. 01;85(1):35–43. increase the total number of ovulations in a woman’s life-
time. Oral contraceptive pills prevent ovulation an ecrease
4. D. Uterine brois, also known as uterine leiomyomas, the risk of ovarian cancer (E). They can increase the risk of
are benign smooth muscle tumors of the uterus. These most breast cancer, which can persist for about 10 years after the
commonly become symptomatic in patients between 40 an cessation of oral contraceptive pills. After this time, the risk
50 years ol, with prevalence ranging from 0% to 80%. returns to baseline. Krukenberg tumor refers to an ovarian
However, most are asymptomatic; bleeing cause by leio- tumor that has metastasize from another site, classically
myomas is the most common inication for hysterectomy in the stomach. The classic pathology associate with this is a
the Unite States (C). Malignant egeneration occurs in less signet ring cell. Women with ovarian cancer may complain
than 1% of cases an is usually encountere in the postmeno- of vague abominal pain or pressure, nausea, early satiety,
pausal years. High levels of pregnancy hormones (estrogen constipation, abominal swelling, loss of weight, urinary
an progesterone) frequently cause signicant enlargement frequency, an abnormal vaginal bleeing. Transvaginal
of preexisting myomas, which may lea to istortion of ultrasoun an CA-15 shoul be performe uring the
the uterine cavity resulting in recurrent miscarriages, intra- initial workup. However, staging is complete with surgery
uterine growth restriction, abruption, preterm labor, an (unlike cervical cancer) (B). This allows for the best evalua-
pain from egeneration (A). Diagnosis is usually mae by tion of the extent of isease an thus etermines the nee
transvaginal ultrasoun, but MRI, CT, an hysterosalpin- for ajuvant therapy. Interestingly, bilateral ovarian involve-
gography can also be performe an help to istinguish ment is still consiere stage 1 isease (C). In patients with
344 PArt i Patient Care

localize ovarian cancer (stage 1 an some cases of stage ) that repeat pap smear in 1 year along with HPV testing is
who wish to retain fertility, a unilateral oophorectomy, peri- appropriate.
toneal biopsies, an unilateral lymphaenectomy may be References: Khan MJ, Smith-McCune KK. Treatment of cervical
performe, with hysterectomy an contralateral oophorec- precancers: back to basics. Obstet Gynecol. 014;13(6):1339–1343.
tomy elaye until after completion of chilbearing. In all Massa LS, Einstein MH, Huh WK, et al. 01 upate consensus
other situations, a total abominal hysterectomy with BSO is guielines for the management of abnormal cervical cancer screen-
ing tests an cancer precursors. Obstet Gynecol. 013;11(4):89–846.
recommene. Although few ranomize clinical trials have
evaluate the concept of “ebulking surgery” to reuce the
volume of ovarian cancer to a microscopic resiual, it is gen-
8. B. Either transvaginal or transabominal ultrasoun can
be use concurrently with serum or urine beta-hCG level
erally accepte that patients with smaller volumes of tumor
to ientify an estimate the gestational age of an intrauter-
following staging laparotomy have an improve survival
ine pregnancy. Transvaginal ultrasoun is a more accurate
when cytoreuction is performe compare with patients in
way of etermining gestational age at a signicantly earlier
whom cytoreuction is unable to be performe. The goal of
point in the pregnancy. With the avent of quantitative urine
cytoreuction is to minimize the iameter of the remaining
tests, the accuracy an sensitivity of etecting pregnancy is
isease because survival is irectly proportional to the tumor
comparable to serum beta-hCG. The earliest an intrauterine
volume following cytoreuction.
gestational sac can be visualize is with a transvaginal ultra-
References: NIH Consensus Development Panel on Ovarian
Cancer. Ovarian cancer: screening, treatment, an follow-up. JAMA.
soun in a patient with a urine or serum beta-hCG greater
1995;73(6):491–497. than 1500 mIU/mL (A, C–E). In fact, nonvisualization of an
Ries LAG, Eisner MP, Kosary CL, et al., es. SEER cancer statistics intrauterine sac on transvaginal ultrasoun in a patient with
review, 1975–2001. National Cancer Institute; 004. a urine or serum beta-hCG of more than 1500 mIU/mL is
concerning for an ectopic pregnancy. The intrauterine sac
7. A. The goal of screening for cervical cancer is to circum- is visible on trans-abominal ultrasoun when the urine or
vent progression of cancer while avoiing the overtreatment serum beta-hCG is greater than 6000 mIU/ml.
of lesions that are likely to regress. This patient ha a con- Reference: Grossman D, Berichevsky K, Larrea F, Beltran
cerning pap smear result that require follow-up with col- J. Accuracy of a semi-quantitative urine pregnancy test com-
poscopy an biopsy. Prior to colposcopy, biopsy was one pare to serum beta-hCG measurement: a possible screening tool
with a large excisional proceure, such as LEEP or col knife for ongoing pregnancy after meication abortion. Contraception.
007;76():101–104.
conization. However, irecte biopsies are now possible
with colposcopy. Cervical intraepithelial neoplasia (CIN) is a
premalignant conition ranging from low to high grae that
9. E. A common an potentially life-threatening cause of
abominal pain in women is ectopic pregnancy. An ectopic
can be ientie on cervical biopsy an is groupe into three
pregnancy is ene as gestation in which implantation has
broa categories: CIN 1 is mil ysplasia, CIN  is moerate
taken place in a site other than the enometrium; 97% of
to marke ysplasia, an CIN 3 is severe ysplasia. Because
cases occur in the fallopian tubes. In a patient who is hemo-
cervical biopsies with colposcopy on’t sample the entire cer-
ynamically stable an is intereste in reprouction later in
vix, invasive cancer can’t be conrme (or exclue). Thus,
life, methotrexate can be given in a single ose with a meian
CIN is most useful to help ientify women who woul most
success rate of 84%. Methotrexate has a higher rate of fail-
benet from excisional cervical biopsies to rule out cancer or
ure if the beta-hCG level is greater than 5000 mIU/mL, if the
can simply procee with less invasive screening. CIN -3 has
intrauterine gestational sac is greater than 4 cm with no fetal
a much simpler algorithm for management compare to CIN
cariac activity, or if the intrauterine gestational sac is greater
1. Since the risk of concurrent cancer an/or progression to
than 3.5 cm with fetal cariac activity. Ectopic pregnancies
cancer is high, treatment is recommene for all women
are often seen in patients with pelvic inammatory isease
(C–E). With the avent of colposcopy, this can be performe
(PID). Women with HPV are not at risk for PID (A). Even
with ablative (cryoablation or laser ablation) or excisional
though patients with IUDs have a 5% risk of ectopic preg-
methos. Ablative options are popular among reprouc-
nancy, the overall risk is still lower than those who o not use
tive-age women as the risk of averse outcomes such as
contraception (B). Base on multiple stuies, the ata have
preterm elivery an prenatal mortality is lower than with
consistently shown laparoscopic surgery to be safer than
excisional techniques. The ecrease risk of averse events
open laparotomy (D). However, laparotomy shoul be per-
is possible because the epth of tissue estruction is lower
forme in patients with an acute abomen that are hemoy-
when compare to excisional techniques. This explains why
namically unstable (C). These patients have shorter hospital
excision offers a more accurate sample an is preferre in
stays, less bloo loss, an less use of postoperative narcotics.
oler women or younger women who are not concerne
Finally, patients that are Rh-negative will nee to receive an
with future fertility. The management of CIN 1, or mil ys-
anti-D globulin injection within 7 hours of meical or sur-
plasia, epens on the age of the patient. Women age 1 to
gical intervention.
4 are at a very low risk for cervical cancer, so these women
References: Hajenius PJ, Engelsbel S, Mol BW, et al. Ranomise
can be manage conservatively with a repeat pap smear in 1
trial of systemic methotrexate versus laparoscopic salpingostomy in
year (not 6 months) (B). If the Pap smear is concerning at that tubal pregnancy. Lancet. 1997;350(9080):774–779.
time, repeat colposcopy an biopsy shoul be performe. Murphy AA, Nager CW, Wujek JJ, Kettel LM, Torp VA, Chin
For women oler than 5 with CIN 1, the management is tai- HG. Operative laparoscopy versus laparotomy for the man-
lore base on current HPV status, results of any previous agement of ectopic pregnancy: a prospective trial. Fertil Steril.
Pap smears, an patient preference, but most woul agree 199;57(6):1180–1185.
CHAPtEr 25 Gynecology 345

10. B. This patient has ovarian vein thrombophlebitis (OVT). an excessive thrombolysis. In DIC, excessive prouction
Patients with OVT usually present with fever an abominal of thrombin leas to wiesprea intravascular brin epo-
pain within 1 week after elivery or surgery. The majority sition an brinolysis (A). This results in the epletion of
(80%) occur on the right sie. Some patients may also have coagulation factors an platelets, along with the prouction
nausea an/or ileus. Postpartum pelvic thrombophlebitis is of brin egraation proucts, causing a profoun bleeing
often precee by Virchow tria involving (1) enothelial iathesis. This can result in massive hemorrhage, thrombo-
amage with elivery, () venous stasis as a result of preg- sis, an multiorgan failure. Since DIC is consiere a con-
nancy-inuce ovarian venous ilatation an low postpar- sumptive process, bloo transfusions are not consiere the
tum ovarian venous pressures, an (3) the hypercoagulable enitive management. In fact, they may worsen symptoms.
state of pregnancy. The iagnosis is often challenging an a The best management is to treat the unerlying cause (B).
iagnosis of exclusion, but one clinical clue is often persistent DIC in pregnancy occurs in 0.03% to 0.035% of cases but oes
fever espite broa-spectrum IV antibiotics. There is no sin- not occur in isolation; most cases are initiate by a trigger. In
gle imaging moality that has proven to be most effective in pregnancy, these triggers inclue postpartum hemorrhage,
assisting with a iagnosis. Ultrasoun can be useful but may preeclampsia, HELLP synrome (hemolysis, elevate liver
be limite by bowel gas. Both CT an MRI are useful (A). The enzymes, low platelet count), acute fatty liver isease, amni-
current recommene management is antibiotic therapy in otic ui embolism, sepsis, an traumatic placental abrup-
conjunction with systemic anticoagulation (C–E). tion (as in the above patient). Although placental abruption
Reference: Nezhat C, Farhay P, Lemyre M. Septic pelvic is often accompanie by vaginal bleeing, patients with a
thrombophlebitis following laparoscopic hysterectomy. JSLS. conceale placental abruption can present with an absence of
009;13(1):84–86. vaginal bleeing (C). Since the above patient is unstable with
DIC seconary to traumatic placental abruption, the best
11. E. Disseminate intravascular coagulation (DIC) is a management is elivery of the fetus. This will often resolve
pathologic isruption of hemostasis. Massive activation of obstetric conitions initiating DIC. Ultrasoun can be a use-
the clotting cascae results in wiesprea thrombosis, which ful ajunct in equivocal cases of placental abruption (D).
leas to the epletion of platelets an coagulation factors
Head and Neck
ZACHARY N. WEITZNER AND JAMES WU 26
ABSITE 99th Percentile High-Yields
I. High Yiel Anatomy
A. Branches of external caroti artery: superior thyroi artery, ascening pharyngeal artery, lingual artery,
facial artery, occipital artery, posterior auricular artery, maxillary artery, supercial temporal artery
B. The external branch of the superior laryngeal nerve runs with the superior thyroi artery; supplies
motor innervation to the cricothyroi muscle; if injure, pitch of voice is altere
C. The recurrent laryngeal nerve runs with the inferior thyroi artery; supplies motor innervation to all
intrinsic laryngeal muscles besies the cricothyroi an moves vocal cors into an aucte position; if
unilateral injury, will have hoarseness; if bilateral injury, will have strior an compromise airway
D. Phrenic nerve runs on anterior surface of anterior scalene
E. Long thoracic nerve runs along mile scalene
F. Lymph noe levels of the neck:
1. Levels I–V: these lymph noes are remove uring a lateral neck issection
. Level VI: these lymph noes are remove uring a central neck issection
G. Lanmarks for surgical airway:
1. Cricothyroiotomy: mae in the cricothyroi membrane (inferior to the thyroi cartilage, superior to
the cricoi cartilage)
. Tracheostomy: usually between the n an 3r tracheal rings; if lower than the 4th tracheal ring—
increase risk for tracheoinnominate stula

II. Eyes, Ears, Nose


A. Temporal bone fractures: facial nerve can be injure proximally (leas to ipsilateral facial roop);
evaluate for cerebrospinal ui (CSF) leak an sensorineural hearing loss
B. Rhinorrhea: evaluate for CSF leak (test for tau protein an beta--transferrin)
C. Epistaxis: anterior can be packe, posterior is more likely to require intervention—embolization of
internal maxillary or ethmoi artery
D. Lacerations:
1. Eyeli: orbicularis oculi close in separate layer
. Lip: vermillion borer must be approximate
3. Auricle: hematomas raine to prevent cauliower ear; cartilage must be covere
4. Facial nerve: if anterior to vertical line from lateral canthus, nerve oes not nee to be repaire; if
posterior, nerve shoul be repaire

III. Oropharynx
A. Squamous cell carcinoma (SCC)
1. Risk factors: HPV 16 an 18, Plummer-Vinson synrome (glossitis, iron eciency anemia,
esophageal web); HPV-relate hea an neck masses are most commonly foun in the oropharynx
B. Benign isease (always start with airway, breathing, circulation)

347
348 PArt i Patient Care

1. Luwig angina: parapharyngeal abscess or infection of oor of mouth which can lea to meiastinitis
an airway compromise; treat with rainage of lateral neck
. Peritonsillar abscess: untreate pharyngitis causing ysphagia, trismus, ysphagia, uvular eviation;
treat with rainage
3. Retropharyngeal abscess: fever, oynophagia, pooling secretions; secure airway emergently then
rain

IV. Nasopharynx
A. Malignancies linke to Epstein-Barr virus (EBV), Asian ethnicities
B. Benign isease
1. Nasopharyngeal angiobroma: vascular tumor causing epistaxis in young patients, treat with
embolization an resection

V. Salivary Glans
A. Major salivary glans: paroti, submanibular, sublingual
B. Benign tumors in orer of frequency: pleomorphic aenoma, monomorphic aenoma, Warthin tumor,
oncocytoma
1. Treat with excision, supercial parotiectomy with facial nerve preservation
. Warthin can present bilaterally; it is also known as papillary cystaenoma lymphomatosum
C. Malignant tumors: mucoepiermoi carcinoma (most common), aenoi cystic carcinoma
1. Treat by local excision, may nee total parotiectomy if invaes eep lobe
D. Sialaenitis an parotitis: seen in elerly an ehyrate patients; ue to obstruction of salivary glan;
most common bacteria Staphylococcus aureus; treat with IVF, lozenges, massage, an antibiotics; shoul
rule out cancer; may nee to incise uct an remove stone

VI. Neck
A. Thyroglossal uct cyst
1. Extens from resiual foramen cecum at tongue base to lower anterior neck: miline mass that rises
with swallow
. Treatment is Sistrunk proceure (excision of the cyst, tract, an central hyoi bone) to reuce risk of
infection an malignancy
B. Branchial cleft cysts
1. First: associate with external auitory canal an paroti
. Secon: superior anterior borer of sternocleiomastoi muscle between internal caroti artery an
external caroti artery to tonsils
3. Thir: mile anterior borer of sternocleiomastoi muscle posterior to common caroti artery to
pyriform sinus
C. Raical neck issection: removal of noes in levels I to V, sternocleiomastoi muscle, internal jugular
vein, CN XI
D. Moie raical neck issection: removal of noes with preservation of one or more of the
sternocleiomastoi muscle, internal jugular vein, CN XI
E. When performing a neck issection, raise platysmal aps more than 1.5 cm below the inferior borer of
the manible to avoi injury to the marginal manibular nerve

VII. Workup of New Neck Mass


A. Differential: SCC, melanoma, thyroi cancer, lymphoma, brachial cleft cyst
B. Evaluation:
1. History, exam of skin an scalp, oral cavity, exible laryngoscopy
. Neck ultrasoun
3. Fine-neele aspiration
4. If SCC: Direct laryngoscopy uner anesthesia, esophagoscopy, bronchoscopy (panenoscopy)
5. Possible to have SCC in neck noe with unknown primary, manage with moie raical neck
issection
CHAPtEr 26 Head and Neck 349

Fig. 26.1 Neck Dissection.

Fig. 26.2 Relationship Between Neck Nerves are Vasculature.

AL GRAWANY
350 PArt i Patient Care

Questions
1. A 4-year-ol boy presents to clinic with 5. A 44-year-ol male with recurrent melanoma
progressive neck swelling over the past few of the posterior scalp an cervical aenopathy
weeks. He has felt otherwise well without arrives at clinic to iscuss the risks of cervical
fevers, weight loss, or ecrease playfulness. lymph noe issection. The nerve most likely to
A -cm mass is palpable beneath the angle of be injure uring this proceure is the:
the manible without uctuance. The overlying A. Spinal accessory nerve
skin is violaceous but without rainage. He B. Long thoracic nerve
has no cranial neuropathy. Fine neele aspirate C. Lesser occipital nerve
emonstrates aci-fast bacteria. What is the best D. Transverse cervical nerve
treatment for this conition? E. Phrenic nerve
A. A 10-ay course of amoxicillin
B. Incision an rainage 6. Which of the following is true regaring
C. Lymphaenectomy nasopharyngeal carcinoma?
D. Azithromycin an rifampin until symptom A. It is not associate with alcohol
resolution B. Most patients present with cervical lymph
E. Chemotherapy noe metastasis
C. The stanar of care involves surgical excision
2. Which of the following is true regaring cleft lip followe by chemoraiation
an cleft palate? D. Plummer-Vinson synrome increases the risk
A. The majority of cases of cleft lip an cleft for its evelopment
palate are ue to congenital synromes E. It is commonly confuse with otitis externa
B. Repair of cleft palate shoul be elaye until
approximately 1 months of age 7. Which of the following is true regaring
C. For cleft palate, echocariography for cariac epistaxis?
abnormalities is unnecessary A. The vast majority of blees are from the
D. Repair of cleft lip shoul be performe within posterior part of the nose
a week of birth in full-term infants B. Posterior blees most commonly arise from
E. Tube feeing is usually necessary for cleft the sphenopalatine artery
palate to ensure preoperative growth an C. Anterior blees have a signicant mortality
evelopment risk
D. Posterior blees are best manage by applying
3. A 6-year-ol male with recurrent otitis meia igital pressure to the nose
presents to the ED with fever an right-sie E. Anterior blees often require packing
earache. Methylene blue conrms a sinus tract combine with a Foley catheter
from the right submanibular area to the external
auitory canal. Which of the following branchial 8. Which of the following statements is true
cleft cysts oes this patient most likely have? regaring paroti glan tumors?
A. First A. The majority are malignant
B. Secon B. Pleomorphic aenoma (benign mixe tumor)
C. Thir is the most common type
D. Fourth C. Pleomorphic aenomas are manage by total
E. Fifth parotiectomy
D. For malignant tumors resection of the facial
4. The brachial plexus is locate: nerve is usually require
A. Posterior to the mile scalene muscle E. For benign tumors, the most commonly
B. Anterior to the mile scalene muscle injure nerve uring resection is the facial
C. Anterior to the anterior scalene muscle nerve
D. Posterior to the posterior scalene muscle
E. Anterior to the posterior scalene muscle
CHAPtEr 26 Head and Neck 351

9. A 65-year-ol male presents with a persistent rm 13. A -year-ol male presents with a well-ene
lateral neck mass that measures approximately anterior neck mass, locate miline an above
.5 cm. Careful history an physical examination the cricoi cartilage. The mother reports no other
of the hea an neck are negative. The next step meical history. It elevates when he swallows
in the management is: an is nontener. He has no cervical aenopathy.
A. Positron emission tomography Which of the following is recommene before
B. Compute tomography scan of the hea an consiering surgical excision?
neck A. Compute tomography (CT) scan of the neck
C. Fine-neele aspiration of the neck mass B. Thyroi scintigraphy
D. Chest raiograph C. Fine-neele biopsy
E. Panenoscopy (esophagouoenoscopy, D. Magnetic resonance imaging (MRI) of the neck
bronchoscopy, laryngoscopy) E. Ultrasoun

10. A 1-week-ol male infant with trisomy 1 14. A 45-year-ol male with squamous cell carcinoma
presents with a large posterolateral neck mass at the oor of the mouth is recovering from a
extening into the axilla that transilluminates. resection, a manibular ap reconstruction, an
The mass has been growing continuously for the a tracheostomy performe at the thir tracheal
past several weeks. Optimal management woul ring. Several hours later, the surgical resient
consist of: gets calle to the postoperative recovery suite
A. Raiation therapy because the patient evelops some bleeing at the
B. Repeat neele aspirations tracheostomy site. Which of the following is true?
C. Raical wie excision A. Making the tracheostomy at the secon tracheal
D. Observation ring coul have prevente this complication
E. Conservative excision B. He shoul be taken to the operating room
(OR) to unergo a meian sternotomy
11. A 54-year-ol man presents with a tener left C. He likely has a traumatic injury of the anterior
neck mass with a raining sinus. Microscopic jugular vein
examination reveals sulfur granules. Optimal D. Immeiate bronchoscopy shoul be performe
management woul be: E. Overinating the tracheostomy cuff shoul be
A. Penicillin avoie
B. Raical excision
C. Penicillin an surgical rainage 15. A 15-year-ol male arrives at the emergency
D. Trimethoprim-sulfamethoxazole epartment (ED) with recurrent right-sie
E. Trimethoprim-sulfamethoxazole an surgical epistaxis an nasal obstruction. Vital signs
rainage are normal. Nasal enoscopy reveals a
esh-appearing mass in the right nares. His
12. A 50-year-ol male presents with a right-sie hemoglobin is 1 g/L. MRI emonstrates a mass
slow-growing roune neck mass locate in the pterygopalatine fossa with anterior bowing
anterior to the sternocleiomastoi. The mass of the posterior maxillary wall. Treatment consists
appears to move sie to sie only. CT of the neck of:
is performe an emonstrates wiening of the A. Placing nasal packing an ischarging home
caroti bifurcation by a well-ene tumor blush. B. Intraoperative biopsy of mass
The mass is 3 cm. Optimal management consists C. Aministering utamie
of: D. Raiation therapy
A. Raiographic embolization E. Enoscopic surgical excision of the mass
B. Raiation therapy
C. Chemotherapy 16. The most common cause of hearing loss in an
D. Surgical excision ault is:
E. Raiographic embolization followe by A. Acute otitis meia
surgical excision B. Chronic otitis meia
C. Otosclerosis
D. Cerumen
E. Presbycusis
352 PArt i Patient Care

17. An elerly patient being treate with 19. Which of the following is true regaring
chemotherapy for metastatic colon cancer carcinoma of the lip?
presents with swelling of the cheek an pain. A. Upper lip carcinoma is more common
WBC is normal an patient is afebrile. Initial B. The majority present with noal metastasis
treatment for this conition consists of: C. Squamous cell carcinoma is the most common
A. Besie incision an rainage type of cancer in the lower lip
B. Paroti massage, lozenges, an hyration D. Raiation therapy is the treatment of choice
C. Supercial parotiectomy for most lip cancers
D. IV antibiotics E. Prophylactic neck issection is usually inicate
E. Enoscopic uct exploration
20. Which of the following is true regaring salivary
18. The most likely site of origin for a metachronous glan tumors?
cancer in a patient with a history of laryngeal A. Paroti tumors are more likely to be malignant
cancer is the: than submanibular glan tumors
A. Esophagus B. Submanibular glan tumors are more likely to
B. Lung be malignant than minor salivary glan tumors
C. Floor of mouth C. Pleomorphic aenomas may unergo
D. Tongue malignant egeneration
E. Hypopharynx D. Warthin tumors are malignant
E. Facial nerve palsy is common in benign tumors

Answers
1. C. This boy presents with likely nontuberculous myco- such, cleft lip may present either unilaterally or bilaterally,
bacterial lymphaenitis (scrofula), an iagnosis can be con- an rarely in the miline. Cleft palate results from failure of
rme with FNA staine for aci-fast bacteria. This conition fusion of bilateral palatal shelves an thus always occurs in
typically affects healthy chilren uner the age of 5 years ol the miline. The majority of cases of cleft lip an cleft palate
an is most frequently ue to Mycobacterium avium. Scrof- are nonsynromic, but many congenital synromes may be
ula typically presents as a nonpainful mass with signicant associate with cleft lip an cleft palate, such as Treacher-
overlying skin iscoloration that progresses to stulization Collins synrome, DiGeorge synrome, an Pierre Robin
an sinus tracts. The treatment for uncomplicate isolate sequence (A). Because of this, evaluation of newborns with
mycobacterial lymphaenitis is lymphaenectomy without cleft lip an cleft palate shoul inclue assessment for con-
antimicrobials. In the event the patient is not a caniate for comitant cariovascular, skeletal, an neurologic abnormal-
lymphaenectomy, which can be ue to parent preference ities (C). Postnatally, the management of cleft lip an cleft
or neurovascular involvement, a course of macrolie plus palate revolves aroun airway management an feeing
rifampin or ethambutol can be chosen (D). Unfortunately, optimization to ensure optimal growth an evelopment.
antimicrobial therapy alone frequently leas to prolonge This can usually be accomplishe with frequent oral fee-
course of illness with increase likelihoo of isguring ings (E). Surgical repair of cleft lip may occur earlier than
complications compare to lymphaenectomy. Incision an cleft palate, as early repair of cleft palate may result in mi-
rainage is the treatment of choice for simple abscesses, an face hypoplasia. Cleft lip repair optimally occurs between 
incising mycobacterial lymphaenitis leas to stulization an 6 months of age, while cleft palate repair occurs between
(B). Antimicrobials combine with lymphaenectomy may 9 an 18 months of age (D).
be require in the setting of bacterial superinfection (A). References: Lewis CW, Jacob LS, Lehmann CU, SECTION ON
Chemotherapy is reserve for the treatment of lymphoma, ORAL HEALTH. The primary care peiatrician an the care of chil-
which typically presents as a neck mass without overlying ren with cleft lip an/or cleft palate. Pediatrics. 017;139(5):e017068.
skin changes associate with B cell symptoms such as fevers, Cockell A, Lees M. Prenatal iagnosis an management of orofa-
cial clefts. Prenat Diagn. 000;0():149–51.
weight loss, an fatigue (E).
3. A. There are only four branchial cleft cysts (E). The above
2. B. Cleft lip an cleft palate are common congenital abnor- patient has a rst branchial cleft cyst presenting with recur-
malities seen in newborns. Variants inclue isolate cleft lip, rent infection. The accompanying sinus tract typically tra-
cleft lip with cleft palate, an isolate cleft palate. Develop- verses from the submanibular area to the external auitory
mentally, cleft lip is the result of the failure of fusion of the canal, an it is a result of incomplete closure of the ectoerm
lateral, meian, an maxillary mesoermal processes. As uring evelopment. Denitive intervention involves a
CHAPtEr 26 Head and Neck 353

supercial parotiectomy. The most common branchial cleft In a ranomize stuy, the 3-year survival rate was 46% for
cyst is a secon branchial cleft cyst, which appears anterior patients ranomize to raiation therapy an 76% for the
to the sternocleiomastoi muscle an can also present with chemotherapy an raiation therapy group. Surgery is gen-
recurrent infections (B). Thir branchial cleft cysts are rare erally not inicate (C).
but most commonly appear on the left sie near the lateral References: Al-Sarraf M, LeBlanc M, Giri PG, et al. Chemora-
neck (C). Fourth branchial cleft cysts also appear on the lat- iotherapy versus raiotherapy in patients with avance nasopha-
eral neck an can lea to neck swelling an airway compro- ryngeal cancer: phase III ranomize Intergroup stuy 0099. J Clin
mise (D). Oncol. 1998;16(4):1310–1317.
Chen L, Gallicchio L, Boy-Linsley K, et al. Alcohol consump-
References: Pincus RL. Congenital neck masses an cysts. In:
tion an the risk of nasopharyngeal carcinoma: a systematic review.
Bailey BJ, e. Head and neck surgery—otolaryngology. 3r e. Lippin-
Nutr Cancer. 009;61(1):1–15.
cott Williams an Wilkins; 001.
Tomita N, Fuwa N, Ariji Y, Koaira T, Mizoguchi N. Factors asso-
Zhong Z, Zhao E, Liu Y, Liu P, Wang Q, Xiao S. Management an
ciate with noal metastasis in nasopharyngeal cancer: an approach
classication of rst branchial cleft anomalies. Lin Chuang Er Bi Yan
to reuce the raiation el in selecte patients. Br J Radiol.
Hou Tou Jing Wai Ke Za Zhi. 013;7(13):691–694.
011;84(999):65–70.
4. B. The subclavian vein, artery, an brachial plexus are all
part of the posterior neck triangle, an their relative relation 7. B. It is important to recognize that epistaxis has the poten-
to the scalene muscles is important to appreciate uring neck tial to be life threatening. Epistaxis has anterior an posterior
an upper extremity issection. Aitionally, the pathway sources. Anterior epistaxis is most common (A) an is cause
that each of these structures takes in the neck, upper thorax, by trauma in most cases, which causes rupture of supercial
an upper extremity helps in unerstaning the pathophys- mucosal vessels (Kiesselbach plexus). Most anterior blees
iology of thoracic outlet synrome (TOS). The most com- stop with simple irect pressure (E) an are not consiere
mon type of TOS is neurogenic, presenting with sensory to be angerous (C). If this fails, then anterior packing is per-
an motor loss in the ulnar nerve istribution. The brachial forme. Posterior blees are more angerous an potentially
plexus an subclavian artery pass posterior to the anterior life threatening. Bleeing is most commonly from a branch
scalene muscle but anterior to the mile scalene muscle (A, of the sphenopalatine artery, the terminal branch of the inter-
C–E). The subclavian vein passes anterior to the anterior sca- nal maxillary artery. It is associate with hypertension an
lene muscle an can evelop an area of narrowing between atherosclerosis. Direct pressure cannot tamponae posterior
the rst rib an clavicle. blees. Treatment involves posterior packing (D). Posterior
packing has the potential to compromise the airway an
5. A. Although all the nerves liste are at risk uring a cervi- cause hypoventilation; therefore, patients nee to be amit-
cal lymph noe issection, the most commonly injure nerve te to a monitore setting. Part of the mortality risk associ-
uring cervical issection is the spinal accessory nerve also ate with posterior blees can be attribute to the patient
known as cranial nerve eleven (CN XI) (B–E). The supercial population that is frequently affecte—the elerly with sig-
course of this nerve at the posterior neck triangle makes it nicant unerlying isease.
particularly susceptible to injury. It travels through the ster-
nocleiomastoi muscle. It can lea to trapezius palsy pre- 8. B. Most salivary glan tumors are in the paroti glan,
senting with shouler weakness an pain. The phrenic nerve an approximately 80% of paroti glan tumors are benign
travels anterior to the anterior scalene muscle an passes (A). Submanibular an sublingual glan tumors are
posterior to the subclavian vein before entering the chest. approximately 50% malignant, an minor salivary glan
References: Lima LP e, Amar A, Lehn CN. Spinal accessory tumors are preominantly malignant. The largest salivary
nerve neuropathy following neck issection. Braz J Otorhinolaryngol. glan is the paroti glan. The most common type of paroti
011;77():59–6. glan tumor is a pleomorphic aenoma (also calle a benign
Wiater JM, Bigliani LU. Spinal accessory nerve injury. Clin Orthop mixe tumor). Bilateral lesions are extremely rare (0.%
Relat Res. 1999;368(368):5–16. of all paroti glan tumors). The most commonly injure
nerve in paroti surgery is the greater auricular nerve (E).
6. B. Nasopharyngeal carcinoma is associate with Epstein- The treatment of choice for benign paroti tumors is a
Barr virus. In fact, Epstein-Barr virus titers can be use to supercial parotiectomy (C). For malignant tumors, every
follow the response to treatment. Nasopharyngeal carcinoma effort shoul be mae to preserve the facial nerve if it is not
is enemic in certain areas of southern China. Previously, invae by the tumor (D).
alcohol was not thought to increase the risk for nasopharyn- Reference: Huang JT, Li W, Chen XQ, Shi RH, Zhao YF. Synchro-
geal carcinoma, but a recent systemic review suggests heavy nous bilateral pleomorphic aenomas of the paroti glan: Bilateral
alcohol use may have a contributing role (A). Plummer-Vin- pleomorphic aenomas. J Investig Clin Dent. 01;3(3):5–7.
son synrome has not been shown to be associate with
nasopharyngeal carcinoma (D). Nasopharyngeal carcinoma 9. C. In aults, the most likely etiology of a persistent neck
often presents with a mile ear effusion an can initially mass larger than  cm is cancer. Most often the cancer is from
be confuse with otitis meia (E). The majority of patients the hea an neck an is squamous cell carcinoma. Careful
(up to 90%) have cervical lymph noe metastasis on presen- physical examination is essential. If the physical examina-
tation. Whites born in the Unite States have a lower risk tion is unremarkable, the next step is to establish whether
of eveloping nasopharyngeal carcinoma, whereas whites the mass is malignant. This is best achieve by ne-neele
born in China have an increase risk. Several stuies have aspiration. Once metastatic cancer is conrme, panenos-
emonstrate that a combination of chemotherapy an rai- copy with guie biopsies is performe in the OR uner
ation yiels a higher survival rate than either moality alone. general anesthesia to locate the primary mass (E). CT scan of
354 PArt i Patient Care

the hea an neck an chest raiograph are also performe in an effort to excise completely, the issection is carrie too
to assist in locating the mass (B, D). If the primary mass is close to the artery. Because of their vascular nature, biopsy
still not localize, the role of positron emission tomography is contrainicate. Routine preoperative embolization is not
is ebatable (A). Several stuies have shown that it has a necessary but shoul be consiere in large tumors (>4 cm)
low sensitivity an oes not alter outcome. If the mass is not (A, E). Raiation therapy may be consiere for long-term
localize after panenoscopy, an excisional biopsy shoul be tumor control in patients that are not caniates for surgery
performe. Aenocarcinoma woul suggest a primary lung, (e.g., inaccessible site) (B). Chemotherapy has no role in the
breast or gastrointestinal tumor. management of these tumors (C). Excising the caroti bifur-
References: Grau C, Johansen LV, Jakobsen J, Geertsen P, Aner- cation shoul be avoie (E).
sen E, Jensen BB. Cervical lymph noe metastases from unknown References: Daviovic LB, Djukic VB, Vasic DM, Sinjelic RP,
primary tumours. Results from a national survey by the Danish Soci- Duvnjak SN. Diagnosis an treatment of caroti boy paragangli-
ety for Hea an Neck Oncology. Radiother Oncol. 000;55():11–19. oma: 1 years of experience at a clinical center of Serbia. World J Surg
Kole AC, Nieweg OE, Pruim J, et al. Detection of unknown Oncol. 005;3(1):10.
occult primary tumors using positron emission tomography. Cancer. Hinerman RW, Amur RJ, Morris CG, Kirwan J, Menenhall
1998;8(6):1160–1166. McGuirt WF. The neck mass. Med Clin North WM. Denitive raiotherapy in the management of paraganglio-
Am. 1999;83(1):19–34. mas arising in the hea an neck: a 35-year experience. Head Neck.
008;30(11):1431–1438.
10. E. The presentation is consistent with a cystic hygroma
(CH) given the age of the patient, the location of the mass, an 13. E. This patient has a thyroglossal uct cyst, a remnant
the fact that it transilluminates. CH occurs more commonly in of thyroi glan escent an the most common miline con-
patients with trisomy 1 an Turner synrome. CH is a lym- genital malformation of the neck. Though present at birth,
phatic malformation. Most present in the posterior neck, an these o not often appear until age  as baby fat recees. It
the next most common site is the axilla. More than half present presents as an anterior miline cystic mass that moves with
at birth, an the remainer become apparent within the rst swallowing or sticking out the tongue. Denitive manage-
 years of life as baby fat recees. On occasion, intralesional ment involves surgical intervention. The operation, known
bleeing can cause the mass to grow signicantly in a short as the Sistrunk proceure, removes the cyst, tract, an cen-
amount of time. Complete surgical excision is preferre; how- tral portion of the hyoi bone, as well as a portion of the
ever, if the mass is ajacent to nerves, it is best manage with tongue base up to the foramen cecum. However, given the
a conservative excision (C). Raiation has no role in the man- increase association of an ectopic thyroi glan in patients
agement of CH (A). Although repeate neele aspirations (B) with a thyroglossal uct cyst, preoperative imaging nees
may shrink the mass, it will only be a temporary intervention. to be performe to conrm the correct anatomic location
Observing the mass is an appropriate consieration for patients of the thyroi glan. This will help avoi excising an ecto-
that are asymptomatic (e.g., the mass is not growing) (D). pic thyroi glan inavertently uring the Sistrunk pro-
ceure. Ultrasonography is the preferre option since it is
11. C. Actinomyces israelii an other Actinomyces species noninvasive, wiely available, an cost effective. Thyroi
occur in the normal ora of the mouth an tonsillar crypts. scintigraphy is equally as effective but is use less often (B).
They are anaerobic, gram-positive, branching lamentous Aitionally, ultrasoun has several avantages over scin-
bacteria. They o not stain aci-fast positive (unlike M. tuber- tigraphy, incluing the absence of ionizing raiation, an it
culosis or Actinomycetes). The face an neck are the most has the ability to characterize the thyroglossal uct cyst with
common sites of infection an usually evelop after minor high elity. MRI or CT scan is not require for the iagno-
trauma or tooth extraction. Actinomyces infections generally sis an shoul not be performe in young patients (A, D).
occur in association with other bacteria. The infection tens to Fine-neele aspiration (FNA) biopsy is appropriate for a sus-
form abscesses that later rain. Microscopic examination may pecte thyroi noule (C). Serial exams/observation woul
reveal the classic appearance of sulfur granules, which are not be appropriate because these cysts have an increase risk
masses of lamentous organisms. Optimal treatment is with of recurrent infections an malignant transformation.
penicillin an surgical rainage, not antibiotics alone (A). Sur-
gical excision can be consiere for complicate cases (e.g., 14. C. Bleeing from aroun the tracheostomy site coul
brotic lesions, extensive abscesses) (B). However, it is rarely have ire consequences an shoul be evaluate quickly.
successful without concurrent antibiotic therapy. Although The lag time between tracheostomy creation an hemor-
Nocaria is also an anaerobic, gram-positive, branching l- rhage helps narrow own the possible etiology. Hemorrhage
amentous bacteria, it is consiere a weakly aci-fast organ- within the rst 48 hours is more likely to be seconary to
ism. It is treate with trimethoprim-sulfamethoxazole (D, E). local trauma such as injury to the inferior thyroi artery or
anterior jugular veins. Aitionally, this patient has likely
12. D. Wiening of the caroti bifurcation by a well-e- receive heparin since he ha a ap reconstruction per-
ne tumor blush (lyre sign) on CT is consiere a pathog- forme. Systemic coagulopathy coul also contribute to con-
nomonic ning for a caroti boy tumor. Patients typically tinue bleeing in the immeiate postoperative perio. The
present in the fourth or fth ecae with a slow-growing rst line of management involves applying irect pressure,
roune neck mass. It is usually locate anterior to the ster- which can be performe by overinating the tracheostomy
nocleiomastoi near the angle of the manible. Caroti cuff (E). If this oes not control bleeing an the patient
boy tumors can only be move from sie to sie, not up continues to have stable vital signs, a bronchoscopy can be
or own, because of their location within the caroti sheath consiere (D). However, if there is any concern for mas-
(Fontaine sign). Treatment of caroti boy tumors is surgi- sive hemorrhage or airway compromise, the patient shoul
cal. One angerous pitfall in excising these tumors is that, be immeiately returne to the OR for neck exploration.
CHAPtEr 26 Head and Neck 355

Tracheoinnominate stula (TIF) is a rare an fatal compli- always be remove because it serves as a protective layer for
cation that requires, at a minimum, 48 hours to evelop. It the skin of the ear canal an helps protect against infection.
often presents with a heral blee that will progress to mas- Patients that present with hearing loss, earache, or fullness
sive exsanguination. Performing a tracheostomy above the shoul have cerumen remove. Otitis meia is more likely
thir tracheal ring will help ecrease the risk of eveloping to result in hearing loss in chilren (A, B). Presbycusis is a
this complication (A). If TIF is suspecte, placing one’s n- sensorineural hearing loss an affects oler patients (E).
ger through the tracheostomy with igital pressure applie References: Isaacson JE, Vora NM. Differential iagnosis an
between the TIF an the posterior surface of the sternum can treatment of hearing loss. Am Fam Physician. 003;68(6):115–113.
control bleeing until the patient is taken to the OR to have a Rolan PS, Smith TL, Schwartz SR, et al. Clinical practice guie-
meian sternotomy an stula ligation performe (B). line: cerumen impaction. Otolaryngol Head Neck Surg. 008;139(3
Suppl ):S1–S1.
References: Grant CA, Dempsey G, Harrison J, Jones T.
Tracheo-innominate artery stula after percutaneous trache-
ostomy: three case reports an a clinical review. Br J Anaesth. 17. B. This patient presents with uncomplicate parotitis.
006;96(1):17–131. Initial treatment consists of paroti massage, sialagogues,
Muhamma JK, Major E, Woo A, Patton DW. Percutaneous il- an IV hyration (B). Parotitis is frequently seen in elerly
atational tracheostomy: haemorrhagic complications an the vascu- patients with poor oral intake an ehyration. Occlusion of
lar anatomy of the anterior neck. A review base on 497 cases. Int J Stensen uct by a stone can lea to bacterial infection, most
Oral Maxillofac Surg. 000;9(3):17–. frequently with S. aureus. However, with normal vital signs
an white bloo cell levels, it is unlikely this patient has sup-
15. E. A young aolescent male presenting with severe purative parotitis at this time. Initial treatment for suppura-
unilateral epistaxis an a esh-appearing nasal mass has tive parotitis is IV antibiotics (D), an if treatment fails or
juvenile nasal angiobroma until proven otherwise. This patient emonstrates signs of sepsis, incision an rainage,
is a highly vascular benign neoplasm arising from aroun enoscopic uct exploration, or even parotiectomy may be
the pterygopalatine fossa. Patients may report history of require (A, C, E). The most common glan to evelop sia-
recurrent epistaxis, nasal obstruction, an/or ischarge. If lolithiasis is the submanibular glan (prouces over 90%
there is any concern about airway compromise ue to mas- of stones).
sive bleeing, the patient shoul be intubate. If the patient Reference: Pfaff J, Moore GP. Otolaryngology. In: Marx JA,
has symptomatic bloo loss, he shoul be transfuse with Rosen P, es. Rosen’s emergency medicine: concepts and clinical practice.
bloo proucts. The next step is to conrm the iagnosis Vol. II, 5th e. Mosby; 00:55–88.
with MRI or CT scan an look for extension of the broma
into the sinuses. Biopsy of the mass is avoie because it 18. B. Patients with hea an neck cancers have an approx-
can lea to life-threatening hemorrhage (B). Nasal packing imately 14% risk of eveloping a secon primary tumor.
shoul be use initially to help stop bleeing. However, the Most of these are metachronous (beyon 6 months). For
patient shoul be amitte an observe (A). Aitionally, laryngeal cancer patients, the most common metachronous
nasal packing for a prolonge perio of time can lea to toxic malignancy is lung cancer (C–E). For patients with oral cav-
shock synrome seconary to Staphylococcus aureus, an ity an pharyngeal cancers, the most common metachronous
as such, a patient ischarge with nasal packing that is to cancer is esophageal (A).
remain in place for a prolonge perio of time shoul also
be given oral antibiotics. The testosterone receptor blocker 19. C. Ninety percent to 95% of lip cancers occur in the
utamie has been reporte to shrink small tumors but is lower lip (A). Sun exposure an tobacco use are the most
not the stanar recommenation (C). If bleeing contin- important risk factors. Lip cancers occur most often in
ues, the patient will nee to be taken to the angiography elerly white men. They are most often ue to squamous cell
suite for embolization of the internal maxillary artery. The carcinoma. Upper lip cancers are usually basal cell carcino-
enitive intervention is surgical excision, which can now mas. The most common presentation is an ulcerative lesion
be performe with a transnasal enoscopic approach (E). on the vermilion or skin surface (B). Early-stage lesions can
Coagulation stuies woul be inicate. Raiation therapy be treate with surgery or raiation therapy, but surgical
use to be a treatment option, but it is no longer performe, resection is preferre an is the treatment of choice for larger
particularly in aolescents (D). lesions (D, E).
References: English GM, Hemenway WG, Cuny RL. Surgical
treatment of invasive angiobroma. Arch Otolaryngol Head Neck Surg.
20. C. Salivary glan neoplasms are rare. Most arise in the
197;96(4):31–318.
paroti glan. The ratio of malignant to benign tumors var-
Gullane PJ, Davison J, O’Dwyer T, Forte V. Juvenile angio-
broma: a review of the literature an a case series report. Laryngo- ies by site. Paroti glan tumors are 80% benign an 0%
scope. 199;10(8):98–933. malignant, submanibular glan an sublingual glan
Nicolai P, Schreiber A, Bolzoni Villaret A. Juvenile angiobroma: tumors are 50% benign an 50% malignant, an minor sali-
evolution of management. Int J Pediatr. 01;01:41545. vary glan tumors are 5% benign an 75% malignant (A, B).
Warthin tumor is the secon most common benign salivary
16. D. Hearing loss can be ivie into two categories tumor an is strongly relate to smoking (D). Facial nerve
incluing conuctive an sensorineural loss. Conuctive involvement is highly suggestive of a malignant tumor (E).
hearing loss occurs more commonly with cerumen (earwax) Although benign, pleomorphic aenomas have a known risk
being the major contributor. Otosclerosis can also lea to con- of malignant transformation that becomes as high as 10% to
uctive hearing loss (C). The majority of patients are asymp- 5% when present beyon 15 years. Fine-neele aspiration is
tomatic, an contrary to popular belief, cerumen shoul not useful in the iagnosis.
Nervous System
ERIC O. YEATES AND RICHARD EVERSON 27
ABSITE 99th Percentile High-Yields
I. Traumatic Brain Injury (TBI)
A. Glasgow Coma Scale (GCS): useful for classifying injury severity an prognostication
1. GCS ≤ 8 is severe TBI (consier intubation), GCS 9 to 1 is moerate TBI, GCS ≥ 13 is mil TBI
B. Treatment goals (prevent seconary insults to the brain)
1. ICP goal < 0 mmHg; CPP = MAP-ICP, CPP goal ≥ 60 mmHg (aults), CPP goal ≥40 mmHg
(peiatrics); consier pressors after appropriate volume resuscitation to achieve CPP goal
. Temperature 36.0 to 37.0°C
3. PaO 80 to 10 mmHg, PaCO 35 to 40 mmHg
4. Soium 145 to 155, hemoglobin > 7, platelets ≥ 75, INR ≤ 1.4, glucose 80 to 180
C. ICP monitoring
1. Inications: GCS < 8 with structural amage on CT, GCS < 8 with normal CT an  of the following:
age > 40-years, systolic bloo pressure < 90 mmHg, abnormal motor posturing
. External ventricular rain (EVD) is preferre as it is iagnostic an therapeutic
D. Approach to management of elevate ICP
1. In orer of intervention to be attempte: hea of be to 30 egrees, seation, hypertonic saline or mannitol
(contrainicate if systemic hypotension), short-term mil hyperventilation (PaCO2 30–35 mmHg),
ventricular rainage, barbiturates, paralysis, an ecompressive craniectomy
. Hypertonic saline an/or mannitol shoul both be given as boluses an not continuous infusions as
they will equilibrate an thus become ineffective; the goal of these interventions is to create an acute
osmotic isequilibrium, which can only be achieve with a bolus
E. Nutrition: start enteric feeing within 4 to 48 hours, postpyloric preferre
F. Venous thromboembolism (VTE) prophylaxis: very high risk of VTE in TBI
1. Brain Trauma Founation guielines (016) leave the timing an choice of agent to the clinician’s
jugment; however, most start low-molecular weight heparin (LMWH) 48 hours after the last stable CT
G. Anticoagulation reversal agents

Drug Reversal Half-life (hrs)


Warfarin Prothrombin complex concentrate 20–60
(preferred), vitamin-K
Dabigatran Idarucizumab 10–20
Edoxaban Andexanet alpfa 10–15
Rivaroxaban Andexanet alpfa 5–10
Apixaban Andexanet alpfa 10–12

357
358 PArt i Patient Care

II. Spinal Cor Injuries

Syndrome Epidemiology Aěected spinal Clinical Prognosis


tracts presentation
Central cord Most common incomplete Bilateral central Motor deęcits in upper Good prognosis
spinal cord injury corticospinal extremities more than although full
syndrome, commonly and lateral lower extremities functional recovery
in elderly with cervical spinothalamic is rare
spondylosis and spinal tracts
stenosis, hyperextension
injury
Anterior Infarction of anterior Corticospinal and Motor loss, pain, and Worst prognosis
cord spinal artery or trauma spinothalamic temperature loss of incomplete
(e.g., penetrating tracts (proprioception syndrome; low
trauma, burst fracture of and vibratory sense chance (10%–20%)
vertebra, Ěexion injury) preserved) of motor recovery
Posterior Very rare, caused by Posterior columns Loss of proprioception, Recovery variable
cord infarction of posterior light touch, vibratory and related to
spinal artery, trauma sense (motor, pain, completeness of
(e.g., penetrating and temperature lesion
trauma), multiple sensation preserved)
sclerosis
Brown- Most commonly due to Hemisection of the Ipsilateral motor and Best prognosis for
Séquard penetrating injury cord proprioception loss, functional motor
contralateral pain and activity recovery
temperature loss (99% ambulate)

Questions
1. Which of the following is true regaring the 2. Which of the following is true regaring gunshot
management of severe traumatic brain injury wouns to the hea?
(TBI) in aults? A. Suicie attempts have the same mortality rate
A. A CT scan is require prior to placement of an as assaults or accients
intracranial monitoring evice B. The incience of vascular injury is low
B. External ventricular rains (EVD) are C. Extene antibiotic prophylaxis is
preferre over intraparenchymal intracranial recommene
pressure monitors if both are available D. Bihemispheric injuries are a signicant risk
C. The goal cerebral perfusion pressure (CPP) is factor for mortality
greater than 40 mmHg E. GCS on arrival is not a signicant preictor of
D. Decompressive craniectomy oes not lower mortality
mortality in cases of refractory intracranial
hypertension as compare to meical 3. Which of the following is true regaring primary
management brain tumors?
E. Heparin is the preferre agent for VTE A. Meulloblastomas are the most common
chemoprophylaxis malignant tumors in aults
B. Aults with glioblastoma have a 5-year
survival rate of aroun 30%
C. Corticosterois are use for symptomatic
peritumoral vasogenic eema
D. Brain tumors in infants typically present with
focal neurologic ecits
E. In chilren over the age of 10, infratentorial
tumors are more common than supratentorial
CHAPtEr 27 Nervous System 359

4. A 6-year-ol intubate male is opening his eyes 7. Which of the following is true regaring Cushing
to voice an attempts to open his mouth. His only tria?
consistent motor movement is to occasionally A. The pulse pressure narrows
withraw from painful stimuli. What is his B. The heart rate increases
current GCS score? C. It oes not lea to changes on
A. GCS 4T electrocariogram
B. GCS 8T D. It is associate with hypocarbia
C. GCS 9T E. It is a late manifestation of increase
D. GCS 11T intracranial pressure
E. GCS 13T
8. Which of the following is true regaring rupture
5. An 88-year-ol female is brought by ambulance intracranial aneurysms?
to the ED after being struck by a vehicle while A. Following repair, ui restriction is
crossing the street. She is only responsive to recommene
painful stimuli an is promptly intubate for B. Most arise from the posterior circulation
airway protection. Her seconary exam reveals C. The initial stuy of choice is a contrast-
only a small abrasion to the left forehea. Her enhance hea CT
systolic bloo pressure suenly increases to D. Following repair, the risk of cerebral
the 00s, an her left pupil becomes ilate an vasospasm causing stroke persists for 3 weeks
unresponsive to light. What is the next best course E. Outcomes are overall quite favorable
of action?
A. Hypertension control with nicaripine 9. An 85-year-ol female presents to the ED after
continuous infusion falling an striking her chin on the kitchen
B. Placement of intraparenchymal intracranial counter. She is unable to lift her arms or hans off
pressure monitor the be an oes not respon to painful stimuli.
C. Immeiate mannitol bolus However, she is able to wiggle her toes an
D. Rectal lorazepam an initiation of seems to feel pain at her feet. She has a history
levetiracetam of cervical raiculopathy. A igital rectal exam
E. Raise hea of be reveals goo sphincter tone an squeeze pressure.
What is the most likely incomplete spinal cor
6. A 17-year-ol boy presents to the ED via injury that she has sustaine?
ambulance after new-onset seizure activity that A. Posterior cor synrome
starte 30 minutes ago. He is unable to provie a B. Anterior cor synrome
goo history because of wor ning issues but C. Caua equina synrome
is able to convey that his hea hurts. His parents D. Brown-Séquar synrome
state that he felt completely normal until about E. Central cor synrome
4 weeks ago when he began to complain of left
ear pain. Vital signs reveal a mil tachycaria 10. Which of the following is true regaring hea
an high fever. Physical exam shows absent light trauma an/or intracranial hemorrhage?
reex in the left eye an papilleema. Which of A. The most common cause of subarachnoi
the following is contrainicate in the workup hemorrhage is rupture of a berry aneurysm
an subsequent treatment of his conition? B. Epiural hematoma is typically associate
A. Lumbar puncture with acceleration-eceleration injuries
B. Compute tomography with intravenous C. A single episoe of systolic bloo pressure
contrast (BP) less than 90 mmHg oubles the mortality
C. Stereotactic neele aspiration rate in patients with hea trauma
D. Surgical ebriement D. Xanthochromia is virtually pathognomonic for
E. Corticosterois acute subural hemorrhage
E. In the absence of other nings, reimaging for
cerebral contusion is generally unnecessary
360 PArt i Patient Care

11. A 5-year-ol male is being evaluate in the 13. Neurogenic thoracic outlet synrome most
emergency epartment (ED) after sustaining a commonly affects which nerve?
blow to the hea with an unknown object uring A. Raial
an assault. He has a 6 cm, stellate laceration B. Ulnar
with an unerlying scalp hematoma. Compute C. Meian
tomography (CT) scan shows evience of a skull D. Musculocutaneous
fracture. In which of the following situations can E. Axillary
this patient be manage nonoperatively?
A. Fracture penetrates ura but not brain 14. A 4-ay-ol female infant weighing 1400 g born
B. 0.5 cm of skull epression at 8 weeks’ gestation is being monitore in the
C. Involvement of the frontal sinus only neonatal critical care unit because of multiple
D. Pneumocephalus episoes of apnea an ifculty with feeing.
E. Gross woun contamination Supplemental oxygen has been sufcient to
maintain saturations. Over the last several
12. A 45-year-ol female arrives at the ED after iving hours, she has ha waxing an waning alertness
hea-rst into a half-empty swimming pool. She an ecrease spontaneous eye movements.
is combative an appears intoxicate. She is not Her fontanelle appears to be full. Which of the
able to move her lower extremities or trunk. You following is the most appropriate next step?
observe her lifting her arms an bening at the A. Immeiate aministration of furosemie an
elbows but are unable to assess any movement in acetazolamie
her hans. It has been 30minutes since she rst B. Besie intracranial ultrasoun
sustaine her injury. Which of the following is C. Lumbar puncture
true regaring this patient? D. Noncontrast CT of hea
A. The likely site of her injury is C3-C4 E. Aminister IV steroi bolus
B. In the absence of other injuries,
methylprenisolone shoul be aministere
immeiately
C. This is a rare spinal cor injury after a iving
accient
D. Anticoagulation shoul be starte within  to
3 ays an continue for  to 3 months
E. Mean arterial pressure shoul be maintaine
between 65 an 75 mmHg for the rst 7 ays

Answers
1. B. One of the rst ecision points in managing a All efforts shoul be mae to maintain an aequate CPP
patient with severe TBI is the placement of an intracra- with techniques incluing seation, ventricular rainage,
nial pressure (ICP) monitor. ICP monitors are inicate in mannitol, hypertonic saline, an paralytics. If intracranial
patients with a CT scan showing intracranial hemorrhage hypertension persists espite these measures, ecompres-
an who have a GCS of less than 8 (or higher than 8 but sive craniectomy is often utilize, though there is still some
with a high risk of progression). Aitionally, ICP mon- controversy regaring its outcomes. In a ranomize con-
itors are also inicate in patients with a low GCS who trolle trial in 016, ecompressive craniectomy for refrac-
are having emergent extracranial surgery (A). A CT scan tory intracranial hypertension resulte in lower mortality
is not neee in this scenario. Though EVDs an intrapa- compare to meical treatment alone (D). Although Brain
renchymal pressure monitors can both be use to measure Trauma Founation guielines leave the choice of VTE
ICP, EVDs are preferre as they are both iagnostic an chemoprophylaxis to the clinician’s jugement, a national
therapeutic (B). Once an ICP monitor is place, CPP can be atabase stuy incluing over 10,000 patients emon-
calculate with CPP = mean arterial pressure (MAP)−ICP. strate LMWH to be associate with reuce mortality
The goal CPP is greater than 60 mmHg in aults (C). How- an thromboembolic complications, regarless of timing
ever, the goal CPP is >40 mmHg for peiatrics patients. of prophylaxis initiation in severe TBI patients (E).
CHAPtEr 27 Nervous System 361

References: ACS Trauma Quality Improvement Program. Best have cranial nerve palsies or cerebellar ysfunction. The
Practices in the Management of Traumatic Brain Injury. American Col- caveat to this rule is in infants (who will not noticeably is-
lege of Surgeons, Committee on Trauma; January 015. https:// play these ecits) who more commonly present with mac-
www.facs.org/-/meia/files/quality-programs/trauma/tqip/ rocephaly, irritability, failure to thrive, loss of evelopmental
tbi_guielines.ashx.
milestones, an vomiting (D).
Kolias PJ, Timofeev AG, IS, et al. Trial of ecompressive cra-
References: Lapointe S, Perry A, Butowski NA. Primary brain
niectomy for traumatic intracranial hypertension. N Engl J Med.
tumours in aults. Lancet. 018;39(10145):43–446.
016;375(1):1119–1130.
Uaka YT, Packer RJ. Peiatric brain tumors. Neurol Clin.
Benjamin E, Recinos G, Aiol A, Inaba K, Demetriaes D. Phar-
018;36(3):533–556.
macological thromboembolic prophylaxis in traumatic brain inju-
ries: low molecular weight heparin is superior to unfractionate
heparin. Ann Surg. 017;66(3):463–469. 4. B. The Glasgow Coma Scale uses the combine scores
from the motor, verbal, an speech sections to give an esti-
2. D. Gunshot wouns to the hea have a high morbiity mate of a patient’s level of functional status. The scoring is as
an mortality. In a large meta-analysis, factors preictive of follows. For eye opening: 4: Spontaneously, 3: To verbal com-
mortality inclue age greater than 40 years, GCS less than man, : To pain, 1: No response. Best motor response scores:
9 on arrival, xe an ilate pupils, ural penetration, 6: Obeys comman, 5: Localizes pain, 4: Flexion withrawal,
bihemispheric injuries, multilobar injuries, tranventricular 3: Flexion abnormal (ecorticate), : Extension (ecerebrate),
injuries, an suicie attempts (A, D, E). In fact, suicies ha 1: No response, an for Best verbal response: 5: Oriente an
a six times higher rate of mortality compare to assaults converses, 4: Disoriente an converses, 3: Inappropriate
or accients. Another interesting ning in this stuy was wors; cries, : Incomprehensible souns, 1: No response. If
that vascular injuries were very common (38%–50%) with the patient is intubate, the maximum score that he or she
intracranial aneurysm, arterial issection, arterial occlu- can get in the verbal category is 1T (the letter T inicating
sion, an arteriovenous stulas being the most common intubate) an maximum overall score of 11T. This patient
types in escening orer of incience (B). There is a lack opens his eyes to voice commans but not spontaneously,
of high-quality evience regaring the management of this which correlates with an eye score of 3. The best calculate
type of injury. Though surgery is associate with lower mor- motor score is a 4 for withrawing from pain. This places his
tality, it is unclear whether this is a result of surgery itself or total GCS at 1T (verbal) + 3 (eye opening) + 4 (motor) = 8T.
ue to patient selection. The rate of CNS infection after pen-
etrating TBI is less than 10% an there is no reuction in the 5. C. Without a CT scan, one cannot be sure of the exact eti-
risk of infection with prophylactic antibiotics (C). However, ology of these neurologic nings, but, base on the history
surgical intervention an ICP monitoring appear to be risk an physical exam nings, this likely represents a close
factors for infection, regarless of prophylactic use. hea injury with an elevate intracranial pressure (ICP). A
References: Maragkos GA, Papavassiliou E, Stippler M, Filip- “blown” pupil in the setting of hea trauma is consistent
piis AS. Civilian gunshot wouns to the hea: prognostic factors with uncal herniation, which is often fatal an will cause
affecting mortality: meta-analysis of 1774 patients. J Neurotrauma. permanent neurologic ecits if not treate promptly. Sys-
018;35():605–614. tolic bloo pressure greater than 180 mmHg can aggravate
Harmon LA, Haase DJ, Kufera JA, et al. Infection after penetrat- vasogenic brain eema an intracranial hypertension. How-
ing brain injury-An Eastern Association for the Surgery of Trauma ever, systemic hypertension may be a physiologic response
multicenter stuy oral presentation at the 3n annual meeting of to reuce cerebral perfusion. Thus, early an aggressive
the Eastern Association for the Surgery of Trauma, January 15–19, treatment of hypertension shoul be avoie until ICP mon-
019, in Austin, Texas. J Trauma Acute Care Surg. 019;87(1):61–67.
itoring has been establishe (A). While this patient likely
nees an ICP monitor, a iagnosis still nees to be mae
3. C. The types an presentations of brain tumors are signi-
before surgical treatment or invasive monitoring (B). Ai-
cantly ifferent in chilren an aults. In aults, the major-
tionally, an external ventricular rain is a better choice in
ity of tumors are benign, with meningiomas being the most
this patient because it allows therapeutic rainage of cere-
common. The most common malignant tumor is glioblas-
brospinal ui. Current inications for a mannitol bolus are
toma, which carries a 5-year survival rate of 5% (A, B). The
for situations just like the above—a quick bailout maneuver
management is typically focuse on maximal resection an
to be use as a brige to more enitive therapies. Mannitol
is sometimes followe by raiation. Other consierations are
immeiately improves cerebral perfusion ue to the fact that
seizure management an corticosteroi use for symptomatic
it ecreases bloo viscosity an therefore increases cerebral
peritumoral vasogenic eema (C). In chilren, brain tumors
bloo ow an cerebral oxygen elivery. Its osmotic prop-
are relatively more common an are the most common cause
erties take 15 to 30 minutes to work. There is some evience
of eath among chilhoo cancers. In chilren up to 14 years
that prolonge or scheule use will rener it ineffective at
ol the most common brain tumor is a glioma, but pituitary
best an potentially harmful. Immeiately following man-
tumors are the most common in chilren 15 years an oler.
nitol, the patient nees a CT scan an shoul be evaluate
The most common malignant brain tumor in chilren is a
for possible surgical rainage of an intracranial hematoma.
meulloblastoma. The location of brain tumors in chilren
Lorazepam an levetiracetam (Keppra) are both meications
also varies by age, with chilren age 4 to 10 years ol being
use for the treatment of seizures, which is not consistent
more likely to have infratentorial tumors. All other ages are
with her exam at this time (D). Raising the hea of the be
more likely to have supratentorial tumors (E). Supratentorial
can lower ICP, but with a blown pupil, the patient nees
tumors ten to present with focal neurologic ecits epen-
more aggressive treatment (E).
ing on the exact location, an infratentorial tumors ten to
362 PArt i Patient Care

Reference: Brain Trauma Founation, American Association of cerebral autoregulation is compromise, these patients
Neurological Surgeons, Congress of Neurological Surgeons. Guie- shoul be given volume to maintain aequate cerebral per-
lines for the management of severe traumatic brain injury. J Neu- fusion pressure (A).
rotrauma. 007;4 Suppl 1:S91–S95. Reference: Keey A. An overview of intracranial aneurysms.
McGill J Med. 006;9():141–146.
6. A. The tria of heaache, focal neurologic ecits, an
fevers shoul raise concern for brain abscess; however, this
9. E. Central cor synrome is the most common type of
classic presentation is present in less than half of all patients.
incomplete spinal cor injury an is primarily foun in
The most common presenting symptom is a heaache, which
patients that suffere a hyperextension injury in the setting
is present in approximately 70% of patients. They arise pri-
of previous cervical spine abnormalities. Symptoms inclue
marily by two forms of sprea: hematogenously from istant
muscle weakness of the upper extremities with relative spar-
sites an irect sprea from contiguous sites of infection (oti-
ing of the lower extremities. Sensory function is variable.
tis meia being most common). This leas to a wie array of
Posterior cor synrome is a relatively rare entity typically
potential pathogens, though the most common are Strepto-
cause by infarction of the posterior spinal artery. Classic
coccus spp. an Staphylococcus spp. Initial iagnosis shoul
presentation inclues sparing of muscles with the loss of
be obtaine by CT scan with contrast, which will show a
proprioception an vibration sensation below the level of the
rim-enhancing collection (B). Lumbar puncture is generally
lesion with preservation of most motor function (A). Ante-
not iagnostic an contrainicate in the setting of elevate
rior cor synrome can be cause by either infarction of the
ICP. Changes in cerebrospinal ui volume in this setting
anterior spinal artery or, less frequently, by fracture or isloca-
can precipitate herniation. All patients shoul be starte on
tion of vertebrae. It is characterize by loss of motor function,
broa-spectrum antibiotics, which can be tailore once cul-
pain sensation, an temperature sensation but preservation
tures are obtaine. Total uration of treatment is typically 4
of touch an proprioception (B). Caua equina synrome can
to 6 weeks. Traitional management inclue surgical rain-
be cause by trauma, mass lesions, or lumbar spinal stenosis
age an excision of the abscess cavity (D). However, serial
an occurs at the level that the spinal cor has split into nerve
neele aspiration has now become the treatment of choice
roots. Symptoms can be variable but generally inclue pares-
unless the abscess is traumatic in origin (potentially has for-
thesia of the perineum, anus, an external genitalia (“sale
eign ebris), fungal, multiloculate, or oes not improve
anesthesia”), bilateral or unilateral paralysis, an inconti-
with neele aspiration (C). Corticosterois are controversial
nence of bowel an blaer (C). Brown-Séquar synrome
in this setting but may be consiere when there is substan-
is hemisection of the spinal cor from a mass lesion or more
tial mass effect from the abscess (E).
commonly trauma. It causes an ipsilateral loss of motor, pro-
References: Brouwer MC, Coutinho JM, van e Beek D. Clinical
prioception, an vibration sensation with contralateral loss of
characteristics an outcome of brain abscess: systematic review an
meta-analysis. Neurology. 014;8(9):806–813.
pain an temperature sensation (D).
Muzumar D, Jhawar S, Goel A. Brain abscess: an overview. Int J
Surg. 011;9():136–144. 10. C. Traumatic brain injuries are among the most com-
mon presenting symptoms in emergency epartments in
7. E. Cushing tria is a vasomotor an respiratory response the Unite States, with over 1.7 million amissions each
to an elevate ICP that inclues braycaria, irregular year. The early recognition an management of brain injury
breathing, an elevation in systolic bloo pressure with is critical in this patient population because it is consiere
a wiene pulse pressure (A). The increase ICP leas to the most common cause of trauma-relate eath in patients
impaire respiration, which worsens hypercarbia (D). Typ- reaching the hospital alive. Preventing seconary injury is an
ically, Cushing tria is a late sign of elevate ICP an sug- important part of management, an this involves maintain-
gests imminent herniation. In aition to braycaria on ing cerebral perfusion pressure greater than 60 mmHg. One
ECG, Mayer waves can be seen with elevate ICP (B). The prospective trial foun that a single episoe of hypotension
waves are cyclic changes in arterial bloo pressure brought with a systolic bloo pressure of less than 90 mmHg ouble
about by oscillations in baroreceptor an chemoreceptor mortality in patients with brain injury. Trauma is consiere
reex control systems an are note on ECG (C). the most common etiology of subarachnoi hemorrhage, fol-
lowe by rupture of berry aneurysms (A). In nontraumatic
8. D. Intracranial aneurysms affect 4% of the population cases, patients may report mil “sentinel” heaaches in the
but are asymptomatic in the majority of cases, an most prior weeks leaing up to a severe, unrelenting, “thuner-
patients are unaware of the iagnosis. Risk factors inclue clap” heaache. Noncontrast compute tomography (CT)
female gener, polycystic kiney isease, an Marfan syn- scan is the iagnostic tool of choice to look for hyperensi-
rome. The majority of the aneurysms occur in the circle ties suggestive of acute bleeing. Aitionally, xanthochro-
of Willis with the anterior communicating artery being the mia of cerebrospinal ui is consiere pathognomonic for
most frequent site (B). When the aneurysm ruptures, it can subarachnoi hemorrhage (D). Epiural hematoma is gener-
result in intraparenchymal an subarachnoi hemorrhage, ally the result of irect trauma to the skull causing isruption
which is a catastrophic event with a mortality rate up to of arterial vessels, particularly the mile meningeal artery.
50% (E). Noncontrast CT hea is the stuy of choice to con- It initially presents with unconsciousness from the concus-
rm the iagnosis (C). Bleeing on brain parenchyma elic- sive effects of the injury, followe by a “luci” interval that
its a vasospasm response, which can result in stroke an progresses to somnolence, lethargy, an eventually a coma
patients are at increase risk for 1 ays; thus, most neu- as the hematoma grows. Noncontrast CT scan will emon-
rosurgeons will start calcium channel blockers. Because strate a lentiform (biconvex), hyperense clot that oes not
CHAPtEr 27 Nervous System 363

cross suture lines. Acute subural hematoma is generally References: Bailes JE, Herman JM, Quigley MR, et al. Diving
the result of acceleration-eceleration injuries that tear the injuries of the cervical spine. Surg Neurol. 1990;34(3):155–158.
briging veins as the brain shifts in relation to the ura (B). Theoore N, et al. Guielines for the management of acute
Patients are often unconscious from the moment of impact. cervical spine an spinal cor injuries: 013 upate. Neurosurgery.
013;7():1–59.
Noncontrast CT scan will emonstrate a hyperense, lunar
(crescent-shape) lesion that oes not cross the miline.
13. B. Neurologic symptoms occur in 95% of cases of tho-
Cerebral contusion is ue to the brain irectly striking the
racic outlet synrome. The lower  nerve roots of the bra-
skull in either a coup or countercoup mechanism after a
chial plexus, C8 an T1, are most commonly (90%) involve,
close hea injury. Lesions on noncontrast CT scans are typ-
proucing pain an paresthesias in the ulnar nerve istribu-
ically scattere, hyperense, an intraparenchymal, though
tion (A, C–E). The secon most common anatomic pattern
they can also present as hypoense lesions. There is a sig-
involves the upper three nerve roots of the brachial plexus,
nicant propensity for these lesions to worsen, an repeat
C5, C6, an C7, with symptoms referre to the neck, ear,
imaging is typically recommene in the rst 4 hours (E).
upper chest, upper back, an outer arm in the raial nerve
References: Chesnut RM, Marshall LF, Klauber MR, et al. The
role of seconary brain injury in etermining outcome from severe
istribution.
hea injury. J Trauma. 1993;34():16–.
Faul M, Xu L, Wal MM, Coronao VG. Traumatic brain injury in 14. B. Intraventricular hemorrhage (IVH) occurs in approx-
the United States: emergency department visits, hospitalizations and deaths imately 15% to 0% of infants born with a birth weight of less
2002–2006. Centers for Disease Control an Prevention, National than 1500 g. Because of the frequency of this conition, serial
Center for Injury Prevention an Control; 010. https://www.cc. ultrasoun screening is recommene for all premature
gov/traumaticbraininjury/pf/blue_book.pf infants an any infants that show signs of IVH. In premature
infants, the relative fragility of the germinal matrix makes
11. B. Any skull fracture with an overlying laceration is them sensitive to changes in cerebral bloo ow with subse-
consiere an open fracture. Traitional teaching is that all of quent hemorrhage into the ventricles. Preisposing factors in
these patients shoul be taken to the operating room to prevent aition to prematurity inclue maternal chorioamnionitis
infection. However, there seems to be a subset of patients that or preeclampsia, an neonatal respiratory istress, hypoten-
can be treate expectantly without signicant increases in mor- sion, or anemia. While 5% to 50% of infants can have clini-
biity. Nonoperative management of open skull fracture can be cally silent IVH, symptoms range from nonspecic changes
consiere in patients without evience of ural penetration, in alertness to stupor or coma. Once it has been iagnose,
signicant intracranial hematoma, frontal sinus involvement, management is largely supportive to prevent long-term com-
woun infection, pneumocephalus, or gross woun contami- plications such as posthemorrhagic hyrocephalus (PHH).
nation (A, C–E). Aitionally, patients with less than 1 cm of Prior to the avent of intracranial ultrasoun, CT scan was
skull epression can be manage nonoperatively. utilize to make the iagnosis, but has now been largely
Reference: Bullock MR, Chesnut R, Ghajar J, et al. Surgical abanone (D). Once the iagnosis is establishe, treatment
management of epresse cranial fractures. Neurosurgery. 006;58 is supportive, incluing correction of anemia (patients can
(3 Suppl):S56–60. suffer major bleeing), hypotension, aciosis, an ventila-
tory support. Treatments to try to prevent hyrocephalus
12. D. Although it is ifcult to ascertain the exact level have been largely ineffective. Though furosemie an acet-
of spine injury in a noncooperative patient, complete paral-
azolamie have been use in oler chilren with PHH, they
ysis of the lower extremities an the trunk with preserva-
o not seem to alter the course in premature infants an
tion of her shoulers an elbows most likely inicates an
coul potentially be eleterious (A). Serial lumbar puncture
injury at C5 or below (A). The most common spinal injury
has been trie with no signicant change in eterioration or
after a iving accient is C5 followe by C6 (C). The use of
progression to permanent ventricular rainage proceures
sterois in spinal cor injury has been controversial. How-
(C, E). Temporary ventricular rainage with transition to
ever, recent level 1 evience recommens against the use of
permanent rainage proceures if necessary is currently the
sterois in the management of acute spinal cor injury (B).
treatment of choice for PHH with elevate intracranial pres-
Among trauma victims, patients with spinal cor injury an
sures. Ultimately, if signicant hyrocephalus persists, the
hea injury have the highest risk of venous thromboembolic
infant may nee a ventriculoperitoneal shunt.
events (VTEs). Without prophylaxis, the risk of VTE is about
References: Mazzola CA, Chouhri AF, Auguste KI, et al. Pei-
40% after complete spinal cor injury. Mechanical prophy- atric hyrocephalus: systematic literature review an evience-base
laxis with compression evices shoul be starte immei- guielines. Part : management of posthemorrhagic hyrocephalus
ately. Anticoagulation shoul be starte within 7 hours an in premature infants. J Neurosurg Pediatr. 014;14 Suppl 1:8–3.
continue for  to 3 months. Low-molecular-weight heparin Robinson S. Neonatal posthemorrhagic hyrocephalus from pre-
is preferre over heparin. Mean arterial pressure shoul be maturity: pathophysiology an current treatment concepts: a review.
maintaine between 85 an 90 mmHg for the rst 7 ays (E). J Neurosurg Pediatr. 01;9(3):4–58.
PART II MEDICAL KNOWLEDGE

Anesthesia
ERIC O. YEATES AND CATHERINE M. KUZA 28
ABSITE 99th Percentile High-Yields
I. American Society of Anesthesiologists Physical Status (ASA PS)

ASA PS Deęnition Examples


I Normal healthy patient Young with no comorbidities
II Mild systemic disease Well-controlled hypertension, current smoker
III Severe systemic disease Poorly controlled hypertension, morbid obesity,
stable angina, prior myocardial infarction,
controlled congestive heart failure with no
symptoms, end-stage renal disease on scheduled
dialysis
IV Severe systemic disease that is a Unstable angina, congestive heart failure with
constant threat to life symptoms, sepsis, end-stage renal disease not on
scheduled dialysis
V Moribund patient not expected to Ischemic bowel, intracranial hemorrhage with
survive without the operation midline shift, ruptured abdominal aortic aneurysm
VI Brain-dead patient undergoing Traumatic brain-injured patient with no brain stem
organ donation reĚexes

II. Opioi Equivalents

Opioid Oral Parenteral


Morphine 30 mg 10 mg
Hydromorphone 7.5 mg 1.5 mg
Hydrocodone 30 mg N/A
Fentanyl N/A 0.1 mg
Codeine 200 mg N/A
Oxycodone 20 mg N/A
Tramadol 120 mg N/A

365
366 PArt ii Medical Knowledge

III. Local Anesthetic Maximum Dose

Medication Without epinephrine With epinephrine


Lidocaine 5 mg/kg 7 mg/kg
Bupivacaine/Ropivacaine 2.5 mg/kg 3 mg/kg

0.5% = 5 mg/ml, 1% = 10 mg/ml, 2% = 20 mg/mg

IV. Common Intravenous Inuction Agents

Medication Mechanism of action Side eěects


Propofol GABA agonist Hypotension, propofol infusion syndrome
Etomidate Mechanism unclear, modulates or Adrenal suppression, myoclonus
activates GABA
Ketamine NMDA receptor antagonist Emergence delirium, hypertension,
arrhythmias, increased intracranial pressure
Midazolam GABA agonist Nausea, vomiting, delirium

V. Neuromuscular Blocking Drugs for Rapi Sequence Intubation

Mechanism of Onset Histamine Reversal


Medication action (min) release Metabolism Side eěects agent
Succinylcholine Depolarizing 1–1.5 Yes Plasma Hyperkalemia, N/A
cholinesterase malignant
hyperthermia,
bradycardia
Rocuronium Nondepolarizing 1.5–3 Yes Hepatic, Allergy Sugammadex
metabolites
excreted
renally
Vecuronium Nondepolarizing 3–4 No Hepatic Allergy Sugammadex
Cisatracurium/ Nondepolarizing 5–7 No Hofmann Bronchospasm Neostigmine
atracurium elimination

VI. Steroi Potency: hyrocortisone < prenisone < methylprenisolone < examethasone

VII. Malignant Hyperthermia: rare, severe reaction to meications use uring general anesthesia
A. Genetics: rare, autosomal ominant isorer cause by a mutation in the ryanoine receptor, locate on
the sarcoplasmic reticulum (in skeletal muscle)
B. Triggering meications: volatile anesthetics (halothane, sevourane, esurane, isourane, enurane) or
epolarizing muscle relaxants (succinylcholine, ecamethonium)
C. Signs/symptoms: can occur immeiately an as late as 4 hours postoperatively
1. Hyperthermia, tachycaria, increase en-tial CO, muscle rigiity, rhabomyolysis, lactic aciosis
D. Diagnosis: acutely, the iagnosis is clinical
1. Conrmatory testing or testing of close relatives who have suffere from malignant hyperthermia;
this inclues a skeletal muscle biopsy followe by a caffeine-halothane contracture test (CHCT);
the muscle is expose to halothane an caffeine with a positive test causing signicant muscle
contraction; testing must take place in centers specialize in iagnosing malignant hyperthermia
E. Treatment: stop all anesthetics, aminister .5 mg/kg of IV antrolene which inhibits calcium ion release
from the sarcoplasmic reticulum (can aminister aitional 1–.5 mg/kg boluses, max cumulative ose
of 10 mg/kg), cooling, correction of hyperkalemia, an ui resuscitation.
F. Outcomes: mortality approximately 5%
CHAPtEr 28 Anesthesia 367

VIII. Propofol Infusion Synrome: rare synrome triggere by high ose (>4 mg/kg/hr) infusion >48 hours
A. Mechanism: unknown, but possibly ue to the impairment of fatty aci metabolism
B. Risk factors: chilren, concomitant catecholamine or steroi infusion, severe critical illness
C. Signs/symptoms: metabolic aciosis, arrhythmias (most often braycaria), rhabomyolysis,
hyperlipiemia, hepatomegaly (not splenomegaly), renal failure, cariovascular collapse
D. Treatment: immeiate cessation of propofol, early hemoialysis, supportive care
E. Screening tool: aily CPK an lactate levels

IX. Differential Diagnosis of Intraoperative Changes in En-Tial CO

Rising end-tidal CO 2 Dropping end-tidal CO 2


Hypoventilation Hyperventilation
Rebreathing Circuit disconnected
Malignant hyperthermia Inadvertent extubation
Sepsis Endotracheal tube obstruction
Hyperthyroidism (thyroid storm) Hypothermia
Pheochromocytoma Pulmonary embolism
Rhabdomyolysis Cardiac arrest
Reduced cardiac output
Tension pneumothorax
368 PArt ii Medical Knowledge

QUESTIONS
1. A 35-year-ol man involve in a motorcycle 4. Which of the following is most likely associate
cycle collision sustains a large laceration to his with opioi abuse an postsurgical prescribing
right thigh. The ecision is mae to washout an patterns?
close the woun at besie with the assistance A. The majority of opiois abuse in the US
of proceural seation. The patient has an oral originate from international rug cartels
airway in place, is breathing spontaneously, an B. Heroin users rarely report previously abusing
is maintaining aequate oxygen saturation with prescription opiois
a simple face mask. With painful stimulation, C. 30 pills of 5 mg oxycoone are the
he awakens briey an is able to follow simple recommene amount to be prescribe after
commans. What level of seation is this patient laparoscopic cholecystectomy
currently uner? D. New persistent opioi use after surgery
A. Minimal seation is more common after major proceures
B. Moerate seation compare to minor proceures
C. Conscious seation E. Preoperative tobacco use is a signicant risk
D. Deep seation factor for new persistent opioi use after
E. General anesthesia surgery

2. Which of the following is associate with opioi 5. A 75-year-ol woman is brought to the operating
tolerance? room for laparoscopic cholecystectomy. She has
A. Characterize by pronounce cravings an a history of progressive ementia an is unable
compulsive rug taking to provie a meical history. Fifteen minutes
B. Decrease analgesic effect of opiois evelops into the operation performe uner general
before ecrease effects on respiratory anesthesia, the anesthesiologist reports ifculty
epression ventilating the patient, an she evelops a iffuse
C. Increase sleeping an eating, epression, an maculopapular rash with urticaria. Which of the
pupillary constriction following is the most likely offening agent?
D. Constipation resolves over time with long- A. Rocuronium
term opioi use B. Latex
E. Genetic components associate with opioi C. Cefazolin
use have not been ientie D. Sevourane
E. Propofol
3. A 68-year-ol woman is unergoing a
laparoscopic liver resection. An arterial line an 6. A 9-year-ol man unergoes a laparoscopic
central line are place prior to surgical incision. cholecystectomy for symptomatic cholelithiasis.
As the hepatic parenchyma is being ivie, the Shortly after inuction, the anesthesiologist notes
anesthesiologist reports suen hypotension an an increase in core boy temperature an en-
a rop in en-tial CO. There is no break in the tial CO. After aministration of antrolene
ventilatory circuit. There is only minimal bleeing an aborting the operation, his status improves.
at this time. There are ST changes note on the Which of the following is most likely associate
EKG. Which if the following is the next best step with this iagnosis?
in management of this conition? A. It is an autosomal recessive isorer
A. Transthoracic echocariography (TTE) B. Genetic analysis is require for iagnostic
examination of the heart conrmation
B. Aminister epinephrine C. It is more common in elerly patients
C. Aminister ui bolus D. It may present as late as 4 hours after
D. Emergently place a pulmonary artery catheter anesthesia
line E. Mortality rate is less than 1%
E. Release (esufate) pneumoperitoneum
CHAPtEr 28 Anesthesia 369

7. After excision of multiple subcutaneous lipomas 11. A 9-year-ol boy has been in the peiatric
uner local anesthesia, a 4-year-ol woman intensive care unit for the last 7 ays after
seizes violently. What is the maximum safe ose presenting to the hospital with inuenza
of a local anesthetic agent in a 70-kg woman? infection leaing to respiratory failure requiring
A. 10 to 0 mL 1% liocaine mechanical ventilation. He is receiving
B. 40 to 50 mL % liocaine with epinephrine continuous fentanyl an propofol infusions
C. 40 to 50 mL 1% liocaine with epinephrine for pain control an seation, respectively.
D. 40 to 50 mL 0.5% liocaine This morning he evelope braycaria, an
E. 40 to 50 mL 1% liocaine without epinephrine his urinary output ecrease. He is note to
have hepatomegaly on physical examination.
8. A 0-year-ol man is about to unergo Laboratory values show an elevate creatinine,
arthroscopic surgery on his left shouler. During hyperlipiemia, hyperkalemia, an lactic aciosis.
anesthetic inuction with succinylcholine, the Which of the following is the best next step in
anesthesiologist note trismus that persiste management?
for > minutes, an the mouth coul not be A. Start bicarbonate infusion
opene to perform irect laryngoscopy or place B. Perform liver biopsy
an enotracheal tube. The anesthesiologist was C. Initiate hemoialysis
able to bag mask ventilate the patient. The en- D. Initiate treatment with low-ose epinephrine
tial CO, heart rate, an temperature remaine E. Discontinue propofol an start
normal. Which of the following is the next best exmeetomiine infusion
step in management?
A. Aminister an aitional ose of 12. A 55-year-ol iabetic man unerwent a right-
succinylcholine sie vieo-assiste thoracoscopic surgery
B. Procee with surgery if the patient can be (VATS) for an empyema yesteray. This morning
intubate he is complaining of pain along his meial left
C. Cancel surgery an sen the patient home forearm an has paresthesia of his fourth an
D. Cancel surgery, aminister antrolene, an fth igits. Which of the following risk factors are
amit for 4-hour observation most likely associate with this complication?
E. Cancel surgery, amit for 4-hour observation, A. Male sex
an refer for muscle biopsy B. Emergency surgery
C. Supine positioning uring surgery
9. Which of the following is the best immeiate way D. Hyperthermia uring surgery
to conrm placement of an enotracheal tube in E. Diabetic neuropathy
the airway after intubation?
A. Direct visualization of tube passing through 13. Which of the following is true regaring invasive
the vocal cors lines use for the monitoring of surgical patients?
B. Auscultation of lungs A. Trauma patients show improve mortality
C. Observation of conensation within tube with placement of a pulmonary artery catheter
D. Pulse oximetry (PAC)
E. Capnography B. A normal Allen Test before raial artery
cannulation will reuce incience of han
10. A 65-year-ol man is unergoing urgent surgery ischemia
for gangrenous cholecystitis. The patient has a C. PAC will provie irect measurement of
history of moerate aortic valve stenosis that was systemic vascular resistance
recently iagnose on echocariography but he D. Systolic bloo pressure measure on a raial
enies any symptoms. Which of the following artery catheter will typically be higher than the
woul be most important goal in the anesthetic aortic pressure
management? E. A right bunle branch block seen on
A. Preloa reuction electrocariogram is consiere a
B. Afterloa reuction contrainication for PAC placement
C. Avoiance of hypotension
D. Heart rate goal of >90 beats per minute
E. Use of epherine for hypotension
370 PArt ii Medical Knowledge

14. A 47-year-ol woman is recovering from 17. Which of the following parameters is most
pneumonia complicate by multiorgan likely to preict successful iscontinuation of
system ysfunction. She is currently receiving mechanical ventilation?
hemoialysis after eveloping renal failure A. Rapi shallow breathing inex (RSBI) (f/VT)
seconary to sepsis. This morning, a rapi less than 105
response was calle for respiratory epression B. Negative inspiratory force (NIF) −0 to −30 cm
an confusion, which improve after the HO
aministration of naloxone. Which of the C. Successful spontaneous breathing trial (SBT)
following meications most likely contribute to D. Respiratory rate less than 30 breaths per
her respiratory compromise? minute
A. Fentanyl E. Tial volume greater than 5 mL/kg
B. Hyromorphone
C. Morphine 18. A 66-year-ol woman presents in septic shock
D. Methaone ue to a perforate uoenal ulcer. She is taken
E. Oxycoone urgently to the operating room for an exploratory
laparotomy. Due to persistent hypotension,
15. A 37-year-ol woman unerwent a percutaneous opamine is infuse by the anesthesiologist
besie tracheostomy tube placement. On an is eventually titrate to a rate of 15 mcg/kg
postoperative ay 1, she evelope signicant per minute. At that rate, which of the following
subcutaneous emphysema of the neck over receptors is exerting the preominant effect?
the course of an hour, an her current oxygen A. α1-Arenergic
saturation is 80%. A respiratory therapist B. α-Arenergic
attempte irectional suctioning, but they were C. β1-Arenergic
unable to pass the catheter. What is the most D. β-Arenergic
appropriate next step in management? E. Dopaminergic
A. Remove the tracheostomy tube an attempt
recannulation with a smaller caliber cannula 19. A 55-year-ol man with a history of chronic
B. Remove the tracheostomy tube an obstructive pulmonary isease (COPD)
recannulate over a suction catheter unergoes an interscalene regional block with
C. Remove the tracheostomy tube an bupivacaine for surgery of a left humerus
recannulate over a beroptic bronchoscope fracture. Soon after placement of the block, the
D. Replace the tracheostomy tube using a patient evelops signicant yspnea. Breath
percutaneous tracheostomy kit souns are equal to auscultation an clear. Which
E. Bag mask ventilation an prepare for of the following factors is the most likely cause of
orotracheal intubation his shortness of breath?
A. Pneumothorax
16. Which of the following correctly pairs the B. COPD exacerbation
invasive mechanical ventilation moe with its C. Inavertent intravascular injection of
mechanism of action? bupivacaine
A. Synchronize intermittent mechanical D. Air embolism
ventilation (SIMV): every breath has a E. An elevate left hemiiaphragm
manate volume
B. Airway pressure release ventilation (APRV): 20. At the en of a surgery, an anticholinesterase
maintains continuous positive airway pressure is aministere to a patient to reverse the
(CPAP) with an intermittent release phase neuromuscular blockae. Which of the following
C. Assist-control (AC) ventilation: patient muscles woul be expecte to recover rst?
etermines the rate an volume of breaths A. Diaphragm
D. CPAP: two ifferent pressure settings for B. Auctor pollicis
inhalation an exhalation C. Ocular muscles
E. High-frequency oscillatory ventilation D. Pharyngeal
(HFOV): high respiratory rate with large tial E. Quariceps femoris
volumes
CHAPtEr 28 Anesthesia 371

21. A 40-year-ol man with obesity, hypertension, 23. A patient is given benzocaine spray in
cirrhosis, iabetes mellitus, an chronic anticipation of a besie exible laryngoscopy.
kiney isease (CKD) stage 1 unergoes After several minutes, he evelops a heaache
general anesthesia for repair of an incarcerate an shortness of breath. Pulse oximetry shows an
inguinal hernia. He takes insulin, echothiopate, SpO of 85%, while an arterial bloo gas shows
amloipine, an simvastatin at home. Propofol an SaO of 80% with a PaO of 150 mmHg.
an pancuronium are use for inuction. At Which of the following is the most appropriate
the en of the proceure, a peripheral nerve treatment?
stimulator emonstrates no recovery of muscle A. Intubation
twitches espite 60 minutes of time elapsing. B. Intravenous methylene blue
Which of the following unerlying factors is most C. Discontinue benzocaine an aminister
likely responsible for this conition? prilocaine
A. Diabetes mellitus D. Metoclopramie
B. Obesity E. Thiosulfate
C. Stage 1 CKD
D. Pancuronium
E. Simvastatin

22. Which of the following is associate with


barbiturate toxicity?
A. Myocarial epression
B. Acute tubular necrosis
C. Hepatotoxicity
D. Peripheral neuropathy
E. Seizures

ANSWERS

1. D. Level of seation is a continuum ene by the tolerance to respiratory epression, which partially explains
patient's response to the meications aministere. During the high overose rates. Tolerance within the colon typically
minimal seation, patients have a normal response to ver- oes not evelop an results in chronic constipation (D).
bal stimulation (A). During moerate seation, patients Depenance is characterize by the unpleasant response
have purposeful responses to verbal or tactile stimulation to stopping or reucing intake of the rug, also referre to
(B). While uner eep seation, repeate verbal or painful as withrawal symptoms. Opioi withrawal symptoms
stimulation is neee to achieve purposeful movements. inclue lacrimation, piloerection, muscle aches, nausea,
Intervention on the airway may be require at this level of vomiting, iarrhea, pupillary ilation, insomnia, tachycar-
seation. Uner general anesthesia, the patient is unarous- ia, hyperreexia, an hypertension (C). Aiction is much
able even with painful stimulus (E). Moerate seation an less preictable an less common than both tolerance an
conscious seation are terms that are often use interchange- epenance an is characterize by pronounce cravings,
ably (C). obsessive thinking, compulsive rug taking, an an inability
Reference: Practice guielines for moerate proceural seation to refrain from use (A). It is also now believe that opioi
an analgesia 018: a report by the American Society of Anesthesiol- aiction has a fairly strong genetic component with herita-
ogists Task Force on Moerate Proceural Seation an Analgesia, bility rates similar to iabetes an hypertension (E).
the American Association of Oral an Maxillofacial Surgeons, Amer- References: Volkow ND, McLellan AT. Opioi abuse in chronic
ican College of Raiology, American Dental Association, American pain–misconceptions an mitigation strategies. N Engl J Med.
Society of Dentist Anesthesiologists, an Society of Interventional 016;374(13):153–163.
Raiology. Anesthesiology. 018;18(3):437–479. Akbarali HI, Inkisar A, Dewey WL. Site an mechanism of mor-
phine tolerance in the gastrointestinal tract. Neurogastroenterol Mot.
2. B. Long-term opioi use commonly results in tolerance 014;6(10):1361–1367.
an physical epenence. Tolerance escribes a ecrease
in opioi potency with repeate aministration. Tolerance 3. E. Given the unexplaine hypotension an ecrease in
to analgesic effects of opiois evelops more quickly than en-tial CO, this patient most likely has a CO embolism.
372 PArt ii Medical Knowledge

Clinically signicant CO embolism is very rare uring lap- References: Brummett CM, Waljee JF, Goesling J, et al. New per-
aroscopic surgery but has a mortality rate of approximately sistent opioi use after minor an major surgical proceures in US
8%. CO embolism is thought to be cause by either intra- aults. JAMA Sur. 017;15(6):e170504.
vascular injection of CO into a vessel with either a Veress Hill MV, McMahon ML, Stucke RS, et al. Wie variation an
excessive osage of opioi prescriptions for common general surgi-
neele or trocar uring initial insufation, or by gas enter-
cal proceures. Ann Surg. 017;65(4):709–714.
ing an injure vessel later uring the operation. Signs of a
CO embolism are unexplaine hypotension, hypoxia, or a
5. A. A stuy one in France from 1997 to 004 looke at
suen ecrease in en-tial CO. Transesophageal echocar-
all patients who ha immeiate hypersensitivity reaction
iography (TEE) is the most sensitive metho for etecting
presume to be from allergic reaction. Of the 1816 patients
CO embolism, though often not necessary when clinical sus-
that met criteria for the stuy, the top three offening agents
picion is high (A). Precorial oppler is the most sensitive
for immeiate hypersensitivity reaction were neuromuscular
noninvasive test. If CO embolism is suspecte, insufation
blocking agents (58%), latex (0%), an antibiotics (13%) (B,
shoul be stoppe an the abomen esufate immei-
C). Allergy to inhale anesthetics an hypnotics was much
ately. Though historically it has been recommene to place
less common (D, E). In chilren, latex was more common
the patient in the left lateral an Trenelenburg position to
than neuromuscular blocking agents, but the sample size for
move the air bubble out of the pulmonary artery, new evi-
this population was much lower.
ence suggests that neither of the above positions results in
References: Butterworth J, Mackey D, Wasnick J, etal., es. Inha-
signicant hemoynamic improvements. Rather, for pro- lation anesthetics. In: Morgan & Mikhail's clinical anesthesiology. 5th e.
ceures below the level of the heart, the patient shoul be McGraw-Hill; 013;44–88.
place in the reverse Trenelenburg position to reuce fur- Butterworth J, Mackey D, Wasnick J, etal., es. Intravenous anes-
ther air entrainment. Vasopressor aministration an a ui thetics. In: Morgan & Mikhail's clinical anesthesiology. 5th e. McGraw-
bolus are reasonable interventions for persistent hypoten- Hill; 013;141–156.
sion, but shoul be one after reucing the risk of further Di Leo E, Delle Donne P, Calogiuri GF, Macchia L, Nettis E. Focus
air entrapment (B, C). Pulmonary artery catheters have been on the agents most frequently responsible for perioperative anaphy-
shown to be ineffective at aspirating air with a success rate laxis. Clin Mol Allergy. 018;16:16.
Mertes PM, Alla F, Tréchot P, Auroy Y, Jougla E, Groupe ’Etues
between 6% an 16% an shoul not be the next step in man-
es Réactions Anaphylactoïes Peranesthésiques. Anaphylaxis
agement (D). A "mill-wheel" murmur is present in less than
uring anesthesia in France: an 8-year national survey. J Allergy Clin
half of patients. Immunol. 011;18():366–373.
References: Cottin V, Delafosse B, Viale JP. Gas embolism uring
laparoscopy: a report of seven cases in patients with previous
abominal surgical history. Surg Endosc. 1996;10():166–169. 6. D. This patient likely has malignant hyperthermia, a rare
Mirski M, Lele AV, Fitzsimmons L, et al. Diagnosis an treatment autosomal ominant isorer of skeletal muscle (A). The con-
of vascular air embolism. Anesthesiology. 007;106:164–177. ition is characterize by a hypermetabolic state triggere by
exposure to inhalation anesthetics (sevourane, esurane,
4. E. Opioi abuse has risen substantially in the US in isourane) an/or succinylcholine. The oler anesthetic agents
recent years, prompting research investigating the causes of associate with this reaction inclue halothane an enurane.
this new epiemic. Though the majority of opiois abuse in It is not cause by nitrous oxie, intravenous anesthetic agents,
the US originate from legitimate prescriptions, only 0% of or other neuromuscular blockers (except for succinylcholine).
opioi users were the intene recipients of the initial pre- Malignant hyperthermia occurs when uncontrolle amounts
scription (A). The majority of opioi abusers receive pills of intracellular calcium accumulate in skeletal muscle. Symp-
for free from family members or friens with excessive pills toms may evelop as early as 30 minutes after anesthetic
or from other methos of iversion. Opioi abuse can also aministration an as late as 4 hours postoperatively. Even
lea to further illicit rug use, as 50% to 85% of heroin users after treatment with antrolene, patients nee to be monitore
report having previously abuse prescription opiois (B). As because they can have a refractory response an go back into
excessive opioi prescriptions appear to be one of the inciting a malignant hyperthermic crisis. The initial clues occur in the
factors in opioi abuse, aitional attention has been place operating room after inuction. Rather than achieving com-
on prescribing patterns after surgery. A large retrospective plete paralysis, the anesthesiologist may notice rigiity in the
stuy showe that new persistent opioi use was fairly com- masseter muscle. Other nings inclue an increase in en-
mon after both major an minor surgical proceures, with an tial CO, tachycaria, an an increase in temperature. It is
incience of aroun 6%. The incience was not signicantly imperative that all anesthetics are immeiately stoppe an
ifferent between major an minor surgeries inicating that antrolene given (.5 mg/kg every 5 minutes) until resolution
pain is not the riving factor for this postsurgical complication of symptoms. Dantrolene stabilizes muscle channels in the sar-
(D). Risk factors inepenently associate with new persistent coplasmic reticulum. The mortality rate was previously 30%,
opioi use inclue preoperative tobacco use, alcohol an sub- but recent evience suggests the mortality rate is now approx-
stance abuse isorers, moo isorers, anxiety, an preop- imately 5% (E). A functional test on skeletal muscle biopsy
erative pain isorers (E). To aress the overprescribing of (caffeine halothane contracture test) is use for iagnosis (B).
opiois after surgery, one stuy ientie the number of pills More than 50% of the families show linkage of the invitro con-
(equivalent to 5 mg oxycoone) that woul fully supply the tracture test phenotype to the gene encoing the skeletal mus-
nees of 80% of patients unergoing a number of ifferent cle ryanoine receptor. The test requires a muscle biopsy with
operations. Examples of these nees inclue 5 pills after a par- exposure of the muscle to halothane an caffeine. A positive
tial mastectomy, 15 pills after a laparoscopic cholecystectomy, test will cause signicant muscle contraction. The majority of
an 15 pills after an open inguinal hernia repair (C). cases occur in chilren or young aults (C).
CHAPtEr 28 Anesthesia 373

References: Jurkat-Rott K, McCarthy T, Lehmann-Horn F. Genet- inclue fevers, increase en-tial CO, generalize muscle
ics an pathogenesis of malignant hyperthermia. Muscle Nerve. rigiity, autonomic instability, an rhabomyolysis. The inci-
000;23(1):4–17. ence of patients who evelop masseter spasms an go on
Ellinas H, Albrecht MA. Malignant hyperthermia upate. Anes- to evelop malignant hyperthermia is unknown. It shoul
thesiol Clin. 00;38(1):165–181.
be note that isolate masseter spasm is not pathognomonic
7. C. There are relatively few sie effects of local anes- for malignant hyperthermia. The surgery shoul be cancele
thetic agents such as liocaine, unless they are inavertently an the patient amitte for at least 4 hours of observation
injecte intravenously or aministere in oses higher than to watch for the evelopment of rhabomyolysis or malig-
recommene. Toxicity begins with neurologic signs an nant hyperthermia; the patient shoul not be sent home
symptoms such as light-heaeness, facial paresthesias, prior to 4 hours of observation an monitoring in the hos-
blurre vision, an tinnitus. It can progress to lethargy, pital (C). In the absence of hemoynamic instability, elevate
tremors, an tonic-clonic seizures. Neurologic symptoms CO, or fever, it is unnecessary to aminister antrolene (D).
precee the more severe cariovascular symptoms, which However, these patients shoul be referre to a center that
inclue hypertension an tachycaria (early symptoms) can perform the necessary testing, incluing genetic testing
an later hypotension, cariovascular collapse, braycaria an a caffeine halothane contracture test (muscle biopsy
or conuction abnormalities, an even cariac arrest. The test). After muscle biopsy, the tissue is only viable for sev-
maximum oses for local injection of liocaine are 5 mg/kg eral hours, so testing must take place in centers specialize
without epinephrine an 7 mg/kg with epinephrine because in iagnosing malignant hyperthermia (E).
the vasoconstriction elays the systemic release of liocaine. References: Schneierbanger D, Johannsen S, Roewer N, Schus-
Because a 1% solution of liocaine contains 10 mg/mL, an ter F. Management of malignant hyperthermia: iagnosis an treat-
ment. Ther Clin Risk Manag. 014;10:355–36.
easy way to remember this is to multiply the patient's weight
Bauer SJ, Orio K, Aams BD. Succinylcholine inuce masseter
by either 5 (no epinephrine) or 7 (with epinephrine) an then
spasm uring rapi sequence intubation may require a surgical air-
ivie by 10. Therefore, for this patient: 70 kg × 5 mg/kg = 350 way: case report. Emerg Med J. 005;:456–458.
mg an ivie by 10 mg/mL = 35 mL of 1% liocaine. For lio- Sheikh MM, Riaz A, Umair HM, Waqar M, Muneeb A. Succi-
caine with epinephrine, 70 kg × 7 mg/kg = 490 mg an ivie nylcholine-inuce masseter muscle rigiity successfully manage
by 10 = 49 mL of 1% liocaine. For a % liocaine solution, one with propofol an laryngeal mask airway: a case report an brief
woul ivie by 0 (4.5 mL an 17.5 mL, respectively, with review. Cureus. 00;1(7):e9376.
an without epinephrine), an for a 0.5% solution, one woul
ivie by 5 (70 mL an 98 mL, respectively, with an without 9. E. Although irect visualization of the tube passing
epinephrine). Patients who experience local anesthetic sys- through the vocal cors, auscultation of the lungs, visual-
temic toxicity (LAST) shoul be treate by iscontinuing the ization of conensation within the tube, an pulse oximetry
local anesthetic, aministering uis, support with 100% FiO, are goo ajuncts to conrm initial placement of the eno-
hyperventilation, an aministering 0% intralipi with a bolus tracheal tube, interpretation is subjective an not as accu-
of 1 to 1.5 mL/kg over one minute. The bolus can be repeate rate as more objective methos for conrming the position
every 3 minutes up to a total ose of 3 mL/kg, followe by of the enotracheal tube within the trachea (A–C). Both the
an infusion of 0.5 mL/kg/min which is continue until the American College of Emergency Physicians an the Amer-
patient is hemoynamically stable for at least 10 minutes. CPR ican Society of Anesthesiologists recommen capnography
an epinephrine shoul be use in cariac arrest, an bicarbon- or en-tial CO etection evices as the preferre conr-
ate shoul be use in aciosis. Benzoiazepines are preferre matory test for tracheal intubation (E). Patients shoul have
over propofol to manage seizures. a continuous uniform waveform of en-tial CO with simi-
References: Warren JA, Thoma RB, Georgescu A, Shah SJ. Intra- lar amplitues to conrm tracheal intubation; however, this
venous lipi infusion in the successful resuscitation of local anes- oes not ifferentiate a tracheal from a bronchial intubation.
thetic-inuce cariovascular collapse after supraclavicular brachial A capnographic waveform that shows en-tial CO etec-
plexus block. Anesth Analg. 008;106(5):1578–1580. tion but oes not have a continuous waveform or the ampli-
Neal JM, Mulroy MF, Weinberg GL, American Society of Regional tues get smaller an smaller until no aitional en-tial
Anesthesia an Pain Meicine. American Society of Regional Anes- CO can be etecte is inicative of an esophageal intuba-
thesia an Pain Meicine checklist for managing local anesthetic sys-
tion. Direct visualization of the tube passing through the
temic toxicity: 01 version. Reg Anesth Pain Med. 01;37(1):16–18.
Cao D, Hear K, Foran M, Koyfman A. Intravenous lipi emul-
cors is not always reliable, as the cors can be misienti-
sion in the emergency epartment: a systematic review of recent lit- e or the tube can be isloge from the trachea before it is
erature. J Emerg Med. 015;48(3):387–97. secure (A). Auscultation of the lungs is not always reliable
because it is possible to get referre souns from the stom-
8. E. Masseter muscle rigiity, or trismus, is consiere a ach (B). Conensation within the tube can occur even with
normal reaction to the aministration of neuromuscular esophageal intubation (C). Pulse oximetry is also not reliable,
blocking agents. However, if this conition persists for more as hypoxia with esophageal intubation can be very elaye if
than 0 to 30 secons, it is consiere an abnormal response, the patient is preoxygenate well (D).
an the clinician nees to have a high level of concern for References: American Society of Anesthesiologists Task Force
on Management of the Difcult Airway. Practice guielines for man-
malignant hyperthermia, an nonemergent surgeries shoul
agement of the ifcult airway: an upate report by the American
be cancele (B). Persistent trismus is not a sign of inaequate Society of Anesthesiologists Task Force on Management of the Dif-
neuromuscular blockae, an thus aitional neuromus- cult Airway. Anesthesiology. 003;98(5):169–77.
cular blocker aministration is not inicate (A). Masseter Grmec S. Comparison of three ifferent methos to conrm tra-
spasm is an early inicator of susceptibility to malignant cheal tube placement in emergency intubation. Intensive Care Med.
hyperthermia. Other markers for malignant hyperthermia 00;8(6):701–704.
374 PArt ii Medical Knowledge

10. C. While asymptomatic aortic stenosis is not a contra- Hemphill S, McMenamin L, Bellamy MC, et al. Propofol infusion
inication to surgery, it requires careful intraoperative mon- synrome: a structure literature review an analysis of publishe
itoring. The increase pressures require to overcome the case reports. Br J Anaesth. 019;1(4):448–459.
Ichikawa T, Okuyama K, Kamata K, et al. Suspecte propofol
stenosis cause concentric hypertrophy of the left ventricle,
infusion synrome uring normal targete propofol concentration. J
which in turn reuces the compliance of the ventricle. This
Anesth. 00;34(4):619–63.
makes these patients heavily preloa epenent for ventric- Mirrakhimov AE, Voore P, Halytskyy O, et al. Propofol infu-
ular lling, an careful attention shoul be pai to maintain- sion synrome in aults: a clinical upate. Crit Care Res Pract.
ing aequate intravascular volume (A). In aition, up to 015;015:60385.
40% of the left ventricular en-iastolic volume (LVEDV) is Schroeppel TJ, Fabian TC, Clement LP, et al. Propofol infusion
provie by the atrial kick. Atrial arrhythmias can quickly synrome: a lethal conition in critically injure patients eliminate
lea to heart failure an shoul be aggressively treate, pref- by a simple screening protocol. Injury. 014;45(1):45–49.
erably with ebrillation. Braycaria (<50 bpm) shoul
also be avoie because patients have a xe stroke volume 12. E. The American Society of Anesthesiologists has ien-
an cariac output is epenent on the heart rate. Hypotension tie perioperative peripheral nerve injuries as among the
an reuctions in afterloa will reuce coronary artery lling top three meical malpractice claims irecte at anesthesi-
an increase the likelihoo of cariac ischemia (B). Hypoten- ologists an operating room staff. Despite the fact that over
sion shoul be preferentially treate with selective α-arenergic 60% inclue ocumentation of appropriate paing an
agents such as phenylephrine, which increase SVR an prevent position, almost half of the cases involve payment. Of these
tachycaria (C, E). Sinus tachycaria an hypertension can injuries, ulnar an brachial plexus injuries appear to be the
precipitate ischemia an shoul be treate by increasing the most common. These injuries appear to happen by one of
epth of anesthesia (D). Because of the potential for braycar- several mechanisms: irect nerve amage, stretch/compres-
ia an hypotension with beta-blockers, these agents shoul sion, an ischemia or toxicity of locally injecte meications.
be use with caution, an short-acting agents, such as esmolol, Risk factors relate to the patient inclue hypertension, ia-
are preferre. To summarize, the goals of anesthesia in patients betes, smoking, extremes in boy mass inex, an malnu-
with aortic stenosis are to avoi hypotension, ensure aequate trition (A, E). Chronically ysfunctional nerves (such as in a
LVEDV, an maintain coronary perfusion pressure, normal patient with iabetic neuropathy) may be particularly sus-
sinus rhythm, an preloa. ceptible to an acute insult. Intraoperative risk factors inclue
References: Christ M, Sharkova Y, Gelner G, Maisch B. Pre- hypothermia, hypovolemia, hypotension, hypoxemia, an
operative an perioperative care for patients with suspecte electrolyte abnormalities (B–D). Because it can take several
or establishe aortic stenosis facing noncariac surgery. Chest. ays for enervation of the affecte muscles to take place,
005;18(4):944–953. EMG is often normal in the immeiate postoperative perio.
Anesthesia for patients with cariovascular isease. In: Butter- However, EMG shoul still be one early because any abnor-
worth IV JF, Mackey DC, Wasnick JD, es. Morgan & Mikhail's clinical malities likely represent a preexisting neuropathy that was
anesthesiology. 6th e. McGraw-Hill; 018. simply exacerbate by the operation. Most of these injuries
will heal with time. However, operative intervention can be
11. E. Propofol infusion synrome is a clinical conition performe epening on the severity of injury an failure to
that is associate with higher oses of propofol infusion improve with conservative measures.
(>4 mg/kg/hr) that is continue for more than 48 hours. It References: Chui J, Murkin JM, Posner KL, Domino KB. Periop-
is associate with metabolic aciosis, arrhythmias (most often erative peripheral nerve injury after general anesthesia: a qualitative
braycaria), rhabomyolysis, hyperlipiemia, hepatomegaly systematic review. Anesth Analg. 018;17(1):134–143.
(not splenomegaly), renal failure, an eventual cariovascu- Lalkhen AG, Bhatia K. Perioperative peripheral nerve injuries.
Contin Educ Anaesth Crit Care Pain. 01;1(1):38–4.
lar collapse. A liver biopsy is not neee for iagnosis (B). The
Welch MB, Brummett CM, Welch TD, et al. Perioperative
rst case report was in the peiatric population, an though
peripheral nerve injuries: a retrospective stuy of 380,680 cases
the correlation with age is unclear, chilren may be at a higher uring a 10-year perio at a single institution. Anesthesiology.
risk. Once it has been iagnose, the rst step in management is 009;111(3):490–497.
immeiate cessation of propofol, an another seating meica- Winfree C, Kline DG. Intraoperative positioning nerve injuries.
tion shoul be starte (E). However, treatment is largely ineffec- Surg Neurol. 005;63(1):5–18.
tive, especially in the setting of arrhythmias. Renal replacement
may be utilize in patients who evelop hyperkalemia an 13. D. Though invasive hemoynamic monitoring of the
rhabomyolysis but is not the most important next step (C). critically ill patient provies valuable information, compli-
Soium bicarbonate aministration to treat lactic aciosis is not cations of placement must always be measure against the
recommene (A). The combination of high-ose propofol with potential avantages. PAC was consiere the stanar of
exogenous catecholamines (i.e., phenylephrine, norepineph- care for many critically ill patients but is being use less
rine, epinephrine) or steroi aministration appears to trigger frequently now because of multiple stuies showing no
the synrome. Catecholamines an sterois aggravate propofol improvement in mortality (A). Base on these nings, inva-
inhibition of fatty aci metabolism, promoting rapi an irre- sive hemoynamic monitoring is no longer recommene
versible peripheral an cariac muscle injury (D). A screening for routine use. However, there may still be a role in patients
tool for this conition has been propose that inclues aily with unknown volume status, severe cariogenic shock,
CPK an lactate levels because these may be the rst inica- pulmonary artery hypertension, or severe unerlying car-
tions that propofol infusion synrome has evelope. iopulmonary isease. Before placement, an electrocario-
References: Foale V, La Monaca E. Propofol infusion synrome: gram must be obtaine to rule out left bunle branch block
an overview of a perplexing isease. Drug Saf. 008;31(4):93–303. (LBBB). There is a high incience of a temporary right bunle
CHAPtEr 28 Anesthesia 375

branch block with placement, an in the setting of an LBBB, an all its metabolites, so it may be use in ialysis-epenent
a complete heart block coul be incite. However, there is no patients (E).
contrainication to placement in most other arrhythmias (E). References: Dean M. Opiois in renal failure an ialysis
The PAC irectly measures cariac output, central venous patients. J Pain Symptom Manag. 004;8(5):497–504.
pressure, mixe venous oxyhemoglobin saturation, right- Leuppi-Taegtmeyer A, Duthaler U, Hammann F, et al. Pharmaco-
sie cariac pressures, an pulmonary artery pressures. kinetics of oxycoone/naloxone an its metabolites in patients with
en-stage renal isease uring an between haemoialysis sessions.
From this information, systemic vascular resistance, cariac
Nephrol Dial Transplant. 019;34(4):69–70.
inex, an oxygen elivery/uptake can be calculate (C).
Schumacher, M etal. Opioi agonists & antagonists. In: Katzung
Invasive arterial bloo pressure monitoring provies contin- BG, Trevor AJ, es. Basic & clinical pharmacology. 13th e. McGraw-
uous measurement of bloo pressure as well as easy access Hill; 014.
to arterial bloo gas samples. However, it too comes with
potential complications, the most signicant being arterial 15. E. Early islogement of the tracheostomy tube is an
thrombosis. The raial artery is generally preferre because infrequent but potentially evastating complication asso-
of aequate collaterals through the ulnar, relative ease of can- ciate with placement. In general, the rst tube exchange
nulation, an lower incience of infection. Though the Allen happens between postoperative ays 3 an 7. Acciental
test is currently consiere manatory before raial arterial removal before a planne exchange can potentially cause
line placement, it oes not seem to accurately preict risk of loss of the airway. Aitionally, manipulation of the eno-
han ischemia. Several trials looking at the Allen test have tracheal tube by ancillary staff may promote a false passage.
shown isagreement on what constitutes a positive test, high Despite positioning of the tracheostomy in a false passage,
variability among observers, an inconsistent preiction of patients may be able to maintain some oxygenation espite
collateral ow when compare with less subjective tests such its location. This can manifest as respiratory istress (in a
as ultrasoun (B). It is also important to remember that sys- nonventilate patient) an with subcutaneous emphysema.
tolic bloo pressure in the raial artery will be higher than Replacement of the tracheostomy tube can be attempte via
in the aorta but mean arterial pressure shoul be preserve. multiple methos incluing trying to use a small caliber can-
A higher systolic pressure occurs with istal progression, nula, using a suction catheter or beroptic bronchoscope as
smaller arterial caliber, an oler age. a guie, or using the equipment in a percutaneous tracheos-
References: Barone JE, Malinger RV. Shoul an Allen test be per- tomy kit (A–D). However, if the patient is unstable an rap-
forme before raial artery cannulation? J Trauma. 006;61():468–470. ily esaturating, securing the airway is the main priority
Fischer J, e. Cariovascular monitoring an support. In: Fisch-
an oral enotracheal is warrante.
er's mastery of surgery. 6th e. Wolters Kluwer Health/Lippincott
References: Halum SL, Ting JY, Plowman EK, et al. A multi-in-
Williams & Wilkins;01.
stitutional analysis of tracheotomy complications. Laryngoscope.
Vincent JL. The pulmonary artery catheter. J Clin Monit Comput.
01;1(1):38–45.
01;6(5): 341–345.
Subroto, P, Colson, Y. (014). Tracheostomy. In: Sugarbaker DJ,
Bueno R, Colson YL, Jaklitsch MT, Krasna MJ, Mentzer S, es. Adult
14. C. Opioi pain meications unergo metabolism pre- chest surgery. n e. McGraw-Hill Professional; 014.
ominantly in the liver into a variety of metabolites that are
generally excrete in the urine. Morphine is metabolize via 16. B. Data showing improve survival of one moe of
glucuroniation by the liver, brain, an kiney into the active ventilation over another in specic isease states is inconsis-
metabolites morphine-3-glucuronie an morphine-6-glu- tent at best. However, theoretically, each moe of ventilation
curonie. The glucuronie metabolites are then eliminate offers certain avantages an isavantages. The conven-
via bile an preominantly urine. Morphine-6-glucuronie tional moes of mechanical ventilation can be consiere on
is more selective for mu-receptors an is a more potent anal- a spectrum base on the amount of support that is provie
gesic than morphine. In the setting of renal injury, the metabo- to the patient an how the machine supports patient-initiate
lites can persist for long perios of time an cause respiratory breaths. At the lowest en of the spectrum is CPAP, in which
epression. Thus, morphine an coeine (which is a prorug all breaths are triggere by the patient an no aitional sup-
that is metabolize into morphine) shoul be avoie in renal port is provie. The mechanical ventilator simply provies
failure an in patients on ialysis. Hyromorphone oes have a constant pressure an allows patients to breath at a rate
an active metabolite, but it oes not have increase potency as an volume that they etermine (D). Pressure support venti-
is seen with morphine's metabolites (B). Fentanyl an metha- lation (PSV) allows the patient to etermine the rate an vol-
one are likely the safest meications to use in ialysis patients ume of breaths but provies aitional pressure to support
because all of the metabolites are inactive (A, D). Oxycoone a patient-triggere breath. SIMV allows the clinician to man-
is metabolize into noroxycoone (which is ve times less ate a certain number of breaths per minute at a set volume
potent than oxycoone) an oxymorphone (which is 8 times or pressure but allows the patient to breath spontaneously in
as potent as oxycoone). However, both these metabolites are between the machine-triggere breaths. It is frequently com-
metabolize to noroxymorphone, which is weakly active, an bine with PSV to provie aitional pressure to support the
oxycoone an all its metabolites also unergo glucuronia- patient-triggere breaths. Proponents avocate that it allows
tion, resulting in metabolites which are inactive at the mu-re- patients to exercise their respiratory muscles, but this comes
ceptor, an thus o not have signicant opioiergic effects. at the expense of an increase effort of breathing, which can
Since the metabolites o not appear to have signicant active potentially fatigue the iaphragm (A). AC allows patients
effects on opioi receptors, it is eeme safe to use in renal to trigger breaths, but every breath has a manate volume
patients, although it is consiere a secon-line agent. Fur- or pressure (C). This allows patients to change their work of
thermore, hemoialysis increases the clearance of oxycoone breathing simply by increasing the respiratory rate (RR), an,
376 PArt ii Medical Knowledge

because every breath is machine elivere, it has the lowest Discontinuing Ventilatory Support” was publishe in Chest
associate work of breathing. APRV is a moe esigne to by a task force specically assemble to assess current wean-
maximize alveolar recruitment by maintaining relatively ing strategies. The basis of these recommenations was
constant higher pressures with an intermittent release phase. that aily evaluation of reainess for extubation shoul be
In this moe, there is a high pressure (P high) which is set one, unerlying conitions correcte, an that the venti-
for a prolonge time (T high) to recruit alveoli an maintain lator shoul be iscontinue as early as possible. Delaying
aequate lung volume, with a time-cycle release phase to a extubation in patients reay for spontaneous breathing was
lower pressure (P low) for a shorter amount of time (T low), associate with an increase in mortality, increase nosoco-
where CO removal occurs. This moe of ventilation uses an mial pneumonia, an a prolonge hospital stay. The most
extreme inverse I:E (inspiratory:expiratory) ratio (which is- preictive factor for successful extubation was successful
tinguishes it from BiPAP (biphasic positive airway pressure)) SBT (A, B, D, E). Current recommenations inclue a aily
to allow for more time at a higher pressure to promote alve- screening for SBT reainess; which inclues an improve-
olar opening an recruitment. APRV is not routinely use ment in the unerlying isease state, aequate gas exchange
an it is reserve as a rescue moe for treating acute respi- (high PaO, low FiO, low PEEP:FiO ratio), hemoynamic
ratory istress synrome (ARDS) or acute lung injury (ALI). stability, an either the iscontinuation of vasopressors or
Its avantages inclue: increase comfort for the patient, ecreasing vasopressor requirements, an the patient's abil-
allowing spontaneous ventilation an ecrease seation ity to generate a spontaneous breath. Patients shoul also
requirements, alveolar recruitment, improve oxygenation, ieally be able to follow simple commans, be neurologically
an hemoynamic stability. HFOV is a moe of ventilation intact (in orer to protect their airway once extubate), an
that works off of the assumption that high airway pressures not require frequent suctioning of secretions (i.e., suction-
can be tolerate by patients as long as they are not sustaine ing every 1 hour woul not make the patient a goo can-
for prolonge perios of time. The goal is to maintain the iate for extubation trial). Once these parameters are met,
lungs at a relatively constant mean airway pressure with the patient shoul unergo an SBT. The moality of the SBT
sinusoial ow oscillation, which can recruit alveoli but oes (CPAP versus pressure support versus T-piece) is not signi-
not result in overistention. Therefore, patients receive small cant, as one has not been emonstrate to be superior to the
tial volumes, which are below the volume of ea space (E) others. Clinicians shoul use the moality their institution
with a high RR. Current approve ventilators in the Unite uses, an the one with which they have the most experience.
States o not support spontaneous breathing with HFOV; A patient is consiere to have “faile” an SBT if any of the
heavy seation or paralysis is generally require with HFOV. following criteria are met: worsening gas exchange, hemoy-
Aitionally, it can result in hemoynamic instability, pul- namic instability, signicant increase in respiratory rate (RR),
monary barotrauma (e.g., pneumothorax), an increase change in mental status, iaphoresis, or signs of increase
infections. This is also a rescue moe of ventilation that may work of breathing. Patients that pass an SBT shoul be con-
be use in patients with ALI/ARDS or burn patients with siere for immeiate extubation. Though several specic
inhalation injury. Current recommenations are to limit its values help guie clinicians in etermining a patient's rea-
use to high-volume centers because of the increase train- iness to extubate (e.g., RSBI <105, tial volume 4–6 mL/kg,
ing require of the staff an the time-intensive nature of NIF −0 to −30 cm HO, minute ventilation 10–15 L/min,
treatment. RR <30 bpm), stuies have emonstrate that they are not
References: Brochar L, Lellouche F. Pressure-support ventila- iniviual preictors of extubation success. RSBI is consi-
tion. In: Tobin MJ, e. Principles and practice of mechanical ventilation. ere an excellent preictor of faile extubation but shoul
3r e. McGraw-Hill Meical; 013. not be use solely to etermine reainess for extubation, nor
Froese A, Ferguson N. High-frequency ventilation. In: Tobin MJ, is it a preictor of successful extubation. Although the goal
e. Principles and practice of mechanical ventilation. 3r e. McGraw-
is early extubation, patients really nee to be optimize an
Hill Meical; 013.
emonstrate that they have a high likelihoo of extubation,
Higgins J, Estetter B, Hollan D, Smith B, Derak S. High-fre-
quency oscillatory ventilation in aults: respiratory therapy issues.
as premature extubation may result in reintubation, which is
Crit Care Med. 005;33(3 Suppl):S196–S03. associate with an 8-fol increase risk of nosocomial pneu-
Mancebo J. Assist-control ventilation. In: Tobin MJ, e. Principles monia an a 6- to 1-fol increase risk of mortality.
and practice of mechanical ventilation. 3r e. McGraw-Hill Meical; References: Celli B. Mechanical ventilatory support. In:
013. Kasper DL, Fauci AS, Hauser SL, Longo DL, Jameson JL, Loscalzo
Putensen C. Airway pressure release ventilation. In: Tobin MJ, J, es. Harrison's principles of internal medicine. 19th e. McGraw-
e. Principles and practice of mechanical ventilation. 3r e. McGraw- Hill; 015.
Hill Meical; 013. MacIntyre NR, Cook DJ, Ely EW Jr, et al. Evience-base guie-
Sassoon C. Intermittent manatory ventilation. In: Tobin MJ, e. lines for weaning an iscontinuing ventilatory support: a collective
Principles and practice of mechanical ventilation. 3r e. McGraw-Hill task force facilitate by the American College of Chest Physicians;
Meical; 013. the American Association for Respiratory Care; an the American
College of Critical Care Meicine. Chest. 001;10(6 Suppl):375S–95S.
McConville JF, Kress JP. Weaning patients from the ventilator. N
17. C. The term weaning when escribing iscontinuation Engl J Med. 01;367(3):33–39.
of mechanical ventilation refers to an ol concept of slowly
reucing ventilator support until a patient is reay to take 18. A. Dopamine is an α- an β-arenergic agonist that
over the work of breathing on his or her own. Because this exerts a variable effect epenent on the ose. However,
practice is no longer encourage, there has been a push to regarless of ose, its effect on α- an β-arenergic receptors
change the term to liberation from the mechanical ventila- is generally weaker than epinephrine an norepinephrine (B,
tor. In 001, “Evience-Base Guielines for Weaning an D). At lower oses (1– mcg/kg per minute), its preominant
CHAPtEr 28 Anesthesia 377

effect is on the opaminergic receptors, causing renal an Reference: McGrath CD, Hunter JM. Monitoring of neuromus-
visceral vasoilation (E). As you increase the ose to 3 to 10 cular block. Contin Educ Anaesth Crit Care Pain. 006;6(1):7–1.
mcg/kg per minute, the β1-arenergic receptors preomi-
nate; this is most similar to the effects of obutamine or low- 21. D. Failure to regain muscle twitches after neuromuscu-
ose epinephrine. This causes an increase in cariac output, lar blockae shoul raise concern for a pseuocholinesterase
primarily by increasing stroke volume (C). As you increase eciency. This patient was given succinylcholine for inuc-
the ose further to greater than 10 mcg/kg per minute, the tion an because pseuocholinesterase is necessary for the
α1-arenergic receptors preominate, leaing to peripheral egraation of succinylcholine, neuromuscular blockae
vasoconstriction; this is most similar to the effects of phenyl- was not reverse. Pseuocholinesterase eciency can be
ephrine. Dopamine infusions o not signicantly affect α- either acquire or ue to genetic abnormality, inherite in an
an β-arenergic receptors. autosomal recessive fashion. Conitions that lower plasma
Reference: Han J, Cribbs SK, Martin GS. Sepsis, severe sepsis, pseuocholinesterase are chronic infections (i.e., tuberculo-
an septic shock. In: Hall JB, Schmit GA, Kress JP, es. Principles of sis), extensive burns, liver isease, malnutrition, malignancy,
critical care. 4th e. McGraw-Hill; 014. an uremia (A–C). Meications that lower plasma pseuo-
cholinesterase are anticholinesterase inhibitors, chlorprom-
19. E. An interscalene nerve block is a frequently per- azine, contraceptives, cyclophosphamie, echothiophate eye
forme an generally well-tolerate anesthesia ajunct for rops, esmolol, glucocorticois, metoclopramie, an pancu-
upper extremity surgery. Local anesthetic is injecte into ronium, among others (E). Unfortunately, treatment is mainly
the interscalene groove, which then isperses to block the supportive an patients must be maintaine on mechanical
brachial plexus (C5-T1). However, the origin of the phrenic ventilation until spontaneous recovery takes place. Pseuo-
nerve (C3-C5 nerve roots) is in close proximity to the tar- cholinesterase also affects the metabolism of ester local anes-
gete area of the block, especially high in the neck, an ipsi- thetics an up to 50% of the metabolism of cocaine, which
lateral iaphragmatic paralysis is possible. One small stuy increases their risk of life-threatening cocaine toxicity. The
showe a 100% incience of iaphragm ysfunction when iagnosis is conrme by a laboratory assay emonstrating
evaluate with ultrasoun. This complication is generally ecrease plasma cholinesterase enzyme activity.
well tolerate by patients with an aequate pulmonary Reference: Soliay FK, Conley YP, Henker R. Pseuocholines-
reserve, but it can be very problematic for patients with terase eciency: a comprehensive review of genetic, acquire, an
lung isease. Ultrasoun guiance, targeting the brachial rug inuences. AANA J. 010;78(4):313–30.
plexus at a lower level in the neck, an lower volumes of
anesthetic agent are use to help prevent this complication, 22. A. Barbiturates are a class of meications that were pre-
but cranial sprea of the agent is still possible. Pneumo- viously use for anesthetic inuction an seizures. However,
thorax is a known complication, but with bilateral breath they have largely been replace by other agents for these two
souns it woul be unlikely (A). COPD exacerbation is inications. As such, barbiturate toxicity is relatively rare.
also unlikely with such an acute onset an clear breathing They are central nervous system epressants an can cause
souns (B). Air embolism is also unlikely with a percutane- effects ranging from rowsiness to general anesthesia. They
ous block (D). The toxic oses of intravenous bupivacaine inhibit neuron ring an are protective against seizures, as
are associate with cariac an neurotoxicity, not isolate they ecrease the seizure threshol (E). Higher oses of bar-
yspnea (C). biturates inhibit the respiratory rive an normal rhythmic
Reference: Urmey WF, Talts KH, Sharrock NE. One hunre respiration. Hepatotoxicity is not seen in barbiturate toxicity;
percent incience of hemiiaphragmatic paresis associate with however, it oes inhibit CYP enzymes, which can increase
interscalene brachial plexus anesthesia as iagnose by ultrasonog- concentrations of other rugs that unergo hepatic egra-
raphy. Anesth Analg. 1991;7(4):498–503. ation (C). At the level of the peripheral nervous system,
barbiturates ecrease transmission through the autonomic
20. A. Not all muscles respon in the same fashion to neu- nervous system an suppress nicotinic receptors, which con-
romuscular blockae. In general, central muscles (e.g., ia- tribute to hypotension but not peripheral neuropathy (D). At
phragm) have a greater bloo supply an will have a quicker anesthetic oses, barbiturates o minimally suppress cariac
onset an quicker recovery from paralysis compare with reexes because of suppression of the autonomic ganglia,
peripheral muscles (e.g., quariceps femoris), which will which is only problematic in patients with unerlying car-
have slower onset an slower recovery (E). Because of the iac isease. However, at toxic oses, there is irect suppres-
variability in muscle relaxant uration an the potentially sion of cariac contractility. Renal injury is likely seconary
evastating complications of incomplete recovery before to hypotension rather than having any irect effect on the
extubation, many argue that quantitative train of four testing kineys (B).
shoul be routine for all cases. One important exception to Reference: Mihic S, Harris R. Hypnotics an seatives. In: Brun-
this rule involves the muscles of the upper airway an phar- ton LL, Chabner BA, Knollmann BC, es. Goodman & Gilman's: the
ynx, which have quick onset but slow offset (D). The ocu- pharmacological basis of therapeutics. 1th e. McGraw-Hill; 015.
lar muscles ten to behave like central muscles an, for this
reason, are an ieal muscle group to monitor at inuction 23. B. Benzocaine toxicity can manifest as methemoglobin-
an uring the operation because they will serve as a surro- emia. In this conition, the ferrous component of hemoglo-
gate for measuring aequate blockae of the central muscles bin is oxiize to form ferric hemoglobin, which oes not
(C). Conversely, auctor pollicis is a goo muscle group to effectively carry oxygen. Mil to moerate methemoglo-
monitor at the en of anesthesia because return of function binemia can cause marke cyanosis but is generally well
will ensure that the central muscles an pharynx have recov- tolerate an oes not typically require mechanical intuba-
ere from blockae (B). tion (A). Pulse oximetry will not reliably assess the egree
378 PArt ii Medical Knowledge

of hypoxemia. It will be falsely elevate initially an can be with supplemental oxygen an intravenous methylene blue,
falsely low after treatment with methylene blue. Methemo- which will reuce hemoglobin back to the ferrous state (B).
globinemia will result in a falsely low saturation when SaO Prilocaine has similar toxicity to bupivacaine an will not
is greater than 85%, an a falsely high saturation when SaO treat the unerlying issue (C). Metoclopramie is use for
is <85%. The partial pressure of oxygen in the bloo (PaO) elaye gastric emptying (D). Thiosulfate is use in the treat-
will remain normal, so stanar arterial bloo gas analyzers, ment of cyanie toxicity (E).
which calculate the oxygen saturation base off of the PaO, Reference: Blanc P. Methemoglobinemia. In: Olson KR, e. Poi-
will show a falsely elevate oxygen saturation. Treatment is soning & drug overdose. 6th e. McGraw-Hill; 01.
Fluids, Electrolytes, and
Acid-Base Balance
JORDAN M. ROOK, AREG GRIGORIAN, AND CHRISTIAN DE VIRGILIO 29
ABSITE 99th Percentile High-Yields
I. Physiology/Pathology
A. Total boy water in liters = 0.6 × weight in kg (in males); 0.5 × weight in kg (in females)
1. Intracellular water is 40% of total boy weight an extracellular water is 0% of total boy weight;
plasma accounts for about 5% of total boy weight

II. Resuscitative Fluis: replaces water an electrolyte losses seconary to pathologic processes
A. Crystallois: contains water-soluble molecules
1. Sepsis: aults, initial bolus: 30 mL/kg (0 mL/kg in chilren)
2. Common crystalloi/colloi:
a) Lactate Ringer’s: 5 electrolytes—Na (130 mEq/L), K (4 mEq/L), Ca (.7 mEq/L), Cl (109 mEq/L),
lactate (8 mEq/L)
b) Normal saline (0.9%):  electrolytes—Na, Cl (both 154 mEq/L)

III. Pathology
A. Vomiting/high nasogastric tube output
1. Metabolic isturbance: hypochloremia, hypokalemia, metabolic alkalosis
2. Mechanism: loss of HCl an volume -> kineys retain bicarb an Na+ an excrete K+, H+, Cl−
3. Treatment: normal saline
B. High output ostomy, stula, iarrhea
1. Metabolic isturbance: non–anion gap metabolic aciosis
2. Mechanism: pancreatic ui, enteric ui (small intestine), an iarrhea rich in bicarb
3. Treatment: lactate Ringer’s
C. Synrome of inappropriate antiiuretic hormone secretion (SIADH)
1. Metabolic isturbance: euvolemic hyponatremia
2. Diagnosis: urine osmolality abnormally high
3. Treatment: ui restriction, hypertonic saline, vasopressin antagonist in severely symptomatic
refractory cases (tolvaptan or emeclocycline)
D. Diabetes Insipius (DI) (most commonly occurs after brain injury)
1. Nephrogenic DI: kineys unresponsive to vasopressin/DDAVP; Central DI: posterior pituitary
oes not release vasopressin/DDAVP; both types of DI result in lack of aquaporin channels in istal
convolute tubules an subsequent inability to reabsorb free water
2. Metabolic isturbance: hypovolemic hypernatremia
3. Diagnosis: urine osmolality abnormally low (<300 mmol/L) with high serum osmolarity
(>80 mmol/L) an serum soium (>14 mEq/L)
4. Treatment: central DI respons to exogenous ADH (esmopressin); nephrogenic DI manage with
supportive care an free water

379
AL GRAWANY
380 PArt ii Medical Knowledge

E. Refeeing synrome (shift from fat to carbohyrate metabolism)


1. Metabolic isturbance: hypophosphatemia (carbohyrate leas to insulin spike an subsequent
extracellular phosphate riven intracellular to prouce ATP), hypokalemia, hypocalcemia,
hypomagnesemia
2. Diagnosis: electrolyte abnormalities in malnourishe patient being fe; hypophosphatemia can lea
to respiratory istress (iaphragm nees lots of ATP)
3. Most common cause of mortality is cariac complication
4. Treatment: aggressive electrolyte repletion; slowly increase calorie intake

IV. Important formulas:


A. Anion gap: AG = (Na+ + K+) – (Cl− + HCO3−)
1. Abnormal is >10–15
. Aniongap: MUDPILES—methanol, uremia, iabetic ketoaciosis, paraalehye, isoniazi, lactic
aciosis, ethylene glycol, salicylates
3. Non–anion gap: iarrhea, nasogastric tube losses, high ileostomy output, small bowel stula
B. Osmolarity: Osm = ( × Na+) + (BUN/.8) + (Glucose/18)

V. Hyponatremia (Na+ <135 mEq/L)


A. Symptoms: lethargy, heaaches, nausea, vomiting, confusion, seizures
B. Diagnosis
1. Hypertonic (osmolality >90 mOsm/kg): ilutional rop in serum soium ue to osmotically active
molecules
a) Most often cause by hyperglycemia (can also occur with mannitol an contrast)
b) Soium correction: a  mEq/L of Na+ for each 100 mg/L the bloo glucose is above 100 mg/L
. Isotonic (osmolality 75–90 mOsm/kg): pseuohyponatremia
a) Laboratory error ue to over-ilution in the presence of abnormally high amounts of solute
b) Most common causes are hypertriglyceriemia, familial hypercholesterolemia, multiple myeloma
3. Hypotonic (osmolality <75 mOsm/kg): excess free water
a) Low urine osmolality (<85 mOsm/kg): antiiuretic hormone (ADH) inepenent
(1) Tea an toast/Beer potomania: ue to low solute intake
(a) Treatment: high solute iet
() Psychogenic polyipsia: excessive water intake
(a) Treatment: water restriction
b) High urine osmolality (>85 mOsm/kg): ADH epenent
(1) Hypovolemic: gastrointestinal losses, renal losses (iuretics, mineralocorticoi eciency),
thir-spacing
(a) Treatment: normal saline
() Euvolemic: SIADH, arenal insufciency, hypothyroiism
(a) Treatment: free water restriction, hypertonic saline, vaptans (SIADH)
(3) Hypervolemic: heart failure, cirrhosis, nephrotic synrome
(a) Treatment: water an Na+ restriction, iuresis
C. Treatment: Na+ shoul be correcte at a rate <8 to 1 mEq/L over 4 hours in orer to avoi osmotic
emyelination synrome (ODS) characterize by encephalopathy, seizures, ysphagia, neuromuscular
weakness, lethargy, tremors, an paralysis; severe case of ODS is “locke-in synrome”
1. Chronic hyponatremia, hypokalemia, liver isease, malnutrition, alcoholism are higher risk for ODS
an shoul be correcte <8 mEq/L over 4 hours; initial level of serum soium oes not necessarily
correlate with the evelopment of ODS

VI. Hypernatremia (Na+ > 145 mEq/L): water ecit an/or excess solute
A. Symptoms: restlessness, confusion, seizures
B. Diagnosis
1. Hypovolemic
a) High urine soium (>0 mEq/L): renal losses (iuretics, postobstructive, intrinsic renal isease)
b) Low urine soium (<0 mEq/L): extrarenal losses (burns, iarrhea, stulas)
CHAPtEr 29 Fluids, Electrolytes, and Acid-Base Balance 381

2. Euvolemic
a) Low urine osmolality (<300 mOsm/kg) iabetes insipius (nephrogenic or central)
b) High urine osmolality (>300 mOsm/kg) insensible losses, hypoipsia
3. Hypervolemic
a) High urine soium (>0 mEq/L): hyperalosteronism, Cushing synrome, exogenous soium
C. Treatment: Na+ shoul be correcte at a rate of <8 to 10 mEq/L over 4 hours to avoi cerebral eema
Na
1. Free water ecit = 0.6 × kg × ( −1)
140

VII. Other common electrolyte abnormalities

Electrolyte
abnormality Common causes Symptoms EKG ęndings Treatment
Hyperkalemia Renal failure, tissue Weakness with Peaked T waves, Calcium gluconate (stabilize
trauma, acidosis hyporeĚexia P wave cardiac membrane),
ĚaĴening, PR insulin + glucose
prolongation, and albuterol (shift
wide QRS intracellular), Kayexalate,
furosemide, dialysis
Hypokalemia Hydrochlorothiazide, Weakness, ileus FlaĴened/ Potassium supplementation
furosemide, inverted T
gastrointestinal wave, ST
losses depression,
U wave, QTc
prolonged
Hypercalcemia Cancer with bony Lethargy, nausea, Short ST, wide T Normal saline,
metastases, vomiting, wave Lasix, calcitonin,
multiple myeloma, hypotension bisphosphonates
hyperparathyroidism
Hypocalcemia After Perioral Prolonged ST, Calcium supplementation
parathyroidectomy numbness/ Long QTc
or thyroidectomy, tingling,
furosemide, hyperreĚexia,
pancreatitis, low Chvostek sign,
vitamin D Trousseau sign
Hypermagnesemia Renal failure, laxatives, Lethargy, Prolonged PR, Calcium, normal saline,
antacids areĚexia, QRS widening furosemide, dialysis
paralysis, com
Hypomagnesemia Diuresis, chronic TPN, Irritability, Tall T wave, ST Magnesium
EtOH abuse confusion, depression supplementation
hyperreĚexia,
tetany, seizures
Hyperphosphatemia Renal failure Muscle cramps, Prolonged QTc Sevelamer, low phosphate in
perioral tingling diet, dialysis
Hypophosphatemia Refeeding syndrome Muscle weakness, Various Phosphate supplementation
diĜculty
weaning oě
ventilator
382 PArt ii Medical Knowledge

QUESTIONS
1. A 34-year-ol G1P0 1-week pregnant female with 4. A 44-year-ol male with poorly controlle type 
a history of severe asthma requiring multiple prior iabetes mellitus presents with acute cholecystitis
hospitalizations is postoperative ay 1 from an an a bloo glucose of 800 mg/L, Na+ of
uncomplicate laparoscopic cholecystectomy. On 10 mEq/L, an anion gap of , an positive urine
morning rouns she is foun to be in moerate ketones. He is initiate on IV uis, an insulin
respiratory istress with accessory muscle use, rip, an antibiotics. What is true regaring his
iaphoresis, an tachypnea to 30 breaths per soium?
minute espite nebulize ipratropium an A. The correcte soium is 17 mEq/L
albuterol an IV hyrocortisone. She is saturating B. The soium shoul not be correcte faster
99% on L nasal canula. Her heart rate is 90 beats than 5 mEq/L in 4 hours
per minute. An arterial bloo gas emonstrates a C. The correcte soium is 134 mEq/L
pH of 7.40, PaO of 97, PaCO of 4, an HCO3 of D. It is impossible to correct for soium in the
4. What is the next best step in management? setting of ketones
A. CTA of the chest E. His hyponatremia is ue to glucose-inuce
B. Intravenous (IV) magnesium sulfate iuresis
C. Transfer to intensive care unit (ICU) for
observation 5. A 76-year-ol female is postoperative ay 4 from
D. Continue breathing treatments sigmoi colon resection. Her postoperative course
E. Intubation has been uneventful, but she has not yet starte
passing atus. Overnight, the urinary output has
2. A 16-year-ol girl arrives via ambulance after the ecrease to 0 cc/hour, an the patient has ha
family became concerne that she was behaving several episoes of emesis. Lab work inclues
strangely. She appears isoriente an will answer a bloo urea nitrogen (BUN) of 40 mg/L an
simple questions but is evasive in answering serum creatinine of 1.5 mg/L. Urinary soium
questions about events leaing up to her arrival. is 10 mEq/L. What is the most likely etiology of
Vital signs are normal except for a respiratory rate of oliguria in this patient?
7 an a boy mass inex (BMI) of 16. Arterial bloo A. Postoperative ileus
gas an basic metabolic panel are consistent with a B. Intraabominal hemorrhage
metabolic alkalosis. Which of the following tests will C. Intraoperative hypotension
be most helpful in establishing a iagnosis? D. Inavertent ligation of the left ureter
A. Urine rug screen E. Drug-inuce nephrotoxicity
B. Compute tomography of the brain
C. Spot urine chlorie concentration 6. Which of the following is true regaring soium
D. Electrocariogram (ECG) an water maintenance in the geriatric patient?
E. Abominal ultrasoun A. There is an increase in the ratio of intracellular
to extracellular water
3. A 64-year-ol female with a past meical history B. A hyperactive thirst response preisposes
of breast cancer with iffuse osseous metastases geriatric patients to hyponatremia
is amitte to the general surgery service for C. Elevate antiiuretic hormone levels
nonoperative management of small bowel preispose patients to soium retention
obstruction thought to be ue to ahesive isease. D. Atrial natriuretic peptie level increases with
Her amission labs emonstrate a calcium of aging
1.6 mg/L with an albumin of .0 g/L. She is E. There is a relative increase in the activity of the
given a normal saline bolus followe by a rip renin-angiotensin-alosterone system
at 00 mL/hr. IV calcitonin is also aministere.
What is the best next step in management?
A. Aminister IV furosemie
B. Aminister IV sevelamer
C. Convert to maintenance uis with lactate
Ringer’s
D. Aminister IV zoleronic aci
E. Aminister IV hyrocortisone
CHAPtEr 29 Fluids, Electrolytes, and Acid-Base Balance 383

7. A 50-year-ol type I iabetic male is amitte has roppe to 0 mL/hr. Delivere tial
to the hospital for the workup of vague volumes on the mechanical ventilator have also
abominal pain an malaise. Past meical signicantly ecrease. Physical exam reveals
history inclues total proctocolectomy with a tense abomen, abominal ui wave, an
ileostomy for ulcerative colitis. Routine laboratory anasarca. Current blaer pressure is 5 mm Hg.
values inclue: pH 7.6, pCO 4 mm Hg, pO The most appropriate initial management is:
100 mm Hg, soium 19 mEq/L, potassium A. Neuromuscular blockae
.9 mEq/L, chlorie 110 mEq/L, an bicarbonate B. Immeiate ecompressive laparotomy
1 mEq/L. Which of the following is the most C. Percutaneous rainage of intraabominal ui
likely iagnosis? D. Continuous renal replacement therapy
A. Excessive ileostomy output E. Change resuscitative ui to albumin
B. Kiney failure
C. Diabetic ketoaciosis 12. Which of the following is true regaring serum
D. Lactic aciosis osmolarity an serum osmolality?
E. Methanol intoxication A. Large proteins are the most important
contributors to serum osmolality
8. A 6-year-ol female was recently iagnose on B. The presence of an osmolar gap inicates the
upper enoscopy with a near obstructing istal presence of a foreign molecule that reaily
gastric tumor but was subsequently lost to follow- istributes across cell membranes
up. She now returns to the ED with 4 hours of C. The ifference between serum osmolarity an
nonbilious vomiting an abominal pain. What serum osmolality is highly variable epening
is the most signicant contributing factor to on the physiologic state
hypokalemia in this patient? D. Soium is multiplie by two in the calculation
A. Intracellular shift for serum osmolarity because of its increase
B. Increase excretion in the urine osmotic activity
C. Loss of potassium with emesis E. The number of molecules, an not the size,
D. Metabolic aciosis is the most important contributor to serum
E. Hypokalemic ui replacement osmolarity

9. Which of the following is consiere a normal 13. An elerly patient presents to the emergency
physiologic change in pregnancy? epartment (ED) with increase thirst an
A. Decrease in bloo pH urinary output. Which of the following nings
B. Decrease in minute ventilation woul be most helpful to suggest iabetes
C. Increase vital capacity insipius (DI) as the likely etiology in this
D. Right-shift of oxyhemoglobin issociation patient?
curve A. Hypernatremia
E. Relative leukopenia B. Hyperglycemia
C. Hyponatremia
10. Which of the following is true regaring D. Low urine osmolality
ehyration an/or hypovolemia in chilren? E. High serum-to-urine osmolality ratio
A. Chilren only nee to lose 5% of total boy
water to prouce signicant symptoms of 14. A 58-year-ol male alcoholic presents to the ED
hypovolemia complaining of increase abominal girth over
B. Hypovolemia refers to a reuction in free the last several weeks. He unerwent a iagnostic
water ultrasoun 1 year ago, which showe evience
C. Dehyration will primarily result in of cirrhosis. Physical exam reveals pitting eema
extracellular ui losses of the lower extremities an positive abominal
D. Profoun hypernatremic hypovolemia shoul ui wave. In aition to alcohol cessation, what
be correcte initially with hypotonic uis is the next step in management?
E. Oral ui replacement is aequate in most A. Free water restriction
chilren with insensible ui losses B. Transjugular intrahepatic portosystemic shunt
C. Intravenous furosemie with transition to PO
11. A 45-year-ol male with congestive heart failure once ascites resolves
is being treate in the ICU for sepsis seconary D. Strict soium restriction (<1 g/ay)
to pneumonia. Over the last 4 hours, he has E. Combination of oral furosemie an
receive 11 L of crystalloi an was starte on spironolactone
vasopressors for hypotension. His urine output
384 PArt ii Medical Knowledge

15. A 5-year-ol female is postoperative ay creatinine level increase from 1.0 to 1.6 mg/L.
1 from a laparoscopic, converte to open Urinalysis reveals no evience of proteinuria or
cholecystectomy for acute cholecystitis. Since microhematuria. Which of the following is the
surgery, she has ha one episoe of emesis, initial step in management?
urinary output has ecrease to 0.3 cc/kg A. Flui resuscitation with normal saline
per hour, an serum soium is foun to be B. Cessation of iuretics
131 mEq/L. Serum creatinine is normal, but C. Terlipressin an albumin
antiiuretic hormone (ADH) level is elevate. D. Initiation of continuous renal replacement
What is the most likely cause of these nings? therapy
A. Synrome of inappropriate antiiuretic E. Transjugular intrahepatic portosystemic shunt
hormone secretion (SIADH) (TIPS)
B. Normal physiologic response to surgery
C. Acute kiney injury 19. A 4-year-ol female unerwent a jejunal
D. Emesis resection complicate by abominal compartment
E. Congestive heart failure synrome an an open abomen after a motor
vehicle collision. She is eventually ischarge
16. A 65-year-ol male with massive intracranial home but returns 1 week later with copious
hemorrhage after a rupture intracranial output of yellowish ui from her miline
aneurysm is currently in the neurosurgical woun. She has note iminishe urinary output,
intensive care unit (ICU). Two ays ago, he is tachycaric, an has ecrease skin turgor.
unerwent intravascular coiling of the lesion. What combination of electrolyte abnormalities is
Because of increase urinary output over the most likely present in this patient?
last 4 hours, a urine soium was measure an A. Hyponatremia, hypokalemia, an metabolic
foun to be 35 mEq/L. Current labs inclue a aciosis
serum soium of 18 mEq/L an a hemoglobin B. Hypokalemia, hypochloremia, an metabolic
of 18 g/L. Central venous pressure is  mm Hg. alkalosis
Which of the following is the most appropriate C. Hyponatremia, hyperkalemia, an metabolic
initial treatment? aciosis
A. Normal saline D. Hypernatremia an metabolic aciosis
B. Free water restriction E. Hyperkalemia an metabolic alkalosis
C. Desmopressin
D. Demeclocycline 20. A 55-year-ol male is amitte to the hospital
E. Tolvaptan with altere mental status. Parameics report
that they foun multiple empty beer cans in
17. A 75-year-ol male is in the ICU ue to sepsis his home. He is foun to have a serum alcohol
5 ays after a colectomy for a perforate concentration of 55 mg/L an a serum soium
iverticulitis. While the nurse is checking his concentration of 118 mEq/L. Flui resuscitation
bloo pressure, his han went into a spasm. is initiate with normal saline an soium
Which of the following is the most likely etiology? levels return to normal by the next morning. On
A. Hypercalcemia hospital ay 5, he evelops spastic quariplegia
B. Hypermagnesemia an is unresponsive to external stimuli. Which of
C. Hypomagnesemia the following is true regaring this conition?
D. Hyponatremia A. It coul have been prevente with the use of
E. Hyperkalemia hypertonic saline
B. Desmopressin can be use as an ajunct to
18. A 48-year-ol male with past meical history ui replacement to prevent this complication
of alcoholic cirrhosis an refractory ascites is C. Cerebral aaptions to hyponatremia take up to
amitte to the ICU recovering from spontaneous a week to evelop
bacterial peritonitis (SBP). He is now off the D. Recovery is impossible after the onset of
antibiotics, an there is no evience of continue neurologic symptoms
infection. Over the course of his hospitalization, his E. Injury is restricte to the pons
CHAPtEr 29 Fluids, Electrolytes, and Acid-Base Balance 385

ANSWERS
1. E. This pregnant patient is experiencing an acute asthma will be associate with a respiratory aciosis, not a metabolic
exacerbation with acute respiratory failure in the postop- alkalosis (A). Electrocariogram may show signs of hypoka-
erative setting. While the patient is oxygenating well as lemia, but that is a common ning in metabolic alkalosis
inicate by her PaO an pulse oximetry, her bloo gas is (D). Compute tomography an abominal ultrasoun may
concerning for impening hypercarbic respiratory failure. be useful in the workup of altere mental status, but laxative
As a result of a progesterone-inuce increase in alveolar abuse or self-inuce vomiting is a much more likely iag-
ventilation uring pregnancy, arterial PCO typically falls nosis in this scenario (B, E).
to a plateau of 7 to 3 mm Hg. Furthermore, uring acute
asthma exacerbations, respiratory rive increases resulting 3. D. This patient presents with severe hypercalcemia sec-
in hyperventilation an ecrease PaCO. As such, a PaCO onary to malignancy. While this patient rst appears to have
of 4 in this patient inicates airway narrowing an ynamic moerate hypercalcemia, ene as calcium between 1 an
hyperination so severe that alveolar ventilation has alreay 14 mg/L, after correcting for low serum albumin, her cal-
ecrease espite increase respiratory rive. Given this cium corrects to 14.4, which is consiere severe, an therefore
clinical picture, the patient shoul be intubate for manage- prompts aggressive treatment. Hypercalcemia of malignancy
ment of her hypercarbic respiratory failure an exhaustion presents by three primary mechanisms: (1) tumor secretion of
as inicate by her use of accessory muscles of respiration, parathyroi hormone-relate protein (PTHrP), () osteolytic
iaphoresis, an tachypnea on physical exam. The rst pri- metastases, an (3) tumor prouction of 1,5 ihyroxyvita-
ority in this patient shoul be to establish a safe airway an min D. In up to 80% of cases, hypercalcemia of malignancy
begin mechanical ventilation to reuce the patient’s work of is seconary to excretion of PTHrP. In approximately 0% of
breathing. Furthermore, this can provie a valuable brige cases, osteolytic metastases are responsible for hypercalcemia.
while waiting for bronchoilators an glucocorticoi mei- Breast cancer is known to cause hypercalcemia through both
cations to reuce airway swelling. It woul be unsafe to only mechanisms. Hypercalcemic patients are often ehyrate,
continue breathing treatments or aminister IV magnesium since the hypercalcemic state impairs the kiney’s ability to
which may not reverse airway eema in a timely enough concentrate urine. Initial treatment is via a normal saline bolus
manner to account for this patient’s ecompensating respira- followe by a 00 cc/hr rip, which shoul be titrate for a
tory status (B, D). Pulmonary embolism is unlikely given no urine output of 1 to  mL/kg/hr. Lactate Ringer’s shoul
evience of hypoxia or tachycaria (A). This patient shoul be avoie ue to their calcium content (C). Once the patient
be transferre to the ICU, but only after aressing her respi- is renere euvolemic, ajunct agents can be ae. Calci-
ratory failure with intubation (C). tonin reuces osteoclast activity an, in turn, ecreases serum
Reference: Brenner B, Corbrige T, Kazzi A. Intubation an calcium. Often volume expansion an calcitonin are all that
mechanical ventilation of the asthmatic patient in respiratory failure. is neee an can reuce serum calcium in 1 to 4 hours.
Proc Am Thorac Soc. 009;6(4):371–379. However, in this case, the hypercalcemia is likely a long-term
problem. As such, the concurrent aministration of bisphos-
2. C. Severe metabolic alkalosis leas to hypoventilation phonates is encourage, particularly for severe cases of hyper-
ue to inhibition of the respiratory center in the meulla. calcemia of malignancy. Multiple ranomize, controlle
The etiology of metabolic alkalosis is generally clear from trials support the fact that bisphosphonates (zoleronate or
history (excessive emesis, iuretic use) alone. However, in pamironate) are potent an relatively safe meications for
scenarios where the patient is unable, or unwilling, to pro- the treatment of moerate to severe hypercalcemia of malig-
vie a history (such as bulimia), the measurement of urine nancy. It shoul be note that this rug takes ays to rener its
chlorie concentration can provie important iagnostic effect. Glucocorticois are not emonstrate to have benet in
information. When metabolic alkalosis is associate with the treatment of hypercalcemia of malignancy (E). Sevelamer,
hypovolemia, the urine chlorie concentration will be a phosphate biner, is not inicate for use in hypercalcemia
appropriately low (<0 mEq/L) in response to the corre- of malignancy (B). Furosemie has not been emonstrate as
sponing hypochloremia an volume contraction. Examples an effective meication for management of hypercalcemia of
of chlorie-responsive metabolic alkalosis inclue excessive malignancy. However, loop iuretics o cause calciuresis, but
vomiting or laxative abuse, such as in anorexia-nervosa or they inuce ui loss, an their utility is therefore more lim-
bulimia-nervosa. Diuretic use is another common etiology, ite (A).
though recent use will increase the urine chlorie concen- References: LeGran S, Leskuski D, Zama I. Narrative review:
tration. Chlorie unresponsive metabolic alkalosis (urine furosemie for hypercalcemia: an unproven yet common practice.
chlorie concentration >0 mEq/L) can be associate with Ann Intern Med. 008;149(4):59–63.
hypervolemia in the setting of excessive mineralocorticoi Major P, Lortholary A, Hon J, et al. Zoleronic aci is superior
concentrations (primary alosteronism) or conitions that to pamironate in the treatment of hypercalcemia of malignancy: a
mimic mineralocorticoi excess (licorice ingestion). Disor- poole analysis of two ranomize, controlle clinical trials. J Clin
ers that lea to increase urinary salt wasting (Bartter or Oncol. 001;19():558–567.
Gitelman synrome) will also be chlorie unresponsive but
will be associate with hypovolemia. Most rugs of abuse 4. C. This patient presents with iabetic ketoaciosis as
that woul lea to altere mental status an hypoventilation emonstrate by his elevate bloo glucose, increase anion
386 PArt ii Medical Knowledge

gap, an positive urine ketones. Aitionally, he presents contributes to renal salt an water wasting. The renin-angio-
with hypertonic hyponatremia with a measure serum tensin-alosterone system is also suppresse, leaing to ys-
soium of 10 mEq/L in the setting of hyperglycemia. The regulation of soium an potassium balance (E).
presence of elevate extracellular glucose results in increase References: El-Sharkawy AM, Sahota O, Maughan RJ, Lobo DN.
serum tonicity an the shift of water from the intracellular The pathophysiology of ui an electrolyte balance in the oler
to the extracellular space, thereby lowering serum soium ault surgical patient. Clin Nutr. 014;33(1):6–13.
concentration. The calculation to correct for this is to a Miller M. Disorers of ui balance. In: Halter JB, Ouslaner JG,
 mEq/L of Na+ for each 100 mg/L the bloo glucose is Tinetti ME, Stuenski S, High KP, Asthana S, es. Hazzard’s geriatric
medicine and gerontology. 6th e. McGraw-Hill; 009.
above 100 mg/L. As such, this patient’s serum soium cor-
rects to 134 mEq/L (A). Clinician must be extremely iligent
in correcting serum soium in a controlle manner (less than
7. A. A low pH with a corresponing low pCO an low
bicarbonate is inicative of a metabolic aciosis. Calculation
8–1 mEq/L per 4 hrs) so as to not cause osmotic emye-
lination synrome (B). Ketones o not affect the correction of the anion gap [19 (Na)–110 (Cl) – 1 (HCO3)] reveals a
of serum soium in the setting of hyperglycemia (D). Glu- value of 7, which is consistent with a non-anion gap met-
cosuria can contribute to signicant total boy epletion of abolic aciosis (normal 8–16). The patient’s history of total
soium an potassium; however, in this case the soium cor- proctocolectomy an non–anion gap metabolic aciosis is
rects to a normal value an thus is more likely to represent consistent with gastrointestinal (GI) losses from excessive
hypertonic hyponatremia (E). ileostomy output. All of the other answer choices liste will
Reference: Emmett M, Sterns RH. Flui, electrolyte, an aci- contribute to an anion gap (B–E).
base isturbances. J Am Soc Nephrol. 013;1:191.
8. B. Gastric outlet obstruction an large volume emesis
5. A. The rst step in ientifying the etiology of oliguria is result in signicant volume loss in aition to hyrogen an
an aequate history an analysis of the BUN:creatinine ratio. chlorie ions. Though gastric juice has a higher concentration
A BUN:creatinine ratio of greater than 0 with a history of of potassium than serum, at 10 mEq/L, the overall potassium
hypoperfusion or hypotension is virtually iagnostic of pre- content is low an relatively insignicant compare with
renal azotemia. However, no such history is provie in this the loss of hyrogen an chlorie (C). This subsequently
vignette. At this point, urinalysis is necessary. A low urinary leas to a hypochloremic metabolic alkalosis, not an aciosis
soium concentration (<0 mEq/L) or a low fractional excre- (D). The volume epletion, initially, is counteracte by the
tion of soium (<1%) is inicative of a prerenal cause of acute mobilization of extravascular uis so the kiney maintains
kiney injury. In the presence of emesis an failure to pass a relatively constant ow. Initially, the kiney respons by
atus, ileus is the most likely iagnosis (A). Ileus or small excreting the excess bicarbonate in the urine in combination
bowel obstruction can lea to signicant intraabominal with soium an potassium to balance the negative charge.
ui sequestration that, without aequate ui resuscita- However, as more soium is lost an hypovolemia becomes
tion, ecreases renal bloo ow an subsequently urinary more apparent, the renin-angiotensin-alosterone system is
output. The low urinary soium is the result of physiologi- activate. This increases the absorption of soium an water,
cally elevate ADH seconary to the hypovolemia. Though but potassium continues to be excrete, leaing to hypoka-
intraabominal hemorrhage woul lea to a similar clinical lemia. Eventually, the kiney will begin to compensate for
picture, bleeing is more common earlier in the postopera- the hypokalemia by exchanging potassium ions for hyro-
tive perio (POD 0-1) (B). You woul also expect the conse- gen ions, which perpetuates the alkalosis an causes the
quences of intraoperative hypotension to present earlier (C). paraoxical aciuria associate with excessive loss of gastric
Drug-inuce nephrotoxicity is an intrinsic acute kiney contents. Though alkalosis oes cause an intracellular shift
injury, an urinary soium woul not be low (E). Inaver- of potassium ions, the effect is variable an oes not account
tent ligation of the ureter typically oes not present with oli- for the signicant hypokalemia seen with metabolic alkalosis
guria unless both sies are affecte (D). (A). Before replacement of the hypokalemia, volume expan-
sion with crystalloi is recommene, which will reuce the
6. D. Numerous physiologic changes associate with effects of alosterone an potassium loss in the urine (E).
aging iminish the geriatric population’s ability to aapt to References: Aronson PS, Giebisch G. Effects of pH on potas-
changes in the environment or health, especially regaring sium: new explanations for ol observations. J Am Soc Nephrol.
the maintenance of water an electrolyte balance. Loss of 011;(11):1981–1989.
lean boy mass ecreases total boy water an ecreases the Lee Hamm L, Hering-Smith KS, Nakhoul NL. Aci-base an
ratio of intracellular to extracellular water (A). This results in potassium homeostasis. Semin Nephrol. 013;33(3):57–64.
a iminishe ability to respon to ui losses because there
is less water to mobilize from the intracellular space. In ai- 9. D. Pregnancy causes a number of physiologic changes,
tion, the oler population has a iminishe thirst response either to improve conitions for the eveloping fetus or as a
to changes in serum osmolality (B). The kiney itself also sie effect of the increase metabolic emans place on the
unergoes structural changes that result in a iminishe glo- mother. From a respiratory stanpoint, the changes are pri-
merular ltration rate, which ecreases the kineys’ ability marily relate to the increase prouction of progesterone
to ilute urine in response to a water loa. However, the ki- an the mass effect of the uterus on the iaphragm. Proges-
ney also shows a iminishe ability to concentrate the urine terone acts on the central nervous system to lower CO levels.
in response to ehyration. This is partly ue to reuce In an effort to lower pCO, the tial volume an respiratory
responsiveness to ADH in the age kiney (C). On the other rate increase, causing an increase in minute ventilation (B).
han, atrial natriuretic peptie levels increase, which further This reuction in the pCO causes a respiratory alkalosis
CHAPtEr 29 Fluids, Electrolytes, and Acid-Base Balance 387

(A). The mass effect from the uterus causes a reuction in pressures or ecrease tial volumes in a pressure moe of
inspiratory an expiratory reserve, as well as functional an ventilation. It is further subivie into primary an secon-
resiual capacity. However, vital capacity remains relatively ary epening on the etiology. Primary abominal compart-
unchange (C). The increase metabolic emans require ment synrome refers to etiologies that arise in the abomen
an increase in oxygen elivery. This is accomplishe by an (such as volvulus or colonic pseuo obstruction), an current
increase in cariac output. The total bloo volume increases recommenations are for immeiate ecompressive laparot-
proportionally to the cariac output, but the increase in omy (B). However, in seconary abominal compartment
plasma volume is greater than the increase in re bloo cell synrome, such as cirrhotics or patients with congestive heart
mass, which causes ilutional anemia. The increase cariac failure with tense ascites, nonsurgical treatments can rst be
output proportionally increases the glomerular ow rate attempte. In 011, Cheatham an others treate abomi-
of the kiney an reuces circulating urea. The oxyhemo- nal compartment synrome from ascites with percutaneous
globin issociation curve also shifts to the right to facilitate rainage, an 81% of stuy participants were successfully
unloaing of oxygen to the fetus. In aition to the increase treate without a ecompressive laparotomy (C). While the
afnity of fetal hemoglobin for oxygen, there is an increase in most recent consensus guielines release by the Worl Soci-
,3-DPG, which further facilitates elivery of oxygen to the ety of the Abominal Compartment Synrome still avocate
fetus. There is a mil reuction in platelets, likely because surgical intervention for abominal compartment synrome,
of increase platelet aggregation from hypercoagulability. they also maintain that the use of percutaneous catheter
However, there is an increase in circulating white bloo rainage for the treatment of obvious intraperitoneal ui
cells (E). contributing to abominal compartment synrome shoul
be use in place of ecompressive laparotomy when it is
10. E. Though frequently use interchangeably, ehyra- technically feasible because it may alleviate the nee for sur-
tion an hypovolemia are separate clinical entities. Dehyra- gery. While neuromuscular blockae an iuresis may help
tion refers to a reuction in free water (ui loss in excess of with the treatment of intraabominal hypertension, worsen-
solute loss), while hypovolemia is a loss of circulating extra- ing kiney an lung function require immeiate interven-
cellular volume (B). This is an important istinction because tion (A, D). The role of albumin in abominal compartment
of the istribution of total boy water. Two-thirs of total synrome is still controversial (E).
boy water is intracellular, which means that ehyration References: Cheatham M, Safcsak K. Percutaneous catheter
will primarily result in intracellular ui losses (C). In fact, ecompression in the treatment of elevate intraabominal pres-
almost 10% of total boy water nees to be lost before sig- sure. Chest. 011;140(6):148–1435.
Kirkpatrick AW, Roberts DJ, De Waele J, et al. Intra-abominal
nicant signs of hypovolemia manifest. Hypovolemia from
hypertension an the abominal compartment synrome: upate
ehyration is relatively rare in people with access to water
consensus enitions an clinical practice guielines from the
because the increase in plasma osmolality stimulates a strong Worl Society of the Abominal Compartment Synrome. Intensive
thirst response, which is why it typically only presents when Care Med. 013;39(7):1190–106.
people are reliant on others (chilren an the elerly) (A).
In hypovolemia, the serum soium will correspon with 12. E. Osmolarity an osmolality represent the number
the type of ui lost an any prehospital replacement that of osmotically active solutes (osmoles) in a given solution.
has taken place. Insensible losses, such as sweating, will Osmolarity is the number of osmoles in a liter of solution,
result in hypernatremia because the ui lost is hypotonic an osmolality is the number of osmoles in a kg of water.
to plasma an increases in ADH will result in soium an Because the volume of a solution can vary slightly epen-
water retention. Secretory iarrhea or bleeing, on the other ing on temperature, osmolality is technically more precise,
han, results in ui losses that are isotonic to plasma an but uner normal physiologic conitions the terms are essen-
on’t have a irect effect on serum soium levels. However, tially interchangeable because 1 L of water weighs 1 kg (C).
replacement of these losses with hypotonic uis will lea to Because the kinetic energy of issolve solutes is base on the
hyponatremia. Profoun hypernatremic hypovolemia man- number, an not the size, large proteins like albumin have a
ates rapi intravascular volume replacement with intra- relatively low contribution compare to more abunant mol-
venous isotonic uis. After the severe volume epletion ecules, like soium (A). In orer to contribute osmotic pres-
is treate, the replacement of the free water ecit can take sure across a semipermeable membrane, the issolve solute
place more slowly. Care shoul be taken to avoi rapi cor- must not be able to reaily iffuse across the membrane.
rection of hypernatremia because it can precipitate cerebral Thus, a foreign molecule that reaily istributes intracellu-
eema (D). Unless there are irect contrainications, such as larly oes not contribute to serum osmolality or an osmolar
altere mental status or vascular compromise, oral replace- gap (B). The equation for the calculation of serum osmolality
ment therapy is likely aequate an is the preferre replace- is  [Na] + [glucose]/18 + [BUN]/.8. Soium is multiplie
ment strategy by the American Acaemy of Peiatrics. by two to account for the corresponing anions (chlorie an
Reference: Spanorfer PR, Alessanrini EA, Joffe MD, Localio bicarbonate) that woul otherwise nee to be ae sepa-
R, Shaw KN. Oral versus intravenous rehyration of moerately rately (D). Serum osmolality can also be irectly measure,
ehyrate chilren: a ranomize, controlle trial. Pediatrics.
normally by freezing point epression, an compare to the
005;115():95–301.
calculate value. If there is a signicant ifference between
the calculate an measure serum osmolality, it inicates the
11. C. Abominal compartment synrome is ene as a presence of an osmotically active foreign solute, like methanol.
sustaine intraabominal pressure of greater than 0 mm Hg
associate with new-onset organ failure. Early clinical signs 13. E. DI is a isease process characterize by either a low
are oliguric acute kiney injury an increase peak airway level of ADH (central DI) or iminishe renal response to
388 PArt ii Medical Knowledge

ADH (nephrogenic DI). The rst step in the evaluation of References: Runyon BA. Management of adult patients with asci-
polyuria is the measurement of serum electrolytes, serum tes due to cirrhosis: update 2012. AASLD Practice Guideline. American
glucose, an urine an serum osmolality. In the absence of Association for the Stuy of Liver Diseases; 01. https://www.
osmotic iuresis from hyperglycemia (i.e., iabetes melli- aasl.org/sites/efault/files/019-06/14100_Guieline_Asci-
tes_4UFb_015.pf.
tus), primary polyipsia, central DI, an nephrogenic DI are
Wong P, Price JC, Herlong H. Cirrhosis an its complications.
the most common etiologies (B). All three entities will show
In: McKean SC, Ross JJ, Dressler DD, Brotman DJ, Ginsberg JS, es.
increase prouction of ilute urine, or low urine osmolal- Principles and practice of hospital medicine. McGraw-Hill; 01.
ity (D). However, in primary polyipsia, this is a normal
response to increase water intake. Serum soium levels will 15. B. After a major operation, there is both an enocrine
generally be low because of the increase intake of water. an a cytokine response to the stress. This can be partly inhib-
Increase urinary output with hyponatremia, low urine ite by blocking painful stimuli from reaching the central
osmolality, an low or normal serum osmolality is virtually nervous system, but it is also meiate by the effects of local
iagnostic (C). Diabetes insipius, on the other han, will be tissue amage. Of the numerous physiologic responses, the
associate with low urine osmolality in the presence of ele- retention of soium an water is likely the most signicant.
vate serum osmolality. Though hypernatremia is possible This is epenent on multiple factors, incluing the effects of
because of the excessive loss of water in the urine, in general, anesthetic rugs, renal vasoconstriction from catecholamines,
patients are able to compensate for the increase urinary out- increase plasma cortisol an alosterone, an increase
put with increase oral intake of water (A). Suspicions can be secretion of antiiuretic hormone (ADH). During an opera-
conrme with a water eprivation test. In primary polyip- tion, ADH levels will increase up to 100 × normal. Though
sia, the urinary output will ecrease an the urine osmolal- they begin to rop at the en of the operation, they remain
ity will increase as the test progresses because the stimulus elevate for several ays. This response is largely seconary
for the polyuria has been remove. However, patients with to the loss of intravascular volume by sequestration in injure
DI lack the ability to concentrate urine, so the prouction of tissues, or “thir-spacing,” ehyration from prolonge fast-
ilute urine will continue, espite rising serum osmolality. ing, an insensible losses uring the operation. This results
Once the patient’s serum osmolality increases to a sufcient in postoperative oliguria an hyponatremia. In this setting,
level, the aministration of vasopressin will ifferentiate the elevate level of ADH is not “inappropriate”; instea,
between nephrogenic an central DI. In central DI, the vaso- it is a normal physiologic response to stress an ecrease
pressin will allow the kineys to concentrate the urine. In intravascular volume. In critically ill patients, the ADH level
nephrogenic DI, no response to exogenous vasopressin will may get inappropriately high ue to ysregulation of the
be expecte because the problem is the kiney’s response to, hypothalamus-pituitary axis, resulting in SIADH. This subse-
not the absence of, ADH. quently leas to secretion of the natriuretic pepties to inuce
loss of soium an water, resulting in a euvolemic state.
14. E. The mobilization of ascites in cirrhotic patients Aitionally, the loss of soium is much greater than that
requires a negative soium balance. This is accomplishe of water, such that patients with SIADH have a signicantly
through limiting oral intake of soium an initiating lower level of soium compare with the mil hyponatremia
iuresis. In the absence of signicant hyponatremia (<15 seen postoperatively (A). By enition, this patient cannot
mEq/L), free water restriction is generally not inicate have acute kiney injury with a normal creatinine clearance
(A). The problem lies in the inappropriate retention of (C). Excess vomiting can present as hyponatremia, but a sin-
soium by the kiney, not excess free water. Diuresis shoul gle episoe of emesis is unlikely to prouce this effect (D).
be initiate with an initial goal of negative 1 L/ay, though Congestive heart failure is an unlikely cause of hyponatremia
500 mL/ay is likely aequate in the absence of peripheral without other associate symptoms (E).
eema. Oral spironolactone an furosemie shoul be ini- Reference: Rassam SS, Counsell DJ. Perioperative electrolyte an
tiate at an initial ose of 100 mg an 40 mg, respectively, ui balance. Contin Educ Anaesth Crit Care Pain. 005;5(5):157–160.
per ay. These can be increase to a maximum aily ose
of 400 mg spironolactone an 160 mg furosemie. Simul- 16. A. In a neurologically injure patient with hyponatre-
taneous aministration of these two meications potenti- mia an elevate urinary soium, the two most likely iag-
ates the natriuretic effect of each an limits the potassium noses are SIADH or an isolate natriuresis from elevate
imbalance that can be seen with either agent alone. Unlike atrial natriuretic peptie (cerebral salt wasting synrome).
ascites seconary to heart failure, intravenous aministra- SIADH can have a natriuresis component as escribe in
tion of iuretics in cirrhotics with new-onset ascites shoul question 1. Though they have similar laboratory nings,
generally be avoie because it can frequently result in the hyponatremia in cerebral salt wasting is cause by exces-
azotemia (C). While strict soium restriction will result sive urinary losses of soium as oppose to excess water
in faster mobilization of ascites, the iet is more ifcult retention with SIADH. This means that the only measurable
to ahere to an can potentially worsen any malnutrition ifference between SIADH an cerebral salt wasting is the
that is present; a soium restriction of less than  g/ay intravascular volume status of the patient; hypovolemia for
is generally all that is require (D). All patients shoul be the latter, an euvolemia or hypervolemia for the former.
consiere for liver transplant because the onset of asci- Cerebral salt wasting is classically escribe as a patient
tes is associate with a signicantly worsene prognosis. with a subarachnoi hemorrhage an a suen increase in
Patients with ascites refractory to iuretics can be consi- urine output, which leas to hyponatremia an hypovole-
ere for serial paracentesis or portosystemic shunt. Trans- mia. The cause of cerebral salt wasting synrome has not
jugular intrahepatic portosystemic shunt is preferre over been completely characterize, an it is unclear whether
surgical shunts (B). natriuretic factors are release from the brain or are simply a
CHAPtEr 29 Fluids, Electrolytes, and Acid-Base Balance 389

ownstream consequences of hormonal effects from the brain common cause of renal failure in patients with cirrhosis is
injury. The propose theoretic mechanism is excessive release prerenal azotemia, so the cessation of iuretics an volume
of atrial natriuretic peptie (ANP) from the cariac myocytes expansion with human albumin an not normal saline (A) is
in the right atrium. However, there are some authors who the initial step when acute kiney injury is suspecte. Failure
argue that this is simply a manifestation of SIADH because to respon to these measures raises concern for the hepato-
ANP levels will naturally rise to counteract the effects of ADH. renal synrome. The current iagnostic criteria for hepatore-
Regarless, the low CVP an elevate hemoglobin in this nal system inclue: cirrhosis with ascites, serum creatinine
patient inicate a reuction in intravascular volume, which greater than 1.5 mg/L, no improvement in serum creati-
shoul be replace with normal saline. Flui restriction, in an nine after at least  ays of iuretic withrawal an volume
attempt to treat SIADH, coul potentially cause worsening expansion with albumin, absence of shock, no current or
cerebral ischemia (B). Desmopressin is an ADH analogue use recent treatment with nephrotoxic rugs, an the absence
to treat central iabetes insipius (inaequate prouction of of parenchymal kiney isease (no proteinuria, no microhe-
ADH), which is characterize by excessive output of ilute maturia, an a normal renal ultrasoun). In aition, urine
urine an normal to high plasma soium (C). Demeclocycline soium is very low (<10 mEq/L). The most important phys-
is a tetracycline antibiotic that blocks the responsiveness of iologic change in hepatorenal synrome is splanchnic vaso-
the renal collecting tubules to ADH; it is use off-label as an ilation, which causes a cascae effect resulting in increase
ajunct to treat SIADH that is unresponsive to ui restriction sympathetic nerve activity, increase activity of the renin-an-
(D). “Vaptans” are a category of meications that function as giotensin system, increase nonosmotic vasopressin release,
vasopressin receptor antagonists an have also been use to renal vasoconstriction, abolishe autoregulation of the ki-
treat SIADH (E). ney, activation of the hepatorenal reex, an a ecrease in
References: Robinson AG. The posterior pituitary (neurohy- renal bloo ow. Treatment epens on the severity of ill-
pophysis). In: Garner DG, Shoback D, es. Greenspan’s basic & clini- ness an whether or not the patient is in the ICU. In the crit-
cal endocrinology. 9th e. McGraw-Hill; 011. ically ill, treatment with albumin an norepinephrine can be
Ropper AH. The hypothalamus an neuroenocrine isorers. initiate. In the non–critically ill, terlipressin (a vasopressor
In: Ropper AH, Samuels MA, Klein JP, es. Adams & Victor’s princi-
analogue) an albumin volume expansion have shown the
ples of neurology. 10th e. McGraw-Hill; 014.
greatest incience of renal recovery (C). However, in coun-
tries where terlipressin is unavailable, like the Unite States,
17. C. Hypomagnesemia is one of the most common elec- therapy can be initiate with miorine an octreotie. The
trolyte abnormalities in hospitalize patients (11%–65%) an ieal treatment is liver transplantation but is limite by avail-
particularly in critically ill patients. Most patients are asymp- ability. Dialysis or renal replacement therapy shoul only be
tomatic but can become symptomatic as the level rops use as a brige to transplant because it hasn’t been shown
below 1. mg/L. Symptoms can manifest as simple neu- to ecrease mortality or improve renal recovery (D). TIPS can
romuscular irritability, as emonstrate above by the pres- be consiere in patients with refractory ascites, but its role
ence of Trousseau sign (spasm of the forearm an han with in the treatment of hepatorenal synrome is unclear (E).
occlusion of the brachial artery) or, in more serious cases, as References: Israelsen M, Gluu L, Kraq A. Acute kiney injury
tetany, nystagmus, an seizures. Depletion of magnesium an hepatorenal synrome in cirrhosis. J Gastroenterol Hepatol.
also leas to both atrial an ventricular arrhythmias. How- 015;30():36–43.
ever, hypomagnesemia commonly presents in the presence Lenz K, Buer R, Kapun L, Voglmayr M. Treatment an manage-
of other electrolyte eciencies, an the iniviual contribu- ment of ascites an hepatorenal synrome: an upate. Therap Adv
tion of magnesium is often ifcult to etermine. Replace- Gastroenterol. 015;8():83–100.
ment therapy for symptomatic magnesium eciency is Runyon BA. Management of adult patients with ascites due to cir-
manatory, but the treatment of asymptomatic hypomag- rhosis: update 2012. AASLD Practice Guideline. American Association
for the Stuy of Liver Diseases; 01. https://www.aasl.org/sites/
nesemia is less well ene. Rubeiz etal. showe increase
efault/les/019-06/14100_Guieline_Ascites_4UFb_015.pf.
mortality in patients with hypomagnesemia on amission
to the meical ICU or war. Similarly, a review article pub- 19. A. The corresponing electrolyte abnormalities seen
lishe in the Journal of Clinical Meicine Research, which with hypovolemia are heavily epenent on the composi-
inclue 0 ifferent stuies, showe a correlation between tion of the corresponing secretions that are lost. Because of
low magnesium levels an increase averse outcomes an the relatively higher concentration of bicarbonate an potas-
mortality in patients with sepsis. Hypercalcemia, hypermag- sium in small bowel an pancreatic secretions, it is common
nesemia, hyponatremia, an hyperkalemia woul not pres- for excessive losses to result in hypokalemia an metabolic
ent with neuromuscular irritability (A, B, D, E). aciosis. The soium content is generally isotonic, or even
References: McEvoy C, Murray PT. Electrolyte isorers in crit- slightly hypotonic, to plasma. However, patients with an
ical care. In: Hall JB, Schmit GA, Kress JP. es. Principles of critical
intact thirst mechanism will typically replace uis with
care. 4th e. McGraw-Hill; 014.
Rubiez G, Thill-Baharozian M, Harie D. Association of hypo-
free water, making hyponatremia much more common on
magnesemia an mortality in acutely ill meical patients. Critical presentation. Stomach secretions are high in hyrogen an
Care Medicine. 1993;1():03–09. chlorie, which results in a hypochloremic metabolic alkalo-
Velissaris D, Karamouzos V, Pierrakos C, et al., Hypomagne- sis. The renal response to these losses results in hypokalemia
semia in critically ill sepsis patients. J Clin Med Res. 015;7(1): (B). The highest concentration of potassium in any gastro-
911–918. intestinal secretion is saliva, followe by the large intestine.
Excessive losses of these uis frequently present with hypo-
18. B. More than 50% of patients with cirrhosis an renal kalemia. Sweat is typically hypotonic to plasma an effec-
failure will ie within 1 month of the iagnosis. The most tively results in free water loss, though the sweat glan’s
390 PArt ii Medical Knowledge

ability to absorb soium oes iminish as output increases. it has now been escribe in other areas of the brain as well
If oral intake is inaequate, this can lea to a hypernatremic (E). While some recovery has been escribe weeks after
metabolic aciosis (D). Hyponatremia, hyperkalemia, an the onset of neurologic symptoms an there has been some
mil metabolic aciosis can be seen in arenal insufciency ata to support reinstitution of hyponatremia to improve
(C). Hyperkalemia is not typically seen in conjunction with prognosis (D), prevention is the mainstay of treatment. This
metabolic alkalosis because the renal response to alkalosis involves slow correction of chronic or unknown chronicity
causes the wasting of potassium in the urine (E). hyponatremia by no more than 9 mEq/L per ay. In cases
of associate hypovolemia, volume replacement can remove
20. B. Osmotic emyelination synrome (ODS), formally the stimulus for ADH release, resulting in free water iuresis
known as central pontine myelinolysis, is a conition brought an an increase rate of soium correction. For this reason,
on by a change in serum osmolality classically escribe esmopressin has been avocate for use in this scenario to
with the rapi correction of chronic hyponatremia. Chronic allow the prouction of more concentrate urine an prevent
hyponatremia results in the loss of osmotically active solutes rapi autocorrection of soium. The use of hypertonic saline
an water from brain cells, which protects against cerebral is generally unnecessary unless the cause of hyponatremia is
eema. This process starts with the initiation of hyponatremia clearly acute by history an there are signs of cerebral eema
an is generally completely in place by 48 hours (C), which or elevate intracranial pressures (A). As one might expect,
is why hyponatremia that evelops over this time perio is cerebral mechanisms eal with chronic hypernatremia by
generally not associate with signicant symptoms. While increasing the concentration of these same osmotically active
the exact mechanism is unknown, stuies in animals have solutes an rapi correction can result in cerebral eema.
shown that the areas of the brain that are slowest at replac- Reference: Mount DB. Flui an electrolyte isturbances. In:
ing the lost solutes are the most likely to unergo emyelin- Kasper D, Fauci A, Hauser S, Longo D, Jameson J, Loscalzo, es.
ation. This process was originally escribe in the pons, but Harrison’s principles of internal medicine. 19th e. McGraw Hill; 014.
Immunology
KRISTOFER E. NAVA AND SAAD SHEBRAIN 30
ABSITE 99th Percentile High-Yields
I. Innate Immunity
A. Cells
1. Phagocytes—enrites, macrophages, neutrophils
. Mast cells, eosinophils, basophils (mast cell is the most important cell involved in anaphylaxis)
3. Complement
a) Classic pathway—activate by antigen-antiboy complex (activation of classic complement
pathway: IgM > IgG); factors C1, C, C4
b) Alternate pathway—activate by bacteria/enotoxins: factors B, D, an P (properin)
c) MB-lectin pathway, triggere by mannan-bining lectin, a normal serum constituent that bins
some encapsulate bacteria
) Anaphylatoxins: C3a, C4a, C5a
e) Opsonins: C3b, C4b
f) Membrane attack complex: C5b-C9b

II. Acquire Immunity


A. Lymphoi organs
1. Primary lymphoi organs—liver, bone, thymus
. Seconary lymphoi organs—spleen, lymph noes
B. Cell-meiate immunity: effective against intracellular pathogens, major histocompatibility complex-
(MHC-) restricte, that is, only recognize antigen presente on MHC
1. CD4 “T helper” cells—cytokine release
a) IL-1: fever (increase prouction of prostaglanin, PGE, by hypothalamus)
b) IL-: maturation of cytotoxic T cells
c) IL-4: maturation of B cells into plasma cells
) IL-6: fever, acute phase reactant
e) IL-8: neutrophil recruitment, chemotaxis
f) IL-10: antiinammatory (“octreotie of the immune system”)
g) TNF-α: cachexia
h) IFN-γ: macrophage activation
. CD8 “cytotoxic T” cells—kill infecte cells
a) Suppressor cells—regulate CD4 an CD8 cells
b) Cytotoxic cells—recognize infecte cells expressing MHC-I, for example, viruses, intracellular
pathogens
C. Antiboy (humoral)-meiate immunity: effective against extracellular pathogens, not MHC-restricte
1. Antiboy prouction occurs as a result of plasma cell maturation from B cells stimulation via IL-4
(CD4 cells)
. Ten percent of plasma cells become memory cells, which can be reactivate for antiboy prouction

391
392 PArt ii Medical Knowledge

3. Antiboies (Abs):
a) All Abs have two antigen binings sites (except IgM)
b) All Abs have two regions: (1) constant region—recognize by effector cells, () variable region—
boun to antigen
c) IgM—largest (5 omains, 10 bining sites) → cannot cross placenta; initial Ab prouce after
antigen exposure, most common Ab in spleen; opsonin
) IgG—most abundant Ab; Ab prouce after seconary antigen exposure, can cross the placenta an
is responsible for neonatal immunity; opsonin
e) IgA—present on mucosal surfaces (Peyer patches), present in breast milk
f) IgE—type I hypersensitivity reactions an parasitic infestations
D. Antigen presentation
1. Extracellular pathogens, for example, bacteria: pathogen is engulfe by antigen-presenting cells
an fuse into phagosomes, which then unergo proteolytic egraation; these proteins are then
repackage an presente on the cell surface boun to MHC-II for presentation to CD4 cells
. Intracellular pathogens, forexample, viruses: infecte cells prouce viral proteins, which are loae
onto MHC-I an then presente on the cell surface to CD8 cells
E. Hypersensitivity reactions
1. Type I: IgE bins to basophils/mast cells leaing to release of histamine, 5-HT, an braykinin; for
example, allergies, anaphylaxis
. Type II: antigen-Ab complex (IgM or IgG), for example, ABO incompatibility, hyperacute rejection
3. Type III: antigen-Ab complex eposition, for example, systemic lupus erythematosus, rheumatoi
arthritis
4. Type IV: elaye (T cells)—APCs presents to CD4 → macrophage activation (IFN- γ); only
hypersensitivity not related to Ab., for example, chronic graft rejection
CHAPtEr 30 Immunology 393

Questions
1. Which immunoglobulin is responsible for 6. Which of the following is true regaring the
neonatal immune function, an how is it immeiate cellular response to a paper cut injury?
transmitte? A. L-selectin is expresse on enothelial cells
A. IgM, from breast milk B. The majority of the cytokine response is
B. IgA, crossing placenta release by circulating platelets
C. IgG, from breast milk C. ICAM expresse on enothelial cells bins to
D. IgA, from breast milk beta- integrin on leukocytes
E. IgM, crossing placenta D. This is not affecte by iabetes mellitus
E. Selectins are involve in platelet ahesion
2. A 19-year-ol male with a known history of HIV
is noncompliant with his retroviral meication 7. Which of the following is true regaring
an is foun to have a CD4 count of 4 cells/ cyclosporine?
mm3. What antibiotic prophylaxis is inicate? A. It is primarily excrete by the kineys
A. Daptomycin B. It is associate with thrombocytosis
B. Cephalexin C. It inhibits the release of IL-
C. Fluconazole D. It inhibits activation of B cells
D. Azithromycin E. It is more potent than FK-506
E. Clinamycin
8. A 8-year-ol male with type A bloo evelops
3. Which of the following is true regaring a high fever, chills, jaunice, an hematuria
apoptosis? shortly after receiving a bloo transfusion. The
A. It oes not occur uring embryogenesis nurse checks the bloo bag an realizes this
B. It is characterize by a loss of membrane patient receive type B onor bloo. Which of the
integrity following is true regaring this conition?
C. It inuces an inammatory response A. This is an example of serum sickness
D. CD-8 T cells can initiate apoptosis in cells that B. He evelope a T cell-meiate response
are virally infecte C. Direct Coombs test will emonstrate IgG
E. p53 inhibits apoptosis while BCL- promotes boun to re bloo cells
apoptosis D. His symptoms are a result of an
overexaggerate response from mast cells
4. Spontaneous regression of cancer ue to the E. This response oes not involve complement
immune system is best exemplie by which of activation
the following malignancies?
A. Melanoma 9. Which of the following is true regaring
B. Thymoma cytokines?
C. Colon A. IL- is a major enogenous pyrogen
D. Pancreas B. IL-6 is consiere a potent stimulus for the
E. Lung prouction of acute phase reactants
C. IL-10 is responsible for enhancing macrophage
5. Which of the following is true regaring the function
immune response to bacterial infection? D. Neutrophils are consiere the largest
A. CD-4 T cells transform B cells into plasma cells proucers of tumor necrosis factor (TNF)-a
B. Class-1 MHC molecules present bacteria- E. During an inammatory response, C-reactive
erive proteins protein prouction is ampene
C. Cells infecte by bacteria are estroye by
cytotoxic T cells
D. Activate CD-4 T cells secrete antiboies
E. Class- MHC cells are present on all nucleate
cells
394 PArt ii Medical Knowledge

10. A 4-year-ol male patient with HIV presents effusion concerning for an empyema an surgical
with fever, ry cough, an shortness of breath. consultation is requeste. Tracheal aspirate
Workup emonstrates Pneumocystis carinii culture is negative for any other organisms. The
pneumonia. He is amitte to the intensive patient is unable to speak in complete sentences
care unit an treate with trimethoprim/ an is using accessory muscles. He inicates
sulfamethoxazole (TMP-SMX). The resient that he has been noncompliant with all his
performs a meication reconciliation an starts meications. What is the most likely cause of his
the patient on highly active antiretroviral therapy worsening symptoms?
(HAART) as it was present in the patient’s A. Natural history of P. carinii pneumonia
electronic meical recor. His symptoms improve B. Incorrect antibiotic therapy
the next ay. However, on hospital ay three, C. Lymphocyte hyperactivity
the patient has worsening leukocytosis an D. Poor penetration of antibiotics in lung
hypotension, requiring initiation of vasopressors. parenchyma
Chest x-ray emonstrates a loculate pleural E. Superimpose bacterial infection

Answers
1. D. During the neonatal perio of evelopment, the continues lifelong an promotes the growth of healthy cells
immature immune system relies on exogenous transplacen- an tissue while facilitating the isposal of infecte, am-
tal IgG an IgA in the breast milk (B, C). IgM, the largest age, or transforme cells that may give rise to cancer. The
immunoglobulin, is too large to cross the placenta (A, E). two pathways of apoptosis have in common the activation of
Outsie of allergens an parasitic infections, IgE oes not caspases, which serves as the nal step for cell estruction.
play a major role in early in neonatal immunology. The intrinsic pathway is regulate by two important genes; p53
Reference: Pierzynowska K, Woliński J, Weström B, Pierzynowski promotes apoptosis while BCL-2 inhibits apoptosis (p for p53
SG. Maternal immunoglobulins in infants—Are they more than just a an promotes) (E). Li-Fraumeni synrome is characterize by
form of passive immunity? Front Immunol. 00;11:855. oi:10.3389/ an absence of p53 an thus apoptosis oes not occur, leaing to
mmu.00.00855. large soli tumors. The extrinsic pathway is activate by sev-
eral external “eath” receptors that are expresse in infecte
2. D. Antibiotic prophylaxis in patients with HIV an cells or cells with DNA amage. CD-8 T cells are responsible
low CD4 counts is essential in preventing opportunistic for recognizing the FAS-eath receptor in virally infecte cells
infections. For patients with CD4 counts <00, prophylaxis an initiating cell estruction. Apoptosis is characterize by
with trimethoprim-sulfamethoxazole (TMP-SMX) against DNA fragmentation an compartmentalization of cytoplasmic
Pneumocystis jirovecii (previously Pneumocystis carinii) an particles into apoptotic boies, which are then broken own
Toxoplasma gondii is inicate to prevent pneumonia an further by activate caspases an ultimately unergo phago-
encephalitis, respectively. When the CD4 count is <50, ai- cytosis by macrophages without inucing an inammatory
tional prophylaxis with azithromycin or clarithromycin is response (C). In contrast, cell necrosis is characterize by a vio-
neee against Mycobacterium avium complex (MAC) which lation of the cell membrane, release of cytoplasmic proucts,
can cause pneumonia. Fluconazole is not inicate unless an a subsequent inammatory response (B).
the patient lives in an area enemic to cocciiomycosis an Reference: Elmore S. Apoptosis: a review of programme cell eath.
has positive serology (C). Daptomycin is not inicate as a Toxicol Pathol. 007;35(4):495–516. oi:10.1080/01963070130337.
prophylactic antibiotic but can be use to treat gram-positive
infections in HIV patients (A). Similarly, clinamycin an 4. A. Spontaneous regression of malignant tumors refers to
cephalexin are not use as prophylactic meications in HIV cases of complete or partial tumor estruction an/or invo-
patients but can be use to treat uncomplicate skin infec- lution without any particular therapy. This occurs in most
tions within this population (B, E). cancers, but certain tumors regress more commonly. Mela-
Reference: Aberg JA, Gallant JE, Ghanem KG, Emmanuel P, noma, testicular germ cell tumors, an neuroblastoma are
Zingman BS, Horberg MA. Infectious Diseases Society of America. cancers that regress with increase frequency. This is ue
Primary care guielines for the management of persons infecte to a combination of cell apoptosis, immune meiators, an
with HIV: 013 upate by the HIV Meicine Association of the Infec- tumor microenvironment. Regression not only occurs in pri-
tious Diseases Society of America. Clin Infect Dis. 014 Jan;58(1). mary tumors but also can occur in metastases. The remaining
answer choices regress less frequently (B–E).
3. D. Apoptosis (programme cell eath) is a critical pro- Reference: Ricci SB, Cerchiari U. Spontaneous regression of
cess governing homeostasis an begins uring embryo- malignant tumors: importance of the immune system an other fac-
genesis with the sheing of skin between igits (A). This tors (Review). Oncol Lett. 010;1(6):941–945. oi:10.389/ol.010.176.
CHAPtEr 30 Immunology 395

5. A. The only cells capable of initiating humoral immu- occur quickly an are antiboy- an complement-meiate
nity to bacterial invasion are antigen-presenting cells, which while type IV is a elaye response an is T cell-meiate.
inclue enrites, macrophages, an B cells. This begins Type I is the only IgE-meiate reaction an occurs when a
with enocytosis an processing of bacterial proteins, which stimulus activates eosinophils, which in turn activate mast
are couple to class  MHC molecules an are expresse on cells an basophils, resulting in a systemic release of bra-
the cell surface (B). Next, CD-4 T cells recognize the bacterial ykinin, serotonin, an histamine (D). Type I reactions are
protein motif an bin to the receptor. The newly activate our immune system’s aaptation as a protective mechanism
CD-4 T cell ns B cells boun to the bacterial antigen an against parasites, which is less threatening in the moern
helps transform them into plasma cells (secreting antiboies) age. Instea, type I hypersensitivity reactions occur most fre-
an memory B cells conferring long-term immunity to a par- quently with exposure to allergens such as bee stings, peanut
ticular bacterial antigen (D). The immune response to a viral exposure, or hay fever. Type II hypersensitivity is an IgG- an
infection works by a ifferent mechanism. Firstly, all nucle- IgM-meiate response resulting in complement activation
ate cells (most notably absent are re bloo cells) have class (opsonization), cell lysis, an phagocytosis (E). In the case of
1 MHC molecules, which are able to bin to viral proteins ABO incompatibility, patients will present with wiesprea
an translocate to the cell surface (E). This is then recognize hemolysis. A irect Coombs test will emonstrate IgG boun
by CD-8 or cytotoxic T cells an marke for estruction (C). to RBC an an inirect Coombs test will measure free anti-
boies in the serum. Of note, not all type II hypersensitivity
6. C. The immune response involve in healing a paper reactions are cytotoxic; myasthenia gravis is a noncytotoxic
cut or similar small injury is a complex one. There are three variant of type II hypersensitivity. Type III hypersensitivity
stages incluing platelet rolling, tight ahesion, an emigra- is an immune complex–meiate response in which immune
tion. The amage enothelial cell expresses E-selectin (E conglomerates eposit into healthy tissue an thus inict
for enothelium), which bins to P-selectin on platelets an amage; serum sickness, systemic lupus erythematosus (SLE),
L-selectin on leukocytes (A). This promotes weak bining, an rheumatoi arthritis are examples of this (A). Type IV
which allows for platelet rolling initiating a platelet plug. hypersensitivity is a elaye reaction an is precee by T-cell
Circulating macrophages release cytokines an chemokines, sensitization (B). Tuberculosis skin test an contact ermatitis
which inuce the expression of various enothelial recep- are consiere type IV hypersensitivity reactions.
tors an attract other immune moulators (B). One of these
newly expresse enothelial receptors inclues ICAM, a type 9. B. Cytokines are largely responsible for cell signaling
of integrin that promotes stable bining allowing for plate- uring an inammatory response. TNF-a an IL-1 are the
let ahesion. Next, PECAM an VCAM are expresse on the two main cytokines responsible for propagating the inam-
enothelial surface, which facilitates emigration of circulat- matory response uring the early stages of injury an/or
ing leukocytes from the vasculature towar the inamma- infection. The largest proucers of these cytokines are mac-
tory stimulus. Selectins are involve in platelet rolling while rophages (D). Both are responsible for soliciting aitional
integrins are involve in platelet ahesion (E). This response cytokine prouction an immune cell recruitment. IL-1, in
is ampene in patients with iabetes an those with chronic particular, is consiere the primary enogenous pyrogen
steroi use, which helps explain why these patients have if- (A). It regulates the thermal set point in the hypothalamus
culty with woun healing (D). The most notable synrome (by bining to the CD-11 family receptor), resulting in fever.
affecting this process is leukocyte ahesion eciency, which Alveolar macrophages proucing IL-1 have classically been
is characterize by efunct integrin molecules leaing to taught to surgical resients as being responsible for the fever
recurrent bacterial infection an the classic presenting sign seen in patients with atelectasis. Aitionally, corticosterois
of elaye umbilical cor sloughing. can inhibit prouction of IL-1; this may explain why patients
with acute arenal insufciency evelop a high fever (ue
7. C. Cyclosporine is an immune moulator that was com- to the isinhibition of IL-1). Some authors o not agree that
monly use in transplant patients as maintenance therapy. It atelectasis is involve in postoperative fever an others have
has largely been replace by tacrolimus. Cyclosporine works suggeste that IL-6 more closely correlates with postopera-
by inhibiting cyclophilin protein on calcineurin an thereby tive fever. IL- is primarily prouce by T cells an helps
inhibits synthesis of IL- an IL-4, which are interleukins recruit an activate aitional T cells an enhances interac-
that activate T cells (D). FK-506 works by a similar mecha- tion between T an B cells. IL-6 is the most potent stimulus
nism but is consiere more potent than cyclosporine (E). of hepatic acute phase reactants incluing C-reactive pro-
The averse effects of cyclosporine inclue nephrotoxicity, tein, amyloi A, an ceruloplasmin (E). In contrast, prealbumin
gingival hyperplasia, hirsutism, an thrombocytopenia (B). an transferrin prouction ecrease uring inammation; this
The rug unergoes hepatic metabolism an is primarily explains why prealbumin as a measure of nutritional status
excrete in bile. Less than 5% unergo renal excretion (A). can’t be interprete without measuring one of the acute phase
Reference: Burckart GJ, Starzl TE, Venkataramanan R, et al. reactants. IL-10 is consiere the largest inhibitor of the inam-
Excretion of cyclosporine an its metabolites in human bile. Trans- matory response incluing the function of macrophages (C).
plant Proc. 1986;18(6 Suppl. 5):46–49. References: Losa García JE, Roriguez FM, Martín e Cabo
MR, et al. Evaluation of inammatory cytokine secretion by
8. C. This patient has evelope a type II hypersensitivity human alveolar macrophages. Mediators Inamm. 1999;8(1):43–51.
reaction from receiving an incorrect bloo type transfusion. oi:10.1080/0969359990711.
The cause of ABO incompatibility in bloo transfusion is Mavros MN, Velmahos GC, Falagas ME. Atelectasis as a cause
clerical error as emonstrate in the above case. There are of postoperative fever: where is the clinical evience? Chest.
four types of hypersensitivity reactions. The rst three types 011;140():418–44. oi:10.1378/chest.11-017.
396 PArt ii Medical Knowledge

10. C. This patient with HIV presents with an AIDS-ening with HAART. It usually occurs in patients whose pretreat-
opportunistic infection (P. carinii pneumonia). As such, he ment CD4 count is <100 cells/mm3 an requires evience of
likely has a low CD-4 count. In fact, HIV patients with CD-4 a positive virologic response to therapy, temporal association
counts <00 cells/mL shoul receive prophylactic treatment with initiation of therapy, systemic signs of inammation,
with TMP-SMX. This antibiotic is the rug of choice for an ruling out other etiologies of systemic inammation
P. carinii pneumonia as it has excellent lung penetration (B, (e.g., bacterial infection, rug-rug reaction) (E). IRIS in the
D). Most patients recover with this treatment (A). Patients context of a recently treate P. carinii pneumonia may present
that are noncompliant with HAART are at risk for immune with initial improvement of symptoms followe by wors-
reconstitution inammatory synrome (IRIS). This refers to a ening pulmonary symptoms, high fever, hypoxia, an even
group of inammatory isorers that arise after the initiation acute respiratory failure.
of antiretroviral therapy in AIDS patients. IRIS is more likely Reference: Sharma SK, Soneja M. HIV & immune reconstitu-
to occur in patients who are younger, male with a low CD-4 tion inammatory synrome (IRIS). Indian J Med Res. 011;134(6):
count, or who have an active infection an are noncompliant 866–877.
Infection and
Antimicrobial Therapy
ERIC O. YEATES AND JEFFRY NAHMIAS 31
ABSITE 99th Percentile High-Yields
I. Surgical Care Improvement Project (SCIP) Recommenations: Prevention of Postoperative Infection
A. Prophylactic antibiotics shoul be given within 1 hour prior to incision.
B. Antibiotics shoul cover the most likely pathogens to be encountere uring the operation.
C. Prophylactic antibiotics shoul be iscontinue after skin closure in clean an clean-contaminate cases.
D. Maintain euglycemia in the rst  postoperative ays.
E. Surgical site hair can be remove with electrical clippers, but not by shaving.
F. Urinary catheters shoul be remove before postoperative ay  when possible.
G. Maintain normothermia perioperatively.

II. Surgical Woun Classication System

Classięcation Description Examples Surgical site infection


Clean Uninfected wounds without Skin mass removal, tunneled 1%–5%
entry into the gastrointestinal, central-venous catheter
genitourinary, or respiratory placement, cardiac surgery
systems
Clean/contaminated Wounds with operative entry Cholecystectomy, routine 3%–11%
into the gastrointestinal, appendectomy,
genitourinary, or respiratory pancreaticoduodenectomy
system, but no gross spillage
Contaminated Gross spillage of gastrointestinal InĚamed appendicitis, 10%–17%
contents bile spillage during
cholecystectomy
Dirty/infected Incision through or operating in Perforated diverticulitis, >27%
existing infection incision and drainage of
abscess, traumatic wound >
4 hours old

III. Common Causes of Postoperative Fever an Timing


A. Win (pneumonia): POD 1-5
B. Water (urinary tract infection): POD 3-9
C. Walk (venous thromboembolism): POD 6-11
D. Woun (surgical site infection): POD 5-14

IV. Surviving Sepsis Guielines


A. Flui resuscitation
1. Resuscitation shoul begin ASAP with 30cc/kg of crystalloi within the rst 3 hours

397
398 PArt ii Medical Knowledge

. Flui challenges with crystalloi (either balance or saline) shoul be continue as long as
hemoynamics improve
3. Mean arterial pressure goal of 65 mmHg
4. Resuscitation shoul attempt to normalize lactate
B. Diagnosis
1. Appropriate cultures shoul be obtaine prior to starting antibiotics if it results in no substantial
elay
C. Antimicrobial therapy
1. IV antibiotics (empiric broa-spectrum coverage) shoul be starte as soon as possible an within
one hour of iagnosis of sepsis or septic shock
. Antibiotics shoul be narrowe once pathogen is ientie an sensitivities are establishe
3. Antibiotics shoul continue for 7 to 10 ays for most serious infections
4. Procalcitonin can be use to support shortening uration of antibiotics
D. Source control
1. Interventions to achieve source control shoul be implemente as soon as possible
. Intravascular evices that are possible sources of sepsis shoul be remove
E. Vasoactive meications
1. Mean arterial pressure goal of 65 mmHg
. First-line vasopressor is norepinephrine
3. Either vasopressin (up to 0.03 U/min) or epinephrine as secon-line
4. Recommen invasive monitoring with arterial catheter for patients requiring vasopressors
F. Corticosterois
1. IV hyrocortisone at 00 mg/ay in ivie oses (e.g., 50 mg every 6 hours) can be use for shock
refractory to ui resuscitation an vasopressors
G. Bloo proucts
1. RBC transfusion at <7.0 g/L (can have higher threshol in acute myocarial ischemia, severe
hypoxia, acute hemorrhage)
H. Glucose control
1. Bloo glucose goal shoul be less than 180 mg/L
. Point-of-care testing shoul be interprete with caution in patients with shock
I. Renal replacement therapy
1. Consier continuous or intermittent renal replacement therapy (RRT) in patients with sepsis an
acute kiney injury
. Continuous RRT shoul be utilize in hemoynamically unstable patients
J. Bicarbonate therapy
1. Recommen against soium bicarbonate therapy in patients with hypoperfusion-inuce lactic
aciemia with pH >7.15
K. Stress ulcer prophylaxis
1. Proton pump inhibitor or histamine- receptor antagonist is recommene in patients with sepsis/
septic shock who have risk factors for gastrointestinal bleeing
L. Nutrition
1. Early enteral feeing when feasible
. Recommen against omega-3 fatty aci as immune supplement in critically ill patients

V. Antibiotic Mechanisms by Site of Action


A. Cell wall synthesis
1. Beta Lactams (penicillins, cephalosporins, carbapenems, monobactams)
. Vancomycin
B. Nuclei aci synthesis
1. Folate synthesis: sulfonamies, trimethoprim
. DNA gyrase: quinolones
3. RNA polymerase: rifampin
C. Protein synthesis
1. 50S: macrolies, clinamycin, linezoli
. 30S: tetracyclines, aminoglycosies
CHAPtEr 31 Infection and Antimicrobial Therapy 399

Questions
1. A previously healthy 55-year-ol male presents D. Inwelling urinary catheter use longer than
with perforate iverticulitis an peritonitis. He ays after surgery is associate with a longer
unergoes an emergent exploratory laparotomy length of stay, but no ifference in urinary
an is foun to have purulent peritonitis. A tract infections
Hartmann's proceure is performe an his E. Intravenous insulin infusion reuces surgical
abomen is copiously irrigate. When shoul site infections after cariac surgery compare
antibiotics be stoppe in this patient? to sliing-scale subcutaneous insulin
A. Within 4 hours after the operation injections
B. Postoperative ay 4
C. Postoperative ay 7 4. A 47-year-ol female with history of pulmonary
D. Postoperative ay 14 sarcoiosis is iscovere to have a right upper
E. Duration shoul be etermine by lobe mass on chest raiograph that is outline
procalcitonin levels by a crescent of air superiorly. On a left lateral
ecubitus lm, the crescent of air shifts to remain
2. A 75-year-ol-male with ementia is amitte for in a nonepenent position. The patient is
a moerate traumatic brain injury after a groun- currently asymptomatic. What is the next step in
level fall. He weighs 50 kg. His hospital course management?
has been complicate by urinary retention an A. Diagnostic bronchoscopy with
he now has an inwelling urinary catheter. On bronchoalveolar lavage
hospital ay 4, he evelops worsening confusion B. CT-guie biopsy
throughout the ay. On evaluation, his heart C. IV voriconazole
rate is 105 beats per min, bloo pressure is 9/64 D. Pulmonary wege resection
mmHg, an his respiratory rate is 30 breaths per E. No further workup or treatment is require
min. Labs reveal a lactic aci of 5.4 mmol/L an a
metabolic aciosis with a pH of 7.18. Which of the 5. Which of the following proles for hepatitis B
following is true regaring the management of his surface antigen (HBsAg), hepatitis B surface
conition? antiboy (anti-HBs), total hepatitis B core
A. Vasopressors shoul be initiate immeiately antiboy (anti-HBc), an IgM antiboy against
B. A 1.5 L colloi ui bolus shoul be hepatitis B core antigen (IgM anti-HBc) woul
aministere as soon as possible you expect for a patient with chronic hepatitis B
C. A  L colloi ui bolus shoul be infection?
aministere as soon as possible A. HbsAg−, anti-HBs−, anti-HBc−, IgM anti-HBc−
D. Soium bicarbonate shoul be aministere to B. HbsAg−, anti-HBs+, anti-HBc+, IgM anti-HBc−
correct his aciosis C. HbsAg−, anti-HBs+, anti-HBc−, IgM anti-HBc−
E. A 1.5 L crystalloi ui bolus shoul be D. HbsAg+, anti-HBs−, anti-HBc+, IgM anti-HBc+
aministere as soon as possible E. HbsAg+, anti-HBs−, anti-HBc+, IgM anti-HBc−

3. Which of the following is true regaring the 6. Which of the following is true regaring
prevention of postoperative infections? occupational risk of hepatitis in health-care
A. Prophylactic antibiotics shoul be given workers?
anytime between 1 hour prior to incision an A. The risk of transmission is greater for hepatitis
before the en of the operation C than for hepatitis B
B. Cefazolin woul be an appropriate choice of B. If the expose person has been vaccinate for
prophylactic antibiotics for an elective sigmoi hepatitis B, no hepatitis B treatment is neee
colectomy C. If the patient has hepatitis C, the expose
C. Electrical clipping an manual shaving prior person shoul be given ribavirin
to an operation have similar infection rates D. Most hepatitis B transmissions are the result of
neelestick injuries
E. Hepatitis B virus can survive on rie bloo
for at least a week
400 PArt ii Medical Knowledge

7. A 3-year-ol male is recovering in the ICU 1 an has a history of vancomycin-resistant


ay after extensive ebriement of the left leg S. aureus bacteremia. Chest raiograph shows
for a necrotizing soft-tissue infection (NSTI). a 4-cm air-ui level within the right lung. He
He is intubate an requiring 80% FiO. He reports a 0-poun weight loss over the past 5
has leukocytosis that has been rising an an months. Appropriate management inclues:
elevate serum lactate. He is on broa-spectrum A. Aministration of intravenous aptomycin
antibiotics. Which of the following is the best next B. Thoracotomy an ecortication
step in management? C. Pulmonary lobectomy
A. Amputation of left leg D. Percutaneous rain placement
B. Secon-look operation E. Diagnostic bronchoscopy
C. A antifungal coverage
D. CT scan of the leg 12. Which of the following is true regaring the
E. Start pressors management of parapneumonic effusions?
A. They shoul be raine with tube
8. Which of the following is least likely to contribute thoracostomy as soon as etecte
to a surgical site infection? B. Intrapleural brinolytics are highly efcacious
A. American Society of Anesthesiologists physical in patients with loculate effusions
status C. Large iameter chest tubes (>8 French) are
B. Length of operation require for aequate rainage
C. Serum glucose level D. Treatment of the organizing phase requires
D. Boy temperature open rainage (e.g., Eloesser ap)
E. Hemoglobin level E. If it progresses to an empyema, vieo-assiste
thoracoscopic surgery shoul be performe if
9. The organism most commonly associate with it oes not respon to chest tube rainage
acute mesenteric lymphaenitis is:
A. Campylobacter jejuni 13. Which of the following is true regaring antibiotic
B. Escherichia coli mechanisms?
C. Enterococcus A. Penicillin-erivative antibiotics bin to the
D. Yersinia enterocolitica bacterial cell membrane an increase its
E. Pinworms permeability
B. Piperacillin-tazobactam works partly by
10. Forty-eight hours after total mastectomy, high bining β-lactamases
fever, iarrhea, vomiting, reness of the skin of C. Metroniazole, though limite, has some effect
the entire boy, an hypotension evelop in a against aerobic bacteria
30-year-ol patient. The mastectomy incision D. Linezoli competitively inhibits the 30S
appears unremarkable. The following ay iffuse ribosome
esquamation evelops. The most likely etiology E. Clinamycin, like the macrolies, reversibly
is: bins the 50S ribosome
A. Clostridium perfringens
B. Clostridium difcile 14. Which of the following shoul be use as part
C. β-Hemolytic Streptococcus of a screening tool to ientify non-ICU patients
D. Staphylococcus aureus that are at increase risk of organ failure from
E. Staphylococcus epidermidis infection?
A. Temperature higher than 38°C
11. A 5-year-ol male smoker with chronic B. Heart rate greater than 90/min
obstructive pulmonary isease (COPD) presents C. Altere mentation
to the emergency epartment (ED) complaining D. White bloo cell (WBC) count greater than
of fevers an foul-tasting sputum for the past 4 1,000/mm3
weeks. He was recently amitte to an outsie E. PaCO less than 3 mmHg
hospital for treatment of a COPD exacerbation
CHAPtEr 31 Infection and Antimicrobial Therapy 401

15. A 56-year-ol HIV-positive (with a low CD4 17. A 6-year-ol man is postoperative ay 6 from
count) patient presente to the ED with a an elective laparoscopic sigmoi colectomy for
spontaneous pneumothorax an unerwent a recurrent iverticulitis. He ha return of bowel
tube thoracostomy proceure. While trying to function  ays ago an was getting reay to be
re-cap the 0-gauge neele use for anesthetizing ischarge home. Throughout his hospital course,
the skin, the resient who performe the he has been having low-grae fevers. He is now
proceure was inavertently stuck resulting complaining of tenesmus an urinary retention.
in visible bleeing from the skin. Which of the Which of the following represents the most
following is true regaring this exposure? appropriate next step in management?
A. Postexposure prophylaxis with a -rug A. Transition to nonnarcotic pain meications
regimen shoul be aministere for 8 weeks B. Blaer scan an in-an-out catheterization as
B. Postexposure prophylaxis with a 3-rug neee
regimen shoul be aministere for 4 weeks C. Abominal raiography
C. Potential HIV infection shoul be isclose to D. Compute tomography (CT)
future patients E. Diagnostic laparoscopy
D. At least 6 months of postexposure treatment is
recommene 18. A 60-year-ol man presents with gas gangrene
E. The hollow bore neele use for this of his left leg requiring below-knee amputation.
proceure lowers the risk of HIV transmission Woun cultures were positive for Clostridium
septicum. Aitional workup shoul inclue:
16. A 45-year-ol HIV-positive male presents to the A. Hea CT scan
ED with perianal pain for the past two ays. B. Bronchoscopy
Physical exam reveals a small area of tenerness C. Colonoscopy
in the right posterolateral position istal to the D. HIV serology
external sphincter that is extremely tener. His E. Chest CT scan
CD4 count is 550 cells/mL, an he is currently
on highly active antiretroviral therapy (HAART). 19. Which of the following is true regaring tetanus?
Which of the following is the most correct A. It is highly contagious
management of this patient? B. Trismus is usually the rst sign
A. Intravenous (IV) antibiotics C. It is cause by a gram-negative anaerobic ro
B. Incision an rainage uner local anesthesia in D. A prior history of surviving tetanus provies
the ED immunity
C. Oral antibiotics an incision an rainage E. The iagnosis is establishe by emonstrating
uner local anesthesia in the ED the organisms in a woun
D. IV antibiotics, exam uner anesthesia (EUA),
an if an area of uctuance is ientie, then 20. Which of the following is associate with an
incision an rainage an biopsy enotoxin?
E. IV antibiotics, EUA, incision an rainage, A. Streptococcus pyogenes
an biopsy of the area of tenerness even if no B. Bacteroides fragilis
uctuance is ientie C. Clostridium tetani
D. S. aureus
E. C. perfringens
402 PArt ii Medical Knowledge

Answers
1. B. The Stuy to Optimize Peritoneal Infection Therapy infections. Their most recent recommenations aress tim-
(STOP-IT) trial was a ranomize controlle trial esigne ing of antibiotics, choice of antibiotics, hair removal tech-
to etermine the optimal length of antibiotic treatment after niques, normothermia, euglycemia, an inwelling urinary
source control in patients with intraabominal infections. catheter use. Prophylactic antibiotics shoul be given within 1
Patients were ranomize to receive antibiotics for 4 ays hour prior to the incision (A). Aitionally, the chosen antibi-
postoperatively versus  ays after the resolution of fever, otic shoul cover the most likely pathogen to be encountere
leukocytosis, an ileus. The meian uration of antibiotics uring the operation. Cefazolin oes not have appropriate
was 4 ays versus 8 ays postoperatively, an there was no anaerobic coverage for a colectomy (B). Surgical site hair
ifference in surgical-site infections, recurrent intraabomi- shoul be remove with electrical clippers, rather than shav-
nal infections, or mortality. Therefore, antibiotics shoul con- ing, on the ay of or ay prior to surgery. Shaving has been
tinue only for 4 ays postoperatively in most cases (A, C–E). shown to have a higher rate of meiastinitis in patients uner-
Procalcitonin may be useful in helping etermine the ura- going open-heart surgery (C). Inwelling urinary catheter
tion of antibiotic therapy. A single absolute value is less use- use longer than  ays after an operation is associate with
ful than the tren over several ays. It is best use in cases higher rates of urinary tract infection an it is recommene
where systemic inammation is present without an obvious to remove catheters prior to this point when possible (D).
infectious etiology. If the procalcitonin level eclines with Maintaining euglycemia for the rst 48 hours postoperatively
antibiotic therapy, it is reasonable to complete a 5 to 10-ay has been shown to ecrease surgical site infection. Aition-
course epening on the suspecte source(s). However, if ally, intravenous insulin infusion postoperatively was associ-
the level is normal an/or oes not change with antibiotics, ate with ecrease eep sternal woun infections in open
it woul be reasonable to stop antibiotic therapy. cariac surgery compare to sliing-scale insulin injections.
Reference: Sawyer RG, Clarige JA, Nathens AB, et al. Trial of References: Furnary AP, Zerr KJ, Grunkemeier GL, Starr A. Con-
short-course antimicrobial therapy for intraabominal infection. N tinuous intravenous insulin infusion reuces the incience of eep
Engl J Med. 015;37(1):1996–005. sternal woun infection in iabetic patients after cariac surgical
proceures. Ann Thorac Surg. 1999;67():35–360.
2. E. This patient is showing multiple signs of sepsis with Rosenberger LH, Politano AD, Sawyer RG. The surgical care
the most likely source being a catheter-associate urinary improvement project an prevention of post-operative infec-
tract infection. The Surviving Sepsis Campaign is a joint col- tion, incluing surgical site infection. Surg Infect (Larchmt).
011;1(3):163–168.
laboration committe to reucing morbiity an mortality
relate to sepsis an septic shock. Upate recommena-
tions etailing the ieal management of sepsis an septic 4. E. Aspergillus species are wiely isperse in the envi-
shock are provie perioically. Flui resuscitation shoul ronment an, when implicate as a pathogen, primarily
begin as soon as possible with 30 cc/kg of crystalloi (C). affect the lungs. It typically presents as one of four syn-
Albumin (colloi) bolus has not been consistently emon- romes: aspergilloma, allergic bronchopulmonary aspergil-
strate to be associate with improve outcomes in patients losis (ABPA), chronic necrotizing Aspergillus pneumonia, an
with septic shock an is signicantly more costly than crys- invasive aspergillosis. Aspergilloma typically presents as an
talloi. As such, crystallois are preferre over collois (B). asymptomatic raiographic ning in patients with a preex-
Empiric broa-spectrum intravenous antibiotics shoul also isting cavitary lung isease such as sarcoiosis. A soft-tissue
be aministere within one hour. However, appropriate cul- mass within a cavity that is surroune by a crescent of air
tures shoul be obtaine prior to starting antibiotics if this (Mona sign) is iagnostic, an because the aspergilloma is
incurs no substantial elay. If hypotension persists espite not aherent to the cavity walls, the air will remain in a non-
ui resuscitation, vasopressors shoul be initiate with epenent position. Biopsy or bronchoscopy is not inicate
a MAP goal of >65 mmHg (A). Aitionally, intravenous or necessary for iagnosis (A, B). As long as the patient is
hyrocortisone at 00 mg/ay shoul be consiere if shock asymptomatic, no further workup or treatment is necessary.
is refractory to both ui resuscitation an vasopressors (C). The most common symptom associate with aspergilloma
This patient has an elevate lactate which shoul be mea- is hemoptysis, which can occasionally be life threatening.
sure at regular intervals an utilize as an enpoint for In this setting, an emergency bronchial artery embolization
aequate resuscitation. Though he oes have a signicant shoul be performe followe by surgical resection (D).
aciemia, aministration of soium bicarbonate is not rec- ABPA is a noninvasive hypersensitivity isease that, if left
ommene for correction as long as his pH is greater than untreate, can lea to brotic lung isease. Therapy is aime
7.15 an certainly shoul not occur prior to aequate ui at the treatment of acute exacerbations either with inhale
resuscitation (D). bronchoilators/sterois (mil isease) or systemic cortico-
Reference: Rhoes A, Evans LE, Alhazzani W. Surviving Sepsis sterois (severe isease) to prevent long-term sequelae. Serial
Campaign: international guielines for management of sepsis an chest raiographs, pulmonary function tests, an IgE levels
septic shock. Crit Care Med. 017;45(3):486–55. shoul be monitore because permanent pulmonary am-
age can take place even in asymptomatic patients. Invasive
3. E. The Surgical Care Improvement Project (SCIP) is a pro- aspergillosis an chronic necrotizing Aspergillus pneumonia
gram esigne to reuce the rates of postoperative surgical are both treate with intravenous antifungals (C). Invasive
CHAPtEr 31 Infection and Antimicrobial Therapy 403

isease can be rapily fatal an is typically only foun in 7. B. NSTI is a broa term that encompasses infections lim-
immunocompromise hosts. High-risk transplant patients, ite to skin an subcutaneous tissue (necrotizing cellulitis)
such as bone marrow recipients, receive prophylactic agents an those involving the fascia (necrotizing fasciitis) an
to prevent invasive infection. muscle (myonecrosis). They can be extremely ifcult to
Reference: Limper AH, Knox KS, Sarosi GA, et al. An ofcial accurately iagnose early on because fewer than half present
American Thoracic Society statement: treatment of fungal infections with obvious har signs of NSTI, such as bullae, skin necro-
in ault pulmonary an critical care patients. Am J Respir Crit Care sis, gas on raiograph, an crepitus. Other signs inclue
Med. 011;183(1):96–18. tense eema, violaceous skin color, severe pain, an neuro-
logic ecit. Several laboratory values have been shown to
5. E. Hepatitis B surface antigen is foun on the surface be useful in istinguishing NSTI from simple cellulitis. The
of the hepatitis B virus an is foun in high quantities in LRINEC (Laboratory Risk Inicator for Necrotizing Fasci-
the serum of iniviuals with acute or chronic infection. itis) score uses the total WBC count, hemoglobin, soium,
Antiboies against this antigen (anti-HBs) are consiere glucose, serum creatinine, an C-reactive protein levels. A
to represent an immunity to the virus either from previous simpler moel uses an amission WBC count greater than
infection or vaccination. All patients with chronic hepatitis 15.4 × 109/L an/or a serum soium level less than 135
B infection will be anti-HBs negative. Antiboies against mEq/L. This latter moel is more useful for its negative pre-
hepatitis core antigen (anti-HBc) appear at the onset of ictive value (99%). A low serum soium level is theorize
symptoms an persist for life, though they o not confer to be the result of either a sepsis-inuce synrome of inap-
immunity to the isease. Vaccination will not prouce anti- propriate antiiuretic hormone or arenal insufciency, but
boies to hepatitis B core antigen. Presence of these anti- this has not been conrme. Risk factors for NSTI inclue
boies inicates either active or previous infection with iabetes, illicit IV rug abuse, immunosuppression, an
hepatitis B but oes not confer a timeline associate with liver isease. Seventy percent to 80% of NSTIs are ue to
that infection. However, IgM against hepatitis B core anti- polymicrobial infection. Of those that are cause by a sin-
gen is only present for the rst 6 months of infection, so gle organism, Klebsiella, S. pyogenes, an C. perfringens are the
its presence inicates a recent exposure to the virus. The most common. The NSTI is subivie into two categories;
aforementione serologic proles represent: A, suscepti- type I infections are cause by polymicrobial infection with
ble to infection; B, immunity from previous infection; C, aerobic an anaerobic bacteria (e.g., Clostridium an Bacteroi-
immunity from vaccination; D, acute infection; E, chronic des spp.), which work synergistically to prouce infection.
infection. Type II infections are cause by group A Streptococcus with
6. E. The risk of eveloping hepatitis B from a neelestick or without Staphylococcus. Treatment inclues rapi amin-
injury is far greater than that of hepatitis C, particularly istration of broa-spectrum antimicrobial agents, aggressive
when the patient is hepatitis Be surface antigen (HBesAg) ui resuscitation, an aggressive surgical ebriement.
positive (A). If the patient's bloo is both HBeAg an HBsAg The mortality rate remains at 0% to 40% an is higher with
positive, the risk of eveloping clinical hepatitis is very high surgical elays, particularly beyon 4 hours. A rising WBC
(%–31%). If the bloo is HBsAg positive but HBeAg nega- count an lactate after ebriement are highly suggestive
tive, the risk rops to 1% to 6% (although seroconversion is of progression of the NSTI. A secon-look operation is often
still high at 3%–37%). Hepatitis B is highly infectious, an require an shoul be performe for this patient in orer
the virus can survive on rie bloo an on environmental to ensure that no aitional tissues have become involve
surfaces for at least a week. The majority of health-care work- since the initial ebriement. Amputation may be neces-
ers infecte with hepatitis B o not recall a neelestick expo- sary, but only a secon-look operation will inicate whether
sure, though they were in contact with a hepatitis B–positive this is the case (A). CT scan in the postoperative setting may
patient (D). For health-care workers who have never been not be useful because interpretation can be ifcult secon-
vaccinate for hepatitis B, or are seronegative, treatment with ary to postsurgical changes (D). With septic shock, pressors
both HBIG (immunoglobulin prepare from human plasma may be necessary, but this woul not be the enitive treat-
known to contain a high titer of antiboy to HBsAg) an the ment (E). Aitionally, no hemoynamic parameters (bloo
hepatitis B vaccine is recommene (B). Data on clinical hep- pressure, central venous pressure) are provie that woul
atitis C following exposure is lacking. However, the average inicate that pressors are neee. Similarly, aing antifun-
incience of anti-HCV seroconversion from an HCV-positive gal coverage can be consiere, but this is not a enitive
source is very low (only 1.8%), suggesting that the risk of intervention (C).
transmission from a neelestick injury is very low. In fact, References: Anaya DA, Dellinger EP. Surgical infections an
choice of antibiotics. In: Townsen CM, Jr, Beauchamp RD, Evers
some stuies suggest that the risk of hepatitis C transmission
BM, Mattox KL, es. Sabiston textbook of surgery: the biological basis of
from a soli surgical neele is negligible. No effective pro-
modern surgical practice. 17th e. W.B. Sauners; 004:57–8.
phylaxis for HCV has been ientie. Immunoglobulin an Dunn DL, Beilman GJ. Surgical infections. In: Brunicari FC,
antiviral agents are not recommene for HCV postexposure Anersen DK, Billiar TR, etal., es. Schwartz's principles of surgery.
prophylaxis (C). 8th e. McGraw-Hill; 005:109–18.
Reference: Kuhar DT, Henerson DK, Struble KA, et al. Updated Wall DB, Klein SR, Black S, e Virgilio C. A simple moel to
U.S. public health service guidelines for the management of occupational help istinguish necrotizing fasciitis from nonnecrotizing soft tissue
exposures to HIV and recommendations for postexposure prophylaxis. infection. J Am Coll Surg. 000;191(3):7–31.
Division of Healthcare Quality Promotion, National Center for Wong CH, Khin LW, Heng KS, Tan KC, Low CO. The LRINEC
Emerging an Zoonotic Infectious Diseases, Center for Disease (Laboratory Risk Inicator for Necrotizing Fasciitis) score: a tool for
Control an Prevention (CDC); 013. https://stacks.cc.gov/view/ istinguishing necrotizing fasciitis from other soft tissue infections.
cc/0711. Crit Care Med. 004;3(7):1535–1541.
404 PArt ii Medical Knowledge

Yaghoubian A, e Virgilio C, Dauphine C, Lewis RJ, Lin M. Use is clear preoperatively (which is usually not the case), treat-
of amission serum lactate an soium levels to preict mortality ment is supportive because it is a self-limite isease. Ultra-
in necrotizing soft-tissue infections. Arch Surg. 007;14(9):840–846. soun has emerge as a useful tool in chilren to suggest this
iagnosis. Finings inclue enlarge, hypoechoic mesenteric
8. E. Risk for surgical site infections is relate to several lymph noes (at least one more than 8 mm in iameter) an
factors, incluing microbial contamination uring surgery, the absence of an iname (ilate) appenix. The iagnosis
length of operation, an patient factors such as iabetes, can also be mae with CT by the emonstration of enlarge,
nutritional state, obesity, an immunosuppression (can- clustere mesenteric lymph noes in the right lower qua-
cer, renal failure, immunosuppressive rugs) (B–D). The rant in the absence of acute appenicitis, but there is increas-
National Nosocomial Infection Surveillance Risk Inex is a ing reluctance to expose chilren to the raiation associate
useful tool to assess the risk of woun infection. This inex with CT scanning. The iagnosis is sometimes mae uring
inclues (1) American Society of Anesthesiologists physical laparoscopy.
status score higher than , () class III or IV wouns, an
(3) uration of an operation greater than the 75th percen- 10. D. A rare cause of infection in the rst 48 hours after
tile for that particular proceure (A). Wouns are classie an operation is woun toxic shock synrome. Toxic shock
as clean (class I) (e.g., hernia repair, breast biopsy), clean/ synrome is an acute onset, multiorgan illness that resem-
contaminate (class II) (e.g., cholecystectomy, elective gas- bles severe scarlet fever. It was originally escribe in men-
trointestinal surgery), contaminate (class III) (e.g., bowel struating women in association with tampon use, but it
injury from trauma or inavertent enterotomy), an irty has been increasingly recognize in postsurgical wouns.
(class IV) (e.g., perforate appenicitis, iverticulitis, nec- In the majority of cases, the illness is cause by S. aureus
rotizing soft-tissue infections [NSTIs]). Hemoglobin levels strains that express toxic shock synrome toxin-1, entero-
have not been shown to increase the risk of woun infection. toxin B, or enterotoxin C. It has rarely been escribe in
In a ranomize stuy of patients unergoing colorectal association with S. pyogenes (group A streptococci) (C). The
surgery, surgical woun infections were foun in 19% who remaining answer choices are not associate with toxic
were permitte to become hypothermic but in only 6% who shock synrome (A, B, E). Half of the postsurgical toxic
were actively kept normothermic. In a ranomize stuy of shock synrome cases present early, within 48 hours of
clean surgery (breast, varicose vein, hernia), those who were operation. Symptoms inclue fever, iarrhea, vomiting, if-
actively warme 30 minutes before surgery ha only a 5% fuse reness of the skin, an hypotension. This is followe
woun infection rate versus 14% in nonwarme patients. a ay or two later by iffuse esquamation. Physical exam-
Active control of glucose via continuous infusion was shown ination nings of woun infection are often unremark-
to ecrease sternal woun infection in iabetic patients able. Woun rainage an antibiotics are recommene.
unergoing cariac surgery. The main concern with aggres- Aministration of clinamycin may be helpful because it
sive glucose control, however, is that it may incite episoes inhibits exotoxin prouction.
of hypoglycemia. A recent stuy also highlighte the risk of Reference: Reingol AL, Dan BB, Shans KN, Broome CV.
bloo transfusion in woun infection, likely the result of its Toxic-shock synrome not associate with menstruation. A review
immunosuppressive effects. of 54 cases. Lancet. 198;1(86):1–4.
References: Campbell DA Jr, Henerson WG, Englesbe MJ, et al.
Surgical site infection prevention: the importance of operative ura- 11. E. Lung abscesses typically present with an inolent
tion an bloo transfusion–results of the rst American College of course over several weeks. Patients often complain of fevers,
Surgeons-National Surgical Quality Improvement Program Best purulent sputum, an cough. Single lung abscesses are fre-
Practices Initiative. J Am Coll Surg. 008;07(6):810–80. quently monomicrobial an are usually associate with
Furnary AP, Zerr KJ, Grunkemeier GL, Starr A. Continuous intra- aspiration pneumonia. As such, they are typically foun in
venous insulin infusion reuces the incience of eep sternal woun
segments of the lung that are epenent in the supine posi-
infection in iabetic patients after cariac surgical proceures. Ann
tion (i.e., the posterior segment of the upper lobes or the
Thorac Surg. 1999;67():35–360.
Kurz A, Sessler DI, Lenhart R. Perioperative normothermia to superior segments of the lower lobes). An air-ui level on
reuce the incience of surgical-woun infection an shorten hospi- a chest raiograph an purulent sputum are virtually iag-
talization: stuy of Woun Infection an Temperature Group. N Engl nostic of an anaerobic lung infection. However, coinfection
J Med. 1996;334(19):109–115. with antibiotic-resistant gram-positive organisms is pos-
Melling AC, Ali B, Scott EM, Leaper DJ. Effects of preoperative sible in patients with frequent hospitalizations. Most lung
warming on the incience of woun infection after clean surgery: a abscesses will resolve with antibiotics alone, but aptomycin
ranomise controlle trial. Lancet. 001;358(985):876–880. cannot be use to treat lung infections because it is inhibite
by pulmonary surfactant (A). In aition to intravenous (IV)
9. D. Acute mesenteric aenitis presents most commonly in antibiotics, a patient with risk factors for lung cancer (e.g.,
chilren an young aults. It can frequently be confuse with smoking, recent weight loss) shoul unergo bronchoscopy
appenicitis in chilren. Usually, an upper respiratory infec- to rule out an unerlying neoplasm (obstruction leaing to
tion is present or has recently resolve. The abominal pain infectious process). Surgical treatment may be necessary
is usually iffuse, but involuntary guaring on exam is rare. for infections that fail to respon to meical management,
Laboratory values are of little help in establishing the iag- abscesses greater than 6 cm in size, an abscesses seconary
nosis. More than 50% have an elevate WBC count. Although to an obstructe bronchus from a foreign boy or neoplasm.
infection with the other answer choices can lea to mesen- This typically involves either lobectomy or pneumonectomy
teric lymphaenitis, Y. enterocolitica is the most commonly (C). Percutaneous rain placement can be consiere in
associate organism in chilren (A–C, E). If the iagnosis patients who are poor surgical caniates (D). Thoracotomy
CHAPtEr 31 Infection and Antimicrobial Therapy 405

an ecortication are treatment options for empyema, not tetracyclines inhibit the 30S ribosome. Linezoli, on the other
lung abscess (B). han, inhibits the 50S ribosome subunit. Several other antibi-
References: Manal K. Thoracic infections. In: Yuh DD, Vricella otics (macrolies, linezoli, chloramphenicol) also inhibit the
LA, Yang SC, Doty JR. es. Johns Hopkins textbook of cardiothoracic sur- 50S ribosome; however, it is a slightly ifferent process (D).
gery. n e. McGraw-Hill; 014. Clinamycin is a lincosamie antibiotic, which interferes
with the amino acyl-tRNA complex (E). Aminoglycosies
12. E. Parapneumonic effusion refers to the accumulation an tetracycline antibiotics inhibit the 30S ribosome.
of pleural ui in response to a respiratory infection. It is
generally ivie into three stages: exuative, brinopuru- 14. C. The Third International Consensus Denitions for Sepsis
lent, an organizing. The rst (exuative) stage is character- and Septic Shock, publishe in JAMA in 016, reene the
ize by the evelopment of sterile pleural ui in response current enition use for sepsis an septic shock. The panel
to increase capillary permeability. After 5 ays, bacteria came to the conclusion that the previously use enition
begin to enter the ui an inammatory cells follow. This of sepsis (+ SIRS criteria an a source of infection) was too
marks the beginning of the brinopurulent phase. In gen- nonspecic an generally unhelpful in the ientication of
eral, new effusions shoul unergo iagnostic thoracentesis patients at increase risk of mortality from infection (A, B, D,
to rule out an empyema. If transuative, antibiotic treatment E). Instea, the committee recommene a besie screening
of the pneumonia is all that is require (A). Urgent rainage tool calle the quick Sequential Organ System Failure score
via tube thoracostomy is recommene for frankly purulent (qSOFA) for ientication of patients that are likely to have a
effusions or those with bacteria on Gram stain or culture. The poor outcome as the result of an infection. If a patient meets
iameter of the chest tube oes not seem to be important so two of the three criteria (respiratory rate >/min, altere
long as smaller caliber tubes are routinely ushe to prevent mental status, an systolic bloo pressure <100 mmHg), fur-
blockage of the catheter (C). As the brinopurulent phase ther workup an treatment for sepsis is inicate. The term
progresses, loculations begin to form within the collection, sepsis has also been change to represent a more serious phys-
making rainage with a single catheter or tube thoracostomy iologic process. Sepsis is now ene as an infection with 
ifcult. Several stuies have been one evaluating the use or more points on the Sequential (Sepsis-Relate) Organ Fail-
of intrapleural brinolytics, such as alteplase, to prevent pro- ure scoring system, or SOFA score. This score takes objective
gression to surgery. However, the results are controversial criteria for multiple organ systems (respiration, cariovascu-
at best, an a 008 Cochrane Review of the practice foun lar, coagulation, liver, central nervous system, an renal) an
no consistent benet (B). At this stage, vieo-assiste thora- assigns a score base on the amount of organ ysfunction. A
coscopic ebriement an ahesiolysis are viable options, score of  or more is associate with a 10% or greater increase
though a certain number of patients will still nee to be con- in mortality. Finally, the term septic shock has been reene
verte to thoracotomy (E). After  to 3 weeks of untreate as sepsis that requires vasopressors to keep the mean arterial
infection, broblasts begin to form a pleural peel an the pressure (MAP) greater than 65 mmHg an a lactate level
nal (organization) stage is reache. Once this membrane greater than .0 mmol/L. The term severe sepsis is no longer
has forme, formal ecortication via thoracotomy is gen- being encourage as a formal iagnosis.
erally necessary. In patients that are unt for surgery, open Reference: Singer M, Deutschman CS, Seymour CW, et al. The
rainage (e.g., Eloesser ap) may be consiere. However, Thir International Consensus Denitions for Sepsis an Septic
this subjects patients to months of ressing changes an sig- Shock (Sepsis-3). JAMA. 016;315(8):801–810.
nicant morbiity (D).
References: Cameron R, Davies HR. Intra-pleural brinolytic 15. B. With a bloo exposure, the rst step is to eter-
therapy versus conservative management in the treatment of ault mine the risk (severity) of the exposure an the risk to the
parapneumonic effusions an empyema. Cochrane Database Syst Rev. patient. The risk of puncture by a hollow neele with fresh
008;:CD0031. bloo is greater than the risk of puncture with a soli (sur-
Davies HE, Davies RJO, Davies CWH, BTS Pleural Disease Guie-
gical) neele, which is greater than the risk of splashing of a
line Group. Management of pleural infection in aults: British Tho-
racic Society Pleural Disease Guieline 010. Thorax. 010;65(Suppl
few bloo rops on mucous membranes or nonintact skin,
):ii41–ii53. which is greater than the risk of bloo rops on intact skin
Light RW. Parapneumonic effusions an empyema. Proc Am Tho- (no risk) (E). Depening on the combination of severity of
rac Soc. 006;3(1):75–80. exposure an severity of HIV, the recommenation is either a
basic regimen of two rugs (4 weeks of ziovuine an lami-
13. B. All penicillin-erivative antibiotics (β-lactams) vuine) or an expane one of three rugs (basic regimen
inhibit the nal step of bacterial cell wall synthesis by bining plus either ininavir or nelnavir for 4 weeks). Given that
transpeptiases or penicillin-bining proteins (A). Cephalo- the health-care worker ha visible skin penetration by fresh
sporins work by the same mechanism but are more resistant bloo with a hollow, large bore neele (high-exposure sever-
to egraation by β-lactamases. Tazobactam, sulbactam, an ity), an the patient escribe has a low CD4 count (high-
clavulanic aci bin β-lactamases an are frequently com- risk HIV status), the recommenation woul be a three-rug
bine with penicillin-erivative antibiotics to increase their regimen (A). The 3-rug regimen is recommene whenever
effectiveness. Examples of this inclue piperacillin-tazobac- a hollow neelestick pierces the skin an the patient is HIV
tam an amoxicillin-clavulanic aci. Metroniazole is an positive, regarless of his or her viral loa or CD4 count.
antibiotic that only has action against anaerobic bacteria by With a soli neele (as in the OR), because the risk of trans-
inhibiting nucleic aci synthesis. It is not effective in aerobic mission risk is lower, the severity of HIV is consiere, an
cells because it requires reuction to its active state, which a -rug regimen is recommene if the patient is low-risk
only takes place in anaerobic cells (C). Aminoglycosies an HIV positive (no active infection, low viral loa, high CD4
406 PArt ii Medical Knowledge

count); a 3-rug regimen is recommene if the patient is Steele SR, Kumar R, Feingol DL, Rafferty JL, Buie WD, Stan-
high-risk HIV positive. Follow-up testing to conrm HIV ars Practice Task Force of the American Society of Colon an Rec-
negative status in health-care workers is recommene 3 to 6 tal Surgeons. Practice parameters for the management of perianal
months later (D). Part of the initial evaluation of the expose abscess an stula-in-ano. Dis Colon Rectum. 011;54(1):1465–1474.
Vasilevsky CA. Anorectal abscess an stula. In: Beck DE, Wexner
health-care worker shoul involve counseling regaring
SD, Hull TL, etal, es. The ASCRS manual of colon and rectal surgery.
appropriate precautions incluing the use of barrier protec- n e. Springer; 013:45–7.
tion, not to onate bloo, practicing safe sex, an to avoi
breastfeeing if possible. If the HIV status of the patient is 17. D. Over 80% of all intraabominal abscesses are
unknown, it epens on the perceive risk of HIV an type postsurgical. They typically arise from one of two mecha-
of exposure. So if it is a soli neele, in a patient at a low risk nisms: persistent walle off infection after the resolution
for HIV, prophylaxis is generally not recommene; whereas of peritonitis or after an anastomotic breakown or perfo-
with a large-bore hollow neele, prophylaxis is generally ration that is effectively controlle by peritoneal efense
recommene until the patient tests negative. The average mechanisms. Presentation can be highly variable epen-
risk of HIV transmission after a percutaneous exposure to ing on their location, ranging from hiccoughing with sub-
HIV-infecte bloo is overall very low (approximately 0.3%). phrenic abscesses to a palpable mass in the paracolic gutter
For health-care workers there is no nee to stop working or or even sepsis. Pelvic abscesses can also present primarily
to inform patients of a possible exposure (C). The most recent with urinary or fecal symptoms such as urinary retention or
statement from the American College of Surgeons states that tenesmus, a recurrent inclination to evacuate bowels. These
“HIV-infecte surgeons may continue to practice an per- typically present on postoperative ays 5 to 7, an suspi-
form invasive proceures an surgical operations unless cious symptoms shoul be evaluate with an abominal
there is clear evience that a signicant risk of transmission CT with intravenous an potentially oral contrast epen-
of infection exists through an inability to meet basic infection ing on the clinical scenario. Plain abominal raiography
control proceures” an that “the HIV status of a surgeon has been essentially replace by CT because of increase
is personal health information an oes not nee to be is- iagnostic sensitivity an specicity for intraabominal
close to anyone.” pathology (C). Though narcotic pain meications or uner-
References: American College of Surgeons. Statement on the lying benign prostatic isease can cause urinary retention
surgeon an HIV infection. Bull Am Coll Surg. 004;89(5):7–9.
after surgery, a more serious etiology must be rule out rst
Kuhar DT, Henerson DK, Struble KA, et al. Upate US Pub-
(A, B). Almost all intraabominal abscesses can be treate
lic Health Service guielines for the management of occupational
exposures to human immunoeciency virus an recommena- with percutaneous rainage an antibiotics. In the absence
tions for postexposure prophylaxis. Infect Control Hosp Epidemiol. of iffuse peritonitis, operative intervention is likely unnec-
013;34(9):875–89. essary (E).
Reference: Tawaros PS, Simpson J, Fischer JE, Rotstein OD.
Abominal abscess an enteric fistulae. In: Zinner MJ, Ashley SW,
16. E. Anorectal isease is the most common inica- es. Maingot's abdominal operations. 1th e. McGraw-Hill; 013.
tion for surgery in the HIV-infecte patient, an it can fre-
quently be the rst presenting symptom for an uniagnose 18. C. C. septicum has been associate with colonic an
patient. However, iagnosis can be ifcult because HIV hematologic malignancies. In a review of the literature
patients with anorectal abscesses may be unable to mount involving 16 cases of C. septicum infection, 81% ha an asso-
an aequate response; thus, patients may present without ciate malignancy, incluing 34% with colon carcinoma an
an obvious uctuant abscess (epening on CD4 count). 40% with a hematologic malignancy. In 37%, the malignancy
Aitionally, they often have signicant tenerness that is was occult. The survival rate was only 35%. As such, patients
out of proportion to exam nings. Previously, operative iscovere to have an infection with C. septicum shoul have
interventions were avoie because of the risk of perianal an outpatient colonoscopy scheule (A, B, D, E).
sepsis. However, HAART therapy has allowe these patients Reference: Kornbluth AA, Danzig JB, Bernstein LH. Clostriium
to be manage with the same practice stanars as the non- septicum infection an associate malignancy. Report of  cases an
infecte patient with similar outcomes given that they are review of the literature. Medicine (Baltimore). 1989;68(1):30–37.
not neutropenic. Incision an rainage is recommene
for this patient (even if no uctuance is etecte) with a 19. B. Tetanus is an acute, often fatal, isease cause by
concurrent seton placement in the event a stula is iscov- an exotoxin prouce by the gram-positive anaerobic ro,
ere (A). Anoscopy with biopsy shoul also be performe C. tetani, that enters the boy through a woun (C). The
because a perianal abscess may be the presenting symptom mean incubation perio is 7 to 8 ays (range 3–1). In the
of an anal or rectal malignancy, particularly in an HIV-pos- presence of anaerobic (low oxygen) conitions, the spores
itive patient. Orinarily, antibiotics are not recommene germinate. C. tetani prouces two exotoxins: tetanolysin an
for perianal abscess. The exception is the immunocom- tetanospasmin. Tetanospasmin is a neurotoxin an causes
promise patient. Thus for the HIV patient, antibiotics are the clinical manifestations of tetanus. The toxins act at sev-
routinely use, even in the setting of aequate rainage, eral sites within the central nervous system (i.e., periph-
an woun cultures shoul be sent for the ientication of eral motor en plates, spinal cor, brain, an sympathetic
atypical organisms (D). The proceure is best performe in nervous system). The toxin interferes with the release of
the OR uner anesthesia (B, C). neurotransmitters, blocking inhibitor impulses, leaing to
References: Miles AJ, Mellor CH, Gazzar B, Allen-Mersh TG, unoppose muscle contraction an spasm. It is character-
Wastell C. Surgical management of anorectal isease in HIV-positive ize by generalize rigiity an convulsive spasms of skel-
homosexuals. Br J Surg. 1990;77(8):869–871. etal muscles. It typically involves the jaw muscles (hence
CHAPtEr 31 Infection and Antimicrobial Therapy 407

the term lockjaw) an neck (trismus) an then becomes gen- Interestingly, it is the only vaccine-preventable isease that is
eralize. The back spasms can be so intense that they can infectious but not contagious (A). Tetanus toxoi shoul be
lea to vertebral fractures. Intense facial spasms can lea to given as a series of three oses in chilhoo for prophylaxis
a classic appearance known as risus saronicus (saronic an as a booster ose every 10 years. It shoul also be given
smile, a smile of contempt or of pain). Laryngospasm an/or for wouns in patients with an incomplete or unknown his-
spasm of the muscles of respiration leas to interference with tory of the primary three oses an in those whose last ose
breathing. There are no characteristic laboratory nings of was over 10 years ago.
tetanus. Culture of the woun or bloo is not helpful. The
iagnosis is clinical (E). Treatment inclues human tetanus 20. B. As a general rule, gram-positive organisms prouce
immunoglobulin, airway protection by early enotracheal exotoxins, an gram-negative organisms have enotoxins. S.
intubation an, if neee, tracheostomy, IV magnesium for pyogenes prouces streptokinase, which acts as a brinolytic
muscle spasm prevention, high calorie replenishment, an (A). B. fragilis, a gram-negative organism, oes not prouce
benzoiazepines. Due to the extreme potency of the toxin, an exotoxin an has efective lipopolysaccharie an lipi
contracting tetanus oes not result in immunity (D). Tetanus A. C. tetani prouces tetanospasmin, which acts as a neu-
immune globulin (TIG) is recommene for iniviuals with rotoxin (C). S. aureus prouces hemolysin an leukociin,
tetanus. Active immunization with tetanus toxoi shoul which amage plasma membranes of the host, an exfolia-
begin or continue as soon as the person's conition has sta- tin, which cleaves esmosomes (D). C. perfringens prouces
bilize. Tetanus is not transmittable from person to person. heat-labile enterotoxin causing watery iarrhea (E).
Nutrition and Metabolism
ERIC O. YEATES, AREG GRIGORIAN, AND CHRISTIAN DE VIRGILIO 32
ABSITE 99th Percentile High-Yields
I. Daily Caloric Nees an Calculations
A. Estimate aily nees: 0 to 5 kcal/kg (approx. 50% carbohyrates, 5% protein, 5% fat)
B. 1 g of carbs = 4 kcal, 1 gof protein = 4kcal, 1g of fat = 9kcal
C. Critically ill patients shoul receive 30 kcal/kg of nutritional support a ay; patients who are seate
an mechanically ventilate have a lower expeniture of energy an shoul receive 5 kcal/kg/ay;
paralyze patients shoul receive 0 kcal/kg/ay
D. Most critically ill patients shoul receive 1. to .0 g protein/kg/ay; protein requirements are
increase in the obese, BMI of 30 to 40 shoul receive .0 g protein/kg/ay an BMI >40 shoul
receive .5g protein/kg/ay as part of an overall strategy of hypocaloric feeing
E. The presence of a 40% burn requires nutritional support with .5 g protein/kg/ay
F. Critically ill patients with renal failure shoul receive 1.5 to 1.75 g protein/kg/ay, whereas patients
with renal failure on continuous renal replacement therapy require .5 g protein/kg/ay
G. Respiratory quotient (RQ): estimates basal metabolic rate; ratio of carbon ioxie prouce by the boy
to oxygen consume by the boy
1. RQ > 1.0: overfeeing, can lea to ifculty weaning from ventilator ue to hypercarbia
. RQ = 1: carbohyrate utilization
3. RQ = 0.8 to 0.9: protein utilization (average 0.85; mixture of fat, protein, carb metabolization)
4. RQ = 0.7: fat utilization
5. RQ < 0.7: starvation

II. Amino Acis: Builing Blocks for Proteins


A. Essential: not mae by the boy, must be ingeste; phenylalanine, valine, threonine, tryptophan,
isoleucine, methionine, histiine, leucine, lysine
B. Nonessential: boy can prouce these: alanine, asparagine, aspartic aci, glutamic aci, serine
C. Conitional amino acis: boy can prouce these, but are essential in times of stress: Arginine, cysteine,
glutamine, glycine, ornithine, proline, tyrosine

III. Nitrogen Balance: To Stuy Protein Metabolism


A. Nitrogen balance = protein intake/6.5 – (4-hour urine nitrogen + 4g)
B. 4g approximates losses via sweat an feces
C. Positive = anabolism; negative = catabolism

IV. Starvation
A. Certain cells (brain, re bloo cells) primarily use glucose for energy (except when starving)
B. During starvation, insulin ecreases an glucagon increases, leaing to an increase in glycogenolysis,
lipolysis, an ketogenesis

409
410 PArt ii Medical Knowledge

C. Glycogen stores (glucose supply) are eplete after 1 to  ays


D. Next, fatty acis (can’t cross bloo-brain barrier) are oxiize to make ketones (for brain)
E. With further starvation, muscle (protein) breaks own to provie alanine for gluconeogenesis

V. Vitamin an Mineral Deciencies

Deęciency Manifestation/disease
Vitamin A Night blindness
Vitamin B1 (Thiamine) Wernicke’s encephalopathy, Beriberi
Vitamin B3 (Niacin) Pellagra (diarrhea, dermatitis, dementia)
Vitamin B6 (Pyridoxine) Anemia, peripheral neuropathy
Vitamin B12 (Cyanocobalamin) Megaloblastic anemia, peripheral neuropathy
Vitamin C Impaired collagen cross-linking, scurvy
Vitamin D Rickets, osteomalacia
Vitamin E Neuropathy
Vitamin K Coagulopathy
Chromium Hyperglycemia, neuropathy
Copper Anemia, leukopenia, muscle weakness
Iodine Goiter
Phosphate Diaphragm muscle weakness, arrhythmia, confusion
Selenium Cardiomyopathy, weakness
Zinc Delayed wound healing, hair loss, acne
Essential faĴy acids Dermatitis, hair loss, easy bruising, delayed wound healing
Essential amino acids Decreased immune function

VI. Nutritional Deciencies Associate With Surgeries/Surgical Diseases

Surgery/surgical diseases Related nutritional deęciency


Gastric bypass Deęciency of vitamin B12, folate, zinc, iron, copper, calcium, vitamin D
Gastric sleeve resection Deęciency of vitamin B12, folate, zinc, iron
Carcinoid syndrome Tryptophan deęciency (due to conversion to serotonin) causes pellagra
Blind loop syndrome Deęciency of vitamin B12, folate, iron, vitamin E
Refeeding syndrome Hypophosphatemia, hypomagnesemia, hypokalemia
CHAPtEr 32 Nutrition and Metabolism 411

Questions
1. A 45-year-ol male is iagnose with severe lysis of ahesions. She subsequently evelops
gallstone pancreatitis. It is currently hospital ay a postoperative ileus an is starte on total
 an he is not requiring vasopressor support or parenteral nutrition on postoperative ay 3. A
invasive mechanical ventilation. He still reports ay later, she rapily evelops weakness, altere
mil epigastric pain. Which of the following mental status, an hypoxic respiratory failure
is true regaring the ieal management of his requiring intubation. Which of the following is
nutrition? true regaring her conition?
A. He shoul be starte on an oral iet as A. Thiamine eciency is the most likely cause of
tolerate her symptoms
B. A nasoenteric tube shoul be place with tube B. Alcoholism is not a risk factor for eveloping
fees starte at a trophic rate an avance to this conition
goal as tolerate C. This conition rarely occurs with enteral
C. Nasojejunal feeing is preferre over nutrition
nasogastric feeing D. This conition coul have potentially been
D. Total parenteral nutrition (TPN) is preferre avoie by starting TPN at a slower rate
over enteral nutrition E. She shoul be given a calcium infusion
E. Enteric feeing shoul not be consiere until
abominal pain has resolve 4. Which of the following amino acis can
be synthesize e novo in humans in any
2. Which of the following is true regaring physiologic state?
immunonutrition? A. Tryptophan
A. Alanine is a substrate in the prouction of B. Tyrosine
nitric oxie (NO) C. Glycine
B. Glutamine has been shown to reuce raiation D. Serine
injury to the small bowel E. Any branche-chain amino aci
C. For cancer patients unergoing surgery,
immunonutrition reuces postoperative 5. Which of the following is true regaring the use
infectious complications of preoperative TPN to prevent postoperative
D. For patients unergoing major abominal complications?
surgery, immunonutrition reuces mortality A. It is useful even if use for as little as 3 ays
E. Glutamine has been shown to ecrease B. It is efcacious if the patient has lost more than
ventilator time in critically ill patients 15% weight before surgery
C. There is no evience that it lowers the
3. A 55-year-ol alcoholic female is amitte complication rate
to the hospital for a small bowel obstruction. D. Slightly overfeeing for 7 ays is
Her serum albumin is 1.8 g/L (normal range recommene as a means to maximize
is 3.4–5.4 g/L) an prealbumin is 8 mg/L replacement of caloric ecits
(normal range is 15–36 mg/L). On hospital E. TPN is efcacious even in mil to moerate
ay 5, she fails nonoperative management an malnutrition
unergoes an exploratory laparotomy with
412 PArt ii Medical Knowledge

6. Which of the following is true regaring 9. Which of the following is true regaring the risk
nutritional eciencies after a partial gastrectomy of hypoglycemia following cessation of total
with a Billroth II (gastrojejunostomy) parenteral nutrition (TPN)?
reconstruction? A. It commonly occurs in patients with liver
A. Calcium absorption will be minimally affecte isease
B. Iron eciency anemia is more common with a B. Tapering of TPN is recommene so as to
Billroth I (gastrouoenostomy) than a Billroth avoi this complication
II C. This complication is relatively common
C. Vitamin B1 eciency will present with a low D. It is more likely to occur in a iabetic patient
mean corpuscular volume E. It is more likely to occur in patients with renal
D. The stomach has no intrinsic absorptive ability isease
E. Carbohyrate absorption is not impaire after
surgery 10. Which of the following is true about the
pharmacologic treatment of cancer cachexia?
7. Which of the following is true regaring nutrition A. There is no evience that ghrelin mimetics are
nees an requirements? of benet
A. Preterm infants may nee up to  g/kg per B. Cannabinois are superior to megestrol acetate
ay of protein in stimulating weight gain
B. 1 g of fat provies 4 kcals of energy C. When initiate early, megestrol acetate has
C. A respiratory quotient (RQ) greater than 1.0 been emonstrate to improve survival
suggests overfeeing D. Megestrol is a progesterone erivative
D. Ventilate critically ill patients require more E. Anabolic sterois lea to improve long-term
aily caloric intake than nonventilate weight gain
critically ill patients
E. Obese patients require less aily protein intake 11. Which of the following is true regaring energy
compare to nonobese patients homeostasis uring perios of starvation?
A. The largest source of energy after glycogen is
8. A 3-year-ol male was amitte 7 ays ago for eplete is free fatty acis
multisystem trauma incluing multiple long-bone B. Skeletal muscle has the largest store of
fractures, subural hematoma, an pulmonary glycogen available systemically
contusions an is still on the ventilator. Which of C. Glucose is converte to lactate in the liver
the following is true regaring tools for assessing D. Re bloo cells metabolize glucose aerobically
nutritional status? E. The brain is unable to utilize ketones
A. Use of serial measurements of albumin an
prealbumin is the “gol stanar” for trauma 12. The most important amino aci use for
patients gluconeogenesis by the liver is:
B. Measurement of nitrogen balance A. Glutamine
unerestimates nitrogen input B. Serine
C. The Mini Nutritional Assessment is esigne C. Alanine
specically for hospitalize patients D. Tyrosine
D. Creatinine height inex may overestimate lean E. Asparagine
boy mass in trauma patients
E. Transferrin is the serum protein that correlates 13. Poor glucose control is a manifestation of
the closest to nitrogen balance eciency of:
A. Zinc
B. Copper
C. Chromium
D. Molybenum
E. Selenium
CHAPtEr 32 Nutrition and Metabolism 413

14. Which of the following is true regaring long- 15. Which of the following amino acis has shown
term TPN? potential for increasing the absorptive capability
A. Fat is consiere the nutritional basis of TPN of the intestine in patients that have unergone
B. It may lea to a mucin gel matrix of cholesterol large segment small bowel resection?
crystals an calcium bilirubinate in the A. Glutamine
gallblaer B. Serine
C. Hepatic ysfunction relate to TPN is less C. Alanine
likely to be lethal in infants than in aults D. Tyrosine
D. It has not been shown to lea to hepatic E. Arginine
brosis
E. Carnitine supplementation has been shown to
reverse TPN-relate liver amage

Answers
1. B. Though avancing to an oral iet as tolerate is rec- human boy an is also the major metabolic fuel for entero-
ommene in mil acute pancreatitis, this is not the recom- cytes an other cells within the immune system. Aminis-
mene management in moerate to severe pancreatitis (A). tration of glutamine has been shown to have no effect on
Instea, patients with moerate to severe acute pancreatitis reucing raiation injury (B). A number of large systematic
shoul have a nasoenteric/oroenteric tube place an enteral reviews have shown benets of immunonutrients in var-
nutrition starte in the rst 1 to  hospital ays (B). Mil epi- ious subsets of surgical patients. For example, in surgical
gastric pain is not a contrainication to enteric feeing (E). cancer patients an patients unergoing major abominal
With regars to the level at which to fee, three ranomize surgery, immunonutrition reuces infectious complications
controlle trials showe no ifference in tolerance or clinical an shortens length of stay, but oes not ecrease mortal-
outcomes between gastric an jejunal feeing (C). The use ity (C, D). In burn patients, initial clinical trial ata suggests
of parenteral nutrition rather than enteral nutrition has also that glutamine may reuce mortality, length of stay, an
been explore in multiple metaanalyses of ten ranomize gram-negative bacteremia though the enitive RE-EN-
clinical trials. These have shown that those receiving enteral ERGIZE trial is ongoing. However, there may be subsets of
nutrition ha lower infectious morbiity, shorter length of patients, like the critically ill, who may be harme by immu-
stay, fewer surgical interventions, an ecrease mortality nonutrition. Two large multicenter ranomize controlle
(D). Guielines from the Society of Critical Care Meicine trials of critically ill ventilate patients showe that supple-
an the American Society for Parenteral an Enteral Nutri- mentation with glutamine an/or antioxiants may increase
tion now recommen consieration of probiotics in severe 6-month mortality (E).
pancreatitis for those receiving early enteral nutrition, which References: Caler PC. Immunonutrition. BMJ. 003;37(7407):
is base on a 010 metaanalysis that showe a reuction in 117–118.
infection an hospital LOS. Suchner U, Kuhn KS, Fürst P. The scientic basis of immunonu-
Reference: McClave SA, Taylor BE, Martinale RG, et al. Guie- trition. Proc Nutr Soc. 000;59(4):553–563.
lines for the Provision an Assessment of Nutrition Support Therapy Probst P, Ohmann S, Klaiber U, et al. Meta-analysis of immuno-
in the Ault Critically Ill Patient: Society of Critical Care Meicine nutrition in major abominal surgery. BJS. 017;104(1):1594–1608.
(SCCM) an American Society for Parenteral an Enteral Nutrition Wischmeyer PE. Glutamine in burn injury. Nutr Clin Pract.
(A.S.P.E.N.). J Parenter Enter Nutr. 016;40():159–11. 019;34(5):681–687.
Yu K, Zheng X, Wang G, et al. Immunonutrition vs stanar
nutrition for cancer patients: a systematic review an meta-analysis
2. C. Immunonutrition is the ability to moulate the
(part 1). J Parenter Enter Nutr. 00;44(5):74–767.
immune system using specic nutrients. This strategy has
van Zanten AR, Hofman Z, Heylan DK. Consequences of the
most often been utilize in critically ill an surgical patients REDOXS an METAPLUS Trials: the en of an era of glutamine an
who often require exogenous nutrients through enteral or antioxiant supplementation for critically ill patients? J Parenter
parenteral routes. The nutrients most robustly stuie for Enter Nutr. 015;39(8):890–89.
immunonutrition are arginine, glutamine, omega-3 fatty
acis, branche chain amino acis, an nucleoties. Arginine 3. D. This patient evelope refeeing synrome, which
an glutamine, two amino acis that have been of particular is a potentially fatal metabolic isturbance after the rein-
interest in this el, have unique properties that may explain stitution of nutrition in a malnourishe patient (low serum
their mechanism of action. Arginine, via the arginine eam- albumin an prealbumin). Prolonge starvation leas to the
inase pathway, is a unique substrate for prouction of NO severe epletion of a number of minerals, though serum con-
(A). Glutamine is the most prevalent free amino aci in the centrations remain relatively normal ue to compensatory
414 PArt ii Medical Knowledge

intra/extracellular shifts. During refeeing, insulin is Thus, improperly aministere TPN increases the risk of
release, leaing to stimulation of glycogen, fat, an protein catheter-relate an noncatheter-relate infection (C). Buzby
synthesis, which requires phosphate, magnesium, an other propose the following guielines: (1) Postoperative TPN
cofactors. Phosphate, magnesium, an potassium (through shoul be consiere when oral or enteral feeing is not
the ATPase symporter) are all taken up into cells, leaing to anticipate within 7 to 10 ays in previously well-nourishe
a suen ecrease in serum levels. Refeeing synrome is patients or within 5 to 7 ays in previously malnourishe
cause by these epletions with hypophosphatemia being or critically ill patients. () Preoperative TPN shoul be con-
the most common an most severe isturbance (A). Some siere in patients who cannot or shoul not eat or receive
common clinical manifestations of severe hypophosphate- enteral feeings if the operation must be elaye for more
mia are arrhythmias, metabolic aciosis, seizures, elirium, than 3 to 5 ays. (3) Preoperative TPN shoul be consiere
hyperglycemia, an profoun weakness, sometimes man- in the most severely malnourishe surgical caniates if an
ifesting as iaphragm insufciency requiring mechanical operative elay is not contrainicate. In patients with only
ventilatory support. Risk factors for refeeing synrome mil to moerate egrees of malnutrition, preoperative TPN
inclue anorexia nervosa, malnutrition, chronic alcoholism, is not inicate.
cancer, recent surgery, elerly patients with comorbiities, References: Bozzetti F, Gavazzi C, Miceli R, et al. Perioper-
BMI <16, recent unintentional weight loss, an recent fast- ative total parenteral nutrition in malnourishe, gastrointestinal
ing (B). Refeeing synrome can evelop with either enteral cancer patients: a ranomize, clinical trial. J Parenter Enteral Nutr.
or parenteral nutrition, but may be more common with 000;4(1):7–14.
Buzby GP. Overview of ranomize clinical trials of total par-
enteral nutrition ue to the release of incretins (C). To pre-
enteral nutrition for malnourishe surgical patients. World J Surg.
vent refeeing synrome, it is recommene that nutrition
1993;17():173–177.
be starte at no more than 50% of normal aily requirements
for those who have not eaten in 5 ays (D). In patients at 6. E. The main eciencies of clinical concern that can be
high-risk of refeeing synrome, nutrition can be increase seen after gastrectomy are iron (most common), calcium,
to meet full nees over 4 to 7 ays. Treatment of refeeing an vitamin B1. Stomach aci helps reuce ietary iron
synrome shoul focus on the rapi correction of electrolyte from a ferric to a ferrous state, which allows it to be actively
abnormalities, with hypophosphatemia, hypomagnesemia, absorbe in the uoenum an jejunum. This can put
an hypokalemia being the most common (E). patients at risk of iron-eciency anemia following partial
References: McKnight CL, Newberry C, Sarav M, et al. Refee- or total gastrectomy. It oes occur more commonly with a
ing synrome in the critically ill: a literature review an clinician’s
Billroth II compare to a Billroth I (B). Calcium absorption
guie. Curr Gastroenterol Rep. 019;1(11):58.
Mehanna HM, Moleina J, Travis J. Refeeing synrome: what it
takes place primarily in the uoenum by an active process
is, an how to prevent an treat it. BMJ. 008;336(7659):1495–1498. that is regulate by vitamin D an parathyroi hormone.
After gastrectomy with Billroth II reconstruction, patients
4. D. Amino acis are the builing blocks use for the syn- are at risk for nutritional eciencies primarily because of
thesis of proteins. The nonessential amino acis are those quicker gastric emptying an anatomically bypassing the
that can be create e novo without an exogenous source. uoenum (A). Parietal cells locate in the gastric funus
In humans, these inclue alanine, aspartic aci, asparagine, an corpus are responsible for the prouction of intrinsic
glutamic aci, an serine. Essential amino acis are those factor, which is require for the absorption of vitamin B1 in
that cannot be synthesize an require an exogenous source: the terminal ileum. Vitamin B1 eciency will present with
phenylalanine, threonine, tryptophan, methionine, lysine, a megaloblastic anemia (increase MCV) an peripheral
an histiine (A–E). In aition, all the branche-chain neuropathy (C). While the stomach oes not typically absorb
amino acis (leucine, isoleucine, an valine [LIV]) are essen- many nutrients, it can absorb some lipi-soluble compouns
tial amino acis. A thir category of amino acis inclues such as alcohol, aspirin, an nonsteroial antiinammatory
those that can become essential in certain physiologic states, rugs (NSAIDs) (D). Though fatty aci absorption has been
such as premature infants or severe states of istress. These shown to be affecte after gastrectomy, there is no evience
inclue arginine, cysteine, glycine, glutamine, ornithine, pro- that carbohyrate absorption is impaire in any way.
line, an tyrosine. Patients with phenylketonuria (PKU) nee References: Guyton AC, Hall JE. Textbook of medical physiology.
to keep their intake of phenylalanine low, an because it is 11th e. WB Sauners; 005.
Lee JH, Hyung WJ, Kim HI, et al. Metho of reconstruction gov-
the precursor to tyrosine, it can become an essential amino
erns iron metabolism after gastrectomy for patients with gastric can-
aci in this isease state (B, C).
cer. Ann Surg. 013;58(6):964–969.

5. B. Proviing nutritional intervention shoul be limite 7. C. Critically ill patients shoul receive 30 kcal/kg of
to patients with severe malnutrition an immunologic ys- nutritional support a ay. Patients who are seate an
function. In a Veterans Affairs multicenter trial, malnour- mechanically ventilate have a lower expeniture of energy
ishe patients who lost more than 15% of their baseline an shoul receive 5 kcal/kg/ay (D). The aily recom-
boy weight ha ecrease operative septic complications mene protein requirement in an ault is approximately
when they receive preoperative nutritional intervention for 0.8 g/kg per ay. However, this can increase in the setting
7 to 10 ays (A). However, in the group stratie as hav- of physiologic stress. Most critically ill patients shoul
ing mil to moerate malnutrition, the ecrease in surgical receive 1. to .0 g protein/kg/ay. Burn patients’ protein
complications was more than offset by the increase in cathe- requirement is closer to  to .5 g/kg per ay. There is also an
ter-relate infectious complications (E). TPN-inuce hyper- increase eman for protein in peiatric patients because
glycemia is likely a contributor to averse outcomes (D). of active growth, with the largest being preterm infants who
CHAPtEr 32 Nutrition and Metabolism 415

may nee 3 to 4g/kg per ay (A). 1g of fat provies 9kcal no ifference in symptomatic hypoglycemia or serum glu-
of energy (B). Protein requirements are increase in the cose measurements between a TPN-epenent group ran-
obese but shoul be part of an overall strategy of hypoca- omize to abrupt cessation versus step-wise tapering (B).
loric feeing (E). The RQ is the ratio of carbon ioxie pro- However, in the iabetic patient, an in those with poor glu-
uce to oxygen consume, an it can be use to estimate cose control, tapering of TPN shoul be consiere (A–E).
which energy source is the primary substrate for energy References: Eisenberg PG, Gianino S, Clutter WE, Fleshman
prouction. However, it must be measure at a steay state. JW. Abrupt iscontinuation of cycle parenteral nutrition is safe. Dis
By knowing the RQ, you are able to etermine the primary Colon Rectum. 1995;38(9):933–939.
substrate being use for energy prouction: greater than 1 Nirula R, Yamaa K, Waxman K. The effect of abrupt cessation of
total parenteral nutrition on serum glucose: a ranomize trial. Am
for lipogenesis (overfeeing state), 1.0 for carbohyrates, 0.8
Surg. 000;66(9):866–869.
for proteins, an 0.7 for fatty acis. This can then be extrapo-
late to the nutritional state of the patient by knowing what
10. D. Cancer-relate cachexia/anorexia has been associ-
substrates are being use at various phases of fasting. A nor-
ate with failure of cancer treatment, elay in initiation of
mal RQ is aroun 0.85 because the boy is using about 50%
treatment, increase treatment toxicity, early iscontinuation
carbohyrates an 50% fatty acis. The overfe state is pre-
of treatment, an shorter survival in terminal cancer patients.
ominate by conversion of glucose into fats an correlates
It has even been implicate as a irect cause of eath in 0%
with an RQ of more than 1. Starving patients are primarily
to 40% of cancer patients. Current National Comprehen-
using fatty acis as the primary fuel source an have an RQ
sive Cancer Network (NCCN) Guielines recommen early
of less than 0.7.
screening an early treatment of this conition. Before ini-
References: Barrett KE, Boitano S, Barman SM, Brooks HL.
tiation of appetite stimulation, treatable causes of anorexia
Ganong’s review of medical physiology. 3r e. McGraw-Hill Meical;
009.
such as oral caniiasis or epression shoul be aresse.
Guyton AC, Hall JE. Textbook of medical physiology. 11th e. WB Megestrol acetate (Megace) is the most wiely stuie an,
Sauners; 005. so far, most efcacious meication available to help improve
appetite an weight gain in this patient population. Mege-
8. D. Nutritional assessment in hospitalize patients is lim- strol acetate is a synthetic, orally active erivative of proges-
ite by multiple confouning factors. While there are lots of terone. It has been foun to improve appetite, caloric intake,
tools available for nutritional assessment, no single item has an nutritional status in several clinical trials. A stuy con-
proven to be infallible in assessing a patient’s nutritional sta- ucte in 010 emonstrate that megestrol acetate use in
tus. Current Eastern Association for the Surgery of Trauma combination with olanzapine was associate with improve-
(EAST) Guielines for nutritional assessment in the trauma ments in weight gain, appetite, nausea, an overall quality of
patient use nitrogen balance as the “gol stanar” by which life when compare with megestrol acetate alone, even when
all other tests are evaluate (A). Though nitrogen balance is a correcte for improvements in epression. Unfortunately,
fairly accurate measurement of nutritional status, it is limite megestrol acetate, either alone or in combination with olan-
by the impracticality of 4-hour urine collection an the often zapine, has not been emonstrate consistently in the litera-
inaccurate recoring of aily nitrogen input. Nitrogen out- ture to improve survival (C). It is also important to note that
put is often unerestimate an input is often overestimate 1 in 3 patients using megestrol acetate will have a throm-
(B). Of the serum proteins, prealbumin seems to correlate the boembolic event; therefore, it shoul be use with caution
closest with nitrogen balance (E). Many of the serum proteins in susceptible patients. While it has been emonstrate that
use for nutrition assessment—albumin, prealbumin, trans- sterois have results equivalent to megestrol acetate, they
ferrin, an retinol-bining protein—are altere in times of are short-live an patients quickly return to baseline after
stress or infection, so most sources recommen incluing an cessation of the rug (E). Cannabinois have been looke at
acute phase reactant such as CRP to put these values in con- extensively in chemotherapy-relate nausea an AIDS-ca-
text. While the creatinine height inex can give you an esti- chexia, but stuies one in the cancer population ten to
mate of lean boy mass, changes in creatinine excretion from show inferiority to megestrol acetate (B). Ghrelin mimetics
systemic processes (e.g., trauma, renal isease, etc.) can make have been emonstrate to improve lean boy mass (A).
the results unreliable. The Mini Nutritional Assessment is References: Nagaya N, Kojima M, Kangawa K. Ghrelin, a novel
specically esigne for the elerly (C). growth hormone-releasing peptie, in the treatment of cariopul-
References: Elmafa I, Meyer AL. Developing suitable methos monary-associate cachexia. Intern Med (Tokyo). 006;45(3):17–134.
of nutritional status assessment: a continuous challenge. Adv Nutr. Navari RM, Brenner MC. Treatment of cancer-relate anorexia
014;5(5):590S–598S. with olanzapine an megestrol acetate: a ranomize trial. Support
Jacobs DG, Jacobs DO, Kusk KA, et al. Practice management Care Cancer. 010;18(8):951–956.
guielines for nutritional support of the trauma patient. J Trauma. Ohnuma T. Treatment of cachexia. In: Kufe DW, Pollock RE,
004;57(3):660–679. Weichselbaum RR, et al., es. Holland-Frei cancer medicine review: com-
Norton JA. Essential practice of surgery: basic science and clinical panion to Holland-Frei cancer medicine. 6th e. BC Decker; 003.
evidence. Springer; 003.
11. A. After a meal, carbohyrates are rapily use, an
9. D. Hypoglycemia following the abrupt cessation of TPN any excess is store as fatty acis or as glycogen (primarily
has been reporte, though it is very rare (C). Hypoglycemia in the liver an skeletal muscle). Though the skeletal muscle
can present with iaphoresis, confusion, agitation, tachycar- has proportionally more glycogen store, it is not available
ia, an, if severe, iabetic coma. Most patients will tolerate systemically uring fasting because these cells lack glu-
abrupt cessation of TPN, an tapering is generally unneces- cose-6-phosphatase, which is the nal step neee for the
sary. Two stuies publishe in 1995 an 000 both showe creation of glucose from glycogen (B). As such, the glycogen
416 PArt ii Medical Knowledge

stores are use only locally. Liver stores of glycogen are nor- an cariomyopathy, loss of pigmentation, an erythrocyte
mally use within 16 to 36 hours, but it can be shorter in macrocytosis (E).
certain isease states. After glycogen stores are eplete, the
boy turns to the breakown of skeletal muscle an lipis 14. B. Glucose is consiere the nutritional basis of TPN,
for energy. The largest source of energy is free fatty acis, but while fat is consiere the nutritional basis of peripheral par-
they are a relatively poor source of free glucose. While amino enteral nutrition (PPN) (A). Liver ysfunction is commonly
acis from protein breakown can be use for gluconeo- observe in patients receiving TPN. It evelops in 40% to
genesis in the liver (early in starvation) an kiney (late in 60% of infants who require long-term TPN for intestinal fail-
starvation), most proteins serve an important role in boily ure. The clinical spectrum inclues cholestasis, biliary sluge
functions. Lactate an glycerol can also be use as substrates (mucin gel matrix of cholesterol crystals an calcium biliru-
for gluconeogenesis (C). During prolonge fasting, tissues binate), cholelithiasis, hepatic brosis with progression to
that are able to use alternate fuel sources (i.e., breakown biliary cirrhosis, an the evelopment of portal hypertension
proucts of fatty acis) begin to o so, an subsequently, an liver failure (D). Preisposing factors inclue short gut
the breakown of muscle slows an breakown of boy fat synrome, a history of bacterial overgrowth, an recurrent
increases. However, gluconeogenesis never completely stops sepsis or a chronic inammatory state. Lack of enteral fee-
because several cells are heavily reliant on glucose as a fuel ing contributes by leaing to reuce gut hormone secretion,
source. Re bloo cells are solely reliant on the anaerobic ecrease bile ow, an biliary stasis. Deciencies in partic-
conversion of glucose to lactate because they lack the mito- ular nutrients such as carnitine, taurine, cysteine, an S-a-
chonria require for the utilization of fatty acis or for the enosylmethionine are also implicate in TPN-relate liver
aerobic breakown of glucose (D). In aition, white bloo isease. Hepatic steatosis may be improve with carnitine
cells, cells in the arenal meulla, an peripheral nerves are supplementation, but there is no evience that it will reverse
all obligate glucose users. While the brain is heavily reli- TPN-relate liver amage (E). Hepatic ysfunction is more
ant on glucose as a fuel source, it can use ketones to some serious an lethal in infants epenent on TPN compare
egree(E). with aults (C). Even when enteral feeing is begun an
References: Brunicari FC, Anersen DK, Billiar TR, Dunn DL, TPN is iscontinue, hepatic ysfunction may persist an
Hunter JG, Matthews JB, Pollock RE, es. Schwartz’s principles of sur- may progress to cirrhosis an eath. The ultimate solution is
gery. 10th e. McGraw Hill Eucation; 015. combine liver an small bowel transplantation.
Cahill GF. Fuel metabolism in starvation. Annu Rev Nutr. Reference: Kelly D. Liver complications of peiatric parenteral
006;6(1):1–. nutrition: epiemiology. Nutrition. 1998;14(1):153–157.

12. C. In humans, the main substrates for gluconeogenesis 15. A. In two ranomize stuies, patients with short gut
are lactate, pyruvate, amino acis, an, to a lesser extent, synrome seconary to small bowel resection were seen
glycerol. This is primarily stimulate by glucagon. Alanine to have moest improvements after the aministration of
is the most important amino aci precursor in gluconeogen- supplemental glutamine, exogenous growth hormone, an
esis. When the liver has exhauste all of its alanine supply, a moie iet with increase ber. One stuy showe an
the kiney takes over gluconeogenesis where glutamine may improvement in calorie, protein, an carbohyrate absorp-
be use for gluconeogenesis (A). Aitionally, alanine an tion as well as a reuction in stool volume. However, the
phenylalanine are the only amino acis that increase uring secon stuy faile to show an increase in the absorption of
times of stress. Serine, tyrosine, an asparagine are not sub- macronutrients an only showe an improvement in electro-
strates for gluconeogenesis (B–D, E). lyte absorption an a reuction in elaye gastric emptying.
These specic interventions seeme to exert bowel-specic
13. C. Chromium is a cofactor involve in the utilization trophic effects, which may inuence nutritional absorp-
of insulin at the tissue level, an eciency often manifests tion. However, it is unclear whether this is through a irect
as a suen iabetic state in which bloo sugar is ifcult or inirect mechanism. It is important to keep in min that
to control, along with peripheral neuropathy an enceph- these stuies were one on a small number of patients, but
alopathy. Zinc eciency has numerous manifestations, they show some promise in patients that woul otherwise be
incluing alopecia, poor woun healing, immunosuppres- completely TPN epenent. The remaining answer choices
sion, night blinness or photophobia, impaire taste or have not been shown to improve intestinal absorption ef-
smell, neuritis, an a variety of skin isorers (A). Copper ciency (B–E).
eciency manifests as microcytic anemia, pancytopenia, References: Byrne TA, Morrissey TB, Nattakom TV, Ziegler
TR, Wilmore DW. Growth hormone, glutamine, an a moie iet
epigmentation, an osteopenia (B). Essential mineral an
enhance nutrient absorption in patients with severe short bowel syn-
vitamin eciency may occur with increase frequency in rome. J Parenter Enteral Nutr. 1995;19(4):96–30.
patients receiving long-term parenteral nutrition. Molybe- Scolapio JS, Camilleri M, Fleming CR, et al. Effect of growth
num eciency is characterize by the toxic accumulation hormone, glutamine, an iet on aaptation in short-bowel
of sulfur-containing amino acis an encephalopathy (D). synrome: a ranomize, controlle stuy. Gastroenterology.
Selenium eciency may result in iffuse skeletal myopathy 1997;113(4):1074–1081.
Oncology and Tumor Biology
ALEXANDRA MOORE, AREG GRIGORIAN, AND CHRISTIAN DE VIRGILIO 33
ABSITE 99th Percentile High-Yields
I. Principles of Raiation Therapy
A. External beam raiation therapy: high energy electrons
1. Damages DNA uring replication (M phase of mitosis) leaing to apoptosis
. Raiation effectiveness: ouble-strane DNA breaks ue to oxygen free raicals
a) Tissue hypoxia signicantly reuces raiation amage an effectiveness; larger tumors are
relatively hypoxic an thus more resistant to raiation compare to smaller tumors

II. Principles of Chemotherapy


A. Most agents act uring DNA replication (the S phase of mitosis)

III. Common Genetic Mutations in Specic Tumor Types


A. RAS: colorectal, neuroblastoma, pancreas, blaer
B. HER/neu: breast, ovarian, stomach
C. MYC: cervical, colon, breast, lung, gastric

IV. Monoclonal Antiboies


A. Human epiermal growth factor receptor  (HER) gene amplication:
1. Trastuzumab: inhibits HER-HER3 imerization; use in HER+ breast an stomach cancer
. Pertuzumab: bins extracellular HER omain
B. Vascular enothelial growth factor (VEGF) inhibitor:
1. Bevacizumab: irectly bins VEGF, inhibits angiogenesis
a) Metastatic colon cancer
b) Sie effects inclue hypersensitivity type-1 reaction, viscous perforation, an impaire woun
healing ue to inhibition of angiogenesis
C. Epiermal growth factor receptor (EGFR) inhibitor:
1. Cetuximab: irectly bins an inhibits EGFR
a) Metastatic KRAS wil-type colorectal cancer (no benet in KRAS mutation), hea an neck
squamous cell cancer
D. Tyrosine kinase inhibitors
1. Imatinib: tyrosine kinase inhibitor; neoajuvant or ajuvant use in GIST
. Sunitinib: tyrosine kinase inhibitors; n line therapy in GIST if imatinib is ineffective; also use in
unresectable neuroenocrine tumors
3. Sorafenib: multitargete tyrosine kinase inhibitor; use in hepatocellular an gallblaer carcinomas
E. Immune checkpoint inhibitors:
1. Ipilimumab: CTLA-4 checkpoint inhibitor
a) Improves survival in unresectable, noe-positive or metastatic melanoma
. Pembrolizumab: PD1 checkpoint inhibitor
a) Melanoma, lung cancer
417
418 PArt ii Medical Knowledge

V. Systemic Biomarkers in Cancer


A. CEA: Colon cancer. Particularly for etecting recurrence after surgery
1. >00 ug/L before resection = poor prognosis
B. CA 19-9: Pancreatic cancer
1. >1000U/mL = likely metastatic isease
C. CA-15: Epithelial ovarian cancer
D. Inhibin-A: epithelial stroma tumors such as mucinous an enometrioi carcinoma an sex cor-stromal
tumors such as granulosa cell tumor an Sertoli-Leyig cell tumor
E. AFP: hepatocellular carcinoma (HCC), nonseminomatous germ cell tumors
F. β-hCG: germ cell tumors
G. Calcitonin: meullary thyroi cancer
H. Thyroglobulin: papillary thyroi cancer; particularly for etecting recurrence after surgery an
raioactive ioine therapy
I. Chromogranin A: carcinoi tumors
Chemotherapeutic Sie Effects

Agent Side eěects


Bleomycin Pulmonary ębrosis
Carboplatin Myelosuppression
Cisplatin Nephrotoxicity, neurotoxicity, ototoxicity
Cyclophosphamide Hemorrhagic cystitis (treat with Mesna), gonadal dysfunction, SIADH
Doxorubicin Cardiac toxicity (irreversible)
Methotrexate Nephrotoxicity, nausea (treat with folinic acid)
Oxaliplatin Peripheral neuropathy
Taxols Neuropathy
Trastuzumab Cardiomyopathy (reversible)
Vinblastine Myelosuppression
Vincristine Peripheral neuropathy

Hereitary Cancer Synromes

Gene Syndrome Associated cancers


APC Familial adenomatous Colon, duodenum, gastric, thyroid, desmoid tumors
polyposis (FAP)
TP53 Li Fraumeni Breast (90%), colon, lung, sarcomas, brain, adrenal
DNA Mismatch Repair Hereditary nonpolyposis Colorectal, endometrial, gastric, ovarian, GU tract,
(MLH1, MSH2, MSH6, colorectal cancer hepatobiliary, small bowel, and CNS
PMS2, EPCAM) (HNPCC)
SMAD4, BMPR1A Juvenile polyposis Colon, gastric
STK11 Peuĵ-Jeghers Hamartomas—GI and mucocutaneous. Malignancies—breast,
colon, pancreatic, gastric, ovarian, lung, small intestine,
endometrial, testicular, esophageal
PTEN Cowden Breast, facial lesions, GI hamartomas, thyroid, endometrial
BRCA1 Breast (87% risk), ovarian (40%–60%)
BRCA2 Breast (80%), ovarian (15%–20%), prostate, pancreatic
CHAPtEr 33 Oncology and Tumor Biology 419

Multiple Enocrine Neoplasia (MEN) Synromes

MC Clinical Organs/tumors What to treat


Syndrome Gene Protein Inheritance presentation involved ęrst
MEN 1 11q13 Menin Autosomal Hypercalcemia Pancreatic NET Pituitary Hyperparathyroidism
Dominant Hyperparathyroidism
MEN 2A 10q11.21 RET Autosomal Medullary Medullary Thyroid CA Pheochromocytoma
Dominant Thyroid Pheochromocytoma
Cancer Hyperparathyroidism
MEN2B 10q11.21 RET Autosomal Medullary Medullary Thyroid CA Pheochromocytoma
Dominant Thyroid Pheochromocytoma
Cancer Mucosal Neuromas
Marfanoid Habitus
420 PArt ii Medical Knowledge

Questions
1. A 44-year-ol male with a history of hypertension 4. An 87-year-ol female presents to the emergency
well controlle on meications is foun to be epartment (ED) with weight loss, vomiting,
anemic an with a positive fecal occult bloo obstipation, an a istene abomen. She has
test uring his yearly physical. He notes a family not ha a bowel movement in 3 ays. Past history
history that inclues eaths ue to colon cancer is signicant for a non-ST segment elevation
in of both his mother at age 46 an his maternal myocarial infarction (NSTEMI) 6 weeks earlier.
granfather at age 51. He unergoes colonoscopy A compute tomography (CT) scan with oral
which emonstrates four aenomatous polyps in contrast shows evience of an obstructing mass
the ascening colon as well as an aenocarcinoma in the sigmoi colon. However, the lumen oes
of the ileocecal junction. This patient is most appear to be patent. Her vitals are stable. Which
likely to have which of the following? of the following is the best recommenation?
A. A mutation in the TP53 gene A. Diverting ileostomy
B. A mutation in the PMS gene B. Diverting transverse colostomy
C. A mutation in the PTEN gene C. Open sigmoi resection with proximal
D. A mutation in the STK11 gene colostomy
E. A mutation in the aenomatous polyposis coli D. Colonoscopy with placement of a temporizing
(APC) gene stent followe by elective surgery
E. Laparoscopic sigmoi resection with proximal
2. An otherwise healthy 68-year-ol woman colostomy
is iagnose with locally avance gastric
aenocarcinoma. There is no evience of istant 5. A patient with metastatic sigmoi colon cancer
metastases. Her tumor is biopsie an note to is about to unergo chemotherapy, an the
have HER2/neu overexpression. She is starte oncologist recommens the use of an anti-EGFR
on an appropriate chemotherapy regimen. After monoclonal antiboy. Which of the following
her secon cycle, she presents with new-onset genetic proles is most likely to benet from the
yspnea on exertion an orthopnea. Which of aition of this agent?
the following chemotherapeutic agents is likely A. K-ras wiltype gene
responsible for her symptoms? B. BRAF mutation
A. Bleomycin C. NRAS
B. 5-Fluorouracil D. PIK3CA mutation
C. Vinblastine E. K-ras mutant gene
D. Trastuzumab
E. Cisplatin 6. Which of the following patients shoul be
referre to a genetic counselor for BRCA testing?
3. A 70-year-ol otherwise healthy male with a A. Family history of breast cancer in mother at
history of colon aenocarcinoma that was treate the age of 55
with a formal resection returns two years later B. Both parents are Sepharic Jews
with a 3-cm lesion on his liver. Workup conrms C. Aopte an unknown family history,
a colorectal metastasis with no evience of sprea evelope breast cancer at 55
elsewhere. Which of the following is the most D. 55-year-ol female with breast cancer in
appropriate next step? bilateral breasts
A. Chemotherapy only E. 55-year-ol female with an inammatory
B. Surgical resection only breast cancer
C. Surgical resection followe by chemotherapy
D. Chemotherapy followe by surgical resection
E. Surgical resection followe by raiation
CHAPtEr 33 Oncology and Tumor Biology 421

7. A 55-year-ol male presents to the ED with 10. Which of the following is true regaring the
vomiting an an inability to tolerate oral intake evelopment of skin cancers?
for the last week. CT scan shows a signicantly A. Ultraviolet (UV) raiation both initiates an
istene stomach, with a thickene mass near promotes DNA amage
the pylorus. Upper enoscopy shows a large B. UVA is the ultraviolet frequency most
mass in the stomach that partly occlues the responsible for chronic skin amage
istal lumen. Biopsy is consistent with low-grae C. An increase level of skin melanin increases
mucosa-associate lymphoi tissue (MALT) the risk of eveloping basal cell carcinoma
lymphoma. He takes proton-pump inhibitors D. UV raiation amages the DNA mismatch
for aci reux. Which of the following is true repair gene
regaring his conition? E. Mutations in the BCL- gene are a known
A. Triple antibiotic therapy for eraication of mechanism for the evelopment of skin cancer
Helicobacter pylori shoul be starte regarless
of whether the patient is H. pylori positive or 11. A 43-year-ol male is iagnose with a high-
negative grae right lower extremity osteosarcoma an
B. The patient shoul be given chemotherapy unergoes surgical resection an ajuvant
along with triple antibiotic therapy chemotherapy with MAP (methotrexate,
C. Gastrectomy has no role in the treatment of oxorubicin, an cisplatin). After the thir
gastric MALT lymphoma treatment cycle, the patient evelops severe
D. Raiotherapy has no role in the treatment of nausea, vomiting, an altere mental status.
gastric MALT lymphoma Workup reveals increase liver transaminases, a
E. Surgery is recommene for patients who o reuction in glomerular ltration rate (GFR), as
not respon to triple antibiotic therapy well as leukopenia an thrombocytopenia. What
meication can potentially reverse these effects?
8. Which of the following is true regaring the A. Cobalamin
interaction between raiation therapy an tumor B. Folinic aci
cells? C. Folic aci
A. Raiation therapy leas to cancer cell eath D. Folate
by irectly inhibiting aenosine triphosphate E. Omeprazole
(ATP) prouction in the mitochonria
B. Larger tumors are more sensitive to raiation 12. Which of the following statements is true
therapy regaring patterns of metastatic sprea?
C. As the energy use in raiation therapy A. The most common metastatic location for
increases, collateral amage to overlying skin breast cancer is the arenal glan
also increases B. The most common metastatic location for
D. The S phase of the cell cycle is most sensitive melanoma is the small bowel
to raiation effects C. Metastases to the arenal glan most
E. Correcting anemia can increase the efcacy of commonly originate in the lungs
raiotherapy D. The most common metastatic location for
colon cancer is the lungs
9. A 60-year-ol male with cirrhosis presents to E. The transverse colon is frequently the rst
clinic with a newly iagnose 4-cm hepatocellular location of metastatic sprea of pancreatic
carcinoma (HCC) in segment 6. There is no cancer
evience of gross vascular invasion an no
regional noal or extrahepatic istant metastases.
His international normalize ratio (INR) is 1.8,
creatinine is 1.0 mg/L, bilirubin is 3.1 mg/L,
an albumin is .6 mg/L, an his compute
tomography (CT) scan shows no evience of
ascites. Which of the following woul be the best
treatment option?
A. Transarterial chemoembolization (TACE)
B. Liver resection
C. Raiofrequency ablation (RFA)
D. Irreversible electroporation
E. Liver transplantation
422 PArt ii Medical Knowledge

13. Which of the following statements is true 14. A 77-year-ol male who resies in a subacute care
regaring the human protein p53? facility has just nishe ajuvant chemotherapy
A. Germline mutations of the p53 gene result in (FOLFOX an Bevacizumab) for metastatic
Cowen Synrome colon cancer. Despite a normal albumin, minimal
B. The unregulate growth seen with human weight loss, an meticulous local woun care,
papillomavirus (HPV) is partly ue to bining his nurses have been unable to aequately treat
an inactivation of the p53 protein a nonhealing sacral ecubitus ulcer. The woun
C. The p53 gene suppresses the translation base looks clean, an he has no signs of systemic
process in DNA sequencing an cell growth infection. Which of the following is true?
D. Overexpression of this gene leas to A. The sacral woun shoul be preemptively
uncontrolle cell growth ebrie to avoi infection an facilitate
E. Mutations frequently result in benign woun healing
neoplastic growth rather than malignancy B. Supplemental enteral nutrition will facilitate
faster woun healing
C. Rescue therapy can be attempte with leucovorin
D. The patient shoul be converte to Cetuximab
E. Barriers to healing will likely resolve in 6 months

Answers
1. B. This patient has Lynch synrome (hereitary nonpol- Mayer RJ. Lower gastrointestinal cancers. In: Jameson J, Fauci
yposis colorectal cancer—HNPCC). The Amsteram crite- AS, Kasper DL, Hauser SL, Longo DL, Loscalzo J. es. Harrison’s
ria ene the criteria necessary for iagnosis. They inclue principles of internal medicine. 0th e. McGraw-Hill; 018.
Morris A. Epiemiology—clinical risk factors—familial cancer
three or more family members who have been iagnose
synromes. In: Greenel LJ, Mulhollan MW, es. Greeneld’s sur-
with an HNPCC-associate cancer (colorectal, enometrial,
gery: scientic principles & practice. 5th e. Lippincott Williams an
gastric, ovarian, GU tract, hepatobiliary, small bowel, an Wilkins; 010.
CNS—but most commonly colorectal cancer), one of whom
is a rst-egree relative of the other two; at least two gener- 2. D. Commonly utilize chemotherapy regimens in gas-
ations of family members involve; an at least one mem- tric cancer inclue FLOT (5-FU, leucovorin, oxaliplatin, an
ber iagnose with colorectal cancer prior to the age of 50. ocetaxel) as well as capecitabine, cisplatin, an epirubicin.
In aition, no family members may have been iagnose In cases of HER/neu overexpression, trastuzumab may be
with FAP. HNPCC is characterize by mutations in mis- ae. Trastuzumab is a monoclonal antiboy therapy use in
match repair genes (MLH1, MSH, MSH6, PMS, EPCAM), the treatment of HER/neu overexpressing cancers (most com-
resulting in microsatellite instability (MSI), an is inherite monly breast an GI origins). Trastuzumab can cause revers-
in an autosomal ominant fashion. Aitionally, there is a ible cariomyopathy (D). 5-uorouracil (5-FU) is a component
high frequency of cancers in HNPCC arising in the proxi- of the FLOT regimen for gastric cancer, but cariomyopathy
mal colon when compare to other hereitary colorectal can- is not a common sie effect (B). Bleomycin is utilize in the
cer synromes (B). A mutation in the TP53 gene results in treatment of lymphoma, testicular, ovarian, an cervical can-
Li Fraumeni synrome which is characterize by tumors of cers an can cause pulmonary brosis (A). Vinblastine is not
the breast (90%), colon, lung, brain, an arenal, as well as a typical therapeutic agent in the treatment of gastric cancer
sarcomas (A). PTEN mutations result in Cowen synrome, an can cause myelosuppression (C). Cisplatin is frequently
in which patients evelop tumors of the breast, thyroi, an utilize in the treatment of gastric cancer, but its sie effects
enometrium as well as facial lesions an GI hamartomas inclue nephrotoxicity, neurotoxicity, an ototoxicity (E).
(C). Peutz-Jeghers synrome is ue to a mutation in the References: Sah BK, Zhang B, Zhang H, et al. Neoajuvant
STK11 gene an is characterize by hamartomas (both GI an FLOT versus SOX phase II ranomize clinical trial for patients with
mucocutaneous) as well as malignancies of the breast, colon, locally avance gastric cancer. Nat Commun. 00;11(1):6093.
pancreas, stomach, ovaries, lung, small intestine, enome- Wagner AD, Syn NL, Moehler M, et al. Chemotherapy for
trium, testicles, an esophagus (D). Familial aenomatous avance gastric cancer. Cochrane Database Syst Rev. 017;8:CD004064.
polyposis (FAP) is ue to a mutation in the APC gene an
is typically characterize by the appearance of thousans of 3. C. Recent literature shows a conferre survival benet
aenomatous polyps throughout the colon early in life (E). for the resection of hepatic metastases in colorectal cancer.
References: Greenel LJ, Mulhollan MW, es. Greeneld’s Multiple high-volume centers have emonstrate the 5-year
surgery: scientic principles & practice. 5th e. Lippincott Williams an survival for patients with metastatic colorectal cancer to the
Wilkins; 010. liver to be 5% to 58% with resection of the metastatic lesion.
CHAPtEr 33 Oncology and Tumor Biology 423

Over the last two ecaes, the perioperative mortality associ- number of lymph noes), an etect the presence of any syn-
ate with hepatic resection has fallen signicantly, with most chronous lesions. Aitionally, this can allow for the meical
high-volume centers reporting a 30-ay perioperative mor- optimization of the patient’s comorbiities. Although stent-
tality of less than %. The presence of any of the following ing has multiple benets, a recent prospective ranomize
risk factors ha a negative, an aitive, effect on survival stuy emonstrate no avantage to stenting over emer-
in patients with hepatic metastases from colorectal cancer: gency surgery. However, in an 87-year-ol female with a
(1) noe-positive primary tumor, () isease-free interval recent NSTEMI, operative risk woul be prohibitive. Despite
less than 1 months, (3) multiple liver metastases, (4) larg- the potential immeiate benets of temporizing stents, the
est hepatic metastasis greater than 5 cm, an (5) serum carc- possible implications for the long-term results of oncologic
inoembryonic antigen (CEA) level greater than 00 ng/mL. treatment remain to be seen. However, obstruction must still
Those with none of these risk factors have the greatest 5-year be treate surgically if stenting is not possible (A–C, E).
survival at 60%. Treatment will vary epening on whether References: Abussamet Bozkurt M, Gonenc M, Kapan S,
it is a synchronous or metachronous lesion. Synchronous Kocatasş A, Temizgönül B, Alis H. Colonic stent as brige to sur-
lesions can be safely treate with combine colon an liver gery in patients with obstructive left-sie colon cancer. JSLS.
resection, provie the hepatic resection is limite (<3 seg- 014;18(4):e014.00161.
Park SJ, Lee KY, Kwon SH, Lee SH. Stenting as a brige to sur-
ments). By combining the two surgeries, initiation of aju-
gery for obstructive colon cancer: oes it have surgical merit or onco-
vant chemotherapy is quicker. Interestingly, for synchronous
logic emerit? Ann Surg Oncol. 016;3(3):84–848.
rectal cancer (that is both nonobstructing an nonbleeing) van Hooft JE, Bemelman WA, Olenburg B, et al. Colonic
with liver metastasis, some experts are now avocating liver stenting versus emergency surgery for acute left-sie malignant
resection rst, followe by chemoraiation therapy (because colonic obstruction: a multicentre ranomise trial. Lancet Oncol.
this therapy may ownstage the rectal cancer). For metachro- 011;1(4):344–35.
nous isease, the timing of surgery an chemotherapy is still
controversial but seems to lean more heavily towar a sur- 5. A. In 01, the Foo an Drug Aministration (FDA)
gery-rst treatment strategy (B). Norlinger an colleagues approve cetuximab, an anti-EGFR monoclonal antiboy,
publishe the results of a large ranomize trial compar- to be use with FOLFIRI, as the rst-line treatment of k-ras
ing surgery alone versus perioperative chemotherapy an mutant negative (wiltype) metastatic colorectal cancer. This
surgery in patients with resectable liver metastases, which approval was largely base on the CRYSTAL trial, as well as
showe a higher rate of complications in the preoperative two other supportive stuies. A statistically signicant over-
chemotherapy group an no ifference in survival. Many all survival an progression-free survival were appreciate
have use this to infer that preoperative chemotherapy is in the cetuximab group (3.5 months versus 19.5 months).
eleterious without conferre benet, but the stuy was The recommene ose an scheule for cetuximab is
not powere to examine survival as a primary enpoint (D). 400 mg/m aministere intravenously as a 10-minute
In this potentially curable patient, surgery rst is likely to infusion as an initial ose, followe by 50 mg/m infuse
confer the largest survival benet. Patients with unresect- over 30 minutes weekly in combination with FOLFIRI. Other
able isease, or other poor prognostic inicators, shoul be stuies have emonstrate the negative effects an poor
consiere for systemic chemotherapy, followe by restag- response rate cetuximab has in patients with mutations in
ing an consieration for surgical therapy (A). Raiation is BRAF, NRAS, an PIK3CA (B–D). K-ras mutations are seen
never part of the treatment algorithm for colon cancer (E). in 35% to 45% of patients with colorectal cancer, an this
References: Martin RCG n, Augenstein V, Reuter NP, Scog- group of patients will not benet from cetuximab therapy.
gins CR, McMasters KM. Simultaneous versus stage resection The most common mutations are on chromosome 1 an 13.
for synchronous colorectal cancer liver metastases. J Am Coll Surg. These have also been shown to preict treatment failure with
009;08(5):84–850. cetuximab (E).
Norlinger B, Sorbye H, Glimelius B, et al. Perioperative References: De Roock W, Claes B, Bernasconi D, et al. Effects
FOLFOX4 chemotherapy an surgery versus surgery alone for of KRAS, BRAF, NRAS, an PIK3CA mutations on the efcacy of
resectable liver metastases from colorectal cancer (EORTC 40983): cetuximab plus chemotherapy in chemotherapy-refractory meta-
long-term results of a ranomise, controlle, phase 3 trial. Lancet static colorectal cancer: a retrospective consortium analysis. Lancet
Oncol. 013;14(1):108–115. Oncol. 010;11(8):753–76.
Yin Z, Liu C, Chen Y, et al. Timing of hepatectomy in resectable Tan C, Du X. KRAS mutation testing in metastatic colorectal can-
synchronous colorectal liver metastases (SCRLM): simultaneous or cer. World J Gastroenterol. 01;18(37):5171–5180.
elaye? Hepatology. 013;57(6):346–357. Van Cutsem E, Lenz HJ, Köhne CH, et al. Fluorouracil, leucovo-
rin, an irinotecan plus cetuximab treatment an RAS mutations in
4. D. Symptoms of obstruction are the initial presenting colorectal cancer. J Clin Oncol. 015;33(7):69–700.
symptoms in up to 8% of colorectal cancers. Emergency sur-
gery has been classically consiere the treatment of choice 6. D. In December 013, the US Preventive Services Task
for these patients. However, in the majority of stuies, emer- Force recommene that women who have family members
gency colorectal surgery is burene with higher morbiity with breast, ovarian, fallopian tube, or peritoneal cancer be
an mortality rates when compare with elective surgery, evaluate to see if they have a family history that is asso-
an many patients require temporary colostomy, which ete- ciate with an increase risk of a harmful mutation in one
riorates their quality of life an becomes permanent in 10% of the BRCA genes. Some risk factors that increase the like-
to 40% of cases. The aim of a temporizing stent is to avoi lihoo of having one of these harmful genes inclue breast
emergency surgery an plan for elective surgery (which cancer before 50 years ol, cancer in both breasts in the same
can be laparoscopic) in orer to improve surgical results, woman, both breast an ovarian cancers in the same family,
obtain an accurate tumor staging (harvesting the appropriate multiple breast cancers, known BRCA in the family, cases of
424 PArt ii Medical Knowledge

male breast cancer, an Ashkenazi Jewish escent (B). The is spare by the prouction of higher-energy electrons that
others liste may have an increase risk of eveloping breast travel forwar an achieve full intensity at a epth below
cancer as per the GAIL moel, but they have no increase risk the skin’s surface (C). Tissue hypoxia has been shown to
that woul necessitate genetic counseling (A, E). For aopte signicantly reuce raiation amage an is one of the
patients, the recommenation for genetic testing is given only patient-moiable factors that is actively being researche
if they have ha breast cancer at a younger than 50 years (C). to improve the effectiveness of raiotherapy. The relative
References: U.S. Preventive Services Task Force. Risk assessment, hypoxia within large tumor cells is one of the reasons they
genetic counseling, and genetic testing for BRCA-related cancer in women: ten to be more resistant to raiation (B). Along this theme,
clinical summary of USPSTF Recommendation. U.S. Preventive Services systemic anemia seems to have a eleterious effect on raio-
Task Force; 013. AHRQ Publication No. 1-05164-EF-3. therapy an correction before raiation therapy is helpful.
In regar to the cell cycle, M phase has been foun to be the
7. A. Gastric MALT lymphoma is a subset of slow-growing most vulnerable stage to raiation therapy (D).
non-Hogkin lymphoma that typically occurs in the setting Reference: Harrison LB, Chaha M, Hill RJ, Hu K, Shasha D.
of chronic H. pylori infection. While these tumors were orig- Impact of tumor hypoxia an anemia on raiation therapy out-
inally treate with surgical resection, like most lymphomas, comes. Oncologist. 00;7(6):49–508.
the focus has move away from surgery. Initially, systemic
therapy mimicke that of other gastric lymphomas with 9. E. Once the iagnosis of HCC is establishe, the choice of
goo response rates to systemic chemotherapy an raio- therapy must be iniviualize for each patient an base on
therapy alone, as oppose to surgery. However, as the con- tumor buren, presence of unerlying liver isease, patient
nection between H. pylori an gastric MALT lymphoma performance status, an the overall possibility of sie effects
became more apparent, initial therapy has now move or complications balance with acceptable results. When fea-
towar attempte treatment with H. pylori eraication. For sible, anatomic resection is the treatment of choice in patients
patients who o not respon, have a recurrence, or are met- without liver isease an appears to be superior to simple
astatic at time of iagnosis, chemotherapy an raiation are wege resection. There is a growing boy of evience sug-
recommene (B, D). Zullo et al. were even able to emon- gesting that RFA may be use in select patients with similar
strate treatment response in H. pylori–negative patients an survival benets to surgical resection. Feng et al. ranomize
avocate for a trial of eraication in all patients with gastric 168 patients with small (<4 cm) HCCs to surgical resection or
MALT lymphoma regarless of H. pylori status. While the RFA. There was no statistical ifference in survival between
role of surgical intervention is extremely limite, it remains the two groups, though complications were signicantly
the treatment strategy of choice in patients with complete lower in the RFA group. That being sai, locoregional thera-
gastric outlet obstruction who o not respon to meical pies (RFA, irreversible electroporation, proton beam therapy)
therapy or those with uncontrollable bleeing (C, E). are typically reserve for tumors that are not amenable to sur-
References: Mahvi D, et al. Stomach. In: Townsen CM Jr, Beau- gical resection or as brige therapy to transplant (C). The best
champ RD, Evers BM, et al, es. Sabiston textbook of surgery: the bio-
results have been seen with tumors that are less than 4 cm
logical basis of modern surgical practice. 19th e. W.B. Sauners; 01.
in size. Irreversible electroporation (Nanoknife) therapies
Yoon SS, Coit DG, Portlock CS, Karpeh MS. The iminishing
role of surgery in the treatment of gastric lymphoma. Ann Surg. show some promise but are still not inclue in the current
004;40(1):8–37. National Comprehensive Cancer Network (NCCN) guie-
Zullo A, Hassan C, Riola L, et al. Eraication therapy in Heli- lines for treatment of HCC (D). Patients with liver isease
cobacter pylori-negative, gastric low-grae mucosa-associate an elevate bilirubin are less likely to tolerate any surgical
lymphoi tissue lymphoma patients: a systematic review. J Clin Gas- intervention. In fact, the Barcelona Clinic Liver Cancer group
troenterol. 013;47(10):84–87. ientie the absence of clinically relevant portal hyper-
tension an a normal bilirubin level as major eterminants
8. E. Despite longstaning use in the treatment of cancer, for successful liver resection (B). The only treatment moal-
the complete mechanism of raiotherapy-inuce cancer ity left for cirrhotics with HCC is liver transplantation. The
cell eath has yet to be fully eluciate. Charge particles, most wiely use stanar to choose appropriate patients
usually photons, are elivere to the target cells by one of is known as the Milan criteria, an it is use by the Unite
three mechanisms: external beam, brachytherapy, or as a Network for Organ Sharing (UNOS) to select caniates. The
raioactive isotope (e.g., ioine-131 in thyroi cancer). These Milan criteria are as follows: a single tumor less than or equal
charge particles interact with the outer layer of loosely to 5 cm or up to three tumors with none larger than 3 cm, an
boun electrons in normal atoms. Energy is transferre from no evience of vascular invasion, regional lymphaenopathy,
the photon, an the electron is eecte out of orbit with a or istant isease. TACE is another useful therapy for ini-
lower energy, creating a “free raical.” This effect is calle viuals not eligible for resection or regional treatment ue to
the Compton effect. The energy issipate by these ioniz- severity of their cirrhosis or other comorbiities (A). How-
ing events leas to the isruption of chemical bons, most ever, it is still contrainicate in Chil class C cirrhosis or for
importantly those in DNA. While ionizing raiation has a cases in which the location preclues selective treatment. The
irect effect on DNA in certain cells, it also inirectly affects only chemotherapy currently approve for HCC is sorafenib,
other cells by forming oxygen-free raicals (A). The most which has been shown to slightly improve survival from 7.9
important effect seems to be the creation of ouble-strane to 10.7 months.
DNA breaks. While normal cells can repair this amage to References: Bruix J, Castells A, Bosch J, et al. Surgical resec-
some egree, tumor cells often have amage or inhibite tion of hepatocellular carcinoma in cirrhotic patients: prog-
DNA repair mechanisms. As the energy of the photon beam nostic value of preoperative portal pressure. Gastroenterology.
increases, the penetration of tissue also increases. The skin 1996;111(4):1018–10.
CHAPtEr 33 Oncology and Tumor Biology 425

Mazzaferro V, Regalia E, Doci R, et al. Liver transplantation for for colon cancer to sprea to the lungs, the liver is more com-
the treatment of small hepatocellular carcinomas in patients with cir- mon (D). Pancreatic metastases can be seen throughout the
rhosis. N Engl J Med. 1996;334(11):693–700. abominal cavity, but the liver is frequently the rst location
National Comprehensive Cancer Network. NCCN Clinical Prac- following locally invasive isease (E). While the most com-
tice Guielines in Oncology: Hepatobiliary Cancers. Hepatocellular
mon metastatic tumor of the small bowel is from melanoma,
cancer current guielines. National Comprehensive Cancer Net-
work. 016; Version 1.016.
melanoma frequently spreas to the lungs rst (B).

10. A. UV raiation is a known risk factor for squamous 13. B. p53 is a protein encoe by the tumor suppressor
cell carcinoma, basal cell carcinoma, an possibly malig- gene TP53 that is locate on the short arm of chromosome
nant melanoma. It acts as both an initiator an a promoter 17p13.1. It is important for cell cycle regulation, DNA rep-
of irect DNA amage an amage of DNA repair mecha- lication, an apoptosis in response to DNA amage. The
nisms. The egree of risk epens on the type of UV rays an p53 protein bins to sequences of DNA in the promoter
the intensity of exposure. A higher quantity of melanin in region of other genes to enhance or regulate transcription
skin is protective (C). The UV portion of the electromagnetic (C). p53 typically interacts with an enhances the effects of
spectrum can be ivie into three wavelength ranges— genes involve in the inhibition of cell growth or replication
UVA (30–400 nm), UVB (80–30 nm), an UVC (00–80 (D). Mutations in the TP53 tumor suppressor gene result in
nm). Of these, UVB is the most signicant contributor to skin unregulate cell growth an a preisposition to the evel-
amage (B). The mechanism of carcinogenicity by UVB is by opment of malignant neoplasms (E). Li-Fraumeni synrome
formation of pyrimiine imers in DNA (D). This amage is an autosomal ominant, hereitary isorer character-
can be repaire by the nucleotie excision repair pathway. ize by a germline mutation of the TP53 tumor suppressor
With excessive sun exposure, it is postulate that the capac- gene (A). However, it can also arise sporaically an is seen
ity of this pathway is overwhelme, an some DNA that is in more than half of all human cancers. HPV, for example,
amage remains unrepaire. Mutations in the ras an p53 encoes the protein E6, which bins an inactivates the p53
genes occur early in skin cancers, mainly at the ipyrimiine protein. This, in part, contributes to the evelopment of cer-
sequences. The BCL- gene is involve in regulating cell vical ysplasia.
apoptosis (E). References: Angeletti PC, Zhang L, Woo C. The viral etiology
of AIDS-associate malignancies. Adv Pharmacol. 008;56:509–557.
References: Marcus C, et al. Tumor biology an tumor markers.
Muller PAJ, Vousen KH. P53 mutations in cancer. Nat Cell Biol.
In: Townsen CM Jr, Beauchamp RD, Evers BM, et al., es. Sabiston
013;15(1):–8.
textbook of surgery: the biological basis of moern surgical practice.
19th e. W.B. Sauners; 01.
Ziegler A, Leffell DJ, Kunala S, et al. Mutation hotspots ue to 14. E. Bevacizumab (Avastin) is a humanize monoclonal
sunlight in the p53 gene of nonmelanoma skin cancers. Proc Natl antiboy against vascular enothelial growth factor (VEGF).
Acad Sci U S A. 1993;90(9):416–40. It has been shown to signicantly prolong survival when
ae to intravenous 5-uorouracil-base chemotherapy
11. B. Folinic aci, also known as leucovorin, is frequently in rst-line chemotherapy for metastatic colorectal cancer.
given as “rescue therapy” for methotrexate toxicity. Folinic Unfortunately, bevacizumab has numerous averse effects,
aci is a 5-formyl erivative of tetrahyrofolic aci that oes with elaye woun healing being one of the most preva-
not require the action of ihyrofolate reuctase (DHFR) for lent. The inhibitory effect on VEGF receptors limits angio-
its conversion an therefore is not affecte by methotrex- genesis, which is critical in woun healing. Potentially, the
ate’s inhibitory action on DHFR. While the mechanism is not most evastating complication is spontaneous bowel perfo-
fully unerstoo, proton pump inhibitors, such as omepra- ration, but this is relatively infrequent. The effects of the che-
zole, elay the elimination of methotrexate an can poten- motherapy regimen on woun healing last about 6 months,
tially increase toxicity. These meications shoul be stoppe with no stuies showing an effect on woun healing after
uring therapy, if possible (E). Folate is the natural form of this time perio (E). In a patient that is alreay showing
vitamin B9, while folic aci is the equivalent synthetic form. signs of impaire woun healing, aitional surgery will
Both are reliant on the DHFR for metabolism an will have likely be unhelpful an potentially eleterious, especially in
no effect on methotrexate toxicity (C, D). Cobalamin, or vita- the absence of clinical signs of infection (A). Supplemental
min B1, can be effective in treating megaloblastic anemia, nutrition in the absence of proven nutritional ecit has not
but this will have no effect on the myelosuppression cause been shown to improve woun healing (B). Leucovorin, or
by methotrexate (A). folinic aci, is given in conjunction with 5-FU to reuce sie
References: Jiranantakan T. Methotrexate. In: Olson KR, e. Poi- effects but has no effect on bevacizumab (C). Cetuximab, a
soning & rug overose. 6th e. McGraw-Hill; 01. monoclonal antiboy against epiermal growth factor recep-
Suzuki K, Doki K, Homma M, et al. Co-aministration of proton tor (EGFR), has shown to improve survival when use with
pump inhibitors elays elimination of plasma methotrexate in high-
FOLFIRI compare with bevacizumab. However, woun
ose methotrexate therapy. Br J Clin Pharmacol. 009;67(1):44–49.
healing complications were foun to be no ifferent (D).
12. C. Metastatic sprea to the arenal glans is common References: Heinemann V, von Weikersthal LF, Decker T,
et al. FOLFIRI plus cetuximab versus FOLFIRI plus bevacizumab
with breast an lung cancer, with the latter being more prev-
as rst-line treatment for patients with metastatic colorectal can-
alent. While breast cancer is able to sprea to the brain via cer (FIRE-3): a ranomise, open-label, phase 3 trial. Lancet Oncol.
Batson’s plexus, the most common location of metastatic is- 014;15(10):1065–1075.
ease is the lungs (A). Colon cancer spreas in a preictable Scappaticci FA, Fehrenbacher L, Cartwright T, et al. Surgical
pattern, starting with the corresponing noal basin an then woun healing complications in metastatic colorectal cancer patients
following the portal system to the liver. Though it is possible treate with bevacizumab. J Surg Oncol. 005;91(3):173–180.
Pharmacology
ERIC O. YEATES, AREG GRIGORIAN, AND CHRISTIAN DE VIRGILIO 34
ABSITE 99th Percentile High-Yields
I. Pharmacology Terms
A. Pharmacokinetics: what the boy oes to the rug
1. Bioavailability: fraction of the rug that reaches the systemic circulation
. First-pass effect: rug gets metabolize before reaching systemic circulation (usually in liver)
a) Sublingual/rectal rugs o not have rst-pass metabolism
3. Half-life: the time it takes concentration of the rug to be reuce by 50%
4. Steay-state concentration: the point at which the concentration of the rug stays consistent
a) Takes 4 to 5 half-lives to reach steay-state
B. Pharmacoynamics: what the rug oes to the boy
1. Tachyphylaxis: less effective with subsequent oses of a rug

II. Cytochrome P450 (CYPs): essential for metabolism of many meications incluing coumain
A. Inhibitors: block the metabolic activity of one or more CYP enzymes (can lea to bleeing an
supratherapeutic INR in patients taking coumain unless ose is ecrease)
1. Examples: amioarone, cimetiine, ciprooxacin, uconazole, ketoconazole, metroniazole,
trimethoprim/sulfamethoxazole, isoniazi, uoxetine, verapamil, erythromycin
B. Inucers: increase CYP activity by increasing enzyme synthesis (may prevent coumain from working
unless ose is increase)
1. Examples: carbamazepine, phenytoin, phenobarbital, rifampin, St. John’s Wort

III. List of Antiotes/Reversal Agents

Toxin/drug Antidote/reversal agent


Benzodiazepines Flumazenil
Opioids Naloxone (very short half-life, will need repeat dosing/drip)
Acetaminophen N-acetylcysteine
Cyanide Sodium/amyl nitrite, sodium thiosulfate, hydroxycobalamin
Beta-blocker Glucagon
Methanol/ethylene glycol Fomepizole (preferred), ethanol
Iron Deferoxamine
Atropine Physostigmine, neostigmine
Warfarin Vitamin K
Amatoxin (poisonous mushrooms) Silibinin
Lidocaine Intravenous lipid emulsion

427
428 PArt ii Medical Knowledge

IV. Antiiarrheal
A. Loperamie: mu-opioi receptor agonist
B. Lomotil (iphenoxylate/atropine): opioi receptor agonist

V. Averse Reactions to Psychotropic Drugs


A. Serotonin synrome: altere mental status, autonomic nervous system isturbances, neurologic
manifestations, hyperreexia, clonus, myriasis
1. Pathophysiology: abnormal elevation in serotonin levels ue to rugs that increase its release,
ecrease uptake/metabolism, or increase serotonin precursors; most commonly with concurrent use
of multiple serotonergic agents (can also occur after initiation of single serotoninergic agents)
. Common inciting meications: tricyclic antiepressants, selective serotonin reuptake inhibitor,
serotonin an norepinephrine reuptake inhibitor, monoamine oxiase inhibitors, triptans,
nefazoone, buspirone, mirtazapine, carbamazepine, tramaol, linezoli, MDMA (ecstasy),
extromethorphan, St. John’s wort, lithium, methaone, cocaine, levoopa, reserpine, an
amphetamines
3. Management: iscontinue serotonergic agents, supportive care, can use benzoiazepines for agitation
B. Neuroleptic malignant synrome (NMS): severe muscle rigiity, hyperpyrexia (>38°C), altere mental
status, autonomic instability, hyporeexia, normal pupils
1. Cause: opaminergic receptor antagonism cause by antipsychotic rug use that triggers a series of
homeostatic responses that result in autonomic ysregulation an hyperthermia, muscular rigiity,
an altere mental status
. Common inciting meications: more common with atypical antipsychotics (clozapine, olanzapine,
risperione, quetiapine) than typical antipsychotics (haloperiol, chlorpromazine, prochlorperazine,
uphenazine)
3. Management: iscontinue opaminergic antagonists, supportive care, bromocriptine, an antrolene
CHAPtEr 34 Pharmacology 429

Questions
1. A 75-year-ol male is amitte to the hospital 4. A 57-year-ol Chil class A cirrhotic male
after blunt trauma. Two ays after his trauma presents to the ED with severe left lower quarant
he evelops severe agitation. He is treate pain. Physical exam is concerning for peritonitis,
with quetiapine, with little improvement in his an free air uner the iaphragm is seen on
agitation, an his ose is increase over several chest x-ray. In the operating room (OR), he is
ays. On hospital ay ve, he evelops a high foun to have feculent peritonitis seconary to a
fever, profuse sweating, altere mental status, an perforate sigmoi iverticulitis an unergoes
muscular rigiity. Which of the following is true a Hartmann proceure. The following ay the
regaring this conition? respiratory therapist in the ICU has ifculty
A. This conition is more common in younger ventilating an oxygenating. The patient has
patients complete white out of both lung els on x-ray,
B. Typical antipsychotics are more likely to an the PaO/FiO ratio is 180. Low tial volume
cause this conition compare to atypical ventilation is commence an the ecision to
antipsychotics paralyze the patient is mae. Which agent shoul
C. Acute kiney failure is a complication of this be use?
conition A. Rocuronium
D. There are no known pharmacologic treatment B. Vecuronium
options C. Atracurium besylate
E. It is not associate with rhabomyolysis D. Suxamethonium chlorie
E. Pancuronium
2. Which of the following correctly matches the
toxin an antiote? 5. Which of the following is true with regars to
A. Iron an eferoxamine correcting metabolic aciosis?
B. Warfarin an anexanet alpha A. Giving bicarbonate alone will be efcient in
C. Benzoiazepines an fomepizole correcting an aciosis
D. Organophosphates an acetylcysteine B. Correction will x the pulmonary vasoilation
E. Amatoxin an naloxone seen in metabolic aciosis
C. Aministration of soium bicarbonate can
3. A 35-year-ol female sustains a 35% total boy lea to hyperkalemia
surface area burn. She evelops respiratory D. Soium bicarbonate may interfere with oxygen
istress on her secon ay in the hospital. elivery
An arterial bloo gas emonstrates metabolic E. Lactic aciosis will often improve after soium
aciosis with partial respiratory compensation. bicarbonate aministration
Which topical antimicrobial was this patient most
likely receiving? 6. A 6-year-ol female with a known history of
A. Bacitracin chronic pancreatitis an subtotal gastrectomy
B. Silver sulfaiazine presents to the ED with abominal pain, altere
C. Gentamicin mental status, unsteay gait, an aphasia.
D. Collagenase ointment (Santyl) Physical exam is signicant for ophthalmoplegia
E. Mafenie acetate (Sulfamylon) on the right. The patient is confuse an unable
to answer any questions. Which of the following
is the best treatment?
A. Intravenous glucose
B. Oral vitamin B1
C. Intramuscular vitamin B1
D. Parenteral vitamin B1
E. Intravenous magnesium

AL GRAWANY
430 PArt ii Medical Knowledge

7. Which of the following meications woul lea 11. A 56-year-ol male with non-Hogkin lymphoma
to a patient requiring a higher warfarin ose to presents to the emergency epartment (ED)
remain therapeutic? with mental status changes, ecrease urine
A. Ketoconazole output, an lethargy. He recently was starte on
B. Cimetiine chemotherapy. Physical exam is remarkable for a
C. Amioarone newly place implantable venous access evice
D. Rifampin below the right clavicle. The port site has no
E. Allopurinol evience of erythema. Gentle tapping anterior to
his external auitory canal results in contraction
8. Which of the following meications is associate of his facial muscles on that sie. Which of the
with the evelopment of aortic aneurysms an following is true regaring this conition?
issection? A. Dialysis is unlikely to help
A. Cephalosporins B. The risk of this complication has ecrease
B. Statins in the past years with the avent of newer
C. Metformin therapy agents
D. Fluoroquinolones C. Alkalinization of the urine shoul be
E. Azithromycin performe
D. The stanar initial treatment is allopurinol
9. Which of the following is true regaring the E. Laboratory exam will likely emonstrate a
bioavailability of meications? metabolic alkalosis
A. IV ciprooxacin has a similar bioavailability to
the oral form 12. A 78-year-ol female is recovering in the intensive
B. Drugs that are absorbe in the stomach have care unit (ICU) from a small bowel resection ue
better bioavailability than rugs absorbe in to a strangulate femoral hernia. She is known to
the small intestine have longstaning hearing loss. On postoperative
C. Hyrophobic rugs are better absorbe than ay , she becomes increasingly agitate an
hyrophilic rugs confuse. Laboratory exam an infection workup
D. Sublingual meications have lower are unrevealing. She is attempting to pull out her
bioavailability than meications absorbe intravenous (IV) lines. Which of the following is
through the gastrointestinal tract true regaring her conition?
E. The ose of chloramphenicol nees to be A. Lorazepam may worsen her agitation
ecrease in patients when given IV to B. Low oses of iphenhyramine are often
ecrease the chance of toxicity compare to useful
the oral route C. She shoul be place in physical restraints
D. It is unlikely that a hearing ai coul have
10. A 67-year-ol female is brought into the ED prevente this conition
in septic shock of unknown origin. She is E. Haloperiol is contrainicate
hypotensive, iaphoretic, febrile, an foun to
have a leukocytosis an altere mental status. 13. Intravenous aministration of Halol shoul be
A rapi sequence intubation (RSI) is performe. accompanie by:
On hospital ay , the patient continues to have A. A review of amission electrocariogram
hypotension espite ui resuscitation an (ECG) for a prolonge QT interval
the use of vasopressors. She is given a ose of B. A review of amission ECG for Q waves
hyrocortisone an vastly improves. Which of the C. Continuous ECG monitoring for evelopment
following explains the patient’s symptoms? of peake T waves
A. Poor perfusion of the arenal glan in the D. Continuous O saturation monitoring
setting of shock E. Serial serum creatine phosphokinase (CPK)
B. The use of etomiate uring RSI measurements
C. She is on sterois at home
D. Overuse of vasopressors
E. Pituitary ysfunction with insufcient release
of arenocorticotropic hormone (ACTH)
CHAPtEr 34 Pharmacology 431

14. Which of the following is true regaring the 18. A 58-year-ol male postoperative patient
prophylactic role of histamine  (H) blockers evelops a hypertensive crisis with a bloo
an/or proton pump inhibitors (PPIs) in pressure of 0/100 mmHg an heart rate
hospitalize patients? of 60beats per minute. He is starte on a
A. They have a similar rate of upper nitroprussie rip, an the bloo pressure
gastrointestinal bleeing improves. The patient subsequently evelops
B. Effective stress ulcer prophylaxis involves generalize weakness an becomes unresponsive.
achieving an intragastric pH greater than 7 He is immeiately intubate an an arterial bloo
C. Intravenous aministration of PPI results in gas emonstrates a high anion-gap aciosis with
a higher intragastric pH compare with oral a high SvO. His skin color appears pink, an
aministration he has the smell of bitter almons on his breath.
D. There is no ifference in the rate of nosocomial Which of the following shoul you aminister
pneumonia next?
E. Ventilate patients that receive PPI have lower A. Soium nitrite
mortality rates B. Amyl nitrite
C. Soium thiosulfate
15. Which of the following meications is safe to give D. Hyroxycobalamin
a patient who is 10 weeks pregnant? E. Methylene blue
A. Acetaminophen
B. Aspirin 19. A 75-year-ol male with stage 4 chronic kiney
C. Propylthiouracil (PTU) isease (CKD) an symptomatic peripheral
D. Coumain arterial isease is scheule for a catheter-base
E. Lisinopril angiography. Which of the following shoul be
aministere before the stuy?
16. Choose the meication that is correctly paire A. Alkalinization of the urine with soium
with its mechanism of action bicarbonate intravenously
A. Cyclosporine—purine synthesis inhibitor B. N-acetylcysteine
B. Vincristine—microtubule formation an C. Aggressive ui resuscitation with normal
stabilization saline
C. 5-Fluorouracil—thymiylate synthase D. N-acetylcysteine an aggressive ui
inhibitor resuscitation with normal saline
D. Taxol—microtubule inhibitor E. Alkalinization of the urine with soium
E. Iniximab—vascular enothelial growth factor bicarbonate intravenously, N-acetylcysteine,
(VEGF) inhibitor an aggressive ui resuscitation with normal
saline
17. A 5-year-ol male with atrial brillation
presents to the ED with a large biloma ientie 20. Which of the following meications is paire with
on ultrasonography 1 week after unergoing the correct sie effect?
a laparoscopic cholecystectomy. He complains A. Furosemie—nausea
of abominal pain but oes not appear to be B. Metroniazole—tinnitus
in signicant iscomfort. He was restarte on C. Spironolactone—fulminant hepatic necrosis
warfarin after the operation an his international D. Halothane—gynecomastia
normalize ratio (INR) is currently .7. The plan E. Vancomycin—cutaneous ushing
is to attempt CT-guie rainage the following
ay. How shoul his INR be correcte?
A. Oral vitamin K
B. Slow IV infusion (over 30 minutes) of vitamin K
C. Fresh frozen plasma (FFP)
D. Allow warfarin to autocorrect
E. Prothrombin complex concentrate
432 PArt ii Medical Knowledge

Answers
1. C. This patient most likely has neuroleptic malignant agent for burn wouns. However, it oes not have any anti-
synrome (NMS) resulting from new antipsychotic rug microbial properties an oes not cause metabolic aciosis
use. NMS is a rare conition that most often presents with (D). Mafenie acetate (Sulfamylon) is a topical antibiotic
high fevers, muscle rigiity, elirium, an ysautonomia. It with broa-spectrum activity, incluing against P. aeruginosa.
is thought to be a result of opaminergic D receptor antag- Due to its mechanism as a carbonic anhyrase inhibitor, it
onism cause by antipsychotic rug use that triggers a series can cause metabolic aciosis, which can manifest as respira-
of homeostatic responses that result in autonomic ysreg- tory istress (E).
ulation an hyperthermia, muscular rigiity, an altere References: Barillo DJ. Topical antimicrobials in burn woun
mental status. Atypical antipsychotics like clozapine, olan- care: a recent history. Wounds. 008;0(7):19–198.
zapine, risperione, an quetiapine cause NMS more often Dai T, Huang YY, Sharma SK, Hashmi JT, Kurup DB, Hamblin
than typical antipsychotics like haloperiol, chlorpromazine, MR. Topical antimicrobials for burn woun infections. Recent Pat
Antiinfect Drug Discov. 010;5():14–151.
prochlorperazine, an uphenazine (B). First-time usage,
Pham CH, Collier ZJ, Fang M, Howell A, Gillenwater TJ. The
high osages, changes in osages, an parenteral aminis-
role of collagenase ointment in acute burns: a systematic review an
tration of antipsychotics also make NMS more likely. Other meta-analysis. J Wound Care. 019;8(Suppl ):S9–S15.
risk factors inclue oler age, polypharmacy, multiple
comorbiities, an ehyration (A). NMS is a serious coni- 4. C. In a patient with unerlying liver isease, the para-
tion that can result in complications like aspiration an acute lytic of choice is atracurium besylate or cisatracurium. These
renal failure ue to myoglobinuria an rhabomyolysis an are nonepolarizing neuromuscular blocking agents metab-
carries a mortality risk of approximately 10% (C, E). Most olize by Hoffman egraation, thereby bypassing the liver.
experts recommen prompt treatment by ui resuscitation Cisatracurium is approximately 3 times stronger than atra-
an stopping the offening agent. There is some evience curium besylate an is more commonly use in this patient
that aministering bromocriptine (a opamine agonist) is population. Aitionally, cisatracurium oes not lea to
benecial (D). histamine release, resulting in ushing an hypotension
Reference: Tse L, Barr AM, Scarapicchia V, Vila-Roriguez F. when compare to atracurium besylate, making it a better
Neuroleptic malignant synrome: A review from a clinically ori-
alternative. The remaining answer choices are excrete either
ente perspective. Curr Neuropharmacol. 015;13(3):395–406.
wholly or partly by the liver (A, B, D, E).
2. A. Deferoxamine is the treatment for severe iron toxicity.
Anexanet alpha is the reversal agent for apixaban an rivar- 5. D. Persistent metabolic aciosis can lea to wiesprea
oxaban (B). Vitamin K reverses warfarin. Benzoiazepines ysfunction, but most commonly affects the cariovascu-
can be reverse by umazenil (C). Fomepizole is use to lar an respiratory systems. This will result in peripheral
treat methanol an ethylene glycol poisoning. Organophos- vasoilation an pulmonary vasoconstriction in aition to
phate poisoning is treate with atropine (D). Acetylcysteine enzymatic an hormone ysfunction (B). Sympathetic stim-
is the treatment for acetaminophen overose. Amatoxin ulation functions poorly because catecholamines are unable
comes from poisonous mushrooms an can be treate with to exert their effect on tissue amage by a low pH. Bicar-
silibinin, which is mae from milk thistle (E). Naloxone is bonate as an anion alone cannot be given to a patient. It is
use to treat opioi overoses. therefore paire with a hypertonic soium solution (A). The
Reference: Schaper A, Ebbecke M. Intox, etox, antiotes— use of soium bicarbonate oes have some averse effects,
Evience base iagnosis an treatment of acute intoxications. Eur J incluing hypernatremia, hypokalemia, an a left shift in
Intern Med. 017;45:66–70. the oxyhemoglobin issociation curve (C). The left shift is
concerning because this can increase the afnity hemoglo-
3. E. There are a variety of topical regimens to treat the bin has for oxygen an leave tissue hypoxic, which in turn
bacterial loa within burns. Bacitracin is a commonly use will lea to worsening aciosis (E). The main goal of using
topical antibiotic that can be utilize in patients with sulfa soium bicarbonate is to treat patients who are persistently
allergies, but it has notably poor eschar penetration (A). Sil- severely aciotic an are starting to have negative cariovas-
ver sulfaiazine (Silvaene) is one of the most commonly cular symptoms.
use topical treatments for burns an is a combination of
silver nitrate an the antibiotic soium sulphaiazine. It has 6. D. The patient is emonstrating Wernicke encephalop-
broa-spectrum activity but oes not cover for Pseudomonas athy, which is cause by a eciency in thiamine (vitamin
aeruginosa. It shoul not be use in patients with sulfa aller- B1). Thiamine eciency occurs most commonly in alco-
gies an has the potential to cause neutropenia an throm- hol-epenent patients with poor iets. It may also be seen
bocytopenia (B). Gentamicin has antipseuomonal coverage in postgastrectomy patients who are preispose to large
an was once commonly use as a topical cream for burn gastrointestinal losses an can become ecient in this vita-
ressings. It has the potential to cause ototoxicity an neph- min, as well as hyperemesis graviarum, prolonge malnu-
rotoxicity (C). Clostriial collagenase ointment (Santyl), typ- trition, an prolonge parenteral nutrition. Aministration
ically use as an enzymatic ebriing agent in woun care, of thiamine quickly reverses the symptoms, particularly in
has also been shown to be a safe an effective ebriement the setting of acute Wernicke encephalopathy. Aministering
CHAPtEr 34 Pharmacology 433

glucose before thiamine may be counterprouctive because but less so omestically because it can cause life-threatening
glucose oxiation is a thiamine-intensive process an may aplastic anemia. This rug has better bioavailability when
eplete any remaining thiamine that may be available (A). given orally than IV (E). Serum concentrations of IV chlor-
Magnesium may be inicate, particularly in alcoholic amphenicol are only 70% of those achieve when compare
patients, because thiamine aministration may be refrac- with the oral form.
tory in the setting of hypomagnesemia. However, there is References: Drusano GL, Stanifor HC, Plaisance K, Forrest
no information provie in the vignette to suggest that this A, Leslie J, Calwell J. Absolute oral bioavailability of ciprooxacin.
patient is an alcoholic (E). Vitamin B1 eciency will have a Antimicrob Agents Chemother. 1986;30(3):444–446.
more insiious onset an present with macrocytic anemia, Glazko AJ, Dill WA, Kinkel AW, etal. Absorption an excretion
of parenteral oses of chloramphenicol soium succinate in com-
peripheral neuropathy, an ataxic gait. Confusion, aphasia,
parison with per oral oses of chloramphenicol. Clin Pharmacol Ther.
an ophthalmoplegia are not characteristic of vitamin B1
1977;1:104.
eciency (B, C).

7. D. Cytochrome P450 is a part of the superfamily of 10. B. Etomiate is the preferre anesthetic agent for RSI
proteins containing a heme factor an is involve in the because it has minimal cariopulmonary effects. It is also fre-
metabolism of warfarin. There are inhibitors an inucers quently use in the trauma population because it leas to a
of CYP450 that will enhance or ampen the effect of war- ecrease cerebral metabolic rate an may assist in ecreas-
farin, respectively. Clinically relevant inhibitors of CYP450 ing intracranial pressure. One notable isavantage is that it
inclue amloipine, cimetiine, ciprooxacin, cyclosporine, can result in arenal ysfunction because it is a known inhib-
iltiazem, ketoconazole, isoniazi, an propranolol. Patients itor of cortisol synthesis (11β-hyroxylase). A systematic
using these meications will nee to ecrease the ose of review ientie 1 stuies that t criteria evaluating the
warfarin to maintain the same therapeutic international averse effects of etomiate. It emonstrate that patients
normalize ratio (INR). Inucers of CYP450 inclue barbi- that receive etomiate ha an increase relative risk of
turates, phenytoin, prenisone, rifampin, as well as omepra- 1.64 for arenal insufciency an an increase relative risk
zole. Patients on these meications will nee to increase their for mortality of 1.19. A single ose can suppress the arenal
warfarin osage (A–C, E). glan for up to 7 hours. There is no information given to
suggest this patient is on chronic sterois (C). Hypoperfu-
8. D. The FDA has issue a warning regaring the increase sion of the arenal glans in the setting of shock, overuse of
risk of aortic aneurysm an aortic issection in association vasopressors, an pituitary ysfunction are all possible, but
with the use of uoroquinolones (A). Usage shoul be lim- etomiate is more likely given the use of RSI (A, D, E).
ite to patients with serious infections who o not have Reference: Albert SG, Ariyan S, Rather A. The effect of etomi-
other antibiotic options. Statins have been associate with ate on arenal function in critical illness: a systematic review. Inten-
sive Care Med. 011;37(6):901–910.
a reuce rate of aortic aneurysm growth (B). Metformin is
associate with a reuce rate of aortic aneurysm evelop-
ment (C). There is no association between azithromycin an 11. B. Tumor lysis synrome (TLS) is not uncommonly seen
aortic pathology (E). in patients recently starte on chemotherapy an primarily
Reference: Gopalakrishnan C, Bykov K, Fischer MA, Con- occurs in those with poorly ifferentiate lymphoprolifera-
nolly JG, Gagne JJ, Fralick M. Association of uoroquinolones with tive iseases such as lymphomas or leukemia, but may also
the risk of aortic aneurysm or aortic issection. JAMA Intern Med. occur with soli organ tumors. It is commonly characterize
00;180(1):1596–1605. by electrolyte abnormalities that lea to acute renal failure.
Although hyperphosphatemia an hyperuricemia occur
9. A. Bioavailability of a meication refers to the rate at most commonly, they are often accompanie by hyperkale-
which an aministere rug is absorbe by the circulatory mia, hypocalcemia, an a metabolic lactic aciosis (E). The
system. The bioavailability of a meication that is given above patient has a physical exam sign consistent with hypo-
intravenously theoretically has 100% bioavailability, but calcemia; Chvostek sign is muscle spasm with gentle tapping
this oes not always prove to be the case. Generally, the IV over the facial nerve. Newer monoclonal antiboy therapies
route provies a higher bioavailability when compare with have emonstrate a ecrease risk of causing TLS. Treat-
the oral form. One notable exception is ciprooxacin, which ment inclues aggressive hyration in an attempt to normal-
has similar bioavailability with either IV or oral form. Ai- ize the electrolyte abnormalities an improve renal function.
tionally, the location of absorption is important. Most rugs Although alkalinization of urine was thought to be a useful
absorbe in the small intestine have greater bioavailability ajunct in TLS, there are newer stuies suggesting that it
than rugs absorbe in the stomach because the bowel has may contribute to renal ysfunction. This is now consiere
1000-fol increase surface area for absorption compare a controversial ajunct an is not wiely use (C). Allo-
with the stomach (B–C). Meications absorbe by the intes- purinol is use to treat the hyperuricemia of malignancy;
tines are route to the portal circulation rst an therefore are however, this can lea to an increase risk of xanthine an
initially metabolize by the liver; this is known as “rst-pass calcium phosphate crystals. Newer approaches inclue use
metabolism.” Because of this, the meication has a lower ini- of urate oxiase, which can provie effective treatment while
tial bioavailability. However, this oes not hol true for sub- having a safer prole. Hyration remains the best treatment
lingual, rectal, intramuscular, an subermal meications moality (D). In refractory cases, ialysis can be use (A).
because they o not pass through the liver before their sys- References: Davison MB, Thakkar S, Hix JK, Bhanarkar ND,
temic sprea (D). Another notable exception is chloramphen- Wong A, Schreiber MJ. Pathophysiology, clinical consequences, an
icol, an antibiotic use commonly in eveloping countries treatment of tumor lysis synrome. Am J Med. 004;116(8):546–554.
434 PArt ii Medical Knowledge

Firwana BM, Hasan R, Hasan N, et al. Tumor lysis synrome: References: Kalisvaart KJ, e Jonghe JFM, Bogaars MJ, et al.
a systematic review of case series an case reports. Postgrad Med. Haloperiol prophylaxis for elerly hip-surgery patients at risk for
01;14():9–101. elirium: a ranomize placebo-controlle stuy: haloperiol pro-
Howar SC, Trilio S, Gregory TK, Baxter N, McBrie A. Tumor phylaxis for elirium. J Am Geriatr Soc. 005;53(10):1658–1666.
lysis synrome in the era of novel an targete agents in patients Kaneko T, Cai J, Ishikura T, et al. Prophylactic consecutive
with hematologic malignancies: a systematic review. Ann Hematol. aministration of haloperiol can reuce the occurrence of post-
016;95(4):563–573. operative elirium in gastrointestinal surgery. Yonago Acta Medica,
Jeha S. Tumor lysis synrome. Semin Hematol. 001;38(4, 1999;4:179–184.
Suppl10):4–8.
Marin GR, Majek E. Acute kiney injury seconary to steroi-in- 14. D. Gastrointestinal stress ulceration occurs in 1% to 4%
uce tumor lysis in an aolescent with acute lymphoblastic leu- of all critically ill patients with a 50% mortality rate. The use
kemia: role of urinary alkalinisation an peritoneal ialysis. Arch of PPI versus H-blockers has been wiely stuie. A recent
Argent Pediatr. 01;110(6):e118–e1.
metaanalysis of eight ranomize controlle trials looking at
critically ill patients foun no ifference in the rate of noso-
12. A. Patients in the ICU often experience ICU elirium.
comial pneumonia or mortality in either group (E). However,
It has been shown that anywhere between 0% an 80% of
the use of PPI may lea to an increase risk of Clostridium
elerly patients in the ICU will experience elirium. The
difcile infection. Aitionally, the PPI group i have a
Hospital Eler Life Program (HELP) is an inpatient strategy
ecrease rate of clinically signicant upper gastrointesti-
to prevent ICU elirium an focus on primary prevention
nal bleeing (A). It has been emonstrate that achieving an
with the use of regular reorientation, encouraging proper
intragastric pH greater than 6 results in clot stabilization an
sleep-wake cycles, meeting nutritional nees, early mobi-
increase platelet aggregation (B). Intravenous or oral PPIs
lization activities, an proviing visual an hearing aap-
are equally effective in achieving a prophylactic intragastric
tations for patients with sensory impairments (D). Physical
pH to prevent ulcer formation when given at the same ose
restraints shoul be avoie because they lea to ecrease
an frequency (C).
mobility, increase agitation, greater risk of injury, an pro-
References: Alhazzani W, Alenezi F, Jaeschke RZ, Moayyei P,
longation of elirium (C). Certain patients will still require Cook DJ. Proton pump inhibitors versus histamine  receptor antag-
pharmacologic therapy. Benzoiazepines are not uncom- onists for stress ulcer prophylaxis in critically ill patients: a system-
monly aministere to elerly patients for agitation, insom- atic review an meta-analysis. Crit Care Med. 013;41(3):693–705.
nia, an anxiety. However, they are known to have averse Barkun AN, Barou M, Pham CQD, Martel M. Proton pump
an paraoxical effects so their use shoul be limite. inhibitors vs. histamine  receptor antagonists for stress-relate
Patients may experience rowsiness, epression, confusion, mucosal bleeing prophylaxis in critically ill patients: a meta-analy-
vertigo, insomnia, or worsene agitation. When benzoiaz- sis. Am J Gastroenterol. 01;107(4):507–50.
epines are given to patients with ICU elirium, up to 3% Barkun AN, Cockeram AW, Ploure V, Feorak RN. Review arti-
may experience an averse event, incluing hypotension, cle: aci suppression in non-variceal acute upper gastrointestinal
bleeing. Aliment Pharmacol Ther. 1999;13(1):1565–1584.
ystonia, laryngeal spasm, malignant hyperthermia, glucose
Hartmann M, Ehrlich A, Fuer H, et al. Equipotent inhibition of
ysregulation, an urinary retention. Diphenhyramine gastric aci secretion by equal oses of oral or intravenous pantopra-
may emonstrate a similar effect in the elerly population zole. Aliment Pharmacol Ther. 1998;1(10):107–103.
an shoul be use with caution (B). Haloperiol is often the
rst-line treatment in the management of an aggressive an 15. C. PTU has been proven to be safe uring the rst tri-
agitate patient in the context of elirium (E). mester of pregnancy to treat patients with hyperthyroi-
References: Fong TG, Tulebaev SR, Inouye SK. Delirium in ism, while methimazole has fallen out of favor because of
elerly aults: iagnosis, prevention an treatment. Nat Rev Neurol. the increase risk of congenital hypothyroiism (cretinism).
009;5(4):10–0.
ACE-inhibitors have been linke to congenital malforma-
Girar TD, Panharipane PP, Ely EW. Delirium in the intensive
tions an renal failure (E). Coumain crosses the bloo/baby
care unit. Crit Care. 008;1(Suppl 3):S3.
Kruse WH. Problems an pitfalls in the use of benzoiazepines barrier an can lea to skeletal an CNS efects (D). Aspirin
in the elerly. Drug Safety. 1990;5(5):38–344. an acetaminophen have both been linke to increase mis-
carriages an therefore shoul be avoie if at all possible (A,
13. A. Elerly patients in the hospital will often experi- B). Acetaminophen, which ha previously been thought to
ence agitation that can potentiate to aggressive behavior. be safe, has now been linke to hyperkinetic an behavioral
In this type of situation, the patient will nee to be seate isorers such as autism. It is consiere a category B rug.
before hurting himself or others. Haloperiol (Halol) References: Hackmon R, Blichowski M, Koren G. The safety
is often the rst-line treatment in the management of an of methimazole an propylthiouracil in pregnancy: a systematic
aggressive an agitate patient in the context of elirium. review. J Obstet Gynaecol Can. 01;34(11):1077–1086.
Liew Z, Ritz B, Virk J, Olsen J. Maternal use of acetamino-
Before the aministration of haloperiol, an ECG shoul
phen uring pregnancy an risk of autism spectrum isorers
be performe to rule out a prolonge QT synrome that in chilhoo: a Danish national birth cohort stuy. Autism Res.
the rug can potentiate an lea to life-threatening tor- 016;9(9):951–958.
saes e pointes an/or heart failure. Peake T waves
can occur with hyperkalemia, while Q waves are usually
present following myocarial infarction (B, C). Continuous 16. C. 5-Flourouracil, or 5FU, is a thymiylate synthase
oxygen saturation monitoring is not require before Halol inhibitor that inhibits purine an DNA synthesis. When
aministration (D). Serial serum CPK measurements are use in combination with leucovorin, it has increase activ-
recommene for patients receiving continuous infusions ity an increase toxicity. Cyclosporine is an immunosup-
of propofol (E). pressant that bins to cyclophilin proteins an inhibits genes
CHAPtEr 34 Pharmacology 435

for cytokine synthesis, particularly IL-. Sie effects of cyc- in the SvO. The initial treatment is inhale amyl nitrite fol-
losporine inclue nephrotoxicity, hepatotoxicity, tremors, lowe by intravenous soium nitrite (A). These agents are
seizures, an hemolytic uremic synrome (A). Vincristine is consiere methemoglobin inucers, which allow for methe-
a chemotherapeutic agent that works by inhibiting microtu- moglobin to reversibly bin with cyanie to make cyanomet-
bule formation (B). Taxol is also a chemotherapeutic agent hemoglobin. Soium thiosulfate is then aministere, which
but works by microtubule formation an stabilization (D). helps convert cyanomethemoglobin to thiocyanate, a harm-
Finally, iniximab is a monoclonal antiboy against TNF-α. less metabolite that is renally excrete (C). Hyroxycobal-
By bining to TNF-α, it inhibits its ability to bin to receptors amin, a form of vitamin B1, is a new meication use to
an reuces the autoimmune inammatory response. Beva- reverse the effects of cyanie by bining to cyanie to form
cizumab is a VEGF inhibitor an has been emonstrate to cyanocobalamin, which is then excrete through the urine.
improve survival in patients with metastatic colorectal can- Although this rug shows promise, it is not yet the stanar
cer (E). of care (D). Methylene blue is use in the treatment of met-
hemoglobinemia (E).
17. A. Reversing warfarin epens on the clinical situation.
If the patient is actively bleeing, an therefore reversal is 19. C. N-acetylcysteine, alkalinization of the urine, an
urgent, prothrombin factor concentrate is now preferre aggressive ui resuscitation have all been shown to have
over FFP (C, E). However, in a patient that is therapeutically a theoretic benet, but only intravenous ui hyration
anticoagulate an requires an invasive intervention elec- has consistently emonstrate a clinical benet when use
tively, urgent reversal is not neee. The metabolism of war- in patients with CKD unergoing a contrast stuy (A, B, D,
farin is regulate by iet an concomitant meications. The E). Aitionally, the egree of nephrotoxicity is ose epen-
half-life is 48 to 7 hours, which allows the rug to continue ent an increases with ionize contrast versus nonionize
its effects for about 4 to 6 ays after cessation (D). As such, contrast. In patients with normal renal function, the concept
allowing warfarin to autocorrect can take up to 6 ays. This of contrast-inuce nephropathy has recently come uner
patient is in no istress an there are no urgently signs to scrutiny with several reports suggesting no harm.
intervene, an so the correction can be one slowly with the References: Klima T, Christ A, Marana I, et al. Soium chlorie
aministration of oral vitamin K. This takes up to 4 hours to vs. soium bicarbonate for the prevention of contrast meium-
have an effect an is the ieal choice for a patient unergo- inuce nephropathy: a ranomize controlle trial. Eur Heart J.
ing CT-guie rainage the following ay. IV push (over 3–5 01;33(16):071–079.
minutes) aministration of vitamin K is generally iscour- Sun Z, Fu Q, Cao L, Jin W, Cheng L, Li Z. Intravenous
N-acetylcysteine for prevention of contrast-inuce nephropathy:
age because there is a risk of thrombosis an anaphylaxis
a meta-analysis of ranomize, controlle trials. PLoS One.
(B). However, slow IV infusion over 30 minutes is acceptable,
013;8(1):e5514.
an it usually takes 8 to 1 hours for it to have an effect.
References: DeZee KJ, Shimeall WT, Douglas KM, Shumway
NM, O’Malley PG. Treatment of excessive anticoagulation with
20. E. Furosemie is a loop iuretic an can result in hypo-
phytonaione (vitamin K): a meta-analysis. Arch Intern Med.
calcemia, hypokalemia, gout, ototoxicity, an tinnitus (A).
006;166(4):391–397. Metroniazole is an antibiotic use frequently in patients in
Fiore LD, Scola MA, Cantillon CE, Brophy MT. Anaphylactoi nee of anaerobic coverage an can lea to intractable nau-
reactions to vitamin K. J Thromb Thrombolysis. 001;11():175–183. sea an emesis, particularly if taken with alcohol (isul-
ram-like reaction) (B). Spironolactone is a potassium-sparing
18. B. The patient is experiencing cyanie poisoning, which iuretic that can result in hyperkalemia an gynecomastia
can occur following the aministration of a nitroprussie (C). Halothane is an anesthetic agent that may rarely result
rip. Nitroprussie is metabolize into nitric oxie an cya- in fulminant hepatic failure (D). Vancomycin will inuce
nie. The accumulation of cyanie leas to a left shift in the peripheral vasoilation resulting in cutaneous ushing, an
oxyhemoglobin issociation curve, resulting in ecrease rarely, it can cause re man synrome.
oxygen elivery. This leas to severe lactic aciosis, which Reference: Sivagnanam S, Deleu D. Re man synrome. Crit
is a hallmark of cyanie poisoning. Aitionally, the hemo- Care. 003;7():119–10.
globin holing on to the oxygen content leas to an increase
Preoperative Evaluation
and Perioperative Care
NAVEEN BALAN, AREG GRIGORIAN, AND CHRISTIAN DE VIRGILIO 35
ABSITE 99th Percentile High-Yields
I. Preoperative Meication Management
A. In patients unergoing percutaneous cariac intervention (PCI): most shoul be starte on ouble
antiplatelet therapy with aspirin an clopiogrel
1) Balloon angioplasty—avoi surgery for weeks postangioplasty; elective surgeryafter this perio is
reasonable but continue aspirin an hol clopiogrel 5ays prior
) Bare metal stent (BMS)—avoi surgery for 1-month post-BMS; elective surgery after this perio is
reasonable but continueaspirin an hol clopiogrel 5ays prior
3) Drug-eluting stent (DES)—avoi surgery for 6-months post-DES; elective surgery after this perio is
reasonable but continue aspirin an hol clopiogrel 5 ays prior
4) All emergent surgical inications (e.g. peritonitis, perforate viscus with hemoynamic instability)
shoul procee without elay with cariology consult, iscuss risks/benets of continuation
of aspirin, an consieration of holing clopiogrel epening on risk of cariac event versus
hemorrhage
B. Anticoagulation meication
1) Coumain in atrial-brillation patient requiring surgery: can stop without briging with heparin, as
overall risk of thrombosis is low unless CHADS-VASc score ≥7, then consier briging
a) Age >75 ( points), previous TIA/stroke ( points), iabetes (1 point), previous vascular isease
(1 point), hypertension (1point), CHF (1 point), female (1 point)
) Novel oral anticoagulant (NOAC) rugs
a) Direct-thrombin inhibitors
(1) Bivaliruin, argatroban, an esiruin: parenteral aministration
() Dabigatran: only available oral agent; half-life is 1 to 17 hours; stop  ays before surgery;
renally metabolize so use with caution in patients with renal isease
(a) Reverse with iarucizumab (a monoclonal antiboy that bins abigatran)
b) Factor-Xa inhibitors (all factor-Xa inhibitors en in -xaban)
(1) Rivaroxaban: half-life is 6 to 9hours, an its therapeutic activity wears off after 4 to
5half-lives; rivaroxaban shoul be iscontinue 1 to  ays before a surgical proceure;
in patients with a reuce creatinine clearance, it shoul be iscontinue 3 to 5 ays
before surgery
() Apixaban: factor-Xa inhibitor, half-life of 1 hours; iscontinue  ays prior to surgery;
metabolize by liver so safe for patients with renal isease
(a) Reverse with 4-factor prothrombin complex concentrate
(b) Anexanet alfa is a recombinant analog of factor Xa an may be consiere an antiote
for factor-Xa inhibitors
c) Can restart anticoagulation 6 to 4 hours after a minor proceure, an after  to 3 ays for major
surgery (barring any perioperative bleeing)

437
438 PArt ii Medical Knowledge

C. Steroi therapy: chronic steroi therapy can affect the hypothalamic–pituitary–arenal axis, leaing
to arenal atrophy an a ecrease capability to prouce cortisol leaing to a theoretical risk of
hypotension in the perioperative perio; however, perioperative “stress-ose” sterois are not supporte
by recent evience
1) “Stress-ose” sterois shoul not be routinely aministere; instea, the patient shoul continue
their home ose of sterois perioperatively
) Shoul consier aitional sterois only if the patient evelops refractory hypotension suggestive of
arenal insufciency in the perioperative perio
CHAPtEr 35 Preoperative Evaluation and Perioperative Care 439

Questions
1. A 58-year-ol man presents with a reucible 4. A 8-year-ol woman unergoes ahesiolysis
inguinal hernia. He is not limite in his aily for an acute small bowel obstruction. During the
activities but is bothere by the appearance. He course of the surgery, she requires a segmental
unerwent percutaneous coronary intervention ileal resection with primary anastomosis. On
(PCI) with placement of a rug-eluting stent postoperative ay 6, she is note to have thick
(DES)  months ago an is currently taking bile-colore ui emanating from the miline
aspirin an clopiogrel. What is the most woun. After IV hyration, the next step in the
appropriate management of this patient? management shoul be:
A. Scheule surgery an continue aspirin an A. CT scan of the abomen
clopiogrel B. Water-soluble upper gastrointestinal series
B. Scheule surgery an Continue aspirin an with small bowel follow-through
stop clopiogrel 5 ays prior to the operation C. Fistulogram
C. Scheule surgery an stop aspirin an D. Operative reexploration
clopiogrel 5 ays prior to the operation E. Octreotie
D. Delay surgery for an aitional 4 months
E. Delay surgery for a year post-DES 5. The most important preictor of colonic ischemia
after repair of a rupture abominal aortic
2. A 38-year-ol woman evelops fever, abominal aneurysm is:
pain, an multiple loose nonblooy bowel A. Age
movements following amission for perforate B. Presence of preoperative shock
appenicitis. Her WBC count is 1,000 an has C. Time to operation
normal kiney function. On imaging, there is no D. Presence of associate cariac isease
evience of eep space abscess or ileus an her E. Preoperative patency of inferior mesenteric
stool tests positive for Clostridium difcile. This is artery
her rst episoe. What is the most appropriate
treatment? 6. Five ays after surgery for perforate
A. Fecal transplant appenicitis, liqui stool emanates from the right
B. Oral an rectal vancomycin lower quarant woun. Which of the following is
C. Oral vancomycin an intravenous true about this conition?
metroniazole A. It is most commonly ue to an unrecognize
D. Intravenous metroniazole malignancy
E. Oral vancomycin B. The majority will close spontaneously
C. The patient shoul be place immeiately on
3. Five ays after a laparoscopic Roux-en-Y gastric TPN
bypass, a patient evelops fever with rigors, D. Flui an electrolyte erangements are
hypotension, tachycaria, an pain in the left common
shouler. This most likely represents: E. The patient shoul be returne immeiately to
A. Gas bloat synrome the operating room for surgical repair
B. Internal hernia
C. Woun ehiscence 7. Five ays after a Billroth II gastric resection
D. Gastric volvulus for a bleeing ulcer, high fever, hypotension,
E. Disruption of the gastric pouch–jejunal tachycaria, an generalize peritonitis evelop
anastomosis in the patient. This most likely represents:
A. Postoperative pancreatitis
B. Acalculous cholecystitis
C. Duoenal stump blowout
D. Woun ehiscence
E. Intraabominal hemorrhage

AL GRAWANY
440 PArt ii Medical Knowledge

8. Which of the following moalities is LEAST 12. A 55-year-ol obese female with chronic
likely to assist in the prevention of postoperative obstructive pulmonary isease (COPD) is
pulmonary complications in a 65-year-ol male unergoing preoperative evaluation for ventral
smoker? hernia repair. She has a 30 pack/year smoking
A. Postoperative use of an incentive spirometer history, though she quit 1 year ago. Her COPD
B. Postoperative eep-breathing exercises symptoms are well controlle with her current
C. Postoperative use of continuous positive meication regimen, an her last amission for
airway pressure COPD exacerbation was over  years ago. Which
D. Smoking cessation 1 week before surgery of the following is true regaring risk assessment
E. Placement of a nasogastric tube for postoperative pulmonary complications in
this patient?
9. Which of the following preoperative stuies is A. Higher ASA class is a signicant risk factor
most strongly associate with an increase risk of B. Preoperative pulmonary function tests (PFTs)
pulmonary-relate postoperative complications? shoul be obtaine
A. Bloo urea nitrogen C. A nasogastric tube shoul be use
B. Incentive spirometry postoperatively to ecrease pulmonary
C. Chest raiograph complications
D. Serum albumin D. Upper miline an lower miline
E. Room air arterial bloo gas laparotomy confer similar risk for pulmonary
complications
10. A 67-year-ol male recovering from a pelvic E. A PaCO of more than 45 mmHg is an absolute
exenteration seconary to locally avance rectal contrainication to major abominal surgery
cancer is starte on total parenteral nutrition for
prolonge ileus via a right-sie peripherally 13. A 65-year-ol woman is amitte to the hospital
inserte central catheter (PICC) line. Several with a large bowel obstruction. Workup reveals
ays later his arm becomes swollen. Ultrasoun a sigmoi cancer, an on hospital ay 4, she
conrms clot in the basilic an axillary veins. unergoes laparoscopy with a plan to perform a
What is the appropriate management of his resection with a proximal colostomy. During the
conition? operation, her en-tial carbon ioxie suenly
A. Warm compress an nonsteroial rops, an she evelops tachycaria to the 10s
antiinammatory rugs (NSAIDs) with occasional premature atrial contractions. Her
B. Immeiately remove the line systolic bloo pressure is 80 mmHg. Which of the
C. Immeiately remove line an then start heparin following woul be most helpful in establishing
D. Start heparin an move the line to an alternate the presumptive iagnosis?
site A. Electrocariogram
E. Start heparin, keep the line in place, an B. Cariac enzymes
therapeutic anticoagulation for 3 to 6 months C. Transesophageal echocariogram (TEE)
D. Arterial bloo gas
11. A 76-year-ol iabetic male is amitte to the E. Flexible bronchoscopy
surgical intensive care unit after a fall. His
injuries inclue a right femoral neck fracture an
subarachnoi hemorrhage. He continues to have
intermittent elevation in his intracranial pressure
an is still requiring respiratory support after
 ays. Which of the following is true regaring
nutritional supplementation in this patient?
A. Postpyloric feeing may reuce his risk of
eveloping pneumonia
B. Gastric feeing is associate with a longer
length of ICU stay
C. Diabetic patients have better outcomes with
gastric versus postpyloric feeings
D. Postpyloric feeing more closely simulates
normal physiologic feeing
E. Gastric feeing is associate with increase
total nutrition
CHAPtEr 35 Preoperative Evaluation and Perioperative Care 441

14. A 59-year-ol male with a coronary artery bypass 17. A 69-year-ol patient with a tumor at the
grafting 1 year prior for multivessel isease rectosigmoi junction unergoes laparoscopic
unergoes a right hip replacement surgery. sigmoi colectomy. Postoperative pain is well
His postoperative course is complicate by controlle with patient-controlle thoracic
pneumonia requiring mechanical ventilation. epiural anesthesia. On postoperative ay 1,
Electrocariogram shows a stable Q wave in lea prophylactic anticoagulation is starte with low-
II. Heart rate is 80 beats per minute an bloo molecular-weight heparin (LMWH). The blaer
pressure is 116/8 mmHg. Chest raiograph is unergoing rainage with an inwelling Foley
shows bilateral patchy inltrates. Laboratory catheter. Which of the following is true regaring
exam emonstrates PaO of 70 mmHg, a epiural anesthesia?
white bloo cell count of 17,000 cells/μL, an A. Blaer catheterization shoul continue while
hemoglobin of 7.4 g/L. Which of the following the thoracic epiural is in place
is true regaring the management of his anemia? B. LMWH shoul be hel for 4 hours before
A. Bloo transfusion will lower his risk of removal of the thoracic epiural
eveloping an acute coronary synrome C. The risk of urinary tract infection is the same
B. He shoul be transfuse to a hemoglobin goal regarless of whether the urinary catheter
of 10 g/L is remove on postoperative ay 1 versus
C. Re bloo cell transfusion is inepenently postoperative ay 3
associate with lower mortality D. Risk of urinary retention is not signicantly
D. Bloo transfusion is not necessary at this time higher with early removal of the Foley
E. Hemoglobin-base oxygen carriers offer a catheter
goo alternative to transfusion in this patient E. Unfractionate heparin shoul not be restarte
for at least 4 hours after removal of an epiural
15. Four ays after a pancreaticouoenectomy for catheter
pancreatic aenocarcinoma, a 65-year-ol man
evelops a fever an tachycaria. Exam reveals 18. A 5-year-ol woman evelops a fever of 104°F
tenerness, eema, an erythema over the 1 hours after an open cholecystectomy. On
angle of the jaw. Which of the following is true examination, she has foul-smelling, purulent
regaring this conition? rainage from her woun. She unergoes the
A. It is usually ue to Staphylococcus appropriate treatment, an culture of the woun
B. Massage of the area is benecial grows gram-positive ros. Which of the following
C. It can be prevente with antibiotics is true regaring this patient an her conition?
D. The incience has been increasing A. The causative organism is an aerobe
E. It can be avoie with the use of B. Diabetes is not consiere to be a risk factor
anticholinergics C. Broa-spectrum antibiotics an ui
resuscitation resolve the majority of cases
16. Which of the following is true regaring venous D. The organism prouces an enotoxin
thromboembolism (VTE) prophylaxis in surgical E. Clinamycin shoul be inclue in the
patients? management
A. Intermittent pneumatic compression (IPC)
prevents DVT by increasing circulating tissue 19. A 34-year-ol woman unergoes a subtotal
plasminogen activator (tPA) thyroiectomy for Graves isease. In the recovery
B. Thigh-high IPC is superior to knee-high IPC room, she evelops anxiety an progressive
C. IPC is equivalent to pharmacologic respiratory istress with strior. Her incision is
prophylaxis in the majority of patients bulging an tense on exam. The most important
D. Unfractionate heparin (UFH) is superior to initial step woul be:
lower-molecular-weight heparin (LMWH) A. Nebulize racemic epinephrine
E. LMWH is superior to IPC B. Rapi-sequence intubation
C. Neele aspiration of the neck woun
D. Ultrasoun examination of the neck
E. Rapily opening the incision at the besie
442 PArt ii Medical Knowledge

20. One ay after a left colectomy for recurrent 23. A 45-year-ol male with en-stage renal
iverticulitis, a patient is note to have an isease is unergoing placement of a tunnele
elevation of his serum creatinine. Other hemoialysis catheter. During the operation,
laboratories are unremarkable. He has a urine the anesthesiologist notices a sharp ecline in
output of 30 to 50 mL/hour. A renal ultrasoun the continuous capnography an the calculate
shows no evience of abnormalities with the physiologic ea space is increase. This is
exception of ascites. Compute tomography (CT) followe by massive myocarial infarction an
scan emonstrates iscontinuity of the left ureter cariac arrest. Which of the following is true
with contrast extravasation at the level of the regaring this conition?
pelvic brim. Which of the following about this A. Electrocariogram (ECG) will most commonly
injury is true? emonstrate right heart strain
A. Immeiate reoperation shoul not be B. A congenital heart efect likely contribute to
performe the cariac arrest
B. Placement of ureteral stents woul have C. The patient shoul be positione left sie up
prevente this complication D. Besie transesophageal echocariography is
C. A percutaneous nephrostomy shoul be generally not sensitive enough to etect this
place complication
D. A retrograe stent shoul be place E. Aspiration from the central line is usually
E. A ureteroneocystostomy will likely be the best helpful
option
24. A 65-year-ol man with Barrett esophagus
21. Two ays after sustaining signicant crush injury an new-onset ysphagia is being evaluate
to her bilateral lower extremities from a motor for iagnostic esophagogastrouoenoscopy
vehicle collision, a 3-year-ol female becomes (EGD), enoscopic ultrasoun (EUS), an
oliguric an is only proucing scant ark urine. mucosal biopsy. He is on warfarin for mechanical
Urine ipstick reveals 4+ bloo, an follow-up mitral valve an has a history of embolic stroke
urinalysis shows 5 to 10 re bloo cells per high 10 years ago. What is recommene for his
power el. Prevention of acute kiney injury is anticoagulation regimen before this proceure?
best achieve by which of the following? A. Hol warfarin for 3 to 5 ays an brige with
A. Urgent 4-compartment fasciotomies low-molecular-weight heparin
B. Loop iuretics B. Hol warfarin for 48 to 7 hours, brige with
C. Vigorous IV ui hyration unfractionate heparin, an hol heparin 4 to
D. Alkalization of urine with intravenous soium 6 hours before the proceure
bicarbonate C. Perform EGD an EUS while therapeutic on
E. Mannitol warfarin; if inicate, the mucosal biopsy
can be performe at a later ate after holing
22. Which of the following is true regaring PFTs? warfarin
A. Total lung capacity (TLC) is generally reuce D. Continue warfarin without interruption
with aging E. Hol warfarin 3 to 5 ays before proceure
B. A preoperative force expiratory volume an restart within 4 hours after the proceure
in one secon (FEV1) of less than 1.5 L is a
contrainication for pulmonary lobectomy
C. Diffusion capacity of the lungs for carbon
monoxie (DLCO) will stay relatively constant
with age so long as there is no intrinsic lung
isease
D. Percent-preicte postoperative FEV1 of >40%
is acceptable for a lobectomy but not for a
pneumonectomy
E. Chest wall compliance ecreases with age
CHAPtEr 35 Preoperative Evaluation and Perioperative Care 443

25. A 4-year-ol female with long-staning systemic 26. Which of the following is true regaring the use
lupus erythematosus (SLE) complicate by lupus of beta-blockers in the perioperative perio for
nephritis an ebilitating arthritis is in the ICU patients unergoing noncariac surgery?
following an emergency bowel resection 4 ays A. Starting a beta-blocker within 4 hours
earlier. Over the next several hours, she becomes of surgery may increase the incience of
febrile, hypotensive, an complains of abominal perioperative stroke
pain. She is given ui boluses, but the bloo B. Beta-blockers shoul be stoppe at least 1
pressure oes not respon. Her abominal exam week before surgery
is unremarkable. Laboratory values reveal a C. In low- an intermeiate cariac risk patients,
white bloo cell count of 1,000 cells/L with beta-blockers shoul be initiate  to 3 weeks
eosinophilia, serum Na of 133 mEq/L, serum before surgery
bicarbonate of 0 mEq/L, an serum K of 5.3 D. Beta-blockers shoul be avoie even in the
mEq/L. Which of the following represents the high cariac risk group
best management of this conition? E. Perioperative initiation of beta-blocker
A. Two liters of normal saline followe by 4 mg ecreases the 30-ay mortality
of examethasone
B. Exploratory laparotomy
C. Vasopressin
D. Immeiate aministration of broa-spectrum
antibiotics an 100 mg of hyrocortisone
E. Flui resuscitation, vasopressor support, an
AM cosyntropin test

Answers
1. D. The management of antiplatelet meications in megacolon). Nonsevere an severe infections are treate
patients unergoing noncariac surgery after PCI poses a with either PO vancomycin or PO axomicin (if available)
common surgical ilemma. In general, patients shoul be ×10. In cases of fulminant isease, treatment shoul inclue
stratie by thrombotic risk base on the type an timing of PO vancomycin an consieration of total abominal colec-
PCI as well as hemorrhagic risk base on the type of surgery. tomy with en ileostomy. In cases of fulminant isease with
Patients who unerwent plain ol balloon angioplasty within ileus, rectal vancomycin an intravenous metroniazole
the past  weeks, bare metal stent within the past 1 month, shoul be ae (B, C, D). First recurrences can be treate
an DES within the past 6 months are at high thrombotic risk. with PO vancomycin (usual osing if metroniazole was
In general, surgery shoul be elaye until after this perio use for the initial episoe or a prolonge pulse/tapere
(A–C, E). For most abominal operations, continuing aspirin regiment if a stanar PO vancomycin regimen was use for
while holing clopiogrel 5 ays prior to surgery is appropri- the initial episoe). Alternatively, axomicin may be use if
ate. However, in the case of time-sensitive surgery such as a a stanar PO vancomycin regimen was use for the initial
colectomy in a patient with colon cancer, surgery shoul not episoe. Subsequent recurrences can be treate with antibi-
be elaye an the risks/benets of continuing ouble anti- otics or fecal transplant (A).
platelet therapy shoul be iscusse with the patient. Of note, Reference: McDonal LC, Gering DN, Johnson S, et al. Clini-
there is recent literature supporting the continuation of aspi- cal practice guielines for Clostridium difcile infection in aults an
rin an clopiogrel in patients with colon cancer unergoing chilren: 017 Upate by the Infectious Diseases Society of Amer-
resection, even within a couple months of DES placement. ica (IDSA) an Society for Healthcare Epiemiology of America
(SHEA). Clin Infect Dis. 018;66(7):987–994.
Reference: Banerjee S, Angiolillo DJ, Boen WE, et al. Use of
antiplatelet therapy/DAPT for post-PCI patients unergoing non-
cariac surgery. J Am Coll Cardiol. 017;69(14):1861–1870. 3. E. Fever, chills, tachycaria, hypotension, an peritoneal
irritation occurring together within 1 week of any surgery
2. E. Infection with C. difcile is not an uncommon com- involving a new bowel anastomosis shoul immeiately
plication following antibiotic treatment. While it has been raise suspicion for an anastomotic isruption. Left shouler
classically associate with clinamycin, it can occur fol- pain is often a consequence of left iaphragm irritation an,
lowing treatment with a wie variety of antibiotics. For in this case, correlates with the gastric pouch–jejunal anas-
initial episoes, infection can either be nonsevere (leukocy- tomosis. Water-soluble contrast stuies can ai in the iag-
tosis <15,000, serum creatinine <1.5), severe (leukocytosis nosis an inicate how large the leak is because containe
>15,000, serum creatinine >1.5), or fulminant (shock, toxic leaks can often be manage nonoperatively. However, in
444 PArt ii Medical Knowledge

this patient, hypotension an signs of peritonitis necessitate Otherwise, urgent enoscopy is require to view the colonic
operative exploration an repair of the anastomosis. Gas- mucosa. The majority of cases of colonic ischemia can be
bloat synrome results from the inability to relieve gas from manage nonoperatively with bowel rest, hyration, an IV
the stomach after a funoplication (A). Gastric volvulus can antibiotics. If the patient requires colon resection, mortality
occur after gastric surgery; however, this is extremely rare rates are as high as 75%.
(D). Internal hernia is less likely given the timeline (most
occur beyon a month after surgery) an left shouler pain 6. B. This case represents a cecal stula. The most common
inicative of iaphragmatic irritation (B). Woun ehiscence causes are slippage of the suture or necrosis of the remaining
woul be suspecte if the skin is erythematous, warm, rain- appeniceal stump, leaing to leakage of the enteric contents
ing purulent or serous material, an has fallen apart (C). into the peritoneal cavity (A). Rarely, the stula results from
unrecognize Crohn isease, malignancy, tuberculosis or is-
4. A. This case represents an enterocutaneous stula, likely tal colon obstruction. Cecal stulas are low-output stulas an
resulting from either an anastomotic leak or an unrecog- are not associate with losses of large amounts of ui, elec-
nize intraoperative bowel injury away from the anastomo- trolytes, or nutrients (D). Therefore, TPN is not necessary to
sis. Management of enterocutaneous stulas shoul begin maintain aequate nutrition (C) an mortality rates are low in
with stabilizing the patient via aggressive ui hyration the absence of other serious complications (A). Spontaneous
an control of sepsis (if present). If the patient is manifest- closure is promote in as many as 75% of patients maintaine
ing signs of sepsis, prompt aministration of IV antibiotics on low-resiue iets because absorption is mostly complete
shoul be institute. Sepsis, ehyration, an electrolyte/ by the time the contents reach the cecum (B, E).
nutrient losses are the most evastating early consequences.
Prompt return to the operating room is not recommene 7. C. Duoenal stump blowout occurs after Billroth II oper-
because the peritoneal cavity will likely have highly vascular ations, where back pressure on the uoenal stump results
ahesions, making reentry treacherous, an early attempts in breakown of the stump closure, leaing to abominal
to reclose stulas typically fail (D). Once the patient has been sepsis an peritonitis. Acute pancreatitis is associate post-
stabilize, the best initial stuy is a CT scan of the abomen. operatively with Billroth II gastrectomy an jejunostomy, in
This will ientify whether any intraabominal abscesses are which increase intrauoenal pressure can cause backow
present that might require percutaneous rainage an rule of activate enzymes into the pancreas but is unlikely to
out whether there is a istal obstruction (B, C). Fistulas are cause peritonitis (A). Woun ehiscence is characterize as
loosely categorize as high an low output. High output is suen ramatic rainage of relatively large volumes of a
ene as outputs of more than 500 mL/ay an low output clear, salmon-colore ui an is apparent on physical exam
as less than 00 mL/ay. High-output stulas are less likely (D). Acalculous cholecystitis can also occur postoperatively;
to close an often cause signicant ui, electrolyte, an however, the clinical presentation woul mainly consist of
nutritional challenges. Factors that preict whether a stula right upper quarant pain (B). Intraabominal hemorrhage
will close (mnemonic “FRIEND”) inclue foreign boy, rai- woul be less likely to present with sepsis (E).
ation to the bowel, inammation/infection (such as inam-
matory bowel isease), epithelialization of the stula tract, 8. E. Smoking is a preictor of postoperative pulmonary
neoplasia at the stula site, an distal obstruction. The mor- complications. The respiratory epithelium is altere in smok-
tality rate of enterocutaneous stulas remains signicant at ers, an poor ciliary activity combine with the prouction
10% to 15%. Approximately 50% close spontaneously. Con- of more viscous mucus leas smokers to be more reliant on
servative treatment shoul be continue for at least 6 weeks coughing to clear secretions from their lungs. Several ays
before any reoperation is performe. Operating before 6 after patients have stoppe smoking, there may be a transient
weeks results in higher mortality an stula recurrence rates. increase in sputum volume. The above reasons have typically
Octreotie has not been shown in ranomize trials to ai prevente health professionals from encouraging smoking
in earlier stula closure but oes not ecrease mortality (E). cessation in the weeks leaing up to surgery. However, a
Reference: Sancho JJ, i Costanzo J, Nubiola P, et al. Ran- metaanalysis publishe by the American Meical Associa-
omize ouble-blin placebo-controlle trial of early octreotie
tion has conclue that the concern that stopping smoking
in patients with postoperative enterocutaneous stula. Br J Surg.
only a few weeks before surgery might worsen clinical out-
1995;8(5):638–641.
comes is unfoune an clinicians shoul avise smoking
cessation as soon as possible (D). Postoperative lung expan-
5. B. Colonic ischemia after repair of a rupture abominal
sion moalities (A–C) reuce postoperative pulmonary com-
aortic aneurysm occurs in 1% to 6% of operations but can
plications, although there is no ae benet from using all
occur in up to 5% of cases uner certain circumstances. The
three. Routine use of a nasogastric tube may increase aspira-
greatest risk factor is the presence of prolonge hypotension
tion risk because the tube stents open the gastroesophageal
preoperatively. In a patient with stable bloo pressure, age,
junction. However, selective use in patients with nausea,
time to operation, an the presence of cariac isease have
bloating, an/or vomiting is probably protective.
little effect on the incience of colonic ischemia after aortic
References: Bluman LG, Mosca L, Newman N, Simon DG. Pre-
repair (A–C, D). Patency of a patient’s inferior mesenteric
operative smoking habits an postoperative pulmonary complica-
artery is not a goo preictor of colonic ischemia because tions. Chest. 1998;113(4):883–889.
of signicant avenues of collateral ow (E). Symptoms an Lawrence VA, Cornell JE, Smetana GW, American College of Phy-
signs of ischemia inclue blooy iarrhea, abominal pain/ sicians. Strategies to reuce postoperative pulmonary complications
istention, an elevate white bloo cell count. If the patient after noncariothoracic surgery: systematic review for the American
has evience of peritonitis, urgent reoperation is inicate. College of Physicians. Ann Intern Med. 006;144(8):596–608.
CHAPtEr 35 Preoperative Evaluation and Perioperative Care 445

Myers K, Hajek P, Hins C, McRobbie H. Stopping smoking (which is common in ICU patients), iabetes, an gastro-
shortly before surgery an postoperative complications: a system- esophageal reux shoul be consiere for postpyloric fees
atic review an meta-analysis. Arch Intern Med. 011;171(11):983–989. (C). In terms of timing, there is abunant evience that ear-
lier enteral feeing in critically ill patients results in better
9. D. A serum albumin less than 3.5 g/L is the single most outcomes.
important laboratory preictor of averse pulmonary events Reference: Alkhawaja S, Martin C, Butler RJ, Gwary-Srihar F.
after surgery. Bloo urea nitrogen (>1 mg/L) is also useful, Post-pyloric versus gastric tube feeing for preventing pneumonia
although the correlation is not as strong (A). Routine spirom- an improving nutritional outcomes in critically ill aults. Cochrane
etry for all operations oes not seem to a value beyon a Database Syst Rev. 015;(8):CD008875.
careful history an physical examination (B). An exception
for the use of spirometry woul be for lung resection. Chest 12. A. Patient-relate risk factors for the evelopment
raiograph an arterial bloo gas are iagnostic stuies that of postoperative pulmonary complications inclue: age
woul only be preictive of postoperative complications if more than 50 years, COPD, congestive heart failure, Amer-
there were abnormal nings (C, E). ican Society of Anesthesiologists (ASA) class greater than ,
References: Lawrence VA, Cornell JE, Smetana GW, Ameri- serum albumin less than 3.5 g/L, obstructive sleep apnea,
can College of Physicians. Strategies to reuce postoperative pul- pulmonary hypertension, an current smoking. While a
monary complications after noncariothoracic surgery: systematic preoperative PaCO greater than 45 oes increase the sur-
review for the American College of Physicians. Ann Intern Med.
gical risk, there is currently no enitive number that pro-
006;144(8):596–608.
Qaseem A, Snow V, Fitterman N, et al. Risk assessment for an
hibits abominal surgery (E). Current American College of
strategies to reuce perioperative pulmonary complications for Physicians Guielines recommen against the routine use
patients unergoing noncariothoracic surgery: a guieline from the of preoperative chest raiography or PFT (B). Although
American College of Physicians. Ann Intern Med. 006;144(8):575–580. it is important to ientify patients with COPD, an some
COPD patients may benet from preoperative interventions,
10. E. Thrombosis of supercial an eep veins of the patients who require aitional testing can be ientie by
upper extremity is cause by intravenous catheters in most history of new symptoms or physical examination nings.
cases. Upper extremity DVT oes pose a risk of pulmonary Thus, PFT shoul be restricte to those who have current
embolus, though less risk than pelvic an lower extremity symptoms or signs base on history an physical. Location
DVT. Management begins with anticoagulation an eter- of the surgical incision is an important risk factor for postop-
mining the necessity for the line; in the above case, there erative pulmonary complications, with incisions closer to the
is a continue nee for TPN via a PICC line as the patient iaphragm inferring more risk (D). When patients have been
has not yet emonstrate a return of bowel function. Stu- ientie as high risk for pulmonary complications, current
ies have shown that it is not necessary to remove the PICC evience supports the use of perioperative eep-breathing
line espite the DVT. Therapeutic anticoagulation for 3 to exercises an incentive spirometry. Routine use of nasogas-
6 months is recommene. Thus, removal of the catheter tric ecompression has been associate with increase rates
without anticoagulation is not acceptable because there is a of pneumonia an atelectasis. Current recommenations are
risk of PE (B). Inications to remove a line inclue infection for more selective use in patients with nausea, vomiting, or
an a contrainication to anticoagulation. If the line is to be gastric istention (C).
remove, anticoagulation is still recommene; however, Reference: Qaseem A, Snow V, Fitterman N, et al. Risk assess-
the recommenation is to wait 5 to 7 ays after the initiation ment for an strategies to reuce perioperative pulmonary com-
of heparin before removing it (ue to the theoretic fear that plications for patients unergoing noncariothoracic surgery: a
pulling the line with a fresh clot might isloge the throm- guieline from the American College of Physicians. Ann Intern Med.
006;144(8):575–580.
bus) (C, D). Warm compresses an NSAIDs woul be appro-
priate for supercial thrombophlebitis (A).
Reference: Kucher N. Clinical practice. Deep-vein thrombosis 13. C. The ifferential iagnosis for a suen rop in
of the upper extremities. N Engl J Med. 011;364(9):861–869. en-tial CO in the operating room (OR) inclues an
obstructe airway, acciental extubation, isconnection of
11. A. While there are many theoretic avantages between the circuit, cariac arrest, an pulmonary embolism (PE).
each metho of feeing, a 015 Cochrane review comparing The patient escribe has at least three risk factors for PE
postpyloric an gastric feeings showe only two signi- incluing malignancy, a heart rate greater than 100, an
cant ifferences: lower rates of pneumonia in the postpyloric more than 3 ays of immobilization (Wells criteria). PE is
group an some evience for increase total nutrition eliv- estimate to occur in 1% to % of surgical patients in the
ere in the postpyloric group (E). There was no signicant perioperative perio. While yspnea, anxiety, tachycaria,
ifference in length of ICU stay, mortality, or time on the ven- an tachypnea are the most common nings in awake
tilator (B). There was also no signicant ifference in asso- patients, physical signs of PE will be limite in patients
ciate complications with tube placement between the two uner general anesthesia. In this situation, an astute cli-
stuy groups. While postpyloric feeing was associate with nician can recognize PE as presenting with hypotension,
a longer time to initiation of tube feeing, this i not seem tachycaria, ecrease en-tial CO, an hypoxemia. In
to affect the time it took to reach nutritional goals. Avan- general, laparoscopic proceures have been associate
tages of gastric feeing inclue a better approximation of with a low risk of both fatal an nonfatal PE (E). This com-
normal physiology, ease of placement, an convenience plication is associate with 10% to 15% mortality in the
(D). It may be a reasonable choice in patients without risk perioperative perio (D). Electrocariogram changes have
for aspiration, but patients with elaye gastric emptying been shown to be present in up to 83% of patients, but they
446 PArt ii Medical Knowledge

are generally nonspecic (A). Uncommonly, PE can pres- leukocytosis, as well as signicant eema involving the oor
ent with a prominent S wave in lea 1, Q wave in lea 3, of the mouth. If left uniagnose an untreate, it can lea
an inverte T wave in lea 3; this is suggestive of right to life-threatening sepsis. Initial treatment is with high-ose
heart strain. Despite potential cariovascular consequences broa-spectrum antibiotics with Staphylococcus coverage
of massive PE, an elevate cariac enzyme level occurs in (most common organism) an warm compresses (B). If the
less than 50% of cases an is not specic for PE (B). The patient oes not improve, surgical incision an rainage are
two most sensitive tests that can be one to help iagnose inicate. In extreme cases involving progressive airway
PE are a TEE an calculating the physiologic ea space obstruction, emergent tracheostomy may be inicate. Use
to look for elevations (though it can be time-consuming). of measures to stimulate salivary ow, such as sucking on
TEE has been shown to yiel a iagnosis in an average of cany, seems to help prevent this complication, but pro-
9.6 minutes with a sensitivity of 80% an specicity of 97% phylactic antibiotics are generally not inicate (C). Within
an is ieal in the OR setting. Although TEE is relatively improve oral hygiene, the incience of this rare complica-
poor at visualizing the PE, it is excellent at emonstrat- tion is eclining (D). Aitionally, the use of anticholinergics
ing right heart strain, which provies inirect evience will ecrease salivary ow an increase the risk of evelop-
of PE. Arterial bloo gas in awake patients with PE may ing postoperative parotitis (E).
emonstrate a low CO, but this may not be the case for
a ventilate patient uner general anesthesia (D). Flexible 16. E. VTE prophylaxis is generally ivie into two
bronchoscopy is not helpful in iagnosing PE (E). categories: pharmacologic an mechanical. Mechani-
Reference: Desciak MC, Martin DE. Perioperative pulmonary cal prophylaxis inclues static compression evices (like
embolism: iagnosis an anesthetic management. J Clin Anesth. grauate compression stockings) an IPC evices. While
011;3():153–165 grauate compression stockings work primarily by pre-
venting venous stasis in the legs, IPC combines that with
14. D. Re bloo cell transfusion has been inepenently its effects on the intrinsic brinolytic system. It was orig-
associate with longer intensive care unit (ICU) an hospital inally hypothesize that intermittent compression cause
stays, increase complications, an increase mortality (C). It the release of agents like tPA from the vascular enothe-
is also an inepenent risk factor for multiorgan system fail- lium. However, when these levels are irectly measure,
ure an systemic inammatory response synrome (SIRS). they seem to be relatively constant espite an increase in
Most societal guielines agree that a liberal transfusion strat- tPA activity. The currently propose mechanism is relate
egy (goal of 10 g/L) is no better an likely worse than a to measure ecreases in plasminogen activator inhibitor-1
more restrictive strategy (goal of 7–9 g/L) in the majority (PAI-1), which functions as a tPA inhibitor (A). Currently,
of patients. So transfusion is neee as long as the hemo- there is no evience that one IPC evice is superior to
globin remains above 7 g/L. The Transfusion Requirements another in preventing VTE (B). While there are relatively
in Critical Care (TRICC) trial emonstrate that critically ill few contrainications to mechanical prophylaxis, traumatic
patients without active bleeing fare better with a restric- injury to the extremity an evience of ischemia seconary
tive transfusion strategy. There is also no evience that trans- to peripheral vascular isease are both contrainications.
fusion lowers the risk of acute coronary synromes. (A, B). Aitionally, patients with conrme DVT in the lower
While hemoglobin can improve oxygen elivery to tissues, extremity shoul not be place on IPC. Limitations of its
it has not been shown to lower risk of acute coronary syn- usefulness are primarily relate to interruption in treatment
romes, ecrease time on the mechanical ventilator, improve an improper application. Comparison between mechani-
oxygen consumption, or improve outcomes in patients with cal an pharmacologic VTE prophylaxis shows that in cer-
ault respiratory istress synrome or acute lung injury tain low-risk patients there is an equivalent reuction in
(A). Because of the negative effects of bloo transfusion, the incience of DVT an PE, though combination therapy
alternative methos of managing anemia are actively being is superior to mechanical prophylaxis alone (C). It can be
researche, incluing the use of recombinant human eryth- consiere as sole therapy in low-risk patients an patients
ropoietin (EPO) an hemoglobin-base oxygen carriers. Tri- with contrainications to pharmacologic agents. In terms of
als esigne specically looking at aministering exogenous pharmacologic prophylaxis, UFH an LMWH are the two
EPO in trauma patients, the EPO-1 an EPO- trials showe most commonly use agents. LMWH is generally regare
reuctions in require bloo transfusions an improve as more effective, especially in certain populations (e.g.,
mortality, respectively. While hemoglobin-base oxygen car- trauma patients) (D).
riers show some promise, they are not currently approve References: Comerota AJ, Chouhan V, Haraa RN, et al. The
for use in the Unite States (E). brinolytic effects of intermittent pneumatic compression: mecha-
nism of enhance brinolysis. Ann Surg. 1997;6(3):306–314.
15. A. This patient has postoperative parotitis. This most Ho K, Tan J. Stratie meta-analysis of intermittent pneumatic
commonly occurs in elerly patients with poor oral hygiene, compression of the lower limbs to prevent venous thromboembo-
poor oral intake, prolonge nasogastric tube ecompression, lism in hospitalize patients. Circulation, 013;18(9), 1003–100.
an ehyration, all leaing to a ecrease in saliva prouc- Morris R, Woocock J. Evience-base compression. Ann Surg.
tion. The pathophysiology involves obstruction of the sali- 004;39(), 16–171.
vary ucts with seconary infection an is more common in
the iabetic or immunocompromise patient. Most patients 17. D. Epiural anesthesia is an excellent tool to control
will be iagnose with parotitis 4 to 1 ays postoperatively. postoperative pain an has been shown to ecrease cariac
Signs an symptoms begin with pain an tenerness over morbiity, an as such, it has been gaining popularity in clin-
the angle of the jaw that can then progress to high fevers an ical practice. Routine use of urinary rainage in the setting of
CHAPtEr 35 Preoperative Evaluation and Perioperative Care 447

epiural anesthesia remains controversial. However, postop- further compromising attempts at intubation. Rapily open-
erative ay 1 removal of the Foley with thoracic epiurals has ing the incision at the besie is necessary because urgent
been shown to signicantly ecrease the incience of urinary ecompression is the fastest way to restore proper respira-
tract infections with minimal change to the rate of urinary tory function. Denitive hemostasis must then be obtaine
retention as measure by rates of recatheterization (A–C). in the operating room. Although ultrasonography is an
Current recommenations for the placement an removal of important iagnostic ai for hematomas (D), clinical suspi-
epiural catheters in patients receiving prophylactic LMWH cion is sufcient in this emergent situation an the urgency
is intene to prevent an epiural hematoma an subse- of ecompression oes not permit waiting for an ultrasoun
quent paralysis. For the placement of epiural catheters, examination. Neele aspiration woul not be sufcient (C).
LMWH must be hel at least 4 hours before placement, an While nebulize racemic epinephrine is use for the treat-
it shoul not be remove within 1 hours of the last ose. ment of strior in conitions like croup, it is not appropriate
Prophylactic anticoagulation can be restarte 6 hours after when the cause of strior is external compression of the air-
placement an no sooner than 4 hours after removal of the way (A).
epiural (B). Unfractionate heparin may be restarte after
1 hour (E). 20. E. The ureters rst pass meial to the psoas muscle an
References: FDA Safety Information an Averse Event Report- travel alongsie the transverse processes of the lumbar ver-
ing Program. Low Molecular Weight Heparins: Drug Safety Com- tebrae an cross anterior to the common iliac arteries near
munication – Upate recommenations to ecrease risk of spinal the bifurcation into the internal an external iliac arteries.
column bleeing an paralysis in patients on low molecular weight The anatomic position places the ureters at risk for injury
heparins. U.S. Foo an Drug Aministration; 013. https://www. uring pelvic surgery, an the situation is particularly pre-
fa.gov/meia/87316/ownloa.
carious when inammation, abscess, an/or phlegmon
Henren S. Urinary catheter management. Clin Colon Rectal Surg.
are present. The highest risk of ureteral injury is uring an
013;6(3):178–181.
Horlocker TT. Regional anaesthesia in the patient receiving anti-
abominoperineal resection. During mobilization of the left
thrombotic an antiplatelet therapy. Br J Anaesth. 011;107 Suppl colon an ligation of the inferior mesenteric artery, visual-
1:i96–i106. ization an protection of the ureter from injury are impera-
Townsen CM Jr, Beauchamp RD, Evers BM, Mattox KL. Sabiston tive. Placement of ureteral stents before the operation may
textbook of surgery: the biological basis of modern surgical practice. 17th help to ientify the ureters an assist with ientifying an
e. WB Sauners; 004. injury intraoperatively, but this oes not seem to correlate
Zaouter C, Kaneva P, Carli F. Less urinary tract infection by earlier with a reuction in the number of injuries (B). The presence
removal of blaer catheter in surgical patients receiving thoracic of blue ye in the operative el after intravenous amin-
epiural analgesia. Reg Anesth Pain Med. 009;34(6):54–548.
istration of inigo carmine or methylene blue is iagnostic
for injury to the ureter. The ecision to immeiately reop-
18. E. Postoperative necrotizing soft-tissue infection is erate is base on the elay associate with injury recogni-
a rare but well-escribe complication. The escription tion, the severity of the injury, an whether the patient has
of “ishwater pus” is classic for a postoperative clostri- evelope urosepsis. If iscovere within a week postop,
ial woun infection. The causative organisms are typically reoperation is generally recommene (A). Beyon 10 ays,
Streptococcus pyogenes or Clostridium perfringens. C. perfrin- the inammation present will make reoperation hazarous.
gens is an anaerobic gram-positive ro that prouces alpha- In this latter case, percutaneous nephrostomy an/or ret-
toxin; this is a virulent exotoxin that leas to extensive tissue rograe rainage with a ureteral stent is inicate (C, D).
necrosis an cariovascular collapse. Immunocompromise The type of repair epens on the location an extent of the
patients (iabetes, malignancy, chronic liver isease) are at injury. For miureter injuries, a ureteroureterostomy is pre-
increase risk (A–D). Clinamycin has been shown to limit ferre. For pelvic injuries, ureteroneocystostomy is neee.
toxin prouction, which ecreases the virulence, slows tissue If this is not possible, a psoas hitch or a Boari ap (from the
estruction, an can potentially reuce inammatory cyto- blaer) may be neee.
kine release. Effective therapy requires rapi aministration Reference: Bothwell WN, Bleicher RJ, Dent TL. Prophylactic
of broa-spectrum antibiotics incluing aerobic coverage ureteral catheterization in colon surgery. A ve-year review. Dis
(C) an source control via emergent operative excision of Colon Rectum. 1994;37(4):330–334.
necrotic infecte tissue, incluing fascia. Conservative man-
agement is not appropriate if a necrotizing soft-tissue infec- 21. C. Crush injury to the extremities causing signicant
tion is suspecte (B). muscle injury is often complicate by rhabomyolysis, which
Reference: Hakkarainen TW, Kopari NM, Pham TN, Evans can lea to acute renal failure. Degraation proucts of both
HL. Necrotizing soft tissue infections: review an current con- hemoglobin an myoglobin are toxic to the nephron in aciic
cepts in treatment, systems of care, an outcomes. Curr Probl Surg. urine. Elevate serum creatine phosphokinase, hyperkalemia,
014;51(8):344–36. an the presence of heme without a signicant amount of re
bloo cells on urinalysis are inicative of rhabomyolysis.
19. E. Postoperative hematomas after neck surgery (thy- Management consists of aggressive IV hyration to main-
roi, parathyroi, caroti artery) can have catastrophic tain a urine output of more than 100 mL/hour an shoul
consequences. Physical examination nings can be ecep- begin with infusion rates of at least 00 cc/hour. Myoglobin
tively benign. Attempts at intubation may be hampere by concentrates in the renal tubules precipitates when it comes
tracheal compression an eviation (B). Furthermore, the in contact with Tamm-Horsfall protein. This precipitation is
recent neck issection, combine with the hematoma, causes enhance uner aciic conitions. Routine aministration of
venous an lymphatic obstruction, leaing to airway eema, bicarbonate (D) an mannitol (E) in the prevention of acute
448 PArt ii Medical Knowledge

kiney injury from rhabomyolysis is controversial, but, the- cancer, 3r e: American College of Chest Physicians evience-base
oretically, alkalinization of the urine increases the solubility of clinical practice guielines. Chest. 013;143(5 Suppl):e166S–e190S.
the myoglobin–Tamm-Horsfall protein P complex an shoul
increase myoglobin washout. It also prevents lipi peroxia- 23. B. Venous air embolism is a rare an typically asymp-
tion an renal vasoconstriction an seems to have relatively tomatic conition. Though it is often associate with the
few negative sie effects if use in patients without high placement of central venous access catheters, it has been
serum bicarbonate an without alkalosis. However, it reuces associate with other conitions incluing trauma, hea an
the amount of ionize calcium so shoul be use with caution neck proceures, an neurosurgical proceures. When sus-
in patients with hypocalcemia. Historical treatment of rhab- pecte, the patient shoul be immeiately place in Trene-
omyolysis has inclue force iuresis with mannitol, but lenburg position an left lateral ecubitus or right sie up
its routine use is being questione now in the literature an (Durant maneuver) (C). This maneuver is esigne to trap
may actually increase the risk of eveloping renal failure. A the air embolus in the right ventricle an prevent it from
retrospective stuy publishe in The Journal of Trauma in 004 going into the pulmonary arteries. Physical exam nings
looking at over 000 patients with elevate creatine kinase inclue jugular venous istention, millwheel murmur, an
showe no ifference in renal failure, nee for hemoialysis, a sucking soun as air enters the venous system through
or mortality in patients receiving bicarbonate an manni- a catheter. The most sensitive besie test for iagnosis is
tol versus volume resuscitation alone. Mannitol may ai in likely transesophageal echocariography, which can etect
ecreasing muscle swelling an compartment pressures, but even small volumes of air (D). ECG suggestive of right heart
the mainstay of treatment remains ecompression of muscle strain is associate with pulmonary emboli (A). Treatment
compartments (A). However, in the case of crush injury, nor- inclues correct positioning as previously escribe, increas-
mal compartment pressures woul not change your strategy ing inspire oxygen, mechanical ventilation, hyperbaric oxy-
for preventing acute kiney injury because tissue amage gen, an, as a last resort, close-chest cariac massage to try
alone coul cause the release of myoglobin. Loop iuretics are to force the air out of the pulmonary arteries an into the
not use in the prevention of acute kiney injury in the setting smaller capillaries of the lung. An attempt can be mae to
of rhabomyolysis (B). While retrograe urethrogram woul aspirate the air from the ventricle either through an existing
assist in the iagnosis of misse urethral injury, the urine is central line or irectly through the chest wall, but the return
positive on the ipstick from myoglobin, not hemoglobin. of air with these proceures is generally low (E). Myocar-
References: Brown CVR, Rhee P, Chan L, Evans K, Demetriaes ial infarction is uncommon with venous air embolism an
D, Velmahos GC. Preventing renal failure in patients with rhabo- is typically the result of the air entering the arterial system
myolysis: o bicarbonate an mannitol make a ifference? J Trauma. via a congenital heart efect, such as a patent foramen ovale,
004;56(6):1191–1196. an occluing the coronary arteries. The volume for a fatal
Holt SG, Moore KP. Pathogenesis an treatment of renal ysfunc- venous air embolism is typically estimate at 3 to 5 mL/kg
tion in rhabomyolysis. Intensive Care Med. 001;7(5):803–811. injecte at a rate of 100 mL/s, but these are largely base on
animal stuies. The volume is much lower if the air enters
22. E. Pulmonary function testing generally inclues three the arterial system.
separate tests: spirometry, lung volumes, an the iffusion Reference: Gory S, Rowell S. Vascular air embolism. Int J Crit
capacity of the lungs. Expecte changes with aging inclue Illn Inj Sci. 013;3(1):73–76.
an increase in the functional resiual capacity an the resi-
ual volume, with a corresponing ecrease in the vital 24. D. This patient is scheule for a low-risk enoscopic
capacity. This reciprocal change generally means the TLC is proceure an represents a high risk for thromboembolic
preserve (A). DLCO will also ecrease with age (C). Com- events (mechanical valves an previous thromboembolic
pliance of the lung can be misleaing because even though events), so anticoagulation shoul be continue without
the compliance of the lung tissue itself increases with age, the interruption. When consiering enoscopic proceures for
chest wall compliance is signicantly reuce. In general, patients on anticoagulation or antiplatelet therapy, three
this means that the overall compliance of the pulmonary main things nee to be consiere: the urgency of the pro-
system is reuce. Preoperative pulmonary function tests ceure, the patient’s risk of thromboembolic events (an in
are manatory for the evaluation of potential pulmonary this case, the type of valve), an risk of bleeing uring the
resection. The preoperative values to remember are FEV1 propose intervention. If the anticoagulation is temporary
greater than  L for pneumonectomy, FEV1 greater than 1.5 L (e.g., treatment of venous thrombosis) or iscontinuation
for lobectomy, FEV1 greater than 80% preicte, an DLCO will be safer at a later ate (e.g., recent myocarial infarc-
greater than 80% preicte. However, these numbers are not tion [MI] with stent placement) an the enoscopy is com-
absolute inications, an failure to meet them simply neces- pletely elective (such as screening), the enoscopy shoul be
sitates more workup; this inclues getting a ventilation/per- elaye. The type of prosthetic valve matters too. Prosthetic
fusion scan to etermine the contribution of the preicte mitral valves have a much higher risk of thrombosis than
segment (B). If the percent-preicte postoperative FEV1 an aortic valves (much higher ow) with cessation of anticoag-
DLCO are greater than 60%, then the patient is a caniate ulation. As such, briging is typically not neee for aortic
for resection of the propose segment without further testing valves. Low-risk enoscopic proceures can safely be per-
(D). If the percent-preictive postoperative FEV1 an DLCO forme on therapeutic anticoagulation or antiplatelet ther-
are less than 60%, exercise tolerance shoul be teste. apy an these meications shoul be continue regarless of
Reference: Brunelli A, Kim AW, Berger KI, Arizzo-Harris DJ. the intervention. Examples of these proceures are iagnos-
Physiologic evaluation of the patient with lung cancer being con- tic enoscopy with mucosal biopsy, ERCP without sphinc-
siere for resectional surgery: iagnosis an management of lung terotomy, EUS, enteroscopy, an stent eployment. In all
CHAPtEr 35 Preoperative Evaluation and Perioperative Care 449

high-risk enoscopic proceures (polypectomy, sphincterot- is possible, this vignette provies insufcient ata to point
omy, therapeutic ilation, ne-neele aspiration, enoscopic to this iagnosis (C, D). Exploratory laparotomy is not an
hemostasis, tumor ablation, cyst gastrostomy, an treatment appropriate option for the above patient (B).
of varices), anticoagulation shoul be iscontinue with or References: Brunicari FC, Anersen DK, Billiar TR, Dunn DL,
without briging. However, aspirin an NSAIDs can safely Hunter JG, Matthews JB, Pollock RE. Schwartz’s principles of surgery.
be continue in all enoscopic proceures. For patients on 10th e. McGraw-Hill Eucation; 015.
antiplatelet therapy with agents other than aspirin, they Kelly KN, Domajnko B. Perioperative stress-ose sterois. Clin
Colon Rectal Surg. 013;6(3):163–167.
shoul be hel 7 to 10 ays before the proceure unless the
Marik PE, Varon J. Requirement of perioperative stress oses
thromboembolic risks are high, in which case patients may
of corticosterois: a systematic review of the literature. Arch Surg.
nee to be switche to aspirin or, in the case of ual anti- 008;143(1):1–16.
platelet therapy, aspirin continue an the other agent is-
continue. If the thromboembolic risk is low, anticoagulation
can be stoppe an simply restarte after the proceure. For
26. A. The 008 POISE trial was a ranomize controlle
trial to measure the effects of perioperative initiation of
anticoagulation with warfarin in patients with high thrombo-
beta-blockers. The control group receive a placebo while
embolic risk, brige therapy with LMWH or unfractionate
the stuy arm was starte on metoprolol on the ay of sur-
heparin shoul be consiere. Use of LMWH an mechani-
gery an receive it for 30 ays postoperatively. The stuy
cal valves is controversial because of reporte events of fatal
foun that patients who receive metoprolol ha a lower
thromboembolism on LMWH in these patients. In general,
incience of myocarial infarction, cariac revasculariza-
anticoagulation can be restarte within 4 hours after the
tion, an clinically signicant atrial brillation. However,
proceure (A–C, E).
patients in the stuy arm also ha increase mortality,
Reference: ASGE Stanars of Practice Committee, Anerson
MA, Ben-Menachem T, et al. Management of antithrombotic agents
stroke, hypotension, an braycaria (A, E). This increase
for enoscopic proceures. Gastrointest Endosc. 009;70(6):1060–1070. in mortality was not seen in the previously publishe
DECREASE trials, which also showe a reuction in myo-
25. A. Refractory hypotension in the postoperative perio carial infarction. However, several of these stuies were
in patients with conitions such as SLE that are commonly retracte because of falsie ata an questionable ata
treate with sterois shoul raise concern for acute arenal collection techniques. Without any other large ranomize
insufciency. When the iagnosis is suspecte, treatment trials to counter the POISE trial, it has largely become the
shoul begin immeiately before conrmatory tests become basis for current guielines regaring perioperative use of
available (E). Initial treatment consists of: volume resuscita- beta-blockers. The 014 ACC/AHA guielines for periop-
tion, laboratory stuies (electrolytes, glucose, arenocorti- erative beta-blocker therapy can be summarize as: (1)
cotropic hormone [ACTH], cortisol), an aministration of Beta-blockers shoul be continue if patients are on them
either 4 mg of examethasone or 100 mg of hyrocortisone. chronically. () Management of beta-blockers after surgery
Dexamethasone is preferre because it will not interfere shoul be base on clinical jugment to avoi negative
with cosyntropin stimulation testing, which shoul be one consequences such as hypotension or braycaria (B). (3)
the next morning to conrm the iagnosis. Glucocorticois Beta-blockers shoul not be starte on the ay of surgery.
can then be tapere to regular maintenance oses. Routine (4) It is unclear what the risk of starting beta-blockers is in
aministration of “stress-ose sterois” for patients on long- the  to 45 ays before surgery (C). (5) It shoul be consi-
term corticosterois is not supporte by evience. It is now ere in high-risk iniviuals (D).
recommene that patients on long-term sterois shoul not References: POISE Stuy Group, Devereaux PJ, Yang H, et al.
be given “stress-ose” perioperative corticosterois. They Effects of extene-release metoprolol succinate in patients uner-
going non-cariac surgery (POISE trial): a ranomise controlle
shoul be continue on their regular maintenance ose with
trial. Lancet. 008;371(967):1839–1847.
the consieration of aitional sterois only if they evelop
Wijeysunera DN, Duncan D, Nkone-Price C, etal. Periopera-
refractory hypotension suggestive of arenal insufciency. tive beta blockae in noncariac surgery: a systematic review for the
While the cosyntropin stimulation test can be instrumental 014 ACC/AHA guieline on perioperative cariovascular evalua-
in etecting acute arenal insufciency, its usefulness as a tion an management of patients unergoing noncariac surgery:
preoperative measure for assessing risk of postoperative a report of the American College of Cariology/American Heart
arenal crisis is lacking sufcient ata to support its routine Association Task Force on practice guielines. J Am Coll Cardiol.
use. While septic shock in the early postoperative perio 014;64():406–45.
Transfusion and Disorders
of Coagulation
CAITLYN BRASCHI, JOON Y. PARK, AND ERIC R. SIMMS 36
ABSITE 99th Percentile High-Yields
I. Coagulation Cascae an Factors
A. Factor I = brinogen, factor IA = brin, factor II = prothrombin, factor IIA = thrombin
B. Intrinsic pathway of coagulation: initiate by expose subenothelial collagen, prekallikrein, high
molecular weight kininogen; also involves factors VIII, IX, XI, XII; if impaire, PTT will be elevate
C. Extrinsic pathway of coagulation: involves factor VII; if impaire, PT/INR will be elevate
D. Common pathway of coagulation: involves factors I, II, V, X
E. Factor VII has the shortest half-life of all coagulation factors
F. Protein C an S breakown factors V an VIII
G. Factor VIII is the only coagulation factor not mae in the liver (mae in the enothelium); von
Willebran factor (vWF) also mae in the enothelium

II. Bloo Proucts an Drugs for Coagulation Reversal

Contents/mechanism
of action Indications Notes
Fresh Frozen All coagulation factors, Warfarin reversal, DIC, TTP, INR of FFP is 1.4–1.6; takes 1–2
Plasma (FFP) vWF, Antithrombin III liver disease, AT III deęciency, hours to thaw
(ATIII) Factor V deęciency
Cryoprecipitate I, VIII, XIII, vWF DIC, vWD type III, hemophilia Highest concentration of
A, hypoębrinogenemia ębrinogen (Factor I)
Prothrombin 3-factor: II, IX, X (not used in Warfarin reversal in life-threatening Immediate warfarin reversal
complex clinical practice) 4-factor: bleed (intracranial hemorrhage),
concentrate II, VII, IX, X, C, S reversal of direct Xa inhibitors
(PCC) (rivaroxaban, apixaban)
Recombinant VIII Hemophilia A Transfuse to 100% normal factor
factor VIII VIII levels before major surgery
Recombinant IX Hemophilia B Transfuse to 100% normal factor
factor IX IX levels before major surgery
Recombinant X Reverse direct Xa inhibitor Not widely available
factor Xa
Vitamin K Cofactor of carboxylation of Nonurgent warfarin reversal Warfarin reversal begins after
coagulation factors II, VII, 6–10 hours, full eěect after
IX, X, C, S 1–2 days
Tranexamic acid Binds plasmin (inhibits Traumatic hemorrhagic shock Must be given within 3 hours
(TXA) ębrinolysis) with hyperębrinolysis of injury for beneęt; if patient
does not have hyperębrinolysis,
TXA increases risk for
thromboembolic events

451
452 PArt ii Medical Knowledge

Contents/mechanism
of action Indications Notes
Aminocaproic Binds plasmin (inhibits tPA-associated bleed DIC
acid ębrinolysis)
Protamine Binds to and inhibits Heparin overdose with Only partially eěective against
sulfate heparin associated bleed LMWH
Desmopressin V2 agonist, causes release of vWD type I/II, uremia vWD type III has absent vWF, so
(DDAVP) vWF and factor VIII from DDAVP ineěective
endothelium and platelets

III. Anticoagulants

Mechanism of action Notes


Heparin Indirect thrombin and factor X Ineěective in ATIII deęciency (give FFP with
inhibitor via ATIII heparin)
Aěects the intrinsic pathway (PTT)
Low-molecular-weight Indirect factor X inhibitor via ATIII Superior DVT prophylaxis in cancer patients
heparin (LMWH)
Coumadin Vitamin K antagonist Contraindicated in pregnancy
Extrinsic pathway (PT)
Argatroban, bivalirudin, Direct thrombin inhibitors Used in HIT (argatroban ęrst line; bivalirudin
hirudin, dabigatran if liver failure); Hirudin is irreversible
inhibitor; Dabigatran reversal: idarucizumab
tPA, streptokinase, urokinase Active plasminogen Monitor ębrinogen levels
Apixaban, rivaroxaban Direct factor Xa inhibitor Rivaroxaban reversed with andexanet alfa

IV. Transfusion Reactions

Related
Reaction Clinical ęndings Cause products Treatment
Febrile, Fever, pruritus, shivering as Cytokines from non- All products Stop transfusion (although
nonhemolytic transfusion is being given; leukoreduced donor (rarely no long-term eěects,
most common transfusion product plasma) need to evaluate why
reaction patient is febrile), control
symptoms (antipyretics,
antihistamines)
Febrile hemolytic Fevers, chills, hypotension, ABO incompatibility Usually RBCs Stop transfusion, give
chest/back pain; DIC, Ěuids, hemodynamic
hematuria, renal failure support
Urticarial Hives IgE reaction to product All products Symptomatic treatment
component (antihistamines)
Anaphylactic Hives, hypotension, Recipient anti-IgA All products Stop transfusion,
wheezing, angioedema, antibodies aĴack donor resuscitation,
hypoxemia IgA antibodies, often in epinephrine
IgA-deęcient patients
Sepsis Fevers, chills, hypotension, Microorganism in stored Usually platelets Antibiotics, hemodynamic
leukocytosis product (stored at support
room temp)
Transfusion- Respiratory distress, “Two-hit”: neutrophil All products Stop transfusion,
related lung hypoxemia, fever, sequestration and ventilatory support
injury (TRALI) hypotension, leukopenia, activation by donor
bilateral inęltrates on CXR; product
within 6 hrs of transfusion
Transfusion- Respiratory distress, Fluid overload; All products Diuresis, ventilatory
associated hypoxemia, JVD, underlying cardiac or support
circulatory hypertension, pulmonary renal dysfunction
overload edema; 6–12
(TACO) posĴransfusion
CHAPtEr 36 Transfusion and Disorders of Coagulation 453

V. Thromboelastography (TEG)
A. TEG is the best way to etermine which bloo proucts shoul be given to a bleeing patient
B. Interpretation:
1. R time—time to initial clot formation—if high, lacking coagulation factors -> give FFP
. K time—time to brin cross linking—if high, lacking brinogen -> give cryoprecipitate
3. a angel—rate of clot formation—if low, lacking brinogen -> give cryoprecipitate
4. MA (max amplitue)—maximum clot strength—if low, lacking platelets (contributes most to clot
strength) -> give platelet
5. LY30—rate of clot lysis—if high, increase brinolysis -> give TXA an/or aminocaproic aci
454 PArt ii Medical Knowledge

Questions
1. A 19-year-ol male is evaluate in the trauma 4. A 34-year-ol woman with no past meical
bay following a motorcycle accient. He is history presents with 6 weeks of left lower
foun to be hypotensive with an open book extremity pain an marke swelling an is foun
pelvic fracture. CT angiography of the pelvis to have a left iliofemoral DVT on CT venogram.
oes not emonstrate active extravasation. A She is given a heparin bolus, an a heparin rip
thrombelastography is performe showing an is starte. She then unergoes catheter-irecte
elevate K time an high LY30. He has receive thrombolysis (CDT). Which of the following is
bloo proucts an is 5 hours post injury. Which true?
of the following is true? A. The half-life of alteplase (tPA) is 4 to 6 hours
A. Cryoprecipitate woul not benet this patient B. Bleeing from inavertent overose may
B. K time is a measure of the time to initial clot benet from aministration of aminocaproic
formation aci
C. This ning is inicative of ecrease C. Bleeing risk best is best monitore by
brinogen levels following INR
D. FFP is inicate D. The rate of intracranial bleeing following
E. The patient shoul receive tranexamic aci CDT is higher than systemic thrombolysis
(TXA) E. The heparin rip shoul be continue uring
CDT
2. The risk of posttransfusion sepsis is greatest with:
A. Packe re bloo cells 5. Which of the following is true regaring the use
B. Cryoprecipitate of intraoperative bloo salvage (autotransfusion)?
C. Fresh frozen plasma A. Use of intraoperative bloo salvage may lea
D. Platelets to coagulopathy
E. Whole bloo B. Malignancy is an absolute contrainication
C. Autotransfusion can still be utilize if sterile
3. Which of the following is correct with regar to water is being use in the el
unfractionate heparin (UFH) an low-molecular- D. Most major abominal surgeries woul benet
weight heparin (LMWH)? from its use
A. LMWH is contrainicate while breastfeeing E. Activate clotting time (ACT) shoul be use
B. UFH is associate with fewer cases of heparin- intraoperatively to monitor for coagulopathy
inuce thrombocytopenia (HIT)
C. Protamine is more effective in reversing 6. A 49-year-ol female with a history of von
LMWH compare to UFH Willebran isease type 3 presents for scheule
D. LMWH oes not nee to be ose-ajuste in lobectomy for lung cancer. Which of the following
obese patients is the correct perioperative management?
E. LMWH is consiere superior in trauma A. Aminister 1 unit of FFP in the preoperative
patients with traumatic brain injury (TBI) holing area
B. Transfuse recombinant factor IX to 100%
normal levels prior to surgery
C. DDAVP shoul be given prior to incision
D. He can procee without any intervention for
von Willebran type 3
E. Preoperative von Willebran factor
concentrate shoul be aministere
CHAPtEr 36 Transfusion and Disorders of Coagulation 455

7. A 75-year-ol woman with a history of atrial 11. Which of the following oes not affect the
brillation on coumain presents to the ED with bleeing time?
a painful, enlarging bulge in her abominal wall. A. Aspirin
She is iagnose with a rectus sheath hematoma. B. von Willebran isease
Her INR is supratherapeutic at 5. She enies any C. Hemophilia A
recent coughing episoes or trauma. However, D. Severe thrombocytopenia
she reports starting a new meication. Which of E. Qualitative platelet isorers
the following meications coul have contribute
to her conition? 12. A eciency of which of the following factors
A. Cimetiine woul increase INR but not prolong the PTT?
B. Carbamazepine A. II
C. Rifampin B. V
D. Phenobarbital C. VII
E. Phenytoin D. IX
E. X
8. Persistent life-threatening bleeing in a patient
with Hemophilia A with high titers of inhibitors 13. The most important preoperative assessment to
(factor VIII alloantiboies) is best treate with: etermine the risk of abnormal intraoperative
A. A higher ose of factor VIII bleeing is:
B. Fresh frozen plasma A. Bleeing time
C. Cryoprecipitate B. Activate partial thromboplastin time (aPTT)
D. Recombinant factor VIIa C. International normalize ratio (INR)
E. DDAVP (esmopressin) D. History an physical examination
E. Platelet count
9. A 76-year-ol male is unergoing a laparoscopic
colectomy for sigmoi colon cancer. Which of 14. Glanzmann thrombasthenia is characterize by:
the following is the best prophylaxis for venous A. Normal bleeing time
thromboembolic events (VTEs)? B. Treatment response to DDAVP (esmopressin)
A. Leg compression evice infusion
B. Unfractionate heparin (UFH) until fully C. Autosomal ominant inheritance
ambulatory D. Defect in platelet aggregation
C. Leg compression evice intraoperatively, UFH E. Prolonge INR
until fully ambulatory
D. Leg compression evice intraoperatively, 15. Cryoprecipitate contains a low concentration of
LMWH until fully ambulatory which of following?
E. Leg compression evice intraoperatively, A. Fibrinogen
LMWH for 4 weeks after surgery B. Factor VIII
C. von Willebran factor
10. A 50-year-ol male unergoes a resection of a D. Fibronectin
large retroperitoneal leiomyosarcoma. There is E. Factor XI
an estimate bloo loss of 750 cc. The next ay,
the patient is foun to be anemic an is given  16. Which of the following is most likely to be useful
units of bloo. Halfway through the rst unit, in the treatment of bleeing in the uremic patient?
the patient evelops chills an his temperature A. Desmopressin
increases from 37 to 39°C. Which of the following B. Cryoprecipitate
is true in regar to this patient’s conition? C. Fresh frozen plasma
A. The transfusion oes not nee to be stoppe D. Recombinant human erythropoietin
B. This occurs more commonly when given E. Estrogens
packe re bloo cells versus platelets
C. Filtration is more effective than leukocyte
washing in preventing this conition
D. Aspirin is more effective than acetaminophen
in treating this conition
E. Pretransfusion aministration of
acetaminophen an iphenhyramine is the
most effective prevention
456 PArt ii Medical Knowledge

17. A 60-year-ol man with iabetes presents with 21. A 31-year-ol woman in her thir trimester
right upper quarant pain an leukocytosis. of pregnancy presents with fever, heaaches,
The patient has an elevate INR of .5 an a an myalgia. She is a former intravenous
prolonge PTT of 60 secons, a low brinogen rug user. She enies pruritus, but her skin
level, an a platelet count of 70,000 cells/μL. appears jaunice. Bloo pressure is normal.
An ultrasoun scan reveals gas in the wall of Her laboratory exam is remarkable for elevate
the gallblaer. The most important part in aspartate aminotransferase (AST) an alanine
management of this patient woul be: transaminase (ALT), hyperbilirubinemia as
A. Aministration of fresh frozen plasma well as thrombocytopenia, anemia, an severe
B. Aministration of cryoprecipitate hypoglycemia. From which of the following
C. Checking the D-imer assay conitions is she most likely suffering?
D. Emergent cholecystectomy A. HELLP (hemolysis, elevate liver enzymes,
E. Aministration of platelets low platelet count) synrome
B. Acute fatty liver of pregnancy (AFLP)
18. Which of the following is true in regar to von C. Intrahepatic cholestasis of pregnancy (ICP)
Willebran isease (vWD)? D. Preeclampsia
A. It is the secon most common congenital E. Hepatitis E
efect in hemostasis
B. Type 1 vWD is transmitte in an autosomal 22. A 35-year-ol man has been in the intensive
recessive fashion care unit sepsis ue to enterocutaneous stulas,
C. DDAVP (esmopressin) is helpful in type 3 ventilator epenence, an pneumonia for 
vWD weeks. He is receiving nutrition parenterally. The
D. Increase partial thromboplastin time (PTT) INR is .0. The aPTT is normal. The total bilirubin
rules out vWD level is normal. The platelet count is normal.
E. DDAVP is ineffective for type B vWD Which of the following is the most likely etiology?
A. Factor VIII eciency
19. A 40-year-ol female presents with a swollen B. DIC
left lower extremity, an ultrasoun conrms C. Vitamin K eciency
a eep venous thrombosis (DVT). The patient D. Primary brinolysis
is starte on therapeutic heparin but espite E. Chronic liver isease
progressively increasing the ose, the pharmacy
is having ifculty achieving a therapeutic partial 23. Which of the following electrolyte abnormalities
thromboplastin time (PTT) after 4 hours. Which are the most likely to occur with massive bloo
of the following is the best option? transfusion?
A. Convert from unfractionate heparin to low- A. Hypocalcemia, hypokalemia, an metabolic
molecular-weight heparin aciosis
B. Aminister fresh frozen plasma B. Hypercalcemia, hyperkalemia, an metabolic
C. Start a irect thrombin inhibitor alkalosis
D. Place an inferior vena cava lter C. Hypocalcemia, hyperkalemia, an metabolic
E. Continue to increase heparin ose as neee alkalosis
D. Hyponatremia, hyperkalemia, an metabolic
20. Which of the following is true regaring alkalosis
prothrombin complex concentrate (PCC)? E. Hyponatremia, hyperkalemia, an metabolic
A. Three-factor an 4-factor PCC refer to varying aciosis
concentrations of factor II
B. It is thawe more rapily than fresh frozen
plasma (FFP)
C. PCC reverses warfarin to an international
normalize ratio (INR) less than 1.5 within
30 minutes
D. PCC lowers INR as profounly as recombinant
factor VIIa
E. It reverses the anticoagulant effect of
abigatran
CHAPtEr 36 Transfusion and Disorders of Coagulation 457

24. A 75-year-ol male with a history of atrial 28. A 55-year-ol patient unergoes surgery, uring
brillation presents to the ED with an acute onset which bloo transfusions were given. One
of left lower extremity pain an pulselessness. week later, skin lesions evelop that appear to
Heparin is starte. He is foun to have an be purpura. The platelet count ecreases from
occlue popliteal artery. The clot is successfully 50,000 cells/μL to 10,000 cells/μL an an upper
cleare with thrombolytic therapy. He remains on gastrointestinal blee evelops. The patient has
a heparin rip with plans to convert to warfarin. not been receiving any meication that coul
However, on hospital ay 5 his platelet count affect platelets. Which of the following is true
rops to 160,000 u/L (from an amission level about this conition?
of 370,000 u/L). Which of the following is true A. It is more common in mile-age men
with regar to the rop in platelet count an the B. Severe bleeing is best manage by platelet
concern for heparin-inuce thrombocytopenia transfusions
(HIT)? C. It can occur without prior antigenic exposure
A. Because the platelet count is above 100,000 u/L, D. It is an antiboy-meiate reaction
heparin can be continue E. Platelet counts are typically higher than with
B. The risk of recurrent thrombosis at this point is heparin-inuce thrombocytopenia
low
C. Because the platelet count in’t rop until 29. Which of the following is true in regar to
ay 5, the concern for HIT is low clopiogrel (Plavix)?
D. HIT is less common in men A. It functionally mimics the pathophysiology of
E. Warfarin shoul be starte Bernar-Soulier isease
B. It has been linke to fatal episoes of
25. A 1-month-ol infant with mil skeletal pulmonary hypertension
abnormalities suffers a cariac arrest an passes C. It is recommene that clopiogrel be stoppe
away. On autopsy, he is foun to have extensive 3 ays before a major operation
thrombosis in his coronary arteries. Which of the D. It inhibits platelet aggregation within  hours
following is the most likely unerlying conition? of oral aministration
A. Factor V Leien mutation E. It can inhibit the release of von Willebran
B. Prothrombin gene mutation factor
C. Antithrombin III eciency
D. Homocystinuria 30. Which of the following factors has the shortest
E. Protein eciency half-life?
A. I
26. The most common cause of transfusion-relate B. II
eath is: C. VII
A. Infection D. IX
B. ABO incompatibility E. X
C. Acute lung injury
D. Delaye transfusion reaction 31. A 9-year-ol female is unergoing splenectomy
E. Graft-versus-host reaction for iiopathic thrombocytopenic purpura.
Intraoperatively, the surgeon notes a signicant
27. A 35-year-ol female evelops postpartum amount of bleeing at the splenic hilum
hemorrhage an requires a transfusion of packe uring mobilization. The surgeon woul like
re bloo cells an platelets. Twelve hours after to temporarily stop bleeing with a hemostatic
transfusion, the patient abruptly evelops rigors an agent. Which of the following woul be the least
chills. Her temperature increases to 39°C, her bloo effective choice for this patient?
pressure rops from 110/70 to 70/40 mmHg, an her A. Microbrillar collagen
heart rate increases from 80 to 10 beats per minute. B. Oxiize cellulose
Urine output rops, although the urine is clear. C. Thrombin
Despite attempts at resuscitation, the patient expires D. Fibrin sealant
within 4 hours. The eath is most likely ue to: E. Glutaralehye cross-linke peptie
A. Gram-positive sepsis
B. ABO incompatibility
C. Acute lung injury
D. Anaphylaxis
E. Gram-negative sepsis
458 PArt ii Medical Knowledge

Answers
1. C. The use of thromboelastography (TEG) has become 4. B. Alteplase, a tissue plasminogen activator (tPA),
more common in the setting of hemorrhagic shock seconary is the rug most commonly use in CDT an has a very
to trauma or cirrhosis. TEG provies real-time information short half-life (5 minutes) (A). tPA triggers the activation
about clotting activity an can guie resuscitation. K time of plasminogen into plasmin which then breaks brin cross
refers to the time to brin cross linking an an elevate K links to issolve clot. Aminocaproic aci is the treatment
time inicates a eciency of brinogen (B). Therefore, trans- of overose or reversal of tPA. Fibrinogen levels shoul
fusion of cryoprecipitate woul be inicate for this patient be monitore closely following thrombolysis. Low lev-
(A). Platelets woul be inicate in the event of a low MA. els of brinogen (usually less than 100 or 150 epening
R time on a TEG result refers to the time to initial clot forma- on clinical practice), are inicative of an increase risk of
tion, an if this is prolonge, transfusion of FFP is inicate bleeing events (C). Although CDT has lower rates of intra-
(D). A high LY30 is consistent with hyperbrinolysis an cranial hemorrhage than systemic thrombolysis (0%–1%
suggests the patient woul benet with TXA aministration. versus 3%–6%, respectively), the patient shoul be moni-
Trauma patients with massive hemorrhage receiving TXA tore closely while unergoing treatment (D). Many of the
have reuce all-cause mortality. However, this benet is same absolute contrainications to systemic thrombolysis
only seen for patients receiving TXA within 3 hours of injury. are true for CDT incluing recent stroke, active bleeing,
TXA aministration past the 3-hour mark is associate with an intracranial trauma. Systemic heparin shoul be hel
worse outcomes (E). uring lytic therapy ue to the risk of bleeing (E).
Reference: Roberts I, Shakur H, Coats T, et al. The CRASH- Reference: Fleck D, Albaawi H, Shamoun F, Knuttinen G,
trial: a ranomise controlle trial an economic evaluation of the Naiu S, Oklu R. Catheter-irecte thrombolysis of eep vein
effects of tranexamic aci on eath, vascular occlusive events an thrombosis: literature review an practice consierations. Cardiovasc
transfusion requirement in bleeing trauma patients. Health Technol Diagn Ther. 017;7(S3):S8–S37.
Assess. 013;17(10):1–79.
5. A. Intraoperative bloo salvage is recommene for
2. D. The risk of posttransfusion sepsis is greatest with clean (non-GI, noncontaminate) proceures with an esti-
platelet transfusion. The risk is the greatest in transfusion mate bloo loss of 500 to 1000 mL (e.g., cariac, liver, vas-
of poole platelet concentrates from multiple onor ver- cular, orthopeic cases) or more (D). This involves removing
sus single-onor platelet transfusion. Platelets are store at the patient’s bloo with a suction catheter uring surgery
°C which makes this bloo prouct the most vulnerable from the operative el. The bloo is then ltere, washe
to bacterial colonization an growth. If bacteria contamina- an returne to the patient. It has been shown to reuce the
tion of aministere bloo proucts is suspecte, the trans- amount of allogenic transfusion require. It also theoreti-
fusion shoul be stoppe immeiately an bloo cultures cally increases operating room efciency as there is less time
obtaine. neee to request an prepare allogenic prouct. Absolute
contrainications inclue mixture with other uis, partic-
3. E. The rate of serious bleeing complications has been ularly sterile water, as this hypotonic solution can lea to
shown to be lower with the use of LMWH compare to UFH. hemolysis (C). Malignancy is not an absolute contrainica-
It has also been shown to be associate with improve mor- tion; however, the risks an benets shoul be assesse on
tality in trauma patients with TBI. However, LMWH oes a case-by-case basis (B). Intraoperative bloo salvage only
not have a completely effective reversal agent available. Only replaces re bloo cells an therefore patients are at risk of
60% of the anticoagulant effect of LMWH can be reverse coagulopathy an ilution of coagulation factors. ACT mon-
with the aministration of protamine (C). Higher rates of itoring is use in the setting of systemic heparinization, not
major bleeing events have been shows in patients with for the use of re cell salvage. Goal ACT varies by provier
renal insufciency with the use of both UFH an LMWH. an proceure but 150 to 00 secons for routine anticoagu-
LMWH is renally cleare, however, an therefore shoul be lation is commonly use (E).
avoie in the setting of reuce creatinine clearance. UFH References: American Society of Anesthesiologists Task Force
unergoes excretion via the reticuloenothelial system an on Perioperative Bloo Management. Practice guielines for periop-
enothelial cells (D). Although both UFH an LMWH are erative bloo management: an upate report by the American
associate with the evelopment of HIT, this is more com- Society of Anesthesiologists Task Force on Perioperative Bloo Man-
monly seen after exposure to UFH (B). Either UFH, LMWH, agement. Anesthesiology. 015;1():41–75.
or warfarin can be safely use while breastfeeing (A). Carless PA, Henry DA, Moxey AJ, O’Connell D, Brown T, Fergus-
Patients with obesity have a larger volume of istribution son DA. Cell salvage for minimising perioperative allogeneic bloo
transfusion. Cochrane Database Syst Rev. 010;(4):CD001888.
of lipophilic rugs such as LMWH an as such will require
ose ajusting to reach aequate levels for thromboprophy-
laxis (D). 6. E. von Willebran isease (vWD) is the most common
Reference: Crowther MA, Berry LR, Monagle PT, Chan AKC. congenital bleeing isorer. Patients with WVD type 3
Mechanisms responsible for the failure of protamine to inactivate have the most severe bleeing iathesis among patients
low-molecular-weight heparin: inactivation of low-molecular-weight with VWD. In this type of VWD, there is an absence of von
heparin by protamine. Br J Haematol. 00;116(1):178–186. Willebran factor (vWF). DDAVP causes release of vWF an
CHAPtEr 36 Transfusion and Disorders of Coagulation 459

factor VIII from enothelial stores, an therefore, patients Kenet G, Lubetsky A, Luboshitz J, Martinowitz U. A new
with type 3 VWD are not responsive to DDAVP (C). The approach to treatment of bleeing episoes in young hemophilia
perioperative management for patients with VWD uner- patients: a single bolus megaose of recombinant activate factor
going major surgery (e.g., cariothoracic, hepatobiliary, VII (NovoSeven): recombinant FVIIa (NovoSeven) megaose. J
Thromb Haemost. 003;1(3):450–455.
neurologic, open vascular) inclues aministration of vWF
(D). For patients with Hemophilia B, recombinant factor IX
9. E. Patients unergoing surgery shoul be assesse for
shoul be aministere to a goal of 100% of normal factor IX
VTE risk an categorize as very low, low, moerate, an
levels preoperatively (B). FFP can be use to correct INR in
high-risk patients. The Caprini score can be use to facilitate
the acute setting (A).
the estimation. A score of 5 or more places a patient at high
Reference: Lavin M, O’Donnell JS. New treatment approaches
risk. Age of 75 years or more = 3 points, cancer =  points,
to von Willebran isease. Hematology Am Soc Hematol Educ Program.
016;016(1):683–689. an major open or laparoscopic surgery longer than 45 min-
utes is also  points. As such this patient woul be consi-
ere high risk. In low-risk patients, mechanical prevention
7. A. This patient has a supratherapeutic INR while on
(compression evice) is recommene. In moerate risk,
coumain. Coumain works by interfering with the gamma-
pharmacologic prophylaxis with either UFH or LMWH is
carboxylation of vitamin K-epenent coagulation factors
recommene. High-risk patients shoul get both mechan-
(factors II, VII, IX, X, protein C, S), an is metabolize by
ical an pharmacologic prophylaxis. The rug shoul be
the cytochrome-P450 in the liver. Several rug interactions
aministere close to surgery an continue until the patient
can lea to altere coumain metabolism. Meications that
is fully ambulatory. Recent ata in high-risk patients (such
inhibit cytochrome-P450 lea to ecrease coumain metab-
as those with cancer) emonstrate enhance VTE prophy-
olism an supratherapeutic INR. Inhibitors of cytochrome-
laxis with extene LMWH for 4 weeks after surgery (A–D).
P450 inclue cimetiine, amioarone, several antibiotics
Interestingly, recent ata inicate that patients unergoing
(macrolies, uoroquinolones, metroniazole, isoniazi, sul-
colectomy for inammatory bowel isease (IBD) are also at
fonamies), voriconazole, an grapefruit juice. Conversely,
very high risk for VTE (though IBD is not inclue in the
inucers of cytochrome-P450 will increase metabolism of
Caprini score).
warfarin ecreasing its effect. These patients may present
Reference: Veovati MC, Becattini C, Ronelli F, et al. A ran-
with a new venous thromboembolism even though they omize stuy on 1-week versus 4-week prophylaxis for venous
have been on the same ose of warfarin for years. Examples thromboembolism after laparoscopic surgery for colorectal cancer.
of cytochrome-P450 inclue carbamazepine, rifampin, phe- Ann Surg. 014;59(4):665–669.
nytoin, an phenobarbital (B–E).
10. C. The patient is likely manifesting a febrile nonhe-
8. D. Hemophilia A is a sex-linke recessive genetic coni- molytic transfusion reaction (FNHTR), the most common
tion an consiere the most common coagulation isorer, bloo transfusion reaction. It occurs in 0.5% to 1.5% of all
accounting for 80% of all inherite coagulation isorers. cases of bloo transfusion (A). It is ene as a rise in tem-
With time, as many as 10% to 15% of patients with factor perature of at least 1.8°C from baseline an is not accounte
VIII–ecient hemophilia A evelop inhibitors (alloanti- for by the patient’s clinical conition. However, FNHTR is
boies) against factor VIII. This is usually from previous a iagnosis of exclusion. As such, it is generally recom-
factor VIII transfusions. In situations in which life-threat- mene to at least temporarily stop the transfusion an
ening hemorrhage evelops, recombinant factor VIIa is the assess the patient. In particular, attention shoul be pai
best option. Another option is porcine factor VIII, but there to aitional symptoms an signs such as respiratory com-
is approximately a 5% cross-reactivity with inhibitors. Fac- promise, cyanosis, back pain, an hypotension; these may
tor VIIa complexes with tissue factor at the site of injury, suggest a hemolytic reaction, TRALI, or sepsis from con-
resulting in an activation of factor X, which then results in taminate bloo. FNHTR is more common in pregnancy
clot formation. Factor VIIa bypasses the requirement for an in patients with immunocompromise states (such as
factors VIII an IX an thus has been shown to be effec- leukemia, lymphoma). It occurs more commonly after the
tive in prevention an treatment of joint hemorrhage, the transfusion of platelets but can also occur with PRBC or
treatment of life-threatening bleeing, an the prevention FFP (B). Pretreatment with acetaminophen was thought to
of surgical bleeing. Restimulation of antiboies to factors reuce the severity of the complication. However, the only
VIII an IX shoul theoretically be less problematic than ranomize controlle trial to ate emonstrate no iffer-
with the use of plasma-erive proucts. The primary con- ence in the rate of FNHTR in patients that were pretreate
cerns with recombinant factor VIIa are the potential for with acetaminophen an iphenhyramine when com-
inucing thrombosis (stroke, eep venous thrombosis) an pare to a placebo (E). The incience of febrile reactions can
the high cost. A higher ose of factor VIII woul not efeat be greatly reuce by the use of leukocyte-reuce bloo
prouction of patient antiboies (A). Both fresh frozen proucts. Filtration removes 99.9% of the white bloo cells
plasma an cryoprecipitate contain factor VII but woul an platelets an is more effective than washing. Leukocyte
be ilute with other factors incluing factor VIII (B, C). reuction prevents almost all febrile transfusion reactions.
DDAVP woul not help a patient with a coagulation efect There is ebate in the literature as to whether leukocyte
(E). Other options that have been use but are only a tem- reuction leas to a ecrease in postoperative infections or
porary x in patients with signicant bleeing are plasma- mortality. Aspirin is not avise given its effects on plate-
pheresis an immune absorption. lets an bleeing (D).
References: DiMichele D. Inhibitors in hemophilia: a primer. Treat- References: Hébert PC, Fergusson D, Blajchman MA, et al.
ment of Hemophilia, 008;(7):1–4. Clinical outcomes following institution of the Canaian universal

AL GRAWANY
460 PArt ii Medical Knowledge

leukoreuction program for re bloo cell transfusions. JAMA. References: Chee YL, Crawfor JC, Watson HG, Greaves M.
003;89(15):1941–1949. Guielines on the assessment of bleeing risk prior to surgery or
Wang SE, Lara PN Jr, Lee-Ow A, et al. Acetaminophen an invasive proceures. British Committee for Stanars in Haematol-
iphenhyramine as premeication for platelet transfusions: a pro- ogy: British Committee for Stanars in Haematology. Br J Haematol.
spective ranomize ouble-blin placebo-controlle trial. Am J 008;140(5):496–504.
Hematol. 00;70(3):191–194. Chee YL, Greaves M. Role of coagulation testing in preicting
bleeing risk. Hematol J. 003;4(6):373–378.
11. C. Bleeing time tests platelet ahesion an aggrega- Klopfenstein CE. Preoperative clinical assessment of hemostatic
tion an will be normal in erangement of the coagulation function in patients scheule for a cariac operation. Ann Thorac
pathways. Hemophilia A is associate with a factor VIII Surg. 1996;6(6):1918–190.
eciency, which manifests as an abnormality in the coag- Suchman AL, Mushlin AI. How well oes the activate partial
thromboplastin time preict postoperative hemorrhage? JAMA.
ulation cascae an presents with a prolonge PTT. Drugs
1986;56(6):750–753.
that inhibit platelet function, such as aspirin (which works
by inhibiting cyclooxygenase), will increase bleeing time 14. D. Glanzmann thrombasthenia is an autosomal reces-
(A). von Willebran isease will result in prolonge blee- sive isorer that results in absence of functional glyco-
ing time because of the qualitative or quantitative eciency protein IIb/IIIa (C). Glycoprotein IIb/IIIa is a receptor for
in Willebran factor, which is require for platelet ahe- brinogen an von Willebran factor an causes platelet
sion to other platelets via the IIb/IIIa receptor (B). Severe ahesion an aggregation. Therefore, bleeing time will be
thrombocytopenia (quantitative) an platelet ysfunction prolonge, but aPTT an INR will be normal (A–E). These
(qualitative) both prolong bleeing time (D, E). Fibrinogen patients will not respon to DDAVP because there is no
eciency also prolongs bleeing time because brinogen is quantitative efect in the enothelial release of von Wille-
require for platelet aggregation. bran factor or factor VIII (von Willebran isease) (B). The
bleeing tenency for patients with Glanzmann’s is variable.
12. C. The INR etects abnormalities in the extrinsic an
Treatment is with platelets. Repeate use of platelet transfu-
common pathways. The extrinsic pathway is triggere by
sions can inuce antiglycoprotein IIb/IIIa alloimmunization,
exposure of the injure vessel to tissue factor an starts with
renering the treatment ineffective. In this circumstance,
factor VII. It then merges with the intrinsic pathway at factor
recombinant factor VIIa may be useful.
X (E) an is followe by activation of factors V an II an
References: ’Oiron R, Ménart C, Trzeciak MC, et al. Use
brinogen (factor I) (A, B). Thus, both the prothrombin time of recombinant factor VIIa in 3 patients with inherite type I
an the PTT will be prolonge in factors I, II, V, an X because Glanzmann’s thrombasthenia unergoing invasive proceures.
they are all part of the common pathway between the intrin- Thromb Haemost. 000;83(5):644–647.
sic an extrinsic pathways. Factor IX is part of the intrinsic Nuren AT. Glanzmann thrombasthenia. Orphanet J Rare Dis.
pathway an a eciency woul prolong PTT only (D). 006;1(1):10.

13. D. The most important element in etecting an 15. E. Cryoprecipitate contains all items liste as well as
increase risk of abnormal bleeing before surgery is a factor XIII. However, it contains low concentrations of factor
etaile history an physical examination. A systematic XI (A–D). Cryoprecipitate was originally create as a treat-
review in 008 emonstrate the poor value of using coagu- ment for hemophilia; however, it is now more often use in
lation tests when it came to ientifying the risk of bleeing patients receiving massive resuscitation in conjunction with
uring an operation (A–C, E). Other stuies have likewise fresh frozen plasma to replenish brinogen levels. Factor
shown that routine use of laboratory testing is neither sensi- XI eciency is also known as hemophilia C or Rosenthal
tive nor specic for etermining increase risk of bleeing. synrome, occurs more often in the Ashkenazi Jewish popu-
One nees to inquire about a history of prolonge bleeing lation, an is treate with fresh frozen plasma (uring blee-
after minor trauma, tooth extraction, menstruation, an in ing episoes).
association with major an minor surgery. In aition, one
must make inquiries into meications an over-the-counter 16. A. The etiology of abnormal bleeing in uremic patients
supplements that might affect hemostasis. If a careful his- is multifactorial, but the most important is impairment of
tory is negative an the planne surgical proceure is minor, platelet function that may be partly ue to a functional efect
then further testing is not necessary. A potential pitfall in in von Willebran factor, which leas to impaire platelet
relying solely on the history is that the history obtaine aggregation. DDAVP (esmopressin) seems to enhance the
might not be sufciently thorough or the patients might not release of von Willebran factor by enothelial cells. A single
recall or recognize that they ha previous abnormal blee- ose of 0.3 to 0.4 mcg/kg is given intravenously or subcu-
ing after an operation. If a major operation is planne that taneously. It has a rapi onset an relatively short ura-
is not a high-bleeing risk, then a platelet count, a bloo tion (4–6 hours). Dialysis is also effective in the treatment
smear, an an aPTT are recommene. If the history sug- of uremic bleeing by removing toxins that cause platelet
gests abnormal bleeing or the operation is either a high ysfunction. Cryoprecipitate has high concentrations of von
bleeing–risk operation or one in which even minor blee- Willebran factor as well as factor VIII an brinogen an
ing may have ire consequences (neurosurgery), then a may also be effective; however, it shoul not be rst-line ther-
bleeing time an INR shoul be ae an a brin clot to apy (B). Recombinant human erythropoietin (Epogen [epoe-
etect abnormal brinolysis. If there is high suspicion for a tin alfa]) has been shown to help uremic bleeing in several
history of abnormal bleeing, a hematology consult shoul stuies as well (D). In aition to stimulating erythropoie-
also be obtaine. sis, Epogen (epoetin alfa) enhances platelet aggregation. The
CHAPtEr 36 Transfusion and Disorders of Coagulation 461

increase re cell mass also seems to isplace platelets from o not make any vWF an therefore DDAVP will have no
the center of the bloo vessel an places them closer to the effect (C).
enothelium. Estrogens have been shown to help with blee- References: Holmberg L, Nilsson IM, Borge L, Gunnarsson M,
ing in men an women. The exact mechanism is unknown, Sjörin E. Platelet aggregation inuce by 1-esamino-8-D-arginine
but it is theorize that they ecrease arginine levels, which vasopressin (DDAVP) in Type IIB von Willebran’s isease. N Engl J
ecreases nitric oxie. This may lea to increases in throm- Med. 1983;309(14):816–81.
Tosetto A, Castaman G. How I treat type  variant forms of von
boxane A an aenosine iphosphate (E). FFP oes not
Willebran isease. Blood. 015;15(6):907–914.
have high concentrations of von Willebran factor an thus
is not effective for uremic bleeing (C).
19. B. Heparin resistance is ene as the nee for more
Reference: Heges SJ, Dehoney SB, Hooper JS, Amanzaeh J,
than 35,000 units in 4 hours to prolong the PTT into the ther-
Busti AJ. Evience-base treatment recommenations for uremic
bleeing. Nat Clin Pract Nephrol. 007;3(3):138–153.
apeutic range or as an activate clotting time (ACT) less than
400 secons espite excessive eman for heparin (>400–
600 IU/kg). Heparin resistance is most commonly the result
17. D. This is a classic presentation of emphysematous of antithrombin-III (ATIII) eciency. Heparin bins to ATIII
cholecystitis complicate by sepsis, which then resulte
causing a conformational change that results in its activation.
in DIC. Elerly male iabetic patients are at higher risk of
Activate ATIII then inactivates thrombin an other prote-
emphysematous cholecystitis, an gas in the gallblaer
ases involve in bloo clotting, most notably factor Xa. ATIII
conrms the iagnosis. DIC leas to a ysregulation of
eciency can be congenital or acquire. Hereitary ATIII
the coagulation cascae, leaing to clotting an resultant
eciency is rare (much less common than factor V Leien
bleeing. The consumption of brinogen, platelets, an
eciency) an can lea to venous thrombosis. Causes of
coagulation factors from the overactivation of the coagu-
acquire ATIII eciency inclue pregnancy, liver isease,
lation cascae results ultimately in iffuse bleeing. There
isseminate intravascular coagulation (DIC), nephrotic
is no specic test for DIC, but thrombocytopenia, hypo-
synrome, major surgery, acute thrombosis, an treatment
brinogenemia, prolonge PT an PTT, an the presence
with heparin. For this latter reason, measurement of ATIII
of increase brin egraation proucts are sufcient to
levels while on heparin is an inaccurate metho of ientify-
suggest the iagnosis of DIC (C). Fresh frozen plasma,
ing heparin resistance. Treatment of heparin resistance con-
platelets, an cryoprecipitate are all important compo-
sists of either aministering FFP or ATIII concentrates. FFP
nents of the treatment, especially for an actively bleeing
has the highest concentration of ATIII, an therefore patients
patient, but the most important part in the management
shoul be initially treate with FFP to replete ATIII in plasma,
of DIC is to ientify an correct the unerlying source,
followe by reaministration of heparin. A irect thrombin
which in this case is by broa-spectrum intravenous (IV)
inhibitor (argatroban) is a potential alternative; however, it
antibiotics an emergent cholecystectomy (A, B, E). With-
has the isavantage of having no way of being reverse in
out removal of the source, DIC will continue to consume
the case of overosage an bleeing (C). A isavantage of
transfuse proucts. The mortality rate from DIC ranges
FFP in the cariac surgery setting is that large volumes may
between 10% an 50%.
be require an it exposes the patient to the risks of trans-
Reference: Levi M, Toh CH, Thachil J, Watson HG. Guielines
fusions, incluing transfusion-relate lung injury (TRALI).
for the iagnosis an management of isseminate intravascular
coagulation. British Committee for Stanars in Haematology. Br J
Thus, in the setting of cariac bypass, ATIII concentrate is
Haematol. 009;145(1):4–33. another alternative (though it is very costly). Low-molecular-
weight heparin has no effect on ATIII eciency an shoul
not be use in this event (A). An inferior vena cava (IVC)
18. E. The most frequent congenital efect in hemostasis is lter woul be inicate if the patient began to blee while
vWD (A). Laboratory tests will emonstrate increase blee-
on heparin but not for heparin resistance. In fact, a lter,
ing time with a normal prothrombin time (PT). Patients may
though protective against PE, increases the risk for DVT, ue
have a normal or increase PTT because von Willebran
to the stasis it may create (D). Most patients achieve thera-
factor (vWF) is consiere a stabilizing factor for factor VIII
peutic PTT within 6 to 18 hours of starting heparin, so simply
(D). There are three types of vWD: Type I is an autosomal
increasing the heparin ose is not appropriate (E).
ominant isease characterize by a low level of vWF an
References: Kearon C, Akl EA, Comerota AJ, et al. Antithrom-
consiere the most common form of vWD (B). Type I is botic therapy for VTE isease: Antithrombotic Therapy an Preven-
treate with DDAVP because this increases circulating vWF tion of Thrombosis, 9th e: American College of Chest Physicians
release from enothelial cells. Type  vWD is also inherite Evience-Base Clinical Practice Guielines. Chest. 01;141(
in an autosomal ominant fashion an is characterize by a Suppl):e419S–e496S.
qualitative efect in which there is an appropriate amount of Spiess BD. Treating heparin resistance with antithrombin or fresh
vWF, but it oes not function properly. Type  has multiple frozen plasma. Ann Thorac Surg. 008;85(6):153–160.
variants, some that can be treate with DDAVP or cryopre-
cipitate. Type b, in particular, when treate with DDAVP 20. C. PCC is an inactivate concentrate of proteins C an
can inuce thrombocytopenia an form platelet complexes S, an factors II, IX, an X, with variable amounts of fac-
leaing to a prothrombotic state. DDAVP is contrainicate tor VII. PCC with normal amounts of factor VII is known
in type b but may be useful in other type  variants. Finally, as 4-factor PCC, while PCC with low levels of factor VII is
type 3 is the most severe form because there is no vWF pro- 3-factor PCC (A). Since 3-factor PCC has low levels of factor
uce by enothelial cells. It is transmitte in an autosomal VII, the aition of fresh frozen plasma is sometimes nec-
recessive fashion. For type 3, the recommene treatment essary for full reversal of warfarin an thus, 4-factor PCC
is recombinant vWF an factor VIII because these patients is superior. When a nonbleeing patient on warfarin nees
462 PArt ii Medical Knowledge

INR reversal, vitamin K is given, either orally (slower acting) turn leas to the liver’s inability to use vitamin K appropri-
or intramuscularly. If a patient is bleeing with an elevate ately. Factors II, VII, IX, an X as well as proteins C an S all
INR, vitamin K an an exogenous clotting factor formulation require vitamin K an will be ecient in these patients (A).
are given. The options are FFP, PCC, or recombinant factor Twenty percent of hospitalize patients given intravenous
VII (less often use). PPC has several avantages over FFP; nutrition over a 3-week perio evelope elevations of INR.
it oes not nee to be thawe (it is lyophilize [i.e., freeze Vitamin K shoul be given at least 6 to 1 hours before a pro-
rie]), it has a more rapi correction of INR, an it can be ceure in patients with aequate liver function. IM route of
infuse faster an with less volume (this also makes it ieal aministration is preferre because an IV push may result
for patients with congestive heart failure or chronic kiney in anaphylaxis. In patients with hepatocellular isease, FFP
isease) (B). Recombinant factor VIIa will lower INR faster or whole bloo is require. Platelets an cryoprecipitate are
than PCC (D). However, the concerns regaring recombinant unrelate to prolonge prothrombin time.
factor VIIa inclue the potential for inucing thrombosis References: Chakraverty R, Davison S, Peggs K, Stross P, Gar-
(stroke, eep venous thrombosis) as well as the high cost. rar C, Littlewoo TJ. The incience an cause of coagulopathies in
PPC oes not reverse the anticoagulant effect of abigatran; an intensive care population. Br J Haematol. 1996;93():460–463.
this can be accomplishe with iarucizumab (E). Crowther MA, McDonal E, Johnston M, Cook D. Vitamin K
eciency an D-imer levels in the intensive care unit: a prospec-
tive cohort stuy. Blood Coagul Fibrinolysis. 00;13(1):49–5.
21. B. AFLP is an uncommon but potentially fatal complica-
Duerksen DR, Papineau N. Clinical research: is routine vitamin K
tion that occurs in the thir trimester of pregnancy or uring
supplementation require in hospitalize patients receiving paren-
the early postpartum perio. It typically presents with a viral teral nutrition? Nutr Clin Pract. 000;15():81–83.
prorome characterize by fever, lethargy, malaise, an nau- Fiore LD, Scola MA, Cantillon CE, Brophy MT. Anaphylactoi
sea an vomiting. It is thought that AFLP may be the result of reactions to vitamin K. J Thromb Thrombolysis. 001;11():175–183.
mitochonrial ysfunction resulting in microvesicular fatty Shearer MJ. Vitamin K in parenteral nutrition. Gastroenterology.
inltration of hepatocytes without signicant inammation 009;137(Suppl. 5):S105–S118.
or necrosis. The mortality rate previously was very high;
however, with prompt iagnosis an treatment, the mater- 23. C. The correct answer is hypocalcemia, hyperkalemia,
nal an perinatal mortality have ecrease to 18% an 3%, an metabolic alkalosis (A, B, D, E). Severe hypocalcemia
respectively. Prompt elivery an intensive supportive care with massive bloo transfusion is uncommon an oes not
are the cornerstones in management of AFLP. Laboratory typically manifest unless the patient is receiving more than 1
abnormalities in AFLP inclue elevations of AST an ALT unit of packe re bloo cells (PRBCs) every 5 minutes. The
(usually less than 1000 IU/L), prolongation of PT an PTT, hypocalcemia is the result of citrate toxicity because the citrate
ecrease brinogen, renal failure, profoun hypoglycemia, in the transfuse bloo bins to circulating calcium in the
an hyperbilirubinemia. Laboratory stuies of AFLP are sim- patient. Because citrate is metabolize in the liver, hypocalce-
ilar to HELLP, but the key ning to help ifferentiate the mia can be more severe in patients with hepatic ysfunction.
two is hypoglycemia, which oes not occur commonly in Aitionally, the citrate is metabolize to bicarbonate lea-
HELLP (A). In aition, patients with HELLP typically have ing to metabolic alkalosis. Potassium concentration of store
preeclampsia, evience of hemolysis, an thrombocytopenia. PRBC is higher than human plasma potassium level. This is
Preeclampsia presents with hypertension, proteinuria, an thought to occur as a result of re bloo cell lysis uring stor-
rapi weight gain an can progress to seizures (eclampsia) age, releasing potassium in the supernatant. The concentration
(D). Patients with ICP report intense pruritus most commonly of potassium in PRBC increases linearly an is approximately
in the hans an soles of the feet that is unrelieve with anti- equal to the number of ays of PRBC storage.
histamines (C). Hepatitis E is cause by a single-strane Reference: Vraets A, Lin Y, Callum JL. Transfusion-associate
RNA virus. In men an nonpregnant women, it tens to be hyperkalemia. Transfus Med Rev. 011;5(3):184–196.
mil. However, it can lea to severe fulminant hepatic failure
in pregnant patients in the thir trimester, with a mortality 24. D. HIT occurs in approximately 1% to 1.% of patients
rate of up to 5% (particularly in eveloping countries) (E). receiving heparin. A scoring system has been evise to
References: Ko H, Yoshia EM. Acute fatty liver of pregnancy. assess risk of HIT, known as the 4 “T”s (Thrombocytopenia,
Can J Gastroenterol. 006;0(1):5–30. Timing, Thrombosis, an other causes for Thrombocytope-
Rahman TM, Wenon J. Severe hepatic ysfunction in pregnancy. nia). Variables that shoul heighten suspicion of HIT inclue
QJM. 00;95(6):343–357. a platelet count rop greater than 50%, occurrence between
Vigil-De Gracia P. Acute fatty liver an HELLP synrome: two is- ays 5 an 10 (it takes time for antiboies to evelop), nair
tinct pregnancy isorers. Int J Gynaecol Obstet. 001;73(3):15–0. of platelet count greater than 0,000 (nair below 10,000
is less likely HIT), no other reason for platelet count rop,
22. C. Several stuies have emonstrate that patients an new skin necrosis or VTE (C). Thus, more important
in the ICU have a high incience of coagulopathy an that than the absolute nair is the percentage rop (A). HIT is
vitamin K eciency is the most common cause (B, D, E). cause by antiboies that attack the heparin-platelet factor
The ifferential iagnosis for an elevate INR with a normal 4 (PF4) complex. Heparin-PF4 antiboies (sometimes calle
aPTT woul inclue a factor VII eciency, warfarin amin- “HIT antiboies”) in the resultant multimolecular immune
istration, the acute phase of liver isease, an vitamin K complex activate platelets via their FcγIIa receptors, causing
eciency. Vitamin K is not stable in patients receiving total the release of prothrombotic platelet-erive microparticles,
parenteral nutrition; therefore, in this case, the prolonge PT platelet consumption, an thrombocytopenia. The micropar-
correlates with vitamin K eciency. Prolonge parenteral ticles in turn promote excessive thrombin generation, fre-
nutrition often leas to cholestatic liver isease, which in quently resulting in thrombosis. Patients receiving any type
CHAPtEr 36 Transfusion and Disorders of Coagulation 463

of heparin at any ose an by any route of aministration nonleukocyte-reuce bloo transfusions have been associ-
are at risk of eveloping HIT antiboies. It oes occur less ate with increase mortality when compare with leukocyte-
commonly in men an occurs more frequently in the elerly. reuce bloo transfusions.
However, not all of those with HIT antiboies will necessar- Reference: Vamvakas E, Blajchman M. Transfusion-relate mor-
ily evelop the clinical synrome. If this is suspecte, heparin tality: the ongoing risks of allogeneic bloo transfusion an the avail-
shoul be iscontinue, an the patient shoul be starte on able strategies for their prevention. Blood. 009;113(15):3406–3417.
a irect thrombin inhibitor (E). If anticoagulation is not initi-
ate, the chance of another thromboembolic event is approx- 27. E. Bacterial contamination of bloo is the most fre-
imately 5% to 10% per ay (B). Diagnosis is performe by quent cause of eath from transfusion-transmitte infec-
an ELISA antiboy test. If these results are equivocal, then a tious isease an is the thir most common cause of eath
conrmatory serotonin release assay shoul be performe. overall in a large series (after acute lung injury an ABO
References: Ahme I, Majee A, Powell R. Heparin inuce incompatibility) (B, C). A key feature of ABO incompati-
thrombocytopenia: iagnosis an management upate. Postgrad bility (hemolytic reaction) is the evelopment of re urine
Med J. 007;83(983):575–58. (hemoglobinuria). Patients also often complain of back pain
Jang IK, Hursting MJ. When heparins promote thrombo- an a sense of oom. Acute lung injury manifests with rapi
sis: review of heparin-inuce thrombocytopenia. Circulation. onset of yspnea an tachypnea aroun 6 hours after trans-
005;111(0):671–683. fusion. Anaphylactic reaction rarely occurs (D). Bacterial
Warkentin TE, Haywar CP, Boshkov LK, et al. Sera from patients contamination now accounts for 1 in every 38,500 cases of
with heparin-inuce thrombocytopenia generate platelet-erive
bloo transfusion. This increase ha coincie with a ra-
microparticles with procoagulant activity: an explanation for the
matic ecrease in viral infections. The highest risk of bacte-
thrombotic complications of heparin-inuce thrombocytopenia.
Blood. 1994;84(11):3691–3699.
rial infection is from poole platelet transfusions because
Wheeler HB. Diagnosis of eep vein thrombosis. Review of many microorganisms can live an propagate uner the
clinical evaluation an impeance plethysmography. Am J Surg. storage conitions of platelets (0–4°C). Gram-negative
1985;150(4A):7–13. sepsis is the most lethal (A), an Yersinia is one of the most
common organisms. Gram-negative sepsis can become
25. D. Although all the answer choices can increase the clinically apparent within 9 to 4 hours after bloo transfu-
risk of venous thromboembolism, homocystinuria is the sion. Cytomegalovirus is the most common infectious agent
most common inherite conition preisposing patients transmitte, but because it is so ubiquitous, it is generally
to arterial thrombosis an affects 5% to 10% of the popula- not a threat to most patients. The exception to that rule is
tion. It is an autosomal recessive isease. Homocystinuria the transplant recipient.
is most commonly cause by a eciency of cystathionine References: Benjamin RJ. Transfusion-relate sepsis: a silent
beta-synthase resulting in an elevate level of homocysteine epiemic. Blood. 016;17(4):380–381.
in plasma an urine. The toxic effect of an elevate level Bihl F, Castelli D, Marincola F, Do RY, Braner C. Transfusion-
of homocysteine in the brain results in mental retaration transmitte infections. J Transl Med. 007;5(1):5.
Kuehnert M, Roth V, Haley N, et al. Transfusion-transmitte bac-
as well as seizures. Skeletal abnormalities (marfanoi hab-
terial infection in the Unite States, 1998 through 000. Transfusion.
itus)may occur seconary to the interference of collagen
001;41(1):1493–1499.
cross-linking. Patients are at increase risk of thrombosis
ue to the isruption of vascular enothelium by homo-
28. D. Transfusion purpura is an uncommon cause of
cysteine leaing to platelet activation an aggregation.
thrombocytopenia an bleeing after transfusion. A small
Patients ientie early to have this conition will benet
minority of patients lack the HPA-1a antigen on their plate-
with aministration of pyrioxine (vitamin B6) to inuce
lets that is present in almost all humans. Transfusion purpura
cystathionine beta-synthase activity. Factor V Leien muta-
requires that the patient has been previously sensitize to the
tion is the most common inherite conition increasing the
HPA-1a antigen; this happens usually by a prior pregnancy
risk of venous thromboembolism followe by prothrombin
or previous bloo transfusion. When these patients later
gene mutation (A, B). Patients that o not have a response
receive bloo proucts that contain a small number of plate-
to the aministration of unfractionate heparin may have
lets with the ubiquitous HPA-1a, they prouce alloantiboies
antithrombin III eciency (C). Protein C eciency is a rare
that attack both the onor’s an the patient’s own platelets
cause of venous thromboembolism (E).
(C). This usually presents 5 to 1 ays after a transfusion an
References: D’Angelo A, Selhub J. Homocysteine an throm-
leas to profoun thrombocytopenia an bleeing that can
botic isease. Blood. 1997;90(1):1–11.
Greico AJ. Homocystinuria: pathogenetic mechanisms. Am J Med
last for weeks. Mortality occurs in 10% to 0% ue to hem-
Sci. 1977;73():10–13. orrhage. Although sensitization can occur after prior bloo
Rosenaal FR. Risk factors for venous thrombosis: prevalence, transfusions, it has become less common with leukocyte-re-
risk, an interaction. Semin Hematol. 1997;34(3):171–187. uce re cells an therefore this issue is most common in
women who have been pregnant (A). Diagnosis is mae by
26. C. The leaing causes of allogeneic bloo transfusion emonstrating platelet alloantiboies with an absence of the
(ABT)–relate mortality in the Unite States (in the orer corresponing antigen on the patient’s platelets. Treatment is
of reporte number of eaths) inclue transfusion-relate primarily with intravenous immunoglobulin (IVIG). Plasma-
acute lung injury (TRALI), ABO an non-ABO hemolytic pheresis an corticosterois are also potential options. Treat-
transfusion reactions, an transfusion-associate sepsis (A, ment with platelet transfusions can exacerbate the isease
B, D). Graft-versus-host reaction is not a common cause process (B). The presentation can easily be confuse with
of ABT (E). Aitionally, it has been emonstrate that heparin-inuce thrombocytopenia without appropriate
464 PArt ii Medical Knowledge

testing. A platelet count of fewer than 15,000 cells/μL is more 30. C. Warfarin acts in the liver by blocking the vitamin
suggestive of transfusion purpura (E). K–epenent factors (II, VII, IX, an X). Of these, factor VII
References: Hillyer CD, Hillyer KL, Strobl FJ, Jefferies LC, Sil- has the shortest half-life (A, B, D, E). A eciency in factor
berstein LE, es. Handbook of transfusion medicine. Acaemic Press; VII manifests by a prolongation of the prothrombin time an
001:38. the international normalize ratio. Vitamin K is critical in the
Lubenow N, Eichler P, Albrecht D. Very low platelet counts in γ-carboxylation of these factors that are synthesize in the
post-transfusion purpura falsely iagnose as heparin-inuce
liver. Patients with hepatic ysfunction woul similarly is-
thrombocytopenia: report of four cases an review of literature.
play prolonge prothrombin time.
Thromb Res. 000;100(3):115–15.

29. D. Clopiogrel (Plavix) irreversibly inhibits platelet 31. A. Hemostatic agents are increasingly use intraopera-
aggregation within  hours of aministration an its effects tively to provie a temporary measure of controlling blee-
last 5 to 7 ays (the half-life of platelets is 1 week) (C, D). It ing when cautery is angerous or inaccessible. Collagen
works by inirectly inhibiting the activation of the glycopro- can provie hemostasis by allowing a large surface area for
tein IIb/IIIa complex (E). It oes this by antagonizing the platelet aherence leaing to thrombus clot. However, this
ADP receptor which, when activate, inserts glycoprotein will not work well in patients with thrombocytopenia. Oxi-
IIb/IIIa receptors on the platelet’s surface. This is functionally ize cellulose promotes re cell lysis generating an articial
similar to Glanzmann thrombasthenia, which is characterize clot an can even be use uring enoscopic proceures. It
by a GpIIb/IIIa receptor eciency on platelets preventing may also have an antimicrobial effect since it ecreases local
brin from linking platelets together. Bernar-Soulier is- tissue pH (B). Thrombin uses bloo as a source of brinogen
ease is characterize by GpIb receptor eciency on platelets to create a clot (C). In contrast, brin sealant is compose
which prevents vWF from linking the platelet to expose col- of both brinogen an thrombin (D). Glutaralehye cross-
lagen on amage tissue (A). Clopiogrel has been shown linke peptie (commonly albumin) forms a scaffol for clot
to ecrease the rate of a combine enpoint of cariovascu- formation an can be use even on wet surfaces (E).
lar eath, myocarial infarction, an stroke in patients with References: Emilia M, Luca S, Francesca B, et al. Topical
hemostatic agents in surgical practice. Transfus Apher Sci. 011;
acute coronary synromes. Use with aspirin increases the
45(3):305–311.
risk of bleeing. Clopiogrel has been associate with the Skinner M, Velazquez-Avina J, Mönkemüller K. Overtube-assiste
evelopment of thrombotic thrombocytopenic purpura, even enoscopic application of oxiize cellulose to achieve hemostasis in
with short-term use (< weeks). Treatment is with plasma anastomotic ulcer bleeing. Gastrointest Endosc. 014;80(5):917–918.
exchange. The mortality rate is as high as 9%. It has not been
associate with pulmonary hypertension (B).
Wound Healing
ERIC O. YEATES, AREG GRIGORIAN, AND CHRISTIAN DE VIRGILIO 37
ABSITE 99th Percentile High-Yields
I. Phases of Woun Healing

Days Cell types Description


Hemostasis 1 Platelets Initial transient vasoconstriction, ADP released, platelets aggregate
and cause thrombosis
InĚammatory 1–10 PMNs (1–3) Vasodilation, PMNs phagocytose debris and bacteria, macrophages
Macrophages (4–5) essential for wound healing and release growth factors/cytokines;
Lymphocytes (5–6) nicotine is a vasoconstrictor that impairs oxygen delivery,
Fibroblasts (6+) increases platelet adhesion, and inhibits proliferation of red blood
cells, ębroblasts, and macrophages, all of which impair wound
healing
Proliferative 5–21 Fibroblasts Fibroblasts deposit collagen and glycosaminoglycans,
neovascularization, granulation tissue, epithelialization
Remodeling 21–365 Fibroblasts Decreased vascularity, type III collagen replaces type I, collagen
cross-linking (max strength is 80% at 8 weeks)

II. Nutrients an Woun Healing

Function Deęciency
Vitamin A Increases inĚammatory response in wounds, stimulates Blindness, rash, delayed wound healing
collagen synthesis, counteracts eěects of steroids or
radiation on wound healing (patient does NOT need to
be vitamin A deęcient to beneęt from this)
Vitamin C Collagen synthesis and crosslinking, angiogenesis, Scurvy (easy bruising, bleeding gums, poor
antioxidant, increases iron absorption; large doses may wound healing)
even inhibit wound healing
Vitamin E Fat-soluble antioxidant, no evidence that it improves Ataxia, peripheral neuropathy, retinopathy,
wound healing or scar appearance impaired immune response
Iron Transports oxygen, metabolism of collagen Fatigue, anemia, impaired wound healing
Zinc Cofactor for collagen formation and many other Rash, alopecia, impaired immune function,
enzymatic reactions in wound healing diarrhea, delayed wound healing,
reduced wound strength
Copper Stimulates ębroblasts proliferation, upregulates collagen Anemia, myelopathy, neuropathy, impaired
production wound healing

465
466 PArt ii Medical Knowledge

Questions
1. Which of the following is true regaring 4. Which of the following is true regaring cell
hyperbaric oxygen therapy (HBOT) an wouns? junctions in humans?
A. HBOT is now wiely aopte in hospitals A. Hemiesmosomes o not interact with
across the Unite States intermeiate laments
B. Topical oxygen treatment (TOT) is as effective B. Tight junctions, by enition, o not allow
as HBOT for wouns involving bone the passage of solutes through ajacent cell
C. Transcutaneous oxygen measurements membranes
(TCOM) are useful in preicting woun C. Connexons allow for irect communication
healing with HBOT between two ajacent cells
D. HBOT ecreases the major amputation rate in D. Aherens junctions are a specialize type of
patients with iabetic foot ulcers tight junction
E. HBOT typically takes at least 6 months to E. Desmosomes function primarily to anchor a
show effectiveness cell to the extracellular matrix

2. Which of the following is true regaring iabetic 5. Which of the following is true regaring
foot ulcers? nutritional status an nonhealing wouns?
A. The lifetime risk of a patient with iabetes A. Short perios of starvation before surgery
eveloping a foot ulcer is approximately 5% generally have minimal effect on woun
B. It is primarily ue to thrombotic occlusion of healing
istal vasculature B. Malnutrition prolongs the inammatory phase
C. Enzymatic ebriement of iabetic foot ulcers of woun healing
shoul be avoie C. Prealbumin will provie an accurate
D. Sharp ebriement of iabetic foot ulcers estimation of nutritional status over the
shoul be avoie previous several weeks
E. Total-contact casts can be use in the setting of D. Nutritional supplements have been shown to
iabetic foot ulcers to promote healing ecrease interval time to complete healing of
pressure ulcers
3. Which of the following is true regaring woun E. Presence of granulation tissue is not preictive
ressings? of aequate woun healing ability
A. Honey ressings are not inicate in wouns
with slough or necrotic tissue 6. A -year-ol female with history of a gunshot
B. Calcium alginate ressings are ieal in woun to the abomen requiring multiple bowel
wouns with a large amount of exuate resections has been on chronic total peripheral
C. Wet-to-ry ressings remain the gol stanar nutrition (TPN) for short gut synrome. She
for woun ressings in most scenarios presents for a clinic follow-up stating that
D. Moisture-retentive ressings have similar rates her hair has starte to fall out, an she has
of infection compare to gauze ressings evelope multiple bruises over her arms an
E. Hyrogels with silver can be use in legs. In aition, she has a iffuse scaly rash an
combination with enzymatic ebriing agents ry skin. In which following nutrients or trace
elements is she likely ecient?
A. Copper
B. Vitamin C
C. Linoleic aci
D. Zinc
E. Selenium
CHAPtEr 37 Wound Healing 467

7. Which of the following is true regaring skin 10. A severely malnourishe 1-year-ol boy presents
antiseptic techniques before surgery? with multiple pigmente spots on his bilateral
A. Ioine-base preps are superior to thighs, bleeing gums, loose an missing teeth,
chlorhexiine for preventing surgical site an several weeping wouns. He recently
infections arrive as a refugee from an unerevelope
B. Chlorhexiine-base preps are safe on all boy country. His meical history is sparse. His iet
surfaces as a preoperative cleanser primarily consiste of cooke grains. Which of
C. Preoperative bathing with chlorhexiine has the following is true regaring the most likely
been shown to reuce incience of surgical site vitamin eciency in this patient?
infections A. It plays an essential step in proteoglycan
D. The bactericial effect of ioine erives synthesis
fromits ability to form an extracellular B. Delaye woun healing is cause by failure to
crystalmatrix an estabilize cell hyroxylate lysine an proline uring collagen
membranes synthesis
E. Povione-ioine was formulate to ecrease C. It oes not affect iron absorption
the availability of molecular ioine D. Exogenous aministration has been shown in
animals to have a corticosteroi-like effect on
8. Which of the following is true about woun woun healing
healing? E. After hyroxylation by the liver an kiney, it
A. Angiogenesis is the major contributor to the helps with bone mineralization
erythema seen in wouns
B. Pain in the rst 48 hours is seconary to newly 11. Which of the following iseases is correctly
active broblasts attempting to contract the paire with the type of collagen affecte?
woun eges A. Alport synrome: type III collagen
C. At 48 hours, phagocytic cells preominate in B. Ehlers-Danlos synrome: type VII collagen
the woun be C. Epiermolysis bullosa: type VII collagen
D. In the rst 36 hours, macrophages are the D. Osteogenesis imperfecta: type II collagen
preominate cells in the woun be E. Bullous pemphigoi: type I collagen
E. While erythema an pain can be normal,
inuration is typically pathologic 12. Which of the following is true regaring the
healing of a small-bowel anastomosis?
9. Which of the following is true regaring kelois A. Leaks are less likely to occur with a han-sewn
an/or hypertrophic scars? anastomosis as compare with staple
A. Kelois are associate with an increase B. There is a ecrease level of collagenase when
eposition of collagen compare to healing skin wouns
B. Low-ose raiation is a better ajunct for C. The serosa plays a minimal role in the healing
treatment of hypertrophic scars of a small-bowel anastomosis
C. Kelois can appear years after a minor injury D. The submucosa provies the most signicant
D. Kelois are much more common after burn strength layer of the anastomosis
injuries than hypertrophic scars E. Free omental aps have been shown to
E. Hypertrophic scars ten to exten beyon improve outcomes when oing a small-bowel
woun borers with time anastomosis

Answers
1. C. HBOT for woun healing remains incompletely of oxygen to the woun be, as well as angiogenesis stimu-
aopte, likely ue to cost an inconsistent efcacy in clini- lation. Sie effects of HBOT are rare but inclue claustropho-
cal trials (A). HBOT typically involves placing the patient in a bia, barotrauma, heaache, an tinnitus. Though there is still
pressurize chamber to aroun .0 atmospheres an amin- ebate on its efcacy, a large systematic review in 015 sug-
istering 100% oxygen for 1 to  hours. These sessions can geste that HBOT improve healing rates at 6 weeks, but not
be performe once or twice aily for  to 4 weeks. HBOT is at one year (E). It also i not ecrease the major amputation
thought to work by elivering high arterial partial pressures rate compare to conventional woun care alone (D). There
468 PArt ii Medical Knowledge

is goo evience that TCOMs taken after HBOT can preict to use in combination with enzymatic ebriing agents, as
the efcacy of therapy with impressive accuracy. Specically, they may become inactive (E). Other wouns have a large
iabetic foot ulcers with TCOMs >00 mmHg heale 90% of amount of exuate which nees to be controlle in orer to
the time with HBOT (C). TOT applies 100% oxygen irectly prevent maceration an improve woun healing. Both foam
to the woun an has been stuie far less than HBOT. ressings an alginate ressings, which can absorb 0 times
Though there are some clinical trials suggesting efcacy in their weight, can be use in this scenario (B). Enzymatic
some patients, it is accepte that TOT oes not penetrate to ebriing agents, like collagenase (e.g., Santyl) an meical-
bone ue to its mechanism of action (B). grae honey, are also commonly use to remove slough an
References: Kranke P, Bennett MH, Martyn-St James M, Schna- necrotic tissues (A).
bel A, Debus SE, Weibel S. Hyperbaric oxygen therapy for chronic Reference: Niezgoa JA, Baranoski S, Ayello EA, et al. Woun
wouns. Cochrane Database Syst Rev. 015;(6):CD00413. treatment options. In: Baranoski S, Ayello EA, es.Wound care essen-
Moon H, Strauss MB, La SS, Miller SS. The valiity of transcuta- tials. 5th e. Wolters Kluwer Health; 00:184–41.
neous oxygen measurements in preicting healing of iabetic foot
ulcers. Undersea Hyperb Med. 016;43(6):641–648. 4. C. All humans have three main types of cell junctions:
Mutluoglu M, Cakkalkurt A, Uzun G, Aktas S. Topical oxygen anchoring junctions, communicating (gap) junctions, an
for chronic wouns: a PRO/CON ebate. J Am Coll Clin Wound Spec.
tight junctions. The rst group (anchoring junctions) is fur-
013;5(3):61–65.
ther subivie into esmosomes, hemiesmosomes, an
aherens junctions (D). Hemiesmosomes an esmosomes
2. E. Diabetic foot ulcers are very common, with the lifetime
both connect with intermeiate laments in the cytoskele-
risk of a patient with iabetes approximately 5% (A). Dia-
ton, but the former connects cells to the unerlying extracel-
betic foot ulcers are mainly cause by peripheral neuropathy
lular matrix, an esmosomes connect ajacent cells to one
leaing to scrapes/cuts of the foot that may go unnotice for
another (A, E). Aherens junctions serve the same purpose
several ays (B). There is also autonomic neuropathy leaing
but use actin laments as their cytoskeletal anchor. Anchor-
to failure of sweating. This manifests as ry skin at risk for
ing junctions, as a whole, provie structural integrity to a tis-
mechanical breakown which can initiate ulcer formation.
sue mae up of iniviual cells. Communicating junctions
Aitionally, autonomic ysregulation of the microcircula-
allow irect chemical communication between ajacent cells.
tion results in poor ow to istal extremities preventing ae-
This is facilitate by six iniviual subunits, calle connex-
quate woun healing. Preventative care is paramount in the
ins, which form a central pore, calle a connexon. When two
prevention of iabetic foot ulcers an inclues maintaining
connexons from ajacent cells come in contact, a channel is
normoglycemia an aily exams for occult scrapes/cuts of
forme allowing communication between the two cells. The
the foot along with aily moisturizer use. Diabetic foot ulcers
nal group, tight junctions, refers to a group of proteins that
shoul be manage with a combination of ebriement an
allow the selective iffusion of molecules base mainly on
off-loaing. Sharp ebriement, enzymatic ebriement,
size, molecular charge, an polarity. These primarily act as
biological ebriement, an autolytic ebriement are all
selective barriers such as in the ifferent layers of the skin (B).
acceptable methos of removing ebris an necrotic tissue
(C, D). Ofoaing is also critical to woun healing, with
nonremovable total-contact casts being the gol stanar.
5. B. Delaye woun healing is a multifactorial problem
with many ientiable risk factors incluing malnutrition,
Contrainications to this type of cast are ischemia, ongoing
vitamin eciencies, smoking, obesity, iabetes, an hypox-
infection, osteomyelitis, an poor skin quality.
emia. However, few systemic factors have been shown to
References: Alexiaou K, Doupis J. Management of iabetic
foot ulcers. Diabetes Ther. 01;3(1):4. spee up woun healing. Short perios of starvation can
Boulton AJM, Armstrong DG, Albert SF, et al. Comprehensive have negative effects on postoperative woun healing (A).
foot examination an risk assessment: a report of the task force of This seems to occur primarily by prolongation of the inam-
the foot care interest group of the American Diabetes Association, matory phase because there are inaequate builing blocks
with enorsement by the American Association of Clinical Enocri- for cell proliferation, protein synthesis, an creation of new
nologists. Diabetes Care. 008;31(8):1679–1685. DNA. The notion that malnutrition plays a key role in the
evelopment of chronic wouns le to multiple stuies
3. B. A large boy of evience supports the utilization of aime at etermining if nutritional supplementation can
a moist woun environment to promote faster woun heal- prevent chronic wouns or spee recovery. A Cochrane
ing an less scar formation. Therefore, the woun ressing review one in 014 looking at 3 ranomize controlle tri-
shoul create an ieal amount of moisture by either aing als evaluating the effect of enteral an parenteral nutrition
moisture to ry wouns or absorbing it from highly exua- on the prevention an treatment of pressure ulcers foun no
tive wouns. Despite the progress mae in woun ress- clear benet of any intervention (D). By knowing the half-
ings, many outate strategies like wet-to-ry ressings are lives an current serum measurements of certain proteins,
overly use. Wet-to-ry ressings, most often gauze, are rst we are able to estimate the synthetic ability of the liver over
allowe to ry on the woun an then are remove, resulting a given time perio. Albumin (14–0 ays), transferrin (8–9
in nonselective ebriement of both slough an healthy tissue ays), an prealbumin (–3 ays) all give a snapshot into
(C). Aitionally, moisture-retentive ressings like hyro- someone’s nutritional status but nee to be combine with
collois an transparent lms have a lower infection rate the entire clinical picture (C). Granulation tissue, if present,
than gauze ressings (D). Some wouns, espite moisture- is preictive of aequate woun healing (E).
retentive ressings, remain too ry an require supple- References: Greenel LJ, Mulhollan MW, es. Greeneld’s
mentation. Hyrogels are useful in this case as they are surgery: scientic principles & practice. 5th e. Lippincott Williams an
hyrating to the woun be, but care shoul be taken not Wilkins; 011.
CHAPtEr 37 Wound Healing 469

Langer G, Fink A. Nutritional interventions for preventing an Mangram AJ, Horan TC, Pearson ML, et al. Guieline for the
treating pressure ulcers. Cochrane Database Syst Rev. 014;(6):CD00316. prevention of surgical site infection. Infect Control Hosp Epidemiol.
Stechmiller JK. Unerstaning the role of nutrition an woun 1990;0:47–80.
healing. Nutr Clin Pract. 010;5(1):61–68.
8. C. Woun healing is typically ivie into 3 or 4 phases:
6. C. A eciency of trace elements an essential fatty acis hemostasis/inammation (combine in the 3-phase moel),
is a relatively rare entity in patients taking foo by mouth. proliferation, an maturation (or remoeling). The hemosta-
However, it has occurre with increase frequency with the sis/inammation phase is initiate with the isruption of
avent an wiesprea use of TPN, particularly in patients capillaries resulting in hemorrhage. This immeiately causes
with a history of short gut synrome. Copper is primarily vasoconstriction to assist with the formation of a platelet
associate with anemia resistant to iron supplementation, plug. After 10 to 15 minutes, local tissue factors an platelets
leukopenia, an neurologic efects (A). Vitamin C eciency, begin to facilitate vasoilation an increase vascular perme-
or scurvy, causes elaye woun healing, bleeing gums, ability. The inltration of ui an cells (mainly neutrophils)
loose teeth, an abnormal bone eposition in chilren (B). causes the woun to become erythematous (A). In aition,
Selenium eciency is associate with a fatal cariomyopa- the woun is warm an eematous (inuration) (E). At this
thy (E). Zinc an essential fatty aci eciency (linoleic aci point, changes in tissue pH an local tissue estruction cause
an alpha-linolenic aci) have many similar features inclu- the woun to be painful (B). The rst cells to arrive after for-
ing elaye woun healing, increase infections, iarrhea, mation of a platelet plug are neutrophils, which on’t seem
an a rash. However, the essential fatty aci rash tens to to be critical to healing an mainly help with phagocytosis of
be scalier an is associate with ry skin, an the rash from bacteria an estruction of ea tissue. Neutrophil preom-
zinc is primarily locate in the perioral area an intertrigi- inance persists for 48 hours, at which point they are largely
nous skin of the ngers an toes. While alopecia an throm- replace by macrophages, which will remain in the woun
bocytopenia can be foun with both conitions, it is more until the completion of healing (D). Macrophages are argu-
closely associate with free fatty aci eciency. Conversely, ably the most important cell in healing because of their effects
the impaire taste, night blinness, an loss of appetite are on angiogenesis, matrix eposition, an remoeling via the
more closely relate with zinc eciency (D). release of cytokines an growth factors. By ay 4, the prolifer-
References: Jeppesen PB, Høy CE, Mortensen PB. Essential ative phase begins an enothelial cells an broblasts begin
fatty aci eciency in patients receiving home parenteral nutrition. to appear in the woun. By ays 5 to 7, there is no longer a
Am J Clin Nutr. 1998;68(1):16–133. signicant population of inammatory cells. The previously
Kumar V, Fausto N, Abbas A, es. Robbins and Cotran pathologic
create matrix of type III collagen is slowly replace with
basis of disease. 7th e. WB Sauners; 004.
type I collagen, angiogenesis takes place, granulation tissue
O’Leary JP, Tabuenca A, Capote LR. The physiologic basis of sur-
gery. 4th e. Wolters Kluwer Health/Lippincott Williams & Wilkins;
begins to form, an woun contraction commences. This
008. phase persists for 3 to 4 weeks an nally gives way to the
remoeling phase. At this point, vascularity ecreases an
7. E. Surgical site infections have been shown to increase collagen is continuing to be synthesize, but it is being bro-
the cost of hospitalizations an length of hospital stays ken own at the same rate an collagen cross-linking occurs.
prompting the Surgical Care Improvement Project (SCIP) to References: Brunicari FC, Anersen DK, Billiar TR, Dunn DL,
aress this major economic buren to moern health care. Hunter JG, Matthews JB, Pollock RE. es. Schwartz’s principles of sur-
gery. 10th e. McGraw-Hill Eucation; 015.
While preoperative bathing with antiseptic solution has been
O’Leary JP, Tabuenca A, Capote LR. The physiologic basis of sur-
shown to ecrease bacterial colonization of skin, it has not
gery. 4th e. Wolters Kluwer Health/Lippincott Williams & Wilkins;
been proven to be associate with ecrease rates of surgi- 008.
cal site infections (C). Multiple preparations for preoperative
skin antisepsis have been esigne; however, the two most
commonly in use are ioine-base an chlorhexiine-base 9. C. Hypertrophic scarring an keloi formation are both
in either an aqueous or alcohol solution. Ioine works pri- examples of pathologic excessive healing. Both are cause
marily by passing through the bacterial cell membrane an by the increase eposition of collagen (A). Formation of
replacing intracellular ions with molecular ioine an oxi- kelois has a large genetic component that is inherite in
izing various structures within the bacterium (D). It is also, an autosomal ominant fashion. It is also more prominent
however, toxic to normal tissues, so it is generally combine in arker-skinne iniviuals. Hypertrophic scarring is gen-
with a carrier molecule (e.g., povione) to reuce the sys- erally cause by a elay in woun healing or by excessive
temic availability of molecular ioine an reuce its toxicity. tensile forces on a new woun an is at a particularly high
In contrast, chlorhexiine works by its ability to estabilize risk of forming after burns (D). They o not sprea outsie of
cellular membranes. A Cochrane review one in 015 com- the borers of the original woun, unlike kelois (E). Hyper-
paring ioine-base an chlorhexiine-base preoperative trophic scars ten to recee with time, but if they persist,
antiseptic techniques foun the latter to be superior in pre- they ten to respon better to surgical excision as compare
venting surgical site infections (A). However, it is generally to kelois. Excision of kelois shoul be performe with
not recommene for use above the chin because of ototoxic- caution, as they ten to reoccur an become bigger. If exci-
ity an potential for causing amage to the cornea in higher sion is planne, it shoul be accompanie by an ajunctive
concentrations (B). treatment such as sterois or low-ose raiation to prevent
References: Dumville JC, McFarlane E, Ewars P, et al. Pre- recurrence (B). Several other ajuncts have also been shown
operative skin antiseptics for preventing surgical woun infections to reuce scarring incluing silicone banages, occlusive
after clean surgery. Cochrane Database Syst Rev. 015;(4):CD003949. ressings, an extremity compression evices.
470 PArt ii Medical Knowledge

References: Gauglitz GG, Korting HC, Pavicic T, Ruzicka T, is oor) an has been associate with Alport an Goopas-
Jeschke MG. Hypertrophic scarring an kelois: Pathomecha- ture synrome (A). Type V collagen is closely associate with
nisms an current an emerging treatment strategies. Mol Med. type I an is in most of the same tissues but with the aition
011;17(1–):113–15. of placental tissue. While there exist clinically signicant col-
Greenel LJ, Mulhollan MW, es. Greeneld’s surgery: scientic
lagens outsie of these main ve, such as type VII (epier-
principles & practice. 5th e. Lippincott Williams an Wilkins; 011.
molysis bullosa) an type XVII (bullous pemphigoi), they
O’Leary JP, Tabuenca A, Capote LR. The physiologic basis of sur-
gery. 4th e. Wolters Kluwer Health/Lippincott Williams & Wilkins; are not nearly as prevalent (E). Ehlers-Danlos is a spectrum
008. of connective tissue isorers that can affect multiple types
of collagen (B). However, the most common is type V (seen
10. B. This patient most likely has scurvy cause by a e- in classic type Ehlers-Danlos).
ciency in vitamin C an is uncommon in the moern age. It is Reference: De Paepe A, Malfait F. Bleeing an bruising in
typically seen in patients with severe malnutrition often from patients with Ehlers-Danlos synrome an other collagen vascular
unerevelope countries without access to fresh fruits an isorers: review. Br J Haematol. 004;17(5):491–500.
vegetables. Patients present with loose or missing teeth, open
sores, pigmente spots on the extremities, bleeing mucous
12. D. While healing of the gastrointestinal tract goes
through the same basic steps as healing of the skin, there are
membranes, vague myalgias, an fatigue. It is a key cofactor
several key ifferences an unique features. Skin wouns
in the hyroxylation of lysine an proline uring collagen
unergo a relatively steay increase of the tensile strength
synthesis; as such, collagen cross-linking is extremely imin-
of the woun over time. In contrast, the increase collage-
ishe in patients with vitamin C eciency. It can even cause
nase activity in the small bowel allows collagen breakown
the involution of previous scars because remoeling con-
to excee collagen eposition on ays 3 to 5 after an anas-
tinues, but patients are unable to synthesize new collagen.
tomosis (B). This is why anastomotic leaks in the gastroin-
Vitamin C is also involve in iron absorption (C). Vitamin
testinal tract occur with increase frequency in this critical
A is another essential vitamin in woun healing an assists
time perio. However, the gastrointestinal tract is quicker
with epithelialization, proteoglycan synthesis, an normal
to reach maximal tensile strength when compare with the
immune function (A). It has also been shown to reverse the
skin. The submucosa provies most of the tensile strength
effects of sterois on woun healing. Vitamin D is consume
for an anastomosis because of the coarse, interwoven bers
in the iet an prouce in the skin. It then unergoes acti-
that make it up. However, the mucosa an serosa are also
vation (hyroxylation) by the liver an kiney to play an
important, an both help provie a quick, leakproof barrier
essential role in calcium metabolism (E). Exogenous vitamin
over the rst several ays (C). One can appreciate this effect
E has been shown in animal trials to cause elaye woun
in action by noting the relatively higher leak rates with por-
healing via an inammatory mechanism similar to cortico-
tions of the GI tract that lack serosa such as the esophagus.
sterois (D).
Multiple ajuncts an techniques have been trie to ecrease
Reference: O’Leary JP, Tabuenca A, Capote LR. The physiologic
basis of surgery. 4th e. Wolters Kluwer Health/Lippincott Williams the rate of anastomotic leaks, an while there may be a tren
& Wilkins; 008. towar fewer leaks with a staple anastomosis in certain cir-
cumstances, there still isn’t conclusive evience that one is
11. C. The most common types of collagen locate in the superior to the other in all cases (A). While omental wrap-
boy inclue types I to V, though there are many more that ping has been shown to improve outcomes in certain situa-
are clinically relevant in certain iseases. Type I collagen tions, a evitalize “omental free ap” will necrose an will
makes up 90% of the boy’s collagen an is foun to some not help with the anastomosis (E).
egree in most tissue, incluing skin, bones, tenons, arterial References: Brunicari FC, Anersen DK, Billiar TR, Dunn DL,
walls, an scars. It is implicate in isease like osteogenesis Hunter JG, Matthews JB, Pollock RE. es. Schwartz’s principles of sur-
imperfecta (D). Type II collagen makes up about 50% of the gery. 10th e. McGraw Hill Eucation; 015.
Egorov VI, Schastlivtsev V, Turusov RA, Baranov AO. Participa-
protein in hyaline cartilage (carTWOlige). Type III collagen
tion of the intestinal layers in supplying of the mechanical strength
is foun in bone, cartilage, an multiple types of connective of the intact an suture gut. Eur Surg Res. 00;34(6):45–431.
tissue, an abnormalities have been foun in Dupuytren Thornton FJ, Barbul A. Healing in the gastrointestinal tract. Neu-
contracture an the formation of aneurysms. Type IV colla- rosurg Clin N Am. 1997;77(3):549–573.
gen is foun primarily in the basement membrane (type four
Conf idence
is ClinicalKey
Evidence-based answers, continually updated

The latest answers, always at your ngertips


A subscription to ClinicalKey draws content from
countless procedural videos, peer-reviewed journals,
patient education materials, and boos authored by
the most respected names in medicine

our patients trust you ou can trust ClinicalKey


Euip yourself with trusted, current content that provides you with
the clinical nowledge to improve patient outcomes

et to now ClinicalKey at storeclinicaleycom

2019v1.0

You might also like